You are on page 1of 546

1

Assignment 1. Questions from chapters 1 and 2 of McMurry and Fay


Question numbers are from the fourth edition.

Chapter 1. Chemistry: Matter and Measurement

1.1 (a) Cd (b) Sb (c) Am

1.2 (a) silver (b) rhodium (c) rhenium (d) cesium (e) argon (f) arsenic

1.3 (a) Ti, metal (b) Te, semimetal (c) Se, nonmetal
(d) Sc, metal (e) At, semimetal (f) Ar, nonmetal

1.4 The three Acoinage metals@ are copper (Cu), silver (Ag), and gold (Au).

1.5 (a) The decimal point must be shifted ten places to the right so the exponent is S10. The
result is 3.72 x 10 10 m.
S

(b) The decimal point must be shifted eleven places to the left so the exponent is 11. The
result is 1.5 x 1011 m.

1.6 (a) microgram (b) decimeter (c) picosecond


(d) kiloampere (e) millimole

5 o 5
1.7 o
C=x ( F _ 32) = x (98.6 _ 32) = 37.0 o C
9 9
K = o C + 273.15 = 37.0 + 273.15 = 310.2 K

1.8 (a) K = oC + 273.15 = S78 + 273.15 = 195.15 K = 195 K


9 9
(b) o F = ( x o C) + 32 = ( x 158) + 32 = 316.4o F = 316 o F
5 5
(c) oC = K S 273.15 = 375 S 273.15 = 101.85oC = 102oC
9 9
o
F = ( x o C) + 32 = ( x 101.85) + 32 = 215.33o F = 215o F
5 5

m 27.43 g
1.9 d= = = 2.212 g/ cm 3
V 12.40 cm 3

1 mL
1.10 volume = 9.37 g x = 6.32 mL
1.483 g

1.11 The actual mass of the bottle and the acetone = 38.0015 g + 0.7791 g = 38.7806 g. The
measured values are 38.7798 g, 38.7795 g, and 38.7801 g. These values are both close to
each other and close to the actual mass. Therefore the results are both precise and
accurate.

1
Chapter 1 S Chemistry: Matter and Measurement
_____________________________________________________________________________

1.12 (a) 76.600 kg has 5 significant figures because zeros at the end of a number and after the
decimal point are always significant.
(b) 4.502 00 x 103 g has 6 significant figures because zeros in the middle of a number are
significant and zeros at the end of a number and after the decimal point are always
significant.
(c) 3000 nm has 1, 2, 3, or 4 significant figures because zeros at the end of a number and
before the decimal point may or may not be significant.
(d) 0.003 00 mL has 3 significant figures because zeros at the beginning of a number are
not significant and zeros at the end of a number and after the decimal point are always
significant.
(e) 18 students has an infinite number of significant figures since this is an exact number.
(f) 3 x 10 5 g has 1 significant figure.
S

(g) 47.60 mL has 4 significant figures because a zero at the end of a number and after the
decimal point is always significant.
(h) 2070 mi has 3 or 4 significant figures because a zero in the middle of a number is
significant and a zero at the end of a number and before the decimal point may or may not
be significant.

1.13 (a) Since the digit to be dropped (the second 4) is less than 5, round down. The result is
3.774 L.
(b) Since the digit to be dropped (0) is less than 5, round down. The result is 255 K.
(c) Since the digit to be dropped is equal to 5 with nothing following, round down. The
result is 55.26 kg.

24.567 g
1.14 (a) + 0.044 78 g This result should be expressed with 3 decimal places. Since the
24.611 78 g
digit to be dropped (7) is greater than 5, round up. The
result is 24.612 g (5 significant figures).

(b) 4.6742 g / 0.003 71 L = 1259.89 g/L


0.003 71 has only 3 significant figures so the result of the division should have only 3
significant figures. Since the digit to be dropped (first 9) is greater than 5, round up. The
result is 1260 g/L (3 significant figures), or 1.26 x 103 g/L.

0.378 mL
+ 42.3 mL
(c) This result should be expressed with 1 decimal place. Since the
_ 1.5833 mL
41.0947 mL
digit to be dropped (9) is greater than 5, round up. The
result is 41.1 mL (3 significant figures).

1.15 The level of the liquid in the thermometer is just past halfway between the 32oC and 33oC

2
Chapter 1 S Chemistry: Matter and Measurement
_____________________________________________________________________________

marks on the thermometer. The temperature is 32.6oC (3 significant figures).

9 9
1.16 (a) Calculation: o
x o C) + 32 = ( x 1064) + 32 = 1947 o F
F=(
5 5
Ballpark estimate: oF . 2 x oC if oC is large. The melting point of gold . 2000oF.

(b) r = d/2 = 3 x 10 6 m = 3 x 10 4 cm; h = 2 x 10 6 m = 2 x 10 4 cm


S S S S

Calculation: volume = πr2h = (3.1416)(3 x 10 4 cm)2(2 x 10 4 cm) = 6 x 10 11 cm3


S S S

Ballpark estimate: volume = πr2h . 3r2h . 3(3 x 10S4 cm)2(2 x 10S4 cm) . 5 x 10S11 cm3

1.17 1 carat = 200 mg = 200 x 10S3 g = 0.200 g


0.200 g
Mass of Hope Diamond in grams = 44.4 carats x = 8.88 g
1 carat
1 ounce = 28.35 g
1 ounce
Mass of Hope Diamond in ounces = 8.88 g x = 0.313 ounces
28.35 g

1.18 An LD50 value is the amount of a substance per kilogram of body weight that is a lethal
dose for 50% of the test animals.

453.6 g 1 kg 4 g
1.19 mass of salt = 155 lb x x x = 281.2 g or 300 g
1 lb 1000 g 1 kg

Understanding Key Concepts

1.20

1.21

3
Chapter 1 S Chemistry: Matter and Measurement
_____________________________________________________________________________

1.22 red B gas; blue B 42; green B sodium

1.23 The element is americium (Am) with atomic number = 95. It is in the actinide series.

1.24 (a) Darts are clustered together (good precision) but are away from the bullseye (poor
accuracy).
(b) Darts are clustered together (good precision) and hit the bullseye (good accuracy).
(c) Darts are scattered (poor precision) and are away from the bullseye (poor accuracy).

1.25 (a) 34.2 mL (3 significant figures) (b) 2.68 cm (3 significant figures)

1.26

The 5 mL graduated cylinder is marked every 0.2


mL and can be read to ∀ 0.02 mL. The 50 mL
graduated cylinder is marked every 2 mL and can
only be read to ∀ 0.2 mL. The 5 mL graduated
cylinder will give more accurate measurements.

1.27 A liquid that is less dense than another will float on top of it. The most dense liquid is
mercury, and it is at the bottom of the cylinder. Because water is less dense than mercury
but more dense than vegetable oil, it is the middle liquid in the cylinder. Vegetable oil is
the least dense of the three liquids and is the top liquid in the cylinder.

Additional Problems
Elements and the Periodic Table

1.28 114 elements are presently known. About 90 elements occur naturally.

1.29 The rows are called periods, and the columns are called groups.

4
Chapter 1 S Chemistry: Matter and Measurement
_____________________________________________________________________________

1.30 There are 18 groups in the periodic table. They are labeled as follows:
1A, 2A, 3B, 4B, 5B, 6B, 7B, 8B (3 groups), 1B, 2B, 3A, 4A, 5A, 6A, 7A, 8A

1.31 Elements within a group have similar chemical properties.

1.32

1.33

1.34
A semimetal is an element with
properties that fall between those of
metals and nonmetals.

1.35 (a) The alkali metals are shiny, soft, low-melting metals that react rapidly with water to
form products that are alkaline.
(b) The noble gases are gases of very low reactivity.
(c) The halogens are nonmetallic and corrosive. They are found in nature only in

5
Chapter 1 S Chemistry: Matter and Measurement
_____________________________________________________________________________

combination with other elements.

1.36 Li, Na, K, Rb, and Cs

1.37 Be, Mg, Ca, Sr, and Ba

1.38 F, Cl, Br, and I

1.39 He, Ne, Ar, Kr, Xe, and Rn

1.40 (a) gadolinium, Gd (b) germanium, Ge (c) technetium, Tc (d) arsenic, As

1.41 (a) cadmium, Cd (b) iridium, Ir (c) beryllium, Be (d) tungsten, W

1.42 (a) Te, tellurium (b) Re, rhenium (c) Be, beryllium
(d) Ar, argon (e) Pu, plutonium

1.43 (a) B, boron (b) Rh, rhodium (c) Cf, californium


(d) Os, osmium (e) Ga, gallium

1.44 (a) Tin is Sn: Ti is titanium. (b) Manganese is Mn: Mg is magnesium.


(c) Potassium is K: Po is polonium. (d) The symbol for helium is He. The
second letter is lowercase.

1.45 (a) The symbol for carbon is C. (b) The symbol for sodium is Na.
(c) The symbol for nitrogen is N. (d) The symbol for chlorine is Cl.

Units and Significant Figures

1.46 Mass measures the amount of matter in an object, whereas weight measures the pull of
gravity on an object by the earth or other celestial body.

1.47 There are only seven fundamental (base) SI units for scientific measurement. A derived
SI unit is some combination of two or more base SI units.
Base SI unit: Mass, kg; Derived SI unit: Density, kg/m3

1.48 (a) kilogram, kg (b) meter, m (c) kelvin, K (d) cubic meter, m3

1.49 (a) kilo, k (b) micro, Φ (c) giga, G (d) pico, p (e) centi, c

9
1.50 A Celsius degree is larger than a Fahrenheit degree by a factor of .
5
1.51 A kelvin and Celsius degree are the same size.

6
Chapter 1 S Chemistry: Matter and Measurement
_____________________________________________________________________________

1.52 The volume of a cubic decimeter (dm3) and a liter (L) are the same.

1.53 The volume of a cubic centimeter (cm3) and a milliliter (mL) are the same.

1.54 Only (a) is exact because it is obtained by counting. (b) and (c) are not exact because
they result from measurements.

4.8673 g
1.55 _ 4.8 g The result should contain only 1 decimal place. Since the digit to
0.0673 g
be dropped (6) is greater than 5, round up. The result is 0.1 g.

1.56 cL is centiliter (10-2 L)

1.57 (a) deciliter (10-1 L) (b) decimeter (10-1 m)


(c) micrometer (10-6 m) (d) nanoliter (10-9 L)

1.58 1 mg = 1 x 10-3 g and 1 pg = 1 x 10-12 g


1 x 10 _ 3 g 1 pg
x = 1 x 109 pg/mg
1 mg 1 x 10 _12 g

35 ng = 35 x 10-9 g
35 x 10 _ 9 g 1 pg
x _12
= 3.5 x 10 4 pg / 35 ng
35 ng 1 x 10 g

1 µL
1.59 1 µL = 10-6 L _6
= 106 µ L/L
10 L
20 x 10 _ 3 L 1 µ L
20 mL = 20 x 10-3 L x _ 6 = 2 x 10 4 µ L/ 20 mL
20 mL 10 L

1.60 (a) 5 pm = 5 x 10-12 m


100 cm
5 x 10-12 m x = 5 x 10-10 cm
1m
1 nm
5 x 10-12 m x _9
= 5 x 10-3 nm
1 x 10 m

3
3 1m  -6 3
(b) 8.5 cm x   = 8.5 x 10 m
 100 cm 

7
Chapter 1 S Chemistry: Matter and Measurement
_____________________________________________________________________________
3
3  10 mm  3 3
8.5 cm x   = 8.5 x 10 mm
 1 cm 

1 x 10 _ 3 g
(c) 65.2 mg x = 0.0652 g
1 mg
1 x 10 _ 3 g 1 pg
65.2 mg x x _12
= 6.52 x 1010 pg
1 mg 1 x 10 g

1.61 (a) A liter is just slightly larger than a quart.


(b) A mile is about twice as long as a kilometer.
(c) An ounce is about 30 times larger than a gram.
(d) An inch is about 2.5 times larger than a centimeter.

1.62 (a) 35.0445 g has 6 significant figures because zeros in the middle of a number are
significant.
(b) 59.0001 cm has 6 significant figures because zeros in the middle of a number are
significant.
(c) 0.030 03 kg has 4 significant figures because zeros at the beginning of a number are
not significant and zeros in the middle of a number are significant.
(d) 0.004 50 m has 3 significant figures because zeros at the beginning of a number are
not significant and zeros at the end of a number and after the decimal point are always
significant.
(e) 67,000 m2 has 2, 3, 4, or 5 significant figures because zeros at the end of a number
and before the decimal point may or may not be significant.

(f) 3.8200 x 103 L has 5 significant figures because zeros at the end of a number and after
the decimal point are always significant.

1.63 (a) $130.95 is an exact number and has an infinite number of significant figures.
(b) 2000.003 has 7 significant figures because zeros in the middle of a number are
significant.
(c) The measured quantity, 5 ft 3 in., has 2 significant figures. The 5 ft is certain and the
3 in. is an estimate.

1.64 To convert 3,666,500 m3 to scientific notation, move the decimal point 6 places to the left
and include an exponent of 106. The result is 3.6665 x 106 m3.

1.65 Since the digit to be dropped (3) is less than 5, round down. The result to 4 significant
figures is 7926 mi or 7.926 x 103 mi.
Since the digit to be dropped (2) is less than 5, round down. The result to 2 significant
figures is 7900 mi or 7.9 x 103 mi.

1.66 (a) To convert 453.32 mg to scientific notation, move the decimal point 2 places to the
left and include an exponent of 102. The result is 4.5332 x 102 mg.

8
Chapter 1 S Chemistry: Matter and Measurement
_____________________________________________________________________________

(b) To convert 0.000 042 1 mL to scientific notation, move the decimal point 5 places to
the right and include an exponent of 10S5. The result is 4.21 x 10S5 mL.
(c) To convert 667,000 g to scientific notation, move the decimal point 5 places to the
left and include an exponent of 105. The result is 6.67 x 105 g.

1.67 (a) Since the exponent is a negative 3, move the decimal point 3 places to the left to get
0.003 221 mm.
(b) Since the exponent is a positive 5, move the decimal point 5 places to the right to get
894,000 m.
(c) Since the exponent is a negative 12, move the decimal point 12 places to the left to
get 0.000 000 000 001 350 82 m3.
(d) Since the exponent is a positive 2, move the decimal point 2 places to the right to get
641.00 km.

1.68 (a) Since the digit to be dropped (0) is less than 5, round down. The result is 3.567 x 104
or 35,670 m (4 significant figures).
Since the digit to be dropped (the second 6) is greater than 5, round up. The result is
35,670.1 m (6 significant figures).
(b) Since the digit to be dropped is 5 with nonzero digits following, round up. The result
is 69 g (2 significant figures).
Since the digit to be dropped (0) is less than 5, round down. The result is
68.5 g (3 significant figures).
(c) Since the digit to be dropped is 5 with nothing following, round down. The result is
4.99 x 103 cm (3 significant figures).
(d) Since the digit to be dropped is 5 with nothing following, round down. The result is
2.3098 x 10 4 kg (5 significant figures).
S

1.69 (a) Since the digit to be dropped (1) is less than 5, round down. The result is 7.000 kg.
(b) Since the digit to be dropped is 5 with nothing following, round down. The result is
1.60 km.
(c) Since the digit to be dropped (1) is less than 5, round down. The result is 13.2 g/cm3.
(d) Since the digit to be dropped (1) is less than 5, round down. The result is 2,300,000.
or 2.300 000 x 106.

1.70 (a) 4.884 x 2.05 = 10.012


The result should contain only 3 significant figures because 2.05 contains 3 significant
figures (the smaller number of significant figures of the two). Since the digit to be
dropped (1) is less than 5, round down. The result is 10.0.

(b) 94.61 / 3.7 = 25.57


The result should contain only 2 significant figures because 3.7 contains 2 significant
figures (the smaller number of significant figures of the two). Since the digit to be
dropped (second 5) is 5 with nonzero digits following, round up. The result is 26.

(c) 3.7 / 94.61 = 0.0391


The result should contain only 2 significant figures because 3.7 contains 2 significant

9
Chapter 1 S Chemistry: Matter and Measurement
_____________________________________________________________________________

figures (the smaller number of significant figures of the two). Since the digit to be
dropped (1) is less than 5, round down. The result is 0.039.

5502.3
24
(d) This result should be expressed with no decimal places. Since the
+ 0.01
5526.31
digit to be dropped (3) is less than 5, round down. The result is
5526.

86.3
+ 1.42
(e) This result should be expressed with only 1 decimal place. Since
_ 0.09
87.63
the digit to be dropped (3) is less than 5, round down. The result is
87.6.

(f) 5.7 x 2.31 = 13.167


The result should contain only 2 significant figures because 5.7 contains 2 significant
figures (the smaller number of significant figures of the two). Since the digit to be
dropped (second 1) is less than 5, round down. The result is 13.

3.41 _ 0.23 3.18


1.71 (a) x 0.205 = x 0.205 = 0.12457 = 0.125
5.233 5.233
Complete the subtraction first. The result has 2 decimal places and 3 significant figures.
The result of the multiplication and division must have 3 significant figures. Since the
digit to be dropped is 5 with nonzero digits following, round up.

5.556 x 2.3 5.556 x 2.3


(b) = = 3.08 = 3.1
4.223 _ 0.08 4.143
Complete the subtraction first. The result of the subtraction should have 2 decimal places
and 3 significant figures (an extra digit is being carried until the calculation is completed).
The result of the multiplication and division must have 2 significant figures. Since the
digit to be dropped (8) is greater than 5, round up.

Unit Conversions

453.59 g
1.72 (a) 0.25 lb x = 113.4 g = 110 g
1 lb

10
Chapter 1 S Chemistry: Matter and Measurement
_____________________________________________________________________________

12 in 2.54 cm 1m
(b) 1454 ft x x x = 443.2 m
1 ft 1 in 100 cm
2 2
 1.6093 km   1000 m 
(c) 2,941,526 mi2 x   x
12
 = 7.6181 x 10 m
2

 1 mi   1 km 

2.54 cm 1m
1.73 (a) 5.4 in x x = 0.14 m
1 in 100 cm
1 kg
(b) 66.31 lb x = 30.08 kg
2.2046 lb
3.7854 L 1 x 10 _ 3 m 3
(c) 0.5521 gal x x = 2.090 x 10 _ 3 m 3
1 gal 1L
mi 1.6093 km 1000 m 1h 1 min m
(d) 65 x x x x = 29
h 1 mi 1 km 60 min 60 s s
3
 1m 
3 3
(e) 978.3 yd x   = 748.0 m
 1.0936 yd 
2 2
 1.6093 km   1000 m 
2 6 2
(f) 2.380 mi x   x  = 6.164 x 10 m
 1 mi   1 km 

2
1 mi 2  5280 ft  3
1.74 (a) 1 acre-ft x x  = 43,560 ft
640 acres  1 mi 
3
 5280 ft  1 acref- ft
(b) 116 mi 3 x   x = 3.92 x 108 acre- ft
 1 mi  43,560 ft
3

1/ 3 ft 12 in 2.54 cm
1.75 (a) 18.6 hands x x x = 189 cm
1 hand 1 ft 1 in
3 3 3 3
 1/ 3 ft   12 in   2.54 cm   1 m 
3 3
(b) (6 x 2.5 x15) hands x   x  x  x  = 0.2 m
 1 hand   1 ft   1 in   100 cm 

200 mg 1000 mL
1.76 (a) x = 2000 mg/L
100 mL 1L
200 mg 1 x 10 _ 3 g 1 µg
(b) x x = 2000 µ g/mL
100 mL 1 mg 1 x 10 _ 6 g
200 mg 1 x 10 _ 3 g 1000 mL
(c) x x = 2 g/L
100 mL 1 mg 1L
200 mg 1 x 10 _ 3 g 1000 mL 1 ng 1 x 10 _ 6 L
(d) x x x x = 2000 ng/ µ L
100 mL 1 mg 1L 1 x 10 _ 9 g 1 µL
(e) 2 g/L x 5 L = 10 g

11
Chapter 1 S Chemistry: Matter and Measurement
_____________________________________________________________________________

14 lb
1.77 8.65 stones x = 121 lb
1 stone

mi 5280 ft 12 in 2.54 cm 1h 2.5 x 10 _ 4 s cm


1.78 55 x x x x x = 0.61
h 1 mi 1 ft 1 in 3600 s 1 shake shake

1 kg
1.79 160 lb x = 72.6 kg
2.2046 lb
20 µ g 1 mg
72.6 kg x x = 1.452 mg = 1.5 mg
1 kg 1 x 103 µ g

Temperature

o 9
1.80 F = ( x oC) + 32
5
9
o
F = ( x 39.9 oC) + 32 = 103.8o F (goat)
5
9
o
F = ( x 22.2oC) + 32 = 72.0o F (Australian spiny anteater)
5

9 
1.81 For Hg: mp is  x (_ 38.87)  + 32 = _ 37.97 o F
5 
9 
For Br2: mp is  x (_ 7.2)  + 32 = 19.0 o F
5 
9 
For Cs: mp is  x (28.40)  + 32 = 83.12o F
5 
 9 
For Ga: mp is  x (29.78)  + 32 = 85.60o F
5 

o 5 o 5
1.82 C=x ( F _ 32) = x (6192 _ 32) = 3422o C
9 9
K = oC + 273.15 = 3422 + 273.15 = 3695.15 K or 3695 K

9 9
1.83 o
F = ( x o C) + 32 = ( x 175) + 32 = 347 o F
5 5

1.84 Ethanol boiling point 78.5oC 173.3oF 200oE


o o
Ethanol melting point S117.3 C S179.1 F 0oE
o o
200 E 200 E
(a) o o
= o
= 1.021 o E/ o C
[78.5 C _ (_117.3 C)] 195.8 C

12
Chapter 1 S Chemistry: Matter and Measurement
_____________________________________________________________________________
o o
200 E 200 E
(b) o
= = 0.5675 o E/ o F
[173.3 F _ (_179.1 F)] 352.4 o F
o

200
(c) o E = x ( o C + 117.3)
195.8
200
H2O melting point = 0oC; o E = x (0 + 117.3) = 119.8o E
195.8
200
H2O boiling point = 100oC; o E = x (100 + 117.3) = 222.0o E
195.8
200 200
(d) o E = x ( o F + 179.1) = x (98.6 + 179.1) = 157.6o E
352.4 352.4
 352.4   352.4  o
(e) o F =  o E x  _ 179.1 = 130 x  _ 179.1 = 50.0 F
 200   200 
Since the outside temperature is 50.0oF, I would wear a sweater or light jacket.

1.85 NH3 boiling point S33.4oC S28.1oF 100oA


NH3 melting point S77.7oC S107.9oF 0oA
o o
100 A 100 A
(a) = = 2.26 o A / o C
[_ 33.4 _ (_ 77.7 o C)] 44.3o C
o o
100 A 100 A
(b) = = 1.25 o A / o F
[_ 28.1 _ (_107.9o F)] 79.8o F
100
(c) o A = x ( o C + 77.7)
44.3
100
H2O melting point = 0oC; o A = x (0 + 77.7) = 175o A
44.3
100
H2O boiling point = 100oC; o A = x (100 + 77.7) = 401o A
44.3
100 100
(d) o A = x ( o F + 107.9) = x (98.6 + 107.9) = 259o A
79.8 79.8

Density
1 x 10 _ 3 g 1 cm 3
1.86 250 mg x = 0.25 g; V = 0.25 g x = 0.18 cm 3
1 mg 1.40 g

453.59 g 1 cm 3
500 lb x = 226, 795 g; V = 226, 795 g x = 161,996 cm 3 = 162, 000 cm 3
1 lb 1.40 g

1L
1.87 For H2: V = 1.0078 g x = 11.2 L
0.0899 g
1L
For Cl2: V = 35.45 g x = 11.03 L
3.214 g

13
Chapter 1 S Chemistry: Matter and Measurement
_____________________________________________________________________________

m 220.9 g g g
1.88 d= = 3
= 11.4 3
= 11 3
V (0.50 x 1.55 x 25.00) cm cm cm

1.89 d = 2.40 mm = 0.240 cm


r = d/2 = 0.120 cm and V = πr2h
m 0.3624 g
d= = 2
= 0.534 g/ cm 3
V ( 3.1416 ) ( 0.120 cm ) ( 15.0 cm )

m 8.763 g 8.763 g g g
1.90 d= = = = 2.331 3
= 2.33 3
V (28.76 _ 25.00) mL 3.76 mL cm cm

1.91 The explosion was caused by a chemical property. Na reacts violently with H2O.

General Problems

1.92 (a) selenium, Se (b) rhenium, Re (c) cobalt, Co (d) rhodium, Rh

1.93 (a) Element 117 is a halogen because it would be found directly below At in group 7A.
(b) Element 119
(c) Element 115 would be found directly below Bi and would be a metal. Element 117
might have the properties of a semimetal.
(d) Element 119, at the bottom of group 1A, would likely be a soft, shiny, very reactive
metal forming a +1 cation.

1.94 NaCl melting point = 1074 K


o
C = K S 273.15 = 1074 S 273.15 = 800.85oC = 801oC
9 9
o
F = ( x o C) + 32 = ( x 800.85) + 32 = 1473.53o F = 1474 o F
5 5
NaCl boiling point = 1686 K
o
C = K S 273.15 = 1686 S 273.15 = 1412.85oC = 1413oC
9 9
o
F = ( x o C) + 32 = ( x 1412.85) + 32 = 2575.13o F = 2575o F
5 5
9  9 
1.95 o
F =  x o C  + 32 =  x (_ 38.9)  + 32 = _ 38.0 o F
5  5 

1 mL
1.96 V = 112.5 g x = 75.85 mL
1.4832 g

lb 453.59 g 1 gal 1L
1.97 15.28 x x x = 1.831 g / mL
gal 1 lb 3.7854 L 1000 mL

14
Chapter 1 S Chemistry: Matter and Measurement
_____________________________________________________________________________

453.59 g 1 mL 1L
1.98 V = 8.728 x 1010 lb x x x = 2.162 x 1010 L
1 lb 1.831 g 1000 mL

2.54 cm 10 mm
1.99 0.22 in x x = 5.6 mm
1 in 1 cm

1 lb 8 pints 1 gal 453.59 g 1L


1.100 (a) density = x x x x = 0.95861 g/mL
1 pint 1 gal 3.7854 L 1 lb 1000 mL
(b) area in m2 =
2 2 2 2
1 mi 2  5280 ft   12 in   2.54 cm   1 m  2
1 acre x x  x  x  x  = 4047 m
640 acres  1 mi   1 ft   1 in   100 cm 
(c) mass of wood =
3 3
128 ft 3  12 in   2.54 cm  0.40 g 1 kg
1 cord x x  x  x x = 1450 kg = 1400 kg
1 cord  1 ft   1 in  1 cm 3
1000 g
(d) mass of oil =
42 gal 3.7854 L 1000 mL 0.85 g 1 kg
1 barrel x x x x x = 135.1 kg = 140 kg
1 barrel 1 gal 1L 1 mL 1000 g
(e) fat Calories =
32 servings 165 Calories 30.0 Cal from fat
0.5 gal x x x = 792 Cal from fat
1 gal 1 serving 100 Cal total

1.101 amount of chocolate =


105 mg caffeine 1.0 ounce chocolate
2.0 cups coffee x x = 14 ounces of chocolate
1 cup coffee 15 mg caffeine
14 ounces of chocolate is just under 1 pound.

1.102 (a) number of Hershey=s Kisses =


453.59 g 1 serving 9 kisses
2.0 lb x x x = 199 kisses = 200 kisses
1 lb 41 g 1 serving
41 g 1 serving 1 mL
(b) Hershey=s Kiss volume = x x = 3.254 mL = 3.3 mL
1 serving 9 kisses 1.4 g
230 Cal 1 serving
(c) Calories/Hershey=s Kiss = x = 25.55 Cal/kiss = 26 Cal/kiss
1 serving 9 kisses
(d) % fat Calories =
13 g fat 9 Cal from fat 1 serving
x x x 100% = 51% Calories from fat
1 serving 1 g fat 230 Cal total

1.103 Let Y equal volume of vinegar and (422.8 cm3 S Y) equal the volume of oil.
Mass = volume x density
397.8 g = (Y x 1.006 g/cm3) + [(422.8 cm3 S Y) x 0.918 g/cm3]
397.8 g = (1.006 g/cm3)Y + 388.1 g S (0.918 g/cm3)Y

15
Chapter 1 S Chemistry: Matter and Measurement
_____________________________________________________________________________

397.8 g S 388.1 g = (1.006 g/cm3)Y S (0.918 g/cm3)Y


9.7 g = (0.088 g/cm3)Y
9.7 g
Y = vinegar volume = 3
= 110 cm3
0.088 g/ cm
oil volume = (422.8 cm S Y) = (422.8 cm3 S 110 cm3) = 313 cm3
3

5 o 5
1.104 o
C= x ( F _ 32) ; Set o C = o F : o C = x ( o C _ 32)
9 9
9
Solve for o C : o C x = o C _ 32
5
o 9
( C x ) _ o C = _ 32
5
o 4
C x = _ 32
5
5
o
C = (_ 32) = _ 40o C
4
The Celsius and Fahrenheit scales Across@ at _ 40o C (_ 40o F).

1.105 Cork: volume = 1.30 cm x 5.50 cm x 3.00 cm = 21.45 cm3


0.235 g
mass = 21.45 cm 3 x = 5.041 g
1 cm 3
Lead: volume = (1.15 cm)3 = 1.521 cm3
11.35 g
mass = 1.521 cm 3 x = 17.26 g
1 cm 3
total mass = 5.041 g + 17.26 g = 22.30 g
total volume = 21.45 cm3 + 1.521 cm3 = 22.97 cm3
22.30 g
average density = = 0.971 g/ cm 3 so the cork and lead will float.
22.97 cm 3

60 s
1.106 Convert 8 min, 25 s to s. 8 min x + 25 s = 505 s
1 min
Convert 293.2 K to oF 293.2 S 273.15 = 20.05oC
9
o
F = ( x 20.05) + 32 = 68.09o F
5
3. o F
Final temperature = 68.09oF + 505 s x 0 = 93.34 o F
60 s
o 5
C = x (93.34 _ 32) = 34.1o C
9

19.7325 g
1.107 Ethyl alcohol density = = 0.7893 g/mL
25.00 mL
total mass = metal mass + ethyl alcohol mass = 38.4704 g

16
Chapter 1 S Chemistry: Matter and Measurement
_____________________________________________________________________________

ethyl alcohol mass = total mass S metal mass = 38.4704 g S 25.0920 g = 13.3784 g
1 mL
ethyl alcohol volume = 13.3784 g x = 16.95 mL
0.7893 g
metal volume = total volume S ethyl alcohol volume = 25.00 mL S 16.95 mL = 8.05 mL
25.0920 g
metal density = = 3.12 g/mL
8.05 mL

1.108 Average brass density = (0.670)(8.92 g/cm3) + (0.330)(7.14 g/cm3) = 8.333 g/cm3
2.54 cm
length = 1.62 in x = 4.115 cm
1 in
2.54 cm
diameter = 0.514 in x = 1.306 cm
1 in
volume = πr2h = (3.1416)[(1.306 cm)/2]2(4.115 cm) = 5.512 cm3
8.333 g
mass = 5.512 cm3 x = 45.9 g
1 cm 3

3
1.109 35 sv = 35 x 109 m
s
3
 3
  100 cm   1 mL   60 s 
(a) gulf stream flow =  35 x 109 m     3 
18
 = 2.1 x 10 mL/min
 s   1 m   1 cm   1 min 

 mL   60 min   1.025 g 
 ( 24 h ) 
21 18
(b) mass of H2O =  2.1 x 1018   = 3.1 x 10 g = 3.1 x 10
 min   1 h   1 mL 
kg

 1000 mL   
(c) time = (1.0 x 1015 L ) 
1 min
  = 0.48 min
 1 L  2.1 x 10 mL 
18

1.110 (a) Gallium is a metal.


(b) Indium, which is right under gallium in the periodic table, should have similar
chemical properties.
0.2133 lb 453.59 g 1 in .3
(c) Ga density = 3
x x 3
= 5.904 g/cm3
1 in . 1 lb (2.54 cm )

(d) Ga boiling point 2204oC 1000oG


Ga melting point 29.78oC 0oG

o o o
1000 G _ 0 G 1000 G
o o
= o
= 0.4599 oG/oC
2204 C _ 29.78 C 2174.22 C

o
G = 0.4599 x (oC S 29.78)
o
G = 0.4599 x (801 S 29.78) = 355oG

17
Chapter 1 S Chemistry: Matter and Measurement
_____________________________________________________________________________

The melting point of sodium chloride (NaCl) on the gallium scale is 355oG.

18
Chapter 1 S Chemistry: Matter and Measurement
_____________________________________________________________________________

Chapter 2. Atoms, Molecules and Ions

2.1
2
First, find the S:O ratio in each compound.
Substance A: S:O mass ratio = (6.00 g S) / (5.99 g O) = 1.00
Substance B: S:O mass ratio = (8.60 g S) / (12.88 g O) = 0.668
S : O mass ratio in substance A 1.00
= = 1.50 =
3
S : O mass ratio in substance B 0.668 2

2.54 cm 1 Au atom
2.2 0.0002 in x x •8
= 2 x 10 4 Au atoms
1 in 2.9 x 10 cm

1.5 x 10 _10 m 1 km 1 time


2.3 1 x 1019 C atoms x x x = 37.4 times . 40 times
C atom 1000 m 40, 075 km

75
2.4 34 Se has 34 protons, 34 electrons, and (75 S 34) = 41 neutrons.

35 37
2.5 17 Cl has (35 S 17) = 18 neutrons. 17 Cl has (37 S 17) = 20 neutrons.

2.6 The element with 47 protons is Ag. The mass number is the sum of the protons and the
neutrons, 47 + 62 = 109. The isotope symbol is 109
47 Ag .

2.7 atomic mass = (0.6917 x 62.94 amu) + (0.3083 x 64.93 amu) = 63.55 amu

1 amu 1 Cu
2.8 2.15 g x _ 24
x = 2.04 x 10 22 Cu atoms
1.6605 x 10 g 63.55 amu

H H
| |
2.9 H _ C _ N _ H
|
H phantomC

2.10 Figure (b) represents a collection of hydrogen peroxide (H2O2) molecules.

2.11 adrenaline, C9H13NO3

2.12 (a) LiBr is composed of a metal (Li) and nonmetal (Br) and is ionic.

19
Chapter 1 S Chemistry: Matter and Measurement
_____________________________________________________________________________

(b) SiCl4 is composed of only nonmetals and is molecular.


(c) BF3 is composed of only nonmetals and is molecular.
(d) CaO is composed of a metal (Ca) and nonmetal (O) and is ionic.
2.13 Figure (a) most likely represents an ionic compound because there are no discrete
molecules, only a regular array of two different chemical species (ions). Figure (b) most
likely represents a molecular compound because discrete molecules are present.

2.14 (a) HF is an acid. In water, HF dissociates to produce H+(aq).


(b) Ca(OH)2 is a base. In water, Ca(OH)2 dissociates to produce OH!(aq).
(c) LiOH is a base. In water, LiOH dissociates to produce OH!(aq).
(d) HCN is an acid. In water, HCN dissociates to produce H+(aq).

2.15 (a) CsF, cesium fluoride (b) K2O, potassium oxide (c) CuO, copper(II) oxide (d) BaS, barium

2.16 (a) vanadium(III) chloride, VCl3 (b) manganese(IV) oxide, MnO2


(c) copper(II) sulfide, CuS (d) aluminum oxide, Al2O3

2.17 red B potassium sulfide, K2S; green B strontium iodide, SrI2; blue B gallium oxide, Ga2O3

2.18 (a) NCl3, nitrogen trichloride (b) P4O6, tetraphosphorus hexoxide


(c) S2F2, disulfur difluoride (d) SeO2, selenium dioxide

2.19 (a) disulfur dichloride, S2Cl2 (b) iodine monochloride, ICl


(c) nitrogen triiodide, NI3

2.20 (a) Ca(ClO)2, calcium hypochlorite


(b) Ag2S2O3, silver(I) thiosulfate or silver thiosulfate
(c) NaH2PO4, sodium dihydrogen phosphate (d) Sn(NO3)2, tin(II) nitrate
(e) Pb(CH3CO2)4, lead(IV) acetate (f) (NH4)2SO4, ammonium sulfate

2.21 (a) lithium phosphate, Li3PO4 (b) magnesium hydrogen sulfate, Mg(HSO4)2
(c) manganese(II) nitrate, Mn(NO3)2 (d) chromium(III) sulfate, Cr2(SO4)3

2.22 Drawing 1 represents ionic compounds with one cation and two anions. Only (c) CaCl2 is
consistent with drawing 1.
Drawing 2 represents ionic compounds with one cation and one anion. Both (a) LiBr and
(b) NaNO2 are consistent with drawing 2.

2.23 (a) HIO4, periodic acid (b) HBrO2, bromous acid (c) H2CrO4, chromic acid

2.24 A normal visual image results when light from the sun or other source reflects off an
object, strikes the retina in our eye, and is converted into electrical signals that are
processed by the brain. The image obtained with a scanning tunneling microscope, by
contrast, is a three-dimensional, computer-generated data plot that uses tunneling current
to mimic depth perception. The nature of the computer-generated image depends on the
identity of the molecules or atoms on the surface, on the precision with which the probe

20
Chapter 2 S Atoms, Molecules, and Ions
______________________________________________________________________________

tip is made, on how the data are manipulated, and on other experimental variables.

Understanding Key Concepts

2.25 Drawing (a) represents a collection of SO2 molecules. Drawing (d) represents a mixture
of S atoms and O2 molecules.

2.26 To obey the law of mass conservation, the correct drawing must have the same number of
red and yellow spheres as in drawing (a). The correct drawing is (d).

2.27 Figures (b) and (d) illustrate the law of multiple proportions. The mass ratio is
2.

2.28. (a) alanine, C3H7NO2 (b) ethylene glycol, C2H6O2 (c) acetic acid, C2H4O2

2.29 A Na atom has 11 protons and 11 electrons [drawing (b)].


A Ca2+ ion has 20 protons and 18 electrons [drawing (c)].
A FS ion has 9 protons and 10 electrons [drawing (a)].

2.30

2.31 (a) MgSO4 (b) Li2CO3 (c) FeCl2 (d) Ca3(PO4)2

Additional Problems
Atomic Theory

2.32 The law of mass conservation in terms of Dalton=s atomic theory states that chemical
reactions only rearrange the way that atoms are combined; the atoms themselves are not
changed.
The law of definite proportions in terms of Dalton=s atomic theory states that the
chemical combination of elements to make different substances occurs when atoms join
together in small, whole-number ratios.

2.33 The law of multiple proportions states that if two elements combine in different ways to
form different substances, the mass ratios are small, whole-number multiples of each
other. This is very similar to Dalton=s statement that the chemical combination of
elements to make different substances occurs when atoms join together in small, whole-
number ratios.

21
Chapter 2 S Atoms, Molecules, and Ions
______________________________________________________________________________

2.34 First, find the C:H ratio in each compound.


Benzene: C:H mass ratio = (4.61 g C) / (0.39 g H) = 12
Ethane: C:H mass ratio (4.00 g C) / (1.00 g H) = 4.00
Ethylene: C:H mass ratio = (4.29 g C) / (0.71 g H) = 6.0
C : H mass ratio in benzene 12 3
= =
C : H mass ratio in ethane 4.00 1
C : H mass ratio in benzene 12 2
= =
C : H mass ratio in ethylene 6.0 1
C : H mass ratio in ethylene 6.0 3
= =
C : H mass ratio in ethane 4.00 2

2.35 First, find the C:O ratio in each compound.


Carbon suboxide: C:O mass ratio = (1.32 g C) / (1.18 g O) = 1.12
Carbon dioxide: C:O mass ratio = (12.00 g C) / (32.00 g O) = 0.375
C : O mass ratio in carbon suboxide 1.12 3
= =
C : O mass ratio in carbon dioxide 0.375 1

2.36 (a) For benzene:


1 amu 1 C atom
4.61 g x _ 24
x = 2.31 x 10 23 C atoms
1.6605 x 10 g 12.011 amu
1 amu 1 H atom
0.39 g x _ 24
x = 2.3 x 10 23 H atoms
1.6605 x 10 g 1.008 amu
C 2.31 x 10 23 C atoms 1 C
= =
H 2.3 x 10 23 H atoms 1 H
A possible formula for benzene is CH.

For ethane:
1 amu 1 C atom
4.00 g x _ 24
x = 2.01 x 10 23 C atoms
1.6605 x 10 g 12.011 amu
1 amu 1 H atom
1.00 g x _ 24
x = 5.97 x 10 23 H atoms
1.6605 x 10 g 1.008 amu
C 2.01 x 10 23 C atoms 1 C
= =
H 5.97 x 10 23 H atoms 3 H
A possible formula for ethane is CH3.

For ethylene:
1 amu 1 C atom
4.29 g x _ 24
x = 2.15 x 10 23 C atoms
1.6605 x 10 g 12.011 amu
1 amu 1 H atom
0.71 g x _ 24
x = 4.2 x 10 23 H atoms
1.6605 x 10 g 1.008 amu
C 2.15 x 10 23 C atoms 1 C
= =
H 4.2 x 10 23 H atoms 2 H

22
Chapter 2 S Atoms, Molecules, and Ions
______________________________________________________________________________

A possible formula for ethylene is CH2.

(b) The results in part (a) give the smallest whole-number ratio of C to H for benzene,
ethane, and ethylene, and these ratios are consistent with their modern formulas.

1 amu 1 C atom
2.37 1.32 g x _ 24
x = 6.62 x 10 22 C atoms
1.6605 x 10 g 12.011 amu
1 amu 1 O atom
1.18 g x _ 24
x = 4.44 x 10 22 O atoms
1.6605 x 10 g 15.9994 amu
C 6.62 x 10 22 C atoms 1.5 C
= = ;
O 4.44 x 10 22 O atoms 1 O
therefore the formula for carbon suboxide is C1.5O, or C3O2.

g
2.38 (a) (1.67 x 10 _ 24 )(6.02 x 10 23 H atoms) = 1.01 g
H atom
This result is numerically equal to the atomic mass of H in grams.
g
(b) (26.558 x 10 _ 24 )(6.02 x 10 23 O atoms) = 16.0 g
O atom
This result is numerically equal to the atomic mass of O in grams.

2.39 The mass of 6.02 x 1023 atoms is its atomic mass expressed in grams.
(a) If the atomic mass of an element is X, then 6.02 x 1023 atoms of this element weighs
X grams.
(b) If the mass of 6.02 x 1023 atoms of element Y is 83.80 g, then the atomic mass of Y is
83.80. Y is Kr.

2.40 Assume a 1.00 g sample of the binary compound of zinc and sulfur.
0.671 x 1.00 g = 0.671 g Zn; 0.329 x 1.00 g = 0.329 g S
1 amu 1 Zn atom
0.671 g x x = 6.18 x 10 21 Zn atoms
1.6605 x 10 _ 24 g 65.39 amu
1 amu 1 S atom
0.329 g x _ 24
x = 6.18 x 10 21 S atoms
1.6605 x 10 g 32.066 amu
Zn 6.18 x 10 21 Zn atoms 1 Zn
= = ; therefore the formula is ZnS.
S 6.18 x 10 21 S atoms 1S

2.41 Assume a 1.000 g sample of one of the binary compounds.


0.3104 x 1.000 g = 0.3104 g Ti; 0.6896 x 1.000 g = 0.6896 g Cl
1 amu 1 Ti atom
0.3104 g x _ 24
x = 3.90 x 10 21 Ti atoms
1.6605 x 10 g 47.88 amu
1 amu 1 Cl atom
0.6896 g x _ 24
x = 1.17 x 10 22 Cl atoms
1.6605 x 10 g 35.453 amu

23
Chapter 2 S Atoms, Molecules, and Ions
______________________________________________________________________________

Cl 1.17 x 10 22 3
= =
Ti 3.90 x 10 21 1
Assume a 1.000 g sample of the other binary compound.
0.2524 x 1.000 g = 0.2524 g Ti; 0.7476 x 1.000 g = 0.7476 g Cl
1 amu 1 Ti atom
0.2524 g x _ 24
x = 3.17 x 10 21 Ti atoms
1.6605 x 10 g 47.88 amu
1 amu 1 Cl atom
0.7476 g x _ 24
x = 1.27 x 10 22 Cl atoms
1.6605 x 10 g 35.453 amu
Cl 1.27 x 10 22 4
= =
Ti 3.17 x 10 21 1

Elements and Atoms

2.42 The atomic number is equal to the number of protons.


The mass number is equal to the sum of the number of protons and the number of neutrons.

2.43 The atomic number is equal to the number of protons.


The atomic mass is the weighted average mass (in amu) of the various isotopes for a
particular element.

2.44 Atoms of the same element that have different numbers of neutrons are called isotopes.

2.45 The mass number is equal to the sum of the number of protons and the number of
neutrons for a particular isotope.
For 14
6 C , mass number = 6 protons + 8 neutrons = 14.
14
For 7 N , mass number = 7 protons + 7 neutrons = 14.

2.46 The subscript giving the atomic number of an atom is often left off of an isotope symbol
because one can readily look up the atomic number in the periodic table.

2.47 Te has isotopes with more neutrons than the isotopes of I.

2.48 (a) carbon, C (b) argon, Ar (c) vanadium, V

137
2.49 55 Cs

220 210 197


2.50 (a) 86 Rn (b) 84 Po (c) 79 Au

140 60
2.51 (a) 58 Ce (b) 27 Co

15
2.52 (a) 7 N , 7 protons, 7 electrons, (15 S 7) = 8 neutrons

24
Chapter 2 S Atoms, Molecules, and Ions
______________________________________________________________________________
60
(b) 27 Co, 27 protons, 27 electrons, (60 S 27) = 33 neutrons

131
(c) 53 I , 53 protons, 53 electrons, (131 S 53) = 78 neutrons

142
(d) 58 Ce, 58 protons, 58 electrons, (142 S 58) = 84 neutrons

27
2.53 (a) Al , 13 protons and (27 S 13) = 14 neutrons
32
(b) S , 16 protons and (32 S 16) = 16 neutrons
64
(c) Zn , 30 protons and (64 S 30) = 34 neutrons
207
(d) Pb , 82 protons and (207 S 82) = 125 neutrons

24 58
2.54 (a) 12 Mg , magnesium (b) 28Ni, nickel
104 183
(c) 46 Pd, palladium (d) 74W, tungsten

202 195
2.55 (a) 80 Hg, mercury (b) 78 Pt , platinum
184 209
(c) 76Os, osmium (d) 83 Bi, bismuth

2.56 (0.199 x 10.0129 amu) + (0.801 x 11.009 31 amu) = 10.8 amu for B

2.57 (0.5184 x 106.9051 amu) + (0.4816 x 108.9048 amu) = 107.9 amu for Ag

2.58 24.305 amu = (0.7899 x 23.985 amu) + (0.1000 x 24.986 amu) + (0.1101 x Z)
Solve for Z. Z = 25.982 amu for 26Mg.

2.59 The total abundance of all three isotopes must be 100.00%. The natural abundance of
29
Si is 4.67%. The natural abundance of 28Si and 30Si together must be 100.00% S 4.67%
= 95.33%. Let Y be the natural abundance of 28Si and [95.33 S Y] the natural abundance
of 30Si.
28.0855 amu = (0.0467 x 28.9765 amu) + (Y x 27.9769 amu)
+ ([0.9533 S Y] x 29.9738 amu)
_1.842
Solve for Y. Y = = 0.922
_1.997
28 30
Si natural abundance = 92.2% Si natural abundance = 95.33 S 92.2 = 3.1%

Compounds and Mixtures, Molecules and Ions

2.60 (a) muddy water, heterogeneous mixture


(b) concrete, heterogeneous mixture
(c) house paint, homogeneous mixture
(d) a soft drink, homogeneous mixture (heterogeneous mixture if it contains CO2 bubbles)

2.61 (a) 18 karat gold, (b) window glass, and (d) liquefied air are homogeneous mixtures.
(c) Tomato juice is a heterogeneous mixture because the liquid contains solid pulp.

25
Chapter 2 S Atoms, Molecules, and Ions
______________________________________________________________________________

2.62 An atom is the smallest particle that retains the chemical properties of an element. A
molecule is matter that results when two or more atoms are joined by covalent bonds. H
and O are atoms, H2O is a water molecule.

2.63 A molecule is the unit of matter that results when two or more atoms are joined by
covalent bonds. An ion results when an atom gains or loses electrons. CH4 is a methane
molecule. Na+ is the sodium cation.

2.64 A covalent bond results when two atoms share several (usually two) of their electrons.
An ionic bond results from a complete transfer of one or more electrons from one atom to
another. The CBH bonds in methane (CH4) are covalent bonds. The bond in NaCl
(Na+Cl ) is an ionic bond.
S

2.65 Covalent bonds typically form between nonmetals. (a) BBBr, (c) BrBCl, and (d) OBBr
are covalent bonds.
Ionic bonds typically form between a metal and a nonmetal. (b) NaBBr is an ionic bond.

2.66 Element symbols are composed of one or two letters. If the element symbol is two letters,
the first letter is uppercase and the second is lowercase. CO stands for carbon and oxygen
in carbon monoxide.

2.67 (a) The formula of ammonia is NH3.


(b) The ionic solid potassium chloride has the formula KCl.
(c) ClS is an anion.
(d) CH4 is a neutral molecule.

2.68 (a) Be2+, 4 protons and 2 electrons (b) Rb+, 37 protons and 36 electrons
(c) Se2 , 34 protons and 36 electrons (d) Au3+, 79 protons and 76 electrons
S

2.69 (a) A +2 cation that has 36 electrons must have 38 protons. X = Sr.
(b) A S1 anion that has 36 electrons must have 35 protons. X = Br.

2.70 C3H8O

2.71 C3H6O3

H H H H
| | | |
2.72 H _ C _ C _ C _ C _ H
| | | |
H H H H

26
Chapter 2 S Atoms, Molecules, and Ions
______________________________________________________________________________

2.73

Acids and Bases

2.74 (a) HI, acid (b) CsOH, base (c) H3PO4, acid
(d) Ba(OH)2, base (e) H2CO3, acid

2.75 (a) HI, one H+ ion (b) H3PO4, three H+ ions (c) H2CO3, two H+ ions

HI(aq)  H+(aq) + I (aq); the anion is I


S S
2.76
H3PO4(aq)  H+(aq) + H2PO4 (aq); the predominant anion is H2PO4
S S

H2CO3(aq)  H (aq) + HCO3 (aq); the predominant anion is HCO3


+ S S

(b) CsOH(aq)  Cs+(aq) + OH (aq); the cation is Cs+


S
2.77
(d) Ba(OH)2(aq)  Ba2+(aq) + 2 OH (aq); the cation is Ba2+
S

Naming Compounds

2.78 (a) KCl (b) SnBr2 (c) CaO (d) BaCl2 (e) AlH3

2.79 (a) Ca(CH3CO2)2 (b) Fe(CN)2 (c) Na2Cr2O7 (d) Cr2(SO4)3 (e) Hg(ClO4)2

2.80 (a) barium ion (b) cesium ion (c) vanadium(III)


ion
(d) hydrogen carbonate ion (e) ammonium ion (f) nickel(II) ion
(g) nitrite ion (h) chlorite ion (i) manganese(II) ion (j) perchlorate ion

2.81 (a) carbon tetrachloride (b) chlorine dioxide


(c) dinitrogen monoxide (d) dinitrogen trioxide

2.82 (a) SO32S (b) PO43S (c) Zr4+ (d) CrO42S (e) CH3CO2S (f) S2O32S

2.83 (a) Zn2+ (b) Fe3+ (c) Ti4+ (d) Sn2+ (e) Hg22+ (f) Mn4+ (g) K+ (h) Cu2+

2.84 (a) zinc(II) cyanide (b) iron(III) nitrite (c) titanium(IV) sulfate
(d) tin(II) phosphate (e) mercury(I) sulfide (f) manganese(IV) oxide
(g) potassium periodate (h) copper(II) acetate

27
Chapter 2 S Atoms, Molecules, and Ions
______________________________________________________________________________

2.85 (a) magnesium sulfite (b) cobalt(II) nitrite (c) manganese(II) hydrogen carbonate (d) zinc(II) chr
(g) aluminum sulfate (h) lithium chlorate

(a) Na+ and SO42 ; therefore the formula is Na2SO4


S
2.86
(b) Ba2+ and PO43 ; therefore the formula is Ba3(PO4)2
S

3+ 2S
(c) Ga and SO4 ; therefore the formula is Ga2(SO4)3

2.87 (a) Na2O2 (b) AlBr3 (c) Cr2(SO4)3

General Problems

2.88 atomic mass = (0.205 x 69.924 amu) + (0.274 x 71.922 amu)


+ (0.078 x 72.923 amu) + (0.365 x 73.921 amu)
+ (0.078 x 75.921 amu) = 72.6 amu

12.011 amu 1.6605 x 10 _ 24 g


2.89 mass of 1 C atom = x = 2.00 x 10 _ 23 g / C atom
1 C atoms 1 amu
1 x 10 _ 5 g
number of C atoms = _ 23
= 5 x 1017 C atoms
2.00 x 10 g / C atom
17
5 x 10 C atoms
time = = 2.5 x 1017 s
2 C atoms / s

2.90 (a) sodium bromate (b) phosphoric acid


(c) phosphorous acid (d) vanadium(V) oxide

2.91 (a) Ca(HSO4)2 (b) SnO (c) Ru(NO3)3 (d) (NH4)2CO3 (e) HI (f) Be3(PO4)2

 3 x 1.0079 amu H 
2.92 For NH3, (2.34 g N)   = 0.505 g H
 14.0067 amu N 
 4 x 1.0079 amu H 
For N2H4, (2.34 g N)   = 0.337 g H
 2 x 14.0067 amu N 

 3.670 g N 
2.93 g N = (1.575 g H)   = 10.96 g N
 0.5275 g H 
From Problem 2.92:
g N 2.34 g N
for NH3, = = 4.63
g H 0.505 g H
g N 2.34 g N
for N2H4, = = 6.94
g H 0.337 g H

28
Chapter 2 S Atoms, Molecules, and Ions
______________________________________________________________________________

g N 10.96 g N
for compound X, = = 6.96 ; X is N2H4
g H 1.575 g H
2.94 TeO42S, tellurate; TeO32S, tellurite.
TeO42 and TeO32 are analogous to SO42 and SO32 .
S S S S

2.95 H2TeO4, telluric acid; H2TeO3, tellurous acid

2.96 (a) IS (b) Au3+ (c) Kr

12.0000 amu X
2.97 = ; X = 12.0005 amu for 12C prior to 1961.
15.9994 amu 16.0000 amu

39.9626 amu X
2.98 = ; X = 39.9641 amu for 40Ca prior to 1961.
15.9994 amu 16.0000 amu

(a) AsO43 , arsenate (b) SeO32 , selenite


S S
2.99
(c) SeO42 , selenate (d) HAsO42 , hydrogen arsenate
S S

2.100 (a) calcium-40, 40Ca


(b) Not enough information, several different isotopes can have 63 neutrons.
(c) The neutral atom contains 26 electrons. The ion is iron-56, 56Fe3+.
(d) Se2S

2.101 Deuterium is 2H and deuterium fluoride is 2HF.


2
H has 1 proton, 1 neutron, and 1 electron.
F has 9 protons, 10 neutrons, and 9 electrons.
2
HF has 10 protons, 11 neutrons, and 10 electrons.
Chemically, 2HF is like HF and is a weak acid.
1
2.102 H35Cl has 18 protons, 18 neutrons, and 18 electrons.
1
H37Cl has 18 protons, 20 neutrons, and 18 electrons.
2 35
H Cl has 18 protons, 19 neutrons, and 18 electrons.
2 37
H Cl has 18 protons, 21 neutrons, and 18 electrons.
3 35
H Cl has 18 protons, 20 neutrons, and 18 electrons.
3 37
H Cl has 18 protons, 22 neutrons, and 18 electrons.
40
2.103 (a) Ar has 18 protons, 22 neutrons, and 18 electrons
40
(b) Ca2+ has 20 protons, 20 neutrons, and 18 electrons
39 +
(c) K has 19 protons, 20 neutrons, and 18 electrons
35 S
(d) Cl has 17 protons, 18 neutrons, and 18 electrons

2.104 (a) Mg2+ and ClS, MgCl2, magnesium chloride


(b) Ca2+ and O2S, CaO, calcium oxide
Li+ and N3 , Li3N, lithium nitride
S
(c)
Al and O2 , Al2O3, aluminum oxide
3+ S
(d)

29
Chapter 2 S Atoms, Molecules, and Ions
______________________________________________________________________________

2.105

2.106

2.107 Mass of H2SO4 solution = 1.3028 g/mL x 40.00 mL = 52.112 g


Total mass of Zn and H2SO4 solution before reaction = 9.520 g + 52.112 g = 61.632 g
Total mass of solution after the reaction = 61.338 g
Because of the conservation of mass, the difference between the two masses is the mass
of H2 produced.
H2 mass = 61.632 g S 61.338 g = 0.294 g
1L
H2 volume = 0.294 g H2 x = 3.27 L H2
0.0899 g H 2

2.108 Molecular mass = (8 x 12.011 amu) + (9 x 1.0079 amu) + (1 x 14.0067 amu)


+ (2 x 15.9994 amu) = 151.165 amu

8 x 12.011
2.109 mass % C = x 100 = 63.565%
151.165
9 x 1.0079
mass % H = x 100 = 6.0008%
151.165
14.0067
mass % N = x 100 = 9.2658%
151.165
2 x 15.9994
mass % O = x 100 = 21.168%
151.165

30
Chapter 2 S Atoms, Molecules, and Ions
______________________________________________________________________________

2.110 (a) Aspirin is likely a molecular compound because it is composed of only nonmetal
elements.
(b) Assume a 100.0 g sample of aspirin. It would contain: 60.00 g C, 4.48 g H, and
35.52 g O.
1 amu 1 C atom
60.00 g x _ 24
x = 3.008 x 1024 C atoms
1.6605 x 10 g 12.011 amu
1 amu 1 H atom
4.48 g x _ 24
x = 2.68 x 1024 H atoms
1.6605 x 10 g 1.008 amu
1 amu 1 O atom
35.52 g x _ 24
x = 1.337 x 1024 O atoms
1.6605 x 10 g 15.999 amu
The atom ratio in aspirin is:
24
C 3.008 x 1024 H 2.68 x 1024 O1.337 x 1024 , divide each subscript by 1 x 10
C3.008 H2.68 O1.337 , divide each subscript by the smallest, 1.337
C3.008 / 1.337 H2.68 / 1.337 O1.337 / 1.337
C2.25H2 O, multiply each subscript by 4
C(2.25 x 4) H(2 x 4) O(1 x 4)
C9H8O4

2.111 (a) Because X reacts by losing electrons, it is likely to be a metal.


(b) Because Y reacts by gaining electrons, it is likely to be a nonmetal.
(c) X2Y3
(d) X is likely to be in group 3A and Y is likely to be in group 6A.

2.112 65.39 amu = (0.4863 x 63.929 amu) + (0.2790 x Z) + (0.0410 x 66.927 amu)
+ (0.1875 x 67.925 amu) + (0.0062 x 69.925 amu)
Solve for Z.
65.39 amu = 47.00 amu + (0.2790 x Z)
65.39 amu S 47.00 amu = 18.39 amu = 0.2790 x Z
18.39 amu/0.2790 = Z
Z = 65.91 amu for 66Zn

31
32
3 Formulas, Equations, and Moles

3.1 2 KClO3 → 2 KCl + 3 O2

3.2 (a) C6H12O6 → 2 C2H6O + 2 CO2


(b) 4 Fe + 3 O2 → 2 Fe2O3
(c) 4 NH3 + Cl2 → N2H4 + 2 NH4Cl

3.3 3 A2 + 2 B → 2 BA3

3.4 (a) Fe2O3: 2(55.85) + 3(16.00) = 159.7 amu


(b) H2SO4: 2(1.01) + 1(32.07) + 4(16.00) = 98.1 amu
(c) C6H8O7: 6(12.01) + 8(1.01) + 7(16.00) = 192.1 amu
(d) C16H18N2O4S: 16(12.01) + 18(1.01) + 2(14.01) + 4(16.00) + 1(32.07) = 334.4 amu

3.5 Fe2O3(s) + 3 CO(g) → 2 Fe(s) + 3 CO2(g)


3 mol CO
0.500 mol Fe2 O3 x = 1.50 mol CO
1 mol Fe2 O3

3.6 C5H11NO2S: 5(12.01) + 11(1.01) + 1(14.01) + 2(16.00) + 1(32.07) = 149.24 amu

3.7 C9H8O4, 180.2 amu; 500 mg = 500 x 10-3 g = 0.500 g


1 mol
0.500 g x = 2.77 x 10_ 3 mol aspirin
180.2 g
6.02 x 1023 molecules
2.77 x 10-3 mol x = 1.67 x 1021 aspirin molecules
1 mol

3.8 salicylic acid, C7H6O3, 138.1 amu; acetic anhydride, C4H6O3, 102.1 amu
aspirin, C9H8O4, 180.2 amu; acetic acid, C2H4O2, 60.1 amu

1 mol C7 H 6 O3 1 mol C4 H 6 O3 102.1 g C4 H 6 O3


4.50 g C7H6O3 x x x = 3.33 g
138.1 g C7 H 6 O3 1 mol C7 H 6 O3 1 mol C4 H 6 O3
C4H6O3

1 mol C7 H 6 O3 1 mol C9 H8 O 4 180.2 g C9 H8 O4


4.50 g C7H6O3 x x x = 5.87 g
138.1 g C7 H 6 O3 1 mol C7 H6 O3 1 mol C9 H8 O 4
C9H8O4

1 mol C7 H 6 O3 1 mol C2 H 4 O 2 60.1 g C2 H 4 O 2


4.50 g C7H6O3 x x x = 1.96 g
138.1 g C7 H 6 O3 1 mol C7 H 6 O3 1 mol C2 H 4 O2

33
C2H4O2

3.9 C2H4, 28.1 amu; C2H6O, 46.1 amu

1 mol C2 H 4 1 mol C2 H6 O 46.1 g C2 H6 O


4.6 g C2 H4 x x x = 7.5 g C2H6O
28.1 g C2 H 4 1 mol C2 H 4 1 mol C2 H6 O
(theoretical yield)
Actual yield 4.7 g
Percent yield = x 100 % = x 100 % = 63 %
Theoretical yield 7.5 g

3.10 CH4, 16.04 amu; CH2Cl2, 84.93 amu; 1.85 kg = 1850 g

1 mol CH 4 1 mol CH 2 Cl2 84.93 g CH 2 Cl2


1850 g CH4 x x x = 9800 g CH2Cl2
16.04 g CH 4 1 mol CH 4 1 mol CH 2 Cl2
(theoretical yield)
Actual yield = (9800 g)(0.431) = 4220 g CH2Cl2

3.11 Li2O, 29.9 amu: 65 kg = 65,000 g; H2O, 18.0 amu: 80.0 kg = 80,000 g
1 mol Li 2 O
65,000 g Li2O x = 2.17 x 103 mol Li2O
29.9 g Li 2 O
1 mol H 2 O
80,000 g H2O x = 4.44 x 103 mol H2O
18.0 g H 2 O
The reaction stoichiometry between Li2O and H2O is one to one. There are twice as many
moles of H2O as there are moles of Li2O. Therefore, Li2O is the limiting reactant.
(4.44 x 103 mol - 2.17 x 103 mol) = 2.27 x 103 mol H2O remaining
18.0 g H 2 O
2.27 x 103 mol H2O x = 40,860 g H2O = 40.9 kg = 41 kg H2O
1 mol H 2 O

3.12 LiOH, 23.9 amu; CO2, 44.0 amu


1 mol LiOH 1 mol CO2 44.0 g CO 2
500.0 g LiOH x x x = 921 g CO2
23.9 g LiOH 1 mol LiOH 1 mol CO 2

3.13 (a) A + B2 → AB2


There is a 1:1 stoichiometry between the two reactants. A is the limiting reactant because
there are fewer reactant A's than there are reactant B2's.
(b) 1.0 mol of AB2 can be made from 1.0 mol of A and 1.0 mol of B2.

3.14 (a) 125 mL = 0.125 L; (0.20 mol/L)(0.125 L) = 0.025 mol NaHCO3


(b) 650.0 mL = 0.6500 L; (2.50 mol/L)(0.6500 L) = 1.62 mol H2SO4

3.15 (a) NaOH, 40.0 amu; 500.0 mL = 0.5000 L


mol NaOH 40.0 g NaOH
1.25 x 0.500 L x = 25.0 g NaOH
L 1 mol NaOH
(b) C6H12O6, 180.2 amu
mol C6 H12 O6 180.2 g C6 H12 O6
0.250 x 1.50 L x = 67.6 g C6 H12 O6
L 1 mol C6 H12 O6

34
Chapter 3 - Formulas, Equations, and Moles
______________________________________________________________________________

3.16 C6H12O6, 180.2 amu;


1 mol C6 H12 O6
25.0 g C6H12O6 x = 0.1387 mol C6H12O6
180.2 g C6 H12 O6
1L
0.1387 mol x = 0.69 L; 0.69 L = 690 mL
0.20 mol

3.17 C27H46O, 386.7 amu; 750 mL = 0.750 L


mol C27 H46 O 386.7 g C27 H46 O
0.005 x 0.750 L x = 1 g C27 H 46 O
L 1 mol C27 H 46 O

x 3.50 M x 75.0 mL
3.18 Mi x Vi = Mf x Vf; Mf = Mi Vi = = 0.656 M
Vf 400.0 mL

3.19 Mi x Vi = Mf x Vf; Mf x V f = 0.500 M x 250.0 mL = 6.94 mL


Vi =
Mi 18.0 M
Dilute 6.94 mL of 18.0 M H2SO4 with enough water to make 250.0 mL of solution. The
resulting solution will be 0.500 M H2SO4.

3.20 50.0 mL = 0.0500 L; (0.100 mol/L)(0.0500 L) = 5.00 x 10-3 mol NaOH


1 mol H 2 SO 4
5.00 x 10-3 mol NaOH x = 2.50 x 10-3 mol H2SO4
2 mol NaOH
1L
volume = 2.50 x 10_ 3 mol x = 0.0100 L; 0.0100 L = 10.0 mL H2SO4
0.250 mol

3.21 HNO3(aq) + KOH(aq) → KNO3(aq) + H2O(l)


25.0 mL = 0.0250 L and 68.5 mL = 0.0685 L
mol KOH 1 mol HNO3
0.150 x 0.0250 L x = 3.75 x 10_ 3 mol HNO3
L 1 mol KOH
3.75 x 10_ 3 mol
HNO3 molarity = = 5.47 x 10_ 2 M
0.0685 L

3.22 From the reaction stoichiometry, moles NaOH = moles CH3CO2H


(0.200 mol/L)(0.0947 L) = 0.018 94 mol NaOH = 0.018 94 mol CH3CO2H
0.018 94 mol
molarity = = 0.758 M
0.0250 L

3.23 For dimethylhydrazine, C2H8N2, divide each subscript by 2 to obtain the empirical
formula. The empirical formula is CH4N. C2H8N2, 60.1 amu or 60.1 g/mol
2 x 12.0 g
% C= x 100 % = 39.9 %
60.1 g
8 x 1.01 g
% H= x 100 % = 13.4 %
60.1 g

35
Chapter 3 - Formulas, Equations, and Moles
______________________________________________________________________________

2 x 14.0 g
% N= x 100 % = 46.6 %
60.1 g

3.24 Assume a 100.0 g sample. From the percent composition data, a 100.0 g sample contains
14.25 g C, 56.93 g O, and 28.83 g Mg.
1 mol C
14.25 g C x = 1.19 mol C
12.0 g C
1 mol O
56.93 g O x = 3.56 mol O
16.0 g O
1 mol Mg
28.83 g Mg x = 1.19 mol Mg
24.3 g Mg
Mg1.19C1.19O3.56; divide each subscript by the smallest, 1.19.
Mg1.19 / 1.19C1.19 / 1.19O3.56 / 1.19
The empirical formula is MgCO3.

1 mol H2 O 2 mol H
3.25 1.161 g H2O x x = 0.129 mol H
18.0 g H 2 O 1 mol H2 O
1 mol CO2 1 mol C
2.818 g CO2 x x = 0.0640 mol C
44.0 g CO2 1 mol CO2
1.01 g H
0.129 mol H x = 0.130 g H
1 mol H
12.0 g C
0.0640 mol C x = 0.768 g C
1 mol C
1.00 g total - (0.130 g H + 0.768 g C) = 0.102 g O
1 mol O
0.102 g O x = 0.006 38 mol O
16.0 g O
C0.0640H0.129O0.006 38; divide each subscript by the smallest, 0.006 38.
C0.0640 / 0.006 38H0.129 / 0.006 38O0.006 38 / 0.006 38
C10.03H20.22O1 The empirical formula is C10H20O.

3.26 The empirical formula is CH2O, 30 amu: molecular mass = 150 amu.
molecular mass 150 amu
= = 5 ; therefore
empirical formula mass 30 amu
molecular formula = 5 x empirical formula = C(5 x 1)H(5 x 2)O(5 x 1) = C5H10O5

3.27 (a) Assume a 100.0 g sample. From the percent composition data, a 100.0 g sample
contains 21.86 g H and 78.14 g B.
1 mol H
21.86 g H x = 21.6 mol H
1.01 g H
1 mol B
78.14 g B x = 7.24 mol B
10.8 g B
B7.24 H21.6; divide each subscript by the smaller, 7.24.

36
Chapter 3 - Formulas, Equations, and Moles
______________________________________________________________________________

B7.24 / 7.24 H21.6 / 7.24 The empirical formula is BH3, 13.8 amu.
27.7 amu / 13.8 amu = 2; molecular formula = B(2 x 1)H(2 x 3) = B2H6.
(b) Assume a 100.0 g sample. From the percent composition data, a 100.0 g sample
contains 6.71 g H, 40.00 g C, and 53.28 g O.
1 mol H
6.71 g H x = 6.64 mol H
1.01 g H
1 mol C
40.00 g C x = 3.33 mol C
12.0 g C
1 mol O
53.28 g O x = 3.33 mol O
16.0 g O
C3.33 H6.64 O3.33; divide each subscript by the smallest, 3.33.
C3.33 / 3.33 H6.64 / 3.33 O3.33 / 3.33 The empirical formula is CH2O, 30.0 amu.
90.08 amu / 30.0 amu = 3; molecular formula = C(3 x 1)H(3 x 2)O(3 x 1) = C3H6O3

3.28 Main sources of error in calculating Avogadro's number by spreading oil on a pond are:
(i) the assumption that the oil molecules are tiny cubes
(ii) the assumption that the oil layer is one molecule thick
(iii) the assumption of a molecular mass of 200 for the oil

3.29 area of oil = 2.0 x 107 cm2


volume of oil = 4.9 cm3 = area x 4 l = (2.0 x 107 cm2) x 4 l
4.9 cm3
l= 7 2
= 6.125 x 10-8 cm
(2.0 x 10 cm )(4)

area of oil = 2.0 x 107 cm2 = l2 x N = (6.125 x 10-8 cm)2 x N

2.0 x 107 cm2


N= _8 2
= 5.33 x 1021 oil molecules
(6.125 x 10 cm )
1 mol oil
moles of oil = (4.9 cm3) x (0.95 g/cm3) x = 0.0233 mol oil
200 g oil
5.33 x 1021 molecules
Avogadro's number = = 2.3 x 1023 molecules/mole
0.0233 mol

Understanding Key Concepts

3.30 The concentration of a solution is cut in half when the volume is doubled. This is best
represented by box (b).

3.31 (c) 2 A + B2 → A2B2

3.32 The molecular formula for cytosine is C4H5N3O.


4 C 1 CO 2
mol CO2 = 0.001 mol cyt x x = 0.004 mol CO2
cyt C

37
Chapter 3 - Formulas, Equations, and Moles
______________________________________________________________________________

5 H 1 H2 O
mol H2O = 0.001 mol cyt x x = 0.0025 mol H2O
cyt 2 H

3.33 reactants, box (d), and products, box (c)

3.34 C17H18F3NO 17(12.01) + 18(1.01) + 3(19.00) + 1(14.01) + 1(16.00) = 309.36 amu

3.35 Because the two volumes are equal (let the volume = y L), the concentrations are
proportional to the number of solute ions.
y L 8 OH _
OH- concentration = 1.00 M x x = 0.67 M
12 H+ yL

3.36 (a) A2 + 3 B2 → 2 AB3; B2 is the limiting reactant because it is completely consumed.


(b) For 1.0 mol of A2, 3.0 mol of B2 are required. Because only 1.0 mol of B2 is
available, B2 is the limiting reactant.
2 mol AB3
1 mol B2 x = 2/3 mol AB3
3 mol B2

O2
3.37 CxHy → 3 CO2 + 4 H2O; x is equal to the coefficient for CO2 and y is equal to 2
times the coefficient for H2O. The empirical formula for the hydrocarbon is C3H8.

Additional Problems
Balancing Equations

3.38 Equation (b) is balanced, (a) is not balanced .

3.39 (a) and (c) are not balanced, (b) is balanced.


(a) 2 Al + Fe2O3 → Al2O3 + 2 Fe (balanced)
(c) 4 Au + 8 NaCN + O2 + 2 H2O → 4 NaAu(CN)2 + 4 NaOH (balanced)

3.40 (a) Mg + 2 HNO3 → H2 + Mg(NO3)2


(b) CaC2 + 2 H2O → Ca(OH)2 + C2H2
(c) 2 S + 3 O2 → 2 SO3
(d) UO2 + 4 HF → UF4 + 2 H2O

3.41 (a) 2 NH4NO3 → 2 N2 + O2 + 4 H2O


(b) C2H6O + O2 → C2H4O2 + H2O
(c) C2H8N2 + 2 N2O4 → 3 N2 + 2 CO2 + 4 H2O

Molecular Masses and Moles

3.42 Hg2Cl2: 2(200.59) + 2(35.45) = 472.1 amu

38
Chapter 3 - Formulas, Equations, and Moles
______________________________________________________________________________

C4H8O2: 4(12.01) + 8(1.01) + 2(16.00) = 88.1 amu


CF2Cl2: 1(12.01) + 2(19.00) + 2(35.45) = 120.9 amu

3.43 (a) (1 x 30.97 amu) + (Y x 35.45 amu) = 137.3 amu; Solve for Y; Y = 3.
The formula is PCl3.
(b) (10 x 12.01 amu) + (14 x 1.008 amu) + (Z x 14.01 amu) = 162.2 amu.
Solve for Z; Z = 2. The formula is C10H14N2.

3.44 One mole equals the atomic mass or molecular mass in grams.
(a) Ti, 47.88 g (b) Br2, 159.81 g (c) Hg, 200.59 g (d) H2O, 18.02 g

1 mol Cr
3.45 (a) 1.00 g Cr x = 0.0192 mol Cr
52.0 g Cr
1 mol Cl2
(b) 1.00 g Cl2 x = 0.0141 mol Cl2
70.9 g Cl2
1 mol Au
(c) 1.00 g Au x = 0.005 08 mol Au
197.0 g Au
1 mol NH3
(d) 1.00 g NH3 x = 0.0588 mol NH3
17.0 g NH3

3.46 There are 2 ions per each formula unit of NaCl. (2.5 mol)(2 mol ions/mol) = 5.0 mol ions

3.47 There are 2 K+ ions per each formula unit of K2SO4.


2 mol K +
1.45 mol K 2 SO4 x = 2.90 mol K +
1 mol K 2 SO4

3.48 There are 3 ions (one Mg2+ and 2 Cl-) per each formula unit of MgCl2.
MgCl2, 95.2 amu
1 mol MgCl 2 3 mol ions
27.5 g MgCl2 x x = 0.867 mol ions
95.2 g MgCl 2 1 mol MgCl 2

3.49 There are 3 F- anions per each formula unit of AlF3.


AlF3, 84.0 amu
1 mol AlF3 3 mol anions
35.6 g AlF3 x x = 1.27 mol F-
84.0 g AlF3 1 mol AlF3

3.28 g
3.50 Molar mass = = 119 g / mol ; molecular mass = 119 amu.
0.0275 mol

221.6 g
3.51 Molar mass = = 386.7 g/mol; molecular mass = 386.7 amu.
0.5731 mol

3.52 FeSO4 , 151.9 amu; 300 mg = 0.300 g

39
Chapter 3 - Formulas, Equations, and Moles
______________________________________________________________________________

1 mol FeSO 4
0.300 g FeSO 4 x = 1.97 x 10_ 3 mol FeSO 4
151.9 g FeSO 4
6.02 x 1023 Fe(II) atoms
1.97 x 10_ 3 mol FeSO4 x = 1.19 x 1021 Fe(II) atoms
1 mol FeSO4

1 mol C 6.02 x 1023 C atoms


3.53 0.0001 g C x x = 5 x 1018 C atoms
12.0 g C 1 mol C

3.54 C8H10N4O2, 194.2 amu; 125 mg = 0.125 g


1 mol caffeine
0.125 g caffeine x = 6.44 x 10-4 mol caffeine
194.2 g caffeine
1 mol caffeine 6.022 x 1023 molecules
0.125 g caffeine x x = 3.88 x 1020 caffeine
194.2 g caffeine 1 mol
molecules

g 6.02 x 1023 eggs


3.55 45 x = 2.7 x 1025 g / mol of eggs
egg 1 mol eggs

1 mol Li
3.56 (a) 1.0 g Li x = 0.14 mol Li
6.94 g Li
1 mol Au
(b) 1.0 g Au x = 0.0051 mol Au
197.0 g Au
(c) penicillin G: C16H17N2O4SK, 372.5 amu
1 mol penicillin G
1.0 g x = 2.7 x 10-3 mol penicillin G
372.5 g penicillin G

23.0 g Na
3.57 (a) 0.0015 mol Na x = 0.034 g Na
1 mol Na
207.2 g Pb
(b) 0.0015 mol Pb x = 0.31 g Pb
1 mol Pb
(c) C16H13ClN2O, 284.7 amu

284.7 g diazepam
0.0015 mol diazepam x = 0.43 g diazepam
1 mol diazepam

Stoichiometry Calculations
79.88 kg TiO 2
3.58 TiO2, 79.88 amu; 100.0 kg Ti x = 166.8 kg TiO2
47.88 kg Ti
2(55.85 g) Fe
3.59 Fe2O3, 159.7 amu; % Fe = x 100 % = 69.94 %
159.7 g Fe2 O3

40
Chapter 3 - Formulas, Equations, and Moles
______________________________________________________________________________

mass Fe = (0.6994)(105 kg) = 73.4 kg

3.60 (a) 2 Fe2O3 + 3 C → 4 Fe + 3 CO2


1 mol Fe 2 O3 3 mol C
(b) Fe2O3, 159.7 amu; 525 g Fe2O3 x x = 4.93 mol C
159.7 g Fe2 O3 2 mol Fe2 O3
12.01 g C
(c) 4.93 mol C x = 59.2 g C
1 mol C

3.61 (a) Fe2O3 + 3 CO → 2 Fe + 3 CO2


(b) Fe2O3, 159.7 amu; CO, 28.01 amu
1 mol Fe2 O3 3 mol CO 28.01 g CO
3.02 g Fe2 O3 x x x = 1.59 g CO
159.7 g Fe2 O3 1 mol Fe2 O3 1 mol CO
3 mol CO 28.01 g CO
(c) 1.68 mol Fe2 O3 x x = 141 g CO
1 mol Fe2 O3 1 mol CO

3.62 (a) 2 Mg + O2 → 2 MgO


(b) Mg, 24.30 amu; O2, 32.00 amu; MgO, 40.30 amu
1 mol Mg 1 mol O2 32.00 g O2
25.0 g Mg x x x = 16.5 g O2
24.30 g Mg 2 mol Mg 1 mol O2
1 mol Mg 2 mol MgO 40.30 g MgO
25.0 g Mg x x x = 41.5 g MgO
24.30 g Mg 2 mol Mg 1 mol MgO
1 mol O2 2 mol Mg 24.30 g Mg
(c) 25.0 g O2 x x x = 38.0 g Mg
32.00 g O2 1 mol O2 1 mol Mg
1 mol O2 2 mol MgO 40.30 g MgO
25.0 g O2 x x x = 63.0 g MgO
32.00 g O2 1 mol O2 1 mol MgO

3.63 C2H4 + H2O → C2H6O; C2H4, 28.05 amu; H2O, 18.02 amu; C2H6O, 46.07 amu
1 mol C2 H 4 28.05 g C2 H 4
(a) 0.133 mol H 2 O x x = 3.73 g C2 H 4
1 mol H 2 O 1 mol C2 H 4
1 mol C2 H 6 O 46.07 g C2 H 6 O
0.133 mol H 2 O x x = 6.13 g C2 H 6 O
1 mol H 2 O 1 mol C2 H 6 O

1 mol H 2 O 18.02 g H 2 O
(b) 0.371 mol C2 H 4 x x = 6.69 g H 2 O
1 mol C2 H 4 1 mol H 2 O
1 mol C2 H 6 O 46.07 g C2 H 6 O
0.371 mol C2 H 4 x x = 17.1 g C2 H 6 O
1 mol C2 H 4 1 mol C2 H 6 O
3.64 (a) 2 HgO → 2 Hg + O2
(b) HgO, 216.6 amu; Hg, 200.6 amu; O2, 32.0 amu

1 mol HgO 2 mol Hg 200.6 g Hg


45.5 g HgO x x x = 42.1 g Hg
216.6 g HgO 2 mol HgO 1 mol Hg

41
Chapter 3 - Formulas, Equations, and Moles
______________________________________________________________________________

1 mol HgO 1 mol O2 32.00 g O2


45.5 g HgO x x x = 3.36 g O2
216.6 g HgO 2 mol HgO 1 mol O2

1 mol O2 2 mol HgO 216.6 g HgO


(c) 33.3 g O2 x x x = 451 g HgO
32.00 g O2 1 mol O2 1 mol HgO

3.65 5.60 kg = 5600 g; TiCl4, 189.7 amu; TiO2, 79.88 amu


1 mol TiCl 4 1 mol TiO 2 79.88 g TiO 2
5600 g TiCl 4 x x x = 2358 g TiO 2 = 2.36 kg TiO 2
189.7 g TiCl 4 1 mol TiCl 4 1 mol TiO 2

1 mol Ag
3.66 2.00 g Ag x = 0.0185 mol Ag
107.9 g Ag
1 mol Cl
0.657 g Cl x = 0.0185 mol Cl
35.45 g Cl
Ag0.0185Cl0.0185 Divide both subscripts by 0.0185. The empirical formula is AgCl.

1 mol Al 1 mol O
3.67 5.0 g Al x = 0.19 mol Al;4.45 g O x = 0.28 mol O
27.0 g Al 16.0 g O
Al0.19O0.28; divide both subscripts by the smaller, 0.19.
Al0.19 / 0.19O0.28 / 0.19
Al1O1.5; multiply both subscripts by 2 to obtain integers. The empirical formula is Al2O3.

Limiting Reactants and Reaction Yield

3 mol H 2
3.68 3.44 mol N2 x = 10.3 mol H2 required.
1 mol N2
Because there is only 1.39 mol H2, H2 is the limiting reactant.
2 mol NH3 17.03 g NH3
1.39 mol H2 x x = 15.8 g NH3
3 mol H 2 1 mol NH3
1 mol N 2 28.01 g N 2
1.39 mol H2 x x = 13.0 g N2 reacted
3 mol H 2 1 mol N2
28.01 g N2
3.44 mol N2 x = 96.3 g N2 initially
1 mol N 2
(96.3 g - 13.0 g) = 83.3 g N2 left over
3.69 H2, 2.016 amu; Cl2, 70.91 amu; HCl 36.46 amu
1 mol H 2
3.56 g H 2 x = 1.77 mol H 2
2.016 g H 2
1 mol Cl2
8.94 g Cl2 x = 0.126 mol Cl2
70.91 g Cl2
Because the reaction stoichiometry between H2 and Cl2 is one to one, Cl2 is the limiting
reactant.

42
Chapter 3 - Formulas, Equations, and Moles
______________________________________________________________________________

2 mol HCl 36.46 g HCl


0.126 mol Cl2 x x = 9.19 g HCl
1 mol Cl2 1 mol HCl

3.70 C2H4, 28.05 amu; Cl2, 70.91 amu; C2H4Cl2, 98.96 amu
1 mol C2 H 4
15.4 g C2H4 x = 0.549 mol C2H4
28.05 g C2 H 4
1 mol Cl2
3.74 g Cl2 x = 0.0527 mol Cl2
70.91 g Cl2
Because the reaction stoichiometry between C2H4 and Cl2 is one to one, Cl2 is the limiting
reactant.
1 mol C2 H 4 Cl2 98.96 g C2 H 4 Cl2
0.0527 mol Cl2 x x = 5.22 g C2H4Cl2
1 mol Cl2 1 mol C2 H 4 Cl2

3.71 (a) NaCl, 58.44 amu; AgNO3, 169.9 amu; AgCl, 143.3 amu; NaNO3, 85.00 amu
NaCl + AgNO3 → AgCl + NaNO3
1 mol NaCl
1.3 g NaCl x = 0.022 mol NaCl
58.44 g NaCl
1 mol AgNO3
3.5 g AgNO3 x = 0.021 mol AgNO3
169.9 g AgNO3
Because the reaction stoichiometry between NaCl and AgNO3 is one to one, AgNO3 is
the limiting reactant.
1 mol AgCl 143.3 g AgCl
0.021 mol AgNO3 x x = 3.0 g AgCl
1 mol AgNO3 1 mol AgCl
1 mol NaNO3 85.00 g NaNO3
0.021 mol AgNO3 x x = 1.8 g NaNO3
1 mol AgNO3 1 mol NaNO3
1 mol NaCl 58.44 g NaCl
0.021 mol AgNO3 x x = 1.2 g NaCl reacted
1 mol AgNO3 1 mol NaCl
(1.3 g - 1.2 g) = 0.1 g NaCl left over

(b) BaCl2, 208.2 amu; H2SO4, 98.08 amu; BaSO4, 233.4 amu; HCl, 36.46 amu
BaCl2 + H2SO4 → BaSO4 + 2 HCl
1 mol BaCl2
2.65 g BaCl2 x = 0.0127 mol BaCl2
208.2 g BaCl2
1 mol H 2 SO4
6.78 g H2SO4 x = 0.0691 mol H2SO4
98.08 g H2 SO4
Because the reaction stoichiometry between BaCl2 and H2SO4 is one to one, BaCl2 is the
limiting reactant.
1 mol BaSO4 233.4 g BaSO4
0.0127 mol BaCl2 x x = 2.96 g BaSO4
1 mol BaCl2 1 mol BaSO4
2 mol HCl 36.46 g HCl
0.0127 mol BaCl2 x x = 0.926 g HCl
1 mol BaCl2 1 mol HCl

43
Chapter 3 - Formulas, Equations, and Moles
______________________________________________________________________________

1 mol H 2 SO 4 98.1 g H 2 SO 4
0.0127 mol BaCl2 x x = 1.25 g H2SO4 reacted
1 mol BaCl2 1 mol H 2 SO 4
(6.78 g - 1.25 g) = 5.53 g H2SO4 left over

3.72 CaCO3, 100.1 amu; HCl, 36.46 amu


CaCO3 + 2 HCl → CaCl2 + H2O + CO2
1 mol CaCO3
2.35 g CaCO3 x = 0.0235 mol CaCO3
100.1 g CaCO3
1 mol HCl
2.35 g HCl x = 0.0645 mol HCl
36.46 g HCl
The reaction stoichiometry is 1 mole of CaCO3 for every 2 moles of HCl. For 0.0235 mol
CaCO3, we only need 2(0.0235 mol) = 0.0470 mol HCl. We have 0.0645 mol HCl;
therefore CaCO3 is the limiting reactant.
1 mol CO2 22.4 L
0.0235 mol CaCO3 x x = 0.526 L CO2
1 mol CaCO3 1 mol CO2

3.73 2 NaN3 → 3 N2 + 2 Na; NaN3, 65.01 amu; N2, 28.01 amu


1 mol NaN3 3 mol N 2 47.0 L
38.5 g NaN3 x x x = 41.8 L
65.01 g NaN3 2 mol NaN3 1.00 mol N 2

3.74 CH3CO2H + C5H12O → C7H14O2 + H2O


CH3CO2H, 60.05 amu; C5H12O, 88.15 amu; C7H14O2, 130.19 amu
1 mol CH3 CO2 H
3.58 g CH3 CO2 H x = 0.0596 mol CH3 CO2 H
60.05 g CH3 CO2 H
1 mol C5 H12 O
4.75 g C5 H12 O x = 0.0539 mol C5 H12 O
88.15 g C5 H12 O

Because the reaction stoichiometry between CH3CO2H and C5H12O is one to one,
isopentyl alcohol (C5H12O) is the limiting reactant.
1 mol C7 H14 O2 130.19 g C7 H14 O2
0.0539 mol C5 H12 O x x = 7.02 g C7 H14 O2
1 mol C5 H12 O 1 mol C7 H14 O2
7.02 g C7H14O2 is the theoretical yield. Actual yield = (7.02 g)(0.45) = 3.2 g.

3.75 K2PtCl4 + 2 NH3 → 2 KCl + Pt(NH3)2Cl2


K2PtCl4, 415.1 amu; NH3, 17.03 amu; Pt(NH3)2Cl2, 300.0 amu
1 mol K 2 PtCl4
55.8 g K2PtCl4 x = 0.134 mol K2PtCl4
415.1 g K 2 PtCl4
1 mol NH3
35.6 g NH3 x = 2.09 mol NH3
17.03 g NH3
Only 2(0.134) = 0.268 mol NH3 are needed to react with 0.134 mol K2PtCl4. Therefore,
the NH3 is in excess and K2PtCl4 is the limiting reactant.

44
Chapter 3 - Formulas, Equations, and Moles
______________________________________________________________________________

1 mol Pt( NH3 )2 Cl2 300.0 g Pt( NH3 )2 Cl2


0.134 mol K2PtCl4 x x = 40.2 g Pt(NH3)2Cl2
1 mol K 2 PtCl 4 1 mol Pt( NH3 )2 Cl2
40.2 g Pt(NH3)2Cl2 is the theoretical yield.
Actual yield = (40.2 g)(0.95) = 38 g Pt(NH3)2Cl2.

3.76 CH3CO2H + C5H12O → C7H14O2 + H2O


CH3CO2H, 60.05 amu; C5H12O, 88.15 amu; C7H14O2, 130.19 amu
1 mol CH3 CO2 H
1.87 g CH3CO2H x = 0.0311 mol CH3CO2H
60.05 g CH3 CO2 H
1 mol C5 H12 O
2.31 g C5H12O x = 0.0262 mol C5H12O
88.15 g C5 H12 O
Because the reaction stoichiometry between CH3CO2H and C5H12O is one to one,
isopentyl alcohol (C5H12O) is the limiting reactant.
1 mol C7 H14 O2 130.19 g C7 H14 O2
0.0262 mol C5H12O x x = 3.41 g C7H14O2
1 mol C5 H12 O 1 mol C7 H14 O2
3.41 g C7H14O2 is the theoretical yield.
Actual yield 2.96 g
% Yield = x 100% = x 100% = 86.8%
Theoretical yield 3.41 g

3.77 K2PtCl4 + 2 NH3 → 2 KCl + Pt(NH3)2Cl2


K2PtCl4, 415.1 amu; NH3, 17.03 amu; Pt(NH3)2Cl2, 300.0 amu
1 mol K 2 PtCl 4
3.42 g K2PtCl4 x = 0.008 24 mol K2PtCl4
415.1 g K 2 PtCl 4
1 mol NH3
1.61 g NH3 x = 0.0945 mol NH3
17.03 g NH3

Only 2 x (0.008 24) = 0.0165 mol of NH3 are needed to react with 0.008 24 mol K2PtCl4.
Therefore, the NH3 is in excess and K2PtCl4 is the limiting reactant.
1 mol Pt( NH3 )2 Cl2 300.0 g Pt( NH3 )2 Cl2
0.008 24 mol K2PtCl4 x x = 2.47 g Pt(NH3)2Cl2
1 mol K 2 PtCl 4 1 mol Pt( NH3 )2 Cl2
2.47 g Pt(NH3)2Cl2 is the theoretical yield. 2.08 g Pt(NH3)2Cl2 is the actual yield.
Actual yield 2.08 g
% Yield = x 100% = x 100% = 84.2%
Theoretical yield 2.47 g

Molarity, Solution Stoichiometry, Dilution, and Titration

1.200 mol HNO3


3.78 (a) 35.0 mL = 0.0350 L; x 0.0350 L = 0.0420 mol HNO3
L
0.67 mol C6 H12 O6
(b) 175 mL = 0.175 L; x 0.175 L = 0.12 mol C6 H12 O6
L

3.79 (a) C2H6O, 46.07 amu; 250.0 mL = 0.2500 L

45
Chapter 3 - Formulas, Equations, and Moles
______________________________________________________________________________

0.600 mol C2 H6 O
x 0.2500 L = 0.150 mol C2H6O
L
(0.150 mol)(46.07 g/mol) = 6.91 g C2H6O

(b) H3BO3, 61.83 amu; 167 mL = 0.167 L


0.200 mol H3 BO3
x 0.167 L = 0.0334 mol H3BO3
L
(0.0334 mol)(61.83 g/mol) = 2.07 g H3BO3

3.80 BaCl2, 208.2 amu


1 mol BaCl2
15.0 g BaCl2 x = 0.0720 mol BaCl2
208.2 g BaCl2
1.0 L
0.0720 mol x = 0.16 L; 0.16 L = 160 mL
0.45 mol

1.00 L
3.81 0.0171 mol KOH x = 0.0489 L; 0.0489 L = 48.9 mL
0.350 mol KOH

3.82 NaCl, 58.4 amu; 400 mg = 0.400 g; 100 mL = 0.100 L


1 mol NaCl
0.400 g NaCl x = 0.006 85 mol NaCl
58.4 g NaCl
0.006 85 mol
molarity = = 0.0685 M
0.100 L

3.83 C6H12O6, 180.2 amu; 90 mg = 0.090 g; 100 mL = 0.100 L


1 mol C6 H12 O6
0.090 g C6 H12 O6 x = 0.000 50 mol C6 H12 O6
180.2 g C6 H12 O6
0.000 50 mol
molarity = = 0.0050 M = 5.0 x 10-3 M
0.100 L

3.84 NaCl, 58.4 amu; KCl, 74.6 amu; CaCl2, 111.0 amu; 500 mL = 0.500 L
1 mol NaCl
4.30 g NaCl x = 0.0736 mol NaCl
58.4 g NaCl
1 mol KCl
0.150 g KCl x = 0.002 01 mol KCl
74.6 g KCl
1 mol CaCl2
0.165 g CaCl2 x = 0.001 49 mol CaCl2
111.0 g CaCl2
0.0736 mol + 0.002 01 mol + 2(0.001 49 mol) = 0.0786 mol Cl-

46
Chapter 3 - Formulas, Equations, and Moles
______________________________________________________________________________

0.0736 mol
Na+ molarity = = 0.147 M
0.500 L
0.001 49 mol
Ca2+ molarity = = 0.002 98 M
0.500 L
0.002 01 mol
K+ molarity = = 0.004 02 M
0.500 L
0.0786 mol
Cl- molarity = = 0.157 M
0.500 L

1 mol Cu
3.85 3.045 g Cu x = 0.047 92 mol Cu; 50.0 mL = 0.0500 L
63.546 g Cu
0.047 92 mol
Cu(NO3)2 molarity = = 0.958 M
0.0500 L

3.86 Mf x Vf = Mi x Vi; Mi x Vi 12.0 M x 35.7 mL


Mf = = = 1.71 M HCl
Vf 250.0 mL

3.87 Mf x Vf = Mi x Vi; Mi x Vi = 0.0913 M x 70.00 mL = 426 mL


Vf =
Mf 0.0150 M

3.88 2 HBr(aq) + K2CO3(aq) → 2 KBr(aq) + CO2(g) + H2O(l)


K2CO3, 138.2 amu; 450 mL = 0.450 L
0.500 mol HBr
x 0.450 L = 0.225 mol HBr
L
1 mol K 2 CO3 138.2 g K 2 CO3
0.225 mol HBr x x = 15.5 g K2CO3
2 mol HBr 1 mol K 2 CO3
3.89 2 C4H10S + NaOCl → C8H18S2 + NaCl + H2O
C4H10S, 90.19 amu; 5.00 mL = 0.005 00 L

0.0985 mol NaOCl


x 0.005 00 L = 4.925 x 10-4 mol NaOCl
L
2 mol C4 H10 S 90.19 g C4 H10 S
4.925 x 10-4 mol NaOCl x x = 0.0888 g C4H10S
1 mol NaOCl 1 mol C4 H10 S

3.90 H2C2O4, 90.04 amu


1 mol H 2 C2 O4 2 mol KMnO 4
3.225 g H 2 C2 O4 x x = 0.0143 mol KMnO 4
90.04 g H 2 C2 O 4 5 mol H 2 C2 O4
1L
0.0143 mol x = 0.0572 L = 57.2 mL
0.250 mol

3.91 H2C2O4, 90.04 amu; 400.0 mL = 0.4000 L; 25.0 mL = 0.0250 L

47
Chapter 3 - Formulas, Equations, and Moles
______________________________________________________________________________

1 mol H 2 C2 O4
12.0 g H 2 C2 O4 x = 0.133 mol H 2 C2 O4
90.04 g H 2 C2 O4
0.133 mol
molarity = = 0.333 M H 2 C2 O4
0.4000 L
H2C2O4(aq) + 2 KOH(aq) → K2C2O4(aq) + 2 H2O(l)

0.333 mol C2 H 2 O4
x 0.0250 L = 0.008 32 mol H2C2O4
L
2 mol KOH
0.008 32 mol H2C2O4 x = 0.0166 mol KOH
1 mol H 2 C2 O4
1L
0.0166 mol x = 0.166 L; 0.166 L = 166 mL
0.100 mol

Formulas and Elemental Analysis

3.92 CH4N2O, 60.1 amu


12.0 g C
% C= x 100 % = 20.0 %
60.1 g
4 x 1.01 g H
% H= x 100 % = 6.72 %
60.1 g
2 x 14.0 g N
% N= x 100 % = 46.6 %
60.1 g
16.0 g O
% O= x 100 % = 26.6 %
60.1 g

3.93 (a) Cu2(OH)2CO3, 221.1 amu


2 x 63.5 g Cu
% Cu = x 100% = 57.4%
221.1 g
5 x 16.0 g O
%O= x 100% = 36.2%
221.1 g
12.0 g C
%C= x 100% = 5.43%
221.1 g
2 x 1.01 g H
%H= x 100% = 0.91%
221.1 g

(b) C8H9NO2, 151.2 amu


8 x 12.0 g C
%C= x 100% = 63.5%
151.2 g
9 x 1.01 g H
%H= x 100% = 6.01%
151.2 g

48
Chapter 3 - Formulas, Equations, and Moles
______________________________________________________________________________

14.0 g N
%N= x 100% = 9.26%
151.2 g
2 x 16.0 g O
%O= x 100% = 21.2%
151.2 g

(c) Fe4[Fe(CN)6]3, 859.2 amu


7 x 55.85 g Fe
% Fe = x 100% = 45.50%
859.2 g
18 x 12.01 g C
%C= x 100% = 25.16%
859.2 g
18 x 14.01 g N
%N= x 100% = 29.35%
859.2 g

3.94 Assume a 100.0 g sample. From the percent composition data, a 100.0 g sample contains
24.25 g F and 75.75 g Sn.
1 mol F
24.25 g F x = 1.276 mol F
19.00 g F
1 mol Sn
75.75 g Sn x = 0.6382 mol Sn
118.7 g Sn
Sn0.6382F1.276; divide each subscript by the smaller, 0.6382.
Sn0.6382 / 0.6382F1.276 / 0.6382 The empirical formula is SnF2.

3.95 (a) Assume a 100.0 g sample of ibuprofen. From the percent composition data, a 100.0 g
sample contains 75.69 g C, 15.51 g O, and 8.80 g H.
1 mol C
75.69 g C x = 6.302 mol C
12.01 g C
1 mol O
15.51 g O x = 0.9694 mol O
16.00 g O
1 mol H
8.80 g H x = 8.71 mol H
1.01 g H
C6.302H8.71O0.9694, divide each subscript by the smallest, 0.9694.
C6.302 / 0.9694H8.71 / 0.9694O0.9694 / 0.9694
C6.5H9O; multiply each subscript by 2 to obtain integers.
The empirical formula is C13H18O2.

(b) Assume a 100.0 g sample of tetraethyllead. From the percent composition data, a
100.0 g sample contains 29.71 g C, 6.23 g H, and 64.06 g Pb.
1 mol C
29.71 g C x = 2.474 mol C
12.01 g C
1 mol H
6.23 g H x = 6.17 mol H
1.01 g H

49
Chapter 3 - Formulas, Equations, and Moles
______________________________________________________________________________

1 mol Pb
64.06 g Pb x = 0.3092 mol Pb
207.2 g Pb
Pb0.3092C2.474H6.17; divide each subscript by the smallest, 0.3092.
Pb0.3092 / 0.3092C2.474 / 0.3092H6.17 / 0.3092 The empirical formula is PbC8H20.

(c) Assume a 100.0 g sample of zircon. From the percent composition data, a 100.0 g
sample contains 34.91 g O, 15.32 g Si, and 49.76 g Zr.
1 mol O
34.91 g O x = 2.182 mol O
16.00 g O
1 mol Si
15.32 g Si x = 0.5454 mol Si
28.09 g Si
1 mol Zr
49.76 g Zr x = 0.5455 mol Zr
91.22 g Zr
Zr0.5455Si0.5454O2.182; divide each subscript by the smallest, 0.5454.
Zr0.5455 / 0.5454Si0.5454 / 0.5454O2.182 / 0.5454 The empirical formula is ZrSiO4.

3.96 Mass of toluene sample = 45.62 mg = 0.045 62 g; mass of CO2 = 152.5 mg = 0.1525 g;
mass of H2O = 35.67 mg = 0.035 67 g
1 mol CO2 1 mol C
0.1525 g CO2 x x = 0.003 465 mol C
44.01 g CO2 1 mol CO2
12.011 g C
mass C = 0.003 465 mol C x = 0.041 62 g C
1 mol C
1 mol H 2 O 2 mol H
0.035 67 g H 2 O x x = 0.003 959 mol H
18.02 g H2 O 1 mol H 2 O
1.008 g H
mass H = 0.003 959 mol H x = 0.003 991 g H
1 mol H
The (mass C + mass H) = 0.041 62 g + 0.003 991 g = 0.045 61 g. The calculated mass
of (C + H) essentially equals the mass of the toluene sample, this means that toluene
contains only C and H and no other elements.
C0.003 465H0.003 959; divide each subscript by the smaller, 0.003 465.
C0.003 465 / 0.003 465H0.003 959 / 0.003 465
CH1.14; multiply each subscript by 7 to obtain integers.
The empirical formula is C7H8.

3.97 5.024 mg = 0.005 024 g; 13.90 mg = 0.013 90 g; 6.048 mg = 0.006 048 g


1 mol CO2 1 mol C
0.013 90 g CO2 x x = 3.158 x 10-4 mol C
44.01 g CO2 1 mol CO2
1 mol H2 O 2 mol H
0.006 048 g H2O x x = 6.713 x 10-4 mol H
18.02 g H 2 O 1 mol H2 O

50
Chapter 3 - Formulas, Equations, and Moles
______________________________________________________________________________

12.01 g C
3.158 x 10-4 mol C x = 0.003 793 g C
1 mol C
1.008 g H
6.713 x 10-4 mol H x = 0.000 676 7 g H
1 mol H
mass N = 0.005 024 g - (0.003 793 g + 0.000 676 7 g) = 0.000 554 g N
1 mol N
0.000 554 g N x = 3.95 x 10-5 mol N
14.01 g N
Scale each mol quantity to eliminate exponents.
C3.158H6.713N0.395; divide each subscript by the smallest, 0.395.
C3.158 / 0.395H6.713 / 0.395N0.395 / 0.395 The empirical formula is C8H17N.

3.98 Let X equal the molecular mass of cytochrome c.


55.847 amu 55.847 amu
0.0043 = ; X= = 13,000 amu
X 0.0043

3.99 Let X equal the molecular mass of nitrogenase.


2 x 95.94 amu 2 x 95.94 amu
0.000 872 = ; X= = 220,000 amu
X 0.000 872

3.100 Let X equal the molecular mass of disilane.


2 x 28.09 amu 2 x 28.09 amu
0.9028 = ; X= = 62.23 amu
X 0.9028
62.23 amu - 2(Si atomic mass) = 62.23 amu - 2(28.09 amu) = 6.05 amu
6.05 amu is the total mass of H atoms.
1 H atom
6.05 amu x = 6 H atoms ; Disilane is Si2H6.
1.01 amu

3.101 Let X equal the molecular mass of MS2.


2 x 32.07 amu 2 x 32.07 amu
0.4006 = ; X= = 160.1 amu
X 0.4006
Atomic mass of M = 160.1 amu - 2(S atomic mass)
= 160.1 amu - 2(32.07 amu) = 95.96 amu
M is Mo.

General Problems

3.102 (a) C6H12O6, 180.2 amu


6 x 12.01 g C
% C= x 100 % = 39.99%
180.2 g

51
Chapter 3 - Formulas, Equations, and Moles
______________________________________________________________________________

12 x 1.008 g H
% H= x 100 % = 6.713%
180.2 g
6 x 16.00 g O
% O= x 100 % = 53.27 %
180.2 g

(b) H2SO4, 98.08 amu


2 x 1.008 g H
% H= x 100 % = 2.055 %
98.08 g
32.07 g S
% S= x 100 % = 32.70 %
98.08 g
4 x 16.00 g O
% O= x 100 % = 65.25 %
98.08 g

(c) KMnO4, 158.0 amu


39.10 g K
% K= x 100 % = 24.75 %
158.0 g
54.94 g Mn
% Mn = x 100 % = 34.77 %
158.0 g
4 x 16.00 g O
% O= x 100 % = 40.51 %
158.0 g

(d) C7H5NO3S, 183.2 amu


7 x 12.01 g C
% C= x 100 % = 45.89 %
183.2 g
5 x 1.008 g H
% H= x 100 % = 2.751 %
183.2 g
14.01 g N
% N= x 100 % = 7.647 %
183.2 g
3 x 16.00 g O
% O= x 100 % = 26.20 %
183.2 g
32.07 g S
% S= x 100 % = 17.51 %
183.2 g

3.103 (a) Assume a 100.0 g sample of aspirin. From the percent composition data, a 100.0 g
sample contains 60.00 g C, 35.52 g O, and 4.48 g H.
1 mol C
60.00 g C x = 4.996 mol C
12.01 g C
1 mol O
35.52 g O x = 2.220 mol O
16.00 g O

52
Chapter 3 - Formulas, Equations, and Moles
______________________________________________________________________________

1 mol H
4.48 g H x = 4.44 mol H
1.01 g H
C4.996H4.44O2.220; divide each subscript by the smallest, 2.220.
C4.996 / 2.220H4.44 / 2.220O2.220 / 2.220
C2.25H2O1; multiply each subscript by 4 to obtain integers.
The empirical formula is C9H8O4.

(b) Assume a 100.0 g sample of ilmenite. From the percent composition data, a 100.0 g
sample contains 31.63 g O, 31.56 g Ti, and 36.81 g Fe.
1 mol O
31.63 g O x = 1.977 mol O
16.00 g O
1 mol Ti
31.56 g Ti x = 0.6591 mol Ti
47.88 g Ti
1 mol Fe
36.81 g Fe x = 0.6591 mol Fe
55.85 g Fe
Fe0.6591Ti0.6591O1.977; divide each subscript by the smallest, 0.6591.
Fe0.6591 / 0.6591Ti0.6591 / 0.6591O1.977 / 0.6591 The empirical formula is FeTiO3.

(c) Assume a 100.0 g sample of sodium thiosulfate. From the percent composition data,
a 100.0 g sample contains 30.36 g O, 29.08 g Na, and 40.56 g S.
1 mol O
30.36 g O x = 1.897 mol O
16.00 g O
1 mol Na
29.08 g Na x = 1.265 mol Na
22.99 g Na
1 mol S
40.56 g S x = 1.265 mol S
32.07 g S
Na1.265S1.265O1.897; divide each subscript by the smallest, 1.265.
Na1.265 / 1.265S1.265 / 1.265O1.897 / 1.265
NaSO1.5; multiply each subscript by 2 to obtain integers.
The empirical formula is Na2S2O3.

3.104 (a) SiCl4 + 2 H2O → SiO2 + 4 HCl


(b) P4O10 + 6 H2O → 4 H3PO4
(c) CaCN2 + 3 H2O → CaCO3 + 2 NH3
(d) 3 NO2 + H2O → 2 HNO3 + NO

3.105 NaH, 24.00 amu; B2H6, 27.67 amu; NaBH4, 37.83 amu
2 NaH + B2H6 → 2 NaBH4
1 mol NaH
8.55 g NaH x = 0.356 mol NaH
24.00 g NaH

53
Chapter 3 - Formulas, Equations, and Moles
______________________________________________________________________________

1 mol B2 H 6
6.75 g B2H6 x = 0.244 mol B2H6
27.67 g B2 H 6
For 0.244 mol B2H6, 2 x (0.244) = 0.488 mol NaH are needed. Because only 0.356 mol
of NaH is available, NaH is the limiting reactant.
2 mol NaBH4 37.83 g NaBH4
0.356 mol NaH x x = 13.5 g NaBH4 produced
2 mol NaH 1 mol NaBH4
1 mol B2 H6 27.67 g B2 H6
0.356 mol NaH x x = 4.93 g B2H6 reacted
2 mol NaH 1 mol B2 H6
B2H6 left over = 6.75 g - 4.93 g = 1.82 g B2H6

3.106 Assume a 100.0 g sample of ferrocene. From the percent composition data, a 100.0 g
sample contains 5.42 g H, 64.56 g C, and 30.02 g Fe.
1 mol H
5.42 g H x = 5.37 mol H
1.01 g H
1 mol C
64.56 g C x = 5.376 mol C
12.01 g C
1 mol Fe
30.02 g Fe x = 0.5375 mol Fe
55.85 g Fe
C5.376H5.37Fe0.5375; divide each subscript by the smallest, 0.5375.
C5.376 / 0.5375H5.37 / 0.5375Fe0.5375 / 0.5375 The empirical formula is C10H10Fe.

amu g
3.107 Mass of 1 HCl molecule = (36.5 )(1.6605 x 10-24 ) = 6.06 x 10-23 g/molecule
molecule amu
 36.5 g/mol 
Avogadro's number =  _ 23
 = 6.02 x 1023 molecules/mol
 6.06 x 10 g/molecule 

3.108 Na2SO4, 142.04 amu; Na3PO4, 163.94 amu; Li2SO4, 109.95 amu; 100.00 mL = 0.10000 L
1 mol Na 2 SO4
0.550 g Na2SO4 x = 0.003 872 mol Na2SO4
142.04 g Na 2 SO4
1 mol Na 3 PO4
1.188 g Na3PO4 x = 0.007 247 mol Na3PO4
163.94 g Na 3 PO4
1 mol Li2 SO4
0.223 g Li2SO4 x = 0.002 028 mol Li2SO4
109.95 g Li2 SO4
(2 x 0.003 872 mol) + (3 x 0.007 247 mol)
Na+ molarity = = 0.295 M
0.100 00 L
2 x 0.002 028 mol
Li+ molarity = = 0.0406 M
0.100 00 L
(1 x 0.003 872 mol) + (1 x 0.002 028 mol)
SO42- molarity = = 0.0590 M
0.100 00 L

54
Chapter 3 - Formulas, Equations, and Moles
______________________________________________________________________________

1 x 0.007 247 mol


PO43- molarity = = 0.0725 M
0.100 00 L

3.109 23.46 mg = 0.023 46 g; 20.42 mg = 0.02042 g; 33.27 mg = 0.033 27 g


1 mol CO2 1 mol C
0.033 27 g CO2 x x = 7.560 x 10_ 4 mol C
44.01 g CO2 1 mol CO2
1 mol H 2 O 2 mol H
0.020 42 g H 2 O x x = 2.266 x 10_ 3 mol H
18.02 g H 2 O 1 mol H2 O
12.01 g C
7.560 x 10_ 4 mol C x = 0.009 080 g C
1 mol C
1.008 g H
2.266 x 10_ 3 mol H x = 0.002 284 g H
1 mol H
mass O = 0.023 46 g - (0.009 080 g + 0.002 284 g) = 0.012 10 g O
1 mol O
0.012 10 g O x = 7.563 x 10_ 4 mol O
16.00 g O
Scale each mol quantity to eliminate exponents.
C0.7560H2.266O0.7563; divide each subscript by the smallest, 0.7560.
C0.7560 / 0.7560H2.266 / 0.7560O0.7563 / 0.7560 The empirical formula is CH3O, 31.0 amu.
62.0 amu / 31.0 amu = 2; molecular formula = C(2 x 1)H(2 x 3)O(2 x 1) = C2H6O2

3.110 High resolution mass spectrometry is capable of measuring the mass of molecules with a
particular isotopic composition.

3.111 (a) CO(NH2)2(aq) + 6 HOCl(aq) → 2 NCl3(aq) + CO2(aq) + 5 H2O(l)


(b) 2 Ca3(PO4)2(s) + 6 SiO2(s) + 10 C(s) → P4(g) + 6 CaSiO3(l) + 10 CO(g)

3.112 The combustion reaction is: 2 C8H18 + 25 O2 → 16 CO2 + 18 H2O


C8H18, 114.23 amu; CO2, 44.01 amu
3.7854 L 1000 mL 0.703 g C8 H18 1 mol C8 H18
pounds CO2 = 1.00 gal x x x x x
1 gal 1L 1 mL 114.23 g C8 H18
16 mol CO2 44.01 g CO2 1 lb
x x = 18.1 pounds CO2
2 mol C8 H18 1 mol CO2 453.59 g

3.113 The reaction is: CaCO3 + 2 HCl → CaCl2 + CO2 + H2O


CaCO3, 100.09 amu; CO2, 44.01 amu
1 mol CaCO3
mol CaCO3 = 6.35 g CaCO3 x = 0.0634 mol CaCO3
100.09 g CaCO3
1L 0.31 mol HCl
mol HCl = 500.0 mL HCl x x = 0.155 mol HCl
1000 mL 1L
Determine the limiting reactant.
2 mol HCl
mol HCl needed = 0.0634 mol CaCO3 x = 0.127 mol HCl needed
1 mol CaCO3

55
Chapter 3 - Formulas, Equations, and Moles
______________________________________________________________________________

Because we have excess HCl, CaCO3 is the limiting reactant.


1 mol CO2 44.01 g CO2
mass CO2 = 0.0634 mol CaCO3 x x = 2.79 g CO2
1 mol CaCO3 1 mol CO2

3.114 AgCl, 143.32 amu; CO2, 44.01 amu; H2O, 18.02 amu
1 mol AgCl 1 mol Cl
mol Cl in 1.00 g of X = 1.95 g AgCl x x = 0.0136 mol Cl
143.32 g AgCl 1 mol AgCl
35.453 g Cl
mass Cl = 0.0136 mol Cl x = 0.482 g Cl
1 mol Cl
1 mol CO2 1 mol C
mol C in 1.00 g of X = 0.900 g CO 2 x x = 0.0204 mol C
44.01 g CO 2 1 mol CO 2
12.011 g C
mass C = 0.0204 mol C x = 0.245 g C
1 mol C
1 mol H 2 O 2 mol H
mol H in 1.00 g of X = 0.735 g H 2 O x x = 0.0816 mol H
18.02 g H 2 O 1 mol H 2 O
1.008 g H
mass H = 0.0816 mol H x = 0.0823 g H
1 mol H
mass N = 1.00 g - mass Cl - mass C - mass H = 1.00 - 0.482 g - 0.245 g - 0.0823 g = 0.19 g N

1 mol N
mol N in 1.00 g of X = 0.19 g N x = 0.014 mol N
14.01 g N
Determine empirical formula.
C0.0204H0.0816N0.014Cl0.0136, divide each subscript by the smallest, 0.0136.
C0.0204 / 0.0136H0.0816 / 0.0136N0.014 / 0.0136Cl0.0136 / 0.0136
C1.5H6NCl, multiply each subscript by 2 to get integers.
The empirical formula is C3H12N2Cl2.

3.115 CaCO3, 100.09 amu


40.08 g Ca
% Ca = x 100% = 40.04%
100.09 g
12.01 g C
%C= x 100% = 12.00%
100.09 g
3 x 16.00 g O
%O= x 100% = 47.96%
100.09 g
Because the mass %’s for the pulverized rock are different from the mass %’s for pure
CaCO3 calculated here, the pulverized rock cannot be pure CaCO3.

3.116 Let SA stand for salicylic acid.


1 mol CO 2 1 mol C
mol C in 1.00 g of SA = 2.23 g CO 2 x x = 0.0507 mol C
44.01 g CO 2 1 mol CO 2

56
Chapter 3 - Formulas, Equations, and Moles
______________________________________________________________________________

12.011 g C
mass C = 0.0507 mol C x = 0.609 g C
1 mol C
1 mol H 2 O 2 mol H
mol H in 1.00 g of SA = 0.39 g H 2 O x x = 0.043 mol H
18.02 g H 2 O 1 mol H 2 O
1.008 g H
mass H = 0.043 mol H x = 0.043 g H
1 mol H
mass O = 1.00 g - mass C - mass H = 1.00 - 0.609 g - 0.043 g = 0.35 g O
1 mol O
mol O in 1.00 g of = 0.35 g N x = 0.022 mol O
16.00 g O
Determine empirical formula.
C0.0507H0.043O0.022, divide each subscript by the smallest, 0.022.
C0.0507 / 0.022H0.043 / 0.022O0.022 / 0.022
C2.3H2O, multiply each subscript by 3 to get integers.
The empirical formula is C7H6O3. The empirical formula mass = 138.12 g/mol

Because salicylic acid has only one acidic hydrogen, there is a 1 to 1 mol ratio between
salicylic acid and NaOH in the acid-base titration.
1L 0.100 mol NaOH 1 mol SA
mol SA in 1.00 g SA = 72.4 mL x x x =
1000 mL 1L 1 mol NaOH
0.00724 mol SA
1.00 g
SA molar mass = = 138 g/mol
0.00724 mol
Because the empirical formula mass and the molar mass are the same, the empirical
formula is the molecular formula for salicylic acid.

1 mol CO2 1 mol C


3.117 (a) mol C = 4.83 g CO2 x x = 0.110 mol C
44.01 g CO2 1 mol CO2
12.011 g C
mass C = 0.110 mol C x = 1.32 g C
1 mol C
1 mol H 2 O 2 mol H
mol H = 1.48 g H2O x x = 0.164 mol H
18.02 g H2 O 1 mol H 2 O
1.008 g H
mass H = 0.164 mol H x = 0.165 g H
1 mol H
109.8 mL = 0.1098 L
mol NaOH = (0.1098 L)(1.00 mol/L) = 0.110 mol NaOH
H2SO4(aq) + 2 NaOH(aq) → Na2SO4(aq) + 2 H2O(l)
1 mol H 2 SO4
mol H2SO4 = 0.110 mol NaOH x = 0.0550 mol H2SO4
2 mol NaOH
1 mol S
mol S = 0.0550 mol H2SO4 x = 0.0550 mol S
1 mol H 2 SO4

57
Chapter 3 - Formulas, Equations, and Moles
______________________________________________________________________________

32.06 g S
mass S = 0.0550 mol S x = 1.76 g S
1 mol S
mass O = 5.00 g - mass C - mass H - mass S = 5.00 g -1.32 g - 0.165 g - 1.76 g = 1.75 g O
1 mol O
mol O = 1.75 g O x = 0.109 mol O
16.00 g O
C0.110H0.164O0.109S0.0550 Divide all subscripts by the smallest.
C0.110 / 0.0550H0.164 / 0.0550O0.109 / 0.0550S0.0550 / 0.0550
The empirical formula is C2H3O2S. The empirical formula mass = 91.1 g/mol
(b) 54.9 mL = 0.0549 L
mol NaOH = (0.0549 L)(1.00 mol/L) = 0.0549 mol NaOH
Because X has two acidic hydrogens, two mol of NaOH are required to titrate 1 mol of X.
1 mol X
mol X = 0.0549 mol NaOH x = 0.0274 mol X
2 mol NaOH
5.00 g
X molar mass = = 182 g/mol
0.0274 mol
Because the molar mass is twice the empirical formula mass, the molecular formula is
twice the empirical formula.
The molecular formula is C(2 x 2)H(2 x 3)O(2 x 2)S(2 x 1) = C4H6O4S2

3.118 Let X equal the mass of benzoic acid and Y the mass of gallic acid in the 1.00 g mixture.
Therefore, X + Y = 1.00 g.
Because both acids contain only one acidic hydrogen, there is a 1 to 1 mol ratio between
each acid and NaOH in the acid-base titration.
In the titration, mol benzoic acid + mol gallic acid = mol NaOH
1 mol BA 1 mol GA
Therefore, X x +Yx = mol NaOH
122 g BA 170 g GA
1L 0.500 mol NaOH
mol NaOH = 14.7 mL x x = 0.00735 mol NaOH
1000 mL 1L
We have two unknowns, X and Y, and two equations.
X + Y = 1.00 g
1 mol BA 1 mol GA
Xx +Yx = 0.00735 mol NaOH
122 g BA 170 g GA
Rearrange to get X = 1.00 g - Y and then substitute it into the equation above to solve for Y.
1 mol BA 1 mol GA
(1.00 g _ Y) x +Yx = 0.00735 mol NaOH
122 g BA 170 g GA
1 mol Y mol Y mol
_ + = 0.00735 mol
122 122 g 170 g
Y mol Y mol 1 mol
_ + = 0.00735 mol _ = _ 8.47 x 10 _ 4 mol
122 g 170 g 122

(_ Y mol)(170 g) + (Y mol)(122 g)
= _ 8.47 x 10 _ 4 mol
(170 g)(122 g)

58
Chapter 3 - Formulas, Equations, and Moles
______________________________________________________________________________

_ 48 Y mol 48 Y
= _ 8.47 x 10 _ 4 mol ; = 8.47 x 10 _ 4
20740 g 20740 g
_4
(20740 g)(8.47 x 10 )
Y= = 0.366 g
48
X = 1.00 g - 0.366 g = 0.634 g
In the 1.00 g mixture there is 0.63 g of benzoic acid and 0.37 g of gallic acid.

3.119 C2H6O, 46.07 amu; H2O, 18.02 amu


Let X = mass of H2O in the 10.00 g sample.
Let Y = mass of ethanol (C2H6O) in the 10.00 g sample.
X + Y = 10.00 g and Y = 10.00 g - X
mass of collected H2O = 11.27 g
 1 mol C2 H6 O 3 mol H 2 O 18.02 g H 2 O 
mass of collected H2O = X +  Y x x x 
 46.07 g C2 H6 O 1 mol C2 H 6 O 1 mol H 2 O 
Substitute for Y.
 1 mol C2 H 6 O 3 mol H 2 O 18.02 g H 2 O 
11.27 g = X +  (10.00 g _ X) x x x 
 46.07 g C2 H 6 O 1 mol C2 H6 O 1 mol H 2 O 
11.27 g = X + (10.00 g - X)(1.173)
11.27 g = X + 11.73 g - 1.173 X
0.173 X = 11.73 g - 11.27 g = 0.46 g
0.46 g
X= = 2.7 g H2O
0.173
Y = 10.00 g - X = 10.00 g - 2.7 g = 7.3 g C2H6O

3.120 FeO, 71.85 amu; Fe2O3, 159.7 amu


Let X equal the mass of FeO and Y the mass of Fe2O3 in the 10.0 g mixture. Therefore,
X + Y = 10.0 g.
1 mol Fe
mol Fe = 7.43 g x = 0.133 mol Fe
55.85 g Fe
mol FeO + 2 x mol Fe2O3 = 0.133 mol Fe
1 mol FeO  1 mol Fe2 O3 
Xx + 2 x  Y x  = 0.133 mol Fe
71.85 g FeO  159.7 g Fe2 O3 
Rearrange to get X = 10.0 g - Y and then substitute it into the equation above to solve for Y.
1 mol FeO  1 mol Fe2 O3 
(10.0 g _ Y) x + 2 x  Y x  = 0.133 mol Fe
71.85 g FeO  159.7 g Fe2 O3 
10.0 mol Y mol 2 Y mol
_ + = 0.133 mol
71.85 71.85 g 159.7 g
Y mol 2 Y mol 10.0 mol
_ + = 0.133 mol _ = - 0.0062 mol
71.85 g 159.7 g 71.85
(_ Y mol)(159.7 g) + (2 Y mol)(71.85 g)
= - 0.0062 mol
(71.85 g)(159.7 g)

59
Chapter 3 - Formulas, Equations, and Moles
______________________________________________________________________________

_ 16.0 Y mol 16.0 Y


= _ 0.0062 mol ; = 0.0062
11474 g 11474 g
Y = (0.0062)(11474 g)/16.0 = 4.44 g = 4.4 g Fe2O3

X = 10.0 g - Y = 10.0 g - 4.4 g = 5.6 g FeO

3.121 AgCl, 143.32 amu;


Find the mass of Cl in 1.68 g of AgCl.
1 mol AgCl 1 mol Cl
mol Cl in 1.68 g of AgCl = 1.68 g AgCl x x = 0.0117 mol Cl
143.32 g AgCl 1 mol AgCl
35.453 g Cl
mass Cl = 0.0117 mol Cl x = 0.415 g Cl
1 mol Cl
All of the Cl in AgCl came from XCl3.
Find the mass of X in 0.634 g of XCl3. Mass of X = 0.634 g - 0.415 g = 0.219 g X
1 mol X
0.0117 mol Cl x = 0.00390 mol X
3 mol Cl
0.219 g
molar mass of X = = 56.2 g/mol; X = Fe
0.00390 mol

3.122 C6H12O6 + 6 O2 → 6 CO2 + 6 H2O; C6H12O6, 180.16 amu; CO2, 44.01 amu
1 mol C6 H12 O6 6 mol CO2 44.01 g CO2
66.3 g C6H12O6 x x x = 97.2 g CO2
180.16 g C6 H12 O6 1 mol C6 H12 O6 1 mol CO2
1 mol C6 H12 O6 6 mol CO2 25.4 L CO2
66.3 g C6H12O6 x x x = 56.1 L CO2
180.16 g C6 H12 O6 1 mol C6 H12 O6 1 mol CO2

3.123 H2C2O4, 90.04 amu; 22.35 mL = 0.02235 L


1 mol H 2 C2 O4 2 mol KMnO 4
0.5170 g H2C2O2 x x = 0.002 297 mol KMnO4
90.04 g H 2 C2 O4 5 mol H 2 C2 O4
0.002 297 mol KMnO 4
KMnO4 molarity = = 0.1028 M
0.022 35 L

3.124 Mass of Cu = 2.196 g; mass of S = 2.748 g - 2.196 g = 0.552 g S


2.196 g
(a) %Cu = x 100% = 79.91%
2.748 g
0.552 g
%S = x 100% = 20.1%
2.748 g
1 mol Cu
(b) 2.196 g Cu x = 0.034 55 mol Cu
63.55 g Cu
1 mol S
0.552 g S x = 0.0172 mol S
32.07 g S

60
Chapter 3 - Formulas, Equations, and Moles
______________________________________________________________________________

Cu0.03455S0.0172; divide each subscript by the smaller, 0.0172.


Cu0.03455 / 0.0172S0.0172 / 0.0172 The empirical formula is Cu2S.

(c) Cu2S, 159.16 amu


5.6 g Cu 2 S 1 mol Cu 2 S 2 mol Cu + ions 6.022 x 1023 Cu + ions
x x x
1 cm3 159.16 g Cu 2 S 1 mol Cu 2 S 1 mol Cu + ions
22 + 3
= 4.2 x 10 Cu ions/cm

3.125 Mass of added Cl = mass of XCl5 - mass of XCl3 = 13.233 g - 8.729 g = 4.504 g
4.504 g
mass of Cl in XCl5 = 5 Cl’s x = 11.26 g Cl
2 C′ls
mass of X in XCl5 = 13.233 g - 11.26 g = 1.973 g X
1 mol Cl
11.26 g Cl x = 0.3176 mol Cl
35.45 g Cl
1 mol X
0.3176 mol Cl x = 0.063 52 mol X
5 mol Cl
1.973 g X
molar mass of X = = 31.1 g/mol; atomic mass =31.1 amu, X = P
0.063 52 mol X

3.126 PCl3, 137.33 amu; PCl5, 208.24 amu


Let Y = mass of PCl3 in the mixture, and (10.00 - Y) = mass of PCl5 in the mixture.
(3)(35.453 g/mol)
fraction Cl in PCl3 = = 0.774 48
137.33 g/mol
(5)(35.453 g/mol)
fraction Cl in PCl5 = = 0.851 25
208.24 g/mol
(mass of Cl in PCl3) + (mass of Cl in PCl5) = mass of Cl in the mixture
0.774 48Y + 0.851 25(10.00 g - Y) = (0.8104)(10.00 g)
Y = 5.32 g PCl3 and 10.00 - Y = 4.68 g PCl5

3.127 100.00 mL = 0.100 00 L; 71.02 mL = 0.071 02 L


0.1083 mol H 2 SO 4
mol H2SO4 = x 0.100 00 L = 0.010 83 mol H2SO4
L
0.1241 mol NaOH
mol NaOH = x 0.071 02 L = 0.008 814 mol NaOH
L
H2SO4 + 2 NaOH → Na2SO4 + 2 H2O
1 mol H 2 SO4
mol H2SO4 reacted with NaOH = 0.008 814 mol NaOH x = 0.004 407 mol H2SO4
2 mol NaOH
mol H2SO4 reacted with MCO3 = 0.010 83 mol - 0.004 407 mol = 0.006 423 mol H2SO4
mol H2SO4 reacted with MCO3 = mol CO32- in MCO3 = mol CO2 produced = 0.006 423 mol CO2
60.01 g CO32 _
(a) CO32-, 60.01 amu; 0.006 423 mol CO32- x = 0.3854 g CO32-
1 mol CO32 _
mass of M = 1.268 g - 0.3854 g = 0.8826 g M

61
Chapter 3 - Formulas, Equations, and Moles
______________________________________________________________________________

0.8826 g
molar mass of M = = 137.4 g/mol; M is Ba
0.006 423 mol
44.01 g CO2 1L
(b) 0.006 423 mol CO2 x x = 0.1571 L CO2
1 mol CO 2 1.799 g

3.128 NH4NO3, 80.04 amu; (NH4)2HPO4, 132.06 amu


Assume you have a 100.0 g sample of the mixture.
Let X = grams of NH4NO3 and (100.0 - X) = grams of (NH4)2HPO4.
Both compounds contain 2 nitrogen atoms per formula unit.
Because the mass % N in the sample is 30.43%, the 100.0 g sample contains 30.43 g N.
1 mol NH 4 NO3
mol NH4NO3 = (X) x
80.04 g
1 mol ( NH4 )2 HPO4
mol (NH4)2HPO4 = (100.0 _ X) x
132.06 g
 1 mol NH4 NO3   1 mol ( NH4 )2 HPO4  
mass N =   (X) x  +  (100.0 _ X) x   x
 80.04 g   132.06 g 
 2 mol N   14.0067 g N 
  x   = 30.43 g
 1 mol ammonium cmpds   1 mol N 
Solve for X.
 X 100.0 _ X 
 + (2)(14.0067) = 30.43
 80.04 132.06 
 X 100.0 _ X 
 +  = 1.08627
 80.04 132.06 
(132.06)(X) + (100.0 _ X)(80.04)
= 1.08627
(80.04)(132.06)
(132.06)(X) + (100.0 - X)(80.04) = (1.08627)(80.04)(132.06)
132.06X + 8004 - 80.04X = 11481.96
132.06X - 80.04X = 11481.96 - 8004
52.02X = 3477.96
3477.96
X= = 66.86 g NH4NO3
52.02
(100.0 - X) = (100.0 - 66.86) = 33.14 g (NH4)2HPO4
mass NH4 NO3 66.86 g
= = 2.018
mass( NH4)2 HPO4 33.14 g
The mass ratio of NH4NO3 to (NH4)2HPO4 in the mixture is 2 to 1.

3.129 Na2CO3 → Na2O + CO2; Na2CO3, 106 amu; Na2O, 62 amu


CaCO3 → CaO + CO2; CaCO3, 100 amu; CaO, 56 amu
In a 0.35 kg sample of glass there would be:
0.12 x 0.35 kg = 0.042 kg = 42 g of Na2O
0.13 x 0.35 kg = 0.045 kg = 45 g of CaO

62
Chapter 3 - Formulas, Equations, and Moles
______________________________________________________________________________

350 g - 42 g - 45 g = 263 g of SiO2

1 mol Na 2 O 1 mol Na 2 CO3 106 g Na 2 CO3


mass Na2CO3 = 42 g Na2O x x x = 72 g
62 g Na 2 O 1 mol Na 2 O 1 mol Na 2 CO3
Na2CO3
1 mol CaO 1 mol CaCO3 100 g CaCO3
mass CaCO3 = 45 g CaO x x x = 80 g CaCO3
56 g CaO 1 mol CaO 1 mol CaCO3
To make 0.35 kg of glass, start with 72 g Na2CO3, 80 g CaCO3, and 263 g SiO2.

3.130 (a) 56.0 mL = 0.0560 L


 1 mol 
mol X2 = (0.0560 L X2)   = 0.00250 mol X2
 22.41 L 
mass X2 = 1.12 g MX2 - 0.720 g MX = 0.40 g X2
0.40 g
molar mass X2 = = 160 g/mol
0.00250 mol
atomic mass of X = 160/2 = 80 amu; X is Br.

2 mol MX
(b) mol MX = 0.00250 mol X2 x = 0.00500 mol MX
1 mol X2
1 mol X 80 g X
mass of X in MX = 0.00500 mol MX x x = 0.40 g X
1 mol MX 1 mol X
mass of M in MX = 0.720 g MX - 0.40 g X = 0.32 g M
0.32 g
molar mass M = = 64 g/mol
0.00500 mol
atomic mass of X = 64 amu; M is Cu.

63
4 Reactions in Aqueous Solution

4.1 (a) precipitation (b) redox (c) acid-base neutralization

4.2 FeBr3 contains 3 Br- ions. The molar concentration of Br- ions = 3 x 0.225 M = 0.675 M

4.3 A2Y is the strongest electrolyte because it is completely dissociated into ions.
A2X is the weakest electrolyte because it is the least dissociated of the three substances.

4.4 (a) Ionic equation:


2 Ag+(aq) + 2 NO3-(aq) + 2 Na+(aq) + CrO42-(aq) → Ag2CrO4(s) + 2 Na+(aq) + 2 NO3-(aq)
Delete spectator ions from the ionic equation to get the net ionic equation.
Net ionic equation: 2 Ag+(aq) + CrO42-(aq) → Ag2CrO4(s)
(b) Ionic equation:
2 H+(aq) + SO42-(aq) + MgCO3(s) → H2O(l) + CO2(g) + Mg2+(aq) + SO42-(aq)
Delete spectator ions from the ionic equation to get the net ionic equation.
Net ionic equation: 2 H+(aq) + MgCO3(s) → H2O(l) + CO2(g) + Mg2+(aq)
(c) Ionic equation:
Hg22+(aq) + 2 NO3-(aq) + 2 NH4+(aq) + 2 Cl-(aq) → Hg2Cl2(s) + 2 NH4+(aq) + 2 NO3-(aq)
Delete spectator ions from the ionic equation to get the net ionic equation.
Net ionic equation: Hg22+(aq) + 2 Cl-(aq) → Hg2Cl2(s)

4.5 (a) CdCO3, insoluble (b) MgO, insoluble (c) Na2S, soluble
(d) PbSO4, insoluble (e) (NH4)3PO4, soluble (f) HgCl2, soluble

4.6 (a) Ionic equation:


Ni2+(aq) + 2 Cl-(aq) + 2 NH4+(aq) + S2-(aq) → NiS(s) + 2 NH4+(aq) + 2 Cl-(aq)
Delete spectator ions from the ionic equation to get the net ionic equation.
Net ionic equation: Ni2+(aq) + S2-(aq) → NiS(s)
(b) Ionic equation:
2 Na+(aq) + CrO42-(aq) + Pb2+(aq) + 2 NO3-(aq) → PbCrO4(s) + 2 Na+(aq) + 2 NO3-(aq)
Delete spectator ions from the ionic equation to get the net ionic equation.
Net ionic equation: Pb2+(aq) + CrO42-(aq) → PbCrO4(s)
(c) Ionic equation:
2 Ag+(aq) + 2 ClO4-(aq) + Ca2+(aq) + 2 Br-(aq) → 2 AgBr(s) + Ca2+(aq) + 2 ClO4-(aq)
Delete spectator ions from the ionic equation, and reduce coefficients to get the net ionic
equation.
Net ionic equation: Ag+(aq) + Br-(aq) → AgBr(s)
(d) Ionic equation:
Zn2+(aq) + 2 Cl-(aq) + 2 K+(aq) + CO32-(aq) → ZnCO3(s) + 2 K+(aq) + 2 Cl-(aq)
Delete spectator ions from the ionic equation to get the net ionic equation.

65
Net ionic equation: Zn2+(aq) + CO32-(aq) → ZnCO3(s)
4.7 3 CaCl2(aq) + 2 Na3PO4(aq) → Ca3(PO4)2(s) + 6 NaCl(aq)
Ionic equation:
3 Ca2+(aq) + 6 Cl-(aq) + 6 Na+(aq) + 2 PO43-(aq) → Ca3(PO4)2(s) + 6 Na+(aq) + 6 Cl-(aq)
Delete spectator ions from the ionic equation to get the net ionic equation.
Net ionic equation: 3 Ca2+(aq) + 2 PO43-(aq) → Ca3(PO4)2(s)

4.8 A precipitate results from the reaction. The precipitate contains cations and anions in a
3:2 ratio. The precipitate is either Mg3(PO4)2 or Zn3(PO4)2.

4.9 (a) Ionic equation:


2 Cs+(aq) + 2 OH-(aq) + 2 H+(aq) + SO42-(aq) → 2 Cs+(aq) + SO42-(aq) + 2 H2O(l)
Delete spectator ions from the ionic equation, and reduce coefficients to get the net ionic
equation.
Net ionic equation: H+(aq) + OH-(aq) → H2O(l)

(b) Ionic equation:


Ca2+(aq) + 2 OH-(aq) + 2 CH3CO2H(aq) → Ca2+(aq) + 2 CH3CO2-(aq) + 2 H2O(l)
Delete spectator ions from the ionic equation, and reduce coefficients to get the net ionic
equation.
Net ionic equation: CH3CO2H(aq) + OH-(aq) → CH3CO2-(aq) + H2O(l)

4.10 HY is the strongest acid because it is completely dissociated.


HX is the weakest acid because it is the least dissociated.

4.11 (a) SnCl4: Cl -1, Sn +4 (b) CrO3: O -2, Cr +6


(c) VOCl3: O -2, Cl -1, V +5 (d) V2O3: O -2, V +3
(e) HNO3: O -2, H +1, N +5 (f) FeSO4: O -2, S +6, Fe +2

4.12 2 Cu2+(aq) + 4 I-(aq) → 2 CuI(s) + I2(aq)


oxidation numbers: Cu2+ +2; I- -1; CuI: Cu +1, I -1; I2: 0
oxidizing agent (oxidation number decreases), Cu2+
reducing agent (oxidation number increases) , I-

4.13 (a) SnO2(s) + 2 C(s) → Sn(s) + 2 CO(g)


C is oxidized (its oxidation number increases from 0 to +2). C is the reducing agent.
The Sn in SnO2 is reduced (its oxidation number decreases from +4 to 0). SnO2 is the
oxidizing agent.
(b) Sn2+(aq) + 2 Fe3+(aq) → Sn4+(aq) + 2 Fe2+(aq)
Sn2+ is oxidized (its oxidation number increases from +2 to +4). Sn2+ is the reducing
agent.
Fe3+ is reduced (its oxidation number decreases from +3 to +2). Fe3+ is the oxidizing
agent.
(c) 4 NH3(g) + 5 O2(g) → 4 NO(g) + 6 H2O(l)
The N in NH3 is oxidized (its oxidation number increases from -3 to +2). NH3 is the
reducing agent.
Each O in O2 is reduced (its oxidation number decreases from 0 to -2). O2 is the
oxidizing agent.

66
Chapter 4 - Reactions in Aqueous Solutions
______________________________________________________________________________

4.14 (a) Pt is below H in the activity series; therefore NO REACTION.


(b) Mg is below Ca in the activity series; therefore NO REACTION.

4.15 Because B will reduce A+, B is above A in the activity series. Because B will not reduce
C+, C is above B in the activity series. Therefore C must be above A in the activity series
and C will reduce A+.

4.16

8 H+(aq) + Cr2O72-(aq) + I-(aq) → 2 Cr3+(aq) + IO3-(aq) + 4 H2O(l)

4.17

2 MnO4-(aq) + Br-(aq) → 2 MnO2(s) + BrO3-(aq)


2 H+(aq) + 2 MnO4-(aq) + Br-(aq) → 2 MnO2(s) + BrO3-(aq) + H2O(l)
2 H+(aq) + 2 OH-(aq) + 2 MnO4-(aq) + Br-(aq) →
2 MnO2(s) + BrO3-(aq) + H2O(l) + 2 OH-(aq)
2 H2O(l) + 2 MnO4-(aq) + Br-(aq) → 2 MnO2(s) + BrO3-(aq) + H2O(l) + 2 OH-(aq)
H2O(l) + 2 MnO4-(aq) + Br-(aq) → 2 MnO2(s) + BrO3-(aq) + 2 OH-(aq)

4.18 (a) MnO4-(aq) → MnO2(s) (reduction)


IO3-(aq) → IO4-(aq) (oxidation)

(b) NO3-(aq) → NO2(g) (reduction)


SO2(aq) → SO42-(aq) (oxidation)

4.19 NO3-(aq) + Cu(s) → NO(g) + Cu2+(aq)


[Cu(s) → Cu2+(aq) + 2 e-] x 3 (oxidation half reaction)

NO3-(aq) → NO(g)
NO3-(aq) → NO(g) + 2 H2O(l)
4 H+(aq) + NO3-(aq) → NO(g) + 2 H2O(l)
[3 e- + 4 H+(aq) + NO3-(aq) → NO(g) + 2 H2O(l)] x 2 (reduction half reaction)

67
Chapter 4 - Reactions in Aqueous Solutions
______________________________________________________________________________

Combine the two half reactions.


2 NO3-(aq) + 8 H+(aq) + 3 Cu(s) → 3 Cu2+(aq) + 2 NO(g) + 4 H2O(l)
4.20 Fe(OH)2(s) + O2(g) → Fe(OH)3(s)
[Fe(OH)2(s) + OH-(aq) → Fe(OH)3(s) + e-] x 4 (oxidation half reaction)

O2(g) → 2 H2O(l)
4 H+(aq) + O2(g) → 2 H2O(l)
4 e- + 4 H+(aq) + O2(g) → 2 H2O(l)
4 e- + 4 H+(aq) + 4 OH-(aq) + O2(g) → 2 H2O(l) + 4 OH-(aq)
4 e- + 4 H2O(l) + O2(g) → 2 H2O(l) + 4 OH-(aq)
4 e- + 2 H2O(l) + O2(g) → 4 OH-(aq) (reduction half reaction)

Combine the two half reactions.


4 Fe(OH)2(s) + 4 OH-(aq) + 2 H2O(l) + O2(g) → 4 Fe(OH)3(s) + 4 OH-(aq)
4 Fe(OH)2(s) + 2 H2O(l) + O2(g) → 4 Fe(OH)3(s)

4.21 31.50 mL = 0.031 50 L; 10.00 mL = 0.010 00 L


0.105 mol BrO3_ 6 mol Fe2+
0.031 50 L x x _
= 1.98 x 10-2 mol Fe2+
1L 1 mol BrO3
_2 2+
1.98 x 10 mol Fe
molarity = = 1.98 M Fe2+ solution
0.010 00 L

4.22 The Na2S2O3, or hypo, is used to solubilize the remaining unreduced AgBr on the film so
that it is no longer sensitive to light. The reaction is
AgBr(s) + 2 S2O32-(aq) → Ag(S2O3)23-(aq) + Br-(aq)

4.23 To convert this negative image into the final printed photograph, the entire photographic
procedure is repeated a second time. Light is passed through the negative image onto
special photographic paper that is coated with the same kind of gelatin–AgBr emulsion
used on the original film. Developing the photographic paper with hydroquinone and
fixing the image with sodium thiosulfate reverses the negative image, and a final, positive
image is produced.

Understanding Key Concepts

4.24 (a) 2 Na+(aq) + CO32-(aq) does not form a precipitate. This is represented by box (1).
(b) Ba2+(aq) + CrO42-(aq) → BaCrO4(s). This is represented by box (2).
(c) 2 Ag+(aq) + SO42-(aq) → Ag2SO4(s). This is represented by box (3).

4.25 In the precipitate there are two cations (blue) for each anion (green). Looking at the ions
in the list, the anion must have a -2 charge and the cation a +1 charge for charge neutrality
of the precipitate. The cation must be Ag+ because all Na+ salts are soluble. Ag2CrO4
and Ag2CO3 are insoluble and consistent with the observed result.

68
Chapter 4 - Reactions in Aqueous Solutions
______________________________________________________________________________

4.26 One OH- will react with each available H+ on the acid forming H2O. The acid is
identified by how many of the 12 OH- react with three molecules of each acid.
(a) Three HF's react with three OH-, leaving nine OH- unreacted (box 2).
(b) Three H2SO3's react with six OH-, leaving six OH- unreacted (box 3).
(c) Three H3PO4's react with nine OH-, leaving three OH- unreacted (box 1).

4.27 The concentration in the buret is three times that in the flask. The NaOCl concentration
is 0.040 M. Because the I- concentration in the buret is three times the OCl- concentration
in the flask and the reaction requires 2 I- ions per OCl- ion, 2/3 or 67% of the I- solution
from the buret must be added to the flask to react with all of the OCl-.

4.28 (a) Ionic equation:


K+(aq) + Cl-(aq) + Ag+(aq) + NO3-(aq) → AgCl(s) + K+(aq) + NO3-(aq)
(b) Ionic equation:
HF(aq) + K+(aq) + OH-(aq) → K+(aq) + F-(aq) + H2O(l)
(c) Ionic equation:
Ba2+(aq) + 2 Cl-(aq) + 2 Na+(aq) + SO42-(aq) → BaSO4(s) + 2 Na+(aq) + 2 Cl-(aq)

Reaction (c) would have the highest initial conductivity because of the 3 net ions for each
BaCl2 (a strong electrolyte).
Reaction (b) would have have the lowest (almost zero) initial conductivity because HF is
a very weak acid/electrolyte.
Reaction (a) would have an intermediate initial conductivity between that for reactions (b) and (c).
Figure (1) is for reaction (a); figure (2) is for reaction (b); and figure (3) is for reaction (c).

4.29 (a) Sr+ + At → Sr + At+ No reaction.


(b) Si + At+ → Si+ + At Reaction would occur.
(c) Sr + Si+ → Sr+ + Si Reaction would occur.

Additional Problems
Aqueous Reactions and Net Ionic Equations

4.30 (a) precipitation (b) redox (c) acid-base neutralization

4.31 (a) redox (b) precipitation (c) acid-base neutralization

4.32 (a) Ionic equation:


Hg2+(aq) + 2 NO3-(aq) + 2 Na+(aq) + 2 I-(aq) → 2 Na+(aq) + 2 NO3-(aq) + HgI2(s)
Delete spectator ions from the ionic equation to get the net ionic equation.
Net ionic equation: Hg2+(aq) + 2 I-(aq) → HgI2(s)
Heat
(b) 2 HgO(s) → 2 Hg(l) + O2(g)
(c) Ionic equation:

69
Chapter 4 - Reactions in Aqueous Solutions
______________________________________________________________________________

H3PO4(aq) + 3 K+(aq) + 3 OH-(aq) → 3 K+(aq) + PO43-(aq) + 3 H2O(l)


Delete spectator ions from the ionic equation to get the net ionic equation.
Net ionic equation: H3PO4(aq) + 3 OH-(aq) → PO43-(aq) + 3 H2O(l)
4.33 (a) S8(s) + 8 O2(g) → 8 SO2(g)
(b) Ionic equation:
Ni2+(aq) + 2 Cl-(aq) + 2 Na+(aq) + S2-(aq) → NiS(s) + 2 Na+(aq) + 2 Cl-(aq)
Delete spectator ions from the ionic equation to get the net ionic equation.
Net ionic equation: Ni2+(aq) + S2-(aq) → NiS(s)
(c) Ionic equation:
2 CH3CO2H(aq) + Ba2+(aq) + 2 OH-(aq) → 2 CH3CO2-(aq) + Ba2+(aq) + 2 H2O(l)
Delete spectator ions from the ionic equation to get the net ionic equation.
Net ionic equation: CH3CO2H(aq) + OH-(aq) → CH3CO2-(aq) + H2O(l)

4.34 Ba(OH)2 is soluble in aqueous solution, dissociates into Ba2+(aq) and 2 OH-(aq), and
conducts electricity. In aqueous solution H2SO4 dissociates into H+(aq) and HSO4-(aq).
H2SO4 solutions conduct electricity. When equal molar solutions of Ba(OH)2 and H2SO4
are mixed, the insoluble BaSO4 is formed along with two H2O. In water BaSO4 does not
produce any appreciable amount of ions and the mixture does not conduct electricity.

4.35 H2O is polar and a good H+ acceptor. It allows the polar HCl to dissociate into ions in
aqueous solution: HCl + H2O → H3O+ + Cl-.
CHCl3 is not very polar and not a H+ acceptor and so does not allow the polar HCl to
dissociate into ions.

4.36 (a) HBr, strong electrolyte (b) HF, weak electrolyte


(c) NaClO4, strong electrolyte (d) (NH4)2CO3, strong electrolyte
(e) NH3, weak electrolyte (f) C2H5OH, nonelectrolyte

4.37 It is possible for a molecular compound to be a strong electrolyte. For example, HCl is a
molecular compound when pure but dissociates completely to give H+ and Cl- ions when
it dissolves in water.

4.38 (a) K2CO3 contains 3 ions (2 K+ and 1 CO32-).


The molar concentration of ions = 3 x 0.750 M = 2.25 M.
(b) AlCl3 contains 4 ions (1 Al3+ and 3 Cl-).
The molar concentration of ions = 4 x 0.355 M = 1.42 M.

4.39 (a) CH3OH is a nonelectrolyte. The ion concentration from CH3OH is zero.
(b) HClO4 is a strong acid.
HClO4(aq) → H+(aq) + ClO4-(aq)
In solution, there are 2 moles of ions per mole of HClO4.
The molar concentration of ions = 2 × 0.225 M = 0.450 M.

Precipitation Reactions and Solubility Rules

70
Chapter 4 - Reactions in Aqueous Solutions
______________________________________________________________________________

4.40 (a) Ag2O, insoluble (b) Ba(NO3)2, soluble


(c) SnCO3, insoluble (d) Fe2O3, insoluble

4.41 (a) ZnS, insoluble (b) Au2(CO3)3, insoluble


(c) PbCl2, insoluble (soluble in hot water) (d) MnO2, insoluble

4.42 (a) No precipitate will form. (b) FeCl2(aq) + 2 KOH(aq) → Fe(OH)2(s) + 2 KCl(aq)
(c) No precipitate will form. (d) No precipitate will form.

4.43 (a) MnCl2(aq) + Na2S(aq) → MnS(s) + 2 NaCl(aq)


(b) No precipitate will form.
(c) 3 Hg(NO3)2(aq) + 2 Na3PO4(aq) → Hg3(PO4)2(s) + 6 NaNO3(aq)
(d) Ba(NO3)2(aq) + 2 KOH(aq) → Ba(OH)2(s) + 2 KNO3(aq)

4.44 (a) Pb(NO3)2(aq) + Na2SO4(aq) → PbSO4(s) + 2 NaNO3(aq)


(b) 3 MgCl2(aq) + 2 K3PO4(aq) → Mg3(PO4)2(s) + 6 KCl(aq)
(c) ZnSO4(aq) + Na2CrO4(aq) → ZnCrO4(s) + Na2SO4(aq)

4.45 (a) AlCl3(aq) + 3 NaOH(aq) → Al(OH)3(s) + 3 NaCl(aq)


(b) Fe(NO3)2(aq) + Na2S(aq) → FeS(s) + 2 NaNO3(aq)
(c) CoSO4(aq) + K2CO3(aq) → CoCO3(s) + K2SO4(aq)

4.46 Add HCl(aq); it will selectively precipitate AgCl(s).

4.47 Add Na2SO4(aq); it will selectively precipitate BaSO4(s).

4.48 Ag+ is eliminated because it would have precipitated as AgCl(s); Ba2+ is eliminated
because it would have precipitated as BaSO4(s). The solution might contain Cs+ and/or
NH4+. Neither of these will precipitate with OH–, SO42–, or Cl–.

4.49 Cl- is eliminated because it would have precipitated as AgCl(s). OH- is eliminated
because it would have precipitated as either AgOH(s) or Cu(OH)2(s). SO42- is eliminated
because it would have precipitated as BaSO4(s). The solution might contain NO3-
because all nitrates are soluble.

Acids, Bases, and Neutralization Reactions

4.50 Add the solution to an active metal, such as magnesium. Bubbles of H2 gas indicate the
presence of an acid.

4.51 We use a double arrow to show the dissociation of a weak acid or weak base in aqueous
solution to indicate the equilibrium between reactants and products.

4.52 (a) 2 H+(aq) + 2 ClO4-(aq) + Ca2+(aq) + 2 OH-(aq) → Ca2+(aq) + 2 ClO4-(aq) + 2 H2O(l)

71
Chapter 4 - Reactions in Aqueous Solutions
______________________________________________________________________________

(b) CH3CO2H(aq) + Na+(aq) + OH-(aq) → CH3CO2-(aq) + Na+(aq) + H2O(l)

4.53 (a) 2 HF(aq) + Ca2+(aq) + 2 OH-(aq) → Ca2+(aq) + 2 F-(aq) + 2 H2O(l)


(b) Mg(OH)2(s) + 2 H+(aq) + 2 NO3-(aq) → Mg2+(aq) + 2 NO3-(aq) + 2 H2O(l)

4.54 (a) LiOH(aq) + HI(aq) → LiI(aq) + H2O(l)


Ionic equation: Li+(aq) + OH-(aq) + H+(aq) + I-(aq) → Li+(aq) + I-(aq) + H2O(l)
Delete spectator ions from the ionic equation to get the net ionic equation.
Net ionic equation: H+(aq) + OH-(aq) → H2O(l)

(b) 2 HBr(aq) + Ca(OH)2(aq) → CaBr2(aq) + 2 H2O(l)


Ionic equation:
2 H+(aq) + 2 Br-(aq) + Ca2+(aq) + 2 OH-(aq) → Ca2+(aq) + 2 Br-(aq) + 2 H2O(l)
Delete spectator ions from the ionic equation to get the net ionic equation.
Net ionic equation: H+(aq) + OH-(aq) → H2O(l)

4.55 (a) 2 Fe(OH)3(s) + 3 H2SO4(aq) → Fe2(SO4)3(aq) + 6 H2O(l)


Ionic equation and net ionic equation are the same.
2 Fe(OH)3(s) + 3 H+(aq) + 3 HSO4-(aq) → 2 Fe3+(aq) + 3 SO42-(aq) + 6 H2O(l)

(b) HClO3(aq) + NaOH(aq) → NaClO3(aq) + H2O(l)


Ionic equation H+(aq) + ClO3-(aq) + Na+(aq) + OH-(aq) → Na+(aq) + ClO3-(aq) + H2O(l)
Delete spectator ions from the ionic equation to get the net ionic equation.
Net ionic equation: H+(aq) + OH-(aq) → H2O(l)

Redox Reactions and Oxidation Numbers

4.56 The best reducing agents are at the bottom left of the periodic table. The best oxidizing
agents are at the top right of the periodic table (excluding the noble gases).

4.57 The most easily reduced elements in the periodic table are in the top-right corner,
excluding group 8A.
The most easily oxidized elements in the periodic table are in the bottom-left corner.

4.58 (a) An oxidizing agent gains electrons.


(b) A reducing agent loses electrons.
(c) A substance undergoing oxidation loses electrons.
(d) A substance undergoing reduction gains electrons.

4.59 (a) In a redox reaction, the oxidation number decreases for an oxidizing agent.
(b) In a redox reaction, the oxidation number increases for a reducing agent.
(c) In a redox reaction, the oxidation number increases for a substance undergoing oxidation.
(d) In a redox reaction, the oxidation number decreases for a substance undergoing
reduction.

72
Chapter 4 - Reactions in Aqueous Solutions
______________________________________________________________________________

4.60 (a) NO2 O -2, N +4 (b) SO3 O -2, S +6


(c) COCl2 O -2, Cl -1, C +4 (d) CH2Cl2 Cl -1, H +1, C 0
(e) KClO3 O -2, K +1, Cl +5 (f) HNO3 O -2, H +1, N +5

4.61 (a) VOCl3 O -2, Cl -1, V +5


(b) CuSO4 O -2, S +6, Cu +2
(c) CH2O O -2, H +1, C 0
(d) Mn2O7 O -2, Mn +7
(e) OsO4 O -2, Os +8
(f) H2PtCl6 Cl -1, H +1, Pt +4

4.62 (a) ClO3- O -2, Cl +5 (b) SO32- O -2, S +4


(c) C2O42- O -2, C +3 (d) NO2- O -2, N +3
(e) BrO- O -2, Br +1 (f) AsO43- O -2, As +5

4.63 (a) Cr(OH)4- O -2, H +1, Cr +3


(b) S2O32- O -2, S +2
(c) NO3- O -2, N +5
(d) MnO42- O -2, Mn +6
(e) HPO42- O -2, H +1, P +5
(f) V2O74- O -2, V +5

4.64 (a) Ca(s) + Sn2+(aq) → Ca2+(aq) + Sn(s)


Ca(s) is oxidized (oxidation number increases from 0 to +2).
Sn2+(aq) is reduced (oxidation number decreases from +2 to 0).
(b) ICl(s) + H2O(l) → HCl(aq) + HOI(aq)
No oxidation numbers change. The reaction is not a redox reaction.

4.65 (a) Si(s) + 2 Cl2(g) → SiCl4(l)


Si(s) is oxidized (oxidation number increases from 0 to +4).
Cl2(g) is reduced (oxidation number decreases from 0 to -1).
(b) Cl2(g) + 2 NaBr(aq) → Br2(aq) + 2 NaCl(aq)
Br-(aq) is oxidized (oxidation number increases from -1 to 0).
Cl2(g) is reduced (oxidation number decreases from 0 to -1).

4.66 (a) Zn is below Na+; therefore no reaction.


(b) Pt is below H+; therefore no reaction.
(c) Au is below Ag+; therefore no reaction.
(d) Ag is above Au3+; the reaction is Au3+(aq) + 3 Ag(s) → 3 Ag+(aq) + Au(s).

4.67 Sr is more metallic than Sb because it is in the same period and to the left of Sb on the
periodic table. Sr is the better reducing agent.
2 Sb3+(aq) + 3 Sr(s) → 2 Sb(s) + 3 Sr2+(aq) will occur, the reverse will not.

4.68 (a) “Any element higher in the activity series will react with the ion of any element lower

73
Chapter 4 - Reactions in Aqueous Solutions
______________________________________________________________________________

in the activity series.”


A + B+ → A+ + B; therefore A is higher than B.
C+ + D → no reaction; therefore C is higher than D.
B + D+ → B+ + D; therefore B is higher than D.
B + C+ → B+ + C; therefore B is higher than C.
The net result is A > B > C > D.
(b) (1) C is below A+; therefore no reaction.
(2) D is below A+; therefore no reaction.
4.69 (a) “Any element higher in the activity series will react with the ion of any element lower
in the activity series.”
2 A + B2+ → 2 A+ + B; therefore A is higher than B.
B + D2+ → B2+ + D; therefore B is higher than D.
A+ + C → no reaction; therefore A is higher than C.
2 C + B2+ → 2 C+ + B; therefore C is higher than B.
The net result is A > C > B > D.
(b) (1) D is below A+; therefore no reaction.
(2) C is above D2+; therefore the reaction will occur.

Balancing Redox Reactions

4.70 (a) N oxidation number decreases from +5 to +2; reduction.


(b) Zn oxidation number increases from 0 to +2; oxidation.
(c) Ti oxidation number increases from +3 to +4; oxidation.
(d) Sn oxidation number decreases from +4 to +2; reduction.

4.71 (a) O oxidation number decreases from 0 to -2; reduction.


(b) O oxidation number increases from -1 to 0; oxidation.
(c) Mn oxidation number decreases from +7 to +6; reduction.
(d) C oxidation number increases from -2 to 0; oxidation.

4.72 (a) NO3-(aq) → NO(g)


NO3-(aq) → NO(g) + 2 H2O(l)
4 H+(aq) + NO3-(aq) → NO(g) + 2 H2O(l)
3 e- + 4 H+(aq) + NO3-(aq) → NO(g) + 2 H2O(l)

(b) Zn(s) → Zn2+(aq) + 2 e-

(c) Ti3+(aq) → TiO2(s)


Ti3+(aq) + 2 H2O(l) → TiO2(s)
Ti3+(aq) + 2 H2O(l) → TiO2(s) + 4 H+(aq)
Ti3+(aq) + 2 H2O(l) → TiO2(s) + 4 H+(aq) + e-

(d) Sn4+(aq) + 2 e- → Sn2+(aq)

4.73 (a) O2(g) → OH-(aq)

74
Chapter 4 - Reactions in Aqueous Solutions
______________________________________________________________________________

O2(g) → OH-(aq) + H2O(l)


3 H+(aq) + O2(g) → OH-(aq) + H2O(l)
3 H+(aq) + 3 OH-(aq) + O2(g) → 4 OH-(aq) + H2O(l)
3 H2O(l) + O2(g) → 4 OH-(aq) + H2O(l)
4 e- + 2 H2O(l) + O2(g) → 4 OH-(aq)

(b) H2O2(aq) → O2(g)


H2O2(aq) → O2(g) + 2 H+(aq)

2 OH-(aq) + H2O2(aq) → O2(g) + 2 H+(aq) + 2 OH-(aq)


2 OH-(aq) + H2O2(aq) → O2(g) + 2 H2O(l) + 2 e-

(c) MnO4-(aq) → MnO42-(aq)


MnO4-(aq) + e- → MnO42-(aq)

(d) CH3OH(aq) → CH2O(aq)


CH3OH(aq) → CH2O(aq) + 2 H+(aq)
CH3OH(aq) + 2 OH-(aq) → CH2O(aq) + 2 H+(aq) + 2 OH-(aq)
CH3OH(aq) + 2 OH-(aq) → CH2O(aq) + 2 H2O(l)
CH3OH(aq) + 2 OH-(aq) → CH2O(aq) + 2 H2O(l) + 2 e-

4.74 (a) Te(s) + NO3-(aq) → TeO2(s) + NO(g)


oxidation: Te(s) → TeO2(s)
reduction: NO3-(aq) → NO(g)
(b) H2O2(aq) + Fe2+(aq) → Fe3+(aq) + H2O(l)
oxidation: Fe2+(aq) → Fe3+(aq)
reduction: H2O2(aq) → H2O(l)

4.75 (a) Mn(s) + NO3-(aq) → Mn2+(aq) + NO2(g)


oxidation: Mn(s) → Mn2+(aq)
reduction: NO3-(aq) → NO2(g)
(b) Mn3+(aq) → MnO2(s) + Mn2+(aq)
oxidation: Mn3+(aq) → MnO2(s)
reduction: Mn3+(aq) → Mn2+(aq)

4.76 (a) Cr2O72-(aq) → Cr3+(aq)


Cr2O72-(aq) → 2 Cr3+(aq)
Cr2O72-(aq) → 2 Cr3+(aq) + 7 H2O(l)
14 H+(aq) + Cr2O72-(aq) → 2 Cr3+(aq) + 7 H2O(l)
14 H+(aq) + Cr2O72-(aq) + 6 e- → 2 Cr3+(aq) + 7 H2O(l)

(b) CrO42-(aq) → Cr(OH)4-(aq)


4 H+(aq) + CrO42-(aq) → Cr(OH)4-(aq)
4 H+(aq) + 4 OH-(aq) + CrO42-(aq) → Cr(OH)4-(aq) + 4 OH-(aq)

75
Chapter 4 - Reactions in Aqueous Solutions
______________________________________________________________________________

4 H2O(l) + CrO42-(aq) → Cr(OH)4-(aq) + 4 OH-(aq)


4 H2O(l) + CrO42-(aq) + 3 e- → Cr(OH)4-(aq) + 4 OH-(aq)

(c) Bi3+(aq) → BiO3-(aq)


Bi3+(aq) + 3 H2O(l) → BiO3-(aq)
Bi3+(aq) + 3 H2O(l) → BiO3-(aq) + 6 H+(aq)
Bi3+(aq) + 3 H2O(l) + 6 OH-(aq) → BiO3-(aq) + 6 H+(aq) + 6 OH-(aq)
Bi3+(aq) + 3 H2O(l) + 6 OH-(aq) → BiO3-(aq) + 6 H2O(l)
Bi3+(aq) + 6 OH-(aq) → BiO3-(aq) + 3 H2O(l)
Bi3+(aq) + 6 OH-(aq) → BiO3-(aq) + 3 H2O(l) + 2 e-

(d) ClO-(aq) → Cl-(aq)


ClO-(aq) → Cl-(aq) + H2O(l)
2 H+(aq) + ClO-(aq) → Cl-(aq) + H2O(l)
2 H+(aq) + 2 OH-(aq) + ClO-(aq) → Cl-(aq) + H2O(l) + 2 OH-(aq)
2 H2O(l) + ClO-(aq) → Cl-(aq) + H2O(l) + 2 OH-(aq)
H2O(l) + ClO-(aq) → Cl-(aq) + 2 OH-(aq)
H2O(l) + ClO-(aq) + 2 e- → Cl-(aq) + 2 OH-(aq)

4.77 (a) VO2+(aq) → V3+(aq)


VO2+(aq) → V3+(aq) + H2O(l)
2 H+(aq) + VO2+(aq) → V3+(aq) + H2O(l)
2 H+(aq) + VO2+(aq) + e- → V3+(aq) + H2O(l)

(b) Ni(OH)2(s) → Ni2O3(s)


2 Ni(OH)2(s) → Ni2O3(s) + H2O(l)
2 Ni(OH)2(s) → Ni2O3(s) + H2O(l) + 2 H+(aq)
2 Ni(OH)2(s) + 2 OH-(aq) → Ni2O3(s) + H2O(l) + 2 H+(aq) + 2 OH-(aq)
2 Ni(OH)2(s) + 2 OH-(aq) → Ni2O3(s) + 3 H2O(l) + 2 e-

(c) NO3-(aq) → NO2(g)


NO3-(aq) → NO2(g) + H2O(l)
2 H+(aq) + NO3-(aq) → NO2(g) + H2O(l)
2 H+(aq) + NO3-(aq) + e- → NO2(g) + H2O(l)

(d) Br2(aq) → BrO3-(aq)


Br2(aq) → 2 BrO3-(aq)
Br2(aq) + 6 H2O(l) → 2 BrO3-(aq)
Br2(aq) + 6 H2O(l) → 2 BrO3-(aq) + 12 H+(aq)
Br2(aq) + 6 H2O(l) + 12 OH-(aq) → 2 BrO3-(aq) + 12 H+(aq) + 12 OH-(aq)
Br2(aq) + 6 H2O(l) + 12 OH-(aq) → 2 BrO3-(aq) + 12 H2O(l)
Br2(aq) + 12 OH-(aq) → 2 BrO3-(aq) + 6 H2O(l) + 10 e-

4.78 (a) MnO4-(aq) → MnO2(s)

76
Chapter 4 - Reactions in Aqueous Solutions
______________________________________________________________________________

MnO4-(aq) → MnO2(s) + 2 H2O(l)


4 H+(aq) + MnO4-(aq) → MnO2(s) + 2 H2O(l)
[4 H+(aq) + MnO4-(aq) +3 e- →MnO2(s) + 2 H2O(l)] x 2 (reduction half reaction)

IO3-(aq) → IO4-(aq)
H2O(l) + IO3-(aq) → IO4-(aq)
H2O(l) + IO3-(aq) → IO4-(aq) + 2 H+(aq)
[H2O(l) + IO3-(aq) → IO4-(aq) + 2 H+(aq) + 2 e-] x 3 (oxidation half reaction)
Combine the two half reactions.
8 H+(aq) + 3 H2O(l) + 2 MnO4-(aq) + 3 IO3-(aq) →
6 H+(aq) + 4 H2O(l) + 2 MnO2(s) + 3 IO4-(aq)
2 H+(aq) + 2 MnO4-(aq) + 3 IO3-(aq) → 2 MnO2(s) + 3 IO4-(aq) + H2O(l)

2 H+(aq) + 2 OH-(aq) + 2 MnO4-(aq) + 3 IO3-(aq) →


2 MnO2(s) + 3 IO4-(aq) + H2O(l) + 2 OH-(aq)
2 H2O(l) + 2 MnO4-(aq) + 3 IO3-(aq) →
2 MnO2(s) + 3 IO4-(aq) + H2O(l) + 2 OH-(aq)
H2O(l) + 2 MnO4-(aq) + 3 IO3-(aq) → 2 MnO2(s) + 3 IO4-(aq) + 2 OH-(aq)

(b) Cu(OH)2(s) → Cu(s)


Cu(OH)2(s) → Cu(s) + 2 H2O(l)
2 H+(aq) + Cu(OH)2(s) → Cu(s) + 2 H2O(l)
[2 H+(aq) + Cu(OH)2(s) + 2 e- → Cu(s) + 2 H2O(l)] x 2 (reduction half reaction)

N2H4(aq) → N2(g)
N2H4(aq) → N2(g) + 4 H+(aq)
N2H4(aq) → N2(g) + 4 H+(aq) + 4 e- (oxidation half reaction)

Combine the two half reactions.


4 H+(aq) + 2 Cu(OH)2(s) + N2H4(aq) → 2 Cu(s) + 4 H2O(l) + N2(g) + 4 H+(aq)
2 Cu(OH)2(s) + N2H4(aq) → 2 Cu(s) + 4 H2O(l) + N2(g)

(c) Fe(OH)2(s) → Fe(OH)3(s)


Fe(OH)2(s) + H2O(l) → Fe(OH)3(s)
Fe(OH)2(s) + H2O(l) → Fe(OH)3(s) + H+(aq)
[Fe(OH)2(s) + H2O(l) → Fe(OH)3(s) + H+(aq) + e-] x 3 (oxidation half reaction)

CrO42-(aq) → Cr(OH)4-(aq)
4 H+(aq) + CrO42-(aq) → Cr(OH)4-(aq)
4 H+(aq) + CrO42-(aq) + 3 e- → Cr(OH)4-(aq) (reduction half reaction)

Combine the two half reactions.


3 Fe(OH)2(s) + 3 H2O(l) + 4 H+(aq) + CrO42-(aq) →
3 Fe(OH)3(s) + 3 H+(aq) + Cr(OH)4-(aq)

77
Chapter 4 - Reactions in Aqueous Solutions
______________________________________________________________________________

3 Fe(OH)2(s) + 3 H2O(l) + H+(aq) + CrO42-(aq) → 3 Fe(OH)3(s) + Cr(OH)4-(aq)


3 Fe(OH)2(s) + 3 H2O(l) + H+(aq) + OH-(aq) + CrO42-(aq) →
3 Fe(OH)3(s) + Cr(OH)4-(aq) + OH-(aq)
3 Fe(OH)2(s) + 4 H2O(l) + CrO42-(aq) → 3 Fe(OH)3(s) + Cr(OH)4-(aq) + OH-(aq)

(d) ClO4-(aq) → ClO2-(aq)


ClO4-(aq) → ClO2-(aq) + 2 H2O(l)
4 H+(aq) + ClO4-(aq) → ClO2-(aq) + 2 H2O(l)
4 H+(aq) + ClO4-(aq) + 4 e- → ClO2-(aq) + 2 H2O(l) (reduction half reaction)

H2O2(aq) → O2(g)
H2O2(aq) → O2(g) + 2 H+(aq)
[H2O2(aq) → O2(g) + 2 H+(aq) + 2 e-] x 2 (oxidation half reaction)

Combine the two half reactions.


4 H+(aq) + ClO4-(aq) + 2 H2O2(aq) → ClO2-(aq) + 2 H2O(l) + 2 O2(g) + 4 H+(aq)
ClO4-(aq) + 2 H2O2(aq) → ClO2-(aq) + 2 H2O(l) + 2 O2(g)

4.79 (a) S2O32-(aq) → S4O62-(aq)


2 S2O32-(aq) → S4O62-(aq)
2 S2O32-(aq) → S4O62-(aq) + 2 e- (oxidation half reaction)

I2(aq) → I-(aq)
I2(aq) → 2 I-(aq)
I2(aq) + 2 e- → 2 I-(aq) (reduction half reaction)

Combine the two half reactions.


2 S2O32-(aq) + I2(aq) → S4O62-(aq) + 2 I-(aq)

(b) Mn2+(aq) → MnO2(s)


Mn2+(aq) + 2 H2O(l) → MnO2(s)
Mn2+(aq) + 2 H2O(l) → MnO2(s) + 4 H+(aq)
Mn2+(aq) + 2 H2O(l) → MnO2(s) + 4 H+(aq) + 2 e- (oxidation half reaction)

H2O2(aq) → 2 H2O(l)
2 H+(aq) + H2O2(aq) → 2 H2O(l)
2 H+(aq) + H2O2(aq) + 2 e- → 2 H2O(l) (reduction half reaction)

Combine the two half reactions.


Mn2+(aq) + 2 H2O(l) + 2 H+(aq) + H2O2(aq) → MnO2(s) + 4 H+(aq) + 2 H2O(l)
Mn2+(aq) + H2O2(aq) → MnO2(s) + 2 H+(aq)
Mn2+(aq) + H2O2(aq) + 2 OH-(aq) → MnO2(s) + 2 H+(aq) + 2 OH-(aq)
Mn2+(aq) + H2O2(aq) + 2 OH-(aq) → MnO2(s) + 2 H2O(l)

78
Chapter 4 - Reactions in Aqueous Solutions
______________________________________________________________________________

(c) Zn(s) → Zn(OH)42-(aq)


4 H2O(l) + Zn(s) → Zn(OH)42-(aq)
4 H2O(l) + Zn(s) → Zn(OH)42-(aq) + 4 H+(aq)
[4 H2O(l) + Zn(s) → Zn(OH)42-(aq) + 4 H+(aq) + 2 e-] x 4 (oxidation half reaction)

NO3-(aq) → NH3(aq)
NO3-(aq) → NH3(aq) + 3 H2O(l)
9 H+(aq) + NO3-(aq) → NH3(aq) + 3 H2O(l)
9 H+(aq) + NO3-(aq) + 8 e- → NH3(aq) + 3 H2O(l) (reduction half reaction)

Combine the two half reactions.


16 H2O(l) + 4 Zn(s) + 9 H+(aq) + NO3-(aq) →
4 Zn(OH)42-(aq) + 16 H+(aq) + NH3(aq) + 3 H2O(l)
13 H2O(l) + 4 Zn(s) + NO3-(aq) → 4 Zn(OH)42-(aq) + 7 H+(aq) + NH3(aq)
13 H2O(l) + 4 Zn(s) + NO3-(aq) + 7 OH-(aq) →
4 Zn(OH)42-(aq) + 7 H+(aq) + 7 OH-(aq) + NH3(aq)
13 H2O(l) + 4 Zn(s) + NO3-(aq) + 7 OH-(aq) →
4 Zn(OH)42-(aq) + 7 H2O(l) + NH3(aq)
6 H2O(l) + 4 Zn(s) + NO3-(aq) + 7 OH-(aq) → 4 Zn(OH)42-(aq) + NH3(aq)

(d) Bi(OH)3(s) → Bi(s)


Bi(OH)3(s) → Bi(s) + 3 H2O(l)
3 H+(aq) + Bi(OH)3(s) → Bi(s) + 3 H2O(l)
[3 H+(aq) + Bi(OH)3(s) + 3 e- → Bi(s) + 3 H2O(l)] x 2 (reduction half reaction)

Sn(OH)3-(aq) → Sn(OH)62-(aq)
Sn(OH)3-(aq) + 3 H2O(l) → Sn(OH)62-(aq)
Sn(OH)3-(aq) + 3 H2O(l) → Sn(OH)62-(aq) + 3 H+(aq)
[Sn(OH)3-(aq) + 3 H2O(l) → Sn(OH)62-(aq) + 3 H+(aq) + 2 e-] x 3
(oxidation half reaction)
Combine the two half reactions.
6 H+(aq) + 2 Bi(OH)3(s) + 3 Sn(OH)3-(aq) + 9 H2O(l) →
2 Bi(s) + 6 H2O(l) + 3 Sn(OH)62-(aq) + 9 H+(aq)
2 Bi(OH)3(s) + 3 Sn(OH)3-(aq) + 3 H2O(l) →2 Bi(s) + 3 Sn(OH)62-(aq) + 3 H+(aq)
2 Bi(OH)3(s) + 3 Sn(OH)3-(aq) + 3 H2O(l) + 3 OH-(aq) →
2 Bi(s) + 3 Sn(OH)62-(aq) + 3 H+(aq) + 3 OH-(aq)
2 Bi(OH)3(s) + 3 Sn(OH)3-(aq) + 3 H2O(l) + 3 OH-(aq) →
2 Bi(s) + 3 Sn(OH)62-(aq) + 3 H2O(l)
2 Bi(OH)3(s) + 3 Sn(OH)3-(aq) + 3 OH-(aq) → 2 Bi(s) + 3 Sn(OH)62-(aq)

4.80 (a) Zn(s) → Zn2+(aq)


Zn(s) → Zn2+(aq) + 2 e- (oxidation half reaction)

79
Chapter 4 - Reactions in Aqueous Solutions
______________________________________________________________________________

VO2+(aq) → V3+(aq)
VO2+(aq) → V3+(aq) + H2O(l)
2 H+(aq) + VO2+(aq) → V3+(aq) + H2O(l)
[2 H+(aq) + VO2+(aq) + e- → V3+(aq) + H2O(l)] x 2 (reduction half reaction)

Combine the two half reactions.


Zn(s) + 2 VO2+(aq) + 4 H+(aq) → Zn2+(aq) + 2 V3+(aq) + 2 H2O(l)

(b) Ag(s) → Ag+(aq)


Ag(s) → Ag+(aq) + e- (oxidation half reaction)

NO3-(aq) → NO2(g)
NO3-(aq) → NO2(g) + H2O(l)
2 H+(aq) + NO3-(aq) → NO2(g) + H2O(l)
2 H+(aq) + NO3-(aq) + e- → NO2(g) + H2O(l) (reduction half reaction)

Combine the two half reactions.


2 H+(aq) + Ag(s) + NO3-(aq) → Ag+(aq) + NO2(g) + H2O(l)

(c) Mg(s) → Mg2+(aq)


[Mg(s) → Mg2+(aq) + 2 e- ] x 3 (oxidation half reaction)

VO43-(aq) → V2+(aq)
VO43-(aq) → V2+(aq) + 4 H2O(l)
8 H+(aq) + VO43-(aq) → V2+(aq) + 4 H2O(l)
[8 H+(aq) + VO43-(aq) + 3 e- → V2+(aq) + 4 H2O(l)] x 2 (reduction half reaction)

Combine the two half reactions.


3 Mg(s) + 16 H+(aq) + 2 VO43-(aq) → 3 Mg2+(aq) + 2 V2+(aq) + 8 H2O(l)

(d) I-(aq) → I3-(aq)


3 I-(aq) → I3-(aq)
[3 I-(aq) → I3-(aq) + 2 e-] x 8 (oxidation half reaction)

IO3-(aq) → I3-(aq)
3 IO3-(aq) → I3-(aq)
3 IO3-(aq) → I3-(aq) + 9 H2O(l)
18 H+(aq) + 3 IO3-(aq) → I3-(aq) + 9 H2O(l)
18 H+(aq) + 3 IO3-(aq) + 16 e- → I3-(aq) + 9 H2O(l) (reduction half reaction)

Combine the two half reactions.


18 H+(aq) + 3 IO3-(aq) + 24 I-(aq) → 9 I3-(aq) + 9 H2O(l)
Divide each coefficient by 3.
6 H+(aq) + IO3-(aq) + 8 I-(aq) → 3 I3-(aq) + 3 H2O(l)

80
Chapter 4 - Reactions in Aqueous Solutions
______________________________________________________________________________

4.81 (a) MnO4-(aq) → Mn2+(aq)


MnO4-(aq) → Mn2+(aq) + 4 H2O(l)
8 H+(aq) + MnO4-(aq) → Mn2+(aq) + 4 H2O(l)
[8 H+(aq) + MnO4-(aq) + 5 e- → Mn2+(aq) + 4 H2O(l)] x 4
(reduction half reaction)
C2H5OH(aq) → CH3CO2H(aq)
C2H5OH(aq) + H2O(l) → CH3CO2H(aq)
C2H5OH(aq) + H2O(l) → CH3CO2H(aq) + 4 H+(aq)
[C2H5OH(aq) + H2O(l) → CH3CO2H(aq) + 4 H+(aq) + 4 e-] x 5
(oxidation half reaction)
Combine the two half reactions.
32 H+(aq) + 4 MnO4-(aq) + 5 C2H5OH(aq) + 5 H2O(l) →
4 Mn2+(aq) + 16 H2O(l) + 5 CH3CO2H(aq) + 20 H+(aq)
12 H (aq) + 4 MnO4-(aq) + 5 C2H5OH(aq) →
+

4 Mn2+(aq) + 11 H2O(l) + 5 CH3CO2H(aq)

(b) Cr2O72-(aq) → Cr3+(aq)


Cr2O72-(aq) → 2 Cr3+(aq)
Cr2O72-(aq) → 2 Cr3+(aq) + 7 H2O(l)

14 H+(aq) + Cr2O72-(aq) → 2 Cr3+(aq) + 7 H2O(l)


14 H+(aq) + Cr2O72-(aq) + 6 e- → 2 Cr3+(aq) + 7 H2O(l) (reduction half reaction)

H2O2(aq) → O2(g)
H2O2(aq) → O2(g) + 2 H+(aq)
[H2O2(aq) → O2(g) + 2 H+(aq) + 2 e-] x 3 (oxidation half reaction)

Combine the two half reactions.


14 H+(aq) + Cr2O72-(aq) + 3 H2O2(aq) →
2 Cr3+(aq) + 7 H2O(l) + 3 O2(g) + 6 H+(aq)
8 H+(aq) + Cr2O72-(aq) + 3 H2O2(aq) → 2 Cr3+(aq) + 7 H2O(l) + 3 O2(g)

(c) Sn2+(aq) → Sn4+(aq)


[Sn2+(aq) → Sn4+(aq) + 2 e-] x 4 (oxidation half reaction)

IO4-(aq) → I-(aq)
IO4-(aq) → I-(aq) + 4 H2O(l)
8 H+(aq) + IO4-(aq) → I-(aq) + 4 H2O(l)
8 H+(aq) + IO4-(aq) + 8 e- → I-(aq) + 4 H2O(l) (reduction half reaction)

Combine the two half reactions.


4 Sn2+(aq) + 8 H+(aq) + IO4-(aq) → 4 Sn4+(aq) + I-(aq) + 4 H2O(l)

81
Chapter 4 - Reactions in Aqueous Solutions
______________________________________________________________________________

(d) PbO2(s) + Cl-(aq) → PbCl2(s)


PbO2(s) + 2 Cl-(aq) → PbCl2(s)
PbO2(s) + 2 Cl-(aq) → PbCl2(s) + 2 H2O(l)
PbO2(s) + 4 H+(aq) + 2 Cl-(aq) → PbCl2(s) + 2 H2O(l)
[PbO2(s) + 4 H+(aq) + 2 Cl-(aq) + 2 e- → PbCl2(s) + 2 H2O(l)] x 2
(reduction half reaction)
H2O(l) → O2(g)
2 H2O(l) → O2(g)
2 H2O(l) → O2(g) + 4 H+(aq)
2 H2O(l) → O2(g) + 4 H+(aq) + 4 e- (oxidation half reaction)

Combine the two half reactions.


2 PbO2(s) + 8 H+(aq) + 4 Cl-(aq) + 2 H2O(l) →
2 PbCl2(s) + 4 H2O(l) + O2(g) + 4 H+(aq)
2 PbO2(s) + 4 H+(aq) + 4 Cl-(aq) → 2 PbCl2(s) + 2 H2O(l) + O2(g)

Redox Titrations

4.82 I2(aq) + 2 S2O32-(aq) → S4O62-(aq) + 2 I-(aq); 35.20 mL = 0.032 50 L


2_
0.150 mol S2 O3 1 mol I2 253.8 g I2
0.035 20 L x x 2_
x = 0.670 g I2
L 2 mol S2 O3 1 mol I2

1 mol I2 2 mol S2 O32 _


4.83 2.486 g I2 x x = 1.959 x 10-2 mol S2O32-
253.8 g I2 1 mol I2
1L
1.959 x 10_ 2 mol x = 0.0784 L; 0.0784 L = 78.4 mL
0.250 mol

4.84 3 H3AsO3(aq) + BrO3-(aq) → Br-(aq) + 3 H3AsO4(aq)


22.35 mL = 0.022 35 L and 50.00 mL = 0.050 00 L
0.100 mol BrO3_ 3 mol H3 AsO3
0.022 35 L x x _
= 6.70 x 10_ 3 mol H3 AsO3
L 1 mol BrO3
_3
6.70 x 10 mol
molarity = = 0.134 M As(III)
0.050 00 L

4.85 As2O3, 197.84 amu; 28.55 mL = 0.028 55 L


1 mol As2 O3 2 mol H3 AsO3 1 mol BrO3_
1.550 g As2O3 x x x
197.84 g As2 O3 1 mol As2 O3 3 mol H3 AsO3
5.223 x 10_ 3 mol
= 5.223 x 10-3 mol BrO3-; KBrO3 molarity = = 0.1829 M
0.028 55 L

4.86 2 Fe3+(aq) + Sn2+(aq) → 2 Fe2+(aq) + Sn4+(aq); 13.28 mL = 0.013 28 L

82
Chapter 4 - Reactions in Aqueous Solutions
______________________________________________________________________________

0.1015 mol Sn 2+ 2 mol Fe3+ 55.847 g Fe3+


0.013 28 L x x x = 0.1506 g Fe3+
L 1 mol Sn 2+ 1 mol Fe3+
0.1506 g
mass % Fe = x 100% = 80.32%
0.1875 g

4.87 Fe2O3, 159.69 amu; 23.84 mL = 0.023 84 L


1 mol Fe2 O3 2 mol Fe3+ 1 mol Sn 2+
1.4855 g Fe2O3 x x x = 0.009 302 mol
159.69 g Fe2 O3 1 mol Fe 2 O3 2 mol Fe3+
Sn2+
0.009 302 mol
Sn2+ molarity = = 0.3902 M
0.023 84 L

4.88 C2H5OH(aq) + 2 Cr2O72-(aq) + 16 H+(aq) → 2 CO2(g) + 4 Cr3+(aq) + 11 H2O(l)


C2H5OH, 46.07 amu; 8.76 mL = 0.008 76 L
0.049 88 mol Cr 2 O72 _ 1 mol C2 H5 OH 46.07 g C2 H5 OH
0.008 76 L x x x
L 2 mol Cr 2 O72 _ 1 mol C2 H5 OH
= 0.010 07 g C2 H5 OH
0.010 07 g
mass % C2H5OH = x 100% = 0.101%
10.002 g

4.89 21.08 mL = 0.021 08 L


0.021 08 L x
5 mol H 2 C2 O 4 1 mol Ca 2+ 40.08 g Ca 2+
9.88 x 10_ 4 mol MnO 4_ overL x x x
2 mol MnO4_ 1 mol H 2 C2 O4 1 mol Ca 2+
= 0.002 09 g = 2.09 mg

General Problems

4.90 (a) [Fe(CN)6]3-(aq) → Fe(CN)6]4-(aq)


([Fe(CN)6]3-(aq) + e- → [Fe(CN)6]4-(aq)) x 4 (reduction half reaction)

N2H4(aq) → N2(g)
N2H4(aq) → N2(g) + 4 H+(aq)
N2H4(aq) → N2(g) + 4 H+(aq) + 4 e-
N2H4(aq) + 4 OH-(aq) → N2(g) + 4 H+(aq) + 4 OH-(aq) + 4 e-
N2H4(aq) + 4 OH-(aq) → N2(g) + 4 H2O(l) + 4 e- (oxidation half reaction)

Combine the two half reactions.


4 [Fe(CN)6]3-(aq) + N2H4(aq) + 4 OH-(aq) →
4 [Fe(CN)6]4-(aq) + N2(g) + 4 H2O(l)

83
Chapter 4 - Reactions in Aqueous Solutions
______________________________________________________________________________

(b) Cl2(g) → Cl-(aq)


Cl2(g) → 2 Cl-(aq)
Cl2(g) + 2 e- → 2 Cl-(aq) (reduction half reaction)

SeO32-(aq) → SeO42-(aq)
SeO32-(aq) + H2O(l) → SeO42-(aq)
SeO32-(aq) + H2O(l) → SeO42-(aq) + 2 H+(aq)
SeO32-(aq) + H2O(l) → SeO42-(aq) + 2 H+(aq) + 2 e-
SeO32-(aq) + H2O(l) + 2 OH-(aq) → SeO42-(aq) + 2 H+(aq) + 2 OH-(aq) + 2 e-
SeO32-(aq) + H2O(l) + 2 OH-(aq) → SeO42-(aq) + 2 H2O(l) + 2 e-
SeO32-(aq) + 2 OH-(aq) → SeO42-(aq) + H2O(l) + 2 e- (oxidation half reaction)

Combine the two half reactions.


SeO32-(aq) + Cl2(g) + 2 OH-(aq) → SeO42-(aq) + 2 Cl-(aq) + H2O(l)

(c) CoCl2(aq) → Co(OH)3(s) + Cl-(aq)


CoCl2(aq) → Co(OH)3(s) + 2 Cl-(aq)
CoCl2(aq) + 3 H2O(l) → Co(OH)3(s) + 2 Cl-(aq)
CoCl2(aq) + 3 H2O(l) → Co(OH)3(s) + 2 Cl-(aq) + 3 H+(aq)
[CoCl2(aq) + 3 H2O(l) → Co(OH)3(s) + 2 Cl-(aq) + 3 H+(aq) + e-] x 2
(oxidation half reaction)
HO2 (aq) → H2O(l)
-

HO2-(aq) → 2 H2O(l)
3 H+(aq) + HO2-(aq) → 2 H2O(l)
3 H+(aq) + HO2-(aq) + 2 e- → 2 H2O(l) (reduction half reaction)
Combine the two half reactions.
2 CoCl2(aq) + 6 H2O(l) + 3 H+(aq) + HO2-(aq) →
2 Co(OH)3(s) + 4 Cl-(aq) + 6 H+(aq) + 2 H2O(l)
2 CoCl2(aq) + 4 H2O(l) + HO2-(aq) → 2 Co(OH)3(s) + 4 Cl-(aq) + 3 H+(aq)
2 CoCl2(aq) + 4 H2O(l) + HO2-(aq) + 3 OH-(aq) →
2 Co(OH)3(s) + 4 Cl-(aq) + 3 H+(aq) + 3 OH-(aq)
2 CoCl2(aq) + 4 H2O(l) + HO2-(aq) + 3 OH-(aq) →
2 Co(OH)3(s) + 4 Cl-(aq) + 3 H2O(l)
2 CoCl2(aq) + H2O(l) + HO2 (aq) + 3 OH-(aq) → 2 Co(OH)3(s) + 4 Cl-(aq)
-

4.91 57.91 mL = 0.057 91 L


1 mol Fe 2+ 55.85 g Fe 2+
0.057 91 L x 0.1018 mol Ce4+ overL x 4+
x 2+
= 0.3292 g Fe 2+
1 mol Ce 1 mol Fe
0.3292 g
mass % Fe = x 100% = 26.80%
1.2284 g

4.92 (a) C2H6 H +1, C -3


(b) Na2B4O7 O -2, Na +1,B +3
(c) Mg2SiO4 O -2, Mg +2, Si +4

84
Chapter 4 - Reactions in Aqueous Solutions
______________________________________________________________________________

4.93 (a) PbO2(s) → Pb2+(aq)


PbO2(s) → Pb2+(aq) + 2 H2O(l)
4 H+(aq) + PbO2(s) → Pb2+(aq) + 2 H2O(l)
[4 H+(aq) + PbO2(s) + 2 e- → Pb2+(aq) + 2 H2O(l)] x 5 (reduction half reaction)

Mn2+(aq) → MnO4-(aq)
4 H2O(l) + Mn2+(aq) → MnO4-(aq)
4 H2O(l) + Mn2+(aq) → MnO4-(aq) + 8 H+(aq)
[4 H2O(l) + Mn2+(aq) → MnO4-(aq) + 8 H+(aq) + 5 e-] x 2
(oxidation half reaction)
Combine the two half reactions.
20 H+(aq) + 5 PbO2(s) + 8 H2O(l) + 2 Mn2+(aq) →
5 Pb2+(aq) + 10 H2O(l) + 2 MnO4-(aq) + 16 H+(aq)
4 H (aq) + 5 PbO2(s) + 2 Mn2+(aq) → 5 Pb2+(aq) + 2 H2O(l) + 2 MnO4-(aq)
+

(b) As2O3(s) → H3AsO4(aq)


As2O3(s) → 2 H3AsO4(aq)
5 H2O(l) + As2O3(s) → 2 H3AsO4(aq)
5 H2O(l) + As2O3(s) → 2 H3AsO4(aq) + 4 H+(aq)
5 H2O(l) + As2O3(s) → 2 H3AsO4(aq) + 4 H+(aq) + 4 e- (oxidation half
reaction)

NO3-(aq) → HNO2(aq)
NO3-(aq) → HNO2(aq) + H2O(l)

3 H+(aq) + NO3-(aq) → HNO2(aq) + H2O(l)


[3 H+(aq) + NO3-(aq) + 2 e- → HNO2(aq) + H2O(l)] x 2
(reduction half reaction)
Combine the two half reactions.
5 H2O(l) + As2O3(s) + 6 H+(aq) + 2 NO3-(aq) →
2 H3AsO4(aq) + 4 H+(aq) + 2 HNO2(aq) + 2 H2O(l)
3 H2O(l) + As2O3(s) + 2 H+(aq) + 2 NO3-(aq) → 2 H3AsO4(aq) + 2 HNO2(aq)

(c) Br2(aq) → Br-(aq)


Br2(aq) → 2 Br-(aq)
Br2(aq) + 2 e- → 2 Br-(aq) (reduction half reaction)

SO2(g) → HSO4-(aq)
2 H2O(l) + SO2(g) → HSO4-(aq)
2 H2O(l) + SO2(g) → HSO4-(aq) + 3 H+(aq)
2 H2O(l) + SO2(g) → HSO4-(aq) + 3 H+(aq) + 2 e- (oxidation half reaction)

85
Chapter 4 - Reactions in Aqueous Solutions
______________________________________________________________________________

Combine the two half reactions.


2 H2O(l) + Br2(aq) + SO2(g) → 2 Br-(aq) + HSO4-(aq) + 3 H+(aq)

(d) I-(aq) → I2(s)


2 I-(aq) → I2(s)
2 I-(aq) → I2(s) + 2 e- (oxidation half reaction)

NO2-(aq) → NO(g)
NO2-(aq) → NO(g) + H2O(l)
2 H+(aq) + NO2-(aq) → NO(g) + H2O(l)
[2 H+(aq) + NO2-(aq) + e- → NO(g) + H2O(l)] x 2 (reduction half reaction)

Combine the two half reactions.


4 H+(aq) + 2 NO2-(aq) + 2 I-(aq) → 2 NO(g) + I2(s) + 2 H2O(l)

4.94 (a) “Any element higher in the activity series will react with the ion of any element lower
in the activity series.”
C + B+ → C+ + B; therefore C is higher than B.
A+ + D → no reaction; therefore A is higher than D.
C+ + A → no reaction; therefore C is higher than A.
D + B+ → D+ + B; therefore D is higher than B.
The net result is C > A > D > B.
(b) (1) The reaction, A+ + C → A + C+, will occur because C is above A in the activity
series.
(2) The reaction, A+ + B → A + B+, will not occur because B is below A in the
activity series.

4.95 (a) Ksp = [Ag+]2[CrO42-]


(b) Ag2CrO4(s) _ 2 Ag+(aq) + CrO42-(aq)
2x x
In a saturated solution 2x = [Ag+] and x = [CrO42-].
Ksp = [Ag+]2[CrO42-] = 1.1 x 10-12 = (2x)2(x) = 4x3; Solve for x; x = 6.5 x 10-5 M
[Ag+] = 2x = 2(6.5 x 10-5 M) = 1.3 x 10-4 M; [CrO42-] = x = 6.5 x 10-5 M

4.96 MgF2(s) _ Mg2+(aq) + 2 F-(aq)


x 2x
[Mg ] = x = 2.6 x 10 M and [F-] = 2x = 2(2.6 x 10-4 M) = 5.2 x 10-4 M in a saturated
2+ -4

solution.
Ksp = [Mg2+][F-]2 = (2.6 x 10-4 M)(5.2 x 10-4 M)2 = 7.0 x 10-11

4.97 65.20 mL = 0.065 20 L

86
Chapter 4 - Reactions in Aqueous Solutions
______________________________________________________________________________

1 mol succinic acid


1.926 g succinic acid x = 0.016 31 mol succinic acid
118.1 g succinic acid
mol NaOH
0.5000 x 0.065 20 L = 0.032 60 mol NaOH
1L
0.032 60 mol NaOH
= 2; therefore succinic acid has two acidic hydrogens.
0.016 31 mol succinic acid

4.98 (a) Add HCl to precipitate Hg2Cl2. Hg22+(aq) + 2Cl-(aq) → Hg2Cl2(s)


(b) Add H2SO4 to precipitate PbSO4. Pb2+(aq) + SO42-(aq) → PbSO4(s)
(c) Add Na2CO3 to precipitate CaCO3. Ca2+(aq) + CO32-(aq) → CaCO3(s)
(d) Add Na2SO4 to precipitate BaSO4. Ba2+(aq) + SO42-(aq) → BaSO4(s)

4.99 (a) Add AgNO3 to precipitate AgCl. Ag+(aq) + Cl-(aq) → AgCl(s)


(b) Add NiCl2 to precipitate NiS. Ni2+(aq) + S2-(aq) → NiS(s)
(c) Add CaCl2 to precipitate CaCO3. Ca2+(aq) + CO32-(aq) → CaCO3(s)
(d) Add MgCl2 to precipitate Mg(OH)2. Mg2+(aq) + 2 OH-(aq) → Mg(OH)2(s)

4.100 All four reactions are redox reactions.


(a) Mn(OH)2(s) → Mn(OH)3(s)
Mn(OH)2(s) + OH-(aq) → Mn(OH)3(s)
[Mn(OH)2(s) + OH-(aq) → Mn(OH)3(s) + e-] x 2 (oxidation half reaction)

H2O2(aq) → 2 H2O(l)
2 H+(aq) + H2O2(aq) → 2 H2O(l)
2 e- + 2 H+(aq) + H2O2(aq) → 2 H2O(l)
2 e- + 2 OH-(aq) + 2 H+(aq) + H2O2(aq) → 2 H2O(l) + 2 OH-(aq)
2 e- + 2 H2O(l) + H2O2(aq) → 2 H2O(l) + 2 OH-(aq)
2 e- + H2O2(aq) → 2 OH-(aq) (reduction half reaction)

Combine the two half reactions.


2 Mn(OH)2(s) + 2 OH-(aq) + H2O2(aq) → 2 Mn(OH)3(s) + 2 OH-(aq)
2 Mn(OH)2(s) + H2O2(aq) → 2 Mn(OH)3(s)

(b) [MnO42-(aq) → MnO4-(aq) + e- ] x 2 (oxidation half reaction)

MnO42-(aq) → MnO2(s)
MnO42-(aq) → MnO2(s) + 2 H2O(l)
4 H+(aq) + MnO42-(aq) → MnO2(s) + 2 H2O(l)
2 e- + 4 H+(aq) + MnO42-(aq) → MnO2(s) + 2 H2O(l) (reduction half reaction)

Combine the two half reactions.


4 H+(aq) + 3 MnO42-(aq) → MnO2(s) + 2 MnO4-(aq) + 2 H2O(l)

87
Chapter 4 - Reactions in Aqueous Solutions
______________________________________________________________________________

(c) I-(aq) → I3-(aq)


3 I-(aq) → I3-(aq)
[3 I-(aq) → I3-(aq) + 2 e- ] x 8 (oxidation half reaction)

IO3-(aq) → I3-(aq)
3 IO3-(aq) → I3-(aq)
3 IO3-(aq) → I3-(aq) + 9 H2O(l)
18 H+(aq) + 3 IO3-(aq) → I3-(aq) + 9 H2O(l)
16 e- + 18 H+(aq) + 3 IO3-(aq) → I3-(aq) + 9 H2O(l) (reduction half reaction)

Combine the two half reactions.


24 I-(aq) + 3 IO3-(aq) + 18 H+(aq) → 9 I3-(aq) + 9 H2O(l)
Divide all coefficients by 3.
8 I-(aq) + IO3-(aq) + 6 H+(aq) → 3 I3-(aq) + 3 H2O(l)

(d) P(s) → HPO32-(aq)


3 H2O(l) + P(s) → HPO32-(aq)
3 H2O(l) + P(s) → HPO32-(aq) + 5 H+(aq)
[3 H2O(l) + P(s) → HPO32-(aq) + 5 H+(aq) + 3 e- ] x 2
(oxidation half reaction)
PO43-(aq) → HPO32-(aq)
PO43-(aq) → HPO32-(aq) + H2O(l)
3 H+(aq) + PO43-(aq) → HPO32-(aq) + H2O(l)
[2 e- + 3 H+(aq) + PO43-(aq) → HPO32-(aq) + H2O(l)] x 3
(reduction half reaction)
Combine the two half reactions and add OH-.
6 H2O(l) + 2 P(s) + 9 H+(aq) + 3 PO43-(aq) →
5 HPO32-(aq) + 10 H+(aq) + 3 H2O(l)
3 H2O(l) + 2 P(s) + 3 PO4 (aq) → 5 HPO32-(aq) + H+(aq)
3-

3 H2O(l) + 2 P(s) + 3 PO43-(aq) + OH-(aq) →


5 HPO32-(aq) + H+(aq) + OH-(aq)

3 H2O(l) + 2 P(s) + 3 PO43-(aq) + OH-(aq) → 5 HPO32-(aq) + H2O(l)


2 H2O(l) + 2 P(s) + 3 PO43-(aq) + OH-(aq) → 5 HPO32-(aq)

4.101 100.0 mL = 0.1000 L; 47.14 mL = 0.047 14 L


mol NaOH
mol HCl and HBr = mol H+ = 0.1235 x 0.047 14 L = 5.8218 x 10-3 mol
1L
mass of AgCl and AgBr = 0.9974 g; mol Ag = mol H+ = 5.8218 x 10-3 mol
107.87 g Ag
mass of Ag = 5.8218 x 10-3 mol Ag x = 0.6280 g Ag
1 mol Ag
mass of Cl and Br = 0.9974 g - 0.6280 g = 0.3694 g of Cl and Br
Let Y = moles Cl and Z = moles Br in 0.3694 g of Cl and Br.
Let (Y + Z) = moles Ag in 0.6280 g Ag.

88
Chapter 4 - Reactions in Aqueous Solutions
______________________________________________________________________________

For Ag: 0.6280 g = (Y + Z) x 107.87 g


For Cl and Br: 0.3694 g = (Y x 35.453 g) + (Z x 79.904 g)
Solve the simultaneous equations for Y and Z.
 0.6280 g 
Rearrange the Ag equation:  _ Z  = Y
 107.87 g 
Substitute for Y in the Cl and Br equation above and solve for Z.
 0.6280 g  
0.3694 g =  _ Z  x 35.453 g  + (Z x 79.904 g)
 107.87 g  
0.1630
Z= = 3.667 x 10-3
44.451
 0.6280 g   0.6280 g 
Y =  _ Z  =  _ 3.667 x 10_ 3  = 2.155 x 10-3
 107.87 g   107.87 g 
2.155 x 10_ 3 mol
HCl molarity = = 0.021 55 M
0.1000 L
3.667 x 10_ 3 mol
HBr molarity = = 0.036 67 M
0.1000 L

4.102 (a) S4O62-(aq) → H2S(aq)


S4O62-(aq) → 4 H2S(aq)
S4O62-(aq) → 4 H2S(aq) + 6 H2O(l)
20 H+(aq) + S4O62-(aq) → 4 H2S(aq) + 6 H2O(l)
18 e- + 20 H+(aq) + S4O62-(aq) → 4 H2S(aq) + 6 H2O(l) (reduction half reaction)

Al(s) → Al3+(aq)
[Al(s) → Al3+(aq) + 3 e-] x 6 (oxidation half reaction)

Combine the two half reactions.


20 H+(aq) + S4O62-(aq) + 6 Al(s) → 4 H2S(aq) + 6 Al3+(aq) + 6 H2O(l)

(b) S2O32-(aq) → S4O62-(aq)


2 S2O32-(aq) → S4O62-(aq)
[2 S2O32-(aq) → S4O62-(aq) + 2 e-] x 3 (oxidation half reaction)

Cr2O72-(aq) → Cr3+(aq)
Cr2O72-(aq) → 2 Cr3+(aq)
Cr2O72-(aq) → 2 Cr3+(aq) + 7 H2O(l)
14 H+(aq) + Cr2O72-(aq) → 2 Cr3+(aq) + 7 H2O(l)
6 e- + 14 H+(aq) + Cr2O72-(aq) → 2 Cr3+(aq) + 7 H2O(l) (reduction half reaction)

Combine the two half reactions.

89
Chapter 4 - Reactions in Aqueous Solutions
______________________________________________________________________________

14 H+(aq) + 6 S2O32-(aq) + Cr2O72-(aq) → 3 S4O62-(aq) + 2 Cr3+(aq) + 7 H2O(l)

(c) ClO3-(aq) → Cl-(aq)


ClO3-(aq) → Cl-(aq) + 3 H2O(l)
6 H+(aq) + ClO3-(aq) → Cl-(aq) + 3 H2O(l)
[6 e- + 6 H+(aq) + ClO3-(aq) → Cl-(aq) + 3 H2O(l)] x 14 (reduction half reaction)

As2S3(s) → H2AsO4-(aq) + HSO4-(aq)


As2S3(s) → 2 H2AsO4-(aq) + 3 HSO4-(aq)
20 H2O(l) + As2S3(s) → 2 H2AsO4-(aq) + 3 HSO4-(aq)
20 H2O(l) + As2S3(s) → 2 H2AsO4-(aq) + 3 HSO4-(aq) + 33 H+(aq)

[20 H2O(l) + As2S3(s) → 2 H2AsO4-(aq) + 3 HSO4-(aq) + 33 H+(aq) + 28 e-] x 3


(oxidation half reaction)

Combine the two half reactions.


84 H+(aq) + 60 H2O(l) + 14 ClO3-(aq) + 3 As2S3(s) →
14 Cl-(aq) + 6 H2AsO4-(aq) + 9 HSO4-(aq) + 42 H2O(l) + 99 H+(aq)
18 H2O(l) + 14 ClO3-(aq) + 3 As2S3(s) →
14 Cl-(aq) + 6 H2AsO4-(aq) + 9 HSO4-(aq) + 15 H+(aq)

(d) IO3-(aq) → I-(aq)


IO3-(aq) → I-(aq) + 3 H2O(l)
6 H+(aq) + IO3-(aq) → I-(aq) + 3 H2O(l)
[6 e- + 6 H+(aq) + IO3-(aq) → I-(aq) + 3 H2O(l)] x 7 (reduction half reaction)

Re(s) → ReO4-(aq)
4 H2O(l) + Re(s) → ReO4-(aq)
4 H2O(l) + Re(s) → ReO4-(aq) + 8 H+(aq)
[4 H2O(l) + Re(s) → ReO4-(aq) + 8 H+(aq) + 7 e-] x 6 (oxidation half reaction)

Combine the two half reactions.


42 H+(aq) + 24 H2O(l) + 7 IO3-(aq) + 6 Re(s) →
7 I-(aq) + 6 ReO4-(aq) + 21 H2O(l) + 48 H+(aq)
3 H2O(l) + 7 IO3 (aq) + 6 Re(s) → 7 I-(aq) + 6 ReO4-(aq) + 6 H+(aq)
-

(e) HSO4-(aq) + Pb3O4(s) → PbSO4(s)


3 HSO4-(aq) + Pb3O4(s) → 3 PbSO4(s)
3 HSO4-(aq) + Pb3O4(s) → 3 PbSO4(s) + 4 H2O(l)
5 H+(aq) + 3 HSO4-(aq) + Pb3O4(s) → 3 PbSO4(s) + 4 H2O(l)
[2 e- + 5 H+(aq) + 3 HSO4-(aq) + Pb3O4(s) → 3 PbSO4(s) + 4 H2O(l)] x 10
(reduction half reaction)

As4(s) → H2AsO4-(aq)
As4(s) → 4 H2AsO4-(aq)

90
Chapter 4 - Reactions in Aqueous Solutions
______________________________________________________________________________

16 H2O(l) + As4(s) → 4 H2AsO4-(aq)


16 H2O(l) + As4(s) → 4 H2AsO4-(aq) + 24 H+(aq)
16 H2O(l) + As4(s) → 4 H2AsO4-(aq) + 24 H+(aq) + 20 e- (oxidation half reaction)

Combine the two half reactions.


26 H+(aq) + 30 HSO4-(aq) + As4(s) + 10 Pb3O4(s) →
4 H2AsO4-(aq) + 30 PbSO4(s) + 24 H2O(l)

(f) HNO2(aq) → NO3-(aq)


H2O(l) + HNO2(aq) → NO3-(aq)
H2O(l) + HNO2(aq) → NO3-(aq) + 3 H+(aq)
H2O(l) + HNO2(aq) → NO3-(aq) + 3 H+(aq) + 2 e- (oxidation half reaction)

HNO2(aq) → NO(g)
HNO2(aq) → NO(g) + H2O(l)
H+(aq) + HNO2(aq) → NO(g) + H2O(l)
[1 e- + H+(aq) + HNO2(aq) → NO(g) + H2O(l)] x 2 (reduction half reaction)

Combine the two half reactions.


3 HNO2(aq) → NO3-(aq) + 2 NO(g) + H2O(l) + H+(aq)

4.103 (a) C4H4O62-(aq) → CO32-(aq)


C4H4O62-(aq) → 4 CO32-(aq)
C4H4O62-(aq) + 6 H2O(l) → 4 CO32-(aq)

C4H4O62-(aq) + 6 H2O(l) → 4 CO32-(aq) + 16 H+(aq)


[C4H4O62-(aq) + 6 H2O(l) → 4 CO32-(aq) + 16 H+(aq) + 10 e-] x 3
(oxidation half reaction)

ClO3-(aq) → Cl-(aq)
ClO3-(aq) → Cl-(aq) + 3 H2O(l)
ClO3-(aq) + 6 H+(aq) → Cl-(aq) + 3 H2O(l)
[6 e- + ClO3-(aq) + 6 H+(aq) → Cl-(aq) + 3 H2O(l)] x 5 (reduction half reaction)

Combine the two half reactions.


3 C4H4O62-(aq) + 18 H2O(l) + 5 ClO3-(aq) + 30 H+(aq) →
12 CO32-(aq) + 48 H+(aq) + 5 Cl-(aq) + 15 H2O(l)
3 C4H4O62-(aq) + 3 H2O(l) + 5 ClO3-(aq) → 12 CO32-(aq) + 18 H+(aq) + 5 Cl-(aq)
3 C4H4O62-(aq) + 3 H2O(l) + 5 ClO3-(aq) + 18 OH-(aq) →
12 CO32-(aq) + 18 H+(aq) + 18 OH-(aq) + 5 Cl-(aq)
3 C4H4O62-(aq) + 3 H2O(l) + 5 ClO3-(aq) + 18 OH-(aq) →
12 CO32-(aq) + 18 H2O(aq) + 5 Cl-(aq)
3 C4H4O62-(aq) + 5 ClO3-(aq) + 18 OH-(aq) → 12 CO32-(aq) + 15 H2O(l) + 5 Cl-(aq)

91
Chapter 4 - Reactions in Aqueous Solutions
______________________________________________________________________________

(b) Al(s) → Al(OH)4-(aq)


Al(s) + 4 OH-(aq) → Al(OH)4-(aq)
[Al(s) + 4 OH-(aq) → Al(OH)4-(aq) + 3 e-] x 11 (oxidation half reaction)

BiONO3(s) → Bi(s) + NH3(aq)


BiONO3(s) → Bi(s) + NH3(aq) + 4 H2O(l)
BiONO3(s) + 11 H+(aq) → Bi(s) + NH3(aq) + 4 H2O(l)
[BiONO3(s) + 11 H+(aq) + 11 e- → Bi(s) + NH3(aq) + 4 H2O(l)] x 3
(reduction half reaction)

Combine the two half reactions.


11 Al(s) + 44 OH-(aq) + 3 BiONO3(s) + 33 H+(aq) →
11 Al(OH)4-(aq) + 3 Bi(s) + 3 NH3(aq) + 12 H2O(l)
11 Al(s) + 11 OH-(aq) + 3 BiONO3(s) + 33 H2O(l) →
11 Al(OH)4-(aq) + 3 Bi(s) + 3 NH3(aq) + 12 H2O(l)
11 Al(s) + 11 OH-(aq) + 3 BiONO3(s) + 21 H2O(l) →
11 Al(OH)4-(aq) + 3 Bi(s) + 3 NH3(aq)

(c) H2O2(aq) → O2(g)


H2O2(aq) → O2(g) + 2 H+(aq)
[H2O2(aq) → O2(g) + 2 H+(aq) + 2 e-] x 4 (oxidation half reaction)

Cl2O7(aq) → ClO2-(aq)
Cl2O7(aq) → 2 ClO2-(aq)
Cl2O7(aq) → 2 ClO2-(aq) + 3 H2O(l)
Cl2O7(aq) + 6 H+(aq) → 2 ClO2-(aq) + 3 H2O(l)
Cl2O7(aq) + 6 H+(aq) + 8 e- → 2 ClO2-(aq) + 3 H2O(l) (reduction half reaction)
Combine the two half reactions.
4 H2O2(aq) + Cl2O7(aq) + 6 H+(aq) → 4 O2(g) + 8 H+(aq) + 2 ClO2-(aq) + 3
H2O(l)
4 H2O2(aq) + Cl2O7(aq) → 4 O2(g) + 2 H+(aq) + 2 ClO2-(aq) + 3 H2O(l)
4 H2O2(aq) + Cl2O7(aq) + 2 OH-(aq) →
4 O2(g) + 2 H+(aq) + 2 OH-(aq) + 2 ClO2-(aq) + 3 H2O(l)
4 H2O2(aq) + Cl2O7(aq) + 2 OH-(aq) → 4 O2(g) + 2 ClO2-(aq) + 5 H2O(l)

(d) Tl2O3(s) → TlOH(s)


Tl2O3(s) → 2 TlOH(s)
Tl2O3(s) → 2 TlOH(s) + H2O(l)
Tl2O3(s) + 4 H+(aq) → 2 TlOH(s) + H2O(l)
Tl2O3(s) + 4 H+(aq) + 4 e- → 2 TlOH(s) + H2O(l) (reduction half reaction)
NH2OH(aq) → N2(g)
2 NH2OH(aq) → N2(g)
2 NH2OH(aq) → N2(g) + 2 H2O(l)
2 NH2OH(aq) → N2(g) + 2 H2O(l) + 2 H+(aq)

92
Chapter 4 - Reactions in Aqueous Solutions
______________________________________________________________________________

[2 NH2OH(aq) → N2(g) + 2 H2O(l) + 2 H+(aq) + 2 e-] x 2 (oxidation half reaction)

Combine the two half reactions.


Tl2O3(s) + 4 H+(aq) + 4 NH2OH(aq) → 2 TlOH(s) + 2 N2(g) + 5 H2O(l) + 4 H+(aq)
Tl2O3(s) + 4 NH2OH(aq) → 2 TlOH(s) + 2 N2(g) + 5 H2O(l)

(e) Cu(NH3)42+(aq) → Cu(s) + 4 NH3(aq)


Cu(NH3)42+(aq) + 2 e- → Cu(s) + 4 NH3(aq) (reduction half reaction)

S2O42-(aq) → SO32-(aq)
S2O42-(aq) → 2 SO32-(aq)
S2O42-(aq) + 2 H2O(l) → 2 SO32-(aq)
S2O42-(aq) + 2 H2O(l) → 2 SO32-(aq) + 4 H+(aq)
S2O42-(aq) + 2 H2O(l) → 2 SO32-(aq) + 4 H+(aq) + 2 e- (oxidation half reaction)

Combine the two half reactions.


Cu(NH3)42+(aq) + S2O42-(aq) + 2 H2O(l) → Cu(s) + 4 NH3(aq) + 2 SO32-(aq) + 4 H+(aq)
Cu(NH3)42+(aq) + S2O42-(aq) + 2 H2O(l) + 4 OH-(aq) →
Cu(s) + 4 NH3(aq) + 2 SO32-(aq) + 4 H+(aq) + 4 OH-(aq)
Cu(NH3)4 (aq) + S2O42-(aq) + 2 H2O(l) + 4 OH-(aq) →
2+

Cu(s) + 4 NH3(aq) + 2 SO32-(aq) + 4 H2O(l)


Cu(NH3)4 (aq) + S2O4 (aq) + 4 OH-(aq) →
2+ 2-

Cu(s) + 4 NH3(aq) + 2 SO32-(aq) + 2 H2O(l)

(f) Mn(OH)2(s) → MnO2(s)


Mn(OH)2(s) → MnO2(s) + 2 H+(aq)
[Mn(OH)2(s) → MnO2(s) + 2 H+(aq) + 2 e-] x 3 (oxidation half reaction)

MnO4-(aq) → MnO2(s)
MnO4-(aq) → MnO2(s) + 2 H2O(l)
MnO4-(aq) + 4 H+(aq) → MnO2(s) + 2 H2O(l)
[MnO4-(aq) + 4 H+(aq) + 3 e- → MnO2(s) + 2 H2O(l)] x 2 (reduction half reaction)

Combine the two half reactions.


3 Mn(OH)2(s) + 2 MnO4-(aq) + 8 H+(aq) → 5 MnO2(s) + 6 H+(aq) + 4 H2O(l)
3 Mn(OH)2(s) + 2 MnO4-(aq) + 2 H+(aq) → 5 MnO2(s) + 4 H2O(l)
3 Mn(OH)2(s) + 2 MnO4-(aq) + 2 H+(aq) + 2 OH-(aq) →
5 MnO2(s) + 4 H2O(l) + 2 OH-(aq)
3 Mn(OH)2(s) + 2 MnO4-(aq) + 2 H2O(l) → 5 MnO2(s) + 4 H2O(l) + 2 OH-(aq)
3 Mn(OH)2(s) + 2 MnO4-(aq) → 5 MnO2(s) + 2 H2O(l) + 2 OH-(aq)

4.104 CuO, 79.55 amu; Cu2O, 143.09 amu


Let X equal the mass of CuO and Y the mass of Cu2O in the 10.50 g mixture. Therefore,

93
Chapter 4 - Reactions in Aqueous Solutions
______________________________________________________________________________

X + Y = 10.50 g.
1 mol Cu
mol Cu = 8.66 g x = 0.1363 mol Cu
63.546 g Cu
mol CuO + 2 x mol Cu2O = 0.1363 mol Cu
1 mol CuO  1 mol Cu 2 O 
Xx + 2 x  Y x  = 0.1363 mol Cu
79.55 g CuO  143.09 g Cu 2 O 
Rearrange to get X = 10.50 g - Y and then substitute it into the equation above to solve
for Y.
1 mol CuO  1 mol Cu 2 O 
(10.50 g _ Y) x + 2 x  Y x  = 0.1363 mol Cu
79.55 g CuO  143.09 g Cu 2 O 
10.50 mol Y mol 2 Y mol
_ + = 0.1363 mol
79.55 79.55 g 143.09 g
Y mol 2 Y mol 10.50 mol
_ + = 0.1363 mol _ = 0.0043 mol
79.55 g 143.09 g 79.55
(_ Y mol)(143.09 g) + (2 Y mol)(79.55 g)
= 0.0043 mol
(79.55 g)(143.09 g)
16.01 Y mol 16.01 Y
= 0.0043 mol ; = 0.0043
11383 g 11383 g
Y = (0.0043)(11383 g)/16.01 = 3.06 g Cu2O

X = 10.50 g - Y = 10.50 g - 3.06 g = 7.44 g CuO

4.105 (a) PbI2, 461.01 amu


Pb(NO3)2(aq) + 2 KI(aq) → PbI2(s) + 2 KNO3(aq)
75.0 mL = 0.0750 L and 100.0 mL = 0.1000 L
mol Pb(NO3)2 = (0.0750 L)(0.100 mol/L) = 7.50 x 10-3 mol Pb(NO3)2
mol KI = (0.1000 L)(0.190 mol/L) = 1.90 x 10-2 mol KI
2 mol KI
mols KI needed = 7.50 x 10-3 mol Pb(NO3)2 x = 1.50 x 10-2 mol KI
1 mol Pb( NO3 )2
There is an excess of KI, so Pb(NO3)2 is the limiting reactant.
1 mol PbI2 461.01 g PbI2
mass PbI2 = 7.50 x 10-3 mol Pb(NO3)2 x x = 3.46 g PbI2
1 mol Pb( NO3 )2 1 mol PbI2

(b) Because Pb(NO3)2 is the limiting reactant, Pb2+ is totally consumed and [Pb2+] = 0.
mol K+ = mol KI = 1.90 x 10-2 mol
2 mol NO3_
mol NO3- = 7.50 x 10-3 mol Pb(NO3)2 x = 0.0150 mol NO3-
1 mol Pb( NO3 )2
mol I = (initial mol KI) - (mol KI needed) = 0.0190 mol - 0.0150 mol = 0.0040 mol I-
-

total volume = 0.0750 L + 0.1000 L = 0.1750 L

94
Chapter 4 - Reactions in Aqueous Solutions
______________________________________________________________________________

0.0190 mol
[K+] = = 0.109 M
0.1750 L
0.0150 mol
[NO3-] = = 0.0857 M
0.1750 L
0.0040 mol
[I-] = = 0.023 M
0.1750 L

Multi-Concept Problems

4.106 NaOH, 40.00 amu; Ba(OH)2, 171.34 amu


Let X equal the mass of NaOH and Y the mass of Ba(OH)2 in the 10.0 g mixture.
Therefore, X + Y = 10.0 g.
1L 1.50 mol HCl
mol HCl = 108.9 mL x x = 0.163 mol HCl
1000 mL 1L
mol NaOH + 2 x mol Ba(OH)2 = 0.163 mol HCl
1 mol NaOH  1 mol Ba(OH )2 
Xx + 2 x  Y x  = 0.163 mol HCl

40.00 g NaOH  171.34 g Ba(OH ) 2

Rearrange to get X = 10.0 g - Y and then substitute it into the equation above to solve for Y.
1 mol NaOH  1 mol Ba(OH )2 
(10.0 g _ Y) x + 2 x  Y x  = 0.163 mol HCl
40.00 g NaOH  171.34 g Ba(OH )2 
10.00 mol Y mol 2 Y mol
_ + = 0.163 mol
40.00 40.00 g 171.34 g
Y mol 2 Y mol 10.00 mol
_ + = 0.163 mol _ = -0.087 mol
40.00 g 171.34 g 40.00
(_ Y mol)(171.34 g) + (2 Y mol)(40.00 g)
= -0.087 mol
(40.00 g)(171.34 g)
_ 91.34 Y mol 91.34 Y
= _ 0.087 mol ; = 0.087
6853.6 g 6853.6 g
Y = (0.087)(6853.6 g)/91.34 = 6.5 g Ba(OH)2
X = 10.0 g - Y = 10.0 g - 6.5 g = 3.5 g NaOH

4.107 100.0 mL = 0.1000 L and 50.0 mL = 0.0500 L


mol Na2SO4 = (0.1000 L)(0.100 mol/L) = 0.0100 mol Na2SO4
mol SO42- = mol Na2SO4 = 0.0100 mol SO42-
2 mol Na +
mol Na+ = 0.0100 mol Na2SO4 x = 0.0200 mol Na+
1 mol Na 2 SO4
mol ZnCl2 = (0.0500 L)(0.300 mol/L) = 0.0150 mol ZnCl2
mol Zn2+ = mol ZnCl2 = 0.0150 mol Zn2+
2 mol Cl _
mol Cl- = 0.0150 mol ZnCl2 x = 0.0300 mol Cl-
1 mol ZnCl2
mol Ba(CN)2 = (0.1000 L)(0.200 mol/L) = 0.0200 mol Ba(CN)2

95
Chapter 4 - Reactions in Aqueous Solutions
______________________________________________________________________________

mol Ba2+ = mol Ba(CN)2 = 0.0200 mol Ba2+


2 mol CN _
mol CN- = 0.0200 mol Ba(CN)2 x = 0.0400 mol CN-
1 mol Ba(CN )2
The following two reactions will take place to form precipitates.
Zn2+(aq) + 2 CN-(aq) → Zn(CN)2(s)
Ba2+(aq) + SO42-(aq) → BaSO4(s)
2 mol CN _
For Zn2+, mol CN- needed = 0.0150 mol Zn2+ x 2+
= 0.0300 mol CN- needed
1 mol Zn
CN- is in excess, so Zn2+ is the limiting reactant and is totally consumed.
mol CN- remaining after reaction= 0.0400 mol - 0.0300 mol = 0.0100 mol CN-

For Ba2+, mol SO42- needed = mol Ba2+ = 0.0200 mol SO42- needed
Ba2+ is in excess, so SO42- is the limiting reactant and is totally consumed.
mol Ba2+ remaining after reaction = 0.0200 mol - 0.0100 mol = 0.0100 mol Ba2+
total volume = 0.1000 L + 0.0500 L + 0.1000 L = 0.2500 L
[Zn2+] = 0
[SO42-] = 0
0.0200 mol
[Na+] = = 0.0800 M
0.2500 L
0.0300 mol
[Cl-] = = 0.120 M
0.2500 L
0.0100 mol
[CN-] = = 0.0400 M
0.2500 L
0.0100 mol
[Ba2+] = = 0.0400 M
0.2500 L

4.108 KNO3, 101.10 amu; BaCl2, 208.24 amu; NaCl, 58.44 amu; BaSO4, 233.40 amu;
AgCl, 143.32 amu
(a) The two precipitates are BaSO4(s) and AgCl(s).
(b) H2SO4 only reacts with BaCl2.
H2SO4(aq) + BaCl2(aq) → BaSO4(s) + 2 HCl(aq)
Calculate the number of moles of BaCl2 in 100.0 g of the mixture.
1 mol BaSO4 1 mol BaCl2
mol BaCl2 = 67.3 g BaSO4 x x = 0.288 mol BaCl2
233.40 g BaSO4 1 mol BaSO4
Calculate mass and moles of BaCl2 in 250.0 g sample.
208.24 g BaCl2 250.0 g
mass BaCl2 = 0.288 mol BaCl2 x x = 150. g BaCl2
1 mol BaCl2 100.0 g
1 mol BaCl2
mol BaCl2 = 150. g BaCl2 x = 0.720 mol BaCl2
208.24 g BaCl2
AgNO3 reacts with both NaCl and BaCl2 in the remaining 150.0 g of the mixture.
3 AgNO3(aq) + NaCl(aq) + BaCl2(aq) → 3 AgCl(s) + NaNO3(aq) + Ba(NO3)2(aq)
Calculate the moles of AgCl that would have been produced from the 250.0 g mixture.

96
Chapter 4 - Reactions in Aqueous Solutions
______________________________________________________________________________

1 mol AgCl 250.0 g


mol AgCl = 197.6 g AgCl x x = 2.30 mol AgCl
143.32 g AgCl 150.0 g
mol AgCl = 2 x (mol BaCl2) + mol NaCl
Calculate the moles and mass of NaCl in the 250.0 g mixture.
2.30 mol AgCl = 2 x 0.720 mol BaCl2 + mol NaCl
mol NaCl = 2.30 mol - 2(0.720 mol) = 0.86 mol NaCl
58.44 g NaCl
mass NaCl = 0.86 mol NaCl x = 50. g NaCl
1 mol NaCl
Calculate the mass of KNO3 in the 250.0 g mixture.
total mass = mass BaCl2 + mass NaCl + mass KNO3
250.0 g = 150. g BaCl2 + 50. g NaCl + mass KNO3
mass KNO3 = 250.0 g - 150. g BaCl2 - 50. g NaCl = 50. g KNO3

4.109 100.0 mL = 0.1000 L; 50.0 mL = 0.0500 L; 250.0 mL = 0.2500 L


After step (2):
BaCl2(aq) + 2 AgNO3(aq) → AgCl(s) + Ba(NO3)2(aq)
mol BaCl2 = (0.1000 L)(0.100 mol/L) = 0.0100 mol BaCl2
mol Ba2+ = mol BaCl2 = 0.0100 mol Ba2+
2 mol Cl _
mol Cl- = 0.0100 mol BaCl2 x = 0.0200 mol Cl-
1 mol BaCl2
mol AgNO3 = (0.0500 L)(0.100 mol/L) = 0.00500 mol AgNO3
mol Ag+ = mol AgNO3 = 0.00500 mol Ag+
mol NO3- = mol AgNO3 = 0.00500 mol NO3-
0.00500 mol Ag+ requires only 0.00500 mol Cl-, so Ag+ is the limiting reactant and totally
consumed.
mol Cl- remaining after reaction = 0.0200 mol - 0.00500 mol = 0.0150 mol Cl-

After step (3):


Ba2+(aq) + H2SO4(aq) → BaSO4(s) + 2 H+(aq)
mol H2SO4 = (0.0500 L)(0.100 mol/L) = 0.00500 mol H2SO4
mol SO42- = mol H2SO4 = 0.00500 mol SO42-
2 mol H +
mol H+ = 0.00500 mol H2SO4 x = 0.0100 mol H+
1 mol H 2 SO4
0.0100 mol Ba2+ requires 0.0100 mol SO42-, so SO42- is the limiting reactant and is totally
consumed.
mol Ba2+ remaining after reaction = 0.0100 mol - 0.00500 mol = 0.00500 mol Ba2+

After step (4):


NH3(aq) + H+(aq) → NH4+(aq)
mol NH3 = (0.2500 L)(0.100 mol/L) = 0.0250 mol NH3
0.0250 mol NH3 requires 0.0250 mol H+, so H+ is the limiting reactant and is totally
consumed.
mol NH3 remaining after reaction = 0.0250 mol - 0.0100 mol = 0.0150 mol NH3
mol NH4+ = mol H+ before reaction = 0.0100 mol NH4+

97
Chapter 4 - Reactions in Aqueous Solutions
______________________________________________________________________________

total volume = 0.1000 L + 0.0500 L + 0.0500 L + 0.2500 L = 0.4500 L


0.00500 mol
[Ba2+] = = 0.0111 M
0.4500 L
0.0150 mol
[Cl-] = = 0.0333 M
0.4500 L
0.00500 mol
[NO3-] = = 0.0111 M
0.4500 L
0.0150 mol
[NH3] = = 0.0333 M
0.4500 L
0.0100 mol
[NH4+] = = 0.0222 M
0.4500 L

4.110 (a) Cr2+(aq) + Cr2O72-(aq) → Cr3+(aq)


[Cr2+(aq) → Cr3+(aq) + e-] x 6 (oxidation half reaction)

Cr2O72-(aq) → Cr3+(aq)
Cr2O72-(aq) → 2 Cr3+(aq)
Cr2O72-(aq) → 2 Cr3+(aq) + 7 H2O(l)
14 H+(aq) + Cr2O72-(aq) → 2 Cr3+(aq) + 7 H2O(l)
6 e- + 14 H+(aq) + Cr2O72-(aq) → 2 Cr3+(aq) + 7 H2O(l) (reduction half reaction)

Combine the two half reactions.


14 H+(aq) + Cr2O72-(aq) + 6 Cr2+(aq) → 8 Cr3+(aq) + 7 H2O(l)

(b) total volume = 100.0 ml + 20.0 mL = 120.0 mL = 0.1200 L


Initial moles:
mol Cr( NO3 )2
0.120 x 0.1000 L = 0.0120 mol Cr(NO3)2
1L
mol HNO3
0.500 x 0.1000 L = 0.0500 mol HNO3
1L
mol K 2 Cr 2 O7
0.250 x 0.0200 L = 0.005 00 mol K2Cr2O7
1L
Check for the limiting reactant. 0.0120 mol of Cr2+ requires (0.0120)/6 = 0.00200 mol
Cr2O72- and (14/6)(0.0120) = 0.0280 mol H+. Both are in excess of the required amounts,
so Cr2+ is the limiting reactant.

14 H+(aq) + Cr2O72-(aq) + 6 Cr2+(aq) → 8 Cr3+(aq) + 7 H2O(l)


Initial moles 0.0500 0.00500 0.0120 0
Change -14x -x -6x +8x
Because Cr2+ is the limiting reactant, 6x = 0.0120 and x = 0.00200
Final moles 0.0220 0.00300 0 0.00160

98
Chapter 4 - Reactions in Aqueous Solutions
______________________________________________________________________________

2 mol K +
mol K+ = 0.00500 mol K2Cr2O7 x = 0.0100 mol K+
1 mol K 2 Cr 2 O7
2 mol NO3_
mol NO3- = 0.0120 mol Cr(NO3)2 x
1 mol Cr( NO3 )2
1 mol NO3_
+ 0.0500 mol HNO3 x = 0.0740 mol NO3-
1 mol HNO3
mol H+ = 0.0220 mol; mol Cr2O7 = 0.00300 mol; mol Cr3+ = 0.01600 mol
2-

Check for charge neutrality.


Total moles of +charge = 0.0100 + 0.0220 + 3 x (0.01600) = 0.0800 mol +charge
Total moles of -charge = 0.0740 + 2 x (0.00300) = 0.0800 mol -charge
The charges balance and there is electrical neutrality in the solution after the reaction.

0.0100 mol K +
K+ molarity = = 0.0833 M
0.1200 L
0.0740 mol NO3_
NO3- molarity = = 0.617 M
0.1200 L
0.0220 mol H +
H+ molarity = = 0.183 M
0.1200 L
0.00300 mol Cr 2 O72 _
Cr2O72- molarity = = 0.0250 M
0.1200 L
0.0160 mol Cr3+
Cr3+ molarity = = 0.133 M
0.1200 L

4.111 (a) (1) I-(aq) → I3-(aq)


3 I-(aq) → I3-(aq)
3 I-(aq) → I3-(aq) + 2 e- (oxidation half reaction)

HNO2(aq) → NO(g)
HNO2(aq) → NO(g) + H2O(l)
H+(aq) + HNO2(aq) → NO(g) + H2O(l)
[e- + H+(aq) + HNO2(aq) → NO(g) + H2O(l)] x 2 (reduction half reaction)

Combine the two half reactions.


3 I-(aq) + 2 H+(aq) + 2 HNO2(aq) → I3-(aq) + 2 NO(g) + 2 H2O(l)

(2) S2O32-(aq) → S4O62-(aq)


2 S2O32-(aq) → S4O62-(aq)
2 S2O32-(aq) → S4O62-(aq) + 2 e- (oxidation half reaction)

I3-(aq) → I-(aq)

99
Chapter 4 - Reactions in Aqueous Solutions
______________________________________________________________________________

I3-(aq) → 3 I-(aq)
2 e- + I3-(aq) → 3 I-(aq) (reduction half reaction)

Combine the two half reactions.


2 S2O32-(aq) + I3-(aq) → S4O62-(aq) + 3 I-(aq)

(b) 18.77 mL = 0.018 77 L; NO2-, 46.01 amu


mol S2 O32 _
0.1500 x 0.018 77 L = 0.002 815 5 mol S2O32-
1L
1 mol I3_ 2 mol NO2_
mass NO2- = 0.002 815 5 mol S2O32- x x x
2 mol S2 O32 _ 1 mol I3_
46.01 g NO2_
_
= 0.1295 g NO2-
1 mol NO2
0.1295 g
mass % NO2- = x 100% = 4.412%
2.935 g

4.112 (a) (1) Cu(s) → Cu2+(aq)


[Cu(s) → Cu2+(aq) + 2 e-] x 3 (oxidation half reaction)

NO3-(aq) → NO(g)
NO3-(aq) → NO(g) + 2 H2O(l)
4 H+(aq) + NO3-(aq) → NO(g) + 2 H2O(l)
[3 e- + 4 H+(aq) + NO3-(aq) → NO(g) + 2 H2O(l)] x 2
(reduction half reaction
Combine the two half reactions.
3 Cu(s) + 8 H+(aq) + 2 NO3-(aq) → 3 Cu2+(aq) + 2 NO(g) + 4 H2O(l)
(2) Cu2+(aq) + SCN-(aq) → CuSCN(s)
[e- + Cu2+(aq) + SCN-(aq) → CuSCN(s)] x 2 (reduction half reaction)

HSO3-(aq) → HSO4-(aq)
H2O(l) + HSO3-(aq) → HSO4-(aq)
H2O(l) + HSO3-(aq) → HSO4-(aq) + 2 H+(aq)
H2O(l) + HSO3-(aq) → HSO4-(aq) + 2 H+(aq) + 2 e-
(oxidation half reaction)
Combine the two half reactions.
2 Cu2+(aq) + 2 SCN-(aq) + H2O(l) + HSO3-(aq) →
2 CuSCN(s) + HSO4-(aq) + 2 H+(aq)

(3) Cu+(aq) → Cu2+(aq)


[Cu+(aq) → Cu2+(aq) + e-] x 10 (oxidation half reaction)

IO3-(aq) → I2(aq)
2 IO3-(aq) → I2(aq)

100
Chapter 4 - Reactions in Aqueous Solutions
______________________________________________________________________________

2 IO3-(aq) → I2(aq) + 6 H2O(l)


12 H+(aq) + 2 IO3-(aq) → I2(aq) + 6 H2O(l)
10 e- + 12 H+(aq) + 2 IO3-(aq) → I2(aq) + 6 H2O(l)
(reduction half reaction)
Combine the two half reactions.
10 Cu+(aq) + 12 H+(aq) + 2 IO3-(aq) → 10 Cu2+(aq) + I2(aq) + 6 H2O(l)

(4) I2(aq) → I-(aq)


I2(aq) → 2 I-(aq)
2 e- + I2(aq) → 2 I-(aq) (reduction half reaction)

S2O32-(aq) → S4O62-(aq)
2 S2O32-(aq) → S4O62-(aq)
2 S2O32-(aq) → S4O62-(aq) + 2 e- (oxidation half reaction)

Combine the two half reactions.


I2(aq) + 2 S2O32-(aq) → 2 I-(aq) + S4O62-(aq)

(5) 2 ZnNH4PO4 → Zn2P2O7 + H2O + 2 NH3

(b) 10.82 mL = 0.01082 L


mol S2O32- = (0.1220 mol/L)(0.01082 L) = 0.00132 mol S2O32-
1 mol I2
mol I2 = 0.00132 mol S2O32- x 2_
= 6.60 x 10-4 mol I2
2 mol S2 O3
10 mol Cu +
mol Cu+ = 6.60 x 10-4 mol I2 x = 6.60 x 10-3 mol Cu+ (Cu)
1 mol I2
-3
g Cu = (6.60 x 10 mol)(63.546 g/mol) = 0.419 g Cu
0.419 g Cu
mass % Cu in brass = x 100% = 77.1% Cu
0.544 g brass
(c) Zn2P2O7, 304.72 amu
2 x 65.39 g
mass % Zn in Zn2P2O7 = x 100% = 42.92%
304.72 g
mass of Zn in Zn2P2O7 = (0.4292)(0.246 g) = 0.106 g Zn
0.106 g Zn
mass % Zn in brass = x 100% = 19.5% Zn
0.544 g brass

4.113 (a) BaSO4, 233.38 amu


1 mol BaSO4 1 mol S
mol S = 7.19 g BaSO4 x x = 0.0308 mol S
233.38 g BaSO4 1 mol BaSO4
0.0308 mol S
theoretical mol S = = 0.0337 mol S
0.913
(b) Assume n = 1:

101
Chapter 4 - Reactions in Aqueous Solutions
______________________________________________________________________________

5 mol Cl
mol Cl in MCl5 = 0.0337 mol S x = 0.168 mol Cl
1 mol S
35.453 g Cl
mass Cl = 0.168 mol Cl x = 5.97 g Cl
1 mol Cl
This is impossible because the initial mass of MCl5 was only 4.61 g.

Assume n = 2:
5 mol Cl
mol Cl in MCl5 = 0.0337 mol S x = 0.0842 mol Cl
2 mol S
35.453 g Cl
mass Cl = 0.0842 mol Cl x = 2.99 g Cl
1 mol Cl
mass M = 4.61g - 2.99 g = 1.62 g M
1 mol M
mol M = 0.0337 mol S x = 0.0168 mol
2 mol S
1.62 g
M molar mass = = 96.4 g/mol; M atomic mass = 96.4 amu
0.0168 mol
96.4 is reasonable and suggests that M is Mo

Assume n = 3:
5 mol Cl
mol Cl in MCl5 = 0.0337 mol S x = 0.0562 mol Cl
3 mol S
35.453 g Cl
mass Cl = 0.0562 mol Cl x = 1.99 g Cl
1 mol Cl
mass M = 4.61g - 1.99 g = 2.62 g M
1 mol M
mol M = 0.0337 mol S x = 0.0112 mol
3 mol S
2.62 g
M molar mass = = 234 g/mol; M atomic mass = 234 amu
0.0112 mol
234 is between Pa and U, which is highly unlikely for a lubricant.

Assume n = 4:
5 mol Cl
mol Cl in MCl5 = 0.0337 mol S x = 0.0421 mol Cl
4 mol S
35.453 g Cl
mass Cl = 0.0421 mol Cl x = 1.49 g Cl
1 mol Cl
mass M = 4.61g - 1.49 g = 3.12 g M
1 mol M
mol M = 0.0337 mol S x = 0.00842 mol
4 mol S
3.12 g
M molar mass = = 371 g/mol; M atomic mass = 371 amu
0.008 42 mol
No known elements have a mass as great as 371 amu.
(c) M is most likely Mo and the metal sulfide is MoS2.
(d) (1) 2 MoCl5(s) + 5 Na2S(s) → 2 MoS2(s) + S(l) + 10 NaCl(s)

102
Chapter 4 - Reactions in Aqueous Solutions
______________________________________________________________________________

(2) 2 MoS2(s) + 7 O2(g) → 2 MoO3(s) + 4 SO2(g)


(3) SO2(g) + 2 Fe3+(aq) + 2 H2O(l) → 2 Fe2+(aq) + SO42-(aq) + 4 H+(aq)
(4) SO42-(aq) + Ba2+(aq) → BaSO4(s)

103
Assignment 2. Questions from chapter 5 of McMurry and Fay
Question numbers are from the fourth edition.

Chapter 5. Periodicity and Atomic Structure

c 3.00 x 108 m/s


ν= = 8.43 x 1018 s 1 = 8.43 x 1018 Hz
S
5.1 Gamma ray =
λ 3.56 x 10 m
_11

c 3.00 x 108 m/s


Radar wave ν= = = 2.91 x 109 sS1 = 2.91 x 109 Hz
λ 10.3 x 10 _ 2 m

5.2 v = 102.5 MHz = 102.5 x 106 Hz = 102.5 x 106 sS1


c 3.00 x 108 m / s
λ= = = 2.93 m
ν 102.5 x 106 s _1
v = 9.55 x 1017 Hz = 9.55 x 1017 s 1
S

c 3.00 x 108 m / s
λ= = = 3.14 x 10 _10 m
ν 9.55 x 1017 s _1

5.3 The wave with the shorter wavelength (b) has the higher frequency. The wave with the
larger amplitude (b) represents the more intense beam of light. The wave with the shorter
wavelength (b) represents blue light. The wave with the longer wavelength (a) represents
red light.

Balmer series: m = 2; R = 1.097 x 10 2 nm 1


S S
5.4
1  1 1 1 1 1 1
= 2.519 x 10 3 nm 1;
S S
= R  2 _ 2 ; = R  2 _ 2 ; λ = 397.0 nm
λ m n  λ 2 7  λ

Paschen series: m = 3; R = 1.097 x 10 2 nm 1


S S
5.5
1  1 1 1 1 1 1
= 5.333 x 10 4 nm 1;
S S
= R  2 _ 2; = R  2 _ 2 ; λ = 1875 nm
λ m n  λ 3 4  λ

Paschen series: m = 3; R = 1.097 x 10 2 nm 1


S S
5.6
1  1 1 1 1 1  1
= R  2 _ 2; = R  2 _ 2 ; = 1.219 x 10S3 nmS1; λ = 820.4 nm
λ m n  λ 3 ∞  λ

5.7 λ = 91.2 nm = 91.2 x 10S9 m


c 3.00 x 108 m/s
ν= = = 3.29 x 1015 sS1
λ 91.2 x 10 _ 9 m
E = hv = (6.626 x 10S34 J≅s)(3.29 x 1015 sS1) = 2.18 x 10S18 J/photon
E = (2.18 x 10S18 J/photon)(6.022 x 1023 photons/mol) = 1.31 x 106 J/mol = 1310 kJ/mol

λ = 1.55 x 10 6 m
S
5.8 IR,
c  3.00 x 108 m / s 
E = h = (6.626 x 10 _ 34 J • s)  23
 (6.022 x 10 / mol)
λ  1.55 x 10
_6
m 

103
Chapter 5 S Periodicity and Atomic Structure
______________________________________________________________________________

E = 7.72 x 104 J/mol = 77.2 kJ/mol

UV, λ = 250 nm = 250 x 10S9 m


c  3.00 x 108 m / s 
E = h = (6.626 x 10 _ 34 J • s)   (6.022 x 10 23 / mol)
λ _9
 250 x 10 m 
5
E = 4.79 x 10 J/mol = 479 kJ/mol

X ray, λ = 5.49 nm = 5.49 x 10 9 m


S

c  3.00 x 108 m / s 
E = h = (6.626 x 10 _ 34 J • s)   (6.022 x 10 23 / mol)
λ _9
 5.49 x 10 m 
E = 2.18 x 10 J/mol = 2.18 x 104 kJ/mol
7

h 6.626 x 10 _ 34 kg m 2 s _1
5.9 λ= = = 2.34 x 10S38 m
mv (1150 kg)(24.6 m/s)

h
5.10 (∆x)(∆mv) ≥ ; uncertainty in velocity = (45 m/s)(0.02) = 0.9 m/s
4 π
h 6.626 x 10 _ 34 kg m 2 s _1
∆ x≥ = = 5 x 10 _ 34 m
4 π(∆ mv) 4 π(0.120 kg)(0.9 m/s)

5.11 n l ml Orbital No. of Orbitals


5 0 0 5s 1
1 S1, 0, +1 5p 3
2 S2, S1, 0, +1, +2 5d 5
3 S3, S2, S1, 0, +1, +2, +3 5f 7
4 S4, S3, S2, S1, 0, +1, +2, +3, +4 5g 9
There are 25 possible orbitals in the fifth shell.

5.12 (a) 2p (b) 4f (c) 3d

5.13 (a) 3s orbital: n = 3, l = 0, ml = 0


(b) 2p orbital: n = 2, l = 1, ml = S1, 0, +1
(c) 4d orbital: n = 4, l = 2, ml = S2, S1, 0, +1, +2

5.14 The g orbitals have four nodal planes.

5.15 The figure represents a d orbital, n = 4 and l = 2.

m = 1, n = 4; R = 1.097 x 10
S2
nm 1
S
5.16
1  1 1 1 1 1  1 1  S2 S1

λ
= R  2 _ 2;
λ
=R  2 _ 2  ; λ = R 1  = 1.097 x 10 nm ; λ = 91.2 nm
m n  1 ∞   

104
Chapter 5 S Periodicity and Atomic Structure
______________________________________________________________________________

 3.00 x 108 m/s 


E = (6.626 x 10 _ 34 J• s)  _9
23
 (6.022 x 10 / mol)
 91.2 x 10 m 
6 3
E = 1.31 x 10 J/mol = 1.31 x 10 kJ/mol

5.17 (a) Ti, 1s2 2s2 2p6 3s2 3p6 4s2 3d2 or [Ar] 4s2 3d2
[Ar] __ __ __
4s 3d

(b) Zn, 1s2 2s2 2p6 3s2 3p6 4s2 3d10 or [Ar] 4s2 3d10
[Ar]
4s 3d

(c) Sn, 1s2 2s2 2p6 3s2 3p6 4s2 3d10 4p6 5s2 4d10 5p2 or [Kr] 5s2 4d10 5p2
[Kr]
5s 4d 5p

(d) Pb,[Xe] 6s2 4f 14 5d10 6p2

For Na+, 1s2 2s2 2p6; for Cl , 1s2 2s2 2p6 3s2 3p6
S
5.18

5.19 The ground-state electron configuration contains 28 electrons. The atom is Ni.

5.20 Cr, Cu, Nb, Mo, Ru, Rh, Pd, Ag, La, Ce, Gd, Pt, Au, Ac, Th, Pa, U, Np, Cm

5.21 (a) Ba; atoms get larger as you go down a group.


(b) W; atoms get smaller as you go across a period.
(c) Sn; atoms get larger as you go down a group.
(d) Ce; atoms get smaller as you go across a period.

5.22 The aurora borealis begins on the surface of the sun with a massive solar flare. These
flares eject a solar "gas" of energetic protons and electrons that reach earth after about 2
days and are then attracted toward the north and south magnetic poles. The energetic
electrons are deflected by the earth's magnetic field into a series of sheetlike beams. The
electrons then collide with O2 and N2 molecules in the upper atmosphere, exciting them,
ionizing them, and breaking them apart into O and N atoms. The energetically excited
atoms, ions, and molecules generated by collisions with electrons emit energy of
characteristic wavelengths when they decay to their ground states. The O2+ ions emit a red
light around 630 nm; N2+ ions emit violet and blue light at 391.4 nm and 470.0 nm; and O
atoms emit a greenish-yellow light at 557.7 nm and a deep red light at 630.0 nm.

Understanding Key Concepts

105
Chapter 5 S Periodicity and Atomic Structure
______________________________________________________________________________

5.23

5.24

5.25 The wave with the larger amplitude (a) has the greater intensity. The wave with the
shorter wavelength (a) has the higher energy radiation. The wave with the shorter
wavelength (a) represents yellow light. The wave with the longer wavelength (b)
represents infrared radiation.

5.26 [Ar] 4s2 3d10 4p1 is Ga.

5.27 There are 34 total electrons in the atom, so there are also 34 protons in the nucleus. The
atom is selenium (Se)
Se, [Ar]
4s 3d 4p

5.28 Ca and Br are in the same period, with Br to the far right of Ca. Ca is larger than Br. Sr
is directly below Ca in the same group, and is larger than Ca. The result is
Sr (215 pm) > Ca (197 pm) > Br (114 pm)

5.29 (a) 3py n = 3, l = 1 (b) 4d z 2 n = 4, l = 2


Additional Problems
Electromagnetic Radiation

106
Chapter 5 S Periodicity and Atomic Structure
______________________________________________________________________________

5.30 Violet has the higher frequency and energy. Red has the higher wavelength.

5.31 Ultraviolet light has the higher frequency and the higher energy. Infrared light has the
higher wavelength.

c 3.00 x 108 m/s


λ= = 5.5 x 10 8 m
S
5.32 =
ν 5.5 x 10 s
15 _1

c 3.00 x 108 m/s


5.33 λ= = = 6.93 x 1010 s _1 = 6.93 x 1010 Hz
ν 4.33 x 10 m
_3

5.34 (a) v = 99.5 MHz = 99.5 x 106 sS1


E = hv = (6.626 x 10 34 J≅s)(99.5 x 106 s 1)(6.022 x 1023 /mol)
S S

E = 3.97 Η 10 2 J/mol = 3.97 Η 10 5 kJ/mol


S S

3 S1
v = 1150 kHz = 1150 x 10 s
E = hv = (6.626 x 10S34 J≅s)(1150 x 103 sS1)(6.022 x 1023 /mol)
E = 4.589 x 10 4 J/mol = 4.589 x 10 7 kJ/mol
S S

The FM radio wave (99.5 MHz) has the higher energy.


(b) λ = 3.44 x 10 9 m
S

c  3.00 x 108 m / s 
E = h = (6.626 x 10 _ 34 J• s)   (6.022 x 10 23 / mol)
λ _9
 3.44 x 10 m 
E = 3.48 x 10 J/mol = 3.48 x 104 kJ/mol
7

λ = 6.71 Η 10S2 m
c  3.00 x 108 m / s 
E = h = (6.626 x 10 _ 34 J• s)  23
 (6.022 x 10 / mol)
λ  6.71 x 10
_2
m 
S3
E = 1.78 J/mol = 1.78 x 10 kJ/mol
The X ray (λ = 3.44 x 10 9 m) has the higher energy.
S

v = 400 MHz = 400 x 106 s 1


S
5.35
E = (6.626 x 10S34 J≅s)(400 x 106 sS1)(6.02 x 1023/mol) = 0.160 J/mol = 1.60 x 10S4
kJ/mol

1000 J 1 mol
5.36 (a) E = 90.5 kJ/mol x x 23
= 1.50 x 10S19 J
1 kJ 6.02 x 10
_19
E 1.50 x 10 J
ν= = = 2.27 x 1014 s 1
S

h 6.626 x 10 J• s
_ 34

c 3.00 x 108 m/s


λ= = = 1.32 x 10S6 m = 1320 x 10S9 m = 1320 nm, near IR
ν 2.27 x 10 s
14 _1

1000 J 1 mol
(b) E = 8.05 x 10 4 kJ/mol x = 1.34 x 10 24 J
S S
x 23
1 kJ 6.02 x 10

107
Chapter 5 S Periodicity and Atomic Structure
______________________________________________________________________________

E 1.34 x 10 _ 24 J
ν= = 2.02 x 109 s 1
S
=
h 6.626 x 10 J• s
_ 34

c 3.00 x 108 m/s


λ= = = 0.149 m, radio wave
ν 2.02 x 109 s _1

1000 J 1 mol
(c) E = 1.83 x 103 kJ/mol x = 3.04 x 10 18 J
S
x 23
1 kJ 6.02 x 10
E 3.04 x 10 _18 J
ν= = 4.59 x 1015 s 1
S
=
h 6.626 x 10 J• s
_ 34

c 3.00 x 108 m/s


λ= = = 6.54 x 10 8 m = 65.4 x 10 9 m = 65.4 nm, UV
S S

ν 4.59 x 1015 s _1

 1 kJ 
5.37 (a) E = h ν = (6.626 x 10 _ 34 J• s)(5.97 x 1019 s _1)  23
 (6.022 x 10 / mol)
 1000 J 
E = 2.38 x 107 kJ/mol
 1 kJ 
(b) E = h ν = (6.626 x 10 _ 34 Jcdot s)(1.26 x 106 s _1)  23
 (6.022 x 10 / mol)
 1000 J 
S7
E = 5.03 x 10 kJ/mol
 3.00 x 108 m / s   1 kJ 
(c) E = h ν = (6.626 x 10 _ 34 J• s)   
23
 (6.022 x 10 / mol)
 2.57 x 10 m   1000 J 
2

S7
E = 4.66 x 10 kJ/mol

h 6.626 x 10 _ 34 kg m 2 s _1
λ= = 2.45 x 10 12 m, γ ray
S
5.38 = _ 31 8
mv (9.11 x 10 kg)(0.99 x 3.00 x 10 m/s)

h 6.626 x 10 _ 34 kg m 2 s _1
5.39 λ= = = 5.28 x 10S15 m, γ ray
mv (1.673 x 10 _ 27 kg)(0.25 x 3.00 x 108 m/s)

5.40 156 km/h = 156 x 103 m/3600 s = 43.3 m/s; 145 g = 0.145 kg
h 6.626 x 10 _ 34 kg m 2 s _1
λ= = = 1.06 x 10 _ 34 m
mv (0.145 kg)(43.3 m/s)

5.41 1.55 mg = 1.55 x 10S3 g = 1.55 x 10S6 kg


h 6.626 x 10 _ 34 kg m 2 s _1
λ= = _6
= 3.10 x 10S28 m
mv (1.55 x 10 kg)(1.38 m/s)
145 g = 0.145 kg; 0.500 nm = 0.500 x 10 9 m
S
5.42

108
Chapter 5 S Periodicity and Atomic Structure
______________________________________________________________________________

h 6.626 x 10 _ 34 kg m 2 s _1
v= = = 9.14 x 10 _ 24 m /s
m λ (0.145 kg)(0.500 x 10 _ 9 m)

750 nm = 750 x 10 9 m
S
5.43
h 6.626 x 10 _ 34 kg m 2 s _1
v= = = 970 m/s
m λ (9.11 x 10 _ 31 kg) (750 x 10 _ 9 m)

Atomic Spectra

For n = 3; λ = 656.3 nm = 656.3 x 10 9 m


S
5.44
c  2.998 x 108 m / s   1 kJ 
E = h = (6.626 x 10 _ 34 Jcdot s)   
23
 (6.022 x 10 / mol)
λ  656.3 x 10 m   1000 J 
_9

E = 182.3 kJ / mol

For n = 4; λ = 486.1 nm = 486.1 x 10S9 m


c  2.998 x 108 m / s   1 kJ 
E = h = (6.626 x 10 _ 34 Jcdot s)   
23
 (6.022 x 10 / mol)
λ  486.1 x 10 m   1000 J 
_9

E = 246.1 kJ / mol

For n = 5; λ = 434.0 nm = 434.0 x 10 9 m


S

c  2.998 x 108 m / s   1 kJ 
E = h = (6.626 x 10 _ 34 Jcdot s)   
23
 (6.022 x 10 / mol)
λ  434.0 x 10 m   1000 J 
_9

E = 275.6 kJ / mol

m = 2, n = 4; R = 1.097 x 10
S2
nm 1
S
5.45
1  1 1 1 1 1  1 1 S3 S1

λ
= R  2 _ 2;
λ
=R  2 _ 2  ; λ = R  4  = 2.74 x 10 nm ; λ = 364.6 nm
m n  2 ∞   

From problem 5.45, for n = 4, λ = 364.6 nm = 364.6 x 10 9 m


S
5.46
c  2.998 x 108 m / s   1 kJ 
E = h = (6.626 x 10 _ 34 Jcdot s)  23
  1000 J  (6.022 x 10 / mol)
λ  364.6 x 10
_9
m  
E = 328.1 kJ / mol

5.47 Brackett series: m = 4, n = 5; R = 1.097 x 10S2 nmS1


1  1 1 1 1 1
= R  2 _ 2  = 2.468 x 10 4 nm 1; λ = 4051 nm
S S
= R  2 _ 2;
λ m n  λ 4 5 
c  2.998 x 108 m / s   1 kJ 
E = h = (6.626 x 10 _ 34 Jcdot s)  23
  1000 J  (6.022 x 10 / mol)
λ  4051 x 10
_9
m   
E = 29.55 kJ / mol, IR
Brackett series: m = 4, n = 6; R = 1.097 x 10S2 nmS1

109
Chapter 5 S Periodicity and Atomic Structure
______________________________________________________________________________

1  1 1 1 1 1
= R  2 _ 2  = 3.809 x 10 4 nm 1; λ = 2625 nm
S S
= R  2 _ 2;
λ m n  λ 4 6 
c  2.998 x 108 m / s   1 kJ 
E = h = (6.626 x 10 _ 34 Jcdot s)  23
  1000 J  (6.022 x 10 / mol)
λ  2625 x 10
_9
m  
E = 45.60 kJ / mol, IR

5.48 λ = 330 nm = 330 x 10S9 m


c  3.00 x 108 m / s   1 kJ 
E = h = (6.626 x 10 _ 34 Jcdot s)  23
  1000 J  (6.022 x 10 / mol)
λ  330 x 10
_9
m  
E = 363 kJ / mol

795 nm = 795 x 10 9 m
S
5.49
c  3.00 x 108 m / s   1 kJ 
E = h = (6.626 x 10 _ 34 Jcdot s)  23
  1000 J  (6.022 x 10 / mol)
λ  795 x 10
_9
m  
E = 151 kJ / mol

Orbitals and Quantum Numbers

5.50 n is the principal quantum number. The size and energy level of an orbital depends on n.
l is the angular-momentum quantum number. l defines the three-dimensional shape of an
orbital.
ml is the magnetic quantum number. ml defines the spatial orientation of an orbital.
ms is the spin quantum number. ms indicates the spin of the electron and can have either
of two values, +2 or S2.

5.51 The Heisenberg uncertainty principle states that one can never know both the position and
the velocity of an electron beyond a certain level of precision. This means we cannot
think of electrons circling the nucleus in specific orbital paths, but we can think of
electrons as being found in certain three-dimensional regions of space around the nucleus,
called orbitals.

5.52 The probability of finding the electron drops off rapidly as distance from the nucleus
increases, although it never drops to zero, even at large distances. As a result, there is no
definite boundary or size for an orbital. However, we usually imagine the boundary
surface of an orbital enclosing the volume where an electron spends 95% of its time.

5.53 A 4s orbital has three nodal surfaces.

110
Chapter 5 S Periodicity and Atomic Structure
______________________________________________________________________________

4s orbital

5.54 Part of the electron-nucleus attraction is canceled by the electron-electron repulsion, an


effect we describe by saying that the electrons are shielded from the nucleus by the other
electrons. The net nuclear charge actually felt by an electron is called the effective
nuclear charge, Zeff, and is often substantially lower than the actual nuclear charge, Zactual.
Zeff = Zactual S electron shielding

5.55 Electron shielding gives rise to energy differences among 3s, 3p, and 3d orbitals in
multielectron atoms because of the differences in orbital shape. For example, the 3s
orbital is spherical and has a large probability density near the nucleus, while the 3p
orbital is dumbbell shaped with a node at the nucleus. An electron in a 3s orbital can
penetrate closer to the nucleus than an electron in a 3p orbital can and feels less of a
shielding effect from other electrons. Generally, for any given value of the principal
quantum number n, a lower value of l corresponds to a higher value of Zeff and to a lower
energy for the orbital.

5.56 (a) 4s n = 4; l = 0; ml = 0; ms = ∀2
(b) 3p n = 3; l = 1; ml = S1, 0, +1; ms = ∀2
(c) 5f n = 5; l = 3; ml = S3, S2, S1, 0, +1, +2, +3; ms = ∀2
(d) 5d n = 5; l = 2; ml = S2, S1, 0, +1, +2; ms = ∀2

5.57 (a) 3s (b) 2p (c) 4f (d) 4d

5.58 (a) is not allowed because for l = 0, ml = 0 only.


(b) is allowed.
(c) is not allowed because for n = 4, l = 0, 1, 2, or 3 only.

5.59 Co 1s2 2s2 2p6 3s2 3p6 4s2 3d7


(a) is not allowed because for l = 0, ml = 0 only.
(b) is not allowed because n = 4 and l = 2 is for a 4d orbital.
(c) is allowed because n = 3 and l = 1 is for a 3p orbital.

5.60 For n = 5, the maximum number of electrons will occur when the 5g orbital is filled:
[Rn] 7s2 5f 14 6d10 7p6 8s2 5g18 = 138 electrons

5.61 n = 4, l = 0 is a 4s orbital. The electron configuration is 1s2 2s2 2p6 3s2 3p6 4s2. The
number of electrons is 20.

0.68 g = 0.68 x 10 3 kg
S
5.62
h h 6.626 x 10 _ 34 kg m 2 s _1
(∆ x)(∆ mv) ≥ ; ∆x≥ = = 8 x 10 _ 31 m
4 π 4 π (∆ mv) 4 π (0.68 x 10 _ 3 kg)(0.1 m / s)

111
Chapter 5 S Periodicity and Atomic Structure
______________________________________________________________________________

1.660 540 x 10 _ 27 kg h
= 6.6465 x 10 27 kg; (∆ x)(∆ mv) ≥
S
5.63 4.0026 amu x
1 amu 4 π
_ 34 2 _1
h 6.626 x 10 kg m s
∆x≥ = 5.833 x 10 10 m
S
=
4 π (∆ mv) 4 π (6.6465 x 10 kg)(0.01 x 1.36 x 10 m / s)
_ 27 3

Electron Configurations

5.64 The number of elements in successive periods of the periodic table increases by the
progression 2, 8, 18, 32 because the principal quantum number n increases by 1 from one
period to the next. As the principal quantum number increases, the number of orbitals in
a shell increases. The progression of elements parallels the number of electrons in a
particular shell.

5.65 The n and l quantum numbers determine the energy level of an orbital in a multielectron
atom.

5.66 (a) 5d (b) 4s (c) 6s

5.67 (a) 2p < 3p < 5s < 4d (b) 2s < 4s < 3d < 4p (c) 3d < 4p < 5p < 6s

5.68 (a) 3d after 4s (b) 4p after 3d(c) 6d after 5f (d) 6s after 5p

5.69 (a) 3s before 3p (b) 3d before 4p (c) 6s before 4f (d) 4f before 5d

5.70 (a) Ti, Z = 22 1s2 2s2 2p6 3s2 3p6 4s2 3d2
(b) Ru, Z = 44 1s2 2s2 2p6 3s2 3p6 4s2 3d10 4p6 5s2 4d6
(c) Sn, Z = 50 1s2 2s2 2p6 3s2 3p6 4s2 3d10 4p6 5s2 4d10 5p2
(d) Sr, Z = 38 1s 2s 2p6 3s2 3p6 4s2 3d10 4p6 5s2
2 2

(e) Se, Z = 34 1s2 2s2 2p6 3s2 3p6 4s2 3d10 4p4

5.71 (a) Z = 55, Cs [Kr] 5s2 4d10 5p6 6s1 (b) Z = 40, Zr [Kr] 5s2 4d2
2 14 10
(c) Z = 80, Hg[Xe] 6s 4f 5d (d) Z = 62, Sm [Xe] 6s 4f 6
2

5.72 (a) Rb, Z = 37 [Kr]


5s
(b) W, Z = 74 [Xe]

6s 4f 5d
(c) Ge, Z = 32 [Ar]
4s 3d 4p
(d) Zr, Z = 40 [Kr]
5s 4d

5.73 (a) Z = 25, Mn [Ar]


4s 3d
(b) Z = 56, Ba [Xe]

112
Chapter 5 S Periodicity and Atomic Structure
______________________________________________________________________________

6s

(c) Z = 28, Ni [Ar]


4s 3d
(d) Z = 47, Ag [Kr]
5s 4d

5.74 4s > 4d > 4f

5.75 K < Ca < Se < Kr

5.76 Z = 116 [Rn] 7s2 5f 14 6d10 7p4

5.77 Z = 119 [Rn] 7s2 5f 14 6d10 7p6 8s1

1s2 2s2 2p4


S
5.78 (a) O 2 unpaired e
2p
2 2 6 2 2
(b) Si 1s 2s 2p 3s 3p 2 unpaired eS
3p
1 S
(c) K [Ar] 4s 1 unpaired e

[Ar] 4s2 3d10 4p3


S
(d) As 3 unpaired e
4p

5.79 (a) Z = 31, Ga (b) Z = 46, Pd

5.80 Order of orbital filling:


1s2s2p3s3p4s3d4p5s4d5p6s4f5d6p7s5f6d7p8s5g
Z = 121

5.81 A g orbital would begin filling at atomic number = 121 (see 5.80). There are nine g
orbitals that can each hold two electrons. The first element to have a filled g orbital
would be atomic number = 138.

Atomic Radii and Periodic Properties

5.82 Atomic radii increase down a group because the electron shells are farther away from the
nucleus.

5.83 Across a period, the effective nuclear charge increases, causing a decrease in atomic radii.

5.84 F < O < S

5.85 (a) K, lower in group 1A (b) Ta, lower in group 5B


(c) V, farther to the left in same period

113
Chapter 5 S Periodicity and Atomic Structure
______________________________________________________________________________

(d) Ba, four periods lower and only one group to the right

5.86 Mg has a higher ionization energy than Na because Mg has a higher Zeff and a smaller
size.

5.87 F has a higher electron affinity than C because of a higher effective nuclear charge and
S
room in the valence shell for the additional electron. In addition, F achieves a noble gas
electron configuration.

General Problems

Balmer series: m = 2; R = 1.097 x 10 2 nm 1


S S
5.88
1  1 1 1 1 1
= R  2 _ 2  = 2.438 x 10 3 nm 1
S S
= R  2 _ 2;
λ m n  λ 2 6 
S9
λ = 410.2 nm = 410.2 x 10 m
c  2.998 x 108 m / s   1 kJ 
E = h = (6.626 x 10 _ 34 Jcdot s)  23
  1000 J  (6.022 x 10 / mol)
λ  410.2 x 10
_9
m   
E = 291.6 kJ / mol

Pfund series: m = 5; R = 1.097 x 10 2 nm


S S1
5.89
1  1 1
= R  2 _ 2
λ m n 
1 1 1
= R  2 _ 2  = 1.341 x 10 4 nm 1; λ = 7458 nm = 7458 x 10 9 m
S S S
n = 6,
λ 5 6 
c  2.998 x 108 m / s   1 kJ 
E = h = (6.626 x 10 _ 34 Jcdot s)  23
  1000 J  (6.022 x 10 / mol)
λ  7458 x 10
_9
m   
E = 16.04 kJ / mol
1 1 1
= R  2 _ 2  = 2.149 x 10 4 nm 1; λ = 4653 nm = 4653 x 10 9 m
S S S
n = 7,
λ 5 7 
c  2.998 x 108 m / s   1 kJ 
E = h = (6.626 x 10 _ 34 Jcdot s)  
23
 (6.022 x 10 / mol)
λ  4653 x 10 m   1000 J 
_9

E = 25.71 kJ / mol
These lines in the Pfund series are in the infrared region of the electromagnetic spectrum.

5.90 Pfund series: m = 5, n = 4; R = 1.097 x 10S2 nmS1


1 1 1  1
= R  2 _ 2  = R   = 4.388 x 10 4 nm 1; λ = 2279 nm
S S

λ 5 ∞   25 

 kJ   1000 J   1 mol 
5.91 (a) E = 142   23 
= 2.36 x 10•19 J
 mol   1 kJ  6.02 x 10 

114
Chapter 5 S Periodicity and Atomic Structure
______________________________________________________________________________

c hc (6.626 x 10•34 J• s)(3.00 x 108 m / s)


E=h , λ= =
λ E 2.36 x 10•19 J
λ = 8.42 x 10 7 m
S
(infrared)

 kJ  1000 J   1 mol 
(b) E =  4.55 x 10•2   23 
= 7.56 x 10•23 J
 mol  1 kJ  6.02 x 10 
c hc (6.626 x 10•34 J• s)(3.00 x 108 m / s)
E=h , λ= =
λ E 7.56 x 10•23 J
S3
λ = 2.63 x 10 m (microwave)
 kJ  1000 J   1 mol 
(c) E =  4.81 x 10 4   = 7.99 x 10•17 J
 mol  1 kJ  6.02 x 10 23 
c hc (6.626 x 10•34 J• s)(3.00 x 108 m / s)
E=h , λ= =
λ E 7.99 x 10•17 J
λ = 2.49 x 10S9 m (X ray)

 1 kJ 
5.92 (a) E = hv = (6.626 x 10S34 J≅s)(3.79 x 1011 sS1)  23
 (6.022 x 10 /mol) = 0.151 kJ/mol
 1000 J 

 1 kJ 
J≅s)(5.45 x 104 s 1) 
S34 S 23 S8
(b) E = hv = (6.626 x 10  (6.022 x 10 /mol) = 2.17 x 10
 1000 J 
kJ/mol

 3.00 x 108 m/s  1 kJ 


(c) E = hv = (6.626 x 10S34 J≅s)  
23
 (6.022 x 10 /mol) = 2.91 kJ/mol
 4.11 x 10 m  1000 J 
_5

v = 9,192,631,770 s 1 = 9.19263 x 109 s 1


S S
5.93
 1 kJ 
E = hv = (6.626 x 10S34 J≅s)(9.19263 x 109 sS1)  23
 (6.022 x 10 /mol) = 3.668 x 10
S3

 1000 J 
kJ/mol

5.94 (a) Ra [Rn] 7s2 [Rn]


7s
(b) Sc [Ar] 4s2 3d1 [Ar]
4s 3d
(c) Lr [Rn] 7s2 5f14 6d1 [Rn]

7s 5f 6d
2 1
(d) B [He] 2s 2p [He]
2s 2p
2 10 4
(e) Te [Kr] 5s 4d 5p [Kr]
5s 4d 5p

115
Chapter 5 S Periodicity and Atomic Structure
______________________________________________________________________________

5.95 (a) row 1 n = 1, l = 0 1s 2 elements


l=1 1p 6 elements
l=2 1d 10 elements

row 2 n = 2, l = 0 2s 2 elements
l = 1 2p 6 elements
l = 2 2d 10 elements
l = 3 2f 14 elements
There would be 50 elements in the first two rows.
(b) There would be 18 elements in the first row [see (a) above]. The fifth element in the
second row would have atomic number = 23.
(c) Z = 12
1s 1p 1d

206.5 x 103 J 1 mol


5.96 206.5 kJ = 206.5 x 103 J; E= x 23
= 3.429 x 10S19 J
1 mol 6.022 x 10
c hc (6.626 x 10 J • s)(3.00 x 10 m / s)
_ 34 8
λ= = 5.797 x 10 7 m = 580. nm
S
E=h , =
λ E 3.429 x 10 _19 J

5.97 780 nm is at the red end of the visible region of the electromagnetic spectrum.
780 nm = 780 x 10 9 m
S

c  3.00 x 108 m / s   1 kJ 
E = h = (6.626 x 10 _ 34 J• s)  23
  1000 J  (6.022 x 10 / mol) = 153 kJ / mol
λ  780 x 10
_9
m  

5.98 (a) Sr, Z = 38 [Kr]


5s
(b) Cd, Z = 48 [Kr]
5s 4d
(c) Z = 22, Ti [Ar]
4s 3d
(d) Z = 34, Se [Ar]
4s 3d 4p

5.99 La ([Xe] 6s2 5d1) is directly below Y ([Kr] 5s2 4d1) in the periodic table. Both have
similar valence electron configurations, but for La the valence electrons are one shell
farther out leading to its larger radius.
Although Hf ([Xe] 6s2 4f 14 5d2) is directly below Zr ([Kr] 5s2 4d2) in the periodic table,
Zr and Hf have almost identical atomic radii because the 4f electrons in Hf are not
effective in shielding the valence electrons. The valence electrons in Hf are drawn in
closer to the nucleus by the higher Zeff.

(418.8 kJ/mol)(4 2) (1350.7 kJ/mol)(4 2)


5.100 For K, Zeff = = 2.26; For Kr, Zeff = = 4.06
1312 kJ/mol 1312 kJ/mol

116
Chapter 5 S Periodicity and Atomic Structure
______________________________________________________________________________

5.101 75 W = 75 J/s; 550 nm = 550 x 10S9 m; (0.05)(75 J/s) = 3.75 J/s


c  3.00 x 10 m / s 
8
E = h = (6.626 x 10 _ 34 J• s)  •19
 = 3.61 x 10 J / photon
λ  550 x 10
_9
m 
3.75 J / s
number of photons = •19
= 1.0 x 1019 photons / s
3.61 x 10 J / photon

5.102 q = (350 g)(4.184 J/g≅oC)(95oC S 20oC) = 109,830 J


λ = 15.0 cm = 15.0 x 10S2 m
 3.00 x 108 m/s 
E = (6.626 x 10 34 J≅s)  = 1.33 x 10 24 J/photon
S S
_2 
 15.0 x 10 m 
109,830 J
number of photons = = 8.3 x 1028 photons
1.33 x 10 _ 24 J/photon

 kJ  1000 J   1 mol 
5.103 E =  310   23 
= 5.15 x 10•19 J
 mol  1 kJ  6.022 x 10 
c hc (6.626 x 10 J • s)(3.00 x 108 m / s)
_ 34
E=h , λ= = 3.86 x 10 7 m = 386 nm
S
=
λ E 5.15 x 10 J_19

5.104 48.2 nm = 48.2 x 10 9 m


S

3.00 x 108 m / s 1 kJ 6.022 x 10 23


E(photon) = 6.626 x 10 _ 34 J• s x x x = 2.48 x 103 kJ/mol
48.2 x 10 _ 9 m 1000 J mol

2  1 kJ   6.022 x 10 
23
_31
E K = E(electron) = ‰ (9.109 x 10 kg) (2.371 x 106 m / s)   
 1000 J   mol 
3
E K = 1.54 x 10 kJ/mol
E(photon) = Ei + EK; Ei = E(photon) B EK = (2.48 x 103) S (1.54 x 103) = 940 kJ/mol

5.105 Charge on electron = 1.602 x 10 19 C; 1 V ≅C = 1 J = 1 kg m2/s2


S

(a) EK = (30,000 V)(1.602 x 10 19 C) = 4.806 x 10 15 J


S S

2 EK 2 x 4.806 x 10 _15 kg m 2 / s 2
EK = 2mv2; v = = = 1.03 x 108 m/s
m 9.109 x 10 _ 31 kg
h 6.626 x 10 _ 34 kg m 2 /s
λ= = _ 31 8
= 7.06 x 10S12 m
mv (9.109 x 10 kg)(1.03 x 10 m/s)
c  3.00 x 108 m / s 
(b) E = h = (6.626 x 10 _ 34 J• s)  •15
 = 1.29 x 10 J / photon
λ  1.54 x 10
_10
m 

117
Chapter 5 S Periodicity and Atomic Structure
______________________________________________________________________________

5.106 Substitute the equation for the orbit radius, r, into the equation for the energy level, E, to
_ Ze 2 _ Z 2e 2
get E = =
 2  2a o n 2
2 n ao 
 Z 
Let E1 be the energy of an electron in a lower orbit and E2 the energy of an electron in a
higher orbit. The difference between the two energy levels is
_ 2 2 _ Z 2e 2 _ Z 2e 2 2 2 2 2 2 2
∆E = E2 S E1 = Z e2 _ = + Ze = Ze _ Ze
2a o n 2 2a o n12 2a o n 22 2a o n12 2a o n12 2a o n 22
e 1 1
2 2
∆E = Z  2
_ 2
2a o  n1 n2 
Because Z, e, and ao are constants, this equation shows that ∆E is proportional to
1 1
 2 _ 2  where n1 and n2 are integers with n2 > n1. This is similar to the Balmer-
 n1 n2 
Rydberg equation where 1/λ or v for the emission spectra of atoms is proportional to
 1 1
 2 _ 2  where m and n are integers with n > m.
m n 

5.107 (a) 0 0 0 0 0 0 0 0 0 ___


1s 2s 2p 3s 3p 4s
Two partially filled orbitals.
(b) The element in the 3rd column and 4th row under these new rules would have an
atomic number of 30 and be in the s-block.

5.108 (a) 3d, n = 3, l = 2


(b) 2p, n = 2, l = 1, ml = S1, 0, +1
3p, n = 3, l = 1, ml = S1, 0, +1
3d, n = 3, l = 2, ml = S2, S1, 0, +1, +2
(c) N, 1s2 2s2 2p3 so the 3s, 3p, and 3d orbitals are empty.
(d) C, 1s2 2s2 2p2 so the 1s and 2s orbitals are filled.
(e) Be, 1s2 2s2 so the 2s orbital contains the outermost electrons.
(f) 2p and 3p ( ) and 3d ( ).

5.109 λ = 1.03 x 10S7 m = 103 x 10S9 m = 103 nm


1  1 1
= R  2 _ 2  , R = 1.097 x 10 2 nm 1
S S

λ m n 
1 1 1 
= (1.097 x 10 _ 2 nm _1)  2 _ 2  , solve for n.
103 nm 1 n 
(1/103 nm) 1
_ 1= _ 2
(1.097 x 10 _ 2 nm _1) n

118
Chapter 5 S Periodicity and Atomic Structure
______________________________________________________________________________

1 1 1
2
= 0.115 ; n 2 = ; n= = 2.95
n 0.115 0.115
The electron jumps to the third shell.

E 7.21 x 10 _19 J
= 1.09 x 1015 s 1
S
5.110 (a) E = hv; v= =
h 6.626 x 10 J • s
_ 34

(b) E(photon) = Ei + EK; from (a), Ei = 7.21 x 10 19 J


S

c  3.00 x 108 m / s 
E(photon) = h = (6.626 x 10 _ 34 J• s)  •19
 = 7.95 x 10 J
λ  2.50 x 10
_7
m 
EK = E(photon) S Ei = (7.95 x 10 19 J) S (7.21 x 10 19 J) = 7.4 x 10 20 J
S S S

Calculate the electron velocity from the kinetic energy, EK.


EK = 7.4 x 10S20 J = 7.4 x 10S20 kgΑm2/s2 = 2mv2 = 2(9.109 x 10S31 kg)v2
2 x (7.4 x 10 _ 20 kg • m 2 / s 2)
v= _ 31
= 4.0 x 105 m/s
9.109 x 10 kg
h 6.626 x 10 _ 34 kg • m 2 /s
= 1.8 x 10 9 m = 1.8
S
deBroglie wavelength = =
m v (9.109 x 10 _ 31 kg)(4.0 x 105 m/s)
nm

Multi-Concept Problem

E 4.70 x 10 _16 J
= 7.09 x 1017 s 1
S
5.111 (a) E = hv; v = =
h 6.626 x 10 _ 34 J • s
c 3.00 x 108 m/s
= 4.23 x 10 10 m = 0.423 x 10 9 m = 0.423 nm
S S
(b) λ = =
ν 7.09 x 10 s
17 _1

h h 6.626 x 10 _ 34 kg • m 2 /s
(c) λ = ; v= = = 1.72 x 106 m/s
mv m λ (9.11 x 10 kg)(4.23 x 10 m)
_ 31 _10

2 _ 31 6 2
mv (9.11 x 10 kg)(1.72 x 10 m/s )
= 1.35 x 10 18 kg Α m2/s2 = 1.35 x 10 18 J
S S
(d) KE = =
2 2

5.112 (a) 5f subshell: n = 5, l = 3, ml = S3, S2, S1, 0, +1, +2, +3


3d subshell: n = 3, l = 2, ml = S2, S1, 0, +1, +2
(b) In the H atom the subshells in a particular energy level are all degenerate, i.e., all
have the same energy. Therefore, you only need to consider the principal quantum
number, n,
to calculate the wavelength emitted for an electron that drops from the 5f to the 3d subshell.
m = 3, n = 5; R = 1.097 x 10S2 nmS1
1  1 1 1 1 1 1
= 7.801 x 10 4 nm 1; λ = 1282 nm
S S
= R  2 _ 2; = R  2 _ 2 ;
λ m n  λ 3 5  λ

(c) m = 3, n = 4; R = 1.097 x 10 2 nm 1
S S

119
Chapter 5 S Periodicity and Atomic Structure
______________________________________________________________________________

1  1 1 1 1 1  1 1
= R  2  = 1.219 x 10 3 nm 1; λ = 820.3 nm
S S
= R  2 _ 2; = R  2 _ 2 ;
λ m n  λ 3 ∞  λ 3 
 3.00 x 10 m/s 
8
E = (6.626 x 10 _ 34 J• s)  _9
23 5
 (6.022 x 10 / mol) = 1.46 x 10 J/mol = 146
 820.3 x 10 m 
kJ/mol

5.113 (a) [Kr] 5s2 4d10 5p6 (b) [Kr] 5s2 4d10 5p5 6s1
*
(c) Both Xe and Cs have a single electron in the 6s orbital with similar effective nuclear
charges. Therefore the 6s electrons in both cases are held with similar strengths and
require almost the same energy to remove.

5.114 (a) Cl2, 70.91 amu


M + Cl2  MCl2
1 mol Cl 2
mol Cl2 = 0.8092 g Cl2 x = 0.01141 mol Cl2
70.91 g Cl 2
1 mol M
mol M = 0.01141 mol Cl2 x = 0.01141 mol M
1 mol Cl 2
1.000 g
molar mass of M = = 87.64 g/mol
0.01141 mol
atomic mass of M = 87.64 amu; M = Sr

9.46 kJ
(b) q = = 829 kJ/mol
0.01141 mol

5.115 (a) H3MO3(aq)  H3MO4(aq)


H3MO3(aq) + H2O(l)  H3MO4(aq)
H3MO3(aq) + H2O(l)  H3MO4(aq) + 2 H+(aq)
[H3MO3(aq) + H2O(l)  H3MO4(aq) + 2 H+(aq) + 2 e ] x 5 (oxidation half reaction)
S

MnO4S(aq)  Mn2+(aq)
MnO4 (aq)  Mn2+(aq) + 4 H2O(l)
S

MnO4 (aq) + 8 H+(aq)  Mn2+(aq) + 4 H2O(l)


S

[MnO4S(aq) + 8 H+(aq) + 5 eS  Mn2+(aq) + 4 H2O(l)] x 2 (reduction half reaction)

Combine the two half reactions.


5 H3MO3(aq) + 5 H2O(l) + 2 MnO4 (aq) + 16 H+(aq) 
S

5 H3MO4(aq) + 10 H+(aq) + 2 Mn2+(aq) + 8 H2O(l)


5 H3MO3(aq) + 2 MnO4S(aq) + 6 H+(aq)  5 H3MO4(aq) + 2 Mn2+(aq) + 3 H2O(l)

(b) 10.7 mL = 0.0107 L


mol MnO4 = (0.0107 L)(0.100 mol/L) = 1.07 x 10 3 mol MnO4
S S S

120
Chapter 5 S Periodicity and Atomic Structure
______________________________________________________________________________

5 mol H 3MO 3
mol H3MO3 = 1.07 x 10 3 mol MnO4 x = 2.67 x 10 3 mol H3MO3
S S S
_
2 mol MnO 4
1 mol M 2O 3
mol M2O3 = 2.67 x 10 3 mol H3MO3 x = 1.34 x 10 3 mol M2O3
S S

2 mol H 3MO 3
2 mol M
(c) mol M in M2O3 = 1.34 x 10S3 mol M2O3 x = 2.68 x 10 3 mol M
S

1 mol M 2O 3
0.200 g
M molar mass = = 74.6 g/mol; M atomic mass = 74.6 amu
2.68 x 10 _ 3 mol
M is As.

(d) E = hv = (6.626 x 10 34 JΑs )(9.07 x 1014 s 1) = 6.01 x 10 19 J/photon


S S S

E = (6.01 x 10 19 J/photon)(6.022 x 1023 photons/mol)(1 kJ/1000 J) = 362 kJ/mol


S

121
122
6 Ionic Bonds and
Some Main-Group Chemistry

6.1 (a) Ra2+ [Rn] (b) La3+ [Xe] (c) Ti4+ [Ar] (d) N3- [Ne]
Each ion has the ground-state electron configuration of the noble gas closest to it in the
periodic table.

6.2 The neutral atom contains 30 e- and is Zn. The ion is Zn2+.

6.3 (a) O2-; decrease in effective nuclear charge and an increase in electron-electron
repulsions lead to the larger anion.
(b) S; atoms get larger as you go down a group.
(c) Fe; in Fe3+ electrons are removed from a larger valence shell and there is an increase
in effective nuclear charge leading to the smaller cation.
(d) H-; decrease in effective nuclear charge and an increase in electron-electron
repulsions lead to the larger anion.

6.4 K+ is smaller than neutral K because the ion has one less electron. K+ and Cl- are
isoelectronic, but K+ is smaller than Cl- because of its higher effective nuclear charge. K
is larger than Cl- because K has one additional electron and that electron begins the next
shell (period). K+, r = 133 pm; Cl-, r = 184 pm; K, r = 227 pm

6.5 (a) Br (b) S (c) Se (d) Ne

6.6 (a) Be 1s2 2s2 N 1s2 2s2 2p3


Be would have the larger third ionization energy because this electron would come from
the 1s orbital.
(b) Ga [Ar] 4s2 3d10 4p1 Ge [Ar] 4s2 3d10 4p2
Ga would have the larger fourth ionization energy because this electron would come from
the 3d orbitals.

6.7 (b) Cl has the highest Ei1 and smallest Ei4.

6.8 Ca (red) would have the largest third ionization energy of the three because the electron
being removed is from a filled valence shell. For Al (green) and Kr (blue), the electron
being removed is from a partially filled valence shell. The third ionization energy for Kr
would be larger than that for Al because the electron being removed from Kr is coming
out of a set of filled 4p orbitals while the electron being removed from Al is coming out
of a half-filled 3s orbital. In addition, Zeff is larger for Kr than for Al. The ease of losing
its third electron is Al < Kr < Ca.

123
Chapter 6 - Ionic Bonds and Some Main-Group Chemistry
______________________________________________________________________________

6.9 Cr [Ar] 4s1 3d5 Mn [Ar] 4s2 3d5 Fe [Ar] 4s2 3d6
Cr can accept an electron into a 4s orbital. The 4s orbital is lower in energy than a 3d
orbital. Both Mn and Fe accept the added electron into a 3d orbital that contains an
electron, but Mn has a lower value of Zeff. Therefore, Mn has a less negative Eea than
either Cr or Fe.

6.10 The least favorable Eea is for Kr (red) because it is a noble gas with filled set of 4p
orbitals. The most favorable Eea is for Ge (blue) because the 4p orbitals would become
half filled. In addition, Zeff is larger for Ge than it is for K (green).

6.11 (a) KCl has the higher lattice energy because of the smaller K+.
(b) CaF2 has the higher lattice energy because of the smaller Ca2+.
(c) CaO has the higher lattice energy because of the higher charge on both the cation and
anion.

6.12 K(s) → K(g) +89.2 kJ/mol


K(g) → K+(g) + e- +418.8 kJ/mol
½ [F2(g) → 2 F(g)] +79 kJ/mol
F(g) + e- → F-(g) -328 kJ/mol
K+(g) + F-(g) → KF(s) -821 kJ/mol
Sum = -562 kJ/mol for K(s) + ½ F2(g) → KF(s)

6.13 The anions are larger than the cations. Cl- is larger than O2- because it is below it in the
periodic table. Therefore, (a) is NaCl and (b) is MgO. Because of the higher ion charge
and shorter cation – anion distance, MgO has the larger lattice energy.

6.14 (a) Li2O, O -2 (b) K2O2, O -1 (c) CsO2, O -½

6.15 (a) 2 Cs(s) + 2 H2O(l) → 2 Cs+(aq) + 2 OH-(aq) + H2(g)


(b) Na(s) + N2(g) → N. R.
(c) Rb(s) + O2(g) → RbO2(s)
(d) 2 K(s) + 2 NH3(g) → 2 KNH2(s) + H2(g)
(e) 2 Rb(s) + H2(g) → 2 RbH(s)

6.16 (a) Be(s) + Br2(l) → BeBr2(s)


(b) Sr(s) + 2 H2O(l) → Sr(OH)2(aq) + H2(g)
(c) 2 Mg(s) + O2(g) → 2 MgO(s)

6.17 BeCl2(s) + 2 K(s) → Be(s) + 2 KCl(s)

6.18 Mg(s) + S(s) → MgS(s); In MgS, the oxidation number of S is -2.

6.19 2 Al(s) + 6 H+(aq) → 2 Al3+(aq) + 3 H2(g)


H+ gains electrons and is the oxidizing agent. Al loses electrons and is the reducing
agent.

124
Chapter 6 - Ionic Bonds and Some Main-Group Chemistry
______________________________________________________________________________

6.20 2 Al(s) + 3 S(s) → Al2S3(s)


6.21 (a) Br2(l) + Cl2(g) → 2 BrCl(g)
(b) 2 Al(s) + 3 F2(g) → 2 AlF3(s)
(c) H2(g) + I2(s) → 2 HI(g)

6.22 Br2(l) + 2 NaI(s) → 2 NaBr(s) + I2(s)


Br2 gains electrons and is the oxidizing agent. I- (from NaI) loses electrons and is the
reducing agent.

6.23 (a) XeF2 F -1, Xe +2 (b) XeF4 F -1, Xe +4


(c) XeOF4 F -1, O -2, Xe +6

6.24 (a) Rb would lose one electron and adopt the Kr noble-gas configuration.
(b) Ba would lose two electrons and adopt the Xe noble-gas configuration.
(c) Ga would lose three electrons and adopt an Ar-like noble-gas configuration (note that
Ga3+ has ten 3d electrons in addition to the two 3s and six 3p electrons).
(d) F would gain one electron and adopt the Ne noble-gas configuration.

6.25 Group 6A elements will gain 2 electrons.

6.26 Only about 10% of current world salt production comes from evaporation of seawater.
Most salt is obtained by mining the vast deposits of halite, or rock salt, formed by
evaporation of ancient inland seas. These salt beds can be up to hundreds of meters thick
and may occur anywhere from a few meters to thousands of meters below the earth's surface.

Understanding Key Concepts

6.27

6.28 (a) shows an extended array, which represents an ionic compound.


(b) shows discrete units, which represent a covalent compound.

125
Chapter 6 - Ionic Bonds and Some Main-Group Chemistry
______________________________________________________________________________

6.29

6.30

(a) Al3+ (b) Cr3+ (c) Sn2+ (d) Ag+

6.31 The first sphere gets larger on going from reactant to product. This is consistent with it
being a nonmetal gaining an electron and becoming an anion. The second sphere gets
smaller on going from reactant to product. This is consistent with it being a metal losing
an electron and becoming a cation.

6.32 (a) I2 (b) Na (c) NaCl (d) Cl2

6.33 (c) has the largest lattice energy because the charges are closest together.
(a) has the smallest lattice energy because the charges are farthest apart.

6.34 Green CBr4: C, +4; Br, -1


Blue SrF2: Sr, +2; F, -1
Red PbS: Pb, +2; S, -2 or PbS2: Pb, +4; S, -2

126
Chapter 6 - Ionic Bonds and Some Main-Group Chemistry
______________________________________________________________________________

6.35

Additional Problems
Ionization Energy and Electron Affinity

6.36 (a) La3+, [Xe] (b) Ag+, [Kr] 4d10 (c) Sn2+, [Kr] 5s2 4d10

6.37 (a) Se2-, [Kr] (b) N3-, [Ne]


6.38 Cr2+ [Ar] 3d4 ↑ ↑
↑ ↑
3d
Fe2+ [Ar] 3d6 ↑↓ ↑ ↑ ↑ ↑
3d
6.39 Z = 30, Zn

6.40 Ionization energies have a positive sign because energy is required to remove an electron
from an atom of any element.

6.41 Electron affinities have a negative sign because energy is released when an electron is
added.

6.42 The largest Ei1 are found in Group 8A because of the largest values of Zeff.
The smallest Ei1 are found in Group 1A because of the smallest values of Zeff.

6.43 Fr would have the smallest ionization energy, and He would have the largest.

6.44 (a) K [Ar] 4s1 Ca [Ar] 4s2


Ca has the smaller second ionization energy because it is easier to remove the second 4s
valence electron in Ca than it is to remove the second electron in K from the filled 3p
orbitals.
(b) Ca [Ar] 4s2 Ga [Ar] 4s2 3d10 4p1
Ca has the larger third ionization energy because it is more difficult to remove the third
electron in Ca from the filled 3p orbitals than it is to remove the third electron (second 4s
valence electron) from Ga.

127
Chapter 6 - Ionic Bonds and Some Main-Group Chemistry
______________________________________________________________________________

6.45 Sn has a smaller fourth ionization energy than Sb because of a smaller Zeff.
Br has a larger sixth ionization energy than Se because of a larger Zeff.

6.46 (a) 1s2 2s2 2p6 3s2 3p3 is P (b) 1s2 2s2 2p6 3s2 3p6 is Ar (c) 1s2 2s2 2p6 3s2 3p6 4s2 is
Ca
Ar has the highest Ei2. Ar has a higher Zeff than P. The 4s electrons in Ca are easier to
remove than any 3p electrons.
Ar has the lowest Ei7. It is difficult to remove 3p electrons from Ca, and it is difficult to
remove 2p electrons from P.

6.47 The atom in the third row with the lowest Ei4 is the 4A element, Si. 1s2 2s2 2p6 3s2 3p2

6.48 Using Figure 6.3 as a reference:


Lowest Ei1 Highest Ei1
(a) K Li
(b) B Cl
(c) Ca Cl

6.49 (a) Group 2A (b) Group 6A

6.50 The relationship between the electron affinity of a univalent cation and the ionization
energy of the neutral atom is that they have the same magnitude but opposite sign.
6.51 The relationship between the ionization energy of a univalent anion and the electron
affinity of the neutral atom is that they have the same magnitude but opposite sign.

6.52 Na+ has a more negative electron affinity than either Na or Cl because of its positive charge.

6.53 Br would have a more negative electron affinity than Br- because Br- has no room in its
valence shell for an additional electron.

6.54 Energy is usually released when an electron is added to a neutral atom but absorbed when
an electron is removed from a neutral atom because of the positive Zeff.

6.55 Ei1 increases steadily across the periodic table from Group 1A to Group 8A because
electrons are being removed from the same shell and Zeff is increasing. The electron
affinity increases irregularly from 1A to 7A and then falls dramatically for Group 8A
because the additional electron goes into the next higher shell.

6.56 (a) F; nonmetals have more negative electron affinities than metals.
(b) Na; Ne (noble gas) has a positive electron affinity.
(c) Br; nonmetals have more negative electron affinities than metals.

6.57 Zn, Cd, and Hg all have filled s and d subshells. An additional electron would have to go
into the higher energy p subshell. This is unfavorable and results in near-zero electron
affinities.

128
Chapter 6 - Ionic Bonds and Some Main-Group Chemistry
______________________________________________________________________________

Lattice Energy and Ionic Bonds

6.58 MgCl2 > LiCl > KCl > KBr

6.59 AlBr3 > CaO > MgBr2 > LiBr

6.60 Li → Li+ + e- +520 kJ/mol


Br + e- → Br- -325 kJ/mol
+195 kJ/mol

6.61 The total energy = (376 kJ/mol) + (-349 kJ/mol) = +27 kJ/mol, which is unfavorable
because it is positive.

6.62 Li(s) → Li(g) +159.4 kJ/mol


Li(s) → Li(g) + e- +520 kJ/mol
½ [Br2(l) → Br2(g)] +15.4 kJ/mol
½ [Br2(g) → 2 Br(g)] +112 kJ/mol
Br(g) + e- → Br-(g) -325 kJ/mol
Li+(g) + Br-(g) → LiBr(s) -807 kJ/mol
Sum = -325 kJ/mol for Li(s) + ½ Br2(l) → LiBr(s)

6.63 (a) Li(s) → Li(g) +159.4 kJ/mol


Li(g) → Li+(g) + e- +520 kJ/mol
½[F2(g) → 2 F(g)] +79 kJ/mol
F(g) + e- → F-(g) -328 kJ/mol
Li+(g) + F-(g) → LiF(s) -1036 kJ/mol
Sum = -606 kJ/mol for Li(s) + ½ F2(g) → LiF(s)

(b) Ca(s) → Ca(g) +178.2 kJ/mol


Ca(g) → Ca+(g) + e- +589.8 kJ/mol
Ca+(g) → Ca2+(g) + e- +1145 kJ/mol
F2(g) → 2 F(g) +158 kJ/mol
2[F(g) + e- → F-(g)] 2(-328) kJ/mol
Ca2+(g) + 2 F- → CaF2(s) -2630 kJ/mol
Sum = -1215 kJ/mol for Ca(s) + F2(g) → CaF2(s)

6.64 Na(s) → Na(g) +107.3 kJ/mol


Na(g) → Na+(g) + e- +495.8 kJ/mol
½ [H2(g) → 2 H(g)] ½(+435.9) kJ/mol
H(g) + e- → H-(g) -72.8 kJ/mol
Na+(g) + H-(g) → NaH(s) -U

129
Chapter 6 - Ionic Bonds and Some Main-Group Chemistry
______________________________________________________________________________

Sum = -60 kJ/mol for Na(s) + ½ H2(g) → NaH(s)

- U = - 60 - 107.3 - 495.8 - 435.9/2 + 72.8 = -808 kJ/mol; U = 808 kJ/mol

6.65 Ca(s) → Ca(g) +178.2 kJ/mol


Ca(g) → Ca (g) + e
+ -
+589.8 kJ/mol
Ca (g) → Ca (g) + e
+ 2+ -
+1145 kJ/mol
H2(g) → 2 H(g) +435.9 kJ/mol
2[H(g) + e → H (g)]
- -
2(-72.8) kJ/mol
Ca2+(g) + 2 H-(g) → CaH2(s) -U
Sum = -186.2 kJ/mol for Ca(s) + H2(g) → CaH2(s)

- U = -186.2 - 178.2 - 589.8 - 1145 - 435.9 + 2(72.8) = -2390 kJ/mol; U = 2390 kJ/mol

6.66 Cs(s) → Cs(g) +76.1 kJ/mol


Cs(g) → Cs+(g) + e- +375.7 kJ/mol
½ [F2(g) → 2 F(g)] +79 kJ/mol
F(g) + e- → F-(g) -328 kJ/mol
Cs+(g) + F-(g) → CsF(s) -740 kJ/mol
Sum = -537 kJ/mol for Cs(s) + ½ F2(g) → CsF(s)

6.67 Cs(s) → Cs(g) +76.1 kJ/mol


Cs(g) → Cs+(g) + e- +375.7 kJ/mol
Cs+(g) → Cs2+(g) + e- +2422 kJ/mol
F2(g) → 2 F(g) +158 kJ/mol
2[F(g) + e- → F-(g)] 2(-328) kJ/mol
Cs2+(g) + 2 F-(g) → CsF2(s) -2347 kJ/mol
Sum = +29 kJ/mol for Cs(s) + F2(g) → CsF2(s)

The overall reaction absorbs 29 kJ/mol.


In the reaction of cesium with fluorine, CsF will form because the overall energy for the
formation of CsF is negative, whereas it is positive for CsF2.

6.68 Ca(s) → Ca(g) +178.2 kJ/mol


Ca(g) → Ca+(g) + e- +589.8 kJ/mol
½[Cl2(g) → 2 Cl(g)] +121.5 kJ/mol
Cl(g) + e- → Cl-(g) -348.6 kJ/mol
Ca+(g) + Cl-(g) → CaCl(s) -717 kJ/mol
Sum = -176 kJ/mol for Ca(s) + ½ Cl2(g) → CaCl(s)

6.69 Ca(s) → Ca(g) + e- +178.2 kJ/mol

130
Chapter 6 - Ionic Bonds and Some Main-Group Chemistry
______________________________________________________________________________

Ca(g) → Ca+(g) + e- +589.8 kJ/mol


Ca (g) → Ca (g)
+ 2+
+1145 kJ/mol
Cl2(g) → 2 Cl(g) +243 kJ/mol
2[Cl(g) + e → Cl (g)]
- -
2(-348.6) kJ/mol
Ca2+(g) + 2 Cl-(g) → CaCl2(s) -2258 kJ/mol
Sum = -799 kJ/mol for Ca(s) + Cl2(g) → CaCl2(s)

In the reaction of calcium with chlorine, CaCl2 will form because the overall energy for
the formation of CaCl2 is much more negative than for the formation of CaCl.

6.70

6.71

Main-Group Chemistry

6.72 Solids: I2; Liquids: Br2; Gases: F2, Cl2, He, Ne, Ar, Kr, Xe

6.73 (a) Li is used in automotive grease. Li2CO3 is a manic depressive drug.

131
Chapter 6 - Ionic Bonds and Some Main-Group Chemistry
______________________________________________________________________________

(b) K salts are used in plant fertilizers.


(c) SrCO3 is used in color TV picture tubes. Sr salts are used for red fireworks.
(d) Liquid He (bp = 4.2 K) is used for low temperature studies and for cooling
superconducting magnets.

6.74 (a) At is in Group 7A. The trend going down the group is gas → liquid → solid. At,
being at the bottom of the group, should be a solid.
(b) At would likely be dark, like I2, maybe with a metallic sheen.
(c) At is likely to react with Na just like the other halogens, yielding NaAt.

6.75 Predicted for Fr: melting point ≈ 23 oC boiling point ≈ 650 oC


density ≈ 2 g/cm3 atomic radius ≈ 275 pm

electrolysis
in CaCl2

6.76 (a) 2 NaCl _ 2 Na(l) + Cl2 (g)


580° C

electrolysis
in Na 3 AlF6

(b) 2 Al2 O3 _ 4 Al(l) + 3 O2 (g)


980° C

(c) Ar is obtained from the distillation of liquid air.


(d) 2 Br-(aq) + Cl2(g) → Br2(l) + 2 Cl-(aq)

6.77 Group 1A metals react by losing an electron. Down Group 1A, the valence electron is
more easily removed. This trend parallels chemical reactivity.
Group 7A nonmetals react by gaining an electron. The electron affinity generally
increases up the group. This trend parallels chemical reactivity.

6.78 Main-group elements tend to undergo reactions that leave them with eight valence
electrons. That is, main-group elements react so that they attain a noble gas electron
configuration with filled s and p sublevels in their valence electron shell.
The octet rule works for valence shell electrons because taking electrons away from a
filled octet is difficult because they are tightly held by a high Zeff; adding more electrons
to a filled octet is difficult because, with s and p sublevels full, there is no low-energy
orbital available.

6.79 Main-group nonmetals in the third period and below occasionally break the octet rule.

6.80 (a) 2 K(s) + H2(g) → 2 KH(s)


(b) 2 K(s) + 2 H2O(l) → 2 K+(aq) + 2 OH-(aq) + H2(g)
(c) 2 K(s) + 2 NH3(g) → 2 KNH2(s) + H2(g)
(d) 2 K(s) + Br2(l) → 2 KBr(s)

132
Chapter 6 - Ionic Bonds and Some Main-Group Chemistry
______________________________________________________________________________

(e) K(s) + N2(g) → N. R.


(f) K(s) + O2(g) → KO2(s)

6.81 (a) Ca(s) + H2(g) → CaH2(s)


(b) Ca(s) + 2 H2O(l) → Ca2+(aq) + 2 OH-(aq) + H2(g)
(c) Ca(s) + He(g) → N. R.
(d) Ca(s) + Br2(l) → CaBr2(s)
(e) 2 Ca(s) + O2(g) → 2 CaO(s)

6.82 (a) Cl2(g) + H2(g) → 2 HCl(g)


(b) Cl2(g) + Ar(g) → N. R.
(c) Cl2(g) + Br2(l) → 2 BrCl(g)
(d) Cl2(g) + N2(g) → N. R.

6.83 (a) 2 Cl-(aq) + F2(g) → 2 F-(aq) + Cl2(g)


F2 gains electrons and is the oxidizing agent. Cl- loses electrons and is the reducing agent.
(b) 2 Br-(aq) + I2(s) → N. R.
(c) 2 I-(aq) + Br2(aq) → 2 Br-(aq) + I2(aq)
Br2 gains electrons and is the oxidizing agent. I- loses electrons and is the reducing agent.

6.84 AlCl3 + 3 Na → Al + 3 NaCl


Al3+ (from AlCl3) gains electrons and is reduced. Na loses electrons and is oxidized.

6.85 2 Mg(s) + O2(g) → 2 MgO(s)


MgO(s) + H2O(l) → Mg(OH)2(aq)

6.86 CaIO3, 215.0 amu; 1.00 kg = 1000 g


126.9 g
%I= x 100% = 59.02%; (0.5902)(1000 g) = 590 g I2
215.0 g

6.87 2 Li(s) + 2 H2O(l) → 2 LiOH(aq) + H2(g); 455 mL = 0.455 L


mass of H2 = (0.0893 g/L)(0.455 L) = 0.0406 g H2
1 mol H 2 2 mol Li 6.94 g Li
0.0406 g H 2 x x x = 0.280 g Li
2.016 g H2 1 mol H 2 1 mol Li

6.88 Ca(s) + H2(g) → CaH2(s); H2, 2.016 amu; CaH2, 42.09 amu
1 mol Ca
5.65 g Ca x = 0.141 mol Ca
40.08 g Ca
g 1 mol H 2
3.15 L H2 x 0.0893 H 2 x = 0.140 mol H2
1 L 2.016 g H 2
Because the reaction stoichiometry between Ca and H2 is one to one, H2 is the limiting
reactant.

133
Chapter 6 - Ionic Bonds and Some Main-Group Chemistry
______________________________________________________________________________

1 mol CaH 2 42.09 g CaH 2


0.140 mol H2 x x x 0.943 = 5.56 g CaH2
1 mol H 2 1 mol CaH 2

6.89 6 Li(s) + N2(g) → 2 Li3N(s); N2, 28.01 amu; Li3N, 34.83 amu
1 mol Li 1 mol N 2 28.01 g N 2 1L
2.87 g Li x x x x = 1.54 L N2
6.941 g Li 6 mol Li 1 mol N 2 1.25 g N 2

6.90 (a) Mg(s) + 2 H+(aq) → Mg2+(aq) + H2(g)


H+ gains electrons and is the oxidizing agent. Mg loses electrons and is the reducing
agent.
(b) Kr(g) + F2(g) → KrF2(s)
F2 gains electrons and is the oxidizing agent. Kr loses electrons and is the reducing agent.
(c) I2(s) + 3 Cl2(g) → 2 ICl3(l)
Cl2 gains electrons and is the oxidizing agent. I2 loses electrons and is the reducing agent.

6.91 (a) 2 XeF2(s) + 2 H2O(l) → 2 Xe(g) + 4 HF(aq) + O2(g)


Xe in XeF2 gains electrons and is the oxidizing agent. O in H2O loses electrons and is the
reducing agent.
(b) NaH(s) + H2O(l) → Na+(aq) + OH-(aq) + H2(g)
H in H2O gains electrons and is the oxidizing agent. H in NaH loses electrons and is the
reducing agent.
(c) 2 TiCl4(l) + H2(g) → 2 TiCl3(s) + 2 HCl(g)
Ti in TiCl4 gains electrons and is the oxidizing agent. H2 loses electrons and is the
reducing agent.

General Problems

6.92 Cu2+ has fewer electrons and a larger effective nuclear charge; therefore it has the smaller
ionic radius.

6.93 S2- > Ca2+ > Sc3+ > Ti4+, Zeff increases on going from S2- to Ti4+.

6.94 Mg(s) → Mg(g) +147.7 kJ/mol


Mg(g) → Mg+(g) + e- +737.7 kJ/mol
½[F2(g) → 2 F(g)] +79 kJ/mol
F(g) + e- → F-(g) -328 kJ/mol
Mg+(g) + F-(g) → MgF(s) -930 kJ/mol
Sum = -294 kJ/mol for Mg(s) + ½ F2(g) → MgF(s)

Mg(s) → Mg(g) +147.7 kJ/mol


Mg(g) → Mg+(g) + e- +737.7 kJ/mol

134
Chapter 6 - Ionic Bonds and Some Main-Group Chemistry
______________________________________________________________________________

Mg+(g) → Mg2+(g) + e- +1450.7 kJ/mol


F2(g) → 2 F(g) +158 kJ/mol
2[F(g) + e- → F-(g)] 2(-328) kJ/mol
Mg (g) + 2 F (g) → MgF2(s) -2952
2+ -
kJ/mol
Sum = -1114 kJ/mol for Mg(s) + F2(g) → MgF2(s)

6.95 In the reaction of magnesium with fluorine, MgF2 will form because the overall energy
for the formation of MgF2 is much more negative than for the formation of MgF.

6.96 (a) Na is used in table salt (NaCl), glass, rubber, and pharmaceutical agents.
(b) Mg is used as a structural material when alloyed with Al.
(c) F2 is used in the manufacture of Teflon, (C2F4)n, and in toothpaste as SnF2.

electrolysis
6.97 (a) 2 HF(l) _ H 2 (g) + F2 (g)
100° C
high temp erature
(b) 3 CaO(l) + 2 Al(l) _ 3 Ca(l) + Al2 O3 (s)

electrolysis
in CaCl2

(c) 2 NaCl(l) _ 2 Na(l) + Cl2 (g)


580° C

6.98 (a) 2 Li(s) + H2(g) → 2 LiH(s)


(b) 2 Li(s) + 2 H2O(l) → 2 Li+(aq) + 2 OH-(aq) + H2(g)
(c) 2 Li(s) + 2 NH3(g) → 2 LiNH2(s) + H2(g)
(d) 2 Li(s) + Br2(l) → 2 LiBr(s)
(e) 6 Li(s) + N2(g) → 2 Li3N(s)
(f) 4 Li(s) + O2(g) → 2 Li2O(s)

6.99 (a) F2(g) + H2(g) → 2 HF(g) (b) F2(g) + 2 Na(s) → 2 NaF(s)


(c) F2(g) + Br2(l) → 2 BrF(g) (d) F2(g) + 2 NaBr(s) → 2 NaF(g) + Br2(l)

6.100 When moving diagonally down and right on the periodic table, the increase in atomic
radius caused by going to a larger shell is offset by a decrease caused by a higher Zeff.
Thus, there is little net change.

6.101 Na(s) → Na(g) +107.3 kJ/mol


Na(g) + e → Na (g)
- -
-52.9 kJ/mol
½[Cl2(g) → 2 Cl(g)] +122 kJ/mol
Cl(g) → Cl+(g) + e- +1251 kJ/mol
Na-(g) + Cl+(g) → ClNa(s) -787 kJ/mol
Sum = +640 kJ/mol for Na(s) + ½ Cl2(g) → Cl+Na-(s)
The formation of Cl+Na- from its elements is not favored because the net energy change is

135
Chapter 6 - Ionic Bonds and Some Main-Group Chemistry
______________________________________________________________________________

positive whereas for the formation of Na+Cl- it is negative.

6.102

6.103 94.2 mL = 0.0942 L


1 mol Cl2 2 mol Cl _
0.0942 L Cl2 x x = 8.41 x 10-3 mol Cl-
22.4 L 1 mol Cl2
Possible formulas for the metal halide are MCl, MCl2, MCl3, etc.
For MCl, mol M = mol Cl- = 8.41 x 10-3 mol M
0.719 g
molar mass of M = = 85.5 g/mol
8.41 x 10_ 3 mol
1 mol M
For MCl2, mol M = 8.41 x 10-3 mol Cl- x _
1111 = 4.20 x 10-3 mol M
2 mol Cl
0.719 g
molar mass of M = = 171 g/mol
4.20 x 10_ 3 mol
1 mol M
For MCl3, mol M = 8.41 x 10-3 mol Cl- x _
= 2.80 x 10-3 mol M
3 mol Cl
0.719 g
molar mass of M = = 257 g/mol
2.80 x 10_ 3 mol
The best match for a metal is with 85.5 g/mol, which is Rb.

6.104 Mg(s) → Mg(g) +147.7 kJ/mol


Mg(g) → Mg+(g) + e- +738 kJ/mol
Mg+(g) → Mg2+(g) + e- +1451 kJ/mol
½[O2(g) → 2 O(g)] +249.2 kJ/mol
O(g) + e- → O-(g) -141.0 kJ/mol
O-(g) + e- → O2-(g) Eea2
Mg2+(g) + O2-(g) → MgO(s) -3791 kJ/mol
Mg(s) + ½O2(g) → MgO(s) -601.7 kJ/mol

147.7 + 738 + 1451 + 249.2 - 141.0 + Eea2 - 3791 = -601.7


Eea2 = -147.7 - 738 - 1451 - 249.2 + 141.0 + 3791 - 601.7 = +744 kJ/mol
Because Eea2 is positive, O2- is not stable in the gas phase. It is stable in MgO because of
the large lattice energy that results from the +2 and -2 charge of the ions and their small size.

136
Chapter 6 - Ionic Bonds and Some Main-Group Chemistry
______________________________________________________________________________

6.105 (a) (i) Ra because it is farthest down (7th period) in the periodic table.
(ii) In because it is farthest down (5th period) in the periodic table.
(b) (i) Tl and Po are farthest down (6th period) but Tl is larger because it is to the left of
Po and thus has the smaller ionization energy.
(ii) Cs and Bi are farthest down (6th period) but Cs is larger because it is to the left of
Bi and thus has the smaller ionization energy.

6.106 (a) The more negative the Eea, the greater the tendency of the atom to accept an electron,
and the more stable the anion that results. Be, N, O, and F are all second row elements.
F has the most negative Eea of the group because the anion that forms, F-, has a complete
octet of electrons and its nucleus has the highest effective nuclear charge.
(b) Se2- and Rb+ are below O2- and F- in the periodic table and are the larger of the four.
Se2- and Rb+ are isoelectronic, but Rb+ has the higher effective nuclear charge so it is
smaller. Therefore Se2- is the largest of the four ions.

6.107 Ca(s) → Ca(g) +178 kJ/mol


Ca(g) → Ca (g) +
+590 kJ/mol
Ca+(g) → Ca2+(g) +1145 kJ/mol
2 C(s) → 2 C(g) 2(+717 kJ/mol)
2 C(g) → C2(g) -614 kJ/mol
C2(g) → C2-(g) -315 kJ/mol
C2 (g) → C2 (g)
- 2-
+410 kJ/mol
Ca2+(g) + C22-(g) → CaC2(s) -U
Ca(s) + 2 C(s) → CaC2(s) -60 kJ/mol
-U = -60 -178 - 590 - 1145 - 2(717) + 614 + 315 - 410 = -2888 kJ/mol
U = 2888 kJ/mol
6.108 Cr(s) → Cr(g) +397 kJ/mol
Cr(g) → Cr+(g) +652 kJ/mol
Cr (g) → Cr (g)
+ 2+
+1588 kJ/mol
Cr2+(g) → Cr3+(g) +2882 kJ/mol
½(I2(s) → I2(g)) +62/2 kJ/mol
½ (I2(g) → 2 I(g)) +151/2 kJ/mol
I(g) + e- → I-(g) -295 kJ/mol
Cl2(g) → 2 Cl(g) +243 kJ/mol
2(Cl(g) + e → Cl (g))
- -
2(-349) kJ/mol
Cr3+(g) + 2 Cl-(g) + I-(g) → CrCl2I(s) -U
Cr(s) + Cl2(g) + ½ I2(g) → CrCl2I(s) - 420 kJ/mol
-U = - 420 - 397 - 652 - 1588 - 2882 - 62/2 - 151/2 + 295 - 243 + 2(349) = -5295.5 kJ/mol
U = 5295 kJ/mol

Multi-Concept Problems

6.109 (a) E = (703 kJ/mol)(1000 J/1 kJ)/(6.022 x 1023 photons/mol) = 1.17 x 10-18 J/photon

137
Chapter 6 - Ionic Bonds and Some Main-Group Chemistry
______________________________________________________________________________

hc
E=
λ
hc (6.626 x 10_ 34 J • s)(3.00 x 108 m/s)
λ= = _ 18
= 1.70 x 10-7 m = 170 x 10-9 m = 170 nm
E 1.17 x 10 J
(b) Bi [Xe] 6s 4f 5d10 6p3
2 14

Bi+ [Xe] 6s2 4f 14 5d10 6p2


(c) n = 6, l = 1
(d) Element 115 would be directly below Bi in the periodic table. The valence electron
is farther from the nucleus and less strongly held than in Bi. The ionization energy for
element 115 would be less than that for Bi.

6.110 (a) Fe [Ar] 4s2 3d6


Fe2+ [Ar] 3d6
3+
Fe [Ar] 3d5
(b) A 3d electron is removed on going from Fe2+ to Fe3+. For the 3d electron, n = 3 and l =
2.
1 mol photons 1000 J
(c) E(J/photon) = 2952 kJ/mol x 23
x = 4.90 x 10-18 J/photon
6.022 x 10 photons 1 kJ
hc
E=
λ
h c (6.626 x 10_ 34 J • s)(3.00 x 108 m/s)
λ= = _ 18
= 4.06 x 10-8 m = 40.6 x 10-9 m = 40.6
E 4.90 x 10 J
nm
(d) Ru is directly below Fe in the periodic table and the two metals have similar electron
configurations. The electron removed from Ru to go from Ru2+ to Ru3+ is a 4d electron.
The electron with the higher principal quantum number, n = 4, is farther from the nucleus,
less tightly held, and requires less energy to remove.

6.111 (a) 58.4 nm = 58.4 x 10-9 m


3.00 x 108 m/s 1 kJ 6.022 x 1023
E(photon) = 6.626 x 10-34 J⋅s x x x = 2049
58.4 x 10-9 m 1000 J mol
kJ/mol
 1 kJ  6.022 x 1023 
EK = E(electron) = ½(9.109 x 10-31 kg)(2.450 x 106 m/s)2   
 1000 J  mol 
EK = 1646 kJ/mol
E(photon) = Ei + EK; Ei = E(photon) - EK = 2049 - 1646 = 403 kJ/mol

(b) 142 nm = 142 x 10-9 m


3.00 x 108 m / s 1 kJ 6.022 x 1023
E(photon) = 6.626 x 10_ 34 J• s x x x = 843 kJ/mol
142 x 10_ 9 m 1000 J mol

138
Chapter 6 - Ionic Bonds and Some Main-Group Chemistry
______________________________________________________________________________

2  1 kJ  6.022 x 10 
23
EK = E(electron) = ‰ (9.109 x 10
_ 31
kg) (1.240 x 10
6
m / s)  
 
 1000 J  mol 
E K = 422 kJ/mol
E(photon) = Ei + EK; Ei = E(photon) – EK = 843 – 422 = 421 kJ/mol

6.112 AgCl, 143.32 amu


35.453 g Cl
(a) mass Cl in AgCl = 1.126 g AgCl x = 0.279 g Cl
143.32 g AgCl
0.279 g Cl
%Cl in alkaline earth chloride = x 100% = 64.0% Cl
0.436 g
(b) Because M is an alkaline earth metal, M is a 2+ cation.
For MCl2, mass of M = 0.436 g - 0.279 g = 0.157 g M
1 mol Cl 1 mol M
mol M = 0.279 g Cl x x = 0.003 93 mol M
35.453 g Cl 2 mol Cl
0.157 g
molar mass for M = = 39.9 g/mol; M = Ca
0.003 93 mol

(c) Ca(s) + Cl2(g) → CaCl2(s)


CaCl2(aq) + 2 AgNO3(aq) → 2 AgCl(s) + Ca(NO3)2(aq)

1 mol Ca
(d) 1.005 g Ca x = 0.0251 mol Ca
40.078 g Ca
1 mol Cl2
1.91 x 1022 Cl2 molecules x = 0.0317 mol Cl2
6.022 x 1023 Cl2 molecules
Because the stoichiometry between Ca and Cl2 is one to one, the Cl2 is in excess.
70.91 g Cl2
Mass Cl2 unreacted = (0.0317 - 0.0251) mol Cl2 x = 0.47 g Cl2 unreacted
1 mol Cl2

6.113 (a) (i) M2O3(s) + 3 C(s) + 3 Cl2(g) → 2 MCl3(l) + 3 CO(g)


(ii) 2 MCl3(l) + 3 H2(g) → 2 M(s) + 6 HCl(g)
(b) HCl(aq) + NaOH(aq) → H2O(l) + NaCl(aq)
144.2 mL = 0.1442 L
mol NaOH = (0.511 mol/L)(0.1442 L) = 0.07369 mol NaOH
1 mol HCl
mol HCl = 0.07369 mol NaOH x = 0.07369 mol HCl
1 mol NaOH
2 mol M
mol M = 0.07369 mol HCl x = 0.02456 mol M
6 mol HCl
2 mol MCl3 1 mol M 2 O3
mol M2O3 = 0.02456 mol M x x = 0.01228 mol M2O3
2 mol M 2 mol MCl3

139
Chapter 6 - Ionic Bonds and Some Main-Group Chemistry
______________________________________________________________________________

0.855 g
molar mass M2O3 = = 69.6 g/mol; molecular mass M2O3 = 69.6 amu
0.01228 mol
69.6 amu _ (3 x 16.0 amu)
atomic mass of M = = 10.8 amu; M = B
2
10.81 g M
(c) mass of M = 0.02456 mol M x = 0.265 g M
1 mol M

6.114 (a) Sr(s) → Sr(g) +164.44 kJ/mol


Sr(g) → Sr (g) + e
+ -
+549.5 kJ/mol
Sr+(g) → Sr2+(g) + e- +1064.2 kJ/mol
Cl2(g) → 2 Cl(g) +243 kJ/mol
2[Cl(g) + e- → Cl-(g)] 2(-348.6) kJ/mol
Sr2+(g) + 2 Cl-(g) → SrCl2(s) -2156 kJ/mol
Sum = -832 kJ/mol for Sr(s) + Cl2(g) → SrCl2(s)
(b) Sr, 87.62 amu; Cl2, 70.91 amu; SrCl2, 158.53 amu
1 mol Sr 1 mol Cl2
20.0 g Sr x = 0.228 mol Sr and 25.0 g Cl2 x = 0.353 mol Cl2
87.62 g Sr 70.91 g Cl2
Because there is a 1:1 stoichiometry between the reactants, the one with the smaller mole
amount is the limiting reactant. Sr is the limiting reactant.
1 mol SrCl2 158.53 g SrCl2
0.228 mol Sr x x = 36.1 g SrCl2
1 mol Sr 1 mol SrCl2
_ 832 kJ
(c) 0.228 mol SrCl2 x = -190 kJ
1 mol SrCl2
190 kJ is released during the reaction of 20.0 g of Sr with 25.0 g Cl2.

6.115 (a) The alkali metal is Li because it is the only alkali metal to form the nitride, M3N.
(b) 4 Li(s) + O2(g) → 2 Li2O(s)
6 Li(s) + N2(g) → 2 Li3N(s)
Li2O(s) + H2O(l) → 2 LiOH(aq)
Li3N(s) + 3 H2O(l) → NH3(aq) + 3 LiOH(aq)
(c) 96.8 mL = 0.0968 L
mol HCl = (0.0968 L)(0.100 mol/L) = 0.009 68 mol HCl
Note, that the HCl neutralized only 20% (0.20) of the total sample.
Let Y = mol Li2O and let Z = mol Li3N
mol HCl = 2Y + 4Z = 0.009 68 mol
1 mol Li
mol Li = 2Y + 3Z = (0.20)(0.265 g Li) x = 0.007 64 mol Li
6.941 g Li
2Y + 4Z = 0.009 68
2Y + 3Z = 0.007 64
2Y = 0.007 64 - 3Z (substitute 2Y into the first equation and solve for Z)
0.007 64 - 3Z + 4Z = 0.009 68
Z = 0.009 68 - 0.007 64 = 0.002 04 mol Li3N

140
Chapter 6 - Ionic Bonds and Some Main-Group Chemistry
______________________________________________________________________________

Y = (0.007 64 - 3Z)/2 = [0.007 64 - 3(0.002 04)]/2 = 0.000 76 mol Li2O


mol Li2 O 0.000 76 mol
XLi2 O = = = 0.271
mol Li2 O + mol Li3 N 0.000 76 mol + 0.002 04 mol
XLi3 N = 1.000 - 0.271 = 0.729

141
7 Covalent Bonds and
Molecular Structure

7.1 (a) SiCl4 chlorine EN = 3.0


silicon EN = 1.8
∆EN = 1.2 The Si–Cl bond is polar covalent.

(b) CsBr bromine EN = 2.8


cesium EN = 0.7
∆EN = 2.1 The Cs+Br- bond is ionic.

(c) FeBr3 bromine EN = 2.8


iron EN = 1.8
∆EN = 1.0 The Fe–Br bond is polar covalent.

(d) CH4 carbon EN = 2.5


hydrogen EN = 2.1
∆EN = 0.4 The C–H bond is polar covalent.

7.2 (a) CCl4 chlorine EN = 3.0


carbon EN = 2.5
∆EN = 0.5

(b) BaCl2 chlorine EN = 3.0


barium EN = 0.9
∆EN = 2.1

(c) TiCl3 chlorine EN = 3.0


titanium EN = 1.5
∆EN = 1.5

(d) Cl2O oxygen EN = 3.5


chlorine EN = 3.0
∆EN = 0.5

Increasing ionic character: CCl4 ~ ClO2 < TiCl3 < BaCl2

7.3 H is positively polarized (blue). O is negatively polarized (red). This is consistent with
the electronegativity values for O (3.5) and H (2.1). The more negatively polarized atom
should be the one with the larger electronegativity.

141
Chapter 7 - Covalent Bonds and Molecular Structure
______________________________________________________________________________

7.4 (a) (b)

7.5

7.6 (a) (b) (c)

(d) (e) (f)

7.7

7.8 Molecular formula: C4H5N3O;


7.9

7.10 (a) (b)

(c) (d)

142
Chapter 7 - Covalent Bonds and Molecular Structure
______________________________________________________________________________

7.11 (a) (b) (c)

(d)

7.12

7.13 (a)

(b)

(c)

(d)

7.14

143
Chapter 7 - Covalent Bonds and Molecular Structure
______________________________________________________________________________

7.15 For nitrogen: Isolated nitrogen valence electrons 5


Bound nitrogen bonding electrons 8
Bound nitrogen nonbonding electrons 0
Formal charge = 5 - ½(8) - 0 = +1

For singly bound Isolated oxygen valence electrons 6


oxygen: Bound oxygen bonding electrons 2
Bound oxygen nonbonding electrons 6
Formal charge = 6 - ½(2) - 6 = -1

For doubly bound Isolated oxygen valence electrons 6


oxygen: Bound oxygen bonding electrons 4
Bound oxygen nonbonding electrons 4
Formal charge = 6 - ½(4) - 4 = 0

7.16 (a)
For nitrogen: Isolated nitrogen valence electrons 5
Bound nitrogen bonding electrons 4
Bound nitrogen nonbonding electrons 4
Formal charge = 5 - ½(4) - 4 = -1

For carbon: Isolated carbon valence electrons 4


Bound carbon bonding electrons 8
Bound carbon nonbonding electrons 0
Formal charge = 4 - ½ (8) - 0 = 0

For oxygen: Isolated oxygen valence electrons 6


Bound oxygen bonding electrons 4
Bound oxygen nonbonding electrons 4
Formal charge = 6 - ½(4) - 4 = 0

(b)
For left oxygen: Isolated oxygen valence electrons 6
Bound oxygen bonding electrons 2
Bound oxygen nonbonding electrons 6
Formal charge = 6 - ½(2) - 6 = -1

For central Isolated oxygen valence electrons 6


oxygen: Bound oxygen bonding electrons 6
Bound oxygen nonbonding electrons 2
Formal charge = 6 - ½(6) - 2 = +1

144
Chapter 7 - Covalent Bonds and Molecular Structure
______________________________________________________________________________

For right Isolated oxygen valence electrons 6


oxygen: Bound oxygen bonding electrons 4
Bound oxygen nonbonding electrons 4
Formal charge = 6 - ½(4) - 4 = 0

7.17 Number of Number of


Bonded Atoms Lone Pairs Shape
(a) O3 2 1 bent
(b) H3O+ 3 1 trigonal pyramidal
(c) XeF2 2 3 linear
(d) PF6- 6 0 octahedral
(e) XeOF4 5 1 square pyramidal
(f) AlH4- 4 0 tetrahedral
(g) BF4- 4 0 tetrahedral
(h) SiCl4 4 0 tetrahedral
(i) ICl4- 4 2 square planar
(j) AlCl3 3 0 trigonal planar

7.18

7.19 (a) tetrahedral (b) seesaw

7.20 Each C is sp3 hybridized. The C–C bond is formed by the overlap
of one singly occupied sp3 hybrid orbital from each C. The C–H
bonds are formed by the overlap of one singly occupied sp3 orbital
on C with a singly occupied H 1s orbital.

7.21

145
Chapter 7 - Covalent Bonds and Molecular Structure
______________________________________________________________________________

The carbon in formaldehyde is sp2 hybridized.

7.22

In HCN the carbon is sp hybridized.

7.23 The central I in I3- has two single bonds and three lone pairs of electrons. The
hybridization of the central I is sp3d. A sketch of the ion showing the orbitals involved in
bonding is shown below.

7.24 Single Bonds Lone Pairs Hybridization of S


SF2 2 2 sp3
SF4 4 1 sp3d
SF6 6 0 sp3d2

146
Chapter 7 - Covalent Bonds and Molecular Structure
______________________________________________________________________________

7.25 (a) sp (b) sp3d

7.26 For He2+ σ*1s ↑


σ1s ↑↓
 number of   number of 
  _  
+  bonding electrons   antibonding electrons  2 _ 1
He2 Bond order = = = 1/ 2
2 2
He2+ should be stable with a bond order of 1/2.

7.27 For B2
σ*2p
π*2p
σ2p
π2p ↑ ↑
σ*2s ↑↓
σ2s ↑↓
 number of   number of 
  _  
 bonding electrons   antibonding electrons  4 _ 2
B2 Bond order = = =1
2 2
B2 is paramagnetic because it has two unpaired electrons in the π2p molecular orbitals.

For C2
σ*2p
π*2p
σ2p
π2p ↑↓ ↑↓
σ*2s ↑↓
σ2s ↑↓
6 _2
C2 Bond order = =2; C2 is diamagnetic because all electrons are paired.
2

7.28

7.29 Handed biomolecules have specific shapes that only match complementary-shaped
receptor sites in living systems. The mirror-image forms of the molecules can’t fit into
the receptor sites and thus don’t elicit the same biological response.

7.30 The mirror image of molecule (a) has the same shape as (a) and is identical to it in all
respects so there is no handedness associated with it. The mirror image of molecule (b) is
different than (b) so there is a handedness to this molecule.

147
Chapter 7 - Covalent Bonds and Molecular Structure
______________________________________________________________________________

Understanding Key Concepts

7.31 As the electrostatic potential maps are drawn, the Li and Cl are at the tops of each map.
The red area is for a negatively polarized region (associated with Cl). The blue area is for
a positively polarized region (associated with Li). Map (a) is for CH3Cl and Map (b) is
for CH3Li.

7.32 (a) square pyramidal (b) trigonal pyramidal


(c) square planar (d) trigonal planar

7.33 (a) trigonal bipyramidal (b) tetrahedral


(c) square pyramidal (4 ligands in the horizontal plane, including one hidden)

7.34 Molecular model (c) does not have a tetrahedral central atom. It is square planar.

7.35 (a) sp2 (b) sp3d2 (c) sp3

7.36 (a) C8H9NO2

(b) & (c)

7.37 (a) C13H10N2O4


(b) and (c)

All carbons that have only single bonds are


sp3 hybridized and have a tetrahedral
geometry. All carbons that have double
bonds are sp2 hybridized and have a trigonal
planar geometry. The two nitrogens are sp2
hybridized and have a trigonal planar
geometry.

148
Chapter 7 - Covalent Bonds and Molecular Structure
______________________________________________________________________________

Additional Problems
Electronegativity and Polar Covalent Bonds

7.38 Electronegativity increases from left to right across a period and decreases down a group.

7.39 Z = 119 would be below francium and have a very low electronegativity.

7.40 K < Li < Mg < Pb < C < Br

7.41 Cl > C > Cu > Ca > Cs

7.42 (a) HF fluorine EN = 4.0


hydrogen EN = 2.1
∆EN = 1.9 HF is polar covalent.

(b) HI iodine EN = 2.5


hydrogen EN = 2.1
∆EN = 0.4 HI is polar covalent.

(c) PdCl2 chlorine EN = 3.0


palladium EN = 2.2
∆EN = 0.8 PdCl2 is polar covalent.

(d) BBr3 bromine EN = 2.8


boron EN = 2.0
∆EN = 0.8 BBr3 is polar covalent.

(e) NaOH Na+ – OH- is ionic


OH- oxygen EN = 3.5
hydrogen EN = 2.1
∆EN = 1.4 OH- is polar covalent.

(f) CH3Li lithium EN = 1.0


carbon EN = 2.5
∆EN = 1.5 CH3Li is polar covalent.

7.43 The electronegativity for each element is shown in parentheses.


(a) C (2.5), H (2.1), Cl (3.0): The C–Cl bond is more polar than the C–H bond because
of the larger electronegativity difference between the bonded atoms.
(b) Si (1.8), Li (1.0), Cl (3.0): The Si–Cl bond is more polar than the Si–Li bond because
of the larger electronegativity difference between the bonded atoms.
(c) N (3.0), Cl (3.0), Mg (1.2): The N–Mg bond is more polar than the N–Cl bond
because of the larger electronegativity difference between the bonded atoms.

149
Chapter 7 - Covalent Bonds and Molecular Structure
______________________________________________________________________________

δ_ δ+ δ+ δ_ δ_ δ+ δ+ δ_
7.44 (a) _ _ (b) _ _
C H C Cl Si Li Si Cl
δ_ δ+
(c) N – Cl _
N Mg

δ_ δ+ δ_ δ+ δ_ δ+
7.45 (a) _ (b) _ (c) _
F H I H Cl Pd
δ_ δ+ δ_ δ+
(d) _ (e) _
Br B O H

Electron-Dot Structures and Resonance

7.46 The octet rule states that main-group elements tend to react so that they attain a noble gas
electron configuration with filled s and p sublevels (8 electrons) in their valence electron
shells. The transition metals are characterized by partially filled d orbitals that can be
used to expand their valence shell beyond the normal octet of electrons.

7.47 (a) AlCl3 Al has only 6 electrons around it. (b) PCl5 P has 10 electrons around it.

7.48 (a) (b) (c)

(d) (e) (f)

7.49 (a) (b) (c)

(d) (e)

150
Chapter 7 - Covalent Bonds and Molecular Structure
______________________________________________________________________________

(f)

7.50 (a)

(b)

(c)

7.51 (a)

(b)

(c)

(d)

7.52

7.53 ; CS2 has two double bonds.

7.54 (a) yes (b) yes (c) yes (d) yes

7.55 (a) yes (b) no (c) yes

7.56 (a) The anion has 32 valence electrons. Each Cl has seven valence electrons (28 total).
The minus one charge on the anion accounts for one valence electron. This leaves three
valence electrons for X. X is Al.
(b) The cation has eight valence electrons. Each H has one valence electron (4 total).

151
Chapter 7 - Covalent Bonds and Molecular Structure
______________________________________________________________________________

X is left with four valence electrons. Since this is a cation, one valence electron was
removed from X. X has five valence electrons. X is P.

7.57 (a) This fourth-row element has six valence electrons. It is Se.
(b) This fourth-row element has eight valence electrons. It is Kr.

7.58 (a) (b)

7.59 (a) (b)


Formal Charges

7.60
For carbon: Isolated carbon valence electrons 4
Bound carbon bonding electrons 6
Bound carbon nonbonding electrons 2
Formal charge = 4 - ½(6) - 2 = -1

For oxygen: Isolated oxygen valence electrons 6


Bound oxygen bonding electrons 6
Bound oxygen nonbonding electrons 2
Formal charge = 6 - ½(6) - 2 = +1

7.61 (a)
For hydrogen: Isolated hydrogen valence electrons 1
Bound hydrogen bonding electrons 2
Bound hydrogen nonbonding electrons 0
Formal charge = 1 - ½(2) - 0 = 0

For nitrogen: Isolated nitrogen valence electrons 5


Bound nitrogen bonding electrons 6
Bound nitrogen nonbonding electrons 2
Formal charge = 5 - ½(6) - 2 = 0

For oxygen: Isolated oxygen valence electrons 6


Bound oxygen bonding electrons 4

152
Chapter 7 - Covalent Bonds and Molecular Structure
______________________________________________________________________________

Bound oxygen nonbonding electrons 4


Formal charge = 6 - ½(4) - 4 = 0

(b)
For hydrogen: Isolated hydrogen valence electrons 1
Bound hydrogen bonding electrons 2
Bound hydrogen nonbonding electrons 0
Formal charge = 1 - ½(2) - 0 = 0

For nitrogen: Isolated nitrogen valence electrons 5


Bound nitrogen bonding electrons 4
Bound nitrogen nonbonding electrons 4
Formal charge = 5 - ½(4) - 4 = -1

For carbon: Isolated carbon valence electrons 4


Bound carbon bonding electrons 8
Bound carbon nonbonding electrons 0
Formal charge = 4 - ½(8) - 0 = 0

(c)
For chlorine: Isolated chlorine valence electrons 7
Bound chlorine bonding electrons 2
Bound chlorine nonbonding electrons 6
Formal charge = 7 - ½(2) - 6 = 0
For oxygen: Isolated oxygen valence electrons 6
Bound oxygen bonding electrons 2
Bound oxygen nonbonding electrons 6
Formal charge = 6 - ½(2) - 6 = -1

For phosphorus: Isolated phosphorus valence electrons 5


Bound phosphorus bonding electrons 8
Bound phosphorus nonbonding electrons 0
Formal charge = 5 - ½(8) - 0 = +1

7.62
For both oxygens: Isolated oxygen valence electrons 6

153
Chapter 7 - Covalent Bonds and Molecular Structure
______________________________________________________________________________

Bound oxygen bonding electrons 2


Bound oxygen nonbonding electrons 6
Formal charge = 6 - ½(2) - 6 = -1

For chlorine: Isolated chlorine valence electrons 7


Bound chlorine bonding electrons 4
Bound chlorine nonbonding electrons 4
Formal charge = 7 - ½(4) - 4 = +1

For left oxygen: Isolated oxygen valence electrons 6


Bound oxygen bonding electrons 2
Bound oxygen nonbonding electrons 6
Formal charge = 6 - ½(2) - 6 = -1

For right oxygen: Isolated oxygen valence electrons 6


Bound oxygen bonding electrons 4
Bound oxygen nonbonding electrons 4
Formal charge = 6 - ½(4) - 4 = 0

For chlorine: Isolated chlorine valence electrons 7


Bound chlorine bonding electrons 6
Bound chlorine nonbonding electrons 4
Formal charge = 7 - ½(6) - 4 = 0

7.63
For sulfur: Isolated sulfur valence electrons 6
Bound sulfur bonding electrons 8
Bound sulfur nonbonding electrons 2
Formal charge = 6 - ½(8) - 2 = 0

For doubly Isolated oxygen valence electrons 6


bound oxygen: Bound oxygen bonding electrons 4
Bound oxygen nonbonding electrons 4
Formal charge = 6 - ½(4) - 4 = 0

For oxygen Isolated oxygen valence electrons 6


bound to Bound oxygen bonding electrons 4
hydrogen: Bound oxygen nonbonding electrons 4
Formal charge = 6 - ½(4) - 4 = 0

154
Chapter 7 - Covalent Bonds and Molecular Structure
______________________________________________________________________________

For hydrogen: Isolated hydrogen valence electrons 1


Bound hydrogen bonding electrons 2
Bound hydrogen nonbonding electrons 0
Formal charge = 1 - ½(2) - 0 = 0

For sulfur: Isolated sulfur valence electrons 6


Bound sulfur bonding electrons 6
Bound sulfur nonbonding electrons 2
Formal charge = 6 - ½(6) - 2 = +1

For oxygen not Isolated oxygen valence electrons 6


bound to Bound oxygen bonding electrons 2
hydrogen: Bound oxygen nonbonding electrons 6
Formal charge = 6 - ½(2) - 6 = - 1

For oxygen Isolated oxygen valence electrons 6


bound Bound oxygen bonding electrons 4
to hydrogen: Bound oxygen nonbonding electrons 4
Formal charge = 6 - ½(4) - 4 = 0

For hydrogen: Isolated hydrogen valence electrons 1


Bound hydrogen bonding electrons 2
Bound hydrogen nonbonding electrons 0
Formal charge = 1 - ½(2) - 0 = 0

7.64 (a)
For hydrogen: Isolated hydrogen valence electrons 1
Bound hydrogen bonding electrons 2
Bound hydrogen nonbonding electrons 0
Formal charge = 1 - ½(2) - 0 = 0

For nitrogen: Isolated nitrogen valence electrons 5


(central) Bound nitrogen bonding electrons 8
Bound nitrogen nonbonding electrons 0
Formal charge = 5 - ½(8) - 0 = +1

For nitrogen: Isolated nitrogen valence electrons 5


(terminal) Bound nitrogen bonding electrons 4
Bound nitrogen nonbonding electrons 4
Formal charge = 5 - ½(4) - 4 = -1

155
Chapter 7 - Covalent Bonds and Molecular Structure
______________________________________________________________________________

For carbon: Isolated carbon valence electrons 4


Bound carbon bonding electrons 8
Bound carbon nonbonding electrons 0
Formal charge = 4 - ½(8) - 0 = 0

(b)
For hydrogen: Isolated hydrogen valence electrons 1
Bound hydrogen bonding electrons 2
Bound hydrogen nonbonding electrons 0
Formal charge = 1 - ½(2) - 0 = 0

For nitrogen: Isolated nitrogen valence electrons 5


(central) Bound nitrogen bonding electrons 6
Bound nitrogen nonbonding electrons 2
Formal charge = 5 - ½(6) - 2 = 0

For nitrogen: Isolated nitrogen valence electrons 5


(terminal) Bound nitrogen bonding electrons 4
Bound nitrogen nonbonding electrons 4
Formal charge = 5 - ½(4) - 4 = -1

For carbon: Isolated carbon valence electrons 4


Bound carbon bonding electrons 6
Bound carbon nonbonding electrons 0
Formal charge = 4 - ½(6) - 0 = +1

Structure (a) is more important because of the octet of electrons around carbon.

7.65
For oxygen: Isolated oxygen valence electrons 6
Bound oxygen bonding electrons 4
Bound oxygen nonbonding electrons 4
Formal charge = 6 - ½(4) - 4 = 0

For left carbon: Isolated carbon valence electrons 4


Bound carbon bonding electrons 8
Bound carbon nonbonding electrons 0
Formal charge = 4 - ½(8) - 0 = 0

156
Chapter 7 - Covalent Bonds and Molecular Structure
______________________________________________________________________________

For right carbon: Isolated carbon valence electrons 4


Bound carbon bonding electrons 6
Bound carbon nonbonding electrons 2
Formal charge = 4 - ½(6) - 2 = -1

For oxygen: Isolated oxygen valence electrons 6


Bound oxygen bonding electrons 2
Bound oxygen nonbonding electrons 6
Formal charge = 6 - ½(2) - 6 = -1

For left carbon: Isolated carbon valence electrons 4


Bound carbon bonding electrons 8
Bound carbon nonbonding electrons 0
Formal charge = 4 - ½(8) - 0 = 0

For right carbon: Isolated carbon valence electrons 4


Bound carbon bonding electrons 8
Bound carbon nonbonding electrons 0
Formal charge = 4 - ½(8) - 0 = 0

The second structure is more important because of the -1 formal charge on the more
electronegative oxygen.

The VSEPR Model

7.66 From data in Table 7.4:


(a) trigonal planar (b) trigonal bipyramidal (c) linear (d) octahedral

7.67 From data in Table 7.4:


(a) T shaped (b) bent (c) square planar

7.68 From data in Table 7.4:


(a) tetrahedral, 4 (b) octahedral, 6 (c) bent, 3 or 4
(d) linear, 2 or 5 (e) square pyramidal, 6 (f) trigonal pyramidal, 4

7.69 From data in Table 7.4:


(a) seesaw, 5 (b) square planar, 6 (c) trigonal bipyramidal, 5
(d) T shaped, 5 (e) trigonal planar, 3 (f) linear, 2 or 5

157
Chapter 7 - Covalent Bonds and Molecular Structure
______________________________________________________________________________

7.70 Number of Number of


Bonded Atoms Lone Pairs Shape
(a) H2Se 2 2 bent
(b) TiCl4 4 0 tetrahedral
(c) O3 2 1 bent
(d) GaH3 3 0 trigonal planar

7.71 Number of Number of


Bonded Atoms Lone Pairs Shape
(a) XeO4 4 0 tetrahedral
(b) SO2Cl2 4 0 tetrahedral
(c) OsO4 4 0 tetrahedral
(d) SeO2 2 1 bent

7.72 Number of Number of


Bonded Atoms Lone Pairs Shape
(a) SbF5 5 0 trigonal bipyramidal
(b) IF4+ 4 1 see saw
(c) SeO32- 3 1 trigonal pyramidal
(d) CrO42- 4 0 tetrahedral

7.73 Number of Number of


Bonded Atoms Lone Pairs Shape
(a) NO3- 3 0 trigonal planar
+
(b) NO2 2 0 linear
(c) NO2- 2 1 bent
7.74 Number of Number of
Bonded Atoms Lone Pairs Shape
(a) PO43- 4 0 tetrahedral
-
(b) MnO4 4 0 tetrahedral
(c) SO42- 4 0 tetrahedral
2-
(d) SO3 3 1 trigonal pyramidal
(e) ClO4- 4 0 tetrahedral
-
(f) SCN 2 0 linear
(C is the central atom)

7.75 Number of Number of


Bonded Atoms Lone Pairs Shape
(a) XeF3+ 3 2 T shaped
(b) SF3+ 3 1 trigonal pyramidal
(c) ClF2+ 2 2 bent

158
Chapter 7 - Covalent Bonds and Molecular Structure
______________________________________________________________________________

(d) CH3+ 3 0 trigonal planar

7.76 (a) In SF2 the sulfur is bound to two fluorines and contains two lone pairs of electrons.
SF2 is bent and the F–S–F bond angle is approximately 109°.
(b) In N2H2 each nitrogen is bound to the other nitrogen and one hydrogen. Each
nitrogen has one lone pair of electrons. The H–N–N bond angle is approximately 120°.
(c) In KrF4 the krypton is bound to four fluorines and contains two lone pairs of
electrons. KrF4 is square planar, and the F–Kr–F bond angle is 90°.
(d) In NOCl the nitrogen is bound to one oxygen and one chlorine and contains one lone
pair of electrons. NOCl is bent, and the Cl–N–O bond angle is approximately 120°.

7.77 (a) In PCl6- the phosphorus is bound to six chlorines. There are no lone pairs of electrons
on the phosphorus. PCl6- is octahedral, and the Cl–P–Cl bond angle is 90o.
(b) In ICl2- the iodine is bound to two chlorines and contains three lone pairs of electrons.
ICl2- is linear, and the Cl–I–Cl bond angle is 180o.
(c) In SO42- the sulfur is bound to four oxygens. There are no lone pairs of electrons on
the sulfur. SO42- is tetrahedral, and the O–S–O bond angle is 109.5o.
(d) In BO33- the boron is bound to three oxygens. There are no lone pairs of electrons on
the boron. BO33- is trigonal planar, and the O–B–O bond angle is 120o.

7.78 H – Ca – H ~ 120o Cb – Cc – N 180o


H – Ca – C b ~ 120o Ca – Cb – H ~ 120o
Ca – Cb – Cc ~ 120o H – Cb – C c ~ 120o

7.79
7.80 All six carbons in cyclohexane are bonded to two other carbons and two hydrogens (i.e.
four charge clouds). The geometry about each carbon is tetrahedral with a C–C–C bond
angle of approximately 109°. Because the geometry about each carbon is tetrahedral, the
cyclohexane ring cannot be flat.

7.81 All six carbon atoms are sp2 hybridized and the bond angles are ~120o. The geometry
about each carbon is trigonal planar.

Hybrid Orbitals and Molecular Orbital Theory

7.82 In a π bond, the shared electrons occupy a region above and below a line connecting the
two nuclei. A σ bond has its shared electrons located along the axis between the two

159
Chapter 7 - Covalent Bonds and Molecular Structure
______________________________________________________________________________

nuclei.

7.83 Electrons in a bonding molecular orbital spend most of their time in the region between
the two nuclei, helping to bond the atoms together. Electrons in an antibonding
molecular orbital cannot occupy the central region between the nuclei and cannot
contribute to bonding.

7.84 See Table 7.5.


(a) sp (b) sp3d (c) sp3d2 (d) sp3

7.85 See Table 7.5.


(a) tetrahedral (b) octahedral (c) linear

7.86 See Table 7.5.


(a) sp3 (b) sp3d2 (c) sp2 or sp3 (d) sp or sp3d (e) sp3d2

7.87 See Tables 7.4 and 7.5.


(a) seesaw, 5 charge clouds, sp3d
(b) square planar, 6 charge clouds, sp3d2
(c) trigonal bipyramidal, 5 charge clouds, sp3d
(d) T shaped, 5 charge clouds, sp3d
(e) trigonal planar, 3 charge clouds, sp2

7.88 (a) sp2 (b) sp3 (c) sp3d2 (d) sp2

7.89 (a) sp3 (b) sp2 (c) sp2 (d) sp3

7.90 The C is sp2 hybridized and the N atoms are sp3 hybridized.

7.91 (a)
(b) H–C–H, ~109o; O–C–O, ~120o; H–N–H, ~107o
(c) N, sp3; left C, sp3; right C, sp2

7.92 σ*2p
π*2p ↑ ↑ ↑ ↑↓ ↑
π2p ↑↓ ↑↓ ↑↓ ↑↓ ↑↓ ↑↓
σ2p ↑↓ ↑↓ ↑↓

160
Chapter 7 - Covalent Bonds and Molecular Structure
______________________________________________________________________________

σ*2s ↑↓ ↑↓ ↑↓
σ2s ↑↓ ↑↓ ↑↓
O2+ O2 O2-

 number of   number of 
  _  
 bonding electrons   antibonding electrons 
Bond order =
2
+ 8 _ 3 8_4
O2 bond order = = 2.5 ; O2 bond order = =2
2 2
_ 8_5
O2 bond order = = 1.5
2
All are stable with bond orders between 1.5 and 2.5. All have unpaired electrons.

7.93 σ*2p
π*2p ↑
σ2p ↑ ↑↓ ↑↓
π2p ↑↓ ↑↓ ↑↓ ↑↓ ↑↓ ↑↓
σ*2s ↑↓ ↑↓ ↑↓
σ2s ↑↓ ↑↓ ↑↓
N2+ N2 N2-

 number of   number of 
  _  
 bonding electrons   antibonding electrons 
Bond order =
2
+ 7_2 8_2
N2 bond order = = 2.5 ; N 2 bond order = =3
2 2
_ 8_3
N2 bond order = = 2.5
2
All are stable with bond orders of either 3 or 2.5. N2+ and N2- contain unpaired electrons.

7.94 p orbitals in allyl cation

allyl cation showing only the σ bonds (each C is sp2 hybridized)

161
Chapter 7 - Covalent Bonds and Molecular Structure
______________________________________________________________________________

delocalized MO model for π bonding in the allyl cation

7.95 p orbitals in NO2-

NO2- showing only the σ bonds (N is sp2 hybridized)

delocalized MO model for π bonding in NO2-

General Problems

7.96

7.97 In ascorbic acid (Problem 7.96) all carbons that have only single bonds are sp3
hybridized. The three carbons that have double bonds are sp2 hybridized.
7.98 Every carbon is sp2 hybridized. There are 18 σ bonds and 5 π bonds.

7.99

162
Chapter 7 - Covalent Bonds and Molecular Structure
______________________________________________________________________________

For
For oxygen: Isolated oxygen valence electrons 6
(top) Bound oxygen bonding electrons 2
Bound oxygen nonbonding electrons 6
Formal charge = 6 - ½(2) - 6 = -1

For oxygen: Isolated oxygen valence electrons 6


(middle) Bound oxygen bonding electrons 4
Bound oxygen nonbonding electrons 4
Formal charge = 6 - ½(4) - 4 = 0

For oxygen: Isolated oxygen valence electrons 6


(left) Bound oxygen bonding electrons 4
Bound oxygen nonbonding electrons 4
Formal charge = 6 - ½(4) - 4 = 0

For oxygen: Isolated oxygen valence electrons 6


(right) Bound oxygen bonding electrons 2
Bound oxygen nonbonding electrons 6
Formal charge = 6 - ½(2) - 6 = -1

For sulfur: Isolated sulfur valence electrons 6


Bound sulfur bonding electrons 8
Bound sulfur nonbonding electrons 0
Formal charge = 6 - ½(8) - 0 = +2

For
For oxygen: Isolated oxygen valence electrons 6
(top) Bound oxygen bonding electrons 2
Bound oxygen nonbonding electrons 6
Formal charge = 6 - ½(2) - 6 = -1
For oxygen: Isolated oxygen valence electrons 6
(middle) Bound oxygen bonding electrons 4
Bound oxygen nonbonding electrons 4
Formal charge = 6 - ½(4) - 4 = 0

For oxygen: Isolated oxygen valence electrons 6


(left) Bound oxygen bonding electrons 2
Bound oxygen nonbonding electrons 6

163
Chapter 7 - Covalent Bonds and Molecular Structure
______________________________________________________________________________

Formal charge = 6 - ½(2) - 6 = -1

For oxygen: Isolated oxygen valence electrons 6


(right) Bound oxygen bonding electrons 4
Bound oxygen nonbonding electrons 4
Formal charge = 6 - ½(4) - 4 = 0

For sulfur: Isolated sulfur valence electrons 6


Bound sulfur bonding electrons 8
Bound sulfur nonbonding electrons 0
Formal charge = 6 - ½(8) - 0 = +2

For
For oxygen: Isolated oxygen valence electrons 6
(top) Bound oxygen bonding electrons 2
Bound oxygen nonbonding electrons 6
Formal charge = 6 - ½(2) - 6 = -1

For oxygen: Isolated oxygen valence electrons 6


(middle) Bound oxygen bonding electrons 6
Bound oxygen nonbonding electrons 2
Formal charge = 6 - ½(6) - 2 = +1

For oxygen: Isolated oxygen valence electrons 6


(left) Bound oxygen bonding electrons 2
Bound oxygen nonbonding electrons 6
Formal charge = 6 - ½(2) - 6 = -1

For oxygen: Isolated oxygen valence electrons 6


(right) Bound oxygen bonding electrons 2
Bound oxygen nonbonding electrons 6
Formal charge = 6 - ½(2) - 6 = -1

For sulfur: Isolated sulfur valence electrons 6


Bound sulfur bonding electrons 8
Bound sulfur nonbonding electrons 0
Formal charge = 6 - ½(8) - 0 = +2

164
Chapter 7 - Covalent Bonds and Molecular Structure
______________________________________________________________________________

7.100

For
For hydrogen: Isolated hydrogen valence electrons 1
Bound hydrogen bonding electrons 2
Bound hydrogen nonbonding electrons 0
Formal charge = 1 - ½(2) - 0 = 0

For carbon: Isolated carbon valence electrons 4


(left) Bound carbon bonding electrons 8
Bound carbon nonbonding electrons 0
Formal charge = 4 - ½(8) - 0 = 0

For nitrogen: Isolated nitrogen valence electrons 5


Bound nitrogen bonding electrons 6
Bound nitrogen nonbonding electrons 2
Formal charge = 5 - ½(6) - 2 = 0

For carbon: Isolated carbon valence electrons 4


(right) Bound carbon bonding electrons 8
Bound carbon nonbonding electrons 0
Formal charge = 4 - ½(8) - 0 = 0

For oxygen: Isolated oxygen valence electrons 6


Bound oxygen bonding electrons 4
Bound oxygen nonbonding electrons 4
Formal charge = 6 - ½(4) - 4 = 0

For
For hydrogen: Isolated hydrogen valence electrons 1
Bound hydrogen bonding electrons 2
Bound hydrogen nonbonding electrons 0
Formal charge = 1 - ½(2) - 0 = 0
For carbon: Isolated carbon valence electrons 4
(left) Bound carbon bonding electrons 8
Bound carbon nonbonding electrons 0
Formal charge = 4 - ½(8) - 0 = 0

165
Chapter 7 - Covalent Bonds and Molecular Structure
______________________________________________________________________________

For nitrogen: Isolated nitrogen valence electrons 5


Bound nitrogen bonding electrons 8
Bound nitrogen nonbonding electrons 0
Formal charge = 5 - ½(8) - 0 = +1

For carbon: Isolated carbon valence electrons 4


(right) Bound carbon bonding electrons 8
Bound carbon nonbonding electrons 0
Formal charge = 4 - ½(8) - 0 = 0

For oxygen: Isolated oxygen valence electrons 6


Bound oxygen bonding electrons 2
Bound oxygen nonbonding electrons 6
Formal charge = 6 - ½(2) - 6 = -1

7.101 They are geometric isomers not resonance forms. In


resonance forms the atoms have the same
geometrical arrangement.

7.102 (a)
For boron: Isolated boron valence electrons 3
Bound boron bonding electrons 8
Bound boron nonbonding electrons 0
Formal charge = 3 – ½(8) – 0 = -1

For oxygen: Isolated oxygen valence electrons 6


Bound oxygen bonding electrons 6
Bound oxygen nonbonding electrons 2
Formal charge = 6 – ½ (6) – 2 = +1

(b) In BF3 the B has three bonding pairs of electrons and no lone pairs. The B is sp2
hybridized and BF3 is trigonal planar.
is bent about the oxygen because of two bonding pairs and two lone
pairs of electrons. The O is sp3 hybridized.
In the product, B is sp3 hybridized (with four bonding pairs of electrons), and the
geometry about it is tetrahedral. The O is also sp3 hybridized (with three bonding pairs
and one lone pair of electrons), and the geometry about it is trigonal pyramidal.
7.103 Both the B and N are sp2 hybridized. All bond angles are ~120o. The overall geometry of

166
Chapter 7 - Covalent Bonds and Molecular Structure
______________________________________________________________________________

the molecule is planar.

7.104 The triply bonded carbon atoms are sp hybridized. The theoretical bond angle for C–C≡C
is 180o. Benzyne is so reactive because the C–C≡C bond angle is closer to 120o and is
very strained.

7.105 (a) (b) (c)

7.106

7.107 Li2 σ*2s


σ2s ↑↓
σ*1s ↑↓
σ1s ↑↓
 number of   number of 
  _  
 bonding electrons   antibonding electrons  4 _ 2
Li2 Bond order = = =1
2 2
The bond order for Li2 is 1, and the molecule is likely to be stable.

7.108 C22-
σ*2p
π*2p
σ2p ↑↓
π2p ↑↓ ↑↓
σ*2s ↑↓
σ2s ↑↓
 number of   number of 
  _  
 bonding electrons   antibonding electrons 
Bond order =
2
2_ 8 _ 2
C2 bond order = = 3 ; there is a triple bond between the two carbons.
2

7.109 (a) (b) (c)

Structure (a) is different from structures (b) and (c) because both chlorines are on the
same carbon. Structures (b) and (c) are different because in (b) both chlorines are on the
same side of the molecule (“cis”) and in (c) they are on opposite sides of the molecule

167
Chapter 7 - Covalent Bonds and Molecular Structure
______________________________________________________________________________

(“trans”). There is no rotation around the carbon-carbon double bond.


7.110 CH4(g) + Cl2(g) → CH3Cl(g) + HCl(g)
Energy change = D (Reactant bonds) - D (Product bonds)
Energy change = [4 DC-H + DCl-Cl] - [3 DC-H + DC-Cl + DH-Cl]
Energy change = [(4 mol)(410 kJ/mol) + (1 mol)(243 kJ/mol]
- [(3 mol)(410 kJ/mol) + (1 mol)(330 kJ/mol) + (1 mol)(432 kJ/mol)] = -109 kJ

7.111

7.112 (a) (b) (c)

(d) (e) (f)

(g) (h)

Structures (a) – (d) make more important contributions to the resonance hybrid because of
only -1 and 0 formal charges on the oxygens.

7.113 (a) (1) (2) (3)


(b) Structure (1) makes the greatest contribution to the resonance hybrid because of the
-1 formal charge on the oxygen. Structure (3) makes the least contribution to the
resonance hybrid because of the +1 formal charge on the oxygen.
(c) and (d) OCN- is linear because the C has 2 charge clouds. It is sp hybridized in all
three resonance structures. It forms two π bonds.

168
Chapter 7 - Covalent Bonds and Molecular Structure
______________________________________________________________________________

7.114

21 σ bonds
5 π bonds
Each C with a double bond is sp2 hybridized.
The –CH3 carbon is sp3 hybridized.

7.115

7.116 (a) σ*3p


π*3p ↑ ↑ ↑↓ ↑↓
π3p ↑↓ ↑↓ ↑↓ ↑↓
σ3p ↑↓ ↑↓
σ*3s ↑↓ ↑↓
σ3s ↑↓ ↑↓
S2 S22-

(b) S2 would be paramagnetic with two unpaired electrons in the π*3p MOs.

 number of   number of 
  _  
 bonding electrons   antibonding electrons 
(c) Bond order =
2
8_4
S2 bond order = =2
2
2_ 8_6
(d) S2 bond order = =1
2
The two added electrons go into the antibonding π*3p MOs, the bond order drops from 2
to 1, and the bond length in S22- should be longer than the bond length in S2.

169
Chapter 7 - Covalent Bonds and Molecular Structure
______________________________________________________________________________

7.117 (a) CO
σ*2p
π*2p
σ2p ↑↓
π2p ↑↓ ↑↓
σ*2s ↑↓
σ2s ↑↓

(b) All electrons are paired, CO is diamagnetic.

(c)
 number of   number of 
  _  
 bonding electrons   antibonding electrons 
Bond order =
2
8_2
CO bond order = =3
2
The bond order here matches that predicted by the electron-dot structure ( ).

(d)

7.118 (a)
The left S has 5 electron clouds (4 bonding, 1 lone pair). The S is sp3d hybridized and the
geometry about this S is seesaw. The right S has 4 electron clouds (2 bonding, 2 lone
pairs). The S is sp3 hybridized and the geometry about this S is bent.

(b)
The left C has 4 electron clouds (4 bonding, 0 lone pairs). This C is sp3 hybridized and
its geometry is tetrahedral. The right C has 3 electron clouds (3 bonding, 0 lone pairs).
This C is sp2 hybridized and its geometry is trigonal planar. The central two C’s have 2
electron clouds (2 bonding, 0 lone pairs). These C’s are sp hybridized and the geometry
about both is linear.

170
Chapter 7 - Covalent Bonds and Molecular Structure
______________________________________________________________________________

7.119

Multi-Concept Problems

7.120 (a)
(b) The oxygen in OH has a half-filled 2p orbital that can accept the additional electron.
For a 2p orbital, n = 2 and l = 1.
(c) The electron affinity for OH is slightly more negative than for an O atom because
when OH gains an additional electron, it achieves an octet configuration.

7.121 (a) (4 orbitals)(3 electrons) = 12 outer-shell electrons


(b) 3 electrons
(c) 1s3 2s3 2p6;
(d)
(e) σ*2p
π*2p ↑↓ ↑
π2p ↑↓↑ ↑↓↑
σ2p ↑↓↑
σ*2s ↑↓↑
σ2s ↑↓↑

 number of   number of 
  _  
 bonding electrons   antibonding electrons 
Bond order =
3
12 _ 6
X2 bond order = =2
3

171
Chapter 7 - Covalent Bonds and Molecular Structure
______________________________________________________________________________

7.122 (a)

(b) Each Cr atom has 6 pairs of electrons around it. The likely geometry about each Cr
atom is tetrahedral because each Cr has 4 charge clouds.

7.123 (a) XOCl2 + 2 H2O → 2 HCl + H2XO3


(b) 96.1 mL = 0.0961 L
mol NaOH = (0.1225 mol/L)(0.0961 L) = 0.01177 mol NaOH
1 mol H +
mol H+ = 0.01177 mol NaOH x = 0.01177 mol H+
1 mol NaOH
Of the total H+ concentration, half comes from HCl and half comes from H2XO3.
0.01177 mol H + 1 mol H 2 XO3
mol H2XO3 = x = 2.943 x 10-3 mol H2XO3
2 2 mol H +
1 mol XOCl2
mol XOCl2 = 2.943 x 10-3 mol H2XO3 x = 2.943 x 10-3 mol XOCl2
1 mol H 2 XO3
0.350 g XOCl2
molar mass XOCl2 = = 118.9 g/mol
2.943 x 10_ 3 mol XOCl2
molecular mass of XOCl2 = 118.9 amu
atomic mass of X = 118.9 amu - 16.0 amu - 2(35.45 amu) = 32.0 amu: X = S

(c)

(d) trigonal pyramidal

7.124 (a)

(b) All three molecules are planar. The first two structures are polar because they both
have an unsymmetrical distribution of atoms about the center of the molecule (the middle
of the double bond), and bond polarities do not cancel. Structure 3 is nonpolar because
the H’s and Cl’s, respectively, are symmetrically distributed about the center of the
molecule, both being opposite each other. In this arrangement, bond polarities cancel.
(c) 200 nm = 200 x 10-9 m
h c (6.626 x 10_ 34 J • s) (3.00 x 108 m / s)
E= = (6.022 x 1023 / mol)
λ 200 x 10_ 9 m
E = 5.99 x 105 J/mol = 599 kJ/mol

172
Chapter 7 - Covalent Bonds and Molecular Structure
______________________________________________________________________________

(d)
The π bond must be broken before rotation can occur.

173
8 Thermochemistry: Chemical Energy

453.59 g 1 kg
8.1 Convert lb to kg. 2300 lb x x = 1043 kg
1 lb 1000 g
mi 1 km 1000 m 1h
Convert mi/h to m/s. 55 x x x = 24.6 m/s
h 0.62137 mi 1 km 3600 s
1 kg⋅ m2/s2 = 1 J; E = ½mv2 = ½(1043 kg)(24.6 m/s)2 = 3.2 x 105 J
1 kJ
E = 3.2 x 105 J x = 3.2 x 102 kJ
1000 J

8.2 (a) and (b) are state functions; (c) is not.

8.3 ∆V = (4.3 L - 8.6 L) = - 4.3 L


w = -P∆V = -(44 atm)( - 4.3 L) = +189.2 L⋅ atm
J
w = (189.2 L⋅ atm)(101 ) = +1.9 x 104 J
L • atm
The positive sign for the work indicates that the surroundings does work on the system.
Energy flows into the system.

8.4 w = -P∆V = - (2.5 atm)(3 L - 2 L) = - 2.5 L⋅atm


 J 
w = (-2.5 L⋅atm) 101  = -252.5 J = -250 J = -0.25 kJ
 L• atm 
The negative sign indicates that the expanding system loses work energy and does work
on the surroundings.

8.5 (a) w = -P∆V is positive and P∆V is negative for this reaction because the system
volume is decreased at constant pressure.
(b) P∆V is small compared to ∆E.
∆H = ∆E + P∆V; ∆H is negative. Its value is slightly more negative than ∆E.

kJ
8.6 ∆Ho = - 484
2 mol H 2
P∆V = (1.00 atm)(-5.6 L) = -5.6 L⋅ atm
J
P∆V = (-5.6 L⋅ atm)(101 ) = -565.6 J = -570 J = -0.57 kJ
L • atm
w = -P∆V = 570 J = 0.57 kJ
_ 121 kJ
∆H =
0.50 mol H 2
∆E = ∆H - P∆V = -121 kJ - (-0.57 kJ) = -120.43 kJ = -120 kJ
8.7 ∆V = 448 L and assume P = 1.00 atm

173
Chapter 8 - Thermochemistry: Chemical Energy
______________________________________________________________________________

w = -P∆V = -(1.00 atm)(448 L) = - 448 L⋅ atm


J
w = -(448 L⋅ atm)(101 ) = - 4.52 x 104 J
L • atm
1 kJ
w = - 4.52 x 104 J x = - 45.2 kJ
1000 J

8.8 (a) C3H8, 44.10 amu; ∆Ho = -2219 kJ/mol C3H8


1 mol C3 H8 - 2219 kJ
15.5 g x x = -780. kJ
44.10 g C3 H8 1 mol C3 H8
780. kJ of heat is evolved.

(b) Ba(OH)2 ⋅ 8 H2O, 315.5 amu; ∆Ho = +80.3 kJ/mol Ba(OH)2 ⋅ 8 H2O
1 mol Ba(OH )2 • 8 H 2 O 80.3 kJ
4.88 g x x = +1.24 kJ
315.5 g Ba(OH )2 • 8 H 2 O 1 mol Ba(OH )2 • 8 H2 O
1.24 kJ of heat is absorbed.

8.9 CH3NO2, 61.04 amu


1 mol CH3 NO2 2441.6 kJ
q = 100.0 g CH3 NO2 x x = 1.000 x 103 kJ
61.04 g CH3 NO2 4 mol CH3 NO2

J
8.10 q = (specific heat) x m x ∆T = (4.18 )(350 g)(3oC - 25oC) = -3.2 x 104 J
g • oC
1 kJ
q = -3.2 x 104 J x = -32 kJ
1000 J
q 96 J
8.11 q = (specific heat) x m x ∆T; specific heat = = = 0.13 J/(g ⋅ oC)
m x ∆ T (75 g)(10 C)
o

8.12 25.0 mL = 0.0250 L and 50.0 mL = 0.0500 L


mol H2SO4 = (1.00 mol/L)(0.0250 L) = 0.0250 mol H2SO4
mol NaOH = (1.00 mol/L)(0.0500 L) = 0.0500 mol NaOH
NaOH and H2SO4 are present in a 2:1 mol ratio. This matches the stoichiometric ratio in
the balanced equation.
q = (specific heat) x m x ∆T
m = (25.0 mL + 50.0 mL)(1.00 g/mL) = 75.0 g
J
q = (4.18 )(75.0 g)(33.9 o C _ 25.0 o C) = 2790 J
g• oC
mol
mol H2SO4 = 0.0250 L x 1.00 H2SO4 = 0.0250 mol H2SO4
L
2.79 x 103 J
Heat evolved per mole of H2SO4 = = 1.1 x 105 J / mol H 2 SO 4
0.0250 mol H 2 SO 4
Because the reaction evolves heat, the sign for ∆H is negative.

174
Chapter 8 - Thermochemistry: Chemical Energy
______________________________________________________________________________

1 kJ
∆H = -1.1 x 105 J x = -1.1 x 102 kJ
1000 J

8.13 CH4(g) + Cl2(g) → CH3Cl(g) + HCl(g) ∆Ho1 = -98.3 kJ


CH3Cl(g) + Cl2(g) → CH2Cl2(g) + HCl(g) ∆Ho2 = -104 kJ
Sum CH4(g) + 2 Cl2(g) → CH2Cl2(g) + 2 HCl(g)
∆Ho = ∆Ho1 + ∆Ho2 = -202 kJ

8.14 (a) A + 2 B → D; ∆Ho = -100 kJ + (-50 kJ) = -150 kJ


(b) The red arrow corresponds to step 1: A + B → C
The green arrow corresponds to step 2: C + B → D
The blue arrow corresponds to the overall reaction.
(c) The top energy level represents A + 2 B.
The middle energy level represents C + B.
The bottom energy level represents D.

8.15

8.16 4 NH3(g) + 5 O2(g) → 4 NO(g) + 6 H2O(g)


∆Horxn = [4 ∆Hof (NO) + 6 ∆Hof (H2O)] - [4 ∆Hof (NH3)]
∆Horxn = [(4 mol)(90.2 kJ/mol) + (6 mol)(- 241.8 kJ/mol)] - [(4 mol)(- 46.1 kJ/mol)]
∆Horxn = -905.6 kJ

8.17 6 CO2(g) + 6 H2O(l) → C6H12O6(s) + 6 O2(g)


∆Horxn = ∆Hof(C6H12O6) - [6 ∆Hof(CO2) + 6 ∆Hof(H2O(l))]
∆Horxn = [(1 mol)(-1260 kJ/mol)] - [(6 mol)(-393.5 kJ/mol) + (6 mol)(-285.8 kJ/mol)]
∆Horxn = +2815.8 kJ = +2816 kJ

8.18 H2C=CH2(g) + H2O(g) → C2H5OH(g)


∆Horxn = D (Reactant bonds) - D (Product bonds)
∆Horxn = (DC=C + 4 DC-H + 2 DO-H) - (DC-C + DC-O + 5 DC-H + DO-H)
∆Horxn = [(1 mol)(611 kJ/mol) + (4 mol)(410 kJ/mol) + (2 mol)(460 kJ/mol)]
- [(1 mol)(350 kJ/mol) + (1 mol)( 350 kJ/mol) + (5 mol)(410 kJ/mol) + (1 mol)(460 kJ/mol)]
∆Horxn = -39 kJ
8.19 2 NH3(g) + Cl2(g) → N2H4(g) + 2 HCl(g)
∆Horxn = D (Reactant bonds) - D (Product bonds)
∆Horxn = (6 DN-H + DCl-Cl) - (DN-N + 4 DN-H + 2 DH-Cl)
∆Horxn = [(6 mol)(390 kJ/mol) + (1 mol)(243 kJ/mol)]

175
Chapter 8 - Thermochemistry: Chemical Energy
______________________________________________________________________________

- [(1 mol)(240 kJ/mol) + (4 mol)(390 kJ/mol) + (2 mol)(432 kJ/mol)] = -81 kJ

13
8.20 C4H10(l) + O2(g) → 4 CO2(g) + 5 H2O(g)
2
∆Horxn = [4 ∆Hof (CO2) + 5 ∆Hof (H2O)] - ∆Hof (C4H10)
∆Horxn = [(4 mol)(-393.5 kJ/mol) + (5 mol)(-241.8 kJ/mol)] - [(1 mol)(-147.5 kJ/mol)]
∆Horxn = -2635.5 kJ
∆HoC = -2635.5 kJ/mol
 kJ  1 mol 
C4H10, 58.12 amu; ∆HoC =  _ 2635.5   = _ 45.35 kJ / g
 mol  58.12 g 
 kJ  g 
∆HoC =  _ 45.35  0.579  = _ 26.3 kJ / mL
 g  mL 

8.21 ∆So is negative because the reaction decreases the number of moles of gaseous molecules.

8.22 The reaction proceeds from a solid and a gas (reactants) to all gas (product). This is more
disordered and ∆So is positive.

8.23 (a) Because ∆Go is negative, the reaction is spontaneous.


(b) Because ∆Go is positive, the reaction is nonspontaneous.

8.24 ∆Go = ∆Ho - T∆So = (-92.2 kJ) - (298 K)(-0.199 kJ/K) = -32.9 kJ
Because ∆Go is negative, the reaction is spontaneous.
Set ∆Go = 0 and solve for T.
∆Go = 0 = ∆Ho - T∆So
∆ o _ 92.2 kJ
T = Ho = = 463 K = 190oC
∆S _ 0.199 kJ/K

8.25 (a) 2 A2 + B2 → 2 A2B


(b) Because the reaction is exothermic, ∆H is negative. There are more reactant
molecules than product molecules. The randomness of the system decreases on going
from reactant to product, therefore ∆S is negative.
(c) Because ∆G = ∆H - T∆S, a reaction with both ∆H and ∆S negative is favored at low
temperatures where the negative ∆H term is larger than the positive - T∆S, and ∆G is
negative.

Understanding Key Concepts

8.26. (a) w = -P∆V, ∆V > 0; therefore w < 0 and the system is doing work on the
surroundings.
(b) Since the temperature has increased there has been an enthalpy change. The system
evolved heat, the reaction is exothermic, and ∆H < 0.

176
Chapter 8 - Thermochemistry: Chemical Energy
______________________________________________________________________________

8.27

8.28

8.29

8.30 ∆H = ∆E + P∆V
∆H _ ∆E = P∆V
∆ H _ ∆ E [_ 35.0 kJ _ (_ 34.8 kJ)] 1 L • atm
∆V = = x = -2 L
P 1 atm 101 x 10_ 3 kJ
∆V = -2 L = Vfinal - Vinitial = Vfinal - 5 L; Vfinal = -2 L - (-5 L) = 3 L
The volume decreases from 5 L to 3 L.

8.31 ∆Ho = +55 kJ


∆So is positive because the chemical system becomes more disordered in going from

177
Chapter 8 - Thermochemistry: Chemical Energy
______________________________________________________________________________

reactant to products.
∆Go = ∆Ho - T∆So; For the reaction to be spontaneous, ∆Go must be negative.
Because ∆Ho and ∆So are both positive, the reaction is spontaneous at some higher
temperatures but nonspontaneous at some lower temperatures.

8.32 The change is the spontaneous conversion of a liquid to a gas. ∆G is negative because the
change is spontaneous. The conversion of a liquid to a gas is endothermic, therefore ∆H
is positive. ∆S is positive because the gas is more disordered than the liquid.

8.33 (a) 2 A3 → 3 A2
(b) Because the reaction is spontaneous, ∆G is negative. ∆S is positive because the
number of molecules increases in going from reactant to products. ∆H could be either
positive or negative and the reaction would still be spontaneous. ∆H is probably positive
because there is more bond breaking than bond making

Additional Problems
Heat, Work, and Energy

8.34 Heat is the energy transferred from one object to another as the result of a temperature
difference between them. Temperature is a measure of the kinetic energy of molecular
motion.
Energy is the capacity to do work or supply heat. Work is defined as the distance moved
times the force that opposes the motion (w = d x F).
Kinetic energy is the energy of motion. Potential energy is stored energy.

8.35 Internal energy is the sum of kinetic and potential energies for each particle in the system.

2
 115 x 103 m 
8.36 Car: EK = ½(1400 kg)   = 7.1 x 105 J
 3600 s 
2
 38 x 103 m 
Truck: EK = ½(12,000 kg)   = 6.7 x 105 J
 3600 s 
The car has more kinetic energy.

8.37 Heat = q = 7.1 x 105 J (from Problem 8.34)


q = (specific heat) x m x ∆T
q 7.1 x 105 J
m= = = 5.7 x 103 g of water
(specific heat) x ∆ T  J 
 4.18  (50 o C _ 20 o C)
 g • o
C 

8.38 w = -P∆V = -(3.6 atm)(3.4 L - 3.2 L) = -0.72 L ⋅ atm

178
Chapter 8 - Thermochemistry: Chemical Energy
______________________________________________________________________________

 101 J 
w = (-0.72 L ⋅ atm)   = -72.7 J = -70 J; The energy change is negative.
 1 L • atm 

8.39 Vinitial = 50.0 mL + 50 mL = 100.0 mL = 0.1000 L


Vfinal = 50.0 mL = 0.0500 L
∆V = Vfinal - Vinitial = (0.0500 L - 0.1000 L) = -0.0500 L
w = -P∆V = -(1.5 atm)(-0.0500 L) = +0.075 L⋅ atm
 J 
w = (+0.075 L ⋅ atm) 101  = +7.6 J
 1 L • atm 
The positive sign for the work indicates that the surroundings does work on the system.
Energy flows into the system.

Energy and Enthalpy

8.40 ∆E = qv is the heat change associated with a reaction at constant volume. Since ∆V = 0,
no PV work is done.
∆H = qp is the heat change associated with a reaction at constant pressure. Since ∆V ≠ 0,
PV work can also be done.

8.41 ∆H is negative for an exothermic reaction. ∆H is positive for an endothermic reaction.

8.42 ∆H = ∆E + P∆V; ∆H and ∆E are nearly equal when there are no gases involved in a
chemical reaction, or, if gases are involved, ∆V = 0 (that is, there are the same
number of reactant and product gas molecules).

8.43 Heat is lost on going from H2O(g) → H2O(l) → H2O(s).


H2O(g) has the highest enthalpy content. H2O(s) has the lowest enthalpy content.

8.44 P∆V = -7.6 J (from Problem 8.39)


∆H = ∆E + P∆V
∆E = ∆H - P∆V = -0.31 kJ - (- 7.6 x 10-3 kJ) = -0.30 kJ

8.45 ∆H = -244 kJ and w = -P∆V = 35 kJ; therefore P∆V = -35 kJ


∆E = ∆H - P∆V = -244 kJ - (-35 kJ) = -209 kJ
For the system: ∆H = -244 kJ and ∆E = -209 kJ
∆H and ∆E for the surroundings are just the opposite of what they are for the system.
For the surroundings: ∆H = 244 kJ and ∆E = 209 kJ

8.46 ∆H = -1255.5 kJ/mol C2H2; C2H2, 26.04 amu


w = -P∆V = -(1.00 atm)(-2.80 L) = 2.80 L⋅ atm
 101 J 
w = (2.80 L⋅ atm)   = 283 J = 0.283 kJ
 1 L • atm 
1 mol C2 H2
6.50 g x = 0.250 mol C2H2
26.04 g C2 H 2

179
Chapter 8 - Thermochemistry: Chemical Energy
______________________________________________________________________________

q = (-1255.5 kJ/mol)(0.250 mol) = -314 kJ


∆E = ∆H - P∆V = -314 kJ - (-0.283 kJ) = -314 kJ

8.47 C2H4, 28.05 amu; HCl, 36.46 amu


w = -P∆V = -(1.00 atm)(-71.5 L) = 71.5 L⋅ atm
 101 J 
w = (71.5 L⋅ atm)   = 7222 J = 7.22 kJ
 1 L• atm 
1 mol C2 H 4
89.5 g C2H4 x = 3.19 mol C2H4
28.05 g C2 H 4
1 mol HCl
125 g HCl x = 3.43 mol HCl
36.46 g HCl
Because the reaction stoichiometry between C2H4 and HCl is one to one, C2H4 is the
limiting reactant.
∆Ho = -72.3 kJ/mol C2H4
q = (-72.3 kJ/mol)(3.19 mol) = -231 kJ
∆E = ∆H - P∆V = -231 kJ - (-7.22 kJ) = -224 kJ

8.48 C4H10O, 74.12 amu; mass of C4H10O = (0.7138 g/mL)(100 mL) = 71.38 g
1 mol
mol C4H10O = 71.38 g x = 0.9626 mol
74.12 g
q = n x ∆Hvap = 0.9626 mol x 26.5 kJ/mol = 25.5 kJ

8.49 Assume 100 mL of H2O = 100 g; H2O, 18.02 amu


1 mol H2 O 40.7 kJ
100 g x x = 226 kJ
18.02 g H 2 O 1 mol H2 O
The heat to vaporize 100 mL of H2O is much greater than that to vaporize 100 mL of
diethyl ether.

8.50 Al, 26.98 amu


1 mol
mol Al = 5.00 g x = 0.1853 mol
26.98 g
_ 1408.4 kJ
q = n x ∆Ho = 0.1853 mol Al x = _ 131 kJ ; 131 kJ is released.
2 mol Al

8.51 Na, 22.99 amu; ∆Ho = -368.4 kJ/2 mol Na = -184.2 kJ/mol Na
1 mol Na _ 184.2 kJ
1.00 g Na x x = -8.01 kJ
22.99 g Na 1 mol Na
8.01 kJ of heat is evolved. The reaction is exothermic.

8.52 Fe2O3, 159.7 amu


1 mol
mol Fe2O3 = 2.50 g x = 0.015 65 mol
159.7 g

180
Chapter 8 - Thermochemistry: Chemical Energy
______________________________________________________________________________

_ 24.8 kJ
q = n x ∆Ho = 0.015 65 mol Fe2O3 x = _ 0.388 kJ ; 0.388 kJ is evolved.
1 mol Fe 2 O3
Because ∆H is negative, the reaction is exothermic.

8.53 CaO, 56.08 amu


1 mol
mol CaO = 233.0 g x = 4.155 mol
56.08 g
464.8 kJ
q = n x ∆Ho = 4.155 mol CaO x = 1931 kJ ; 1931 kJ is absorbed.
1 mol CaO
Because ∆H is positive, the reaction is endothermic.

Calorimetry and Heat Capacity

8.54 Heat capacity is the amount of heat required to raise the temperature of a substance a
given amount. Specific heat is the amount of heat necessary to raise the temperature of
exactly 1 g of a substance by exactly 1oC.

8.55 A measurement carried out in a bomb calorimeter is done at constant volume and
therefore ∆E is obtained.

8.56 Na, 22.99 amu


J 1 mol
specific heat = 28.2 x = 1.23 J / (g • o C)
mol• C 22.99 g
o

8.57 q = (specific heat) x m x ∆T


q 89.7 J
specific heat = = = 0.523 J/(g ⋅ oC)
m x ∆ T (33.0 g)(5.20 o C)
Cm = [0.523 J/(g ⋅ oC)](47.88 g/mol) = 25.0 J/(mol ⋅ oC)

8.58 Mass of solution = 50.0 g + 1.045 g = 51.0 g


q = (specific heat) x m x ∆T
 J 
q =  4.18 (51.0 g)(32.3 o C _ 25.0 o C) = 1.56 x 103 J = 1.56 kJ
 g • o
C 
1 mol
CaO, 56.08 amu; mol CaO = 1.045 g x = 0.018 63 mol
56.08 g
1.56 kJ
Heat evolved per mole of CaO = = 83.7 kJ / mol CaO
0.018 63 mol
Because the reaction evolves heat, the sign for ∆H is negative. ∆H = -83.7 kJ

8.59 C6H6, 78.11 amu; 2 C6H6(l) + 15 O2(g) → 12 CO2(g) + 6 H2O(g)

181
Chapter 8 - Thermochemistry: Chemical Energy
______________________________________________________________________________

 J 
∆E = qV = _ q H2 O = -  4.18 (250.0 g)(7. 48o C) = -7817 J = -7.82 kJ
 g • o
C 
1 mol C6 H 6
0.187 g C6H6 x = 0.002 39 mol C6H6
78.11 g C6 H 6
∆E(per mole) = (-7.82 kJ)/(0.002 39 mol) = -3.27 x 103 kJ/mol
∆E (per gram C6H6) = (-3.27 x 103 kJ/mol)/(78.11 g/mol) = - 41.9 kJ/g

8.60 NaOH, 40.00 amu; HCl, 36.46 amu


1 mol NaOH
8.00 g NaOH x = 0.200 mol NaOH
40.00 g NaOH
1 mol HCl
8.00 g HCl x = 0.219 mol HCl
36.46 g HCl
Because the reaction stoichiometry between NaOH and HCl is one to one, the NaOH is
the limiting reactant.
 J 
qP = -qsoln = -(specific heat) x m x ∆T = -  4.18 (316 g)(33. 5o C _ 25. 0o C) = -11.2 kJ
 g • C
o

∆H = qp/n = (-11.2 kJ)/(0.200 mol) = -56 kJ/mol


When 10.00 g of HCl in 248.0 g of water is added the same temperature increase is
observed because the mass of NaOH is the same and it is still the limiting reactant. The
mass of the solution is also the same.

8.61 NH4NO3, 80.04 amu; assume 125 mL = 125 g H2O


1 mol NH 4 NO3
50.0 g NH4NO3 x = 0.625 mol NH4NO3
80.04 g NH 4 NO3
qp = ∆H x n = (+25.7 kJ/mol)(0.625 mol) = 16.1 kJ = 16,100 J
qsoln = -qp = -16,100 J
qsoln = (specific heat) x m x ∆T
qsoln _ 16,100 J
∆T = = = -22.0oC
(specific heat) x m  J 
 4.18 (50 g + 125 g)
 g • o C 
∆T = -22.0oC = Tfinal - Tinitial = Tfinal - 25.0oC
Tfinal = -22.0oC + 25.0oC = 3.0oC

Hess's Law and Heats of Formation

8.62 The standard state of an element is its most stable form at 1 atm and the specified
temperature, usually 25oC.

8.63 A compound’s standard heat of formation is the amount of heat associated with the
formation of 1 mole of a compound from its elements (in their standard states).

8.64 Hess’s Law – the overall enthalpy change for a reaction is equal to the sum of the
enthalpy changes for the individual steps in the reaction.

182
Chapter 8 - Thermochemistry: Chemical Energy
______________________________________________________________________________

Hess’s Law works because of the law of conservation of energy.

8.65 Elements always have ∆Hof = 0 because the standard state of elements is the reference
point from which all enthalpy changes are measured.

8.66 S(s) + O2(g) → SO2(g) ∆Ho1 = -296.8 kJ


SO2 + ½ O2(g) → SO3(g) ∆Ho2 = -98.9 kJ
Sum S(s) + 3/2 O2(g) → SO3(g) ∆Ho3 = ∆Ho1 + ∆Ho2
o o
∆H f = ∆H 3 = -296.8 kJ + (-98.9 kJ) = -395.7 kJ/mol

8.67 ∆Horxn = [12 ∆Hof(CO2) + 6 ∆Hof(H2O)] - [2 ∆Hof(C6H6)]


-6534 kJ = [(12 mol)(-393.5 kJ/mol) + (6 mol)(-285.8 kJ/mol)] - [(2 mol)(∆Hof(C6H6))]
Solve for ∆Hof(C6H6).
-6534 kJ = -6436.8 kJ - [(2 mol)(∆Hof(C6H6))]
97.2 kJ = (2 mol)(∆Hof(C6H6))
∆Hof(C6H6) = +48.6 kJ/mol

8.68 SO3(g) + H2O(l) → H2SO4(aq) ∆Ho1 = -227.8 kJ


H2(g) + ½ O2(g) → H2O(l) ∆H 2 = ∆Hof = -285.8 kJ
o

S(s) + 3/2 O2(g) → SO3(g) ∆Ho3 = ∆Hof = -395.7


Sum S(s) + H2(g) + 2 O2(g) → H2SO4(aq) ∆Hof (H2SO4) = ?
o o o o
∆H f (H2SO4) = ∆H 1 + ∆H 2 + ∆H 3 = -909.3 kJ

8.69 ∆Horxn = [∆Hof(CH3CO2H) + ∆Hof(H2O)] - ∆Hof(CH3CH2OH)


∆Horxn = [(1 mol)(-484.5 kJ/mol) + (1mol)(-285.8 kJ/mol)] - [(1 mol)(-277.7 kJ/mol)]
∆Horxn = - 492.6 kJ

8.70 C8H8(l) + 10 O2(g) → 8 CO2(g) + 4 H2O(l)


∆Horxn = ∆Hoc = - 4395.2 kJ
∆Horxn = [8 ∆Hof(CO2) + 4 ∆Hof(H2O)] - ∆Hof(C8H8)
- 4395.2 kJ = [(8 mol)(-393.5 kJ/mol) + (4 mol)(-285.8 kJ/mol)]- [(1 mol)(∆Hof(C8H8))]
Solve for ∆Hof(C8H8)
- 4395.2 kJ = - 4291.2 kJ - (1 mol)(∆Hof(C8H8))
-104.0 kJ = -(1 mol)(∆Hof(C8H8))
_ 104.0 kJ
∆Hof(C8H8) = = + 104.0 kJ / mol
_ 1 mol

8.71 C5H12O(l) + 15/2 O2(g) → 5 CO2(g) + 6 H2O(l)


∆Horxn = [5 ∆Hof(CO2) + 6 ∆Hof(H2O)] - ∆Hof(C5H12O)
∆Horxn = [(5 mol)(-393.5 kJ/mol) + (6 mol)(-285.8 kJ/mol)] - [(1 mol)(-313.6 kJ/mol)]
∆Horxn = -3369 kJ

8.72 ∆Horxn = ∆Hof(MTBE) - [∆Hof(2-Methylpropene) + ∆Hof(CH3OH)]


-57.8 kJ = -313.6 kJ - [(1 mol)(∆Hof(2-Methylpropene)) + (-238.7 kJ)]

183
Chapter 8 - Thermochemistry: Chemical Energy
______________________________________________________________________________

Solve for ∆Hof(2-Methylpropene).


-17.1 kJ = (1 mol)(∆Hof(2-Methylpropene))
∆Hof(2-Methylpropene) = -17.1 kJ/mol

8.73 C51H88O6(l) + 70 O2(g) → 51 CO2(g) + 44 H2O(l)


∆Horxn = [51 ∆Hof(CO2) + 44 ∆Hof(H2O)] - ∆Hof(C51H88O6)
∆Horxn = [(51 mol)(-393.5 kJ/mol) + (44 mol)(-285.8 kJ/mol)] - [(1 mol)(-1310 kJ/mol)]
∆Horxn = -3.133 x 104 kJ/mol C51H88O6
C51H88O6, 797.25 amu
kJ 1 mol g
q = -3.133 x 104 x x 0.94 = _ 37 kJ / mL ; 37 kJ released per mL
mol 797.25 g mL

Bond Dissociation Energies

8.74 H2C=CH2(g) + H2(g) → CH3CH3(g)


∆Horxn = D (Reactant bonds) - D (Product bonds)
∆Horxn = (DC=C + 4 DC-H+ DH-H) - (6 DC-H + DC-C)
∆Horxn = [(1 mol)(611 kJ/mol) + (4 mol)(410 kJ/mol) + (1 mol)(436 kJ/mol)]
- [(6 mol)(410 kJ/mol) + (1 mol)(350 kJ/mol)] = -123 kJ

8.75 CH3CH=CH2 + H2O → CH3CH(OH)CH3


∆Horxn = D (Reactant bonds) - D (Product bonds)
∆Horxn = (DC=C + DC-C + 6 DC-H + 2 DO-H) - (2 DC-C + 7 DC-H + DC-O + DO-H)
∆Horxn = [(1 mol)(611 kJ/mol) + (1 mol)(350 kJ/mol) + (6 mol)(410 kJ/mol)
+ (2 mol)(460 kJ/mol)] - [(2 mol)(350 kJ/mol) + (7 mol)(410 kJ/mol)
+ (1 mol)(350 kJ/mol) + (1 mol)(460 kJ/mol)] = -39 kJ

8.76 C4H10 + 13/2 O2 → 4 CO2 + 5 H2O


∆Horxn = D (Reactant bonds) - D (Product bonds)
∆Horxn = (3 DC-C + 10 DC-H + 13/2 DO=O) - (8 DC=O + 10 DO-H)
∆Horxn = [(3 mol)(350 kJ/mol) + (10 mol)(410 kJ/mol) + (13/2 mol)(498 kJ/mol)]
- [(8 mol)(804 kJ/mol) + (10 mol)(460 kJ/mol)] = -2645 kJ

8.77 CH3CO2H + CH3CH2OH → CH3CO2CH2CH3 + H2O


∆Horxn = D (Reactant bonds) - D (Product bonds)
∆Horxn = (DC=O + 2 DC-O + 8 DC-H + 2 DO-H) - (DC=O + 2 DC-O + 8 DC-H + 2 DO-H) = 0 kJ

Free Energy and Entropy

8.78 Entropy is a measure of molecular disorder.

8.79 ∆G = ∆H - T∆S
∆H is usually more important because it is usually much larger than T∆S.
8.80 A reaction can be spontaneous yet endothermic if ∆S is positive (more disorder) and the
T∆S term is larger than ∆H.

184
Chapter 8 - Thermochemistry: Chemical Energy
______________________________________________________________________________

8.81 A reaction can be nonspontaneous yet exothermic if ∆S is negative (more order) and the
temperature is high enough so that the T∆S term is more negative than ∆H.

8.82 (a) positive (more disorder) (b) negative (more order)

8.83 (a) positive (more disorder) (b) negative (more order)


(c) positive (more disorder)

8.84 (a) zero (equilibrium) (b) zero (equilibrium)


(c) negative (spontaneous)

8.85 Because the mixing of gas molecules is spontaneous, ∆G is negative. The mixture of gas
molecules is more disordered so ∆S is positive. For the diffusion of gases, ∆H is
approximately zero.

8.86 ∆S is positive. The reaction increases the total number of molecules.

8.87 ∆S < 0. The reaction decreases the number of gas molecules.

8.88 ∆G = ∆H - T∆S
(a) ∆G = - 48 kJ - (400 K)(135 x 10-3 kJ/K) = -102 kJ
∆G < 0, spontaneous; ∆H < 0, exothermic.
(b) ∆G = - 48 kJ - (400 K)(-135 x 10-3 kJ/K) = +6 kJ
∆G > 0, nonspontaneous; ∆H < 0, exothermic.
(c) ∆G = +48 kJ - (400 K)(135 x 10-3 kJ/K) = -6 kJ
∆G < 0, spontaneous; ∆H > 0, endothermic.
(d) ∆G = +48 kJ - (400 K)(-135 x 10-3 kJ/K) = +102 kJ
∆G > 0, nonspontaneous; ∆H > 0, endothermic.

8.89 ∆G = ∆H - T∆S
(a) ∆G = -128 kJ - (500 K)(35 x 10-3 kJ/K) = -146 kJ
∆G < 0, spontaneous; ∆H < 0, exothermic
(b) ∆G = +67 kJ - (250 K)(-140 x 10-3 kJ/K) = +102 kJ
∆G > 0, nonspontaneous; ∆H > 0, endothermic
(c) ∆G = +75 kJ - (800 K)(95 x 10-3 kJ/K) = -1 kJ
∆G < 0, spontaneous; ∆H > 0, endothermic

8.90 ∆G = ∆H - T∆S; Set ∆G = 0 and solve for T (the crossover temperature).


∆H _ 33 kJ
T= = = 570 K
∆S _ 0.058 kJ / K

8.91 Because ∆H > 0 and ∆S < 0, the reaction is nonspontaneous at all temperatures. There is
no crossover temperature.

185
Chapter 8 - Thermochemistry: Chemical Energy
______________________________________________________________________________

8.92 (a) ∆H < 0 and ∆S > 0; reaction is spontaneous at all temperatures.


(b) ∆H < 0 and ∆S < 0; reaction has a crossover temperature.
(c) ∆H > 0 and ∆S > 0; reaction has a crossover temperature.
(d) ∆H > 0 and ∆S < 0; reaction is nonspontaneous at all temperatures.

8.93 (a) ∆H < 0 and ∆S < 0. The reaction is favored by enthalpy but not by entropy.
∆Go = ∆Ho - T∆So = -217.5 kJ/mol - (298 K)[-233.9 x 10-3 kJ/(K ⋅ mol)] = -147.8 kJ
(b) The reaction has a crossover temperature. Set ∆G = 0 and solve for T (the crossover
temperature).
∆Go = 0 = ∆Ho - T∆So
∆ o 217.5 kJ/mol
T = Ho = = 929.9 K
∆S 233.9 x 10_ 3 kJ/(K • mol)

8.94 T = -114.1oC = 273.15 + (-114.1) = 159.0 K


∆Gfus = ∆Hfus - T∆Sfus; ∆G = 0 at the melting point temperature.
Set ∆G = 0 and solve for ∆Sfus.
∆G = 0 = ∆Hfus - T∆Sfus
∆ 5.02 kJ/mol
∆Sfus = Hfus = = 0.0316 kJ/(K⋅mol) = 31.6 J/(K⋅mol)
T 159.0 K

8.95 T = 61.2oC = 273.15 + (61.2) = 334.4 K


∆Gvap = ∆Hvap - T∆Svap; ∆G = 0 at the boiling point temperature.
Set ∆G = 0 and solve for ∆Svap.
∆G = 0 = ∆Hvap - T∆Svap
∆ H vap 29.2 kJ/mol
∆Svap = = = 0.0873 kJ/(K⋅mol) = 87.3 J/(K⋅mol)
T 334.4 K

General Problems

8.96 Mg(s) + 2 HCl(aq) → MgCl2(aq) + H2(g)


1 mol
mol Mg = 1.50 g x = 0.0617 mol Mg
24.3 g
mol
mol HCl = 0.200 L x 6.00 = 1.20 mol HCl
L
There is an excess of HCl. Mg is the limiting reactant.
 J   J 
q =  4.18 (200 g)(42.9 o C _ 25.0 oC) +  776 o (42.9 o C _ 25.0 o C) =
 g• C
o
 C
4
2.89 x 10 J
1 kJ
q = 2.89 x 104 J x = 28.9 kJ
1000 J
28.9 kJ
Heat evolved per mole of Mg = = 468 kJ / mol
0.0617 mol

186
Chapter 8 - Thermochemistry: Chemical Energy
______________________________________________________________________________

Because the reaction evolves heat, the sign for ∆H is negative. ∆H = - 468 kJ

8.97 (a) C(s) + CO2(g) → 2 CO(g)


∆Horxn = [2 ∆Hof(CO)] - ∆Hof(CO2)
∆Horxn = [(2 mol)(-110.5 kJ/mol)] - [(1 mol)(-393.5 kJ/mol)] = +172.5 kJ
(b) 2 H2O2(aq) → 2 H2O(l) + O2(g)
∆Horxn = [2 ∆Hof(H2O)] - [2 ∆Hof(H2O2)]
∆Horxn = [(2 mol)(-285.8 kJ/mol)] - [(2 mol)(-191.2 kJ/mol)] = -189.2 kJ
(c) Fe2O3(s) + 3 CO(g) → 2 Fe(s) + 3 CO2(g)
∆Horxn = [3 ∆Hof(CO2)] - [∆Hof(Fe2O3) + 3 ∆Hof(CO)]
∆Horxn = [(3 mol)(-393.5 kJ/mol)]
- [(1 mol)(-824.2 kJ/mol) + (3 mol)(-110.5 kJ/mol)] = -24.8 kJ

8.98 2 NO(g) + O2(g) → 2 NO2 (g) ∆Ho1 = 2(-57.0 kJ)


2 NO2 (g) → N2O4(g) ∆Ho2 = -57.2 kJ
Sum 2 NO(g) + O2(g) → N2O4(g)
∆Ho = ∆Ho1 + ∆Ho2 = -171.2 kJ

8.99 ∆G = ∆H - T∆S; at equilibrium ∆G = 0. Set ∆G = 0 and solve for T.


∆G = 0 = ∆H - T∆S
∆H 30.91 kJ/mol
T= = = 332 K = 59oC
∆S 93.2 x 10_ 3 kJ/(K • mol)

8.100 ∆Gfus = ∆Hfus - T∆Sfus; at the melting point ∆G = 0. Set ∆G = 0 and solve for T (the
melting point).
∆G = 0 = ∆Hfus - T∆Sfus
∆ 9.95 kJ
T = H fus = = 279 K
∆ Sfus 0.0357 kJ/K

8.101 HgS(s) + O2(g) → Hg(l) + SO2(g)


(a) ∆Horxn = ∆Hof(SO2) - ∆Hof(HgS)
∆Horxn = [(1 mol)(-296.8 kJ/mol)] - [(1 mol)(-58.2 kJ/mol)] = -238.6 kJ
(b) and (c) Because ∆H < 0 and ∆S > 0, the reaction is spontaneous at all temperatures.

8.102 ∆Horxn = D (Reactant bonds) - D (Product bonds)


(a) 2 CH4(g) → C2H6(g) + H2(g)
∆Horxn = (8 DC-H) - (DC-C + 6 DC-H + DH-H)
∆Horxn = [(8 mol)(410 kJ/mol)] - [(1 mol)(350 kJ/mol) + (6 mol)(410 kJ/mol)
+ (1 mol)(436 kJ/mol)] = +34 kJ
(b) C2H6(g) + F2(g) → C2H5F(g) + HF(g)
∆Horxn = (6 DC-H + DC-C + DF-F) - (5 DC-H + DC-C + DC-F + DH-F)
∆Horxn = [(6 mol)(410 kJ/mol) + (1 mol)(350 kJ/mol) + (1 mol)(159 kJ/mol)]

187
Chapter 8 - Thermochemistry: Chemical Energy
______________________________________________________________________________

- [(5 mol)(410 kJ/mol) + (1 mol)(350 kJ/mol) + (1 mol)(450 kJ/mol)


+ (1 mol)(570 kJ/mol)] = - 451 kJ
(c) N2(g) + 3 H2(g) → 2 NH3(g)
The bond dissociation energy for N2 is 945 kJ/mol.
∆Horxn = (D N2 + 3 DH-H) - (6 DN-H)
∆Horxn = [(1 mol)(945 kJ/mol) + (3 mol)(436 kJ/mol)] - [(6 mol)(390 kJ/mol)] = -87 kJ

8.103 (a) ∆Horxn = ∆Hof(CH3OH) - ∆Hof(CO)


∆Horxn = [(1 mol)(-238.7 kJ/mol)] - [(1 mol)(-110.5 kJ/mol)] = -128.2 kJ
(b) ∆Go = ∆Ho - T∆So = -128.2 kJ - (298 K)(-332 x 10-3 kJ/K) = -29.3 kJ
(c) Step 1 is spontaneous since ∆Go < 0.
(d) ∆Ho, because it is larger than T∆So.
(e) Set ∆G = 0 and solve for T.
∆G = 0 = ∆H - T∆S
∆H 128.2 kJ
T= = = 386 K; The reaction is spontaneous below 386 K.
∆S 332 x 10_ 3 kJ/K
(f) ∆Horxn = ∆Hof(CH4) - ∆Hof(CH3OH)
∆Horxn = [(1 mol)(-74.8 kJ/mol)] - [(1 mol)(-238.7 kJ/mol)] = +163.9 kJ
(g) ∆Go = ∆Ho - T∆So = +163.9 kJ - (298 K)(162 x 10-3 kJ/K) = +115.6 kJ
(h) Step 2 is nonspontaneous since ∆Go > 0.
(i) ∆Ho, because it is larger than T∆So.
(j) Set ∆G = 0 and solve for T.
∆G = 0 = ∆H - T∆S
∆H 163.9 kJ
T= = = 1012 K; The reaction is spontaneous above 1012 K.
∆S 162 x 10_ 3 kJ/K
(k) ∆Gooverall = ∆Go1 + ∆Go2 = -29.3 kJ + 115.6 kJ = +86.3 kJ
∆Hooverall = ∆Ho1 + ∆Ho2 = -128.2 kJ + 163.9 kJ = +35.7 kJ
∆Sooverall = ∆So1 + ∆So2 = -332 J/K + 162 J/K = -170 J/K
(l) The overall reaction is nonspontaneous since ∆Gooverall > 0.
(m) The two reactions should be run separately. Run step 1 below 386 K and run step 2
above 1012 K.

8.104 (a) 2 C8H18(l) + 25 O2(g) → 16 CO2(g) + 18 H2O(g)


(b) C8H18(l) + 25/2 O2(g) → 8 CO2(g) + 9 H2O(g)
∆Horxn = ∆Hoc = -5456.6 kJ
∆Horxn = [8 ∆Hof(CO2) + 9 ∆Hof(H2O)] - ∆Hof(C8H18)
-5456.6 kJ = [(8 mol)(-393.5 kJ/mol) +(9 mol)(-241.8 kJ/mol)] - [(1 mol)(∆Hof(C8H18))]
Solve for ∆Hof(C8H18).
-5456.6 kJ = -5324 kJ - [(1 mol)(∆Hof(C8H18))]
-132.4 kJ = -(1 mol)(∆Hof(C8H18))
∆Hof(C8H18) = +132.4 kJ/mol

8.105 Assume 1.00 kg of H2O; 1 kg⋅m2/s2 = 1 J


Ep = (1.00 kg)(9.81 m/s2)(739 m) = 7250 kg⋅m2/s2 = 7250 J

188
Chapter 8 - Thermochemistry: Chemical Energy
______________________________________________________________________________

q = specific heat x m x ∆T
q 7250 J
∆T = = = 1.73oC (temperature rise)
m x specific heat (1000 g)(4.18 J/(g • o C)

8.106 (a) ∆Stotal = ∆Ssystem + ∆Ssurr and ∆Ssurr = -∆H/T


∆Stotal = ∆Ssystem + (-∆H/T) = ∆Ssystem - ∆H/T
∆Ssystem = ∆Stotal + ∆H/T
∆G = ∆H - T∆S (substitute ∆Ssystem for ∆S in this equation)
∆G = ∆H - T(∆Stotal + ∆H/T) = -T∆Stotal
∆G = -T∆Stotal For a spontaneous reaction, if ∆Stotal > 0 then ∆G < 0.
(b) ∆Go = ∆Ho - T∆So
∆Ho = ∆Go + T∆So
∆ o [∆ G o + T ∆ So] [2879 x 103 J/mol + (298 K)(_ 210 J/(K • mol))]
∆Ssurr = - H = _ =_
T T 298 K
∆Ssurr = -9451 J/(K⋅ mol)

_ 138.4 kJ
8.107 3/2 NO2(g) + 1/2 H2O(l) → HNO3(aq) + 1/2 NO(g) ∆Ho1 =
2
(3)(_ 114.0 kJ)
3/2 NO(g) + 3/4 O2(g) → 3/2 NO2(g) ∆Ho2 =
4
1/2 N2(g) + 3/2 H2(g) → NH3(g) ∆Ho3 = - 46.1 kJ
_ 1169.6 kJ
NH3(g) + 5/4 O2(g) → NO(g) + 6/4 H2O(l) ∆Ho4 =
4
H2O(l) → 1/2 O2(g) + H2(g) ∆Ho5 = +285.8 kJ
Sum 1/2 H2(g) + 1/2 N2(g) + 3/2 O2(g) → HNO3(aq) ∆Ho = -207.4 kJ

8.108 2 CH4(g) + 4 O2(g) → 2 CO2(g) + 4 H2O(l) ∆Ho1 = 2(-890.3 kJ)


C2H6(g) → C2H4(g) + H2(g) ∆Ho2 = +137.0 kJ
3119.4 kJ
2 CO2(g) + 3 H2O(l) → C2H6(g) + 7/2 O2(g) ∆Ho3 =
2
H2O(l) → H2(g) + 1/2 O2(g) ∆Ho4 = +285.8 kJ
Sum 2 CH4(g) → C2H4(g) + 2 H2(g) ∆Ho = +201.9 kJ

8.109 qMo = (110.0 g)(specific heat Mo)(28.0oC - 100.0oC)


q H2 O = (150.0 g)[4.184 J/(g ⋅ oC)](28.0oC - 24.6oC)
qMo = - q H2 O
(110.0 g)(specific heat Mo)(28.0oC - 100.0oC) = - (150.0 g)[4.184 J/(g ⋅ oC)](28.0oC - 24.6oC)
_ (150.0 g)[4.184 J/(g • o C)](28. 0o C _ 24. 6o C)
specific heat Mo = o o
= 0.27 J/(g ⋅ oC)
(110.0 g)(28. 0 C _ 100. 0 C)

8.110 qice tea = -qice

189
Chapter 8 - Thermochemistry: Chemical Energy
______________________________________________________________________________

J
qice tea = (4.18 )(400.0 g)(10.0 o C _ 80.0 o C) = -1.17 x 105 J
g• C o

H2O, 18.02 amu


 1000 J  1 mol H 2 O 
qice = 1.17 x 105 J = (6.01 kJ/mol)  mice x 
 1 kJ  18.02 g H 2 O 
 J 
+  4.18 (mice)(10. 0o C _ 0. 0o C)
 (g • C) 
o

Solve for the mass of ice, mice.


1.17 x 105 J = (3.34 x 102 J/g)(mice) + (41.8 J/g)(mice) = (3.76 x 102 J/g)(mice)
1.17 x 105 J
mice = = 311 g of ice
3.76 x 102 J/g

8.111 There is a large excess of NaOH.


5.00 mL = 0.005 00 L
mol citric acid = (0.005 00 L)(0.64 mol/L) = 0.0032 mol citric acid
q H2 O = (51.6 g)[4.0 J/(g ⋅ oC)](27.9oC - 26.0oC) = 392 J
q rxn = _ q H2 O = -392 J
392 J 1 kJ
∆H = _ x = -123 kJ/mol = -120 kJ/mol citric acid
0.0032 mol 1000 J

8.112 CsOH(aq) + HCl(aq) → CsCl(aq) + H2O(l)


0.200 mol CsOH
mol CsOH = 0.100 L x = 0.0200 mol CsOH
1.00 L
0.400 mol HCl
mol HCl = 0.050 L x = 0.0200 mol HCl
1.00 L
The reactants were mixed in equal mole amounts.
Total volume = 150 mL and has a mass of 150 g
 J 
qsolution =  4.2 (150 g)(24. 28o C _ 22. 50o C) = 1121 J
 (g • o
C) 
qreaction = -qsolution = -1121 J
q reaction _ 1121 J 1 kJ
∆H = = x = -56 kJ/mol CsOH
mol CsOH 0.0200 mol CsOH 1000 J

8.113 NaNO3, 84.99 amu; KF, 58.10 amu


1 mol NaNO3
For NaNO3(s) → NaNO3(aq), q = 20.5 kJ/mol x = 0.241 kJ/g
84.99 g NaNO3
1 mol KF
For KF(s) → KF(aq), q = -17.7 kJ/mol x = -0.305 kJ/g
58.10 g KF
qsoln = (110.0 g)[4.18 J/(g ⋅ oC)](2.22oC) = 1021 J = 1.02 kJ
q rxn = _ qsoln = -1.02 kJ
Let X = mass of NaNO3 and Y = mass of KF

190
Chapter 8 - Thermochemistry: Chemical Energy
______________________________________________________________________________

X + Y = 10.0 g, so Y = 10.0 g - X
q rxn = -1.02 kJ = X(0.241 kJ/g) + Y(- 0.305 kJ/g) (substitute for Y and solve for X)
-1.02 kJ = X(0.241 kJ/g) + (10.0 g - X)(- 0.305 kJ/g)
-1.02 kJ = (0.241 kJ)X - 3.05 kJ + (0.305 kJ)X
2.03 kJ = (0.546 kJ)X
2.03 kJ
X= = 3.72 g NaNO3
0.546 kJ
Y = 10.0 g - X = 10.0 g - 3.72 g = 6.28 g KF = 6.3 g KF

8.114 ∆H
4 CO(g) + 2 O2(g) → 4 CO2(g) 2(-566.0 kJ)
2 NO2(g) → 2 NO(g) + O2(g) +114.0 kJ
2 NO(g) → O2(g) + N2(g) 2(- 90.2 kJ)
4 CO(g) + 2 NO2(g) → 4 CO2(g) + N2(g) - 1198.4 kJ

Multi-Concept Problems

8.115 (a) Each S has 2 bonding pairs and 2 lone pairs of electrons. Each S is sp3 hybridized
and the geometry around each S is bent.
(b) ∆H = D(reactant bonds) - D(product bonds)
∆H = (8 DS-S) - (4 DS=S) = +99 kJ
∆H = [(8 mol)(225 kJ/mol) - [(4 mol)(DS=S)] = +99 kJ
- (4 mol)(DS=S) = 99 kJ - 1800 kJ = -1701 kJ
DS=S = (1701 kJ)/(4 mol) = 425 kJ/mol
(c) σ*3p
π*3p ↑ ↑
π3p ↑↓ ↑↓
σ3p ↑↓
σ*3s ↑↓
σ3s ↑↓
S2
S2 should be paramagnetic with two unpaired electrons in the π*3p MOs.

8.116 (a)

(b) C(g) + ½ O2(g) + Cl2(g) → COCl2(g)


∆Hof = ∆Hof(C(g)) + (½ DO=O + DCl-Cl) - (DC=O + 2 DC-Cl)
∆Hof = (716.7 kJ) + [(½ mol)(498 kJ/mol) + (1 mol)(243 kJ/mol)]
- [(1 mol)(732 kJ/mol) + (2 mol)(330 kJ/mol)]
o
∆H rxn = - 183 kJ per mol COCl2
From Appendix B, ∆Hof(COCl2) = -219.1 kJ/mol
The calculation of ∆Hof from bond energies is only an estimate because the bond energies

191
Chapter 8 - Thermochemistry: Chemical Energy
______________________________________________________________________________

are average values derived from many different compounds.

8.117 (a) (1) 2 CH3CO2H(l) + Na2CO3(s) → 2 CH3CO2Na(aq) + CO2(g) + H2O(l)


(2) CH3CO2H(l) + NaHCO3(s) → CH3CO2Na(aq) + CO2(g) + H2O(l)

(b) CH3CO2H, 60.05 amu; Na2CO3, 105.99 amu; NaHCO3, 84.01 amu
3.7854 L 1000 mL 1.049 g CH3 CO2 H
1 gal x x x = 3971 g CH3CO2H
1 gal 1L 1 mL
1 mol CH3 CO2 H
3971 g CH3CO2H x = 66.13 mol CH3CO2H
60.05 g CH3 CO2 H
For reaction (1)
1 mol Na 2 CO3 105.99 g Na 2 CO3 1 kg
66.13 mol CH 3 CO2 H x x x = 3.505 kg
2 mol CH3 CO 2 H 1 mol Na 2 CO3 1000 g
Na2CO3
For reaction (2)
1 mol NaHCO3 84.01 g NaHCO3 1 kg
66.13 mol CH 3 CO2 H x x x = 5.556 kg
1 mol CH 3 CO2 H 1 mol NaHCO3 1000 g
NaHCO3

(c) 2 CH3CO2H(l) + Na2CO3(s) → 2 CH3CO2Na(aq) + CO2(g) + H2O(l)


∆Horxn = [2 ∆Hof(CH3CO2Na) + ∆Hof(CO2) + ∆Hof(H2O)]
- [2 ∆Hof(CH3CO2H) + ∆Hof(Na2CO3)]
o
∆H rxn = [(2 mol)(-726.1 kJ/mol) + (1 mol)(-393.5 kJ/mol) + (1 mol)(-285.8 kJ/mol)]
- [(2 mol)(- 484.5 kJ/mol) + (1 mol)(-1130.7 kJ/mol)]
∆Horxn = -31.8 kJ for 2 mol CH3CO2H
31.8 kJ
Heat = - x 66.13 mol CH3CO2H = -1050 kJ (liberated)
2 mol CH3 CO2 H
CH3CO2H(l) + NaHCO3(s) → CH3CO2Na(aq) + CO2(g) + H2O(l)
∆Horxn = [∆Hof(CH3CO2Na) + ∆Hof(CO2) + ∆Hof(H2O)]
- [∆Hof(CH3CO2H) + ∆Hof(NaHCO3)]
o
∆H rxn = [(1 mol)(-726.1 kJ/mol) + (1 mol)(-393.5 kJ/mol) + (1 mol)(-285.8 kJ/mol)]
- [(1 mol)(- 484.5 kJ/mol) + (1 mol)(-950.8 kJ/mol)]
∆Horxn = +29.9 kJ for 1 mol CH3CO2H
29.9 kJ
q= x 66.13 mol CH3CO2H = +1980 kJ (absorbed)
1 mol CH3 CO2 H

8.118 (a) 2 K(s) + 2 H2O(l) → 2 KOH(aq) + H2(g)


(b) ∆Horxn = [2 ∆Hof(KOH)] - [2 ∆Hof(H2O)]
∆Horxn = [(2 mol)(- 482.4 kJ/mol)] - [(2 mol)(-285.8 kJ/mol)] = -393.2 kJ
(c) The reaction produces 393.2 kJ/ 2 mol K = 196.6 kJ/ mol K. Assume that the mass of
the water does not change and that the specific heat = 4.18 J/(g⋅oC) for the solution that is
produced.

192
Chapter 8 - Thermochemistry: Chemical Energy
______________________________________________________________________________

1 mol K 196.6 kJ 1000 J


q = 7.55 g K x x x = 3.80 x 104 J
39.10 g K 1 mol K 1 kJ
q = (specific heat) x m x ∆T
q 3.80 x 104 J
∆T = = = 22.7oC
(specific heat) x m [4.18 J/(g • C)](400.0 g)
o

∆T = Tfinal - Tinitial
Tfinal = ∆T + Tinitial = 22.7oC + 25.0oC = 47.7oC

1 mol K 2 mol KOH


(d) 7.55 g K x x = 0.193 mol KOH
39.10 g K 2 mol K
Assume that the mass of the solution does not change during the reaction and that the
solution has a density of 1.00 g/mL.
1.00 mL 1L
solution volume = 400.0 g x x = 0.400 L
1g 1000 mL
0.193 mol KOH
molarity = = 0.483 M
0.400 L
2 KOH(aq) + H2SO4(aq) → K2SO4(aq) + 2 H2O(l)
1 mol H 2 SO4 1000 mL
0.193 mol KOH x x = 174 mL of 0.554 M H2SO4
2 mol KOH 0.554 mol H 2 SO4

8.119 (a) Each N is sp3 hybridized and the geometry about each N is
trigonal pyramidal.

_ 1011.2 kJ
(b) 2/4 NH3(g) + 3/4 N2O(g) → N2(g) + 3/4 H2O(l) ∆Ho1 =
4
+ 286 kJ
1/4 H2O(l) + 1/4 N2H4(l) → 1/8 O2(g) + 1/2 NH3(g) ∆Ho2 =
8
(3)(+314 kJ)
3/4 N2H4(l) + 3/4 H2O(l) → 3/4 N2O(g) + 9/4 H2(g) o
∆H 3 =
4
(9)(_ 285.8 kJ)
9/4 H2(g) + 9/8 O2(g) → 9/4 H2O(l) ∆Ho4 =
4
Sum N2H4(l) +
O2(g) → N2(g) + 2 H2O(l)
∆Ho = -622 kJ

(c) N2H4, 32.045 amu


1 mol N 2 H 4
mol N2H4 = 100.0 g N2H4 x = 3.12 mol N2H4
32.045 g N 2 H 4
q = (3.12 mol N2H4)(622 kJ/mol) = 1940 kJ

193
Chapter 8 - Thermochemistry: Chemical Energy
______________________________________________________________________________

8.120 Assume 100.0 g of Y.


1 mol F
mol F = 61.7 g F x = 3.25 mol F
19.00 g F
1 mol Cl
mol Cl = 38.3 g Cl x = 1.08 mol Cl
35.45 g Cl
Cl1.08F3.25, divide each subscript by the smaller of the two, 1.08.
Cl1.08 / 1.08F3.25 / 1.08
ClF3
(a) Y is ClF3 and X is ClF

(b)
There are five electron clouds around the Cl (3 bonding and 2 lone pairs). The geometry
is T-shaped.

(c) ∆H
Cl2O(g) + 3 OF2(g) → 2 O2(g) + 2 ClF3(g) -533.4 kJ
2 ClF(g) + O2(g) → Cl2O(g) + OF2(g) +205.6 kJ
O2(g) + 2 F2(g) → 2 OF2(g) 2(24.7 kJ)
2 ClF(g) + 2 F2(g) → 2 ClF3(g) -278.4 kJ

Divide reaction coefficients and ∆H by 2.

ClF(g) + F2(g) → ClF3(g) ∆H = -278.4 kJ/2 = -139.2 kJ/mol ClF3

(d) ClF, 54.45 amu


1 mol ClF _ 139.2 kJ
q = 25.0 g ClF x x x 0.875 = -55.9 kJ
54.45 g ClF 1 mol ClF
55.9 kJ is released in this reaction.

194
9 Gases: Their Properties and Behavior

9.1 1.00 atm = 14.7 psi


1 atm 14.7 psi
1.00 mm Hg x x = 1.93 x 10-2 psi
760 mm Hg 1 atm

9.2 1.00 atmosphere pressure can support a column of Hg 0.760 m high. Because the density
of H2O is 1.00 g/mL and that of Hg is 13.6 g/mL, 1.00 atmosphere pressure can support a
column of H2O 13.6 times higher than that of Hg. The column of H2O supported by 1.00
atmosphere will be (0.760 m)(13.6) = 10.3 m.

9.3 The pressure in the flask is less than 0.975 atm because the liquid level is higher on the
side connected to the flask. The 24.7 cm of Hg is the difference between the two pressures.
1 atm
Pressure difference = 24.7 cm Hg x = 0.325 atm
76.0 cm Hg
Pressure in flask = 0.975 atm - 0.325 atm = 0.650 atm

9.4 The pressure in the flask is greater than 750 mm Hg because the liquid level is lower on
the side connected to the flask.
10 mm Hg
Pressure difference = 25 cm Hg x = 250 mm Hg
1 cm Hg
Pressure in flask = 750 mm Hg + 250 mm Hg = 1000 mm Hg

9.5 (a) Assume an initial volume of 1.00 L.


First consider the volume change resulting from a change in the number of moles with the
pressure and temperature constant.
Vi Vf ; V = Vi n f (1.00 L)(0.225 mol)
= f = = 0.75 L
ni nf ni 0.3 mol
Now consider the volume change from 0.75 L as a result of a change in temperature with
the number of moles and the pressure constant.
Vi Vf ; V = Vi T f (0.75 L)(400 K)
= f = = 1.0 L
Ti Tf Ti 300 K
There is no net change in the volume as a result of the decrease in the number of moles of
gas and a temperature increase.

195
Chapter 9 - Gases: Their Properties and Behavior
______________________________________________________________________________

(b) Assume an initial volume of 1.00 L.


First consider the volume change resulting from a change in the number of moles with the
pressure and temperature constant.
Vi Vf ; V = Vi n f (1.00 L)(0.225 mol)
= f = = 0.75 L
ni nf ni 0.3 mol
Now consider the volume change from 0.75 L as a result of a change in temperature with
the number of moles and the pressure constant.
Vi Vf ; V = Vi T f (0.75 L)(200 K)
= f = = 0.5 L
Ti Tf Ti 300 K
The volume would be cut in half as a result of the decrease in the number of moles of gas
and a temperature decrease.

PV (1.000 atm)(1.000 x 105 L)


9.6 n= = = 4.461 x 103 mol CH4
RT  L • atm 
 0.082 06 (273.15 K)
 K • mol 
 16.04 g 
CH4, 16.04 amu; mass CH4 = (4.461 x 103 mol)  4
 = 7.155 x 10 g CH4
 1 mol 

9.7 C3H8, 44.10 amu; V = 350 mL = 0.350 L; T = 20oC = 293 K


1 mol C3 H8
n = 3.2 g x = 0.073 mol C3H8
44.10 g C3 H8
 L • atm 
(0.073 mol) 0.082 06 (293 K)
nRT  K • mol 
P= = = 5.0 atm
V 0.350 L

1 atm
9.8 P = 1.51 x 104 kPa x = 149 atm; T = 25.0oC = 298 K
101.325 kPa
PV (149 atm)(43.8 L)
n= = = 267 mol He
RT  L • atm 
 0.082 06 (298 K)
 K • mol 

9.9 The volume and number of moles of gas remain constant.


nR Pi Pf (2.37 atm)(273 K)
= = ; Tf = Pf Ti = = 301 K = 28oC
V Ti T f Pi 2.15 atm

9.10 (a) The temperature has increased by about 10% (from 300 K to 325 K) while the

196
Chapter 9 - Gases: Their Properties and Behavior
______________________________________________________________________________

amount and the pressure are unchanged. Thus, the volume should increase by about 10%.

(b) The temperature has increased by a factor of 1.5 (from 300 K to 450 K) and the
pressure has increased by a factor of 3 (from 0.9 atm to 2.7 atm) while the amount is
unchanged. Thus, the volume should decrease by half (1.5/3 = 0.5).

(c) Both the amount and the pressure have increased by a factor of 3 (from 0.075 mol to
0.22 mol and from 0.9 atm to 2.7 atm) while the temperature is unchanged. Thus, the
volume is unchanged.

9.11 CaCO3(s) + 2 HCl(aq) → CaCl2(aq) + CO2(g) + H2O(l)


CaCO3, 100.1 amu; CO2, 44.01 amu
1 mol CaCO3 1 mol CO2
mole CO2 = 33.7 g CaCO3 x x = 0.337 mol CO2
100.1 g CaCO3 1 mol CaCO3
44.01 g CO2
mass CO2 = 0.337 mol CO2 x = 14.8 g CO2
1 mol CO2
 L • atm 
(0.337 mol) 0.082 06 (273 K)
nRT  K • mol 
V= = = 7.55 L
P 1.00 atm

9.12 C3H8(g) + 5 O2(g) → 3 CO2(g) + 4 H2O(l)

197
Chapter 9 - Gases: Their Properties and Behavior
______________________________________________________________________________

PV (4.5 atm)(15.0 L)
npropane = = = 2.76 mol C3H8
RT  L • atm 
 0.082 06 (298 K)
 K • mol 
3 mol CO 2
2.76 mol C3H8 x = 8.28 mol CO2
1 mol C3 H8
 L • atm 
(8.28 mol) 0.082 06 (273 K)
nRT  K • mol 
V= = = 186 L = 190 L
P 1.00 atm
PV (1.00 atm)(1.00 L)
9.13 n= = = 0.0446 mol
RT  L • atm 
 0.082 06 (273 K)
 K • mol 
1.52 g
molar mass = = 34.1 g/mol; molecular mass = 34.1 amu
0.0446 mol
Na2S(aq) + 2 HCl(aq) → H2S(g) + 2 NaCl(aq)
The foul-smelling gas is H2S, hydrogen sulfide.

1 mol H 2
9.14 12.45 g H2 x = 6.176 mol H2
2.016 g H 2
1 mol N 2
60.67 g N2 x = 2.166 mol N2
28.01 g N2
1 mol NH3
2.38 g NH3 x = 0.140 mol NH3
17.03 g NH3
ntotal = n H2 + n N2 + n NH3 = 6.176 mol + 2.166 mol + 0.140 mol = 8.482 mol

6.176 mol 2.166 mol 0.140 mol


XH2 = = 0.7281; X N2 = = 0.2554; X NH3 = = 0.0165
8.482 mol 8.482 mol 8.482 mol

9.15 n total = 8.482 mol (from Problem 9.14). T = 90oC = 363 K


 L • atm 
(8.482 mol) 0.082 06 (363 K)
n total RT  K • mol 
P total = = = 25.27 atm
V 10.00 L
P H2 = X H2 • P total = (0.7281)(25.27 atm) = 18.4 atm
P N2 = X N2 • P total = (0.2554)(25.27 atm) = 6.45 atm
P NH3 = X NH3 • P total = (0.0165)(25.27 atm) = 0.417 atm

9.16 P H2 O = X H2 O ⋅ PTotal = (0.0287)(0.977 atm) = 0.0280 atm

9.17 The number of moles of each gas is proportional to the number of each of the different
gas molecules in the container.
ntotal = nred + nyellow + ngreen = 6 + 2 + 4 = 12

198
Chapter 9 - Gases: Their Properties and Behavior
______________________________________________________________________________

6 n yellow 2 n green 4
Xred = n red = = 0.500; Xyellow = = = 0.167; Xgreen = = =
n total 12 n total 12 n total 12
0.333
Pred = Xred ⋅ Ptotal = (0.500)(600 mm Hg) = 300 mm Hg
Pyellow = Xyellow ⋅ Ptotal = (0.167)(600 mm Hg) = 100 mm Hg
Pgreen = Xgreen ⋅ Ptotal = (0.333)(600 mm Hg) = 200 mm Hg

3 RT
9.18 u= , M = molar mass, R = 8.314 J/(K ⋅ mol), 1 J = 1 kg ⋅ m2/s2
M
3 x 8.314 kg m 2 /(s2 K mol) x 310 K
at 37oC = 310 K, u= = 525 m/s
28.01 x 10_ 3 kg / mol
3 x 8.314 kg m2 /(s 2 K mol) x 248 K
at -25oC = 248 K, u= = 470 m/s
28.01 x 10_ 3 kg / mol
3 RT
9.19 u= , M = molar mass, R = 8.314 J/(K ⋅ mol), 1 J = 1 kg ⋅ m2/s2
M
O2, 32.00 amu, 32.00 x 10-3 kg/mol
1.6093 km 1000 m 1 hr 1 min
u = 580 mi/h x x x x = 259 m/s
1 mi 1 km 60 min 60 s
3 RT 3R T
u= ; u2 =
M M
2
M (259 m/s )2 (32.00 x 10_ 3 kg/mol)
T= u = = 86.1 K
3R (3)(8.314 kg • m 2 / s2 • K • mol)
T = 86.1 - 273.15 = -187.0oC

rate O2 83.8 rate O2


9.20 (a) = M Kr = ; = 1.62
rate Kr M O2 32.0 rate Kr
O2 diffuses 1.62 times faster than Kr.
rate C2 H 2 M N2 28.0 rate C2 H 2
(b) = = ; = 1.04
rate N 2 M C2 H 2 26.0 rate N 2
C2H2 diffuses 1.04 times faster than N2.

rate 20 Ne M 22 Ne 22 rate 21 Ne M 22 Ne 22
9.21 22
= 20
= = 1.05 ; 22
= 21
= = 1.02
rate Ne M Ne 20 rate Ne M Ne 21
Thus, the relative rates of diffusion are 20 Ne(1.05) > 21 Ne(1.02) > 22 Ne(1.00) .

 L • atm 
(0.500 mol) 0.082 06 (300 K)
nRT  K • mol 
9.22 P= = = 20.5 atm
V (0.600 L)

199
Chapter 9 - Gases: Their Properties and Behavior
______________________________________________________________________________

nRT a n2
P = _ 2
V _ nb V
 L • atm   L • atm (0.500 mol 2
2
(0.500 mol) 0.082 06 (300 K)  1.35  )
 K • mol  
2
mol 
P= _ = 20.3
[(0.600 L) _ (0.500 mol)(0.0387 L/mol)] (0.600 L )2
atm
9.23 The amount of ozone is assumed to be constant.
Therefore nR = Pi Vi = Pf Vf
Ti Tf
Because V ∝ h, then Pi h i = Pf h f where h is the thickness of the O3 layer.
Ti Tf
 1.6 x 10_ 9 atm  273 K 
hf = Pi x Tf x h i =   -5
(20 x 103 m) = 3.8 x 10 m
Pf Ti  1 atm  230 K 
2
(Actually, V = 4πr h, where r = the radius of the earth. When you go out ~30 km to get to
the ozone layer, the change in r2 is less than 1%. Therefore you can neglect the change in
r2 and assume that V is proportional to h.)

15 mm Hg
9.24 For ether, the MAC = x 100% = 2.0%
760 mm Hg

9.25 (a) Let X = partial pressure of chloroform.


X
MAC = x 100% = 0.77%
760 mm Hg
0.77%
Solve for X. X = 760 mm Hg x = 5.9 mm Hg
100%
(b) CHCl3, 119.4 amu
PV = nRT; n =
PV
=
(5.9 mm Hg )(10.0 L) = 0.00347 mol CHCl
3
RT  L • atm 
 0.082 06 (273 K)
 K • mol 
119.4 g CHCl3
mass CHCl3 = 0.00347 mol CHCl3 x = 0.41 g CHCl3
1 mol CHCl3

Understanding Key Concepts

9.26 (a) The volume of a gas is proportional to the kelvin temperature at


constant pressure. As the temperature increases from 300 K to
450 K, the volume will increase by a factor of 1.5.

200
Chapter 9 - Gases: Their Properties and Behavior
______________________________________________________________________________

(b) The volume of a gas is inversely proportional to pressure at constant


temperature. As the pressure increases from 1 atm to
2 atm, the volume will decrease by a factor of 2.

(c) PV = nRT; The amount of gas (n) is constant.


Therefore nR = Pi Vi = Pf Vf .
Ti Tf
Assume Vi = 1 L and solve for Vf.
Pi Vi Tf = (3 atm)(1 L)(200 K) = = 1 L
Vf
Ti Pf (300 K)(2 atm)
There is no change in volume.

9.27 If the sample remains a gas at 150 K, then drawing (c) represents the gas at this
temperature. The gas molecules still fill the container.

9.28 The two gases should mix randomly and homogeneously and this is best represented by
drawing (c).

9.29 The two gases will be equally distributed among the three flasks.

9.30 The gas pressure in the bulb in mm Hg is equal to the difference in the height of the Hg in
the two arms of the manometer.

201
Chapter 9 - Gases: Their Properties and Behavior
______________________________________________________________________________

9.31 A
When stopcock A is opened, the pressure in the flask will equal the
external pressure, and the level of mercury will be the same in both
arms of the manometer.

9.32 (a) Because there are more yellow gas molecules than there are blue, the yellow gas
molecules have the higher average speed.
(b) Each rate is proportional to the number of effused gas molecules of each type.
Myellow = 25 amu
2
rateblue M yellow 5 25 amu 5 25 amu 25 amu
= ; = ;   = ; Mblue = 2
= 36
rateyellow M blue 6 M blue 6 M blue 5
 
6
amu

9.33

( )( ) (248 K)(760 mm Hg)


(a) P2 = P1 ; P2 = T 2 P1 = = 632 mm Hg
T 2 T1 T1 298 K
The column of Hg will rise to ~130 mm Hg inside the tube (drawing 1). The pressure
inside the tube (632 mm Hg) plus the pressure of 130 mm Hg equals the external
pressure, ~760 mm Hg.
(b) The column of Hg will rise to ~760 mm (drawing 2), which is equal to the external
pressure, ~760 mm Hg.

202
Chapter 9 - Gases: Their Properties and Behavior
______________________________________________________________________________

(c) The pressure inside the tube is equal to the external pressure and the Hg level inside
the tube will be the same as in the dish (drawing 3).

Additional Problems
Gases and Gas Pressure

9.34 Temperature is a measure of the average kinetic energy of gas particles.

9.35 Gases are much more compressible than solids or liquids because there is a large amount
of empty space between individual gas molecules.

1.00 atm
9.36 P = 480 mm Hg x = 0.632 atm
760 mm Hg
101,325 Pa
P = 480 mm Hg x = 6.40 x 104 Pa
760 mm Hg

101,325 Pa 1 kPa
9.37 P = 352 torr x x = 46.9 kPa
760 torr 1000 Pa
760 mm Hg
P = 0.255 atm x = 194 mm Hg
1.00 atm
101,325 Pa
P = 0.0382 mm Hg x = 5.09 Pa
760 mm Hg
9.38 Pflask > 754.3 mm Hg; Pflask = 754.3 mm Hg + 176 mm Hg = 930 mm Hg

9.39 Pflask < 1.021 atm (see Figure 9.4)


1.00 atm
Pdifference = 28.3 cm Hg x = 0.372 atm
76.0 cm Hg
Pflask = 1.021 atm - Pdifference = 1.021 atm - 0.372 atm = 0.649 atm

9.40 Pflask > 752.3 mm Hg (see Figure 9.4)


If the pressure in the flask can support a column of ethyl alcohol (d = 0.7893 g/mL) 55.1
cm high, then it can only support a column of Hg that is much shorter because of the
higher density of Hg.
0.7893 g/mL
55.1 cm x = 3.21 cm Hg = 32.1 mm Hg
13.546 g/mL
Pflask = 752.3 mm Hg + 32.1 mm Hg = 784.4 mm Hg
101,325 Pa
Pflask = 784.4 mm Hg x = 1.046 x 105 Pa
760 mm Hg

9.41 Compute the height of a column of CHCl3 that 1.00 atm can support.
13.546 g/mL
760 mm Hg x = 6941 mm CHCl3; therefore 1.00 atm = 6941 mm CHCl3
1.4832 g/mL
The pressure in the flask is less than atmospheric pressure.

203
Chapter 9 - Gases: Their Properties and Behavior
______________________________________________________________________________

Patm - Pflask = 0.849 atm - 0.788 atm = 0.061 atm


6941 mm CHCl3
0.061 atm x = 423 mm CHCl3
1.00 atm
The chloroform will be 423 mm higher in the manometer arm connected to the flask.

9.42 % Volume
N2 78.08
O2 20.95
Ar 0.93
CO2 0.037
The % volume for a particular gas is proportional to the number of molecules of that gas
in a mixture of gases.
Average molecular mass of air
= (0.7808)(mol. mass N2) + (0.2095)(mol. mass O2)
+ (0.0093)(at. mass Ar) + (0.000 37)(mol. mass CO2)
= (0.7808)(28.01 amu) + (0.2095)(32.00 amu)
+ (0.0093)(39.95 amu) + (0.000 37)(44.01 amu) = 28.96 amu

9.43 The % volume for a particular gas is proportional to the number of molecules of that gas
in a mixture of gases.
Average molecular mass of a diving-gas = (0.020)(mol. mass O2) + (0.980)(at. mass He)
= (0.020)(32.00 amu) + (0.980)(4.00 amu) = 4.56 amu

The Gas Laws

nR Pi Pf Pi T f = P
9.44 (a) = = ; f
V Ti Tf Ti
Let Pi = 1 atm, Ti = 100 K, Tf = 300 K
(1 atm)(300 K)
Pf = Pi T f = = 3 atm
Ti (100 K)
The pressure would triple.

RT Pi Pf
(b) = = ; Pi n f = Pf
V ni nf ni
Let Pi = 1 atm, ni = 3 mol, nf = 1 mol
(1 atm)(1 mol) 1
Pf = Pi n f = = atm
ni (3 mol) 3
1
The pressure would be the initial pressure.
3

(c) nRT = PiVi = PfVf; Pi Vi = P


f
Vf
Let Pi = 1 atm, Vi = 1 L, Vf = 1 - 0.45 L = 0.55 L

204
Chapter 9 - Gases: Their Properties and Behavior
______________________________________________________________________________

(1 atm)(1 L)
Pf = Pi V i = = 1.8 atm
Vf (0.55 L)
The pressure would increase by 1.8 times.

(d) nR = Pi Vi = Pf Vf ; Pi V i T f = P
f
Ti Tf Ti Vf
Let Pi = 1 atm, Vi = 1 L, Ti = 200 K, Vf = 3 L, Ti = 100 K
(1 atm)(1 L)(100 K)
Pf = Pi V i T f = = 0.17 atm
T i Vf (200 K)(3 L)
The pressure would be 0.17 times the initial pressure.

nR Vi Vf Vi T f
9.45 (a) = = ; = Vf
P Ti T f Ti
Let Vi = 1 L, Ti = 400 K, Tf = 200 K
Vi Tf (1 L)(200 K)
Vf = = = 0.5 L
Ti (400 K)
The volume would be halved.

RT Vi Vf Vi n f
(b) = = ; = Vf
P ni nf ni
Let Vi = 1 L, ni = 4 mol, nf = 5 mol
Vi n f (1 L)(5 mol)
Vf = = = 1.25 L
ni (4 mol)
The volume would increase by 1/4.

(c) nRT = Pi Vi = Pf Vf; Pi V i


= Vf
Pf
Let Vi = 1 L, Pi = 4 atm, Pf = 1 atm
Pi Vi = (4 atm)(1 L) = 4 L
Vf =
Pf (1 atm)
The volume would increase by a factor of 4.

(d) Pi V i P f V f ; Pi V i T f
nR = = = Vf
Ti Tf Pf T i
Let Vi = 1 L, Ti = 200 K, Tf = 400 K, Pi = 1 atm, Pf = 2atm
Pi Vi Tf = (1 atm)(1 L)(400 K) = 1 L
Vf =
Pf Ti (2 atm)(200 K)
There is no volume change.

9.46 They all contain the same number of gas molecules.

9.47 For air, T = 50oC = 323 K.

205
Chapter 9 - Gases: Their Properties and Behavior
______________________________________________________________________________

 1.00 atm 
 750 mm Hg x (2.50 L)
PV  760 mm Hg 
n= = = 0.0931 mol air
RT  L • atm 
 0.082 06 (323 K)
 K • mol 
For CO2, T = -10oC = 263 K
 1.00 atm 
 765 mm Hg x (2.16 L)
PV  760 mm Hg 
n= = = 0.101 mol CO2
RT  L • atm 
 0.082 06 (263 K)
 K • mol 
Because the number of moles of CO2 is larger than the number of moles of air, the CO2
sample contains more molecules.

9.48 n and T are constant; therefore nRT = Pi Vi = Pf Vf


(150 atm)(49.0 L)
Vf = Pi Vi = = 7210 L
Pf (1.02 atm)
nR Vi Vf
n and P are constant; therefore = =
P Ti T f
(49.0 L)(308 K)
Vf = Vi Tf = = 51.5 L
Ti (293 K)

9.49 Ti = 20oC = 293 K; nR = Pi Vi = Pf Vf


Ti Tf
(140 atm)(8.0 L)(273 K)
Vf = Pi Vi Tf = = 1.0 x 103 L
Ti Pf (293 K)(1.00 atm)

1 mol CO2
9.50 15.0 g CO2 x = 0.341 mol CO2
44.0 g CO2
 L • atm 
(0.341 mol) 0.082 06 (300 K)
nRT  K • mol 
P= = = 27.98 atm
V (0.30 L)
760 mm Hg
27.98 atm x = 2.1 x 104 mm Hg
1 atm

1 mol N2
9.51 20.0 g N2 x = 0.714 mol N2
28.0 g N 2
PV (6.0 atm)(0.40 L)
T= = = 41 K
nR  L • atm 
(0.714 mol) 0.082 06 
 K • mol 

206
Chapter 9 - Gases: Their Properties and Behavior
______________________________________________________________________________

1 H atom 1 mol H 1000 cm3


9.52 3
x 23
x = 1.7 x 10-21 mol H/L
cm 6.02 x 10 atoms 1L
 L • atm 
(1.7 x 10_ 21 mol) 0.082 06 (100 K)
nRT  K • mol 
P= = = 1.4 x 10-20 atm
V (1 L)
760 mm Hg
P = 1.4 x 10-20 atm x = 1 x 10-17 mm Hg
1.0 atm

9.53 CH4, 16.04 amu; 5.54 kg = 5.54 x 103 g; T = 20oC = 293 K


 1 mol  L • atm 
 5.54 x 103 g x  0.082 06 (293 K)
nRT  16.04 g  K • mol 
P= = = 189.6 atm
V (43.8 L)
101,325 Pa 1 k Pa
P = 189.6 atm x x = 1.92 x 104 kPa
1 atm 1000 Pa
 1000 Pa 1 atm 
17,180 kPa x x (43.8 L)
PV  1 k Pa 101,325 Pa 
9.54 n= = = 308.9 mol
RT  L • atm 
 0.082 06 (293 K)
 K • mol 
39.948 g
mass Ar = 308.9 mol x = 12340 g = 1.23 x 104 g
1 mol

1000 Pa 1 atm
9.55 P = 13,800 kPa x x = 136.2 atm
1 kPa 101,325 Pa
n and T are constant; therefore nRT = PiVi = PfVf
(136.2 atm)(2.30 L)
Vf = Pi Vi = = 250.6 L
Pf (1.25 atm)
1 balloon
250.6 L x = 167 balloons
1.5 L

Gas Stoichiometry

9.56 For steam, T = 123.0oC = 396 K


PV (0.93 atm)(15.0 L)
n= = = 0.43 mol steam
RT  L • atm 
 0.082 06 (396 K)
 K • mol 
1 mol
For ice, H2O, 18.02 amu; n = 10.5 g x = 0.583 mol ice
18.02 g
Because the number of moles of ice is larger than the number of moles of steam, the ice
contains more H2O molecules.

9.57 T = 85.0oC = 358 K

207
Chapter 9 - Gases: Their Properties and Behavior
______________________________________________________________________________

 1.00 atm 
1111 mm Hg x (3.14 L)
PV  760 mm Hg 
nAr = = = 0.156 mol Ar
RT  L • atm 
 0.082 06 (358 K)
 K • mol 
1 mol Cl2
Cl2, 70.91 amu; n Cl2 = 11.07 g Cl2 x = 0.156 mol Cl2
70.91 g Cl2
There are equal numbers of Ar atoms and Cl2 molecules in their respective samples.

9.58 The containers are identical. Both containers contain the same number of gas molecules.
Weigh the containers. Because the molecular mass for O2 is greater than the molecular
mass for H2, the heavier container contains O2.

9.59 Assuming that you can see through the flask, Cl2 gas is greenish and He is colorless.

1L
9.60 room volume = 4.0 m x 5.0 m x 2.5 m x _3 3
= 5.0 x 104 L
10 m
PV (1.0 atm)(5.0 x 104 L)
ntotal = = = 2.23 x 103 mol
RT  L • atm 
 0.082 06 (273 K)
 K • mol 
n O2 = (0.2095)n total = (0.2095)(2.23 x 103 mol) = 467 mol O2
32.0 g
mass O2 = 467 mol x = 1.5 x 104 g O2
1 mol

1 mol O2
9.61 0.25 g O2 x = 7.8 x 10-3 mol O2
32.0 g O2
 L • atm 
(7.8 x 10_ 3 mol) 0.082 06 (310 K)
nRT  K • mol 
V= = = 0.198 L = 0.200 L = 200 mL O2
P 1.0 atm

16.04 g
9.62 (a) CH4, 16.04 amu; d= = 0.716 g/L
22.4 L
44.01 g
(b) CO2, 44.01 amu; d= = 1.96 g/L
22.4 L
32.00 g
(c) O2, 32.00 amu; d= = 1.43 g/L
22.4 L
352.0 g
(d) UF6, 352.0 amu; d= = 15.7 g/L
22.4 L

9.63 Average molar mass = (0.270)(molar mass F2) + (0.730)(molar mass He)
= (0.270)(38.00 g/mol) + (0.730)(4.003 g/mol) = 13.18 g/mol
Assume 1.00 mole of the gas mixture. T = 27.5oC = 300.6 K

208
Chapter 9 - Gases: Their Properties and Behavior
______________________________________________________________________________

 L • atm 
(1.00 mol) 0.082 06 (300.6 K)
nRT  K • mol 
V= = = 26.3 L
P  1.00 atm 
 714 mm Hg x 
 760 mm Hg 
13.18 g
d= = 0.501 g/L
26.3 L

 1.00 atm 
 356 mm Hg x (1.500 L)
PV  760 mm Hg 
9.64 n= = = 0.0290 mol
RT  L • atm 
 0.082 06 (295.5 K)
 K • mol 
0.9847 g
molar mass = = 34.0 g/mol; molecular mass = 34.0 amu
0.0290 mol
9.65 (a) Assume 1.000 L gas sample
PV (1.00 atm)(1.000 L)
n= = = 0.0446 mol
RT  L • atm 
 0.082 06 (273 K)
 K • mol 
1.342 g
molar mass = = 30.1 g/mol; molecular mass = 30.1 amu
0.0446 mol
(b) Assume 1.000 L gas sample
 1.00 atm 
 752 mm Hg x (1.000 L)
PV  760 mm Hg 
n= = = 0.0405 mol
RT  L • atm 
 0.082 06 (298 K)
 K • mol 
1.053 g
molar mass = = 26.0 g/mol; molecular mass = 26.0 amu
0.0405 mol

9.66 2 HgO(s) → 2 Hg(l) + O2(g); HgO, 216.59 amu


1 mol HgO 1 mol O2
10.57 g HgO x x = 0.024 40 mol O2
216.59 g HgO 2 mol HgO
 L• atm 
(0.024 40 mol) 0.082 06 (273.15 K)
nRT  K • mol 
V= = = 0.5469 L
P 1.000 atm

9.67 2 HgO(s) → 2 Hg(l) + O2(g); HgO, 216.59 amu

2 mol HgO 216.59 g HgO


mass HgO = 0.0155 mol O2 x x = 6.71 g HgO
1 mol O2 1 mol HgO

9.68 Zn(s) + 2 HCl(aq) → ZnCl2(aq) + H2(g)

209
Chapter 9 - Gases: Their Properties and Behavior
______________________________________________________________________________

1 mol Zn 1 mol H 2
(a) 25.5 g Zn x x = 0.390 mol H2
65.39 g Zn 1 mol Zn
 L • atm 
(0.390 mol) 0.082 06 (288 K)
nRT  K • mol 
V= = = 9.44 L
P  1.00 atm 
 742 mm Hg x 
 760 mm Hg 
 1.00 atm 
 350 mm Hg x (5.00 L)
PV  760 mm Hg 
(b) n = = = 0.092 56 mol H2
RT  L • atm 
 0.082 06 (303.15 K)
 K • mol 
1 mol Zn 65.39 g Zn
0.092 56 mol H2 x x = 6.05 g Zn
1 mol H 2 1 mol Zn

9.69 2 NH4NO3(s) → 2 N2(g) + 4 H2O(g) + O2(g); NH4NO3, 80.04 amu


1 mol NH4 NO3 7 mol gas
Total moles of gas = 450 g NH4NO3 x x = 19.68
80.04 g NH 4 NO3 2 mol NH4 NO3
mol
T = 450oC = 723 K
 L • atm 
(19.68 mol) 0.082 06 (723 K)
nRT  K • mol 
V= = = 1.17 x 103 L
P (1.00 atm)

9.70 (a) V24h = (4.50 L/min)(60 min/h)(24 h/day) = 6480 L


VCO2 = (0.034)V24h = (0.034)(6480 L) = 220 L
 1.00 atm 
 735 mm Hg x (220 L)
PV  760 mm Hg 
n= = = 8.70 mol CO2
RT  L • atm 
 0.082 06 (298 K)
 K • mol 
44.01 g CO2
8.70 mol CO2 x = 383 g = 380 g CO2
1 mol CO2
(b) 2 Na2O2(s) + 2 CO2(g) → 2 Na2CO3(s) + O2(g); Na2O2, 77.98 amu
3.65 kg = 3650 g
1 mol Na 2 O2 2 mol CO2 1 day
3650 g Na2O2 x x x = 5.4 days
77.98 g Na 2 O2 2 mol Na 2 O2 8.70 mol CO2

9.71 2 TiCl4(g) + H2(g) → 2 TiCl3(s) + 2 HCl(g); TiCl4, 189.69 amu


(a) T = 435oC = 708 K

210
Chapter 9 - Gases: Their Properties and Behavior
______________________________________________________________________________

 1.00 atm 
 795 mm Hg x (155 L)
PV  760 mm Hg 
n H2 = = = 2.79 mol H2
RT  L • atm 
 0.082 06 (708 K)
 K • mol 
2 mol TiCl 4 189.69 g TiCl 4
2.79 mol H2 x x = 1058 g = 1060 g TiCl4
1 mol H2 1 mol TiCl4

2 mol HCl
(b) nHCl = 2.79 mol H2 x = 5.58 mol HCl
1 mol H 2
 L • atm 
(5.58 mol) 0.082 06 (273 K)
n HCl RT  K • mol 
V= = = 125 L HCl
P (1.00 atm)

Dalton's Law and Mole Fraction

9.72 Because of Avogadro's Law (V ∝ n), the % volumes are also % moles.
% mole
N2 78.08
O2 20.95
Ar 0.93
CO2 0.037 In decimal form, % mole = mole fraction.

P N2 = X N2 • P total = (0.7808)(1.000 atm) = 0.7808 atm


PO2 = XO2 • P total = (0.2095)(1.000 atm) = 0.2095 atm
PAr = XAr • P total = (0.0093)(1.000 atm) = 0.0093 atm
PCO2 = XCO2 • P total = (0.000 37)(1.000 atm) = 0.000 37 atm
Pressures of the rest are negligible.

94 mol
9.73 XCH4 = = 0.94; PCH4 = XCH4 • P total = (0.94)(1.48 atm) = 1.4 atm
100 mol
4 mol
X C 2 H6 = = 0.040; PC2H6 = XC2H6 • P total = (0.040)(1.48 atm) = 0.059 atm
100 mol
1.5 mol
XC3H8 = = 0.015; PC3H8 = XC3H8 • P total = (0.015)(1.48 atm) = 0.022 atm
100 mol
0.5 mol
XC4 H10 = = 0.0050; PC4 H10 = XC4H10 • P total = (0.0050)(1.48 atm) = 0.0074 atm
100 mol

9.74 Assume a 100.0 g sample. g CO2 = 1.00 g and g O2 = 99.0 g


1 mol CO2
mol CO2 = 1.00 g CO2 x = 0.0227 mol CO2
44.01 g CO2

211
Chapter 9 - Gases: Their Properties and Behavior
______________________________________________________________________________

1 mol O2
mol O2 = 99.0 g O2 x = 3.094 mol O2
32.00 g O2
ntotal = 3.094 mol + 0.0227 mol = 3.117 mol
3.094 mol 0.0227 mol
X O2 = = 0.993; XCO2 = = 0.007 28
3.117 mol 3.117 mol
PO2 = XO2 • P total = (0.993)(0.977 atm) = 0.970 atm
PCO2 = XCO2 • P total = (0.007 28)(0.977 atm) = 0.007 11 atm

9.75 From Problem 9.76: XHCI = 0.026, XH2 = 0.094, X Ne = 0.88


PHCI = XHCl ⋅ Ptotal = (0.026)(13,800 kPa) = 3.6 x 102 kPa
P H2 = X H2 • P total = (0.094)(13,800 kPa) = 1.3 x 10 kPa
3

PNe = XNe ⋅ Ptotal = (0.88)(13,800) kPa) = 1.2 x 104 kPa

9.76 Assume a 100.0 g sample.


1 mol HCl
g HCl = (0.0500)(100.0 g) = 5.00 g; 5.00 g HCl x = 0.137 mol HCl
36.5 g HCl
1 mol H 2
g H2 = (0.0100)(100.0 g) = 1.00 g; 1.00 g H2 x = 0.496 mol H2
2.016 g H 2
1 mol Ne
g Ne = (0.94)(100.0 g) = 94 g; 94 g Ne x = 4.66 mol Ne
20.18 g Ne
ntotal = 0.137 + 0.496 + 4.66 = 5.3 mol
0.137 mol 0.496 mol 4.66 mol
X HCl = = 0.026; X H2 = = 0.094; X Ne = = 0.88
5.3 mol 5.3 mol 5.3 mol

9.77 Assume a 1.000 L gas sample.


PV (1.000 atm)(1.000 L)
n= = = 0.044 61 mol
RT  L • atm 
 0.082 06 (273.15 K)
 K • mol 
1.413 g
average molar mass = = 31.67 g/mol
0.044 61 mol
31.67 = x • M Ar + (1 _ x) • M N2
31.67 = (x)(39.948) + (1 - x)(28.013)
Solve for x: x = 0.3064, 1 - x = 0.6936
The mixture contains 30.64% Ar and 69.36% N2.
Assume 100 moles of gas.

30.64 mol 69.36 mol


XAr = = 0.3064; X N2 = = 0.6936
100 mol 100 mol

212
Chapter 9 - Gases: Their Properties and Behavior
______________________________________________________________________________

9.78 Ptotal = PH2 + PH2 O ; PH2 = P total _ PH2 O = 747 mm Hg - 23.8 mm Hg = 723 mm Hg
 1.00 atm 
 723 mm Hg x (3.557 L)
PV  760 mm Hg 
n= = = 0.1384 mol H2
RT  L • atm 
 0.082 06 (298 K)
 K • mol 
1 mol Mg 24.3 g Mg
0.1384 mol H2 x x = 3.36 g Mg
1 mol H 2 1 mol Mg

9.79 Ptotal = PCl2 + PH2 O = 755 mm Hg


PCl2 = P total _ PH2 O = 755 mm Hg - 28.7 mm Hg = 726.3 mm Hg
P 726.3 mm Hg
(a) XCl2 = Cl2 = = 0.962
P total 755 mm Hg
(b) NaCl, 58.44 amu

pCl2 V (726.3 mmHg / 760 mmHg atm−1 )(0.597 L)


nCl2 = = = 0.0232 molCl2(g)
RT 0.082 L atmK −1mol−1 ( 27 + 273)K

2 mol NaCl 58.44 g NaCl


0.0232 mol Cl2 x x = 2.71 g NaCl
1mol Cl2 1 mol NaCl

Kinetic-Molecular Theory and Graham's Law

9.80 The kinetic-molecular theory is based on the following assumptions:


1. A gas consists of tiny particles, either atoms or molecules, moving about at random.
2. The volume of the particles themselves is negligible compared with the total volume
of the gas; most of the volume of a gas is empty space.
3. The gas particles act independently; there are no attractive or repulsive forces between
particles.
4. Collisions of the gas particles, either with other particles or with the walls of the
container, are elastic; that is, the total kinetic energy of the gas particles is constant at
constant T.
5. The average kinetic energy of the gas particles is proportional to the Kelvin
temperature of the sample.

9.81 Diffusion – The mixing of different gases by random molecular motion and with frequent
collisions.
Effusion – The process in which gas molecules escape through a tiny hole in a membrane
without collisions.

9.82 Heat is the energy transferred from one object to another as the result of a temperature
difference between them.

213
Chapter 9 - Gases: Their Properties and Behavior
______________________________________________________________________________

Temperature is a measure of the kinetic energy of molecular motion.

9.83 The atomic mass of He is much less than the molecular mass of the major components of
air (N2 and O2). The rate of effusion of He through the balloon skin is much faster.

3 RT 3 x 8.314 kg m 2 /(s2 K mol) x 220 K


9.84 u= = = 443 m/s
M 28.0 x 10_ 3 kg / mol

3 RT 3 x 8.314 kg m2 /(s2 K mol) x 293 K


9.85 For Br2: u = = = 214 m/s
M 159.8 x 10_ 3 kg / mol
3 x 8.314 kg m2 /(s 2 K mol) x T
For Xe: u = 214 m/s =
131.3 x 10_ 3 kg/mol
Square both sides of the equation and solve for T.
3 x 8.314 kg m2 /(s2 K mol) x T
45796 m2/s2 =
131.3 x 10_ 3 kg/mol
T = 241 K = -32oC

3 RT 3 x 8.314 kg m2 /(s2 K mol) x 150 K


9.86 For H2, u= = = 1360 m/s
M 2.02 x 10_ 3 kg/mol
3 x 8.314 kg m2 /(s2 K mol) x 648 K
For He, u= = 2010 m/s
4.00 x 10_ 3 kg / mol
He at 375oC has the higher average speed.

9.87 UF6, 352.02 amu; T = 25oC = 298 K


3 RT 3 x 8.314 kg m 2 /(s2 K mol) x 298 K
u= = = 145 m/s
M 352.02 x 10_ 3 kg/mol
Ferrari
mi 1.6093 km 1000 m 1 hr
145 x x x = 64.8 m/s
hr 1 mi 1 km 3600 s
The UF6 molecule has the higher average speed.

rateH2 MX ; 2.92 MX
9.88 = = ; 2.92 2.02 = M X
rateX M H2 1 2.02
MX = (2.92 2.02 )2 = 17.2 g/mol; molecular mass = 17.2 amu

214
Chapter 9 - Gases: Their Properties and Behavior
______________________________________________________________________________

rate Xe 1 MZ ; 131.29 131.29


9.89 = MZ ; = = MZ ; = MZ
rate Z M Xe 1.86 131.29 1.86 (1.86 )2
MZ = 37.9 g/mol; molecular mass = 37.9 amu; The gas could be F2.

9.90 HCl, 36.5 amu; F2, 38.0 amu; Ar, 39.9 amu
rate HCl 39.9 rate F2 39.9
= M Ar = = 1.05 = M Ar = = 1.02
rate Ar M HCl 36.5 rate Ar M F2 38.0
The relative rates of diffusion are HCl(1.05) > F2(1.02) > Ar(1.00).

9.91 Because CO and N2 have the same mass, they will have the same diffusion rates.

3 x 8.314 kg m2 /(s2 K mol) x T


9.92 u = 45 m/s =
4.00 x 10_ 3 kg / mol
Square both sides of the equation and solve for T.
3 x 8.314 kg m 2 /(s2 K mol) x T
2025 m 2 / s2 =
4.00 x 10_ 3 kg / mol
T = 0.325 K = -272.83oC (near absolute zero)
1000 m 1h
9.93 230 km/h x x = 63.9 m/s
1 km 3600 s
3 x 8.314 kg m2 /(s 2 K mol) x T
u = 63.9 m/s =
32.0 x 10_ 3 kg / mol
Square both sides of the equation and solve for T.
3 x 8.314 kg m2 /(s2 K mol) x T
4083 m2/s2 =
32.0 x 10_ 3 kg / mol
T = 5.24 K = -268oC

General Problems

35 37
rate Cl2 M Cl2 74.0
9.94 37
= 35
= = 1.03
rate Cl2 M Cl2 70.0
rate 35 Cl 37 Cl M 37 Cl2 74.0
37
= 35 37
= = 1.01
rate Cl2 M Cl Cl 72.0
The relative rates of diffusion are 35 Cl2 (1.03) > 35 Cl 37
Cl(1.01) > 37
Cl2 (1.00) .

9.95 Average molecular mass of air = 28.96 amu; CO2, 44.01 amu
44.01 g/mol
P = 760 mm Hg x = 1155 mm Hg
28.96 g/mol

215
Chapter 9 - Gases: Their Properties and Behavior
______________________________________________________________________________

 L • atm 
(1.00 mol) 0.082 06 (1050 K)
nRT  K • mol 
9.96 V= = = 1.1 L
P (75 atm)

9.97 1 atm = 1033.228 g/cm2


column height = (1033.228 g/cm2)(1 cm3/0.89g) = 1200 cm = 12 m

PV (2.15 atm)(7.35 L)
9.98 n= = = 0.657 mol Ar
RT  L • atm 
 0.082 06  (293 K)
 K • mol 
39.948 g Ar
0.657 mol Ar x = 26.2 g Ar
1 mol Ar
mtotal = 478.1 g + 26.2 g = 504.3 g

9.99 This is initially a Boyle's Law problem, because only P and V are changing while n and T
remain fixed. The initial volume for each gas is the volume of their individual bulbs.
The final volume for each gas is the total volume of the three bulbs.
nRT = PiVi = PfVf; Vf = 1.50 + 1.00 + 2.00 = 4.50 L
V (2.13 atm)(1.50 L)
For CO2: Pf = Pi i = = 0.710 atm
Vf (4.50 L)
(0.861 atm)(1.00 L)
For H2: Pf = Pi Vi = = 0.191 atm
Vf (4.50 L)
(1.15 atm)(2.00 L)
For Ar: Pf = Pi Vi = = 0.511 atm
Vf (4.50 L)
From Dalton's Law, Ptotal = PCO2 + P H2 + P Ar
Ptotal = 0.710 atm + 0.191 atm + 0.511 atm = 1.412 atm

9.100 (a) Bulb A contains CO2(g) and N2(g); Bulb B contains CO2(g), N2(g), and H2O(s).

 1.00 atm 
 564 mm Hg x (1.000 L)
PV  760 mm Hg 
(b) Initial moles of gas = n = =
RT  L• atm 
 0.082 06 (298 K)
 K • mol 
Initial moles of gas = 0.030 35 mol
 1.00 atm 
 219 mm Hg x (1.000 L)
PV  760 mm Hg 
mol gas in Bulb A = n = = = 0.011 78 mol
RT  L • atm 
 0.082 06 (298 K)
 K • mol 

216
Chapter 9 - Gases: Their Properties and Behavior
______________________________________________________________________________

 1.00 atm 
 219 mm Hg x (1.000 L)
PV  760 mm Hg 
mol gas in Bulb B = n = = = 0.017 29 mol
RT  L • atm 
 0.082 06 (203 K)
 K • mol 
n H2 O = ninitial - nA - nB = 0.030 35 - 0.011 78 - 0.017 29 = 0.001 28 mol = 0.0013 mol H2O

(c) Bulb A contains N2(g).


Bulb B contains N2(g) and H2O(s).
Bulb C contains N2(g) and CO2(s).

 1.00 atm 
 33.5 mm Hg x (1.000 L)
PV  760 mm Hg 
(d) nA = = = 0.001 803 mol
RT  L • atm 
 0.082 06 (298 K)
 K • mol 
 1.00 atm 
 33.5 mm Hg x (1.000 L)
PV  760 mm Hg 
nB = = = 0.002 646 mol
RT  L • atm 
 0.082 06 (203 K)
 K • mol 
 1.00 atm 
 33.5 mm Hg x (1.000 L)
PV  760 mm Hg 
nC = = = 0.006 472 mol
RT  L • atm 
 0.082 06 (83 K)
 K • mol 
n N2 = nA + nB + nC = 0.001 803 + 0.002 646 + 0.006 472 = 0.010 92 mol N2

(e) n CO2 = ninitial - n H2 O - n N2 = 0.030 35 - 0.0013 - 0.010 92 = 0.0181 mol CO2

9.101 C3H5N3O9, 227.1 amu


1 mol
(a) moles C3H5N3O9 = 1.00 g x = 0.004 40 mol
227.1 g
PV (1.00 atm)(0.500 L)
nair = = = 0.0208 mol air
RT  L • atm 
 0.082 06 (293 K)
 K • mol 

29 mol gas
(b) moles gas from C3H5N3O9 = 0.004 40 mol x
4 mol nitro
moles gas from C3H5N3O9 = 0.0319 mol gas from C3H5N3O9
ntotal = 0.0319 mol + 0.0208 mol = 0.0527 mol

217
Chapter 9 - Gases: Their Properties and Behavior
______________________________________________________________________________

 L • atm 
(0.0527 mol) 0.082 06 (698 K)
nRT  K • mol 
(c) P = = = 6.04 atm
V (0.500 L)

1 mol
9.102 NH3, 17.03 amu; mol NH3 = 45.0 g x = 2.64 mol
17.03 g
2
nRT nRT an
P= or P= _ 2
V (V _ nb) V

(a) At T = 0oC = 273 K


 L • atm 
(2.64 mol) 0.082 06 (273 K)
 K • mol 
P= = 59.1 atm
(1.000 L)
 L • atm   L • atm (2.64 mol 2
2
(2.64 mol) 0.082 06 (273 K)  4.17  )
 K • mol  
2
mol 
P= _
[(1.000 L) _ (2.64 mol)(0.0371 L/mol)] (1.000 L )2
P = 65.6 atm - 29.1 atm = 36.5 atm

(b) At T = 50oC = 323 K


 L • atm 
(2.64 mol) 0.082 06 (323 K)
 K • mol 
P= = 70.0 atm
(1.000 L)
 L • atm   L • atm (2.64 mol 2
2
(2.64 mol) 0.082 06 (323 K)  4.17  )
 K • mol  
2
mol 
P= _
[(1.000 L) _ (2.64 mol)(0.0371 L/mol)] (1.000 L )2
P = 77.6 atm - 29.1 atm = 48.5 atm

(c) At T = 100oC = 373 K


 L • atm 
(2.64 mol) 0.082 06 (373 K)
 K • mol 
P= = 80.8 atm
(1.000 L)
L • atm   L • atm (2.64 mol 2
2

(2.64 mol) 0.082 06 (373 K)  4.17  )
 K • mol  
2
mol 
P= _ 2
[(1.000 L) _ (2.64 mol)(0.0371 L/mol)] (1.000 L )
P = 89.6 atm - 29.1 atm = 60.5 atm
At the three temperatures, the van der Waals equation predicts a much lower pressure
than does the ideal gas law. This is likely due to the fact that NH3 can hydrogen bond
leading to strong intermolecular forces.

218
Chapter 9 - Gases: Their Properties and Behavior
______________________________________________________________________________

 1.00 atm 
 258 mm Hg x (0.500 L)
PV  760 mm Hg 
9.103 (a) ntotal = = = 0.007 06 mol
RT  L • atm 
 0.082 06 (293 K)
 K • mol 
 1 atm 
 344 mm Hg x (0.250 L)
PV  760 mm Hg 
(b) nB = = = 0.004 71 moles
RT  L • atm 
 0.082 06 (293 K)
 K • mol 
0.218 g
(c) d= = 0.872 g/L
0.250 L
0.218 g
(d) molar mass = = 46.3 g/mol, NO2; mol. mass = 46.3 amu
0.004 71 mol
(e) Hg2CO3(s) + 6 HNO3(aq) → 2 Hg(NO3)2(aq) + 3 H2O(l) + CO2(g) + 2 NO2(g)

9.104 CO2, 44.01 amu


1 mol CO 2
mol CO2 = 500.0 g CO2 x = 11.36 mol CO2
44.01 g CO 2

PV = nRT
 L• atm 
(11.36 mol) 0.082 06 (700 K)
nRT  K • mol 
P= = = 816 atm
V (0.800 L)

9.105 (a) Let x = mol CnH2n + 2 in reaction mixture.


 n + 1 3n + 1
Combustion of CnH2n + 2 → nCO2 + (n +1)H2O needs n +  = mol O2
 2  2
 3n + 1
Balanced equation is: CnH2n + 2(g) +   O2(g) → nCO2(g) + (n + 1)H2O(g)
 2 
In going from reactants to products, the increase in the number of moles is
 3 n + 1 n _1
[n + (n + 1)] - 1 +  = per mol of CnH2n + 2 reacted.
 2  2

PV (2.000 atm)(0.4000 L)
Before reaction: total mol = = = 0.032 70 mol
RT  L • atm 
 0.082 06 (298.15 K)
 K • mol 

PV (2.983 atm)(0.4000 L)
After reaction: total mol = = = 0.036 52 mol
RT  L • atm 
 0.082 06 (398.15 K)
 K • mol 
Difference = 0.032 70 mol - 0.036 52 mol = 0.003 82 mol

219
Chapter 9 - Gases: Their Properties and Behavior
______________________________________________________________________________

 n _ 1 2(0.003 82)
Increase in number of mol =   x = 0.003 82 mol; x =
 2  n _1
g Cn H 2 n + 2 0.148 g
Also x = =
molar mass [12.01 n + 1.008(2 n + 2)] g/mol
2(0.003 82) 0.148
So = ; 0.148 n - 0.148 = 0.107 n + 0.0154
n _1 14.026 n + 2.016
0.163
0.041 n = 0.163; n = = 4.0
0.041
CnH2n + 2 is C4H10 (butane); molar mass = (4)(12.01) + (10)(1.008) = 58.12 g/mol

1 mol C4 H10
(b) 0.148 g C4H10 x = 0.002 55 mol C4H10
58.12 g C4 H10
mol O2 initially = total mol - mol C4H10 = 0.032 70 mol - 0.002 55 mol = 0.030 15 mol O2
 n C4 H10   0.002 55 mol 
PC4H10 =   Pinitial =  (2.000 atm) = 0.156 atm
 n total   0.032 70 mol 
 n O2   0.030 15 mol 
PO2 =   Pinitial =  (2.000 atm) = 1.844 atm
 n total   0.032 70 mol 
13
(c) C4H10(g) + O2 → 4 CO2(g) + 5 H2O(g)
2
4 mol CO2
0.002 55 mol C4H10 x = 0.0102 mol CO2
1 mol C4 H10
5 mol H2 O
0.002 55 mol C4H10 x = 0.012 75 mol H2O
1 mol C4 H10
mol O2 unreacted = total mol after reaction - mol CO2 - mol H2O
= 0.03652 mol - 0.0102 mol - 0.01275 = 0.01357 mol O2

 n CO2   0.0102 mol 


PCO2 =   Pfinal =  (2.983 atm) = 0.833 atm
 n total   0.036 52 mol 

 n H2 O   0.012 75 mol 
PH2 O =   Pfinal =  (2.983 atm) = 1.041 atm
 n total   0.036 52 mol 

 n O2   0.013 57 mol 
PO2 =   Pfinal =  (2.983 atm) = 1.108 atm
 n total   0.036 52 mol 

9.106 (a) average molecular mass for natural gas


= (0.915)(16.04 amu) + (0.085)(30.07 amu) = 17.2 amu
1 mol gas
total moles of gas = 15.50 g x = 0.901 mol gas
17.2 g gas

220
Chapter 9 - Gases: Their Properties and Behavior
______________________________________________________________________________

 L • atm 
(0.901 mol) 0.082 06 (293 K)
 K • mol 
(b) P = = 1.44 atm
(15.00 L)

(c) PCH4 = XCH4 • P total = (1.44 atm)(0.915) = 1.32 atm


PC2H6 = XC2 H6 • P total = (1.44 atm)(0.085) = 0.12 atm

(d) ∆Hcombustion(CH4) = -802.3 kJ/mol and ∆Hcombustion(C2H6) = -1427.7 kJ/mol


Heat liberated = (0.915)(0.901 mol)(-802.3 kJ/mol)
+ (0.085)(0.901)(-1427.7 kJ/mol) = -771 kJ

9.107 PV = nRT
PV (3.00 atm)(10.0 L)
n total (initial) = = = 0.980 mol
RT  L • atm 
 0.082 06 (373.1 K)
 K • mol 
PV (2.40 atm)(10.0 L)
n total (final) = = = 0.784 mol
RT  L• atm 
 0.082 06 (373.1 K)
 K • mol 

CS2(g) + 3 O2(g) → CO2(g) + 2 SO2(g)


before reaction (mol) y 0.980 - y 0 0
change (mol) -x -3x +x +2x
after reaction (mol) y - x = 0 0.980 - y - 3x x 2x
n total (final) = (y - x) + (0.980 - y - 3x) + x + 2x = 0.784 mol
0.980 mol - 4x + 3x = 0.784 mol
x = 0.980 mol - 0.784 mol = 0.196 mol
mol CO2 = x = 0.196 mol
 L • atm 
(0.196 mol) 0.082 06 (373.1 K)
nRT  K • mol 
PCO2 = = = 0.600 atm
V (10.0 L)
mol SO2 = 2x = 2(0.196 mol) = 0.392 mol
 L • atm 
(0.392 mol) 0.082 06 (373.1 K)
nRT  K • mol 
PSO2 = = = 1.20 atm
V (10.0 L)
mol O2 = 0.980 mol - y - 3x = 0.980 mol - x - 3x = 0.980 - 4(0.196 mol) = 0.196 mol
PO2 = PCO2 = 0.600 atm

9.108 (a) T = 0oC = 273 K; PV = nRT


PV (0.229 atm)(0.0500 L)
nQ = = = 5.11 x 10-4 mol Q
RT  L • atm 
 0.082 06 (273 K)
 K • mol 

221
Chapter 9 - Gases: Their Properties and Behavior
______________________________________________________________________________

0.100 g Q
Q molar mass = = 196 g/mol
5.11 x 10_ 4 mol Q
Xe molar mass = 131.3 g/mol
On molar mass = 196 g/mol - 131.3 g/mol = 65 g/mol
So, n = 4 and XeO4 is the likely formula for Q.
(b) XeO4(g) → Xe(g) + 2 O2(g)
After decomposition, PTotal = P Xe + PO2 .
Because of the stoichiometry of the decomposition reaction, the partial pressure of O2 is
twice the partial pressure of Xe.
Let x = PXe and 2x = PO2 . PTotal = P Xe + PO2 = x + 2x = 3x = 0.941 atm
0.941 atm
x = = 0.314 atm
3
PXe = x = 0.314 atm; PO2 = PTotal _ PXe = 0.941 atm - 0.314 atm = 0.627 atm

9.109 Ca(ClO3)2, 206.98 amu; Ca(ClO)2, 142.98 amu


(a) Ca(ClO3)2(s) → CaCl2(s) + 3 O2(g)
Ca(ClO)2(s) → CaCl2(s) + O2(g)
(b) T = 700oC = 700 + 273 = 973 K
PV = nRT
PV (1.00 atm)(10.0 L)
n O2 = = = 0.125 mol O2
RT  L • atm 
 0.082 06 (973 K)
 K • mol 
Let X = mol Ca(ClO3)2 and let Y = mol Ca(ClO)2
X(206.98 g/mol) + Y(142.98 g/mol) = 10.0 g
3X + Y = 0.125 mol, so Y = 0.125 mol - 3X (substitute for Y and solve for X)
X(206.98 g/mol) + (0.125 mol - 3X)(142.98 g/mol) = 10.0 g
X(206.98 g/mol) + 17.9 g - X(428.94 g/mol) = 10.0 g
X(206.98 g/mol) - X(428.94 g/mol) = 10.0 g - 17.9 g = -7.9 g
X(-221.96 g/mol) = -7.9 g
X = (-7.9 g)/(-221.96 g/mol) = 0.0356 mol Ca(ClO3)2
Y = 0.125 mol - 3X; Y = 0.125 mol - 3(0.0356 mol) = 0.0182 mol Ca(ClO)2
206.98 g Ca(ClO3 )2
mass Ca(ClO3)2 = 0.0356 mol Ca(ClO3)2 x = 7.4 g Ca(ClO3)2
1 mol Ca(ClO3 )2
mass Ca(ClO)2 = 10.0 g - 7.4 g = 2.6 g Ca(ClO)2

9.110 PCl3, 137.3 amu; O2, 32.00 amu; POCl3, 153.3 amu
2 PCl3(g) + O2(g) → 2 POCl3(g)
1 mol PCl3
mol PCl3 = 25.0 g x = 0.182 mol PCl3
137.3 g PCl3
1 mol O2
mol O2 = 3.00 g x = 0.0937 mol O2
32.00 g O2

222
Chapter 9 - Gases: Their Properties and Behavior
______________________________________________________________________________

Check for limiting reactant.


1 mol O2
mol O2 needed = 0.182 mol PCl3 x = 0.0910 mol O2 needed
2 mol PCl3
There is a slight excess of O2. PCl3 is the limiting reactant.
2 mol POCl3
mol POCl3 = 0.182 mol PCl3 x = 0.182 mol POCl3
2 mol PCl3
mol O2 left over = 0.0937 mol - 0.0910 mol = 0.0027 mol O2 left over
T = 200.0oC = 200.0 + 273.15 = 473.1 K; PV = nRT
 L • atm 
(0.182 mol + 0.0027 mol) 0.082 06 (473.1 K)
nRT  K • mol 
P= = = 1.43 atm
V (5.00 L)

9.111 (a) T = 225oC = 225 + 273 = 498 K


PV = nRT
 L • atm 
(2.00 mol) 0.082 06 (498 K)
o
nRT  K • mol 
P NOCl = = = 0.204 atm
V (400.0 L)

2 NOCl(g) → 2 NO(g) + Cl2(g)


initial (atm) 0.204 0 0
change (atm) -2x +2x +x
equil (atm) 0.204 - 2x 2x x

Ptotal (after rxn) = (0.204 atm - 2x) + 2x + x = 0.246 atm


x = 0.246 atm - 0.204 atm = 0.042 atm
P NO = 2x = 2(0.042) = 0.084 atm
PCl2 = x = 0.042 atm
P NOCl = 0.204 - 2x = 0.204 - 2(0.042) = 0.120 atm
2x 0.084 atm
(b) % NOCl decomposed = x 100 % = x 100 % = 41%
o
P NOCl 0.204 atm

9.112 O2, 32.00 amu; O3, 48.00 amu


3 O2(g) → 2 O3(g)
initial (atm) 32.00 0
change (atm) -3x +2x
after rxn (atm) 32.00 - 3x 2x

PTotal = PO2 + PO3 = 30.64 atm = 32.00 atm - 3x + 2x = 32.00 atm - x


x = 32.00 atm - 30.64 atm = 1.36 atm
PO2 = 32.00 - 3x = 32.00 - 3(1.36 atm) = 27.92 atm
PO3 = 2x = 2(1.36 atm) = 2.72 atm

223
Chapter 9 - Gases: Their Properties and Behavior
______________________________________________________________________________

T = 25oC = 25 + 273 = 298 K; PV = nRT


PV (27.92 atm)(10.00 L)
n O2 = = = 11.42 mol O2
RT  L • atm 
 0.082 06 (298 K)
 K • mol 

PV (2.72 atm)(10.00 L)
n O3 = = = 1.11 mol O3
RT  L • atm 
 0.082 06 (298 K)
 K • mol 
32.00 g O2
mass O2 = 11.42 mol O2 x = 365.4 g O2
1 mol O2
48.00 g O3
mass O3 = 1.11 mol O3 x = 53.3 g O3
1 mol O3
total mass = 365.4 g + 53.3 g = 418.7 g
mass O3 53.3 g
mass % O3 = = x 100% = 12.7 %
total mass 418.7 g

9.113 CaCO3, 100.09 amu; CaO, 56.08 amu


1 mol CaCO3 1 mol CaO or CO2
mol CaO (or CO2) = 25.0 g CaCO3 x x = 0.250
100.09 g CaCO3 1 mol CaCO3
mol
56.08 g CaO
mass CaO = 0.250 mol CaO x = 14.02 g CaO
1 mol CaO
(a) 500.0 mL = 0.5000 L
PV = nRT; n CO2 = 0.250 mol
 L • atm 
(0.250 mol) 0.082 06 (1500 K)
nRT  K • mol 
PCO2 = = = 61.5 atm
V (0.5000 L)
(b) VCaO = (14.02 g)(3.34 g/mL) = 4.20 mL
V = 500.0 mL - 4.20 mL = 495.8 mL = 0.4958 L
nRT a n2
P = _ 2
V _ nb V
 L • atm   L • atm (0.250 mol 2
2
(0.250 mol) 0.082 06  (1500 K)  3.59  )
 K • mol  
2
mol 
P= _ = 62.5 atm
[(0.4958 L) _ (0.250 mol)(0.0427 L/mol)] (0.4958 L )2

Multi-Concept Problems

9.114 CO2, 44.01 amu


CH4(g) + 2 O2(g) → CO2(g) + 2 H2O(g) ∆Ho = -802 kJ
(a) 1.00 atm of CH4 only requires 2.00 atm O2, therefore O2 is in excess.
T = 300oC = 300 + 273 = 573 K; PV = nRT

224
Chapter 9 - Gases: Their Properties and Behavior
______________________________________________________________________________

PV (1.00 atm)(4.00 L)
n CH4 = = = 0.0851 mol CH4
RT  L • atm 
 0.082 06 (573 K)
 K • mol 
PV (4.00 atm)(4.00 L)
n O2 = = = 0.340 mol O2
RT  L • atm 
 0.082 06  (573 K)
 K • mol 
1 mol CO2 44.01 g CO2
mass CO2 = 0.0851 mol CH4 x x = 3.75 g CO2
1 mol CH 4 1 mol CO2
_ 802 kJ
(b) qrxn = 0.0851 mol CH4 x = -68.3 kJ
1 mol CH 4
CH4(g) + 2 O2(g) → CO2(g) + 2 H2O(g)
initial (mol) 0.0851 0.340 0 0
change (mol) -0.0851 -2(0.0851) +0.0851 +2(0.0851)
after rxn (mol) 0 0.340 - 2(0.0851) 0.0851 0.170

total moles of gas = 0.340 ml - 2(0.0851) mol + 0.0851 mol + 0.170 mol = 0.425 mol gas
1000 J
qrxn = -68.3 kJ x = - 68,300 J
1 kJ
qvessel = -qrxn = 68,300 J = (0.425 mol)(21 J/(mol ⋅ oC))( t f - 300oC) +
(14.500
 1000 g 
kg)  (0.449 J/(g • o C))( t f _ 300o C)
 1 kg 
Solve for t f .
68,300 J = (8.925 J/oC + 6510 J/oC)( t f - 300oC) = (6519 J/oC)( t f - 300oC)
68,300 J
o
= 10.5oC = ( t f - 300oC)
6519 J / C
300oC + 10.5oC = t f
o
t f = 310 C
(c) T = 310oC = 310 + 273 = 583 K
 L • atm 
(0.0851 mol) 0.082 06 (583 K)
nRT  K • mol 
PCO2 = = = 1.02 atm
V (4.00 L)

9.115 X + 3 O2 → 2 CO2 + 3 H2O


(a) X = C2H6O
C2H6O + 3 O2 → 2 CO2 + 3 H2O
(b) It is an empirical formula because it is the smallest whole number ratio of atoms. It is
also a molecular formula because any higher multiple such as C4H12O2 does not
correspond to a stable electron-dot structure.

225
Chapter 9 - Gases: Their Properties and Behavior
______________________________________________________________________________

(c)
(d) C2H6O, 46.07 amu
1 mol C2 H6 O
mol C2H6O = 5.000 g C2H6O x = 0.1085 mol C2H6O
46.07 g C2 H6 O
_ 144.2 kJ
∆Hcombustion = = -1328.6 kJ/mol
0.1085 mol
∆Hcombustion = [2 ∆Hof(CO2) + 3 ∆Hof(H2O)] - ∆Hof(C2H6O)

∆Hof(C2H6O) = [2 ∆Hof(CO2) + 3 ∆Hof(H2O)] - ∆Hcombustion


= [(2mol)(-393.5 kJ/mol) + (3 mol)(-241.8 kJ/mol)] - (-1328.6 kJ)
= -183.8 kJ/mol

9.116 (a) 2 C8H18(l) + 25 O2(g) → 16 CO2(g) + 18 H2O(g)


1000 mL 0.792 g
(b) 4.6 x 1010 L C8H18 x x = 3.64 x 1013 g C8H18
1L 1 mL
1 mol C8 H18 16 mol CO2
3.64 x 1013 g C8H18 x x = 2.55 x 1012 mol CO2
114.2 g C8 H18 2 mol C8 H18
44.0 g CO2 1 kg
2.55 x 1012 mol CO2 x x = 1.1 x 1011 kg CO2
1 mol CO 2 1000 g
 L • atm 
(2.55 x 1012 mol) 0.082 06 (273 K)
nRT  K • mol 
(c) V = = = 5.7 x 1013 L of CO2
P (1.00 atm)
(d) 12.5 moles of O2 are needed for each mole of isooctane (from part a).
12.5 mol
12.5 mol O2 = (0.210)(nair); nair = = 59.5 mol air
0.210
 L • atm 
(59.5 mol) 0.082 06 (273 K)
nRT  K • mol 
V= = = 1.33 x 103 L
P (1.00 atm)

9.117 (a) Freezing point of H2O on the Rankine scale is (9/5)(273.15) = 492oR.
PV (1.00 atm)(22.414 L) L • atm
(b) R = = = 0.0456 o
nT ∨
(1.00 mol)(49 2 ) R • mol

 L • atm   L • atm (2.50 mol 2


2
(2.50 mol) 0.0456 o  (525 o
R)  2.253  )
 R • mol  
2
mol 
(c) P = _ 2
[(0.4000 L) _ (2.50 mol)(0.04278 L/mol)] (0.4000 L )

P = 204.2 atm - 88.0 atm = 116 atm

226
Chapter 9 - Gases: Their Properties and Behavior
______________________________________________________________________________

PV (1 atm)(1323 L)
9.118 n = = = 7.25 mol of all gases
RT  L • atm 
 0.082 06  (2223 K)
 K • mol 
7.25 mol gases
(a) 0.004 00 mol “nitro” x = 0.0290 mol hot gases
1 mol _ nitro _

 1.00 atm 
 623 mm Hg x (0.500 L)
PV  760 mm Hg 
(b) n = = = 0.0190 mol B + C + D
RT  L • atm 
 0.082 06 (263 K)
 K • mol 
nA = ntotal - n(B+C+D) = 0.0290 - 0.0190 = 0.0100 mol A; A = H2O

 1.00 atm 
 260 mm Hg x (0.500 L)
PV  760 mm Hg 
(c) n = = = 0.007 00 mol C + D
RT  L • atm 
 0.082 06 (298 K)
 K • mol 
nB = n(B+C+D) - n(C+D) = 0.0190 - 0.007 00 = 0.0120 mol B; B = CO2

 1.00 atm 
 223 mm Hg x (0.500 L)
PV  760 mm Hg 
(d) n = = = 0.006 00 mol D
RT  L • atm 
 0.082 06 (298 K)
 K • mol 
nC = n(C+D) - nD = 0.007 00 - 0.006 00 = 0.001 00 mol C; C = O2
0.168 g
molar mass D = = 28.0 g/mol; D = N2
0.006 00 mol

(e) 0.004 C3H5N3O9(l) → 0.0100 H2O(g) + 0.012 CO2(g) + 0.001 O2(g) + 0.006 N2(g)
Multiply each coefficient by 1000 to obtain integers.
4 C3H5N3O9(l) → 10 H2O(g) + 12 CO2(g) + O2(g) + 6 N2(g)

9.119 CO2, 44.01 amu; H2O, 18.02 amu


1 mol CO2 1 mol C
(a) mol C = 0.3744 g CO2 x x = 0.008 507 mol C
44.01 g CO2 1 mol CO2
12.011 g C
mass C = 0.008 507 mol C x = 0.1022 g C
1 mol C
1 mol H2 O 2 mol H
mol H = 0.1838 g H2O x x = 0.020 400 mol H
18.02 g H 2 O 1 mol H 2 O

227
Chapter 9 - Gases: Their Properties and Behavior
______________________________________________________________________________

1.008 g H
mass H = 0.020 400 mol H x = 0.02056 g H
1 mol H
mass O = 0.1500 g - 0.1022 g - 0.02056 g = 0.0272 g O
1 mol O
mol O = 0.0272 g O x = 0.001 70 mol O
16.00 g O
C0.008 507H0.020 400O0.001 70 Divide each subscript by the smallest, 0.001 70.
C0.008 507 / 0.001 70H0.020 400 / 0.001 70O0.001 70 / 0.001 70
The empirical formula is C5H12O. The empirical formula mass is 88 g/mol.

(b) 1 atm = 101,325 Pa; T = 54.8oC = 54.8 + 273.15 = 327.9 K


PV = nRT
 1.00 atm 
100.0 kPa x (1.00 L)
PV  101.325 kPa 
n= = = 0.0367 mol methyl tert-butyl ether
RT  L • atm 
 0.082 06 (327.9 K)
 K • mol 
3.233 g
methyl tert-butyl ether molar mass = = 88.1 g/mol
0.0367 mol
The empirical formula mass and the molar mass are the same, so the molecular formula
and empirical formula are the same. C5H12O is the molecular formula and 88.15 amu is
the molecular mass for methyl tert-butyl ether.

(c) C5H12O(l) + 15/2 O2(g) → 5 CO2(g) + 6 H2O(l)

(d) ∆Hocombustion = [5 ∆Hof (CO2) + 6 ∆Hof (H2O(l))] - ∆Hof (C5H12O) = -3368.7 kJ


-3368.7 kJ = [(5 mol)(-393.5 kJ/mol) + (6 mol)(-285.8 kJ/mol)] - (1 mol)∆Hof (C5H12O)
(1 mol)∆Hof (C5H12O) = [(5 mol)(-393.5 kJ/mol) + (6 mol)(-285.8 kJ/mol)] + 3368.7 kJ
∆Hof (C5H12O) = -313.6 kJ/mol

228
10 Liquids, Solids, and Changes of State

 1D 
10.1 µ = Q x r = (1.60 x 10-19 C)(92 x 10-12 m)   = 4.41 D
 3.336 x 10 C • m 
_ 30

1.82 D
% ionic character for HF = x 100% = 41%
4.41 D
HF has more ionic character than HCl. HCl has only 17% ionic character.

10.2 (a) SF6 has polar covalent bonds but the molecule is symmetrical (octahedral). The
individual bond polarities cancel, and the molecule has no dipole moment.
(b) H2C=CH2 can be assumed to have nonpolar C–H bonds. In addition, the molecule is
symmetrical. The molecule has no dipole moment.

(c) The C–Cl bonds in CHCl3 are polar covalent bonds, and the
molecule is polar.

(d) The C–Cl bonds in CH2Cl2 are polar covalent bonds, and the
molecule is polar.

10.3

10.4 The N atom is electron rich (red) because of its high electronegativity. The H atoms are
electron poor (blue) because they are less electronegative.

10.5 (a) Of the four substances, only HNO3 has a net dipole moment.
(b) Only HNO3 can hydrogen bond.
(c) Ar has fewer electrons than Cl2 and CCl4, and has the smallest dispersion forces.

10.6 H2S dipole-dipole, dispersion

229
CH3OH hydrogen bonding, dipole-dipole, dispersion
C2H6 dispersion
Ar dispersion
Ar < C2H6 < H2S < CH3OH

10.7 (a) CO2(s) → CO2(g), ∆S is positive.


(b) H2O(g) → H2O(l), ∆S is negative.
(c) ∆S is positive (more disorder).

10.8 ∆G = ∆H - T∆S; at the boiling point (phase change), ∆G = 0.


∆ H vap 29.2 kJ/mol
∆H = T∆S; T = = = 334 K
∆ Svap 87.5 x 10_ 3 kJ/(K • mol)

10.9 The boiling point is the temperature where the vapor pressure of a liquid equals the
external pressure.
P1 = 760 mm Hg; P2 = 260 mm Hg; T1 = 80.1oC
∆Hvap = 30.8 kJ/mol
∆ H vap  1 1
ln P2 = ln P1 +  _ 
R  T1 T 2 
 R  1 1
(ln P 2 _ ln P1) = _

 ∆ H vap  T1 T 2
Solve for T2 (the boiling point for benzene at 260 mm Hg).
1  R  1
_ (ln P 2 _ ln P1) =
 T
T1  ∆ H vap  2

 J 
 8.3145 
1
_ [ln(260) _ ln(760)] K • mol  = 1
353.2 K  30,800 J/mol  T 2
 
 
1
= 0.003 121 K-1; T2 = 320 K = 47oC (boiling point is lower at lower pressure)
T2

(ln P2 _ ln P1)(R)
10.10 ∆Hvap =
1 1
 _ 
 T1 T 2 
P1 = 400 mm Hg; T1 = 41.0 oC = 314.2 K
P2 = 760 mm Hg; T2 = 331.9 K
 J 
[ln (760) _ ln (400)]  8.3145 
 K• mol 
∆Hvap = = 31,442 J/mol = 31.4 kJ/mol
 1 1 
 _ 
 314.2 K 331.9 K 

10.11 (a) 1/8 atom at 8 corners and 1 atom at body center = 2 atoms

230
Chapter 10 - Liquids, Solids, and Changes of State
______________________________________________________________________________

(b) 1/8 atom at 8 corners and 1/2 atom at 6 faces = 4 atoms

d 334 pm
10.12 For a simple cube, d = 2r; r= = = 167 pm
2 2

10.13 For a simple cube, there is one atom per unit cell.
1 mol
mass of one Po atom = 209 g/mol x = 3.4706 x 10-22 g/atom
6.022 x 1023 atoms
unit cell edge = d = 334 pm = 334 x 10-12 m = 3.34 x 10-8 cm
unit cell volume = d3 = (3.34 x 10-8 cm)3 = 3.7260 x 10-23 cm3
mass 3.4706 x 10_ 22 g
density = = = 9.31 g/cm3
volume 3.7260 x 10_ 23 cm3

10.14 There are several possibilities. Here's one.

10.15 For CuCl:


1/8 Cl- at 8 corners and 1/2 Cl- at 6 faces = 4 Cl- (4 minuses)
4 Cu+ inside (4 pluses)
For BaCl2:
1/8 Ba2+ at 8 corners and 1/2 Ba2+ at 6 faces = 4 Ba2+ (8 pluses)
8 Cl- inside (8 minuses)

10.16 (a) In the unit cell there is a rhenium atom at each corner of the cube. The number of
rhenium atoms in the unit cell = 1/8 Re at 8 corners = 1 Re atom.
In the unit cell there is an oxygen atom in the center of each edge of the cube. The
number of oxygen atoms in the unit cell = 1/4 O on 12 edges = 3 O atoms.
(b) ReO3
(c) Each oxide has a -2 charge and there are three of them for a total charge of -6. The
charge (oxidation state) of rhenium must be +6 to balance the negative charge of the oxides.
(d) Each oxygen atom is surrounded by two rhenium atoms. The geometry is linear.
(e) Each rhenium atom is surrounded by six oxygen atoms. The geometry is octahedral.

10.17 The minimum pressure at which liquid CO2 can exist is its triple point pressure of 5.11
atm.

231
Chapter 10 - Liquids, Solids, and Changes of State
______________________________________________________________________________

10.18 (a) CO2(s) → CO2(g)


(b) CO2(l) → CO2(g)
(c) CO2(g) → CO2(l) → supercritical CO2

10.19 (a)

(b) Gallium has two triple points. The one below 1 atm is a solid, liquid, vapor triple
point. The one at 104 atm is a solid(1), solid(2), liquid triple point.
(c) Increasing the pressure favors the liquid phase, giving the solid/liquid boundary a
negative slope. At 1 atm pressure the liquid phase is more dense than the solid phase.

10.20 The molecules in a liquid crystal can move around, as in viscous liquids, but they have a
restricted range of motion, as in solids.

10.21 Liquid crystal molecules have a rigid rodlike shape with a length four to eight times
greater than their diameter.

Understanding Key Concepts

10.22 The electronegative O atoms are electron rich (red), while the rest of the molecule is
electron poor (blue).

10.23 (a) cubic closest-packed (b) simple cubic


(c) hexagonal closest-packed (d) body-centered cubic

10.24 (a) cubic closest-packed


(b) 1/8 S2- at 8 corners and 1/2 S2- at 6 faces = 4 S2-; 4 Zn2+ inside

10.25 (a) 1/8 Ca2+ at 8 corners = 1 Ca2+; 1/2 O2- at 6 faces = 3 O2-; 1 Ti4+ inside
The formula for perovskite is CaTiO3.
(b) The oxidation number of Ti is +4 to maintain charge neutrality in the unit cell.

10.26 (a) normal boiling point ≈ 300 K; normal melting point ≈ 180 K
(b) (i) solid (ii) gas (iii) supercritical fluid

232
Chapter 10 - Liquids, Solids, and Changes of State
______________________________________________________________________________

10.27

10.28 Here are two possibilities.

10.29 (a), (c), (d)

(b) There are three triple points.


(e) The solid phase that is stable at the higher pressure is more dense. The more dense
phase is diamond.

233
Chapter 10 - Liquids, Solids, and Changes of State
______________________________________________________________________________

Additional Problems
Dipole Moments and Intermolecular Forces

10.30 If a molecule has polar covalent bonds, the molecular shape (and location of lone pairs of
electrons) determines whether a molecule has a dipole moment or not. The molecular
shape will determine whether the bond dipoles cancel or not.

10.31 Dipole-dipole forces arise between molecules that have permanent dipole moments.
London dispersion forces arise between molecules as a result of induced temporary
dipoles.

10.32 (a) CHCl3 has a permanent dipole moment. Dipole-dipole intermolecular forces are
important. London dispersion forces are also present.
(b) O2 has no dipole moment. London dispersion intermolecular forces are important.
(c) polyethylene, CnH2n+2. London dispersion intermolecular forces are important.
(d) CH3OH has a permanent dipole moment. Dipole-dipole intermolecular forces and
hydrogen bonding are important. London dispersion forces are also present.

10.33 (a) Xe has no dipole-dipole forces (b) HF has the largest hydrogen bond forces
(c) Xe has the largest dispersion forces

10.34 For CH3OH and CH4, dispersion forces are small. CH3OH can hydrogen bond; CH4
cannot. This accounts for the large difference in boiling points.
For 1-decanol and decane, dispersion forces are comparable and relatively large along
the C–H chain. 1-decanol can hydrogen bond; decane cannot. This accounts for the
55oC higher boiling point for 1-decanol.

10.35 (a) C8H18 has the larger dispersion forces because of its longer hydrocarbon chain.
(b) HI has the larger dispersion forces because of the larger, more polarizable iodine.
(c) H2Se has the larger dispersion forces because of the more polarizable and less
electronegative Se.

10.36 (a) (b)

(c) (d)

234
Chapter 10 - Liquids, Solids, and Changes of State
______________________________________________________________________________

10.37 (a) (b)

(c) (d)

10.38
SO2 is bent and the individual bond dipole moments add to give the molecule a net
dipole moment.
CO2 is linear and the individual bond dipole moments point in opposite directions to
cancel each other out. CO2 has no net dipole moment.

10.39
In both PCl3 and PCl5 the P–Cl bond is polar covalent. PCl3 is trigonal pyramidal and
the bond dipoles add to give the molecule a net dipole moment. PCl5 is trigonal
bipyramidal and the bond dipoles cancel. PCl5 has no dipole moment.

10.40

10.41

Vapor Pressure and Changes of State

10.42 ∆Hvap is usually larger than ∆Hfusion because ∆Hvap is the heat required to overcome all
intermolecular forces.

10.43 Sublimation is the direct conversion of a solid to a gas. A solid can also be converted to
a gas in two steps; melting followed by vaporization. The energy to convert a solid to a
gas must be the same regardless of the path. Therefore ∆Hsubl = ∆Hfusion + ∆Hvap.

235
Chapter 10 - Liquids, Solids, and Changes of State
______________________________________________________________________________

10.44 (a) Hg(l) → Hg(g)


(b) no change of state, Hg remains a liquid
(c) Hg(g) → Hg(l) → Hg(s)

10.45 (a) solid I2 melts to form liquid I2 (b) no change of state, I2 remains a liquid

10.46 As the pressure over the liquid H2O is lowered, H2O vapor is removed by the pump. As
H2O vapor is removed, more of the liquid H2O is converted to H2O vapor. This
conversion is an endothermic process and the temperature decreases. The combination
of both a decrease in pressure and temperature takes the system across the liquid/solid
boundary in the phase diagram so the H2O that remains turns to ice.

10.47 The normal boiling point for ether is relatively low (34.6oC). As the pressure is reduced
by the pump, the relatively high vapor pressure of the ether equals the external pressure
produced by the pump and the liquid boils.

1 mol H 2 O
10.48 H2O, 18.02 amu; 5.00 g H2O x = 0.2775 mol H2O
18.02 g H2 O
q1 = (0.2775 mol)[36.6 x 10-3 kJ/(K ⋅ mol)](273 K - 263 K) = 0.1016 kJ
q2 = (0.2775 mol)(6.01 kJ/mol) = 1.668 kJ
q3 = (0.2775 mol)(75.3 x 10-3 kJ/(K ⋅ mol)](303 K - 273 K) = 0.6269 kJ
qtotal = q1 + q2 + q3 = 2.40 kJ; 2.40 kJ of heat is required.

1 mol H 2 O
10.49 H2O, 18.02 amu; 15.3 g H2O x = 0.8491 mol H2O
18.02 g H2 O
q1 = (0.8491 mol)[33.6 x 10-3 kJ/(K ⋅ mol)](373 K - 388 K) = -0.4279 kJ
q2 = -(0.8491 mol)(40.67 kJ/mol) = -34.53 kJ
q3 = (0.8491 mol)[75.3 x 10-3 kJ/(K ⋅ mol)](348 K - 373 K) = -1.598 kJ
qtotal = q1 + q2 + q3 = -36.6 kJ; 36.6 kJ of heat is released.

1 mol H 2 O
10.50 H2O, 18.02 amu; 7.55 g H2O x = 0.4190 mol H2O
18.02 g H2 O
q1 = (0.4190 mol)[75.3 x 10-3 kJ/(K ⋅ mol)](273.15 K - 306.65 K) = -1.057 kJ
q2 = -(0.4190 mol)(6.01 kJ/mol) = -2.518 kJ
q3 = (0.4190 mol)[36.6 x 10-3 kJ/(K ⋅ mol)](263.15 K - 273.15 K) = -0.1534 kJ
qtotal = q1 + q2 + q3 = -3.73 kJ; 3.73 kJ of heat is released.

1 mol C2 H5 OH
10.51 C2H5OH, 46.07 amu; 25.0 g C2H5OH x = 0.543 mol C2H5OH
46.07 g C2 H5 OH
q1 = (0.543 mol)[65.7 x 10-3 kJ/(K ⋅ mol)](351.55 K - 366.15 K) = -0.521 kJ

236
Chapter 10 - Liquids, Solids, and Changes of State
______________________________________________________________________________

q2 = -(0.543 mol)(38.56 kJ/mol) = -20.94 kJ


q3 = (0.543 mol)[113 x 10-3 kJ/(K ⋅ mol)](263.15 K - 351.55 K) = -5.42 kJ
qtotal = q1 + q2 + q3 = -26.9 kJ; 26.9 kJ of heat is released.

10.52

10.53

10.54 boiling point = 218oC = 491 K


∆G = ∆Hvap - T∆Svap; At the boiling point (phase change), ∆G = 0
∆ H vap 43.3 kJ/mol
∆Hvap = T∆Svap; ∆Svap = = = 0.0882 kJ/(K⋅ mol) = 88.2 J/(K⋅ mol)
T 491 K

∆ Hfus 2.64 kJ/mol


10.55 ∆ Sfus = = = 0.007 12 kJ/(K⋅ mol) = 7.12 J/(K⋅ mol)
T 371 K

(ln P2 _ ln P1)(R)
10.56 ∆Hvap =
1 1 
 _ 
 T1 T 2 
o
T1 = -5.1 C = 268.0 K; P1 = 100 mm Hg
o
T2 = 46.5 C = 319.6 K; P2 = 760 mm Hg

237
Chapter 10 - Liquids, Solids, and Changes of State
______________________________________________________________________________

[ln (760) _ ln (100)][8.3145 x 10_ 3 kJ/(K • mol)]


∆Hvap = = 28.0 kJ/mol
 1 1 
 _ 
 268.0 K 319.6 K 
(ln P2 _ ln P1)(R)
10.57 ∆Hvap =
1 1 
 _ 
 T1 T 2 

P1 = 100 mm Hg; T1 = 5.4oC = 278.6 K


P2 = 760 mm Hg; T2 = 56.8oC = 330.0 K

[ln(760) _ ln(100)][8.3145 x 10_ 3 kJ/(K • mol)]


∆Hvap = = 30.2 kJ/mol
 1 1 
 _ 
 278.6 K 330.0 K 

∆ H vap  1 1
10.58 ln P2 = ln P1 +  _ 
R  T1 T 2 

∆Hvap = 28.0 kJ/mol


P1 = 100 mm Hg; T1 = -5.1oC = 268.0 K; T2 = 20.0oC = 293.2 K
Solve for P2.
28.0 kJ/mol  1 1 
ln P2 = ln (100) +  _ 
[8.3145 x 10 kJ/(K • mol)]  268.0 K 293.2 K 
_3

ln P2 = 5.6852; P2 = e5.6852 = 294.5 mm Hg = 294 mm Hg

∆ H vap  1 1
10.59 ln P2 = ln P1 +  _ 
R  T1 T 2 

∆Hvap = 30.2 kJ/mol


P1 = 100 mm Hg; T1 = 5.4oC = 278.6 K; T2 = 30.0oC = 303.2 K
Solve for P2.
30.2 kJ/mol  1 1 
ln P2 = ln (100) +  _ 
[8.3145 x 10 kJ/(K • mol)]  278.6 K 303.2 K 
_3

ln P2 = 5.6630; P2 = e5.6620 = 288.0 mm Hg = 288 mm Hg

10.60 T(K) Pvap(mm Hg) ln Pvap 1/T


263 80.1 4.383 0.003 802
273 133.6 4.8949 0.003 663
283 213.3 5.3627 0.003 534
293 329.6 5.7979 0.003 413

238
Chapter 10 - Liquids, Solids, and Changes of State
______________________________________________________________________________

303 495.4 6.2054 0.003 300


313 724.4 6.5853 0.003 195

 ∆ H vap  1
ln Pvap =  _  + C; C = 18.2
 R  T
∆ H vap
slope = -3628 K = _
R
∆Hvap = (3628 K)(R) = (3628 K)[8.3145 x 10-3 kJ/(K⋅ mol)] = 30.1 kJ/mol

10.61 T(K) Pvap(mm Hg) ln Pvap 1/T


500 39.3 3.671 0.002 000
520 68.5 4.227 0.001 923
540 114.4 4.7397 0.001 852
560 191.6 5.2554 0.001 786
580 286.4 5.6574 0.001 724
600 432.3 6.0691 0.001 667

 ∆ H vap  1
ln Pvap =  _  + C; C = 18.1
 R  T
∆ H vap
slope = -7219 K = _
R
∆Hvap = (7219 K)(R) = (7219 K)[8.3145 x 10-3 kJ/(K⋅ mol)] = 60.0 kJ/mol
10.62 ∆Hvap = 30.1 kJ/mol

10.63 ∆Hvap = 60.0 kJ/mol

(ln P2 _ ln P1)(R)
10.64 ∆Hvap =
1 1 
 _ 
 T1 T 2 
P1 = 80.1 mm Hg; T1 = 263 K
P2 = 724.4 mm Hg; T2 = 313 K

239
Chapter 10 - Liquids, Solids, and Changes of State
______________________________________________________________________________

[ln (724.4) _ ln (80.1)][8.3145 x 10_ 3 kJ/(K • mol)]


∆Hvap = = 30.1 kJ/mol
 1 1 
 _ 
 263 K 313 K 
The calculated ∆Hvap and that obtained from the plot in Problem 10.62 are the same.

(ln P2 _ ln P1)(R)
10.65 ∆Hvap =
1 1 
 _ 
 T1 T 2 
P1 = 39.3 mm Hg; T1 = 500 K
P2 = 432.3 mm Hg; T2 = 600 K
[ln(432.3) _ ln(39.3)][8.3145 x 10_ 3 kJ/(K • mol)]
∆Hvap = = 59.8 kJ/mol
 1 1 
 _ 
 500 K 600 K 
The calculated ∆Hvap and that obtained from the plot in Problem 10.63 are consistent
with each other. The value from the slope is 60.0 kJ/mol

Structures of Solids

10.66 molecular solid, CO2, I2; metallic solid, any metallic element;
covalent network solid, diamond; ionic solid, NaCl

10.67 molecular solid, covalent molecules; metallic solid, metal atoms;


covalent network solid, nonmetal atoms; ionic solid, cations and anions

10.68 The unit cell is the smallest repeating unit in a crystal.

10.69 From Table 10.10.


Hexagonal and cubic closest packing are the most efficient because 74% of the available
space is used.
Simple cubic packing is the least efficient because only 52% of the available space is
used.

d
2
(362 pm )2
10.70 Cu is face-centered cubic. d = 362 pm; r= = = 128 pm
8 8
362 pm = 362 x 10-12 m = 3.62 x 10-8 cm
unit cell volume = (3.62 x 10-8 cm)3 = 4.74 x 10-23 cm3
1 mol
mass of one Cu atom = 63.55 g/mol x = 1.055 x 10-22 g/atom
6.022 x 1023 atom
Cu is face-centered cubic; there are therefore four Cu atoms in the unit cell.
unit cell mass = (4 atoms)(1.055 x 10-22 g/atom) = 4.22 x 10-22 g
mass 4.22 x 10_ 22 g
density = = = 8.90 g/cm3
volume 4.74 x 10_ 23 cm3

240
Chapter 10 - Liquids, Solids, and Changes of State
______________________________________________________________________________

10.71 Pb is face-centered cubic. d = 495 pm = 4.95 x 10-8 cm


2 2
d (495 pm )
r= = = 175 pm
8 8
unit cell volume = (4.95 x 10-8 cm)3 = 1.2129 x 10-22 cm3
1 mol
mass of one Pb atom = 207.2 g/mol x = 3.4407 x 10-22 g/atom
6.022 x 1023 atoms
Pb is face-centered cubic; there are therefore four Pb atoms in the unit cell.
mass 4(3.4407 x 10_ 22 g)
density = = _ 22 3
= 11.3 g/cm3
volume 1.2129 x 10 cm

1 mol
10.72 mass of one Al atom = 26.98 g/mol x = 4.480 x 10-23 g/atom
6.022 x 1023 atom
Al is face-centered cubic; there are therefore four Al atoms in the unit cell.
unit cell mass = (4 atoms)(4.480 x 10-23 g/atom) = 1.792 x 10-22 g
mass
density =
volume
unit cell mass 1.792 x 10_ 22 g
unit cell volume = = = 6.640 x 10-23 cm3
density 2.699 g/ cm3
unit cell edge = d = 6.640 x 10_ 23 cm3 = 4.049 x 10-8 cm
3

1m
d = 4.049 x 10-8 cm x = 4.049 x 10-10 m = 404.9 x 10-12 m = 404.9 pm
100 cm

10.73 W is body-centered cubic. d = 317 pm


a = edge = d; b = face diagonal; c = body diagonal
b2 = 2a2
c2 = a2 + b2
c2 = a2 + 2a2 = 3a2
c= 3 a
unit cell body diagonal = 3 d = 3 (317 pm) = 549 pm
10.74 unit cell body diagonal = 4r = 549 pm
549 pm
For W, r = = 137 pm
4

1 mol
10.75 mass of one Na atom = 23.0 g/mol x 23
= 3.82 x 10-23 g/atom
6.022 x 10 atoms
Because Na is body-centered cubic; there are two Na atoms in the unit cell.
unit cell mass = 2(3.82 x 10-23 g) = 7.64 x 10-23 g
unit cell mass 7.64 x 10_ 23 g
unit cell volume = = 3
= 7.87 x 10-23 cm3
density 0.971 g/ cm
unit cell edge = d = 3
7.87 x 10_ 23 cm3 = 4.29 x 10-8 cm = 429 pm

241
Chapter 10 - Liquids, Solids, and Changes of State
______________________________________________________________________________

3 d 3 (429 pm)
4R = 3 d; R= = = 186 pm
4 4

1 mol
10.76 mass of one Ti atom = 47.88 g/mol x 23
= 7.951 x 10-23 g/atom
6.022 x 10 atoms
r = 144.8 pm = 144.8 x 10-12 m
100 cm
r = 144.8 x 10-12 m x = 1.448 x 10-8 cm
1m
Calculate the volume and then the density for Ti assuming it is primitive cubic, body-
centered cubic, and face-centered cubic. Compare the calculated density with the actual
density to identify the unit cell.
For primitive cubic:
d = 2r; volume = d3 = [2(1.448 x 10-8 cm)]3 = 2.429 x 10-23 cm3
unit cell mass 7.951 x 10_ 23 g
density = = _ 23 3
= 3.273 g/cm3
volume 2.429 x 10 cm
For face-centered cubic:
d = 2 2 r; volume = d3 = [2 2 (1.448 x 10-8 cm)]3 = 6.870 x 10-23 cm3
4(7.951 x 10_ 23 g)
density = _ 23 3
= 4.630 g/cm3
6.870 x 10 cm
For body-centered cubic:
From Problems 10.73 and 10.74,
3
4r  4(1.448 x 10_ 8 cm) 
d= ; volume = d3 =  -23
 = 3.739 x 10 cm
3
3  3 
_ 23
2(7.951 x 10 g)
density = _ 23 3
= 4.253 g/cm3
3.739 x 10 cm
The calculated density for a face-centered cube (4.630 g/cm3) is closest to the actual
density of 4.54 g/cm3. Ti crystallizes in the face-centered cubic unit cell.

1 mol
10.77 mass of one Ca = 40.08 g/mol x 23
= 6.656 x 10-23 g/atom
6.022 x 10 atom
unit cell edge = d = 558.2 pm = 5.582 x 10-8 cm
unit cell volume = d3 = (5.582 x 10-8 cm)3 = 1.739 x 10-22 cm3
unit cell mass = (1.739 x 10-22 cm3)(1.55 g/cm3) = 2.695 x 10-22 g
unit cell mass
(a) number of Ca atoms in unit cell =
mass of one Ca atom
2.695 x 10_ 22 g
= = 4.05 = 4 Ca atoms
6.656 x 10_ 23 g/atom
(b) Because the unit cell contains 4 Ca atoms, the unit cell is face-centered cubic.

10.78 Six Na+ ions touch each H- ion and six H- ions touch each Na+ ion.

242
Chapter 10 - Liquids, Solids, and Changes of State
______________________________________________________________________________

10.79 For CsCl: (1/8 x 8 corners), so 1 Cl- and 1 minus per unit cell
1 Cs+ inside, so 1 plus per unit cell

10.80 Na+ H- Na+


← 488 pm → unit cell edge = d = 488 pm; Na–H bond = d/2 = 244 pm

10.81 See Problem 10.73.


body diagonal = 3 d = 3 (412.3 pm) = 714.12 pm
Cs–Cl bond = body diagonal/2 = (714.12 pm)/2 = 357.1 pm
Cs–Cl bond length = r Cs+ + r Cl_
357.1 pm = r Cs+ + r Cl_
357.1 pm = r Cs+ + 181 pm
r Cs+ = 357.1 pm - 181 pm = 176 pm

Phase Diagrams

10.82 (a) gas (b) liquid (c) solid

10.83 (a) H2O(l) → H2O(s)


(b) 380oC is above the critical temperature; therefore, the water cannot be liquefied. At
the higher pressure, it will behave as a supercritical fluid.

10.84

243
Chapter 10 - Liquids, Solids, and Changes of State
______________________________________________________________________________

10.85

10.86 (a) Br2(s) (b) Br2(l)

10.87 (a) O2(l) (b) O2 - supercritical fluid

10.88 Solid O2 does not melt when pressure is applied because the solid is denser than the
liquid and the solid/liquid boundary in the phase diagram slopes to the right.

10.89 Ammonia can be liquefied at 25oC because this temperature is below Tc (132.5oC).
Methane cannot be liquefied at 25oC because this temperature is above Tc (-82.1oC).
Sulfur dioxide can be liquefied at 25oC because this temperature is below Tc (157.8oC).

10.90

The starting phase is benzene as a solid, and


the final phase is benzene as a gas.

244
Chapter 10 - Liquids, Solids, and Changes of State
______________________________________________________________________________

10.91

The starting phase is a gas, and the final


phase is a liquid.

10.92 solid → liquid → supercritical fluid → liquid → solid → gas

10.93 gas → solid → liquid → gas → liquid

General Problems

10.94 Because chlorine is larger than fluorine, the charge separation is larger in CH3Cl
compared to CH3F resulting in CH3Cl having a slightly larger dipole moment.

10.95 Because Ar crystallizes in a face-centered cubic unit cell, there are four Ar atoms in the
unit cell.
1 mol
mass of one Ar atom = 39.95 g/mol x 23
= 6.634 x 10-23 g/atom
6.022 x 10 atom
unit cell mass = 4 atoms x mass of one Ar atom
= 4 atoms x 6.634 x 10-23 g/atom = 2.654 x 10-22 g
mass
density =
volume
unit cell mass 2.654 x 10_ 22 g
unit cell volume = = = 1.635 x 10-22 cm3
density 1.623 g/ cm3
unit cell edge = d = 3 1.635 x 10_ 22 cm3 = 5.468 x 10-8 cm
1m
d = 5.468 x 10-8 cm x = 5.468 x 10-10 m = 546.8 x 10-12 m = 546.8 pm
100 cm

245
Chapter 10 - Liquids, Solids, and Changes of State
______________________________________________________________________________

d
2
(546.8 pm )2
r= = = 193.3 pm
8 8

1 mol
10.96 7.50 g x = 0.037 39 mol Hg
200.6 g
q1 = (0.037 39 mol)[28.2 x 10-3 kJ/(K ⋅ mol)](234.2 K - 223.2 K) = 0.011 60 kJ
q2 = (0.037 39 mol)(2.33 kJ/mol) = 0.087 12 kJ
q3 = (0.037 39 mol)[27.9 x 10-3 kJ/(K ⋅ mol)](323.2 K - 234.2 K) = 0.092 84 kJ
qtotal = q1 + q2 + q3 = 0.192 kJ; 0.192 kJ of heat is required.

10.97

∆ H vap  1 1
10.98 ln P2 = ln P1 +  _ 
R  T1 T 2 
∆Hvap = 40.67 kJ/mol
At 1 atm, H2O boils at 100oC; therefore set
T1 = 100oC = 373 K, and P1 = 1.00 atm.
Let T2 = 95oC = 368 K, and solve for P2. (P2 is the atmospheric pressure in Denver.)
40.67 kJ/mol  1 1 
ln P2 = ln(1) +  _ 
[8.3145 x l 0 kJ/(K • mol)]  373 K 368 K 
_3

ln P2 = -0.1782; P2 = e-0.1782 = 0.837 atm

10.99

10.100 ∆G = ∆H - T∆S; at the melting point (phase change), ∆G = 0.


∆ 9.037 kJ/mol
∆H = T∆S; T = Hfus = = 923 K = 650oC
∆ Sfus 9.79 x 10_ 3 kJ/(K • mol)

10.101 melting point = -23.2oC = 250.0 K


∆G = ∆Hfusion - T∆Sfusion
At the melting point (phase change), ∆G = 0
∆Hfusion = T∆Sfusion
∆ 9.37 kJ/mol
∆Sfusion = Hfusion = = 0.0375 kJ/(K ⋅ mol) = 37.5 J/(K ⋅ mol)
T 250.0 K

246
Chapter 10 - Liquids, Solids, and Changes of State
______________________________________________________________________________

(ln P2 _ ln P1)(R)
10.102 ∆Hvap =
1 1 
 _ 
 T1 T 2 
P1 = 40.0 mm Hg; T1 = -81.6oC =191.6 K
P2 = 400 mm Hg; T2 = -43.9oC = 229.2 K
 kJ 
[ln (400) _ ln (40.0)] 8.3145 x 10_ 3 
 K• mol 
∆Hvap = = 22.36 kJ/mol
 1 1 
 _ 
 191.6 K 229.2 K 
Using ∆Hvap = 22.36 kJ/mol
∆ H vap  1 1
ln P 2 = ln P1 +  _ 
R  T1 T 2 
 R  1 1
(ln P 2 _ ln P1) = _

 ∆ H vap  T1 T 2
1  R  1
_ (ln P 2 _ ln P1) =
 T
T1  ∆ H vap  2

P1 = 40.0 mm Hg; T1 = 191.6 K


P2 = 760 mm Hg
Solve for T2 (the normal boiling point).
 kJ 
 8.3145 x 10_ 3  1
1
_ [ln(760) _ ln(40.0)] K • mol =
191.6 K  22.36 kJ/mol  T2
 
 
1
= 0.004 124 33; T2 = 242.46 K = -30.7oC
T2

∆ H vap  1 1
10.103 (a) ln P2 = ln P1 +  _ 
R  T1 T 2 
 R  1 1
(ln P2 _ ln P1) = _

 ∆ H vap  T1 T 2
1  R  1
_ (ln P2 _ ln P1) =

T1  ∆ H vap  T 2
P1 = 100.0 mm Hg; T1 = -23oC = 250 K
P2 = 760.0 mm Hg
Solve for T2, the normal boiling point for CCl3F.

247
Chapter 10 - Liquids, Solids, and Changes of State
______________________________________________________________________________

 kJ 
 8.3145 x 10_ 3 
1
_ [ln(760.0) _ ln(100.0)] K• mol  = 1
250 K  24.77 kJ/mol  T2
 
 
1
= 0.003 319; T2 = 301.3 K = 28.1oC
T2
∆ H vap 24.77 kJ/mol
(b) ∆Svap = = = 0.082 21 kJ/(K ⋅ mol) = 82.2 J/(K ⋅ mol)
T 301.3 K

(ln P2 _ ln P1)(R)
10.104 ∆Hvap =
1 1 
 _ 
 T1 T 2 
P1 = 100 mm Hg; T1 = -110.3oC = 162.85 K
P2 = 760 mm Hg; T2 = -88.5oC = 184.65 K
 kJ 
[ln (760) _ ln (100)] 8.3145 x 10_ 3 
 K• mol 
∆Hvap = = 23.3 kJ/mol
 1 1 
 _ 
 162.85 K 184.65 K 

∆ H vap  1 1
10.105 ln P 2 = ln P1 +  _ 
R  T1 T 2 
 R  1 1
(ln P 2 _ ln P1) = _

 ∆ H vap  T1 T 2
1  R  1
_ (ln P 2 _ ln P1) =

T1  ∆ H vap  T 2
P1 = 760 mm Hg; T1 = 56.2oC = 329.4 K
P2 = 105 mm Hg Solve for T2.
 kJ 
 8.3145 x 10_ 3  1
1
_ [ln(105) _ ln(760)]  K • mol =
329.4 K  29.1 kJ/mol  T2
 
 
1
= 0.003 601; T2 = 277.7 K = 4.5oC
T2

248
Chapter 10 - Liquids, Solids, and Changes of State
______________________________________________________________________________

10.106

Kr cannot be liquified at room temperature because room temperature is above Tc (-63oC).

10.107 (a) Kr(l) (b) supercritical Kr

10.108 For a body-centered cube


4r
4r = 3 edge; edge =
3
4 3
volume of sphere = πr
3
3
 4r  64 r3
volume of unit cell =   =
 3 3 3
4  8
volume of 2 spheres = 2  π r3  = π r3
3  3
 8 3
 πr 
% volume occupied =   x 100% = 68%
3
 64 r 
3
 
3 3

3 d 3 (287 pm)
10.109 From Problem 10.73, 4r = 3 d; r= = = 124 pm
4 4

10.110 unit cell edge = d = 287 pm = 287 x 10-12 m = 2.87 x 10-8 cm


unit cell volume = d3 = (2.87 x 10-8 cm)3 = 2.364 x 10-23 cm3
unit cell mass = (2.364 x 10-23 cm3)(7.86 g/cm3) = 1.858 x 10-22 g
Fe is body-centered cubic; therefore there are two Fe atoms per unit cell.
1.858 x 10_ 22 g
mass of one Fe atom = = 9.290 x 10-23 g/atom
2 Fe atoms
1 atom
Avogadro's number = 55.85 g/mol x _ 23
= 6.01 x 1023 atoms/mol
9.290 x 10 g

249
Chapter 10 - Liquids, Solids, and Changes of State
______________________________________________________________________________

10.111 unit cell edge = d = 408 pm = 408 x 10-12 m = 4.08 x 10-8 cm


unit cell volume = (4.08 x 10-8 cm)3 = 6.792 x 10-23 cm3
unit cell mass = (10.50 g/cm3)(6.792 x 10-23 cm3) = 7.132 x 10-22 g
Ag is face-centered cubic; therefore there are four Ag atoms in the unit cell.
7.132 x 10_ 22 g
mass of one Ag atom = = 1.783 x 10-22 g/atom
4 Ag atoms
1 atom
Avogadro's number = 107.9 g/mol x = 6.05 x 1023 atoms/mol
1.783 x 10_ 22 g

10.112 (a) unit cell edge = 2 r Cl_ + 2 r Na+ = 2(181 pm) + 2(97 pm) = 556 pm
(b) unit cell edge = d = 556 pm = 556 x 10-12 m = 5.56 x 10-8 cm
unit cell volume = (5.56 x 10-8 cm)3 = 1.719 x 10-22 cm3
The unit cell contains 4 Na+ ions and 4 Cl- ions.
1 mol
mass of one Na+ ion = 22.99 g/mol x 23
= 3.818 x 10-23 g/Na+
6.022 x 10 ions
1 mol
mass of one Cl- ion = 35.45 g/mol x 23
= 5.887 x 10-23 g/Cl-
6.022 x 10 ions
unit cell mass = 4(3.818 x 10 g) + 4(5.887 x 10-23 g) = 3.882 x 10-22 g
-23

unit cell mass 3.882 x 10_ 22 g


density = = = 2.26 g/cm3
unit cell volume 1.719 x 10_ 22 cm3

10.113 (a) (1/2 Nb/face)(6 faces) = 3 Nb; (1/4 O/edge)(12 edges) = 3 O


(b) NbO
(c) The oxidation state of Nb is +2.

10.114 Al2O3, ionic (greater lattice energy than NaCl because of higher ion charges);
F2, dispersion; H2O, dipole-dipole, H–bonding; Br2, dispersion (larger and more
polarizable than F2), ICl, dipole-dipole, NaCl, ionic

rank according to normal boiling points: F2 < Br2 < ICl < H2O < NaCl < Al2O3

10.115 Ag2Te, 343. 33 amu; 529 pm = 529 x 10-12 m = 529 x 10-10 cm


unit cell volume = (529 x 10-10 cm)3 = 1.48 x 10-22 cm3
unit cell mass = (1.48 x 10-22 cm3)(7.70 g/cm3) = 1.14 x 10-21 g
343.33 g Ag2 Te / mol
mass of one Ag2Te = 23
= 5.70 x 10-22 g Ag2Te/formula
6.022 x 10 Ag2 Te formula units / mol
unit
1.14 x 10_ 21 g/unit cell
Ag2Te formula units/unit cell = = 2 Ag2Te/unit cell
5.70 x 10_ 22 g/ Ag2 Te
2 Ag2 Te 2 Ag
Ag/unit cell = x = 4 Ag/unit cell
unit cell Ag2 Te

250
Chapter 10 - Liquids, Solids, and Changes of State
______________________________________________________________________________

10.116 (a)

(b) (i) solid (ii) gas (iii) liquid (iv) liquid (v) solid
Multi-Concept Problems

10.117 C2H5OH(l) → C2H5OH(g)


Calculate ∆H and ∆S for this process and assume they do not change as a function of
temperature.
∆Ho = ∆Hof(C2H5OH(g)) - ∆Hof(C2H5OH(l))
∆Ho = [(1 mol)(-235.1 kJ/mol)] - [(1 mol)(-277.7 kJ/mol)] = 42.6 kJ
∆So = So(C2H5OH(g)) - So(C2H5OH(l))
∆So = [(1 mol)(282.6 J/(K ⋅ mol))] - [(1 mol)(161 J/(K ⋅ mol))] = 122 J/K
∆So = 122 x 10-3 kJ/K)
∆Go = ∆Ho - T∆So and at the boiling point, ∆G = 0
0 = ∆Ho - Tbp∆So
Tbp∆So = ∆Ho
∆H o 42.6 kJ
Tbp = = = 349 K
∆S o
122 x 10_ 3 kJ/K
Tbp = 349 - 273 = 76oC

∆ H vap  1 1
ln P2 = ln P1 +  _ 
R  T1 T 2 
∆Hvap = 42.6 kJ/mol
At 1 atm, C2H5OH boils at 349 K; therefore set
T1 = 349 K, and P1 = 1.00 atm.
Let T2 = 25oC = 298 K, and solve for P2.
P2 is the vapor pressure of C2H5OH at 25oC.
42.6 kJ/mol  1 1 
ln P2 = ln(1.00) +  _ 
[8.3145 x l 0 kJ/(K • mol)]  349 K 298 K 
_3

ln P2 = -2.512; P2 = e-2.512 = 0.0811 atm

251
Chapter 10 - Liquids, Solids, and Changes of State
______________________________________________________________________________

760 mm Hg
P2 = 0.0811 atm x = 61.6 mm Hg
1.00 atm

10.118 (a) Let the formula of magnetite be FexOy, then FexOy + y CO → x Fe + y CO2
 1.00 atm 
 751 mm Hg x (1.136 L)
PV  760 mm Hg 
n CO2 = y = = = 0.04590 mol CO2
RT  L • atm 
 0.082 06 (298 K)
 K • mol 
0.04590 mol CO2 = mol of O in FexOy
16.0 g O
mass of O in FexOy = 0.04590 mol O x = 0.7345 g O
1 mol O
mass of Fe in FexOy = 2.660 g - 0.7345 g = 1.926 g Fe
1 mol Fe
(b) mol Fe in magnetite = 1.926 g Fe x = 0.0345 mol Fe
55.85 g Fe
formula of magnetite: Fe 0.0345 O 0.0459 (divide each subscript by the smaller)
Fe 0.0345 / 0.0345 O 0.0459 / 0.0345
FeO 1.33 (multiply both subscripts by 3)
Fe (1 x 3) O (1.33 x 3); Fe3O4
(c) unit cell edge = d = 839 pm = 839 x 10-12 m
100 cm
d = 839 x 10-12 m x = 8.39 x 10-8 cm
1 m
unit cell volume = d3 = (8.39 x 10-8 cm)3 = 5.91 x 10-22 cm3
unit cell mass = (5.91 x 10-22 cm3)(5.20 g/cm3) = 3.07 x 10-21 g
 1.926 g Fe 
mass of Fe in unit cell =  (3.07 x 10_ 21 g) = 2.22 x 10-21 g Fe
 2.660 g 
 0.7345 g O 
mass of O in unit cell =  (3.07 x 10_ 21 g) = 8.47 x 10-22 g O
 2.660 g 
6.022 x 1023 atoms/mol
Fe atoms in unit cell = 2.22 x 10_ 21 g x = 24 Fe atoms
55.847 g/mol

6.022 x 1023 atoms/mol


O atoms in unit cell = 8.47 x 10_ 22 g x = 32 O atoms
16.00 g/mol

 1.00 atm 
 740 mm Hg x (4.00 L)
PV  760 mm Hg 
10.119 (a) n H2 = = = 0.160 mol H2
RT  L • atm 
 0.08206 (296 K)
 K • mol 

M = Group 3A metal; 2 M(s) + 6 H+(aq) → 2 M3+(aq) + 3 H2(g)

252
Chapter 10 - Liquids, Solids, and Changes of State
______________________________________________________________________________

2 mol M
nM = 0.160 mol H2 x = 0.107 mol M
3 mol H 2
mass M = 1.07 cm3 x 2.70 g/cm3 = 2.89 g M
2.89 g M
molar mass M = = 27.0 g/mol; The Group 3A metal is Al
0.107 mol M
1 mol
(b) mass of one Al atom = 26.98 g/mol x 23
= 4.48 x 10-23 g/atom
6.022 x 10 atoms
-12
unit cell edge = d = 404 pm = 404 x 10 m
100 cm
d = 404 x 10-12 m x = 4.04 x 10-8 cm
1 m
unit cell volume = d3 = (4.04 x 10-8 cm)3 = 6.59 x 10-23 cm3
Calculate the density of Al assuming it is primitive cubic, body-centered cubic, and face-
centered cubic. Compare the calculated density with the actual density to identify the
unit cell.
For primitive cubic:
unit cell mass (1 Al)(4.48 x 10_ 23 g/Al atom)
density = = = 0.680 g/cm3
unit cell volume 6.59 x 10_ 23 cm3

For body-centered cubic:


unit cell mass (2 Al)(4.48 x 10_ 23 g/Al atom)
density = = _ 23 3
= 1.36 g/cm3
unit cell volume 6.59 x 10 cm
For face-centered cubic:
unit cell mass (4 Al)(4.48 x 10_ 23 g/Al atom)
density = = = 2.72 g/cm3
unit cell volume 6.59 x 10_ 23 cm3
The calculated density for a face-centered cube (2.72 g/cm3) is closest to the actual
density of 2.70 g/cm3. Al crystallizes in the face-centered cubic unit cell.
d
2
(404 pm )2
(c) r = = = 143 pm
8 8

10.120 (a) M = alkali metal; 500.0 mL = 0.5000 L; 802oC = 1075 K


 1.00 atm 
12.5 mm Hg x (0.5000 L)
PV  760 mm Hg 
nM = = = 9.32 x 10-5 mol M
RT  L • atm 
 0.082 06 (1075 K)
 K • mol 
1.62 mm = 1.62 x 10-3 m; crystal volume = (1.62 x 10-3 m)3 = 4.25 x 10-9 m3
M atoms in crystal = (9.32 x 10-5 mol)(6.022 x 1023 atoms/mol) = 5.61 x 1019 M atoms
Because M is body-centered cubic, only 68% (Table 10.10) of the total volume is
occupied by M atoms.
(0.68)(4.25 x 10_ 9 m)
volume of M atom = 19
= 5.15 x 10-29 m3/M atom
5.61 x 10 M atoms

253
Chapter 10 - Liquids, Solids, and Changes of State
______________________________________________________________________________

4 3
volume of a sphere = πr
3
3(volume) 3 3(5.15 x 10_ 29 m3)
rM = 3 = = 2.31 x 10-10 m = 231 x 10-12 m = 231 pm
4π 4π
(b) The radius of 231 pm is closest to that of K.
(c) 1.62 mm = 0.162 cm
(9.32 x 10_ 5 mol)(39.1 g/mol)
density of solid = 3
= 0.857 g/cm3
(0.162 cm )
(9.32 x 10_ 5 mol)(39.1 g/mol)
density of vapor = 3
= 7.29 x 10-6 g/cm3
500.0 cm

10.121 (a)
 1.00 atm 
 755 mm Hg x (0.500 L)
PV  760 mm Hg 
n X2 = = = 0.0203 mol X2
RT  L • atm 
 0.082 06 (298 K)
 K • mol 
M(s) + 1/2 X2(g) → MX(s)
1 mol M
mol M = 0.0203 mol X2 x = 0.0406 mol M
1 / 2 mol X 2
1.588 g M
molar mass M = = 39.1 g/mol; atomic mass = 39.1 amu ; M = K
0.0406 mol M
(b) From Figure 6.1, the radius for K+ is ~140 pm.
unit cell edge = 535 pm = 2 r K+ + 2 r X_
535 pm _ 2 r K+ 535 pm _ 2(140 pm)
r X_ = = = 128 pm
2 2
From Figure 6.2, X- = F-
(c) Because the cation and anion are of comparable size, the anions are not in contact
with each other.

(d) unit cell contents: 1/8 F- at 8 corners and 1/2 F- at 6 faces = 4 F-


1/4 K+ at 12 edges and 1 K+ inside = 4 K+
39.098 g/mol
mass of one K+ = 23 +
= 6.493 x 10-23 g/K+
6.022 x 10 K /mol

254
Chapter 10 - Liquids, Solids, and Changes of State
______________________________________________________________________________

18.998 g/mol
mass of one F- = 23 _
= 3.155 x 10-23 g/F-
6.022 x 10 F /mol
unit cell mass = (4 K+)(6.493 x 10-23 g/K+) + (4 F-)(3.155 x 10-23 g/F-) = 3.859 x 10-22 g
unit cell volume = [(535 x 10-12 m)(100 cm/m)]3 = 1.531 x 10-22 cm3
mass of unit cell 3.859 x 10_ 22 g
density = = _ 22 3
= 2.52 g/cm3
volume of unit cell 1.531 x 10 cm
(e) K(s) + 1/2 F2(g) → KF(s) is a formation reaction.
_ 22.83 kJ
∆Hof(KF) = = -562 kJ/mol
0.0406 mol

255
11 Solutions and Their Properties

11.1 Toluene is nonpolar and is insoluble in water.


Br2 is nonpolar but because of its size is polarizable and is soluble in water.
KBr is an ionic compound and is very soluble in water.
toluene < Br2 < KBr (solubility in H2O)

11.2 (a) Na+ has the larger (more negative) hydration energy because the Na+ ion is smaller
than the Cs+ ion and water molecules can approach more closely and bind more tightly to
the Na+ ion.
(b) Ba2+ has the larger (more negative) hydration energy because of its higher charge.

11.3 NaCl, 58.44 amu; 1.00 mol NaCl = 58.44 g


1.00 L H2O = 1000 mL = 1000 g (assuming a density of 1.00 g/mL)
58.44 g
mass % NaCl = x 100% = 5.52 mass %
1000 g + 58.44 g

mass of CO2
11.4 ppm = x 106 ppm
total mass of solution
total mass of solution = density x volume = (1.3 g/L)(1.0 L) = 1.3 g
mass of CO2
35 ppm = x 106 ppm
1.3 g
(35 ppm)(1.3 g)
mass of CO2 = 6
= 4.6 x 10-5 g CO2
10 ppm

11.5 Assume 1.00 L of sea water.


mass of 1.00 L = (1000 mL)(1.025 g/mL) = 1025 g
mass NaCl 1025 g x 3.50
x 100% = 3.50 mass %; mass NaCl = = 35.88 g
1025 g 100
There are 35.88 g NaCl per 1.00 L of solution.
 1 mol NaCl 
 35.88 g NaCl x 
 58.44 g NaCl 
M= = 0.614 M
1.00 L

1 kg
11.6 C27H46O, 386.7 amu; CHCl3, 119.4 amu; 40.0 g x = 0.0400 kg
1000 g
 1 mol 
 0.385 g x 
mol C27 H 46 O  386.7 g 
molality = = = 0.0249 mol/kg = 0.0249 m
kg CHCl3 0.0400 kg

255
Chapter 11 - Solutions and Their Properties
______________________________________________________________________________

mol C27 H 46 O
XC27 H46 O =
mol C27 H 46 O + mol CHCl3
 1 mol 
 0.385 g x 
 386.7 g 
XC27 H46 O = = 2.96 x 10-3
 1 mol   1 mol 
 0.385 g x  +  40.0 g x 
 386.7 g   119.4 g 

11.7 CH3CO2Na, 82.03 amu


 1 kg H 2 O 
kg H2O = (0.150 mol CH3 CO2 Na)  = 0.300 kg H2O
 0.500 mol CH3 CO2 Na 
82.03 g CH3 CO2 Na
mass CH3CO2Na = 0.150 mol CH3CO2Na x = 12.3 g CH3CO2Na
1 mol CH3 CO2 Na
mass of solution needed = 300 g + 12.3 g = 312 g

11.8 Assume you have a solution with 1.000 kg (1000 g) of H2O. If this solution is 0.258 m,
then it must also contain 0.258 mol glucose.
180.2 g
mass of glucose = 0.258 mol x = 46.5 g glucose
1 mol
mass of solution = 1000 g + 46.5 g = 1046.5 g
density = 1.0173 g/mL
1 mL
volume of solution = 1046.5 g x = 1028.7 mL
1.0173 g
1L 0.258 mol
volume = 1028.7 mL x = 1.029 L; molarity = = 0.251 M
1000 mL 1.029 L

11.9 Assume 1.00 L of solution.


mass of 1.00 L = (1.0042 g/mL)(1000 mL) = 1004.2 g of solution
60.05 g CH3 CO2 H
0.500 mol CH3CO2H x = 30.02 g CH3CO2H
1 mol CH3 CO2 H
0.500 mol
1004.2 g - 30.02 g = 974.2 g = 0.9742 kg of H2O; molality = = 0.513 m
0.9742 kg

11.10 Assume you have 100.0 g of seawater.


mass NaCl = (0.0350)(100.0 g) = 3.50 g NaCl
mass H2O = 100.0 g - 3.50 g = 96.5 g H2O
1 mol
NaCl, 58.44 amu; mol NaCl = 3.50 g x = 0.0599 mol NaCl
58.44 g
1 kg 0.0599 mol
mass H2O = 96.5 g x = 0.0965 kg H2O; molality = = 0.621 m
1000 g 0.0965 kg

256
Chapter 11 - Solutions and Their Properties
______________________________________________________________________________

M 3.2 x 10_ 2 M
11.11 M = k ⋅ P; k = = = 3.2 x 10-2 mol/(L⋅ atm)
P 1.0 atm

11.12 (a) M = k ⋅ P = [3.2 x 10-2 mol/(L ⋅ atm)](2.5 atm) = 0.080 M


(b) M = k ⋅ P = [3.2 x 10-2 mol/(L ⋅ atm)](4.0 x 10-4 atm) = 1.3 x 10-5 M

11.13 C7H6O2, 122.1 amu; C2H6O, 46.07 amu


 1 mol 
100 g x 
mol C2 H6 O  46.07 g 
Xsolv = = = 0.981
mol C2 H6 O + mol C7 H6 O2  1 mol   1 mol 
100 g x  +  5.00 g x 
 46.07 g   122.1 g 
Psoln = Psolv ⋅ Xsolv = (100.5 mm Hg)(0.981) = 98.6 mm Hg

11.14 Psoln = Psolv ⋅ Xsolv; Psoln = (55.3 _ 1.30) mm Hg = 0.976


Xsolv =
Psolv 55.3 mm Hg
NaBr dissociates into two ions in aqueous solution.
mol H 2 O
Xsolv =
mol H 2 O + mol Na + + mol Br _
 1 mol 
 250 g x 
 18.02 g 
Xsolv = 0.976 =
 1 mol 
 250 g x  + x mol Na + + x mol Br _
 18.02 g 
13.9 mol
0.976 = ; solve for x.
13.9 mol + 2 x mol
0.976(13.9 mol + 2x mol) = 13.9 mol
13.566 mol + 1.952 x mol = 13.9 mol
1.952 x mol = 13.9 mol - 13.566 mol
13.9 mol _ 13.566 mol
x mol = = 0.171 mol
1.952
x = 0.171 mol Na+ = 0.171 mol Br- = 0.171 mol NaBr
102.9 g
NaBr, 102.9 amu; mass NaBr = 0.171 mol x = 17.6 g NaBr
1 mol

11.15 At any given temperature, the vapor pressure of a solution is lower than the vapor
pressure of the pure solvent. The upper curve represents the vapor pressure of the pure
solvent. The lower curve represents the vapor pressure of the solution.

11.16 C2H5OH, 46.07 amu; H2O, 18.02 amu


1 mol C2 H5 OH
(a) 25.0 g C2 H5 OH x = 0.5426 mol C2H5OH
46.07 g C2 H5 OH

257
Chapter 11 - Solutions and Their Properties
______________________________________________________________________________

1 mol H 2 O
100.0 g H2O x = 5.549 mol H2O
18.02 g H 2 O
0.5426 mol
XC2 H5 OH = = 0.08907
0.5426 mol + 5.549 mol
5.549 mol
X H2 O = = 0.9109
0.5426 mol + 5.549 mol
Psoln = XC2 H5 OH PCo 2 H5 OH + X H2 O PoH2 O
Psoln = (0.08907)(61.2 mm Hg) + (0.9109)(23.8 mm Hg) = 27.1 mm Hg
1 mol C2 H5 OH
(b) 100 g C2 H5 OH x = 2.171 mol C2H6O
46.07 g C2 H5 OH
1 mol H 2 O
25.0 g H2O x = 1.387 mol H2O
18.02 g H2 O
2.171 mol
XC2 H5 OH = = 0.6102
2.171 mol + 1.387 mol
1.387 mol
X H2 O = = 0.3898
2.171 mol + 1.387 mol
Psoln = XC2 H5 OH PoC2 H5 OH + X H2 O PoH2 O
Psoln = (0.6102)(61.2 mm Hg) + (0.3898)(23.8 mm Hg) = 46.6 mm Hg

11.17 (a) Because the vapor pressure of the solution (red curve) is higher than that of the first
liquid (green curve), the vapor pressure of the second liquid must be higher than that of
the solution (red curve). Because the second liquid has a higher vapor pressure than the
first liquid, the second liquid has a lower boiling point.

(b)

o
C • kg
11.18 C9H8O4, 180.2 amu; CHCl3 is the solvent. For CHCl3, Kb = 3.63
mol
1 kg
75.00 g x = 0.075 00 kg
1000 g

258
Chapter 11 - Solutions and Their Properties
______________________________________________________________________________

 1 mol  
 1.50 g x 
 o
C • kg   180.2 g  
∆Tb = Kb ⋅ m =  3.63   = 0.40oC
 mol 

0.075 00 kg

 
 
Solution boiling point = 61.7 C + ∆Tb = 61.7 C + 0.40oC = 62.1oC
o o

11.19 MgCl2, 95.21 amu


1 kg
110 g x = 0.110 kg
1000 g
 1 mol  
  7.40 g x 
 C • kg  
o
95.21 g  
∆Tf = Kf ⋅ m ⋅ i = 1.86  o
(2.7) = 3.55 C
 mol  0.110 kg

 
 
Solution freezing point = 0.00 C - ∆Tf = 0.00 C - 3.55 C = -3.55oC
o o o

11.20 ∆Tf = Kf ⋅ m ⋅ i; For KBr, i = 2.


Solution freezing point = -2.95oC = 0.00oC - ∆Tf; ∆Tf = 2.95oC
∆ Tf 2. 95o C
m= = = 0.793 mol/kg = 0.793 m
K f • i 1.86 C • kg (2)
o
 
 mol 

11.21 HCl, 36.46 amu; ∆Tf = Kf ⋅ m ⋅ i


1 kg
190 g x = 0.190 kg
1000 g
Solution freezing point = - 4.65oC = 0.00oC - ∆Tf; ∆Tf = 4.65oC
∆ Tf 4. 65o C
i= = = 1.9
Kf • m  1 mol 
 9.12 g x 
 o
C • kg  36.46 g 
1.86 
 mol  0.190 kg 
 
 

11.22 The red curve represents the vapor pressure of pure chloroform.
(a) The normal boiling point for a liquid is the temperature where the vapor pressure of
the liquid equals 1 atm (760 mm Hg). The approximate boiling point of pure chloroform
is 62oC.
(b) The approximate boiling point of the solution is 69oC.
∆Tb = 69oC - 62oC = 7oC

259
Chapter 11 - Solutions and Their Properties
______________________________________________________________________________

∆ Tb o
7 C
∆Tb = Kb ⋅ m; m= = = 2 mol/kg = 2 m
Kb
o
C• kg
3.63
mol

11.23 For CaCl2 there are 3 ions (solute particles)/CaCl2


Π = MRT; For CaCl2, Π = 3MRT
 L • atm 
Π = (3)(0.125 mol/L)  0.082 06 (310 K) = 9.54 atm
 K • mol 

Π (3.85 atm)
11.24 Π = MRT; M= = = 0.156 M
RT  L • atm 
 0.082 06 (300 K)
 K • mol 

∆ Tf 2.10o C
11.25 ∆Tf = Kf ⋅ m; m = = = 0.0557 mol/kg = 0.0557 m
Kf
o
C • kg
37.7
mol
1 kg
35.00 g x = 0.03500 kg
1000 g
mol
mol = 0.0557 x 0.03500 kg = 0.001 95 mol naphthalene
kg
0.250 g naphthalene
molar mass of naphthalene = = 128 g/mol
0.001 95 mol naphthalene

 1 atm 
149 mm Hg x 
Π  760 mm Hg 
11.26 Π = MRT; M = = = 8.02 x 10-3 M
RT  L • atm 
 0.08206 (298 K)
 K • mol 
300.0 mL = 0.3000 L
(8.02 x 10-3 mol/L)(0.3000 L) = 0.002 406 mol sucrose
0.822 g sucrose
molar mass of sucrose = = 342 g/mol
0.002 406 mol sucrose

260
Chapter 11 - Solutions and Their Properties
______________________________________________________________________________

11.27 (a) and (c)

(b) The mixture will begin to boil at ~50oC.


(d) After two cycles of boiling and condensing, the approximate composition of the
liquid would 90% dichloromethane and 10% chloroform.

11.28 Both solvent molecules and small solute particles can pass through a semipermeable
dialysis membrane. Only large colloidal particles such as proteins can’t pass through.
Only solvent molecules can pass through a semipermeable membrane used for osmosis.

Understanding Key Concepts

11.29 The upper curve is pure ether.


(a) The normal boiling point for ether is the temperature where the upper curve
intersects the 760 mm Hg line, ~ 37oC.
(b) ∆Tb _3oC
∆ o
3C
∆Tb = Kb ⋅ m; m = Tb = _1.5 mol/kg _1.5 m
K b 2.02 C • kg
o

mol
11.30 (a) < (b) < (c)

11.31 At any given temperature, the vapor pressure of a mixture of two pure liquids falls
between the individual vapor pressures of the two pure liquids themselves. Because the
vapor pressure of the mixture is greater than the vapor pressure of the solvent, the second
liquid is more volatile (has a higher vapor pressure) than the solvent.

11.32 Assume that only the blue (open) spheres (solvent) can pass
through the semipermeable membrane. There will be a net
transfer of solvent from the right compartment (pure
solvent) to the left compartment (solution) to achieve
equilibrium.

11.33 At point 1, the temperature should be near the boiling point of the lower boiling solvent,
CHCl3, approximately 62oC.

261
Chapter 11 - Solutions and Their Properties
______________________________________________________________________________

At point 3, the temperature should be about halfway between the two boiling points at
approximately 70oC.
At point 2, the temperature should be about halfway between the temperatures at points 1
and 3, approximately 66oC.

11.34 The vapor pressure of the NaCl solution is lower than that of pure H2O. More H2O
molecules will go into the vapor from the pure H2O than from the NaCl solution. More
H2O vapor molecules will go into the NaCl solution than into pure H2O. The result is
represented by (b).

11.35 (b) ~95oC

Additional Problems
Solutions and Energy Changes

11.36 The surface area of a solid plays an important role in determining how rapidly a solid
dissolves. The larger the surface area, the more solid-solvent interactions, and the more
rapidly the solid will dissolve. Powdered NaCl has a much larger surface area than a
large block of NaCl, and it will dissolve more rapidly.

11.37 (a) a gas in a liquid – carbonated soft drink


(b) a solid in a solid – metal alloys (14-karat gold)
(c) a liquid in a solid – dental amalgam (Hg in Ag)

11.38 Substances tend to dissolve when the solute and solvent have the same type and
magnitude of intermolecular forces; thus the rule of thumb "like dissolves like."

11.39 Both Br2 and CCl4 are nonpolar, and intermolecular forces for both are dispersion forces.
H2O is a polar molecule with dipole-dipole forces and hydrogen bonding. Therefore,
Br2 is more soluble in CCl4.

11.40 Energy is required to overcome intermolecular forces holding solute particles together in
the crystal. For an ionic solid, this is the lattice energy. Substances with higher lattice
energies tend to be less soluble than substances with lower lattice energies.

11.41 SO42- has the larger hydration energy because of its higher charge. Both SO42- and ClO4-
are comparable in size, so size is not a factor.

262
Chapter 11 - Solutions and Their Properties
______________________________________________________________________________

11.42 Ethyl alcohol and water are both polar with small dispersion forces. They both can
hydrogen bond, and are miscible.
Pentyl alcohol is slightly polar and can hydrogen bond. It has, however, a relatively
large dispersion force because of its size, which limits its water solubility.

11.43 The intermolecular forces associated with octane are dispersion forces. Both pentyl
alcohol and methyl alcohol can hydrogen bond. Pentyl alcohol has relatively large
dispersion forces because of its size. Methyl alcohol does not. Pentyl alcohol is soluble
in octane; methyl alcohol is not.

11.44 CaCl2, 110.98 amu


For a 1.00 m solution:
heat released = 81,300 J
mass of solution = 1000 g H2O + 110.98 g CaCl2 = 1110.98 g
q 81,300 J
∆T = = = 17.5 K =
(specific heat)(mass of solution) [4.18 J/(K • g)](1110.98 g)
17.5oC
Final temperature = 25.0oC + 17.5oC = 42.5oC

11.45 NH4ClO4, 117.48 amu


For a 1.00 m solution:
heat absorbed = 33,500 J
mass of solution = 1000 g H2O + 117.48 g NH4ClO4 = 1117.48 g
q _ 33,500 J
∆T = = = -7.2 K =
(specific heat)(mass of solution) [4.18 J/(K• g)](1117.48 g)
-7.2oC
Final temperature = 25.0oC - 7.2oC = 17.8oC

Units of Concentration

moles of solute moles of solute


11.46 molarity = ; molality =
liters of solution kg of solvent

11.47 A saturated solution contains enough solute so that there is an equilibrium between
dissolved solute and undissolved solid.
A supersaturated solution contains a greater-than-equilibrium amount of solute.

11.48 (a) Dissolve 0.150 mol of glucose in water; dilute to 1.00 L.


(b) Dissolve 1.135 mol of KBr in 1.00 kg of H2O.
(c) Mix together 0.15 mol of CH3OH with 0.85 mol of H2O.

11.49 (a) Dissolve 15.5 mg urea in 100 mL water


(b) Choose a K+ salt, say KCl, and dissolve 0.0075 mol (0.559 g) in water; dilute to100
mL.

263
Chapter 11 - Solutions and Their Properties
______________________________________________________________________________

11.50 C7H6O2, 122.12 amu, 165 mL = 0.165 L


mol C7H6O2 = (0.0268 mol/L)(0.165 L) = 0.004 42 mol
122.12 g
mass C7H6O2 = 0.004 42 mol x = 0.540 g
1 mol
Dissolve 4.42 x 10-3 mol (0.540 g) of C7H6O2 in enough CHCl3 to make 165 mL of
solution.

11.51 C7H6O2, 122.12 amu


122.12 g C7 H6 O2
0.0268 mol C7H6O2 x = 3.27 g C7H6O2
1 mol C7 H6 O2
Dissolve 3.27 g of C7H6O2 in 1.000 kg of CHCl3, and take 165 mL of the solution.

11.52 (a) KCl, 74.6 amu


A 0.500 M KCl solution contains 37.3 g of KCl per 1.00 L of solution.
A 0.500 mass % KCl solution contains 5.00 g of KCl per 995 g of water.
The 0.500 M KCl solution is more concentrated (that is, it contains more solute per
amount of solvent).
(b) Both solutions contain the same amount of solute. The 1.75 M solution contains less
solvent than the 1.75 m solution. The 1.75 M solution is more concentrated.

11.53 (a) KI, 166.00 amu; KBr, 119.00 amu; assume 1.000 L = 1000 mL = 1000 g solution
mass KI
10 ppm = x 106 ; mass KI = 0.010 g
1000 g
mass KBr
10,000 ppb = x 109 ; mass KBr = 0.010 g
1000 g
Both solutions contain the same mass of solute in the same amount of solvent. Because
the molar mass of KBr is less than that of KI, the number of moles of KBr is larger than
the number of moles of KI. The KBr solution has a higher molarity than the KI solution.
(b) Because the mass % of the two solutions is the same, they both contain the same
mass of solute and solution. Because the molar mass of KCl is less than that of citric
acid, the number of moles of KCl is larger than the number of moles of citric acid. The
KCl solution has a higher molarity than the citric acid solution.

11.54 (a) C6H8O7, 192.12 amu


192.12 g C6 H8 O7
0.655 mol C6H8O7 x = 126 g C6H8O7
1 mol C6 H8 O7
126 g
mass % C6H8O7 = x 100% = 11.2 mass %
126 g + 1000 g
(b) 0.135 mg = 0.135 x 10-3 g
(5.00 mL H2O)(1.00 g/mL) = 5.00 g H2O
0.135 x 10_ 3 g
mass % KBr = x 100% = 0.002 70 mass % KBr
(0.135 x 10_ 3 g) + 5.00 g

264
Chapter 11 - Solutions and Their Properties
______________________________________________________________________________

5.50 g
(c) mass % aspirin = x 100% = 3.65 mass % aspirin
5.50 g + 145 g

0.655 mol
11.55 (a) molality = = 0.655 m
1.00 kg
(b) KBr, 119.00 amu; 5.00 g = 0.005 00 kg

 1 mol 
 0.135 x 10_ 3 g x 
 119.00 g 
molality = = 2.27 x 10-4 mol/kg = 2.27 x 10-4 m
0.005 00 kg

(c) C9H8O4, 180.16 amu; 145 g = 0.145 kg


 1 mol 
 5.50 g x 
 180.16 g 
molality = = 0.211 mol/kg = 0.211 m
0.145 kg

11.56 PO3 = Ptotal • XO3


PO3 1.6 x 10_ 9 atm
XO3 = = _2
= 1.2 x 10-7
P total 1.3 x 10 atm
Assume one mole of air (29 g/mol)
mol O3 = nair ⋅ XO3 = (1 mol)(1.2 x 10-7) = 1.2 x 10-7 mol O3
48.0 g
O3, 48.00 amu; mass O3 = 1.2 x 10-7 mol x = 5.8 x 10-6 g O3
1 mol
5.8 x 10_ 6 g
ppm O3 = x 106 = 0.20 ppm
29 g

11.57 Assume 1 mL of blood weighs 1 g. 1 dL = 0.1 L = 100 mL = 100 g


10 x 10_ 6 g
ppb = x 109 = 100 ppb
100 g

 1 mol 
 25.0 g x 
 98.08 g 
11.58 (a) H2SO4, 98.08 amu; molality = = 0.196 mol/kg = 0.196 m
1.30 kg
(b) C10H14N2, 162.23 amu; CH2Cl2, 84.93 amu
1 mol C10 H14 N 2
2.25 g C10H14N2 x = 0.0139 mol C10H14N2
162.23 g C10 H14 N 2
1 mol CH 2 Cl2
80.0 g CH2Cl2 x = 0.942 mol CH2Cl2
84.93 g CH 2 Cl2

265
Chapter 11 - Solutions and Their Properties
______________________________________________________________________________

0.0139 mol
XC10 H14 N2 = = 0.0145
0.942 mol + 0.0139 mol
0.942 mol
XCH2Cl2 = = 0.985
0.942 mol + 0.0139 mol

11.59 NaOCl, 74.44 amu


A 5.0 mass % aqueous solution of NaOCl contains 5.0 g NaOCl and 95 g H2O.
 1 mol 
 5.0 g x 
 74.44 g 
molality = = 0.71 mol/kg = 0.71 m
0.095 kg
1 mol NaOCl
5.0 g NaOCl x = 0.0672 mol NaOCl
74.44 g NaOCl
1 mol H 2 O
95 g H2O x = 5.27 mol H2O
18.02 g H2 O
0.0672 mol
X NaOCl = = 0.013
5.27 mol + 0.0672 mol

16.0 g H 2 SO4
11.60 16.0 mass % =
16.0 g H 2 SO4 + 84.0 g H 2 O
H2SO4, 98.08 amu; density = 1.1094 g/mL
1 mL
volume of solution = 100.0 g x = 90.14 mL = 0.090 14 L
1.1094 g
 1 mol 
16.0 g x 
 98.08 g 
molarity = = 1.81 M
0.090 14 L

11.61 C2H6O2, 62.07 amu


A 40.0 mass % aqueous solution of C2H6O2 contains 40.0 g C2H6O2 and 60.0 g H2O.
density = 1.0514 g/mL
1 mL
volume of solution = 100.0 g x = 95.1 mL = 0.0951 L
1.0514 g
 1 mol 
 40.0 g x 
 62.07 g 
molarity = = 6.78 M
0.0951 L

 1 mol 
 40.0 g x 
 62.07 g 
11.62 molality = = 10.7 mol/kg = 10.7 m
0.0600 kg

266
Chapter 11 - Solutions and Their Properties
______________________________________________________________________________

 1 mol 
16.0 g x 
 98.08 g 
11.63 molality = = 1.94 mol/kg = 1.94 m
0.0840 kg

11.64 C19H21NO3, 311.34 amu; 1.5 mg = 1.5 x 10-3 g


 1 mol 
1.5 x 10_ 3 g x 
-3  311.34 g 
1.3 x 10 mol/kg = ; solve for kg of solvent.
kg of solvent
 1 mol 
1.5 x 10_ 3 g x 
 311.34 g 
kg of solvent = = 0.0037 kg
1.3 x 10_ 3 mol/kg
Because the solution is very dilute, kg of solvent ≈ kg of solution.
 1000 g 
g of solution = (0.0037 kg)   = 3.7 g
 1 kg 

11.65 C12H22O11, 342.30 amu


1 mol C12 H22 O11
32.5 g C12H22O11 x = 0.0949 mol C12H22O11
342.30 g C12 H22 O11
0.0949 mol 0.0949 mol
0.850 m = 0.850 mol/kg = ; kg of H2O = = 0.112 kg
kg of H2 O 0.850 mol/kg
1000 g
mass of H2O = 0.112 kg x = 112 g H2O
1 kg

11.66 C6H12O6, 180.16 amu; H2O, 18.02 amu; Assume 1.00 L of solution.
mass of solution = (1000 mL)(1.0624 g/mL) = 1062.4 g
180.16 g
mass of solute = 0.944 mol x = 170.1 g C6H12O6
1 mol
mass of H2O = 1062.4 g - 170.1 g = 892.3 g H2O
1 mol
mol C6H12O6 = 0.944 mol; mol H2O = 892.3 g x = 49.5 mol
18.02 g
mol C6 H12 O6 0.944 mol
(a) XC6H12O6 = = = 0.0187
mol C6 H12 O6 + mol H2 O 0.944 mol + 49.5 mol
mass C6 H12 O6 170.1 g
(b) mass % = x 100% = x 100% = 16.0%
total mass of solution 1062.4 g
mol C6 H12 O6 0.944 mol
(c) molality = = = 1.06 mol/kg = 1.06 m
kg H2 O 0.8923 kg

11.67 C12H22O11, 342.30 amu; Assume 1.00 L of solution.


mass of solution = (1000 mL)(1.0432 g/mL) = 1043.2 g

267
Chapter 11 - Solutions and Their Properties
______________________________________________________________________________

342.30 g C12 H 22 O11


mass of solute = 0.335 mol C12H22O11 x = 114.7 g C12H22O11
1 mol C12 H 22 O11
mass of H2O = 1043.2 g - 114.7 g = 928.5 g H2O
1 mol H 2 O
mol C12H22O11 = 0.335 mol; 928.5 g H2O x = 51.53 mol H2O
18.02 g H 2 O
0.335 mol
XC12 H22O11 = = 0.006 46
51.53 mol + 0.335 mol
114.7 g
mass % C12H22O11 = x 100% = 11.0 mass % C12H22O11
1043.2 g
0.335 mol
molality = = 0.361 mol/kg = 0.361 m
0.9285 kg

Solubility and Henry's Law

mol
11.68 M = k ⋅ P = (0.091 )(0.75 atm) = 0.068 M
L • atm

M 0.195 M
11.69 M = k ⋅ P; k = = = 0.195 mol/(L⋅ atm)
P 1.00 atm
1.00 atm
P = 25.5 mm Hg x = 0.0336 atm
760 mm Hg
mol
M = k ⋅ P = (0.195 )(0.0336 atm) = 6.55 x 10-3 M
L • atm

11.70 M=k⋅P
M 2.21 x 10_ 3 mol/L mol
Calculate k: k = = = 2.21 x 10-3
P 1.00 atm L • atm
Convert 4 mg/L to mol/L:
4 mg = 4 x 10-3 g
 1 mol 
 4 x 10_ 3 g x 
 32.00 g 
O2 molarity = = 1.25 x 10-4 M
1.00 L
mol
1.25 x 10_ 4
M L
P O2 = = = 0.06 atm
k 2.21 x _ 3 mol
10
L • atm

11.71 k = 1.93 x 10-3 mol/(L ⋅ atm)


 1.00 atm 
M = k ⋅ P = [1.93 x 10-3 mol/(L ⋅ atm)]  68 mm Hg x  = 1.73 x 10-4 mol/L
 760 mm Hg 

268
Chapter 11 - Solutions and Their Properties
______________________________________________________________________________

32.00 g O 2 1 mg
1.73 x 10-4 mol/L x x = 5.5 mg/L
1 mol O2 1 x 10_ 3 g

11.72 [Xe] = 10 mmol/L = 0.010 M at STP


M 0.010 M
M = k ⋅ P; k = = = 0.010 mol/(L⋅ atm)
P 1.0 atm

11.73 Assuming H2O as the solvent, NH3 does not obey Henry's law because NH3 can both
hydrogen bond and react with H2O.

Colligative Properties

11.74 The difference in entropy between the solvent in a solution and a pure solvent is
responsible for colligative properties.

11.75 Osmotic pressure is the amount of pressure that needs to be applied to cause
osmosis to stop.

11.76 NaCl is a nonvolatile solute. Methyl alcohol is a volatile solute. When NaCl is added to
water, the vapor pressure of the solution is decreased, which means that the boiling point
of the solution will increase. When methyl alcohol is added to water, the vapor pressure
of the solution is increased which means that the boiling point of the solution will
decrease.

11.77 When 100 mL of 9 M H2SO4 at 0oC is added to 100 mL of liquid water at 0oC, the
temperature rises because ∆Hsoln for H2SO4 is exothermic.
When 100 mL of 9 M H2SO4 at 0oC is added to 100 g of solid ice at 0oC, some of the ice
will melt (an endothermic process) and the temperature will fall because the H2SO4
(solute) lowers the freezing point of the ice/water mixture.

11.78

11.79 Molality is a temperature independent concentration unit. For freezing point depression
and boiling point elevation, molality is used so that the solute concentration is
independent of temperature changes. Molarity is temperature dependent. Molarity can
be used for osmotic pressure because osmotic pressure is measured at a fixed

269
Chapter 11 - Solutions and Their Properties
______________________________________________________________________________

temperature.

11.80 (a) CH4N2O, 60.06 amu; H2O, 18.02 amu


1 mol CH 4 N2 O
10.0 g CH4N2O x = 0.167 mol CH4N2O
60.06 g CH 4 N 2 O
1 mol H 2 O
150.0 g H2O x = 8.32 mol H2O
18.02 g H2 O
8.32 mol
X H2 O = = 0.980
8.32 mol + 0.167 mol
Psoln = PoH2 O • XH2 O = (71.93 mm Hg)(0.980) = 70.5 mm Hg
1 mol LiCl
(b) LiCl, 42.39 amu; 10.0 g LiCl x = 0.236 mol LiCl
42.39 g LiCl
LiCl dissociates into Li+(aq) and Cl-(aq) in H2O.
mol Li+ = mol Cl- = mol LiCl = 0.236 mol
1 mol H 2 O
150.0 g H2O x = 8.32 mol H2O
18.02 g H 2 O
8.32 mol
X H2 O = = 0.946
8.32 mol + 0.236 mol + 0.236 mol
Psoln = PoH2 O • X H2 O =(71.93 mm Hg)(0.946) = 68.0 mm Hg

11.81 C6H12O6, 180.16 amu; CH3OH, 32.04 amu


1 mol C6 H12 O6
16.0 g C6H12O6 x = 0.0888 mol C6H12O6
180.16 g C6 H12 O6
1 mol CH 3 OH
80.0 g CH3OH x = 2.50 mol CH3OH
32.04 g CH3 OH
2.50 mol
XCH3 OH = = 0.966
2.50 mol + 0.0888 mol
Psoln = PoCH3 OH • X CH3 OH = (140 mm Hg)(0.966) = 135 mm Hg

o
C • kg
11.82 For H2O, Kb = 0.51 ; 150.0 g = 0.1500 kg
mol
 o
C • kg  0.167 mol 
(a) ∆Tb = Kb ⋅ m =  0.51   = 0.57oC
 mol  0.1500 kg 
Solution boiling point = 100.00oC + ∆Tb = 100.00oC + 0.57oC = 100.57oC
 o
C • kg  2(0.236 mol) 
(b) ∆Tb = Kb ⋅ m =  0.51   = 1.6oC
 mol  0.1500 kg 
Solution boiling point = 100.00oC + ∆Tb = 100.00oC + 1.6oC = 101.6oC

o
C • kg
11.83 For H2O, Kf = 1.86 ; 150.0 g = 0.1500 kg
mol

270
Chapter 11 - Solutions and Their Properties
______________________________________________________________________________

 o
C • kg  0.167 mol 
(a) ∆Tf = Kf ⋅ m = 1.86   = 2.07oC
 mol  0.1500 kg 
Solution freezing point = 0.00 C - ∆Tf = 0.00 C - 2.07oC = -2.07oC
o o

 o
C • kg  2(0.236 mol) 
(b) ∆Tf = Kf ⋅ m = 1.86   = 5.85oC
 mol  0.1500 kg 
Solution freezing point = 0.00oC - ∆Tf = 0.00oC - 5.85oC = -5.85oC

11.84 ∆Tf = Kf ⋅ m ⋅ i
Solution freezing point = - 4.3oC = 0.00oC - ∆Tf; ∆Tf = 4.3oC
∆ Tf 4. 3o C
i= = = 2.3
K f • m 1.86 C • kg (1.0 mol/kg)
o
 
 mol 

 o
C • kg 
11.85 ∆Tb = Kb ⋅ m ⋅ i =  0.51 (0.75 mol/kg)(1.85) = 0.71oC
 mol 
Solution boiling point = 100.00 C + ∆Tb = 100.00oC + 0.71oC = 100.71oC
o

11.86 Acetone, C3H6O, 58.08 amu, PoC3H6 O = 285 mm Hg


Ethyl acetate, C4H8O2, 88.11 amu, PoC4 H8O2 = 118 mm Hg
1 mol C3 H6 O
25.0 g C3H6O x = 0.430 mol C3H6O
58.08 g C3 H6 O
1 mol C4 H8 O 2
25.0 g C4H8O2 x = 0.284 mol C4H8O2
88.11 g C4 H8 O 2
0.430 mol 0.284 mol
XC3H6 O = = 0.602; XC4 H8O2 = = 0.398
0.430 mol + 0.284 mol 0.430 mol + 0.284 mol
Psoln = PCo 3H6 O • XC3H6 O + PCo 4H8O2 • XC4 H8O2
Psoln = (285 mm Hg)(0.602) + (118 mm Hg)(0.398) = 219 mm Hg

11.87 CHCl3, 119.38 amu, PoCHCl3 = 205 mm Hg; CH2Cl2, 84.93 amu, PoCH2Cl2 = 415 mm Hg
1 mol CHCl3
15.0 g CHCl3 x = 0.126 mol CHCl3
119.38 g CHCl3
1 mol CH 2 Cl2
37.5 g CH2Cl2 x = 0.442 mol CH2Cl2
84.93 g CH 2 Cl2
0.126 mol 0.442 mol
XCHCl3 = = 0.222; XCH2Cl2 = = 0.778
0.126 mol + 0.442 mol 0.126 mol + 0.442 mol
Psoln = PoCHCl3 • XCHCl3 + PoCH2Cl2 • XCH2Cl2
Psoln = (205 mm Hg)(0.222) + (415 mm Hg)(0.778) = 368 mm Hg

271
Chapter 11 - Solutions and Their Properties
______________________________________________________________________________

11.88 In the liquid, Xacetone = 0.602 and Xethyl acetate = 0.398


In the vapor, PTotal = 219 mm Hg
Pacetone = Poacetone ⋅ Xacetone = (285 mm Hg)(0.602) = 172 mm Hg
acetate ⋅ Xethyl acetate = (118 mm Hg)(0.398) = 47 mm Hg
o
Pethyl acetate = Pethyl
Pacetone = 172 mm Hg = 0.785; Pethyl acetate 47 mm Hg
Xacetone = X ethyl acetate = = = 0.215
P total 219 mm Hg P total 219 mm Hg

11.89 In the liquid, XCHCl3 = 0.222 and X CH2Cl2 = 0.778


In the vapor, Ptotal = 368 mm Hg
PCHCl3 = PCHCl3 • X CHCl3 = (205 mm Hg)(0.222) = 45.5 mm Hg
o

PCH2Cl2 = P CH2Cl2 • X CH2Cl2 = (415 mm Hg)(0.778) = 323 mm Hg


o

PCHCl3 45.5 mm Hg PCH2Cl2 323 mm Hg


XCHCl3 = = = 0.124; XCH2Cl2 = = = 0.876
P total 368 mm Hg P total 368 mm Hg

11.90 C9H8O4, 180.16 amu; 215 g = 0.215 kg


∆ 0. 47o C o
C • kg
∆Tb = Kb ⋅ m = 0.47oC; Kb = T b = = 3.6
m  1 mol  mol
 5.00 g x 
 180.16 g 
 0.215 kg 
 
 

11.91 C6H8O6, 176.13 amu; 50.0 g = 0.0500 kg


∆ 1.33 o C o
C • kg
∆Tf = Kf ⋅ m = 1.33oC; Kf = Tf = = 3.90
m  1 mol  mol
 3.00 g x 
 176.13 g 
 0.0500 kg 
 
 

∆ Tb 1.7 6o C
11.92 ∆Tb = Kb ⋅ m = 1.76oC; m= = = 0.573 m
Kb
o
C • kg
3.07
mol
11.93 C6H12O6, 180.16 amu
C • kg
o
For ethyl alcohol, Kb = 1.22 ; 285 g = 0.285 kg
mol
 1 mol 
 26.0 g x 
 o
C • kg  180.16 g 
∆Tb = Kb ⋅ m = 1.22  = 0.618oC
 mol  0.285 kg 
 
 
Solution boiling point = normal boiling point + ∆Tb = 79.1oC
Normal boiling point = 79.1oC - ∆Tb = 79.1oC - 0.618oC = 78.5oC

272
Chapter 11 - Solutions and Their Properties
______________________________________________________________________________

11.94 Π = MRT
(a) NaCl 58.44 amu; 350.0 mL = 0.3500 L
There are 2 moles of ions/mole of NaCl
 1 mol 
 5.00 g x 
 58.44 g  L • atm 
Π = (2)  0.082 06 (323 K) = 13.0 atm
 0.3500 L  K • mol 
 
 
(b) CH3CO2Na, 82.03 amu; 55.0 mL = 0.0550 L
There are 2 moles of ions/mole of CH3CO2Na
 1 mol 
 6.33 g x 
 82.03 g  L • atm 
Π = (2)  0.082 06 (283 K) = 65.2 atm
 0.0550 L  K • mol 
 
 

 1 mol 
 11.5 x 10 g x 
_3

 5990 g   L • atm 
11.95 Π = MRT =  0.082 06 (298 K) = 0.007 11 atm
 0.006 60 L   K • mol 
 
 
760 mm Hg
Π = 0.007 11 atm x = 5.41 mm Hg
1 atm
13.534 mm H 2 O
height of H2O column = 5.41 mm Hg x = 73.2 mm
1.00 mm Hg
1m
height of H2O column = 73.2 mm x = 0.0732 m
1000 mm

Π 4.85 atm
11.96 Π = MRT; M= = = 0.197 M
RT  L • atm 
 0.082 06 (300 K)
 K • mol 

Π 7.7 atm
11.97 Π = MRT; M= = = 0.30 M
RT  L • atm 
 0.082 06 (310 K)
 K • mol 

Uses of Colligative Properties

11.98 Osmotic pressure is most often used for the determination of molecular mass because, of
the four colligative properties, osmotic pressure gives the largest colligative property
change per mole of solute.

11.99 C6H12O6 does not dissociate in aqueous solution. LiCl and NaCl both dissociate into two
solute particles per formula unit in aqueous solution. CaCl2 dissociates into three solute
particles per formula unit in aqueous solution. Assume that you have 1.00 g of each

273
Chapter 11 - Solutions and Their Properties
______________________________________________________________________________

substance. Calculate the number of moles of solute particles in 1.00 g of each substance.

1 mol
C6H12O6, 180.2 amu; moles solute particles = 1.00 g x = 0.005 55 moles
180.2 g
 1 mol 
LiCl, 42.4 amu; moles solute particles = 2 1.00 g x  = 0.0472 moles
 42.4 g 
 1 mol 
NaCl, 58.4 amu; moles solute particles = 2 1.00 g x  = 0.0342 moles
 58.4 g 
 1 mol 
CaCl2, 111.0 amu; moles solute particles = 3 1.00 g x  = 0.0270 moles
 111.0 g 

LiCl produces more solute particles/gram than any of the other three substances. LiCl
would be the most efficient per unit mass.

1 atm
11.100 Π = 407.2 mm Hg x = 0.5358 atm
760 mm Hg
Π 0.5358 atm
Π = MRT; M = = = 0.021 90 M
RT  L • atm 
 0.082 06 (298.15 K)
 K • mol 
1L
200.0 mL x = 0.2000 L
1000 mL
mol cellobiose = (0.2000 L)(0.021 90 mol/L) = 4.380 x 10-3 mol

1.500 g cellobiose
molar mass of cellobiose = = 342.5 g/mol
4.380 x 10_ 3 mol cellobiose
molecular mass = 342.5 amu

1.00 cm Hg
11.101 height of Hg column = 32.9 cm H2O x = 2.43 cm Hg
13.534 cm H 2 O
1.00 atm
Π = 2.43 cm Hg x = 0.0320 atm
76.0 cm Hg
Π 0.0320 atm
Π = MRT; M = = = 0.001 31 M
RT  L • atm 
 0.082 06 (298 K)
 K • mol 
1L
20.0 mL x = 0.0200 L
1000 mL
1.00 g
15.0 mg x = 0.0150 g
1000 mg
mol met-enkephalin = (0.0200 L)(0.001 31 mol/L) = 2.62 x 10-5 mol

274
Chapter 11 - Solutions and Their Properties
______________________________________________________________________________

0.0150 g met- enkephalin


molar mass of met-enkephalin = = 573 g/mol
2.62 x 10_ 5 mol met- enkephalin
molecular mass = 573 amu
11.102 HCl is a strong electrolyte in H2O and completely dissociates into two solute particles
per each HCl.
HF is a weak electrolyte in H2O. Only a few percent of the HF molecules dissociates into
ions.

 1 mol 
 71 g x 
 142.0 g 
11.103 Na2SO4, 142.0 amu; m = = 0.50 mol/kg = 0.50 m
1.00 kg
 o
C • kg 
∆Tb = Kb ⋅ m =  0.51  (0.50 m) = 0.26oC
 mol 
The experimental ∆T is approximately 3 times that predicted by the equation above
because Na2SO4 dissociates into three solute particles (2 Na+ and SO42-) in aqueous
solution.

11.104 First, determine the empirical formula:


Assume 100.0 g of β-carotene.
1 mol H
10.51% H 10.51 g H x = 10.43 mol H
1.008 g H
1 mol C
89.49% C 89.49 g C x = 7.45 mol C
12.01 g C
C7.45H10.43; Divide each subscript by the smaller, 7.45.
C7.45 / 7.45H10.43 / 7.45
CH1.4
Multiply each subscript by 5 to obtain integers. Empirical formula is C5H7, 67.1 amu.
Second, calculate the molecular mass:
∆ 1.17o C
∆Tf = Kf ⋅ m; m = Tf = = 0.0310 mol/kg = 0.0310 m
Kf
o
C • kg
37.7
mol
1 kg
1.50 g x = 1.50 x 10-3 kg
1000 g
mol β-carotene = (1.50 x 10-3 kg)(0.0310 mol/kg) = 4.65 x 10-5 mol

0.0250 g β - carotene
molar mass of β-carotene = = 538 g/mol
4.65 x 10_ 5 mol β - carotene
molecular mass = 538 amu

Finally, determine the molecular formula:


Divide the molecular mass by the empirical formula mass.

275
Chapter 11 - Solutions and Their Properties
______________________________________________________________________________

538 amu
=8; molecular formula is C(8 x 5)H(8 x 7), or C40H56
67.1 amu

11.105 First, determine the empirical formula:


Assume a 100.0 g sample of lysine.
1 mol C
49.29% C 49.29 g C x = 4.10 mol C
12.011 g C
1 mol H
9.65% H 9.65 g H x = 9.57 mol H
1.008 g H
1 mol N
19.16% N 19.16 g N x = 1.37 mol N
14.007 g N
1 mol O
21.89% O 21.89 g O x = 1.37 mol O
15.999 g O
C4.10H9.57N1.37O1.37; Divide each subscript by the smallest, 1.37.
C4.10 / 1.37H9.57 / 1.37N1.37 / 1.37O1.37 / 1.37
Empirical formula is C3H7NO, 73.09 amu
Second, calculate the molecular mass:
∆ 1.37 o C
∆Tf = Kf ⋅ m = 1.37 oC; m = Tf = = 0.171 mol/kg = 0.171 m
Kf
o
C • kg
8.00
mol
1 kg
1.200 g x = 1.200 x 10-3 kg
1000 g
1.00 g
30.0 mg x = 0.0300 g
1000 mg
mol lysine = (1.200 x 10-3 kg)(0.171 mol/kg) = 2.05 x 10-4 mol
0.0300 g lysine
molar mass of lysine = = 146 g/mol
2.05 x 10_ 4 mol lysine
molecular mass = 146 amu
Finally, determine the molecular formula:
Divide the molecular mass by the empirical formula mass.
146 amu
= 2; molecular formula is C(2 x 3)H(2 x 7)N(2 x 1)O(2 x 1), or C6H14N2O2
73.09 amu

General Problems

C • kg
o
11.106 Kf for snow (H2O) is 1.86 . Reasonable amounts of salt are capable of
mol
lowering the freezing point (∆Tf) of the snow below an air temperature of -2oC.
Reasonable amounts of salt, however, are not capable of causing a ∆Tf of more than
30oC which would be required if it is to melt snow when the air temperature is -30oC.

276
Chapter 11 - Solutions and Their Properties
______________________________________________________________________________

11.107 KBr, 119.00 amu; for KBr, i = 2


1 kg
125 g x = 0.125 kg
1000 g
∆Tb = 103.2 oC - 100.0 oC = 3.2 oC
∆ Tb 3.2 o C
∆Tb = Kb ⋅ m ⋅ i; m= = = 3.137 mol/kg = 3.137 m
K b • 2  0.51 C • kg (2)
o
 
 mol 
mol KBr = (0.125 kg)(3.137 mol/kg) = 0.392 mol KBr

119.00 g KBr
mass of KBr = 0.392 mol KBr x = 47 g KBr
1 mol KBr

11.108 C2H6O2, 62.07 amu; ∆Tf = 22.0oC


∆ 22. 0o C
∆Tf = Kf ⋅ m; m = Tf = = 11.8 mol/kg = 11.8 m
K f 1.86 C • kg
o

mol
mol C2H6O2 = (3.55 kg)(11.8 mol/kg) = 41.9 mol C2H6O2
62.07 g C2 H6 O2
mass C2H6O2 = 41.9 mol C2H6O2 x = 2.60 x 103 g C2H6O2
1 mol C2 H6 O2

11.109 The vapor pressure of toluene is lower than the vapor pressure of benzene at the same
temperature. When 1 mL of toluene is added to 100 mL of benzene, the vapor pressure
of the solution decreases, which means that the boiling point of the solution will
increase. When 1 mL of benzene is added to 100 mL of toluene, the vapor pressure of
the solution increases, which means that the boiling point of the solution will decrease.

11.110 When solid CaCl2 is added to liquid water, the temperature rises because ∆Hsoln for
CaCl2 is exothermic.
When solid CaCl2 is added to ice at 0oC, some of the ice will melt (an endothermic
process) and the temperature will fall because the CaCl2 lowers the freezing point of an
ice/water mixture.

11.111 AgCl, 143.32 amu; there are 2 ions/AgCl


Π = 2MRT
 1 mol 
 0.007 x 10_ 3 g x 
 143.32 g   L • atm 
Π=2  0.082 06 (278 K) = 0.002 atm
 0.001 L  K • mol 
 
 

11.112 C10H8, 128.17 amu; ∆Tf = 0.35oC

277
Chapter 11 - Solutions and Their Properties
______________________________________________________________________________

∆ Tf 0. 35o C
∆Tf = Kf ⋅ m;m = = = 0.0684 mol/kg = 0.0684 m
Kf
o
C • kg
5.12
mol
1 kg
150.0 g x = 0.1500 kg
1000 g
mol C10H8 = (0.1500 kg)(0.0684 mol/kg) = 0.0103 mol C10H8
128.17 g C10 H8
mass C10H8 = 0.0103 mol C10H8 x = 1.3 g C10H8
1 mol C10 H8

11.113 Br2, 159.81 amu; CCl4, 153.82 amu


o 760 mm Hg
P Br2 = 30.5 kPa x = 228.8 mm Hg
101.325 kPa
o 760 mm Hg
PCCl4 = 16.5 kPa x = 123.8 mm Hg
101.325 kPa
1 mol Br 2
1.50 g Br2 x = 9.39 x 10-3 mol Br2
159.81 g Br 2
1 mol CCl4
145.0 g CCl4 x = 0.943 mol CCl4
153.82 g CCl4
9.39 x 10_ 3 mol
X Br2 = = 0.009 86
(0.943 mol) + (9.39 x 10_ 3 mol)
0.943 mol
XCCl4 = = 0.990
(0.943 mol) + (9.39 x 10_ 3 mol)
Psoln = PoBr2 • X Br2 + PoCCl4 • X CCl4
Psoln = (228.8 mm Hg)(0.009 86) + (123.8 mm Hg)(0.990) = 125 mm Hg

11.114 NaCl, 58.44 amu; there are 2 ions/NaCl


A 3.5 mass % aqueous solution of NaCl contains 3.5 g NaCl and 96.5 g H2O.
 1 mol 
 3.5 g x 
 58.44 g 
molality = = 0.62 mol/kg = 0.62 m
0.0965 kg
 o
C • kg 
∆Tf = Kf ⋅ 2 ⋅ m = 1.86 (2)(0.62 mol/kg) = 2.3oC
 mol 
Solution freezing point = 0.0 C - ∆Tf = 0.0oC - 2.3oC = -2.3oC
o

 o
C • kg 
∆Tb = Kb ⋅ 2 ⋅ m =  0.51 (2)(0.62 mol/kg) = 0.63oC
 mol 
Solution boiling point = 100.00oC + ∆Tb = 100.00oC + 0.63oC = 100.63oC

11.115 (a) Assume a total mass of solution of 1000.0 g.

278
Chapter 11 - Solutions and Their Properties
______________________________________________________________________________

mass of solute ion


ppm = x 106
total mass of solution
(ppm)(1000.0 g)
For each ion: mass of solute ion = 6
10
Ion Mass Moles
Cl- 19.0 g 0.536 mol
Na+ 10.5 g 0.457 mol
SO42- 2.65 g 0.0276 mol
Mg2+ 1.35 g 0.0555 mol
Ca2+ 0.400 g 0.009 98 mol
K+ 0.380 g 0.009 72 mol
HCO3- 0.140 g 0.002 29 mol
Br- 0.065 g 0.000 81 mol
Total 34.5 g 1.099 mol

Mass of H2O = 1000.0 g - 34.5 g = 965.5 g H2O = 0.9655 kg H2O


1.099 mol
molality = = 1.138 mol/kg = 1.138 m
0.9655 kg
(b) Assume M = m for a dilute solution.
 L • atm 
Π = MRT = (1.138 mol/L)  0.082 06  (300 K) = 28.0 atm
 K • mol 

10.5 g
11.116 (a) 90 mass % isopropyl alcohol = x 100%
10.5 g + mass of H 2 O
Solve for the mass of H2O.
 100 
mass of H2O = 10.5 g x  _ 10.5 g = 1.2 g
 90 
mass of solution = 10.5 g + 1.2 g = 11.7 g
11.7 g of rubbing alcohol contains 10.5 g of isopropyl alcohol.
(b) C3H8O, 60.10 amu
mass C3H8O = (0.90)(50.0 g) = 45 g
1 mol C3 H8 O
45 g C3H8O x = 0.75 mol C3H8O
60.10 g C3 H8 O

11.117 C6H12O6, 180.16 amu; 50.0 mL = 0.0500 L; 17.5 mg = 17.5 x 10-3 g


Π = MRT
 1 atm 
 37.8 mm Hg x 
Π  760 mm Hg 
T= = = 312 K
MR   1 mol  
 17.5 x 10_ 3 g x 
 180.16 g   L • atm 
  0.082 06 
0.0500 L  K • mol 
 
 
 

279
Chapter 11 - Solutions and Their Properties
______________________________________________________________________________

11.118 First, determine the empirical formula.


3.47 mg = 3.47 x 10-3 g sample
10.10 mg = 10.10 x 10-3 g CO2
2.76 mg = 2.76 x 10-3 g H2O
12.01 g C
mass C = 10.10 x 10-3 g CO2 x = 2.76 x 10-3 g C
44.01 g CO2
2 x 1.008 g H
mass H = 2.76 x 10-3 g H2O x = 3.09 x 10-4 g H
18.02 g H 2 O
mass O = 3.47 x 10-3 g - 2.76 x 10-3 g C - 3.09 x 10-4 g H = 4.01 x 10-4 g O
1 mol C
2.76 x 10-3 g C x = 2.30 x 10-4 mol C
12.01 g C
1 mol H
3.09 x 10-4 g H x = 3.07 x 10-4 mol H
1.008 g H
1 mol O
4.01 x 10-4 g O x = 2.51 x 10-5 mol O = 0.251 x 10-4 mol O
16.00 g O
To simplify the empirical formula, divide each mol quantity by 10-4.
C2.30H3.07O0.251; Divide all subscripts by the smallest, 0.251.
C2.30 / 0.251H3.07 / 0.251O0.251 / 0.251
C9.16H12.23O, empirical formula is C9H12O (136 amu)

Second, determine the molecular mass.


1 kg
7.55 mg = 7.55 x 10-3 g estradiol; 0.500 g x = 5.00 x 10-4 kg camphor
1000 g
∆ Tf 2.10o C
∆Tf = Kf ⋅ m; m = = = 0.0557 mol/kg = 0.0557 m
Kf
o
C • kg
37.7
mol
mol estradiol
m=
kg solvent
mol estradiol = m x (kg solvent) = (0.0557 mol/kg)(5.00 x 10-4 kg) = 2.79 x 10-5 mol
7.55 x 10_ 3 g estradiol
molar mass = = 271 g/mol; molecular mass = 271 amu
2.79 x 10_ 5 mol estradiol

Finally, determine the molecular formula:


Divide the molecular mass by the empirical formula mass.
271 amu
= 2; molecular formula is C(2 x 9)H(2 x 12)O(2 x 1), or C18H24O2
136 amu

11.119 CCl3CO2H(aq) _ H+(aq) + CCl3CO2-(aq)


1.00 - x x x

280
Chapter 11 - Solutions and Their Properties
______________________________________________________________________________

∆ Tf 2. 53o C
∆Tf = Kf ⋅ m; m = = = 1.36 m
K f 1.86
o
C • kg
mol
1.36 = 1.00 - x + x + x = 1 + x; x = 0.36
36% of the acid molecules are dissociated.

98.08 g H 2 SO 4
11.120 (a) H2SO4, 98.08 amu; 2.238 mol H2SO4 x = 219.50 g H2SO4
1 mol H 2 SO4
mass of 2.238 m solution = 219.50 g H2SO4 + 1000 g H2O = 1219.50 g
1.0000 mL
volume of 2.238 m solution = 1219.50 g x = 1084.68 mL = 1.0847 L
1.1243 g
2.238 mol
molarity of 2.238 m solution = = 2.063 M
1.0847 L
The molarity of the H2SO4 solution is less than the molarity of the BaCl2 solution.
Because equal volumes of the two solutions are mixed, H2SO4 is the limiting reactant
and the number of moles of H2SO4 determines the number of moles of BaSO4 produced
as the white precipitate.
1 mol BaSO4 233.39 g BaSO4
(0.05000 L) x (2.063 mol H2SO4/L) x x = 24.07 g BaSO4
1 mol H2 SO4 1 mol BaSO4
(b) More precipitate will form because of the excess BaCl2 in the solution.

11.121 KCl, 74.55amu; KNO3, 101.10 amu; Ba(NO3)2, 261.34 amu


 1.00 atm 
 744.7 mm Hg x 
Π  760 mm Hg 
Π = MRT; M = = = 0.040 07 M
RT  L • atm 
 0.082 06 (298 K)
 K • mol 
mol ions
0.040 07 M = ; mol ions = (0.040 07 mol/L)(0.500 L) = 0.020 035 mol ions
0.500 L
mass Cl = 1.000 g x 0.2092 = 0.2092 g Cl
1 mol Cl 1 mol KCl 74.55 g KCl
mass KCl = 0.2092 g Cl x x x = 0.440 g
35.453 g Cl 1 mol Cl 1 mol KCl
KCl
1 mol KCl 2 mol ions
mol ions from KCl = 0.440 g KCl x x = 0.0118 mol ions
74.55 g KCl 1 mol KCl
mol ions from KNO3 and Ba(NO3)2 = 0.020 035 - 0.0118 = 0.008 235 mol ions
Let x = mass KNO3 and y = mass Ba(NO3)2
x + y = 1.000 g - 0.440 g = 0.560 g
 x   y 
 2 +  3 = 0.008 235 mol ions
 101.10   261.34 
x = 0.560 - y
0.0198x + 0.0115y = 0.008 235

281
Chapter 11 - Solutions and Their Properties
______________________________________________________________________________

0.0198(0.560 - y) + 0.0115y = 0.008 235


0.011 09 - 0.0198y + 0.0115y = 0.008 235
0.002 855
0.002 855 = 0.0083y; y = = 0.3440; x = 0.560 - 0.3440 = 0.216
0.0083
0.440 g
mass % KCl = x 100% = 44.0%
1.000 g
0.216 g
mass % KNO3 = x 100% = 21.6%
1.000 g
0.344 g
mass % Ba(NO3)2 = x 100% = 34.4%
1.000 g

11.122 Let x = X H2 O and y = XCH3 OH and assume ntotal = 1.00 mol


(14.5 mm Hg)x + (82.5 mm Hg)y = 39.4 mm Hg
(26.8 mm Hg)x + (140.3 mm Hg)y = 68.2 mm Hg
68.2 _ 140.3 y
x=
26.8
14.5(68.2 _ 140.3 y)
+ 82.5y = 39.4
26.8
(988.9 _ 2034.35 y)
+ 82.5y = 39.4
26.8
2.5
36.90 - 75.91y + 82.5y = 39.4; 6.59 = 2.5; y = = 0.3794
6.59
[68.2 _ 140.3(0.3794)]
x= = 0.5586
26.8
XLiCl = 1 - XH2 O - XCH3 OH = 1 - 0.5586 - 0.3794 = 0.0620
The mole fraction equals the number of moles of each component because
ntotal = 1.00 mol.
42.39 g LiCl
mass LiCl = 0.0620 mol LiCl x = 2.6 g LiCl
1 mol LiCl
18.02 g H2 O
mass H2O = 0.5588 mol H2O x = 10.1 g H2O
1 mol H 2 O
32.04 g CH3 OH
mass CH3OH = 0.3794 mol CH3OH x = 12.2 g CH3OH
1 mol CH3 OH
total mass = 2.6 g + 10.1 g + 12.2 g = 24.9 g

2.6 g
mass % LiCl = x 100% = 10%
24.9 g

10.1 g
mass % H2O = x 100% = 41%
24.9 g

282
Chapter 11 - Solutions and Their Properties
______________________________________________________________________________

12.2 g
mass % CH3OH = x 100% = 49%
24.9 g

11.123 KI, 166.00 amu


∆ Tf 1. 95o C
∆Tf = Kf ⋅ m ⋅ i; m = = = 0.524 mol/kg = 0.524 m
K f • i 1.86 C • kg (2)
o
 
 mol 
Π 25.0 atm
Π = i ⋅ MRT; M = = = 0.511 M = 0.511
i • RT  L • atm 
(2) 0.082 06 (298 K)
 K • mol 
mol/L
1.000 L of solution contains 0.511 mol KI and is 0.524 m.
166.00 g KI
mass KI = 0.511 mol KI x = 84.83 g KI
1 mol KI
Calculate the mass of solvent in this solution.
0.511 mol
0.524 m = 0.524 mol/kg =
mass of solvent
0.511 mol
mass of solvent = = 0.9752 kg = 975.2 g
0.524 mol/kg
mass of solution = mass KI + mass of solvent = 84.83 g + 975.2 g = 1060 g
1060 g
density = = 1.06 g/mL
1000 mL

11.124 Solution freezing point = -1.03oC = 0.00oC - ∆Tf; ∆Tf = 1.03oC


∆ 1. 03o C
∆Tf = Kf ⋅ m; m = Tf = = 0.554 mol/kg = 0.554 m
K f 1.86 C • kg
o

mol
Π (12.16 atm) mol
Π = MRT; M = = = 0.497
RT  L • atm  L
 0.082 06 (298 K)
 K • mol 
Assume 1.000 L = 1000 mL of solution.
mass of solution = (1000 mL)(1.063 g/mL) = 1063 g
1000 g H 2 O
mass of H2O in 1000 mL of solution = x 0.497 mol = 897 g H2O
0.554 mol of solute
mass of solute = total mass - mass of H2O = 1063 g - 897 g = 166 g solute
166 g
molar mass = = 334 g/mol
0.497 mol

11.125 C6H6, 78.11 amu

283
Chapter 11 - Solutions and Their Properties
______________________________________________________________________________

299 mm Hg = PoC6 H6 ⋅ XC6H6 + PoX ⋅ XX


299 mm Hg = (395 mm Hg) ⋅ XC6 H6 + (96 mm Hg) ⋅ XX
XC6 H6 + XX = 1; XX = 1 - XC6H6
299 mm Hg = (395 mm Hg) ⋅ XC6 H6 + (96 mm Hg)(1 - XC6H6 )
299 mm Hg = (395 mm Hg) ⋅ XC6 H6 + 96 mm Hg - (96 mm Hg) ⋅ XC6H6
299 mm Hg - 96 mm Hg = (395 mm Hg) ⋅ XC6H6 - (96 mm Hg) ⋅ XC6H6
203 mm Hg = (299 mm Hg) ⋅ XC6 H6
XC6 H6 = 203 mm Hg/299 mm Hg = 0.679
Assume the mixture contains 1.00 mol (78.11 g) of C6H6. Then a 50/50 mixture will
also contain 78.11 g of X.
1 mol C6 H6
XC6 H6 = = 0.679
1 mol C6 H6 + mol X
1 mol C6H6 = (0.679)(1 mol C6H6 + mol X)
1 mol C6 H6
= 1 mol C6H6 + mol X
0.679
1 mol C6 H6
- 1 mol C6H6 = mol X
0.679
mol X = 0.473 mol
molar mass X = 78.11 g/0.473 mol = 165 g/mol

11.126 (a) NaCl, 58.44 amu; CaCl2, 110.98 amu; H2O, 18.02 amu
1 mol NaCl
mol NaCl = 100.0 g NaCl x = 1.711 mol NaCl
58.44 g NaCl
1 mol CaCl2
mol CaCl2 = 100.0 g CaCl2 x = 0.9011 mol CaCl2
110.98 g CaCl2
mass of solution = (1000 mL)(1.15 g/mL) = 1150 g
mass of H2O in solution = mass of solution - mass NaCl - mass CaCl2
= 1150 g - 100.0 g - 100.0 g = 950 g
1 kg
= 950 g x = 0.950 kg
1000 g
∆Tb = Kb ⋅ ( m NaCl • i + mCaCl2 • i )
 o
C • kg  (1.711 mol NaCl • 2) + (0.9011 mol CaCl2 • 3) 
∆Tb =  0.51   = 3.3oC
 mol  0.950 kg 
o o o o
solution boiling point = 100.0 C + ∆Tb = 100.0 C + 3.3 C = 103.3 C

1 mol H 2 O
(b) mol H2O = 950 g H2O x = 52.7 mol H2O
18.02 g H 2 O
PSolution = Po • X H2 O

PSolution = Po

284
Chapter 11 - Solutions and Their Properties
______________________________________________________________________________

 52.7 mol H 2 O 
•  
 (52.7 mol H 2 O) + (1.711 mol NaCl • 2) + (0.9011 mol CaCl2 • 3) 
PSolution = (23.8 mm Hg)(0.896) = 21.3 mm Hg

11.127 HIO3, 175.91 amu


mass of 1.00 L solution = (1.00 x 103 mL)(1.07 g/mL) = 1.07 x 103 g
1.00 L of solution contains 1 mol (175.91 g) HIO3.
mass of H2O = 1070 g - 175.91 g = 894 g = 0.894 kg
m = 1.00 mol/0.894 kg = 1.12 m
∆Tf = Kf ⋅ m ⋅ i
∆ Tf 2. 78o C
i= = = 1.33; The acid is 33% dissociated.
K f • m 1.86 C • kg (1.12 mol/kg)
o
 
 mol 

11.128 (a) KI, 166.00 amu


Assume you have 1.000 L of 1.24 M solution.
mass of solution = (1000 mL)(1.15 g/mL) = 1150 g
166.00 g KI
mass of KI in solution = 1.24 mol KI x = 206 g KI
1 mol KI
mass of H2O in solution = mass of solution - mass KI = 1150 g - 206 g = 944 g
1 kg
= 944 g x = 0.944 kg
1000 g
1.24 mol KI
molality = = 1.31 m
0.944 kg H 2 O
(b) For KI, i = 2 assuming complete dissociation.
 o
C • kg 
∆Tf = Kf ⋅ m ⋅ i = 1.86 (1.31 m)(2) = 4.87oC
 mol 
Solution freezing point = 0.00oC - ∆Tf = 0.00oC - 4.87oC = - 4.87oC
∆ Tf 4. 46o C
(c) i = = = 1.83
K f • m 1.86 C • kg (1.31 mol/kg)
o
 
 mol 
Because the calculated i is only 1.83 and not 2, the percent dissociation for KI is 83%.

11.129 (a) For NaCl, i = 2 and for MgCl2, i = 3; T = 25oC = 25 + 273 = 298 K
 L • atm 
Π = i ⋅ MRT = [(2)(0.470 mol/L) + (3)(0.068 mol/L)]  0.082 06 (298 K) = 28.0 atm
 K • mol 

(b) Calculate the molarity for an osmotic pressure = 100.0 atm.

285
Chapter 11 - Solutions and Their Properties
______________________________________________________________________________

Π (100.0 atm)
Π = MRT; M = = = 4.09 mol/L
RT  L • atm 
 0.082 06 (298 K)
 K • mol 
Mconc x Vconc = Mdil x Vdil
x [(2)(0.470 mol/L) + (3)(0.068 mol/L)](1.00 L)
Vconc = Mdil Vdil = = 0.28 L
M conc 4.09 mol/L
A volume of 1.00 L of seawater can reduced to 0.28 L by an osmotic pressure of 100.0
atm. The volume of fresh water that can be obtained is (1.00 L - 0.28 L) = 0.72 L.

11.130 NaCl, 58.44 amu; C12H22O11, 342.3 amu


Let X = mass NaCl and Y = mass C12H22O11, then X + Y = 100.0 g.
1 kg
500.0 g x = 0.5000 kg
1000 g
Solution freezing point = -2.25oC = 0.00oC - ∆Tf; = ∆Tf = 0.00oC + 2.25oC = 2.25oC
∆Tf = Kf ⋅ ( m NaCl • i + mC12H22O11 )
 o
C • kg  (mol NaCl • 2) + (mol C12 H 22 O11) 
∆Tb = 1.86   = 2.25oC
 mol  0.5000 kg 
1 mol NaCl
mol NaCl = X g NaCl x = X/58.44 mol
58.44 g NaCl
1 mol C12 H22 O11
mol C12H22O11 = Y g C12H22O11 x = Y/342.3 mol
342.3 g C12 H22 O11

 o
C • kg  ((X/ 58.44) • 2 mol) + ((Y/ 342.3) mol) 
∆Tb = 1.86   = 2.25oC
 mol  0.5000 kg 
X = 100 - Y

 o
C • kg  {[(100 _ Y) / 58.44] • 2 mol]} + [(Y/ 342.3) mol] 
1.86   = 2.25oC
 mol  0.5000 kg 

 [(200 / 58.44) _ (2 Y/ 58.44) + (Y/ 342.3)] mol  2.2 5o C


  = = 1.21 mol/kg
 0.5000 kg   o
C • kg 
1.86 
 mol 
 [(3.42) _ (0.0313 Y)] mol 
  = 1.21 mol/kg
 0.5000 kg 

[(3.42) - (0.0313Y)] = (0.5000 kg)(1.21) = 0.605


- 0.0313 Y = 0.605 - 3.42 = - 2.81
Y = (- 2.81)/(- 0.0313) = 89.9 g of C12H22O11
X = 100.0 g - Y = 100.0 g - 89.9 g = 10.1 g of NaCl

286
Chapter 11 - Solutions and Their Properties
______________________________________________________________________________

Multi-Concept Problems

11.131 (a) 382.6 mL = 0.3826 L; 20.0oC = 293.2 K


PV = nRT
 1.0 atm 
 755 mm Hg x (0.3826 L)
PV  760 mm Hg 
n H2 = = = 0.0158 mol H2
RT  L • atm 
 0.082 06  (293.2 K)
 K • mol 
(b) M + x HCl → x/2 H2 + MClx
x mol HCl
moles HCl reacted = 0.0158 mol H2 x = 0.0316 mol HCl
x/ 2 mol H 2
moles Cl reacted = moles HCl reacted = 0.0316 mol Cl
35.453 g Cl
mass Cl = 0.0316 mol Cl x = 1.120 g Cl
1 mol Cl
mass MClx = mass M + mass Cl = 1.385 g + 1.120 g = 2.505 g MClx
∆ 3. 53o C
(c) ∆Tf = Kf ⋅ m; m = Tf = = 1.90 mol/kg = 1.90 m
K f 1.86 C • kg
o

mol
(d) 25.0 g = 0.0250 kg
mol x mol ions
1.90 m = 1.90 =
kg 0.0250 kg
mol ions = (1.90 mol/kg)(0.0250 kg) = 0.0475 mol ions
(e) mol M = mol ions - mol Cl = 0.0475 mol - 0.0316 mol = 0.0159 mol M
Cl 0.0316 mol
= = 2, the formula is MCl2.
M 0.0159 mol
2.505 g
molar mass = = 157.5 g/mol; molecular mass = 157.5 amu
0.0159 mol
(f) atomic mass of M = 157.5 amu - 2(35.453 amu) = 86.6 amu; M = Sr

11.132 (a) 20.00 mL = 0.02000 L


mol NaOH = (0.02000 L)(2.00 mol/L) = 0.0400 mol NaOH
1 mol CO2
mol CO2 = 0.0400 mol NaOH x = 0.0200 mol CO2
2 mol NaOH
1 mol C
mol C = 0.0200 mol CO2 x = 0.0200 mol C
1 mol CO2
12.011 g C
mass C = 0.0200 mol C x = 0.240 g C
1 mol C
mass H = mass of compound - mass of C = 0.270 g - 0.240 g = 0.030 g H

287
Chapter 11 - Solutions and Their Properties
______________________________________________________________________________

1 mol H
mol H = 0.030 g H x = 0.030 mol H
1.008 g H
The mole ratio of C and H in the molecule is C0.0200 H0.030.
C0.0200 H0.030, divide both subscripts by the smaller of the two, 0.0200.
C0.0200 / 0.0200 H0.030 / 0.0200
C1H1.5, multiply both subscripts by 2.
C(2 x 1) H(2 x 1.5)
C2H3 (27.05 amu) is the empirical formula.
∆ (179.8o C _ 177. 9o C)
(b) ∆Tf = Kf ⋅ m; m = Tf = = 0.050 mol/kg = 0.050 m
Kf
o
C • kg
37.7
mol
1 kg
50.0 g x = 0.0500 kg
1000 g
mol solute = (0.050 mol/kg)(0.0500 kg) = 0.0025 mol
0.270 g
molar mass = = 108 g/mol; molecular mass = 108 amu
0.0025 mol

(c) To find the molecular formula, first divide the molecular mass by the mass of the
empirical formula unit.
108
=4
27
Multiply the subscripts in the empirical formula by the result of this division, 4.
C(4 x 2) H(4 x 3)
C8H12 is the molecular formula of the compound.

11.133 CO2, 44.01 amu; H2O, 18.02 amu


1g 1 mol CO2 1 mol C
mol C = 106.43 mg CO2 x x x = 0.002 418 mol
1000 mg 44.01 g CO2 1 mol CO2
C
12.011 g C
mass C = 0.002 418 mol C x = 0.029 04 g C
1 mol C
1g 1 mol H 2 O 2 mol H
mol H = 32.100 mg H2O x x x = 0.003 563 mol
1000 mg 18.02 g H2 O 1 mol H2 O
H
1.008 g H
mass H = 0.003 563 mol H x = 0.003 592 g H
1 mol H
 1g 
mass O =  36.72 mg x  - 0.029 04 g C - 0.003 592 g H = 0.004 088 g O
 1000 mg 
1 mol O
mol O = 0.004 088 g O x = 0.000 255 5 mol O
16.00 g O
C0.002 418H0.003 563O0.000 255 5 Divide all subscripts by the smallest, 0.000 255 5.
C0.002 418 / 0.000 255 5H0.003 563 / 0.000 255 5O0.000 255 5 / 0.000 255 5

288
Chapter 11 - Solutions and Their Properties
______________________________________________________________________________

C9.5H14O Multiply all subscripts by 2.


C(9.5 x 2)H(14 x 2)O(1 x 2)
Empirical formula is C19H28O2, 288 amu

T = 25oC = 25 + 273 = 298 K


 1 atm 
 21.5 mm Hg x 
Π  760 mm Hg 
Π = MRT; M = = = 0.001 16 mol/L
RT  L • atm 
 0.082 06 (298 K)
 K • mol 
15.0 mL = 0.0150 L
mol solute = (0.001 16 mol/L)(0.0150 L) = 1.74 x 10-5 mol
 1g 
 5.00 mg x 
 1000 mg 
molar mass = = 287 g/mol
1.74 x 10_ 5 mol
The molar mass and the empirical formula mass are essentially identical, so the
molecular formula and the empirical formula are the same. The molecular formula is
C19H28O2.

11.134 AgCl, 143.32 amu


Solution freezing point = - 4.42oC = 0.00oC - ∆Tf; ∆Tf = 0.00oC + 4.42oC = 4.42oC
∆Tf = Kf ⋅ m
∆ 4. 42o C
total ion m = Tf = = 2.376 mol/kg = 2.376 m
K f 1.86 C • kg
o

mol
1 kg
150.0 g x = 0.1500 kg
1000 g
total mol of ions = (2.376 mol/kg)(0.1500 kg) = 0.3564 mol of ions
An excess of AgNO3 reacts with all Cl- to produce 27.575 g AgCl.
1 mol AgCl 1 mol Cl _
total mol Cl- = 27.575 g AgCl x x = 0.1924 mol Cl-
143.32 g AgCl 1 mol AgCl

Let P = mol XCl and Q = mol YCl2.


0.3564 mol ions = 2 x mol XCl + 3 x mol YCl2 = (2 x P) + (3 x Q)
0.1924 mol Cl- = mol XCl + 2 x mol YCl2 = P + (2 x Q)
P = 0.1924 - (2 x Q)
0.3564 = 2 x [0.1924 - (2 x Q)] + (3 x Q) = 0.3848 - (4 x Q) + (3 x Q)
Q = 0.3848 - 0.3564 = 0.0284 mol YCl2
P = 0.1924 - (2 x Q) = 0.1924 - (2 x 0.0284) = 0.1356 mol XCl

1 mol Cl 35.453 g Cl
mass Cl in XCl = 0.1356 mol XCl x x = 4.81 g Cl
1 mol XCl 1 mol Cl

289
Chapter 11 - Solutions and Their Properties
______________________________________________________________________________

2 mol Cl 35.453 g Cl
mass Cl in YCl2 = 0.0284 mol YCl2 x x = 2.01 g Cl
1 mol YCl2 1 mol Cl
total mass of XCl and YCl2 = 8.900 g
mass of X + Y = total mass - mass Cl = 8.900 g - 4.81 g - 2.01 g = 2.08 g
X is an alkali metal and there are 0.1356 mol of X in XCl.
If X = Li, then mass of X = (0.1356 mol)(6.941 g/mol) = 0.941 g
If X = Na, then mass of X = (0.1356 mol)(22.99 g/mol) = 3.12 g but this is not possible
because 3.12 g is greater than the total mass of X + Y. Therefore, X is Li.
mass of Y = 2.08 - mass of X = 2.08 g - 0.941 g = 1.14 g
Y is an alkaline earth metal and there are 0.0284 mol of Y in YCl2.
molar mass of Y = 1.14 g/0.0284 mol = 40.1 g/mol. Therefore, Y is Ca.

42.39 g LiCl
mass LiCl = 0.1356 mol LiCl x = 5.75 g LiCl
1 mol LiCl
110.98 g CaCl2
mass CaCl2 = 0.0284 mol CaCl2 x = 3.15 g CaCl2
1 mol CaCl2

290
12 Chemical Kinetics

12.1 3 I-(aq) + H3AsO4(aq) + 2 H+(aq) → I3-(aq) + H3AsO3(aq) + H2O(l)


∆[ _ ]
(a) - I = 4.8 x 10-4 M/s
∆t
∆[I3 ] 1  ∆[I _ ]   1 
_
= _  =  (4.8 x 10_ 4 M/s) = 1.6 x 10-4 M/s
∆t 3 ∆t  3
∆[ ] +
 ∆[ _ ] 
(b) - H = 2 I3  = (2)(1.6 x 10-4 M/s) = 3.2 x 10-4 M/s
∆t  ∆t 

12.2 2 N2O5(g) → 4 NO2(g) + O2(g)


time [N2O5] [O2]
200 s 0.0142 M 0.0029 M
300 s 0.0120 M 0.0040 M
∆[ N 2 O5] 0.0120 M _ 0.0142 M
Rate of decomposition of N2O5 = _ =_ = 2.2 x 10-5
∆t 300 s _ 200 s
M/s
∆[O2] 0.0040 M _ 0.0029 M
Rate of formation of O2 = = = 1.1 x 10-5 M/s
∆t 300 s _ 200 s

12.3 Rate = k[BrO3-][Br-][H+]2


1st order in BrO3-, 1st order in Br-, 2nd order in H+, 4th order overall
Rate = k[H2][I2], 1st order in H2, 1st order in I2, 2nd order overall
Rate = k[CH3CHO]3/2, 3/2 order in CH3CHO, 3/2 order overall

12.4 H2O2(aq) + 3 I-(aq) + 2 H+(aq) → I3-(aq) + 2 H2O(l)


∆[ _ ]
Rate = I3 = k[H2O2]m[I-]n
∆t
2.30 x 10_ 4 M/s [H 2 O2 ]3 0.200 M
(a) Rate3 = _4
=2 = =2
Rate1 1.15 x 10 M/s [H 2 O2 ]1 0.100 M
Because both ratios are the same, m = 1.
_4
Rate2 2.30 x 10 M/s = 2 [I _ ]2 0.200 M
= _4
= =2
Rate1 1.15 x 10 M/s [I_ ]1 0.100 M
Because both ratios are the same, n = 1.
The rate law is: Rate = k[H2O2][I-]

Rate
(b) k =
[H 2 O2][I_ ]

291
Chapter 12 - Chemical Kinetics
_____________________________________________________________________________

1.15 x 10_ 4 M/s


Using data from Experiment 1: k = = 1.15 x 10-2 /(M ⋅ s)
(0.100 M)(0.100 M)
(c) Rate = k[H2O2][I ] = [1.15 x 10 /(M ⋅ s)](0.300 M)(0.400 M) = 1.38 x 10-3 M/s
- -2

12.5 Rate Law Units of k


Rate = k[(CH3)3CBr] 1/s
Rate = k[Br2] 1/s
Rate = k[BrO3-][Br-][H+]2 1/(M3 ⋅ s)
Rate = k[H2][I2] 1/(M ⋅ s)
Rate = [CH3CHO]3/2 1/(M1/2 ⋅ s)

12.6 (a) The reactions in vessels (a) and (b) have the same rate, the same number of B
molecules, but different numbers of A molecules. Therefore, the rate does not depend on
A and its reaction order is zero. The same conclusion can be drawn from the reactions in
vessels (c) and (d).
The rate for the reaction in vessel (c) is four times the rate for the reaction in vessel (a).
Vessel (c) has twice as many B molecules than does vessel (a). Because the rate
quadruples when the concentration of B doubles, the reaction order for B is two.
(b) rate = k[B]2

[Co( NH3 )5 Br 2+ ]t
12.7 (a) ln = _ kt
[Co( NH3 )5 Br 2+ ]o
3600 s
k = 6.3 x 10-6/s; t = 10.0 h x = 36,000 s
1h
ln[Co(NH3)5Br2+]t = _ kt + ln[Co(NH3)5Br2+]o
ln[Co(NH3)5Br2+]t = _ (6.3 x 10_ 6 /s)(36,000 s) + ln(0.100)
ln[Co(NH3)5Br2+]t = -2.5294; After 10.0 h, [Co(NH3)5Br2+] = e-2.5294 = 0.080 M

(b) [Co(NH3)5Br2+]o = 0.100 M


If 75% of the Co(NH3)5Br2+ reacts then 25% remains.
[Co(NH3)5Br2+]t = (0.25)(0.100 M) = 0.025 M
[Co( NH3 )5 Br 2+ ]t
ln
[Co( NH3 )5 Br 2+ ]t [Co( NH3 )5 Br 2+ ]o
ln = _ kt ; t =
[Co( NH3 )5 Br 2+ ]o _k
 0.025 
ln  
t=  0.100 
= 2.2 x 105 s; t = 2.2 x 105 s x
1h
= 61 h
_6
_ (6.3 x 10 /s) 3600 s

292
Chapter 12 - Chemical Kinetics
_____________________________________________________________________________

12.8

Slope = -0.03989/min = -6.6 x 10-4/s and k = -slope


A plot of ln[cyclopropane] versus time is linear, indicating that the data fit the equation
for a first-order reaction. k = 6.6 x 10-4/s (0.040/min)

12.9 (a) k = 1.8 x 10-5/s


0.693 0.693 1h
t1/2 = = = 38,500 s; t1/2 = 38,500 s x = 11 h
k 1.8 x 10_ 5 /s 3600 s

t1/2 t1/2 t1/2 t1/2


(b) 0.30 M → 0.15 M → 0.075 M → 0.0375 M → 0.019 M
(c) Because 25% of the initial concentration corresponds to 1/4 or (1/2)2 of the initial
concentration, the time required is two half-lives: t = 2t1/2 = 2(11 h) = 22 h

12.10 After one half-life, there would be four A molecules remaining. After two half-lives,
there would be two A molecules remaining. This is represented by the drawing at t = 10
min. 10 min is equal to two half-lives, therefore, t1/2 = 5 min for this reaction. After 15
min (three half-lives) only one A molecule would remain.

12.11

293
Chapter 12 - Chemical Kinetics
_____________________________________________________________________________

(a) A plot of 1/[HI] versus time is linear. The reaction is second-order.


(b) k = slope = 0.0308/(M ⋅ min)
1 1 1  1  1 1 
(c) t =  _ = _ = 260 min
k [HI ]t [HI ]o  0.0308 / (M • min)  0.100 M 0.500 M 

(d) It requires one half-life (t1/2) for the [HI] to drop from 0.400 M to 0.200 M.
1 1
t1/2 = = = 81.2 min
k[HI ]o [0.0308 /(Mcdot min)](0.400 M)

12.12 (a) NO2(g) + F2(g) → NO2F(g) + F(g)


F(g) + NO2(g) → NO2F(g)
Overall reaction 2 NO2(g) + F2(g) → 2 NO2F(g)
Because F(g) is produced in the first reaction and consumed in the second, it is a reaction
intermediate.
(b) In each reaction there are two reactants, so each elementary reaction is bimolecular.

12.13 (a) Rate = k[O3][O] (b) Rate = k[Br]2[Ar] (c) Rate = k[Co(CN)5(H2O)2-]

12.14 Co(CN)5(H2O)2-(aq) → Co(CN)52-(aq) + H2O(l) (slow)


Co(CN)5 (aq) + I (aq) → Co(CN)5I (aq)
2- - 3-
(fast)
Overall reaction Co(CN)5(H2O) (aq) + I (aq) → Co(CN)5I (aq) + H2O(l)
2- - 3-

The predicted rate law for the overall reaction is the rate law for the first (slow)
elementary reaction: Rate = k[Co(CN)5(H2O)2-]
The predicted rate law is in accord with the observed rate law.

12.15 (a) Ea = 100 kJ/mol - 20 kJ/mol = 80 kJ/mol


(b) The reaction is endothermic because the energy of the products is higher than the
energy of the reactants.

(c)

   _  1 1
12.16 (a) ln  k 2  =  Ea  _ 
 k1   R  T 2 T1 
k1 = 3.7 x 10-5/s, T1 = 25oC = 298 K
k2 = 1.7 x 10-3/s, T2 = 55oC = 328 K

294
Chapter 12 - Chemical Kinetics
_____________________________________________________________________________

[ln k 2 _ ln k1] R
Ea = _
 1 1
 _ 
 T 2 T1 
[ln(1.7 x 10_ 3) _ ln(3.7 x 10_ 5)][8.314 x 10_ 3 kJ/(Kcdot mol)]
Ea = _ = 104 kJ/mol
 1 1 
 _ 
 328 K 298 K 
(b) k1 = 3.7 x 10-5/s, T1 = 25oC = 298 K
solve for k2, T2 = 35oC = 308 K
 _  1 1
ln k2 =  E a  _  + ln k1
 R  T 2 T1 
 _ 104 kJ/mol  1 1 
ln k2 =   _  + ln (3.7 x 10_ 5)
 8.314 x 10 kJ/(K • mol)  308 K 298 K 
_3

ln k2 = -8.84; k2 = e-8.84 = 1.4 x 10-4/s

12.17 Assume that concentration is proportional to the number of each molecule in a box.
(a) From boxes (1) and (2), the concentration of A doubles, B and C2 remain the same
and the rate does not change. This means the reaction is zeroth-order in A.
From boxes (1) and (3), the concentration of C2 doubles, A and B remain the same and
the rate doubles. This means the reaction is first-order in C2.
From boxes (1) and (4), the concentration of B triples, A and C2 remain the same and the
rate triples. This means the reaction is first-order in B.
(b) Rate = k [B][C2]
(c) B + C2 → BC2 (slow)
A + BC2 → AC + BC
A + BC → AC + B
2 A + C2 → 2 AC (overall)
(d) B doesn’t appear in the overall reaction because it is consumed in the first step and
regenerated in the third step. B is therefore a catalyst. BC2 and BC are intermediates
because they are formed in one step and then consumed in a subsequent step in the
reaction.

12.18 Nitroglycerin contains three nitro groups per molecule. Because the bonds in nitro groups
are relatively weak (about 200 kJ/mol) and because the explosion products (CO2, N2,
H2O, and O2) are extremely stable, a great deal of energy is released (very exothermic)
during an explosion.

12.19 Secondary explosives are generally less sensitive to heat and shock than primary
explosives. This would indicate that secondary explosives should have a higher
activation energy than primary explosives.

12.20 C5H8N4O12(s) → 4 CO2(g) + 4 H2O(g) + 2 N2(g) + C(s)

295
Chapter 12 - Chemical Kinetics
_____________________________________________________________________________

∆Horxn = [4 ∆Hof (CO2) + 4 ∆Hof (H2O)] - ∆Hof (C5H8N4O12)


∆Horxn = [(4 mol)(-393.5 kJ/mol) + (4 mol)(-241.8 kJ/mol)] - [(1 mol)(537 kJ/mol)]
∆Horxn = -3078 kJ

12.21 C5H8N4O12(s) → 4 CO2(g) + 4 H2O(g) + 2 N2(g) + C(s)


C5H8N4O12, 316.14 amu; 1.54 kg = 1.54 x 103 g; 800oC = 1073 K
From the reaction, 1 mole of PETN produces 10 moles of gas.
1 mol PETN 10 mol gas
mol gas = 1.54 x 103 g PETN x x = 48.7 mol
316.14 g PETN 1 mol PETN
PV = nRT
 L • atm 
(48.7 mol) 0.082 06 (1073 K)
nRT  K • mol 
V= = = 4.40 x 103 L
P 0.975 atm

Understanding Key Concepts

12.22 (a) Because Rate = k[A][B], the rate is proportional to the product of the number of A
molecules and the number of B molecules. The relative rates of the reaction in vessels
(a) – (d) are 2 : 1 : 4 : 2.
(b) Because the same reaction takes place in each vessel, the k's are all the same.

12.23 (a) Because Rate = k[A], the rate is proportional to the number of A molecules in each
reaction vessel. The relative rates of the reaction are 2 : 4 : 3.
(b) For a first-order reaction, half-lives are independent of concentration. The half-lives
are the same.
(c) Concentrations will double, rates will double, and half-lives will be unaffected.

12.24 (a) For the first-order reaction, half of the A molecules are converted to B molecules
each minute.

(b) Because half of the A molecules are converted to B molecules in 1 min, the half-life
is 1 minute.

12.25 (a) Two molecules of A are converted to two molecules of B every minute. This means
the rate is constant throughout the course of the reaction. The reaction is zeroth-order.

296
Chapter 12 - Chemical Kinetics
_____________________________________________________________________________

(b)
(c) Rate = k
 6.0 x 1021 molecules  1 mol 
(2)  23

 1.0 L  6.022 x 10 molecules  1 min
k= x = 3.3 x 10-4 M/s
min 60 s

12.26 (a) Because the half-life is inversely proportional to the concentration of A molecules,
the reaction is second-order in A.
(b) Rate = k[A]2
(c) The second box represents the passing of one half-life, and the third box represents
the passing of a second half-life for a second-order reaction. A relative value of k can be
calculated.
1 1
k= = = 0.0625
t1/ 2 [A] (1)(16)
1 1
t1/2 in going from box 3 to box 4 is: t1/2 = = = 4 min
k[A] (0.0625)(4)
(For fourth box, t = 7 min)

12.27 (a) bimolecular (b) unimolecular (c) termolecular

12.28 (a) BC + D → B + CD
(b) 1. B–C + D (reactants), A (catalyst); 2. B---C---A (transition state), D (reactant);
3. A–C (intermediate), B (product), D (reactant); 4. A---C---D (transition state),
B (product); 5. A (catalyst), C–D + B (products)
(c) The first step is rate determining because the first maximum in the potential energy
curve is greater than the second (relative) maximum; Rate = k[A][BC]
(d) Endothermic

12.29

297
Chapter 12 - Chemical Kinetics
_____________________________________________________________________________
Additional Problems
Reaction Rates

mol
12.30 M/s or
L•s

12.31 molecules/(cm3 ⋅ s)

_ ∆[cyclopropane] 0.080 M _ 0.098 M


12.32 (a) Rate = =_ = 3.6 x 10-3 M/min
∆t 5.0 min _ 0.0 min
M 1 min
Rate = 3.6 x 10-3 x = 6.0 x 10-5 M/s
min 60 s
_ ∆[cyclopropane] 0.044 M _ 0.054 M
(b) Rate = =_ = 2.0 x 10-3 M/min
∆t 20.0 min _ l 5.0 min
M 1 min
Rate = 2.0 x 10-3 x = 3.3 x 10-5 M/s
min 60 s

∆[ NO2] (5.59 x 10_ 3 M) _ (6.58 x 10_ 3 M)


12.33 (a) Rate = _ =_ = 2.0 x 10-5 M/s
Deltat 100 s _ 50 s
∆[ NO2] (4.85 x 10 M) _ (5.59 x 10_ 3 M)
_3
(b) Rate = _ =_ = 1.5 x 10-5 M/s
Deltat 150 s _ 100 s

12.34

(a) The instantaneous rate of decomposition of N2O5 at t = 200 s is determined from the
slope of the curve at t = 200 s.
∆[ N 2 O5] (1.20 x 10_ 2 M) _ (1.69 x 10_ 2 M)
Rate = _ = _ slope = _ = 2.4 x 10-5 M/s
Deltat 300 s _ 100 s
(b) The initial rate of decomposition of N2O5 is determined from the slope of the curve at
t = 0 s. This is equivalent to the slope of the curve from 0 s to 100 s because in this time
interval the curve is almost linear.
∆[ ] (1.69 x 10_ 2 M) _ (2.00 x 10_ 2 M)
Initial rate = - N 2 O5 = _ slope = _ = 3.1 x 10-5 M/s
∆t 100 s _ 0 s

298
Chapter 12 - Chemical Kinetics
_____________________________________________________________________________

12.35
(a) The instantaneous rate of decomposition of NO2 at t = 100 s is determined from the
slope of the curve at t = 100 s.
∆[ NO2] (4.00 x 10_ 3 M) _ (7.00 x 10_ 3 M)
Rate = _ = _ slope = _ = 1.8 x 10-5 M/s
Deltat 190 s _ 20 s
(b) The initial rate of decomposition of NO2 is determined from the slope of the curve at
t = 0 s. This is equivalent to the slope of the curve from 0 s to 50 s because in this time
interval the curve is almost linear.
∆[ ] (6.58 x 10_ 3 M) _ (8.00 x 10_ 3 M)
Initial rate = - NO2 = _ slope = _ = 2.8 x 10-5 M/s
∆t 50 s _ 0 s

∆[H2] ∆[ N 2]
12.36 (a) _ =_ 3 ; The rate of consumption of H2 is 3 times faster.
∆t ∆t
∆[ NH3] ∆[ N 2]
(b) =_ 2 ; The rate of formation of NH3 is 2 times faster.
∆t ∆t

∆[O2] 5 ∆[ NH3]
12.37 (a) _ =_ ; The rate of consumption of O2 is 1.25 times faster.
∆t 4 ∆t
∆[NO] ∆[ NH3]
(b) =_ ; The rate of formation of NO is the same.
∆t ∆t
∆[H 2 O] 6 ∆[ NH3]
=_ ; The rate of formation of H2O is 1.5 times faster.
∆t 4 ∆t

∆[ N 2] 1 ∆[H 2] 1 ∆[ NH3]
12.38 N2(g) + 3 H2(g) → 2 NH3(g); - =_ =
∆t 3 ∆t 2 ∆t
∆[I _ ] ∆[S2 O8 2 _ ]
12.39 (a) _ =_ 3 = 3(1.5 x 10-3 M/s) = 4.5 x 10-3 M/s
∆t ∆t
∆[SO 4 ] 2_
∆[S2 O8 2 _ ]
(b) =_ 2 = 2(1.5 x 10-3 M/s) = 3.0 x 10-3 M/s
∆t ∆t

Rate Laws

12.40 Rate = k[NO]2[Br2]; 2nd order in NO; 1st order in Br2; 3rd order overall

12.41 Rate = k[CHCl3][Cl2]1/2; 1st order in CHCl3; 1/2 order in Cl2; 3/2 order overall

L
12.42 Rate = k[H2][ICl]; units for k are or 1/(M ⋅ s)
mol • s

299
Chapter 12 - Chemical Kinetics
_____________________________________________________________________________

12.43 Rate = k[NO]2[H2], units for k are 1/(M2 ⋅ s)

12.44 (a) Rate = k[CH3Br][OH-]


(b) Because the reaction is first-order in OH-, if the [OH-] is decreased by a factor of 5,
the rate will also decrease by a factor of 5.
(c) Because the reaction is first-order in each reactant, if both reactant concentrations are
doubled, the rate will increase by a factor of 2 x 2 = 4.

12.45 (a) Rate = k[Br-][BrO3-][H+]2


(b) The overall reaction order is 1 + 1 + 2 = 4.
(c) Because the reaction is second-order in H+, if the [H+] is tripled, the rate will increase
by a factor of 32 = 9.
(d) Because the reaction is first-order in both Br- and BrO3-, if both reactant
concentrations are halved, the rate will decrease by a factor of 4 (1/2 x 1/2 = 1/4).

12.46 (a) Rate = k[CH3COCH3]m

   7.8 x 10 
_5
ln  Rate2  ln  _5

m=  Rate1  = 
5.2 x 10 
= 1; Rate = k[CH3COCH3]
 [CH3 COCH3 ]2   9.0 x 10 
_3
ln   ln  _3

 [CH3 COCH3 ]1   6.0 x 10 

Rate 5.2 x 10_ 5 M/s


(b) From Experiment 1: k = = = 8.7 x 10-3/s
[CH3 COCH3] 6.0 x 10_ 3 M
(c) Rate = k[CH3COCH3] = (8.7 x 10-3/s)(1.8 x 10-3M) = 1.6 x 10-5 M/s

12.47 (a) Rate = k[CH3NNCH3]m

   2.0 10 
_6
x
ln Rate2  ln _6

m=  Rate1  = 
6.0 10 
x
= 1; Rate = k[CH3NNCH3]
 [CH3 NNCH3 ]2   8.0 10 
_3
x
ln  ln _2

 [CH3 NNCH3 ]1   2.4 10 
x
Rate 6.0 x 10_ 6 M/s
(b) From Experiment 1: k = = _2
= 2.5 x 10-4/s
[CH3 NNCH3] 2.4 x 10 M
(c) Rate = k[CH3NNCH3] = (2.5 x 10-4/s)(0.020 M) = 5.0 x 10-6 M/s

12.48 (a) Rate = k[NH4+]m[NO2-]n

300
Chapter 12 - Chemical Kinetics
_____________________________________________________________________________

   3.6 x 10_ 6     5.4 x 10_ 6 


ln  Rate2  ln   ln  Rate3  ln  
 Rate1   7.2 x 10_ 6   Rate2   3.6 x 10_ 6 
m= = = 1; n= = =1
 [ NH+4 ]2   0.12   [ NO2_ ]3   0.15 
ln  + 
 ln   ln   ln  
 [ NH 4 ]1
 0.24   [ NO
_
2 ]2
 0.10 
Rate = k[NH4+][NO2-]
Rate 7.2 x 10_ 6 M/s
(b) From Experiment 1: k = + _
= = 3.0 x 10-4/(M ⋅ s)
[ NH 4 ][ NO2 ] (0.24 M)(0.10 M)
(c) Rate = k[NH4 ][NO2 ] = [3.0 x 10-4/(M ⋅ s)](0.39 M)(0.052 M) = 6.1 x 10-6 M/s
+ -

12.49 (a) Rate = k[NO]m[Cl2]n

   4.0 x 10_ 2   Rate3   5.0 x 10_ 3 


ln Rate2  ln  ln   ln  
 Rate1   1.0 x 10_ 2   Rate1   1.0 x 10_ 2 
m= = = 2; n = = =1
 [NO ]2   0.26   [Cl2 ]3   0.10 
ln  ln  ln  ln 
 [NO ]1
 0.13  [
 2 1
Cl ]  0.20 
Rate = k[NO]2[Cl2]
Rate 1.0 x 10_ 2 M/s
(b) From Experiment 1: k = = = 3.0/(M2 ⋅ s)
[NO ]2 [Cl2] (0.13 M )2 (0.20 M)
(c) Rate = k[NO]2[Cl2] = [3.0/(M2 ⋅ s)](0.12 M)2(0.12 M) = 5.2 x 10—3 M/s

Integrated Rate Law; Half-Life

[C3 H6 ]t
12.50 ln = _ kt , k = 6.7 x 10-4/s
[C3 H6 ]0
60 s
(a) t = 30 min x = 1800 s
1 min
ln[C3H6]t = _ kt + ln[C3H6]0 = _ (6.7 x 10_ 4 /s)(1800 s) + ln(0.0500) = - 4.202
[C3H6]t = e- 4.202 = 0.015 M

[C3 H6 ]t  0.0100 
ln ln  
(b) t =
[C3 H6 ]0
=  0.0500 
= 2402 s; t = 2402 s x
1 min
= 40 min
_4
_k _ (6.7 x 10 /s) 60 s
(c) [C3H6]0 = 0.0500 M; If 25% of the C3H6 reacts then 75% remains.
[C3H6]t = (0.75)(0.0500 M) = 0.0375 M.

[C3 H6 ]t  0.0375 
ln ln  
t=
[C3 H6 ]0
=  0.0500 
= 429 s; t = 429 s x
1 min
= 7.2 min
_k _ (6.7 x 10_ 4 /s) 60 s

301
Chapter 12 - Chemical Kinetics
_____________________________________________________________________________
[CH3 NC ]t
12.51 ln = _ kt , k = 5.11 x 10-5/s
[CH3 NC ]0
60 min 60 s
(a) t = 2.00 hr x x = 7200 s
1 hr 1 min
ln[CH3NC]t = _ kt + ln[CH3NC]0 = _ (5.11 x 10_ 5 /s)(7200 s) + ln(0.0340) = -3.749
[CH3NC]t = e-3.749 = 0.0235 M

[CH3 NC ]t  0.0300 
ln ln  
(b) t =
[CH3 NC ]0
=  0.0340  = 2449 s; t = 2449 s x
1 min
= 40.8 min
_k _ 5.11 x 10_ 5 /s) 60 s

(c) [CH3NC]0 = 0.0340 M; If 20% of the CH3NC reacts then 80% remains.
[CH3NC]t = (0.80)(0.0340 M) = 0.0272 M.

[CH3 NC ]t  0.0272 
ln ln  
t=
[CH3 NC ]0
=  0.0340  = 4367 s; t = 4367 s x
1 min
= 72.8 min
_k _ (5.11 x 10_ 5 /s) 60 s

0.693 0.693
12.52 t1/2 = = = 1034 s = 17 min
k 6.7 x 10_ 4 /s
[C H ] (0.0625)(0.0500)
ln 3 6 t ln
[C3 H6 ]o (0.0500)
t= = = 4140 s
_k _ 6.7 x 10_ 4 /s
1 min
t = 4140 s x = 69 min
60 s
t1/2 t1/2 t1/2 t1/2
This is also 4 half-lives. 100 → 50 → 25 → 12.5 → 6.25

0.693 0.693
12.53 t1/2 = = = 13,562 s
k 5.11 x 10_ 5 /s
1 min 1 hr
t1/2 = 13,562 s x x = 3.77 hr
60 s 60 min
[CH3 NC ]t (0.125)(0.0340)
ln ln
[CH3 NC ]o (0.0340)
t= = = 40,694 s
_k _ 5.11 x 10_ 5 /s
1 min 1 hr
t = 40,694 s x x = 11.3 hr
60 s 60 min
t1/2 t1/2 t1/2
This is also 3 half-lives. 100 → 50 → 25 → 12.5

302
Chapter 12 - Chemical Kinetics
_____________________________________________________________________________

12.54 t1/2 = 8.0 h


t1/2 t1/2
0.60 M → 0.30 M → 0.15 M requires 2 half-lives so it will take 16.0 h.

12.55 t1/2 = 3.33 h


t1/2 t1/2 t1/2 t1/2
0.800 M → 0.400 M → 0.200 M → 0.100 M → 0.0500 M
requires 4 half-lives so it will take 13.3 h.

1 1
12.56 kt = _ , k = 4.0 x 10-2/(M ⋅ s)
[C4 H6 ]t [C4 H6 ]0

60 min 60 s
(a) t = 1.00 h x x = 3600 s
1 hr 1 min
1 1 1
= kt + = (4.0 x 10_ 2 /(M • s))(3600 s) +
[C4 H 6 ]t [C4 H 6 ]0 0.0200 M
1
= 194/M and [C4H6] = 5.2 x 10-3 M
[C4 H6 ]t


1 1 1 
(b) t =  _ 
k
 [C4 H6 ]t [C4 H6 ]0 
1  1 1 
t= _ = 11,250 s
4.0 x 10 /(M • s)  (0.0020 M) (0.0200 M) 
_2 
1 min 1 hr
t = 11,250 s x x = 3.1 h
60 s 60 min

1 1
12.57 kt = _ , k = 9.7 x 10-6/(M ⋅ s)
[HI ]t [HI ]0

24 hr 60 min 60 s
(a) t = 6.00 day x x x = 518,400 s
1 day 1 hr 1 min
1 1 1
= kt + = (9.7 x 10_ 6 /(M • s))(518,400 s) +
[HI ]t [HI ]0 0.100 M
1
= 15.03/M and [HI] = 0.067 M
[HI ]t

303
Chapter 12 - Chemical Kinetics
_____________________________________________________________________________

1 1 1 
(b) t =  _ 
k [HI ]t [HI ]0 
1  1 1 
t= _ = 4,123,711 s
9.7 x 10 /(M • s)  (0.020 M) (0.100 M) 
_6 
1 min 1 hr 1 day
t = 4,123,711 s x x x = 48 days
60 s 60 min 24 hr

1 1
12.58 t1/2 = = _2
= 1250 s = 21 min
k[C4 H 6 ]o [4.0 x 10 /(Mcdot s)](0.0200 M)
1 1
t = t1/2 = = _2
= 2500 s = 42 min
k[C4 H6 ]o [4.0 x 10 /(Mcdot s)](0.0100 M)

1 1
12.59 t1/2 = = = 1,030,928 s
k[HI ]o [9.7 x 10 /(M• s)](0.100 M)
_6

1 min 1 hr 1 day
1,030,928 s x x x = 12 days
60 s 60 min 24 hr
1 1
t = t1/2 = = = 4,123,711 s
k[HI ]o [9.7 x 10 /(M• s)](0.0250 M)
_6

1 min 1 hr 1 day
4,123,711 s x x x = 48 days
60 s 60 min 24 hr
12.60 time (min) [N2O] ln[N2O] 1/[N2O]
0 0.250 -1.386 4.00
60 0.218 -1.523 4.59
90 0.204 -1.590 4.90
120 0.190 -1.661 5.26
180 0.166 -1.796 6.02

A plot of ln [N2O] versus time is linear. The reaction is first-order in N2O.


k = -slope = -(-2.28 x 10-3/min) = 2.28 x 10-3/min
1 min
k = 2.28 x 10-3/min x = 3.79 x 10-5/s
60 s

12.61 time (s) [NOBr] ln[NOBr] 1/[NOBr]


0 0.0400 -3.219 25.0
10 0.0303 -3.497 33.0

304
Chapter 12 - Chemical Kinetics
_____________________________________________________________________________

20 0.0244 -3.713 41.0


30 0.0204 -3.892 49.0
40 0.0175 -4.046 57.1

A plot of 1/[NOBr] versus time is linear. The reaction is second-order in NOBr.


k = slope = 0.80/(M ⋅ s)

0.693 0.693
12.62 k = = = 2.79 x 10—3/s
t1 / 2 248 s

1 60 s
12.63 t1/2 = ; t1/2 = 25 min x = 1500 s
k[A ]o 1 min
1 1
k= = = 1.8 x 10-2 M-1 s-1
t1/ 2 [A ]0 (1500 s)(0.036 M)

12.64 (a) The units for the rate constant, k, indicate the reaction is zeroth-order.
(b) For a zeroth-order reaction, [A]t - [A]o = -kt
60 s
t = 30 min x = 1800 s
1 min
[A]t = -kt + [A]o = - (3.6 x 10-5 M/s)(1800 s) + 0.096 M = 0.031 M
(c) Let [A]t = [A]o/2
[A ]o / 2 _ [A ]o 0.096 / 2 M _ 0.096 M
t1 / 2 = = = 1333 s
_k _ 3.6 x 10_ 5 M/s
1 min
t1/ 2 = 1333 s x = 22 min
60 s

305
Chapter 12 - Chemical Kinetics
_____________________________________________________________________________

12.65 (a)

A plot of [AB] versus time is linear. The reaction is zeroth-order and k = - slope.
0.140 M _ 0.200 M
k= _ = 7.50 x 10-4 M/min
80.0 min _ 0 min
1 min
k = 7.50 x 10-4 M/min x = 1.25 x 10-5 M/s
60 s
(b) [A]t - [A]o = -kt
[A]t = -kt + [A]o = -(7.50 x 10-4 M/min)(126 min) + 0.200 M = 0.105 M

(c) [A]t - [A]o = -kt


[A ]t _ [A ]o 0.100 M _ 0.200 M
t= = = 133 min
_k _ 7.50 x 10_ 4 M/min

Reaction Mechanisms

12.66 An elementary reaction is a description of an individual molecular event that involves the
breaking and/or making of chemical bonds. By contrast, the overall reaction describes
only the stoichiometry of the overall process but provides no information about how the
reaction occurs.

12.67 Molecularity is the number of reactant molecules or atoms for an elementary reaction.
Reaction order is the sum of the exponents of the concentration terms in the rate law.

12.68 There is no relationship between the coefficients in a balanced chemical equation for an
overall reaction and the exponents in the rate law unless the overall reaction occurs in a
single elementary step, in which case the coefficients in the balanced equation are the
exponents in the rate law.

12.69 The rate-determining step is the slowest step in a multistep reaction. The coefficients in
the balanced equation for the rate-determining step are the exponents in the rate law.

306
Chapter 12 - Chemical Kinetics
_____________________________________________________________________________

12.70 (a) H2(g) + ICl(g) → HI(g) + HCl(g)


HI(g) + ICl(g) → I2(g) + HCl(g)
Overall reaction H2(g) + 2 ICl(g) → I2(g) + 2 HCl(g)
(b) Because HI(g) is produced in the first step and consumed in the second step, it is a
reaction intermediate.
(c) In each reaction there are two reactant molecules, so each elementary reaction is
bimolecular.

12.71 (a) NO(g) + Cl2(g) → NOCl2(g)


NOCl2(g) + NO(g) → 2 NOCl(g)
Overall reaction 2 NO(g) + Cl2(g) → 2 NOCl(g)
(b) Because NOCl2 is produced in the first step and consumed in the second step, NOCl2
is a reaction intermediate.
(c) Each elementary step is bimolecular.

12.72 (a) bimolecular, Rate = k[O3][Cl] (b) unimolecular, Rate = k[NO2]


(c) bimolecular, Rate = k[ClO][O] (d) termolecular, Rate = k[Cl]2[N2]

12.73 (a) unimolecular, Rate = k[I2] (b) termolecular, Rate = k[NO]2[Br2]


(c) bimolecular, Rate = k[CH3Br][OH—] (d) unimolecular, Rate = k[N2O5]

12.74 (a) NO2Cl(g) → NO2(g) + Cl(g)


Cl(g) + NO2Cl(g) → NO2(g) + Cl2(g)
Overall reaction 2 NO2Cl(g) → 2 NO2(g) + Cl2(g)
(b) 1. unimolecular; 2. bimolecular
(c) Rate = k[NO2Cl]

12.75 (a) Mo(CO)6 → Mo(CO)5 + CO


Mo(CO)5 + L → Mo(CO)5L
Overall reaction Mo(CO)6 + L → Mo(CO)5L + CO
(b) 1. unimolecular; 2. bimolecular
(c) Rate = k[Mo(CO)6]

12.76 NO2(g) + F2(g) → NO2F(g) + F(g) (slow)


F(g) + NO2(g) → NO2F(g) (fast)

12.77 O3(g) + NO(g) → O2(g) + NO2(g) (slow)


NO2(g) + O(g) → O2(g) + NO(g) (fast)

307
Chapter 12 - Chemical Kinetics
_____________________________________________________________________________

The Arrhenius Equation

12.78 Very few collisions involve a collision energy greater than or equal to the activation
energy, and only a fraction of those have the proper orientation for reaction.

12.79 The two reactions have frequency factors that differ by a factor of 10.

12.80 Plot ln k versus 1/T to determine the activation energy, Ea

Slope = -1.25 x 104 K


Ea = -R(slope) = -[8.314 x 10-3 kJ/(K ⋅ mol)](-1.25 x 104 K) = 104 kJ/mol
12.81 Plot ln k versus 1/T to determine the activation energy, Ea.

Slope = -1.359 x 104 K


Ea = -R(slope) = -[8.314 x 10-3 kJ/(K ⋅ mol)](-1.359 x 104 K) = 113 kJ/mol

   _  1 1
12.82 (a) ln  k 2  =  Ea  _ 
 k1   R  T 2 T1 
k1 = 1.3/(M ⋅ s), T1 = 700 K
k2 = 23.0/(M ⋅ s), T2 = 800 K
[ln k 2 _ ln k1](R)
Ea = _
 1 1
 _ 
 T 2 T1 
[ln (23.0) _ ln (1.3)][8.314 x 10_ 3 kJ/(K • mol)]
Ea = _ = 134 kJ/mol
 1 1 
 _ 
 800 K 700 K 

(b) k1 = 1.3/(M ⋅ s), T1 = 700 K


solve for k2, T2 = 750 K
 _  1 1
ln k2 =  E a  _  + ln k1
 R  T 2 T1 

308
Chapter 12 - Chemical Kinetics
_____________________________________________________________________________

 _ 133.8 kJ/mol  1 1 
ln k2 =   _  + ln (1.3) = 1.795
 8.314 x 10 kJ/(K • mol)  750 K 700 K 
_3

k2 = e1.795 = 6.0/(M ⋅ s)

   _  1 1
12.83 ln  k 2  =  Ea   _ 
 k1   R   T 2 T1 
(a) Because the rate doubles, k2 = 2k1
k1 = 1.0 x 10-3/s, T1 = 25oC = 298 K
k2 = 2.0 x 10-3/s, T2 = 35oC = 308 K
[ln k 2 _ ln k1](R)
Ea = _
 1 1
 _ 
 T 2 T1 
[ln(2.0 x 10_ 3) _ ln(1.0 x 10_ 3)][8.314 x 10_ 3 kJ/(Kcdot mol)]
Ea = _ = 53 kJ/mol
 1 1 
 _ 
 308 K 298 K 

(b) Because the rate triples, k2 = 3k1


k1 = 1.0 x 10-3/s, T1 = 25oC = 298 K
k2 = 3.0 x 10-3/s, T2 = 35oC = 308 K
[ln(3.0 x 10_ 3) _ ln(1.0 x 10_ 3)][8.314 x 10_ 3 kJ/(Kcdot mol)]
Ea = _ = 84 kJ/mol
 1 1 
 _ 
 308 K 298 K 

   _  1 1
12.84 ln  k 2  =  Ea  _ 
 k1   R  T 2 T1 
assume k1 = 1.0/(M ⋅ s) at T1 = 25oC = 298 K
assume k2 = 15/(M ⋅ s) at T2 = 50oC = 323 K
[ln k 2 _ ln k1](R)
Ea = _
 1 1
 _ 
 T 2 T1 
[ln (15) _ ln (1.0)][8.314 x 10_ 3 kJ/(K • mol)]
Ea = _ = 87 kJ/mo1
 1 1 
 _ 
 323 K 298 K 

   _  1 1
12.85 ln  k 2  =  Ea  _ 
 k1   R  T 2 T1 
assume k1 = 1.0/(M ⋅ s) at T1 = 15oC = 288 K
assume k2 = 6.37/(M ⋅ s) at T2 = 45oC = 318 K

309
Chapter 12 - Chemical Kinetics
_____________________________________________________________________________

[ln k 2 _ ln k1](R)
Ea = _
 1 1
 _ 
 T 2 T1 
[ln (6.37) _ ln (1.0)][8.314 x 10_ 3 kJ/(K • mol)]
Ea = _ = 47.0 kJ/mo1
 1 1 
 _ 
 318 K 288 K 

12.86

12.87 (a) (b)

Catalysis

12.88 A catalyst does participate in the reaction, but it is not consumed because it reacts in one
step of the reaction and is regenerated in a subsequent step.

12.89 A catalyst doesn't appear in the chemical equation for a reaction because a catalyst reacts
in one step of the reaction but is regenerated in a subsequent step.

12.90 A catalyst increases the rate of a reaction by changing the reaction mechanism and
lowering the activation energy.

310
Chapter 12 - Chemical Kinetics
_____________________________________________________________________________

12.91 A homogeneous catalyst is one that exists in the same phase as the reactants.
Example: NO(g) acts as a homogeneous catalyst for the conversion of O2(g) to O3(g).
A heterogeneous catalyst is one that exists in a different phase from the reactants.
Example: solid Ni, Pd, or Pt for catalytic hydrogenation, C2H4(g) + H2(g) → C2H6(g).

12.92 (a) O3(g) + O(g) → 2 O2(g) (b) Cl acts as a catalyst.


(c) ClO is a reaction intermediate.
(d) A catalyst reacts in one step and is regenerated in a subsequent step. A reaction
intermediate is produced in one step and consumed in another.
12.93 (a) 2 SO2(g) + 2 NO2(g) → 2 SO3(g) + 2 NO(g)
2 NO(g) + O2(g) → 2 NO2(g)
Overall reaction 2 SO2(g) + O2(g) → 2 SO3(g)
(b) NO2(g) acts as a catalyst because it is used in the first step and regenerated in the
second. NO(g) is a reaction intermediate because it is produced in the first step and
consumed in the second.

12.94 (a) NH2NO2(aq) + OH-(aq) → NHNO2-(aq) + H2O(l)


NHNO2-(aq) → N2O(g) + OH-(aq)
Overall reaction NH2NO2(aq) → N2O(g) + H2O(l)
(b) OH- acts as a catalyst because it is used in the first step and regenerated in the
second. NHNO2- is a reaction intermediate because it is produced in the first step and
consumed in the second.
(c) The rate will decrease because added acid decreases the concentration of OH-, which
appears in the rate law since it is a catalyst.

12.95 The reaction in Problem 12.77 involves a catalyst (NO) because NO is used in the first
step and is regenerated in the second step. The reaction also involves an intermediate
(NO2) because NO2 is produced in the first step and is used up in the second step.

General Problems

12.96 2 AB2 → A2 + 2 B2
(a) Measure the change in the concentration of AB2 as a function of time.
(b) and (c) If a plot of [AB2] versus time is linear, the reaction is zeroth-order and
k = -slope. If a plot of ln [AB2] versus time is linear, the reaction is first-order and
k = -slope. If a plot of 1/[AB2] versus time is linear, the reaction is second-order and
k = slope.

12.97 A → B + C
(a) Measure the change in the concentration of A as a function of time at several different
temperatures.
(b) Plot ln [A] versus time, for each temperature. Straight line graphs will result and k
at each temperature equals -slope. Graph ln k versus 1/K, where K is the kelvin
temperature. Determine the slope of the line. Ea = -R(slope) where R = 8.314 x 10-3

311
Chapter 12 - Chemical Kinetics
_____________________________________________________________________________

kJ/(K ⋅ mol).

12.98 (a) Rate = k[B2][C]


(b) B2 + C → CB + B (slow)
CB + A → AB + C (fast)
(c) C is a catalyst. C does not appear in the chemical equation because it is consumed in
the first step and regenerated in the second step.

12.99 (a)

(b) Reaction 2 is the fastest (smallest Ea), and reaction 3 is the slowest (largest Ea).
(c) Reaction 3 is the most endothermic (positive ∆E), and reaction 1 is the most
exothermic (largest negative ∆E).

12.100 The first maximum represents the potential energy of the transition state for the first
step. The second maximum represents the potential energy of the transition state for the
second step. The saddle point between the two maxima represents the potential energy
of the intermediate products.

12.101 Because 0.060 M is half of 0.120 M, 5.2 h is the half-life.


For a first-order reaction, the half-life is independent of initial concentration. Because
0.015 M is half of 0.030 M, it will take one half-life, 5.2 h.
0.693 0.693
k= = = 0.133/h
t1/ 2 5.2 h
[N O ]
ln 2 5 t = _ kt
[ N 2 O5 ]0
[N O ]
ln 2 5 t  0.015 
ln  
t=
[ N 2 O5 ]0
=  0.480 
= 26 h (Note that t is five half-lives.)
_k _ (0.133 /h)

12.102 (a) The reaction rate will increase with an increase in temperature at constant volume.
(b) The reaction rate will decrease with an increase in volume at constant temperature
because reactant concentrations will decrease.

312
Chapter 12 - Chemical Kinetics
_____________________________________________________________________________

(c) The reaction rate will increase with the addition of a catalyst.
(d) Addition of an inert gas at constant volume will not affect the reaction rate.

12.103 As the temperature of a gas is raised by 10oC, even though the collision frequency increases by on

12.104 (a) Rate = k[C2H4Br2]m[I-]n

   1.74 x 10_ 4 
ln  Rate2  ln  
 Rate1   6.45 x 10_ 5 
m= = =1
 [C2 H4 Br 2 ]2   0.343 
ln   ln  
 [ C 2 H 4 Br 2 ]1
 0.127 

 Rate3 • [C2 H 4 Br 2 ]2   _4

ln   ln  (1.26 x 10 )(0.343) 
Rate2 • [C2 H4 Br 2 ]3  _4
n=  = 
(1.74 x 10 )(0.203) 
=1
 [I ]3 
_
 0.125 
ln  _  ln  
 [I ]2   0.102 

Rate = k[C2H4Br2][I-]

(b) From Experiment 1:


Rate 6.45 x 10_ 5 M/s
k= _
= = 4.98 x 10-3/(M ⋅ s)
[C2 H 4 Br 2][I ] (0.127 M)(0.102 M)
(c) Rate = k[C2H4Br2][I-] = [4.98 x 10-3(M ⋅ s)](0.150 M)(0.150 M) = 1.12 x 10-4 M/s

12.105 (a) From the data in the table for Experiment 1, we see that 0.20 mol of A reacts with
0.10 mol of B to produce 0.10 mol of D. The balanced equation for the reaction is:
2A + B → D
∆A
(b) From the data in the table, initial Rates = _ have been calculated.
∆t
For example, from Experiment 1:
∆A (4.80 M _ 5.00 M)
Initial rate = _ =_ = 3.33 x 10-3 M/s
∆t 60 s
Initial concentrations and initial rate data have been collected in the table below.

EXPT [A]o (M) [B]o (M) [C]o (M) Initial Rate (M/s)
1 5.00 2.00 1.00 3.33 x 10-3
2 10.00 2.00 1.00 6.66 x 10-3
3 5.00 4.00 1.00 3.33 x 10-3
4 5.00 2.00 2.00 6.66 x 10-3

313
Chapter 12 - Chemical Kinetics
_____________________________________________________________________________

Rate = k[A]m[B]n[C]p
From Expts 1 and 2, [A] doubles and the initial rate doubles; therefore m = 1.
From Expts 1 and 3, [B] doubles but the initial rate does not change; therefore n = 0.
From Expts 1 and 4, [C] doubles and the initial rate doubles; therefore p = 1.
The reaction is: first-order in A; zeroth-order in B; first-order in C; second-order overall.
(c) Rate = k[A][C]
(d) C is a catalyst. C appears in the rate law, but it is not consumed in the reaction.
(e) A + C → AC (slow)
AC + B → AB + C (fast)
A + AB → D (fast)
(f) From data in Expt 1:
Rate ∆ D/ ∆ t 0.10 M/ 60 s
k= = = = 3.4 x 10-4/(M ⋅ s)
[A][C] [A][C] (5.00 M)(1.00 M)

12.106 For Ea = 50 kJ/mol


 _ 50 kJ/mol  -9
f = e _ Ea /RT = exp  = 2.0 x 10
 [8.314 x 10
_3
kJ/(K • mol)](300 K) 
For Ea = 100 kJ/mol
 _ 100 kJ/mol  -18
f = e _ Ea / RT = exp  = 3.9 x 10
 [8.314 x 10
_3
kJ/(K • mol)](300 K) 

   _  1 1
12.107 ln  k 2  =  Ea  _ 
 k1   R  T 2 T1 
k2 = 2.5k1
k1 = 1.0, T1 = 20oC = 293 K
k2 = 2.5, T2 = 30oC = 303 K
[ln k 2 _ ln k1](R)
Ea = _
 1 1
 _ 
 T 2 T1 
[ln(2.5) _ ln(1.0)][8.314 x 10_ 3 kJ/(K • mol)]
Ea = _ = 68 kJ/mol
 1 1 
 _ 
 303 K 293 K 

k1 = 1.0, T1 = 120oC = 393 K


k2 = ?, T2 = 130oC = 403 K
Solve for k2.
_  1 1
ln k2 = Ea  _  + ln k1
R  T 2 T1 

314
Chapter 12 - Chemical Kinetics
_____________________________________________________________________________

_ 68 kJ/mol  1 1 
ln k2 =  _  + ln(1.0) = 0.516
[8.314 x 10 kJ/(K • mol)]  403 K 393 K 
_3

k2 = e0.516 = 1.7; The rate increases by a factor of 1.7.

12.108 (a) 2 NO(g) + Br2(g) → 2 NOBr(g)


(b) Since NOBr2 is generated in the first step and consumed in the second step, NOBr2
is a reaction intermediate.
(c) Rate = k[NO][Br2]
(d) It can't be the first step. It must be the second step.

12.109 [A] = -kt + [A]o


[A]o/2 = -kt1/2 + [A]o
[A]o/2 - [A]o = -kt1/2
-[A]o/2 = -kt1/2
[A]o/2 = kt1/2
[A ]o
For a zeroth-order reaction, t1/2 = .
2 k
For a zeroth-order reaction, each half-life is half of the previous one.
For a first-order reaction, each half-life is the same as the previous one.
For a second-order reaction, each half-life is twice the previous one.

12.110 (a) 2 NO(g) _ N2O2(g) (fast)


N2O2(g) + H2(g) → N2O(g) + H2O(g) (slow)
N2O(g) + H2(g) → N2(g) + H2O(g) (fast)
Overall reaction 2 NO(g) + 2 H2(g) → N2(g) + 2 H2O(g)
(b) N2O2 and N2O are reaction intermediates because they are produced in one step of
the reaction and used up in a subsequent step.
(c) Rate = k2[N2O2][H2]
(d) Because the forward and reverse rates in step 1 are equal, k1[NO]2 = k-1[N2O2].
Solving for [N2O2] and substituting into the rate law for the second step gives
Rate = k2[N2O2][H2] = k1 k 2 [NO]2[H2]
k _1
Because the rate law for the overall reaction is equal to the rate law for the rate-
determining step, the rate law for the overall reaction is
Rate = k[NO]2[H2] where k = k1 k 2
k _1

12.111 (a) I-(aq) + OCl-(aq) → Cl-(aq) + OI-(aq)

315
Chapter 12 - Chemical Kinetics
_____________________________________________________________________________

∆[I _ ]
(b) From the data in the table, initial rates = _ have been calculated.
∆t
For example, from Experiment 1:
∆[I_ ] (2.17 x 10_ 4 M _ 2.40 x 10_ 4 M)
Initial rate = _ =_ = 2.30 x 10-6 M/s
Deltat 10 s

Initial concentrations and initial rate data have been collected in the table below.

EXPT [I-]o (M) [OCl-]o (M) [OH-]o (M) Initial Rate (M/s)
1 2.40 x 10-4 1.60 x 10-4 1.00 2.30 x 10-6
2 1.20 x 10-4 1.60 x 10-4 1.00 1.20 x 10-6
3 2.40 x 10-4 4.00 x 10-5 1.00 6.00 x 10-7
4 1.20 x 10-4 1.60 x 10-4 2.00 6.00 x 10-7

Rate = k[I-]m[OCl-]n[OH-]p

From Expts 1 and 2, [I-] is cut in half and the initial rate is cut in half; therefore m = 1.
From Expts 1 and 3, [OCl-] is reduced by a factor of four and the initial rate is reduced
by a factor of four; therefore n = 1.
From Expts 2 and 4, [OH-] is doubled and the initial rate is cut in half; therefore p = -1.
_
[ _ ][ ]
Rate = k I OCl _
[OH ]
From data in Expt 1:
_
Rate [ ] (2.30 x 10_ 6 M/s)(1.00 M)
k = _ OH_ = = 60/s
[I ][OCl ] (2.40 x 10_ 4 M)(1.60 x 10_ 4 M)

(c) The reaction does not occur by a single-step mechanism because OH- appears in the
rate law but not in the overall reaction.

(d) OCl-(aq) + H2O(l) _


HOCl(aq) + OH-(aq) (fast)
HOCl(aq) + I (aq) → HOI(aq) + Cl (aq) (slow)
- -

HOI(aq) + OH-(aq) → H2O(l) + OI-(aq) (fast)


Overall reaction I-(aq) + OCl-(aq) → Cl-(aq) + OI-(aq)

Because the forward and reverse rates in step 1 are equal, k1[OCl-][H2O] =
k-1[HOCl][OH-]. Solving for [HOCl] and substituting into the rate law for the second
step gives
_
[ ][ O][ _ ]
Rate = k2[HOCl][I-] = k1 k 2 OCl H 2_ I
k _1 [OH ]
[H2O] is constant and can be combined into k.

Because the rate law for the overall reaction is equal to the rate law for the rate-
determining step, the rate law for the overall reaction is

316
Chapter 12 - Chemical Kinetics
_____________________________________________________________________________

_
[ ][ _ ] [ O]
Rate = k OCl _ I where k = k1 k 2 H2
[OH ] k _1

12.112 (a) Ratef = kf[A] and Rater = kr[B]

(b)
[B] k f (3.0 x 10_ 3)
(c) When Ratef = Rater, kf[A] = kr[B], and = = =3
[A] k r (1.0 x 10_ 3)

12.113 (a) 1 → 1/2 → 1/4 → 1/8


After three half-lives, 1/8 of the strontium-90 will remain.
0.693 0.693
(b) k = = = 0.0239/y = 0.024/y
t1/ 2 29 y
(Sr _ 90 )t (0.01)
ln ln
(Sr _ 90 )o (1)
(c) t = = = 193 y
_k _ 0.0239 /y

0.693 0.693
12.114 k= = = 1.21 x 10-4/y
t1/ 2 5730 y
14
( C) (2.3)
ln 14 t ln
( C )o (15.3)
t= = _4
= 1.6 x 104 y
_k _ 1.21 x 10 /y

Rate
12.115 Rate = k [N2O4]; k =
[ N 2 O4]
5.0 x 103 M/s
At 25oC, k1 = = 5.0 x 104 s-1
0.10 M
2.3 x 104 M/s
At 40oC, k2 = = 1.5 x 105 s-1
0.15 M

317
Chapter 12 - Chemical Kinetics
_____________________________________________________________________________

25oC = 25 + 273 = 298 K and 40oC = 40 + 273 = 313 K


   _  1 1
ln  k 2  =  Ea  _ 
 k1   R  T 2 T1 
[ln k 2 _ ln k1](R)
Ea = _
 1 1
 _ 
 T 2 T1 
[ln(1.5 x 105) _ ln(5.0 x 104)][8.314 x 10_ 3 kJ/(K • mol)]
Ea = _ = 56.8 kJ/mol
 1 1 
 _ 
 313 K 298 K 

12.116 X → products is a first-order reaction


60 s
t = 60 min x = 3600 s
1 min
[X ]t
ln
[X ]t [X ]o
ln = - kt; k=
[X ]o _t
 0.600
M
ln  
k1 = 
o M
1.000
At 25 C, calculate k1: = 1.42 x 10-4 s-1
_ 3600
s
 0.200
M
ln  
k2 = 
o M
0.600
At 35 C, calculate k2: = 3.05 x 10-4 s-1
_ 3600
s
 0.010 M 
ln  
At an unknown temperature calculate k3. k3 =  0.200 M 
= 8.32 x 10-4 s-1
_ 3600 s
T1 = 25oC = 25 + 273 = 298 K
T2 = 35oC = 35 + 273 = 308 K

Calculate Ea using k1 and k2.


   _  1 1
ln  k 2  =  Ea  _ 
 k1   R  T 2 T1 
[ln k 2 _ ln k1](R)
Ea = _
 1 1
 _ 
 T 2 T1 
[ln(3.05 x 10_ 4) _ ln(1.42 x 10_ 4)][8.314 x 10_ 3 kJ/(Kcdot mol)]
Ea = _ = 58.3
 1 1 
 _ 
 308 K 298 K 
kJ/mol

Use Ea, k1, and k3 to calculate T3.

318
Chapter 12 - Chemical Kinetics
_____________________________________________________________________________

   8.32 x 10_ 4 
ln  k 3  ln  
1  k1  1  1.42 x 10_ 4  1
= + = + = 0.003104/K
T3  _ E a  T1  _ 58.3 kJ/mol  298 K
   
 R   8.314 x 10 kJ/(K • mol) 
_3

1
T3 = = 322 K = 322 - 273 = 49oC
0.003104 /K
At 3:00 p.m. raise the temperature to 49oC to finish the reaction by 4:00 p.m.

12.117 N2O4(g) → 2 NO2(g)


before (mm Hg) 17.0 0
change (mm Hg) -x +2x
after (mm Hg) 17.0 - x 2x

2x = 1.3 mm Hg
x = 1.3 mm Hg/2 = 0.65 mm Hg
Pt(N2O4) = 17.0 - x = 17.0 - 0.65 = 16.35 mm Hg

0.693 0.693
k= = _5
= 5.3 x 104 s-1
t1/ 2 1.3 x 10 s
ln P t ln
16.35
P
ln t = - kt; t = Po = 17 = 7.4 x 10-7 s
4 _1
Po _ k _ 5.3 x 10 s

12.118 (a) When equal volumes of two solutions are mixed, both concemtrations are cut in half.
[H3O+]o = [OH-]o = 1.0 M
When 99.999% of the acid is neutralized, [H3O+] = [OH-] = 1.0 M - (1.0 M x 0.99999)
= 1.0 x 10-5 M
Using the 2nd order integrated rate law:
1 1 1 1 1 
kt = _ ; t =  _ 
[H3 O+ ]t [H3 O+ ]o k [H3 O+ ]t [H3 O+ ]o 
1  1 1 
t= 11 _1 _1  _5
_  = 7.7 x 10-7 s
(1.3 x 10 M s )  (1.0 x 10 M) (1.0 M) 
(b) The rate of an acid-base neutralization reaction would be limited by the speed of
mixing, which is much slower than the intrinsic rate of the reaction itself.

1 1
12.119 2
=8 k t + 2
([O2] ) ([O2 ]o )
1 1
2
= 8(25 M _ 2 s _1)(100.0 s) + 2
([O2] ) (0.0100 M )

319
Chapter 12 - Chemical Kinetics
_____________________________________________________________________________

1
2
= 30,000 M-2
([O2] )
1
[O2] = = 0.005 77 M
30,000 M _ 2

2 NO(g) + O2(g) → 2 NO2(g)


before (M) 0.0200 0.0100 0
change (M) -2x -x +2x
after (M) 0.0200 - 2x 0.0100 - x 2x

[O2] = 0.005 77 M = 0.0100 M - x


x = 0.0100 M - 0.005 77 M = 0.004 23 M
[NO] = 0.0200 M - 2x = 0.0200 M - 2(0.004 23 M) = 0.0115 M
[O2] = 0.005 77 M
[NO2] = 2x = 2(0.004 23 M) = 0.008 46 M

12.120 Looking at the two experiments at 600 K, when the NO2 concentration is doubled, the
rate increased by a factor of 4. Therefore, the reaction is 2nd order.
Rate = k [NO2]2
Calculate k1 at 600 K: k1 = Rate/[NO2]2 = 5.4 x 10-7 M s-1/(0.0010 M)2 = 0.54 M-1 s-1
Calculate k2 at 700 K: k2 = Rate/[NO2]2 = 5.2 x 10-6 M s-1/(0.0020 M)2 = 13 M-1 s-1
Calculate Ea using k1 and k2.
   _  1 1
ln  k 2  =  Ea  _ 
 k1   R  T 2 T1 
[ln k 2 _ ln k1](R)
Ea = _
 1 1
 _ 
 T 2 T1 
[ln(13) _ ln(0.54)][8.314 x 10_ 3 kJ/(K • mol)]
Ea = _ = 111 kJ/mol
 1 1 
 _ 
 700 K 600 K 
Calculate k3 at 650 K using Ea and k1.

Solve for k3.


_ 1 1
ln k3 = Ea  _  + ln k1
R  T3 T1 
_ 111 kJ/mol  1 1 
ln k3 =  _  + ln(0.54) = 1.0955
[8.314 x 10 kJ/(K • mol)]  650 K 600 K 
_3

k3 = e1.0955 = 3.0 M-1 s-1


1 1 1  1 1 
k3 t = _ ; t =  _ 
[ NO2 ]t [ NO2 ]o k 3  [ NO2 ]t [ NO2 ]o 

320
Chapter 12 - Chemical Kinetics
_____________________________________________________________________________

1  1 1 
t= _ = 2.7 x 102 s
(3.0 M s )  (0.0010 M) (0.0050 M) 

_1 _1

12.121 Rate = k [A]x[B]y


Comparing Experiments 1 and 2, the concentration of B does not change, the
concentration of A doubles, and the rate doubles. This means the reaction is first-order
in A (x = 1).
Comparing Experiments 1 and 3, the rate would drop to 0.9 x 10-5 M/s as a result of the
concentration of A being cut in half. Then with the the concentration of B doubling, the
rate increases by a factor of 4, to 3.6 x 10-5 M/s. This means the reaction is second-
order in B (y = 2).
Rate 4.3 x 10_ 5 M/s
At 600 K, k1 = = = 3.4 x 10-4 M-2 s-1
[A][B ]2 (0.50 M)(0.50 M )2
Rate 1.8 x 10_ 5 M/s
At 700 K, k2 = 2
= 2
= 9.0 x 10-3 M-2 s-1
[A][B ] (0.20 M)(0.10 M )
   _  1 1
ln  k 2  =  Ea  _ 
 k1   R  T 2 T1 
[ln k 2 _ ln k1](R)
Ea = _
 1 1
 _ 
 T 2 T1 
[ln (9.0 x 10_ 3) _ ln (3.4 x 10_ 4)][8.314 x 10_ 3 kJ/(Kcdot mol)]
Ea = _ = 114 kJ/mol
 1 1 
 _ 
 700 K 600 K 

12.122 A → C is a first-order reaction.


The reaction is complete at 200 s when the absorbance of C reaches 1.200.
Because there is a one to one stoichiometry between A and C, the concentration of A
must be proportional to 1.200 - absorbance of C. Any two data points can be used to
find k. Let [A]o ∝ 1.200 and at 100 s, [A]t ∝ 1.200 - 1.188 = 0.012
[A ]t  0.012 M 
ln ln  
ln
[A ]t
= - kt; k=
[A ]o
; k=  1.200 M 
= 0.0461 s-1
[A ]o _t _ 100 s
0.693 0.693
t1/ 2 = = = 15 s
k 0.0461 s _ 1

12.123 Rate = k [HI]x


x
Rate2 k [0.30 ]
= x
Rate1 k [0.10 ]

321
Chapter 12 - Chemical Kinetics
_____________________________________________________________________________

1.6 x 10_ 4
log Rate2 log
Rate1 = 1.8 x 10_ 5 0.949
x= = =2
(0.30) (0.30) 0.477
log log
(0.10) (0.10)
2
Rate = k [HI]
Rate 1.8 x 10_ 5 M/s
At 700 K, k1 = = = 1.8 x 10-3 M-1 s-1
[HI ]2 (0.10 M )2
Rate 3.9 x 10_ 3 M/s
At 800 K, k2 = 2
= 2
= 9.7 x 10-2 M-1 s-1
[HI ] (0.20 M )
   _  1 1
ln  k 2  =  Ea  _ 
 k1   R  T 2 T1 
[ln k 2 _ ln k1](R)
Ea = _
 1 1
 _ 
 T 2 T1 
[ln (9.7 x 10_ 2) _ ln (1.8 x 10_ 3)][8.314 x 10_ 3 kJ/(K • mol)]
Ea = _ = 186 kJ/mol
 1 1 
 _ 
 800 K 700 K 
Calculate k4 at 650 K using Ea and k1.
Solve for k4.
_  1 1
ln k4 = Ea  _  + ln k1
R  T 4 T1 
_ 186 kJ/mol  1 1  -3
ln k4 =  _  + ln(1.8 x 10 ) = -8.788
[8.314 x 10 kJ/(K • mol)]  650 K 700 K 
_3

k4 = e-8.788 = 1.5 x 10-4 M-1 s-1

Rate 1.0 x 10_ 5 M/s


[HI] = = = 0.26 M
k4 1.5 x 10_ 4 M _1 s _1

N
12.124 For radioactive decay, ln = - kt
No
0.693 0.693
For 235U, k1 = = 8
= 9.76 x 10-10 y-1
t1/ 2 7.1 x 10 y
0.693 0.693
For 238U, k2 = = 9
= 1.54 x 10-10 y-1
t1/ 2 4.51 x 10 y
For 235U, ln N1 = - k1t and ln N1 + k1t = 0
No 1 No 1
For 238U, ln N 2 = - k2t and ln N 2 + k2t = 0
No 2 No 2

322
Chapter 12 - Chemical Kinetics
_____________________________________________________________________________

Set the two equations that are equal to zero equal to each other and solve for t.
N N2
ln 1 + k1t = ln + k2t
No 1 No 2
ln N1 - ln N 2 = k2t - k1t = (k2 - k1)t
No 1 No 2
 N1 
 
ln  No1 
= (k2 - k1)t, now No1 = No2, so ln N1 = (k2 - k1)t
 N2  N2
 
 No 2 
N1 = 7.25 x 10-3, so ln(7.25 x 10-3) = (1.54 x 10-10 y-1 - 9.76 x 10-10 y-1)t
N2
_ 4.93
t= _ 10 _1
= 6.0 x 109 y
_ 8.22 x 10 y
The age of the elements is 6.0 x 109 y (6 billion years).

Multi-Concept Problems

Ea 6.3 kJ/mol
12.125 (a) k = A e _ RT = (6.0 x 108/(M ⋅ s)) e _ [8.314 x 10_ 3 kJ/(K • mol)](298 K) = 4.7 x 107/(M ⋅ s)

(b) N has 3 electron clouds, is sp2 hybridized, and the molecule is bent.

(c)

(d) The reaction has such a low activation energy because the F–F bond is very weak
and the N–F bond is relatively strong.

12.126 2 HI(g) → H2(g) + I2(g)


1000 mL 0.0101 g
(a) mass HI = 1.50 L x x = 15.15 g HI
1L 1 mL
1 mol HI
15.15 g HI x = 0.118 mol HI
127.91 g HI
0.118 mol
[HI] = = 0.0787 mol/L
1.50 L
∆ [HI]
_ = k [HI ]2 = (0.031/(M ⋅ min))(0.0787 M)2 = 1.92 x 10-4 M/min
∆t
2 HI(g) → H2(g) + I2(g)

∆ [ I2] 1  ∆ [HI]  1.92 x 10_ 4 M/min


= _  = = 9.60 x 10-5 M/min
∆t 2 ∆t  2
(9.60 x 10-5 M/min)(1.50 L)(6.022 x 1023 molecules/mol) = 8.7 x 1019 molecules/min

323
Chapter 12 - Chemical Kinetics
_____________________________________________________________________________

(b) Rate = k[HI]2


1 1  60.0 min  1
= kt + = (0.031/(M ⋅ min))  8.00 h x + =
[HI ]t [HI ]o  1 h  0.0787 M
27.59/M
1
[HI]t = = 0.0362 M
27.59 /M
From stoichiometry, [H2]t = 1/2 ([HI]o - [HI]t) = 1/2 (0.0787 M - 0.0362 M) = 0.0212 M
410oC = 683 K
PV = nRT
n  L• atm 
PH2 =   RT = (0.0212 mol/L) 0.082 06 (683 K) = 1.2 atm
V  K • mol 

12.127 2 NO2(g) → 2 NO(g) + O2(g)


k = 4.7/(M ⋅ s)
(a) The units for k indicate a second-order reaction.
(b) 383oC = 656 K
PV = nRT
 1.000 atm 
 746 mm Hg x 
n P  760 mm Hg 
[NO2]o = = = = 0.01823 mol/L
V RT  L • atm 
 0.082 06 (656 K)
 K • mol 
initial rate = k [ NO2 ]o2 = [4.7/(M ⋅ s)](0.01823 mol/L)2 = 1.56 x 10-3 mol/(L ⋅ s)
initial rate for NO2 1.56 x 10_ 3 mol/(L • s)
initial rate for O2 = = = 7.80 x 10-4 mol/(L ⋅
2 2
s)
initial rate for O2 = [7.80 x 10-4 mol/(L ⋅ s)](32.00 g/mol) = 0.025 g/(L ⋅ s)

1 1 1
(c) = kt + = [4.7 /(M • s)](60 s) +
[ NO 2 ]t [ NO 2 ]0 0.01823 M
1
= 336.9/M and [NO2] = 0.00297 M
[ NO2 ]t

2 NO2(g) → 2 NO(g) + O2(g)


before reaction (M) 0.01823 0 0
change (M) -2x +2x +x
after 1.00 min (M) 0.01823 - 2x 2x x

after 1.00 min [NO2] = 0.00297 M = 0.01823 - 2x


x = 0.00763 M = [O2]
mass O2 = (0.00763 mol/L)(5.00 L)(32.00 g/mol) = 1.22 g O2

324
Chapter 12 - Chemical Kinetics
_____________________________________________________________________________

12.128 (a) N2O5, 108.01 amu


 1 mol N 2 O5 
 2.70 g N 2 O5 x 
 108.01 g N 2 O5 
[N2O5]o = = 0.0125 mol/L
2.00 L
 60.0 s 
ln [N2O5]t = -kt + ln [N2O5]o = -(1.7 x 10-3 s-1) 13.0 min x  + ln (0.0125) = -5.71
 1 min 
[N2O5]t = e-5.71 = 3.31 x 10-3 mol/L
After 13.0 min, mol N2O5 = (3.31 x 10-3 mol/L)(2.00 L) = 6.62 x 10-3 mol N2O5

N2O5(g) → 2 NO2(g) + 1/2 O2(g)


before reaction (mol) 0.0250 0 0
change (mol) -x +2x +1/2x
after reaction (mol) 0.0250 - x 2x 1/2x

After 13.0 min, mol N2O5 = 6.62 x 10-3 = 0.0250 - x


x = 0.0184 mol

After 13.0 min, ntotal = n N2O5 + n NO2 + n O2 = (6.62 x 10-3) + 2(0.0184) + 1/2(0.0184)
ntotal = 0.0526 mol
55oC = 328 K
PV = nRT
 L• atm 
(0.0526 mol) 0.082 06 (328 K)
nRT  K • mol 
Ptotal = = = 0.71 atm
V 2.00 L

(b) N2O5(g) → 2 NO2(g) + 1/2 O2(g)


∆Horxn = 2 ∆Hof(NO2) - ∆Hof(N2O5)
∆Horxn = (2 mol)(33.2 kJ/mol) - (1 mol)(11 kJ/mol) = 55.4 kJ = 5.54 x 104 J
initial rate = k[N2O5]o = (1.7 x 10-3 s-1)(0.0125 mol/L) = 2.125 x 10-5 mol/(L ⋅ s)
initial rate absorbing heat = [2.125 x 10-5 mol/(L ⋅ s)](2.00 L)( 5.54 x 104 J/mol) = 2.4 J/s
(c)
 60.0 s 
ln [N2O5]t = -kt + ln [N2O5]o = -(1.7 x 10-3 s-1) 10.0 min x  + ln (0.0125) = -5.40
 1 min 
[N2O5]t = e-5.40 = 4.52 x 10-3 mol/L
After 10.0 min, mol N2O5 = (4.52 x 10-3 mol/L)(2.00 L) = 9.03 x 10-3 mol N2O5

N2O5(g) → 2 NO2(g) + 1/2 O2(g)


before reaction (mol) 0.0250 0 0
change (mol) -x +2x +1/2x
after reaction (mol) 0.0250 - x 2x 1/2x

After 10.0 min, mol N2O5 = 9.03 x 10-3 = 0.0250 - x


x = 0.0160 mol

325
Chapter 12 - Chemical Kinetics
_____________________________________________________________________________

heat absorbed = (0.0160 mol)(55.4 kJ/mol) = 0.89 kJ

12.129 2 N2O(g) → 2 N2(g) + O2(g)


PO2 (in exit gas) = 1.0 mm Hg; Ptotal = 1.50 atm = 1140 mm Hg
From the reaction stoichiometry:
P N2 (in exit gas) = 2 PO2 = 2.0 mm Hg
P N2 O (in exit gas) = Ptotal - P N2 - PO2 = 1140 - 2.0 - 1.0 = 1137 mm Hg
Assume P N2 O (initial) = Ptotal = 1140 mm Hg (In assuming a constant total pressure in the
tube, we are neglecting the slight change in pressure due to the reaction.)
Volume of tube = πr2l = π(1.25 cm)2(20 cm) = 98.2 cm3 = 0.0982 L
volume of tube 0.0982 L 60 s
Time, t, gases are in the tube = x x = 7.86 s
flow rate 0.75 L/min 1 min
[ N 2 O ]t P N2 O (in exit gas) 1137 mm Hg
At time t, = = = 0.997 37
[ N 2 O ]0 P N2 O (initial) 1140 mm Hg

Ea
Because k = A e _ RT and A = 4.2 x 109 s-1, k has units of s-1. Therefore, this is a first-order
[N O ]
reaction and the appropriate integrated rate law is ln 2 t = _ kt .
[ N 2 O ]0
[N O ]
_ ln 2 t
[ N 2 O ]0 _ ln (0.997 37)
k= = = 3.35 x 10-4 s-1
t 7.86 s
From the Arrhenius equation, ln k = ln A - Ea
RT
Ea 222 kJ/mol
T= = = 885 K
(R)[ln A _ ln k] (8.314 x 10 kJ/(K • mol))[(22.16) _ (_ 8.00)]
_3

12.130 H2O2, 34.01 amu


mass H2O2 = (0.500 L)(1000 mL/1 L)(1.00 g/ 1 mL)(0.0300) = 15.0 g H2O2
1 mol H2 O2
mol H2O2 = 15.0 g H2O2 x = 0.441 H2O2
34.01 g H 2 O2
0.441 mol
[H2O2]o = = 0.882 mol /L
0.500 L
0.693 0.693
k= = = 6.48 x 10-2/h
t1/ 2 10.7 h
ln [H2O2]t = -kt + ln [H2O2]o
ln [H2O2]t = -(6.48 x 10-2/h)(4.02 h) + ln (0.882)
ln [H2O2]t = -0.386; [H2O2]t = e -0.386 = 0.680 mol/L

326
Chapter 12 - Chemical Kinetics
_____________________________________________________________________________

mol H2O2 = (0.680 mol/L)(0.500 L) = 0.340 mol

2 H2O2(aq) → 2 H2O(l) + O2(g)


before reaction (mol) 0.441 0 0
change (mol) - 2x +2x +x
after reaction (mol) 0.441 - 2x 2x x

After 4.02 h, mol H2O2 = 0.340 mol = 0.441 - 2x; solve for x.
2x = 0.101
x = 0.0505 mol = mol O2

1.00 atm
P = 738 mm Hg x = 0.971 atm
760 mm Hg
PV = nRT
 L• atm 
(0.0505 mol) 0.082 06 (293 K)
nRT  K • mol 
V= = = 1.25 L
P 0.971 atm
P∆V = (0.971 atm)(1.25 L) = 1.21 L ⋅ atm

 J 
w = -P∆V = -1.21 L⋅atm = (-1.21 L ⋅ atm) 101  = -122 J
 L• atm 

12.131 (a) CH3CHO(g) → CH4(g) + CO(g)


before (atm) 0.500 0 0
change (atm) -x +x +x
after (atm) 0.500 - x x x

At 605 s, Ptotal = PCH3 CHO + PCH4 + PCO = (0.500 atm - x) + x + x = 0.808 atm
x = 0.808 atm - 0.500 atm = 0.308 atm

The integrated rate law for a second-order reaction in terms of molar concentrations is
1 1
= kt + . The ideal gas law, PV = nRT, can be rearranged to show how P is
[A ]t [A ]o
proportional the the molar concentration of a gas.
n n
P = RT (R and T are constant), so P ∝ = molar concentration
V V
Because of this relationship, the second-order integrated rate law can be rewritten in
terms of partial pressures.
1 1
 _ 
1
= kt +
1
;
1 1
_ = kt ;  P t Po  = k
Pt Po P t Po t

327
Chapter 12 - Chemical Kinetics
_____________________________________________________________________________

P is the partial pressure of CH3CHO.

At t = 0, Po = 0.500 and at t = 605 s, Pt = 0.500 atm - 0.308 atm = 0.192 atm


 1 1 
 _ 
k=  0.192 atm 0.500 atm  = 5.30 x 10-3 atm-1 s-1
605 s

(b) Use the ideal gas law to convert atm-1 to M-1.


n P n 1 V
P= RT ; = ; RT = = M _1
V RT V P n
So, multiply k by RT to convert atm-1 s-1 to M-1 s-1.
k = (5.30 x 10-3 atm-1 s-1)RT
 L • atm  L
k = (5.30 x 10-3 atm-1 s-1)  0.082 06 (791 K) = 0.344 = 0.344 M-1
 K • mol  mol • s
s-1

(c) CH3CHO(g) → CH4(g) + CO(g)


∆Horxn = [∆Hof(CH4) + ∆Hof(CO)] - ∆Hof(CH3CHO)]
∆Horxn = [(1 mol)(-74.8 kJ/mol) + (1 mol)(-110.5 kJ/mol)] - (1 mol)(-166.2 kJ/mol)
∆Horxn = -19.1 kJ per mole of CH3CHO that decomposes

PV = nRT
PV (0.308 atm)(1.00 L)
mol CH3CHO reacted = = = 0.004 74 mol
RT  L • atm 
 0.082 06 (791 K)
 K • mol 
q = (0.004 74 mol)(19.1 kJ/mol)(1000 J/kJ) = 90.6 J liberated after a reaction time of 605 s

328
13 Chemical Equilibrium

[SO3 ]2 [SO2 ]2 [O2]


13.1 (a) Kc = (b) Kc =
[SO2 ]2 [O2] [SO3 ]2

[SO3 ]2 (5.0 x 10_ 2 )2


13.2 (a) Kc = 2
= _3 2 _3
= 7.9 x 104
[SO2 ] [O2] (3.0 x 10 ) (3.5 x 10 )
[SO2 ]2 [O2] (3.0 x 10_ 3 )2 (3.5 x 10_ 3)
(b) Kc = = = 1.3 x 10-5
[SO3 ]2 (5.0 x 10_ 2 )2

_
[ + ][ ]
13.3 (a) K c = H C3 H5 O3
[C3 H6 O3]
[(0.100)(0.0365) ]2
(b) K c = = 1.38 x 10-4
[0.100 _ (0.100)(0.0365)]

[AB][B] (1)(2)
13.4 From (1), K c = = = 1
[A][B2] (1)(2)
[AB][B]
For a mixture to be at equilibrium, must be equal to 1.
[A][B2]
[AB][B] (2)(1)
For (2), = = 1. This mixture is at equilibrium.
[A][B2] (2)(1)
[AB][B] (1)(1)
For (3), = = 0.125. This mixture is not at equilibrium.
[A][B2] (4)(2)
[AB][B] (2)( 2)
For (4), = = 1.0 This mixture is at equilibrium.
[A][B 2] (4)(1)

(PCO2)(PH2) (6.12)(20.3)
13.5 Kp = = = 9.48
(PCO )(PH2 O ) (1.31)(10.0)

13.6 2 NO(g) + O2 _ 2 NO2(g); ∆n = 2 - 3 = -1


Kp = Kc(RT)∆n, Kc = Kp(1/RT)∆n
at 500 K: Kp = (6.9 x 105)[(0.082 06)(500)]-1 = 1.7 x 104
_1
 1 
at 1000 K: Kc = (1.3 x 10 ) 
-2
 = 1.1
 (0.082 06)(1000) 
3
[ ]3 ( P H2 )
13.7 (a) Kc = H 2 3 , Kp = 3
, ∆n = (3) - (3) = 0 and Kp = Kc
[H 2 O ] ( P H2 O )

331
Chapter 13 - Chemical Equilibrium
______________________________________________________________________________

(b) Kc = [H2]2[O2], Kp = (P H2 )2 (PO2) , ∆n = (3) - (0) = 3 and Kp = Kc(RT)3


[HCl ]4 (P HCl )4
(c) Kc = , Kp = 2
, ∆n = (4) - (3) = 1 and Kp = Kc(RT)
[SiCl4][H 2 ]2 (PSiCl4)(P H2 )
1
(d) Kc = 2+
[Hg 2 ][Cl _ ]2

13.8 Kc = 1.2 x 10-42. Because Kc is very small, the equilibrium mixture contains mostly H2
molecules. H is in periodic group 1A. A very small value of Kc is consistent with strong
bonding between 2 H atoms, each with one valence electron.

13.9 The container volume of 5.0 L must be included to calculate molar concentrations.
[ NO2 ]2t (0.80 mol/ 5.0 L )2
(a) Qc = = = 890
[NO ]2t [O2 ]t (0.060 mol/ 5.0 L )2 (1.0 mol/ 5.0 L)
Because Qc < Kc, the reaction is not at equilibrium. The reaction will proceed to the
right to reach equilibrium.
[ NO2 ]2t (4.0 mol/ 5.0 L )2
(b) Qc = 2
= _3 2
= 1.6 x 107
[NO ]t [O2 ]t (5.0 x 10 mol/ 5.0 L ) (0.20 mol/ 5.0 L)
Because Qc > Kc, the reaction is not at equilibrium. The reaction will proceed to the left
to reach equilibrium.

[AB ]2 [AB ]2
13.10 Kc = = 4 ; For a mixture to be at equilibrium, must be equal to 4.
[A 2][B2] [A 2][B2]
[AB ]2 (6 )2
For (1), Qc = = = 36, Qc > Kc
[A 2][B2] (1)(1)
[AB ]2 (4 )2
For (2), Qc = = = 4, Qc = Kc
[A 2][B2] (2)(2)
[AB ]2 (2 )2
For (3), Qc = = = 0.44, Qc < Kc
[A 2][B2] (3)(3)
(a) (2) (b) (1), reverse; (3), forward

[H ]2
13.11 Kc = = 1.2 x 10-42
[ H 2]
_ 42 -22
(a) [H] = K c [H 2] = (1.2 x 10 )(0.10) = 3.5 x 10 M

(b) H atoms = (3.5 x 10-22 mol/L)(1.0 L)(6.022 x 1023 atoms/mol) = 210 H atoms
H2 molecules = (0.10 mol/L)(1.0 L)(6.022 x 1023 molecules/mol) = 6.0 x 1022 H2 molecules

13.12 CO(g) + H2O(g) _ CO2(g) + H2(g)


initial (M) 0.150 0.150 0 0
change (M) -x -x +x +x

332
Chapter 13 - Chemical Equilibrium
______________________________________________________________________________

equil (M) 0.150 - x 0.150 - x x x


2
[CO2][H2] x
Kc = 4.24 = =
[CO][H 2 O] (0.150 _ x )2
Take the square root of both sides and solve for x.
2
x x
4.24 = 2
; 2.06 = ; x = 0.101
(0.150 _ x ) 0.150 _ x
At equilibrium, [CO2] = [H2] = x = 0.101 M
[CO] = [H2O] = 0.150 - x = 0.150 - 0.101 = 0.049 M

13.13 N2O4(g) _ 2 NO2(g)


initial (M) 0.0500 0
change (M) -x +2x
equil (M) 0.0500 - x 2x
-3 [ NO2 ] 2
(2 x )2
Kc = 4.64 x 10 = =
[ N 2 O4] (0.0500 _ x)
4x2 + (4.64 x 10-3)x - (2.32 x 10-4) = 0
Use the quadratic formula to solve for x.
_ (4.64 x 10_ 3) ± (4.64 x 10_ 3 ) _ 4(4)(_ 2.32 x 10_ 4) _ 0.00464 ± 0.06110
2

x= =
2(4) 8
x = -0.008 22 and 0.007 06
Discard the negative solution (-0.008 22) because it leads to a negative concentration of
NO2 and that is impossible.
[N2O4] = 0.0500 - x = 0.0500 - 0.007 06 = 0.0429 M
[NO2] = 2x = 2(0.007 06) = 0.0141 M

13.14 N2O4(g) _ 2 NO2(g)


[ NO2 ]2t (0.0300 mol/L )2
Qc = = = 0.0450; Qc > Kc
[ N 2 O4 ]t (0.0200 mol/L)
The reaction will approach equilibrium by going from right to left.
N2O4(g) _ 2 NO2(g)
initial (M) 0.0200 0.0300
change (M) +x -2x
equil (M) 0.0200 + x 0.0300 - 2x
2 2
[ ] (0.0300 _ 2 x )
Kc = 4.64 x 10-3 = NO2 =
[ N 2 O4] (0.0200 + x)
2 -4
4x - 0.1246x + (8.072 x 10 ) = 0
Use the quadratic formula to solve for x.
_ (_ 0.1246) ± (_ 0.1246 )2 _ 4(4)(8.072 x 10_ 4) 0.1246 ± 0.05109
x= =
2(4) 8
x = 0.0220 and 0.009 19
Discard the larger solution (0.0220) because it leads to a negative concentration of NO2,

333
Chapter 13 - Chemical Equilibrium
______________________________________________________________________________

and that is impossible.


[N2O4] = 0.0200 + x = 0.0200 + 0.009 19 = 0.0292 M
[NO2] = 0.0300 - 2x = 0.0300 - 2(0.009 19) = 0.0116 M

(PCO )(PH2) (1.00)(1.40)


13.15 Kp = = 2.44, Qp = = 1.17, Qp < Kp and the reaction goes to the
(P H2 O ) (1.20)
right to reach equilibrium.
C(s) + H2O(g) _ CO(g) + H2(g)
initial (atm) 1.20 1.00 1.40
change (atm) -x +x +x
equil (atm) 1.20 - x 1.00 + x 1.40 + x
(PCO )(P H2) (1.00 + x)(1.40 + x)
Kp = = 2.44 =
( P H2 O ) (1.20 _ x)
2
x + 4.84x - 1.53 = 0
Use the quadratic formula to solve for x.
_ (4.84) ± (4.84 )2 _ 4(1)(_ 1.53) _ 4.84 ± 5.44
x= =
2(1) 2
x = -5.14 and 0.300
Discard the negative solution (-5.14) because it leads to negative partial pressures and
that is impossible.
PH2 O = 1.20 - x = 1.20 - 0.300 = 0.90 atm
PCO = 1.00 + x = 1.00 + 0.300 = 1.30 atm
PH2 = 1.40 + x = 1.40 + 0.300 = 1.70 atm

13.16 (a) CO(reactant) added, H2 concentration increases.


(b) CO2 (product) added, H2 concentration decreases.
(c) H2O (reactant) removed, H2 concentration decreases.
(d) CO2 (product) removed, H2 concentration increases.
[ ][ ]
At equilibrium, Qc = Kc = CO2 H 2 . If some CO2 is removed from the
[CO][H 2 O]
equilibrium mixture, the numerator in Qc is decreased, which means that
Qc < Kc and the reaction will shift to the right, increasing the H2 concentration.
13.17 (a) Because there are 2 mol of gas on both sides of the balanced equation, the
composition of the equilibrium mixture is unaffected by a change in pressure. The
number of moles of reaction products remains the same.
(b) Because there are 2 mol of gas on the left side and 1 mol of gas on the right side of
the balanced equation, the stress of an increase in pressure is relieved by a shift in the
reaction to the side with fewer moles of gas (in this case, to products). The number of
moles of reaction products increases.
(c) Because there is 1 mol of gas on the left side and 2 mol of gas on the right side of the
balanced equation, the stress of an increase in pressure is relieved by a shift in the
reaction to the side with fewer moles of gas (in this case, to reactants). The number of
moles of reaction product decreases.

334
Chapter 13 - Chemical Equilibrium
______________________________________________________________________________

13.18

13.19 Le Châtelier’s principle predicts that a stress of added heat will be relieved by net
reaction in the direction that absorbs the heat. Since the reaction is endothermic, the
equilibrium will shift from left to right (Kc will increase) with an increase in
temperature. Therefore, the equilibrium mixture will contain more of the offending NO,
the higher the temperature.

13.20 The reaction is exothermic. As the temperature is increased the reaction shifts from right
to left. The amount of ethyl acetate decreases.
[ ][ O]
Kc = CH3 CO2 C2 H5 H 2
[CH3 CO2 H][C2 H5 OH]
As the temperature is decreased, the reaction shifts from left to right. The product
concentrations increase, and the reactant concentrations decrease. This corresponds to
an increase in Kc.

13.21 There are more AB(g) molecules at the higher temperature. The equilibrium shifted to
the right at the higher temperature, which means the reaction is endothermic.

13.22 (a) A catalyst does not affect the equilibrium composition. The amount of CO remains
the same.
(b) The reaction is exothermic. An increase in temperature shifts the reaction toward
reactants. The amount of CO increases.
(c) Because there are 3 mol of gas on the left side and 2 mol of gas on the right side of
the balanced equation, the stress of an increase in pressure is relieved by a shift in the
reaction to the side with fewer moles of gas (in this case, to products). The amount of
CO decreases.
(d) An increase in pressure as a result of the addition of an inert gas (with no volume
change) does not affect the equilibrium composition. The amount of CO remains the same.
(e) Adding O2 increases the O2 concentration and shifts the reaction toward products.
The amount of CO decreases.

13.23 (a) Because Kc is so large, kf is larger than kr.


8.5 x 106 M _1 s _1
(b) Kc = k f ; kr = k f = 34
= 2.5 x 10-28 M-1 s-1
kr Kc 3.4 x 10
(c) Because the reaction is exothermic, Ea (forward) is less than Ea (reverse).
Consequently, as the temperature decreases, kr decreases more than kf decreases, and

335
Chapter 13 - Chemical Equilibrium
______________________________________________________________________________

therefore Kc = k f increases.
kr

13.24 Hb + O2 _ Hb(O2)
If CO binds to Hb, Hb is removed from the reaction and the reaction will shift to the left
resulting in O2 being released from Hb(O2). This will decrease the effectiveness of Hb
for carrying O2.

13.25 The equilibrium shifts to the left because at the higher altitude the concentration of O2 is
decreased.

13.26 There are 26 π electrons.

13.27 The partial pressure of O2 in the atmosphere is 0.2095 atm.


PV = nRT
PV (0.2095 atm)(0.500 L)
n= = = 4.28 x 10-3 mol O2
RT  L • atm 
 0.082 06 (298 K)
 K • mol 
6.022 x 1023 O2 molecules
4.28 x 10-3 mol O2 x = 2.58 x 1021 O2 molecules
1 mol O2
Understanding Key Concepts

13.28 (a) (1) and (3) because the number of A and B's are the same in the third and fourth
box.
[B] 6
(b) Kc = = = 1.5
[A] 4
(c) Because the same number of molecules appear on both sides of the equation, the
volume terms in Kc all cancel. Therefore, we can calculate Kc without including the
volume.

13.29 (a) A2 + C2 _ 2 AC (most product molecules)


(b) A2 + B2 _ 2 AB (fewest product molecules)

[A][AB] (2)(4)
13.30 (a) Only reaction (3), K c = = = 2, is at equilibrium.
[A2][B] (2)(2)
[A][AB] (3)(5)
(b) Qc = = = 15 for reaction (1). Because Qc > Kc, the reaction will
[A 2][B] (1)(1)
go in the reverse direction to reach equilibrium.
[A][AB] (1)(3)
Qc = = = 1/3 for reaction (2). Because Qc < Kc, the reaction will go
[A 2][B] (3)(3)
in the forward direction to reach equilibrium.

13.31 (a) A2 + 2 B _ 2 AB

336
Chapter 13 - Chemical Equilibrium
______________________________________________________________________________

(b) The number of AB molecules will increase, because as the volume is decreased at
constant temperature, the pressure will increase and the reaction will shift to the side of
fewer molecules to reduce the pressure.

13.32 When the stopcock is opened, the reaction will go in the reverse direction because there
will be initially an excess of AB molecules.

13.33 As the temperature is raised, the reaction proceeds in the reverse direction. This is
consistent with an exothermic reaction where "heat" can be considered as a product.

13.34 (a) AB → A + B
(b) The reaction is endothermic because a stress of added heat (higher temperature)
shifts the AB _ A + B equilibrium to the right.
(c) If the volume is increased, the pressure is decreased. The stress of decreased
pressure will be relieved by a shift in the equilibrium from left to right, thus increasing
the number of A atoms.

13.35 Heat + BaCO3(s) _ BaO(s) + CO2(g)

(a) (b)

13.36 (a) (b) (c)

(2)(2)
13.37 This equilibrium mixture has a Kc ∝ and is less than 1. This means that kf < kr.
(3 )2
Additional Problems
Equilibrium Expressions and Equilibrium Constants

[CO][H 2 ]3 [ClF3 ]2 [HF ]2


13.38 (a) K c = (b) K c = (c) Kc =
[CH 4][H 2 O] [F2 ]3 [Cl2] [H 2][F2]

[ CHO ]2 [ ][ ] [NO ]4 [H 2 O ]6
13.39 (a) K c = CH3 2 (b) K c = N 2 O22 (c) K c =
[C 2 H 4 ] [O 2 ] [NO ] [ NH3 ]4 [O2 ]5

337
Chapter 13 - Chemical Equilibrium
______________________________________________________________________________

(PCO )(P H2 )3
13.40 (a) Kp = , ∆n = 2 and Kp = Kc(RT)2
(PCH4)(P H2 O )
2
(PClF3 )
(b) Kp = 3
, ∆n = -2 and Kp = Kc(RT)-2
(PF2 ) (PCl2)
(P HF )2
(c) Kp = , ∆n = 0 and Kp = Kc
(P H2)(P F2)

2
(PCH3 CHO )
13.41 (a) Kp = 2
, ∆n = -1 and Kp = Kc(RT)-1
( P C2 H 4 ) ( P O 2 )
(P N2)(PO2)
(b) Kp = , ∆n = 0 and Kp = Kc
(P NO )2
4 6
(P NO ) (PH2 O )
(c) Kp = 4 5
, ∆n = 1 and Kp = Kc(RT)
(P NH3 ) (PO2 )

[C2 H5 OC2 H5][H 2 O]


13.42 Kc =
[C2 H5 OH ]2

[ OH]
13.43 Kc = HOCH 2 CH 2
[C2 H 4 O][H2 O]

[Isocitrate]
13.44 Kc =
[Citrate]

[citric acid]
13.45 Kc =
[acetic acid][oxaloacetic acid]

13.46 The two reactions are the reverse of each other.


1 1
Kc(reverse) = = _9
= 1.3 x 108
K c (forward) 7.5 x 10

13.47 The two reactions are the reverse of each other.


1 1
Kp(reverse) = = = 1.99 x 10-2
K p (forward) 50.2

[ ][ ] (1.5 x 10_ 2)(3.2 x 10_ 2)


13.48 Kc = PCl3 Cl2 = = 0.058
[PCl5] (8.3 x 10_ 3)

(PClNO )2 (1.35 )2
13.49 Kp = 2
= 2
= 52.0
(P NO ) (PCl2) (0.240 ) (0.608)

338
Chapter 13 - Chemical Equilibrium
______________________________________________________________________________

13.50 The container volume of 2.00 L must be included to calculate molar concentrations.
Initial [HI] = 9.30 x 10-3 mol/2.00 L = 4.65 x 10-3 M = 0.004 65 M
H2(g) + I2(g) _ 2 HI(g)
initial (M) 0 0 0.004 65
change (M) +x +x -2x
equil (M) x x 0.004 65 - 2x
x = [H2] = [I2] = 6.29 x 10-4 M = 0.000 629 M
[HI] = 0.004 65 - 2x = 0.004 65 - 2(0.000 629) = 0.003 39 M
[HI ]2 (0.003 39 )2
Kc = = = 29.0
[H 2][I2] (0.000 629 )2

_
[ + ][ ]
13.51 (a) Kc = H CH3 CO2
[CH3 CO 2 H]
(b) CH3CO2H(aq) _ H+(aq) + CH3CO2-(aq)
initial (M) 1.0 0 0
change (M) -0.0042 +0.0042 +0.0042
equil (M) 1.0 - 0.0042 0.0042 0.0042
(0.0042)(0.0042)
Kc = = 1.8 x 10-5
(1.0 _ 0.0042)

[ ][ O]
13.52 (a) K c = CH3 CO 2 C2 H5 H 2
[CH3 CO 2 H][C2 H5 OH]
(b) CH3CO2H(soln) + C2H5OH(soln) _ CH3CO2C2H5(soln) + H2O(soln)
initial (mol) 1.00 1.00 0 0
change (mol) -x -x +x +x
equil (mol) 1.00 - x 1.00 - x x x
2
(0.65 )
x = 0.65 mol; 1.00 - x = 0.35 mol; Kc = = 3.4
(0.35 )2
Because there are the same number of molecules on both sides of the equation, the
volume terms in Kc cancel. Therefore, we can calculate Kc without including the
volume.

13.53 CH3CO2C2H5(soln) + H2O(soln) _ CH3CO2H(soln) + C2H5OH(soln)


1 1
Kc(hydrolysis) = = = 0.29
K c (forward) 3.4

13.54 ∆n = 1 and Kp = Kc(RT) = (0.575)(0.082 06)(500) = 23.6

13.55 2 SO2(g) + O2(g) _ 2 SO3(g); ∆n = 2 - (2 + 1) = -1 and Kp = 3.30


∆n _1
 1   1 
Kc = Kp   = (3.30)   = 271
 RT   (0.082 06)(1000) 

339
Chapter 13 - Chemical Equilibrium
______________________________________________________________________________

13.56 Kp = PH2 O = 0.0313 atm; ∆n = 1


 1   1 
Kc = Kp   = (0.0313)  = 1.28 x 10-3
 RT   (0.082 06)(298) 

1 atm
13.57 PC10H8 = 0.10 mm Hg x = 1.3 x 10-4 atm
760 mm Hg
-4
Kp = PC10H8 = 1.3 x 10 ; ∆n = 1 - 0 = 1, T = 27oC = 300 K
∆n
 1   1 
Kc = Kp   = (1.3 x 10_ 4)  = 5.3 x 10-6
 RT   (0.082 06)(300) 

[CO2 ]3 (PCO2 )3 1 1
13.58 (a) K c = , Kp = (b) K c = , Kp = 3
[CO ]3 (PCO ) 3
[O2 ] 3
( P O2 )
(c) Kc = [SO3], K p = PSO3 (d) Kc = [Ba2+][SO42-]

[H 2 O ]3 ( P H 2 O )3 1
13.59 (a) Kc = , Kp = 3
(b) Kc = +
[H 2 ]3 (P H2 ) [Ag ][Cl _ ]
6
[HCl ] (PHCl )6
(c) Kc = , Kp = 3
(d) Kc = [CO2], Kp = PCO2
[H 2 O ]3 ( P H2 O )

Using the Equilibrium Constant

13.60 (a) Because Kc is very large, the equilibrium mixture contains mostly product.
(b) Because Kc is very small, the equilibrium mixture contains mostly reactants.
13.61 (a) proceeds hardly at all toward completion
(b) goes almost all the way to completion

13.62 (a) Because Kc is very small, the equilibrium mixture contains mostly reactant.
(b) Because Kc is very large, the equilibrium mixture contains mostly product.
(c) Because Kc = 1.8, the equilibrium mixture contains an appreciable concentration of
both reactants and products.

13.63 (a) Because Kc is very large, the equilibrium mixture contains mostly product.
(b) Because Kc = 7.5 x 10-3, the equilibrium mixture contains an appreciable
concentration of both reactants and products.
(c) Because Kc is very small, the equilibrium mixture contains mostly reactant.

13.64 Kc = 1.2 x 1082 is very large. When equilibrium is reached, very little if any ethanol will
remain because the reaction goes to completion.

13.65 Because Kc is very small, pure air will contain very little O3 (ozone) at equilibrium.

340
Chapter 13 - Chemical Equilibrium
______________________________________________________________________________

[ ]2
3 O2(g) _ 2 O3(g); Kc = O3 3 = 1.7 x 10-56; [O2] = 8 x 10-3 M
[O2 ]
[O3] = [O2 ]3 x K c = (8 x 10_ 3 )3 (1.7 x 10_ 56) = 9 x 10-32 M

13.66 The container volume of 10.0 L must be included to calculate molar concentrations.
[CS2 ]t [H 2 ]4t (3.0 mol/ 10.0 L)(3.0 mol/10.0 L )4
Qc = 2
= 2
= 7.6 x 10-2; Kc = 2.5 x 10-3
[CH 4 ]t [H 2 S ]t (2.0 mol/ 10.0 L)(4.0 mol/10.0 L )
The reaction is not at equilibrium because Qc > Kc. The reaction will proceed from right
to left to reach equilibrium.

[CO ]t [H 2 ]3t (0.15)(0.20 )


3
13.67 Qc = = = 0.69; Kc = 4.7
[H 2 O ]t [CH 4 ]t (0.035)(0.050)
The reaction is not at equilibrium because Qc < Kc. The reaction will proceed from left
to right to reach equilibrium.

[ NH3 ]2
13.68 Kc = = 0.29; At equilibrium, [N2] = 0.036 M and [H2] = 0.15 M
[ N 2][H 2 ]3
[NH3] = [ N 2] x [H 2 ]3 x K c = (0.036)(0.15 )3 (0.29) = 5.9 x 10-3 M

2 [SO3 ]2 1
13.69 Kc = 2.7 x 10 = ; Because [SO3] = [SO2], then 2.7 x 102 =
[SO2 ]2 [O2] [ O2 ]
[O2] = 3.7 x 10-3 M

13.70 N2(g) + O2(g) _ 2 NO(g)


initial (M) 1.40 1.40 0
change (M) -x -x +2x
equil (M) 1.40 - x 1.40 - x 2x
2 2
[NO ] (2 x )
Kc = 1.7 x 10-3 = =
[ N 2][O2] (1.40 _ x )2
Take the square root of both sides and solve for x.
(2 x )2 2x
1.7 x 10_ 3 = 2
; 4.1 x 10-2 = ; x = 2.8 x 10-2
(1.40 _ x ) 1.40 _ x
At equilibrium, [NO] = 2x = 2(2.8 x 10-2) = 0.056 M
[N2] = [O2] = 1.40 - x = 1.40 - (2.8 x 10-2) = 1.37 M

13.71 N2(g) + O2(g) _ 2 NO(g)


initial (M) 2.24 0.56 0
change (M) -x -x +2x
equil (M) 2.24 - x 0.56 - x 2x

341
Chapter 13 - Chemical Equilibrium
______________________________________________________________________________

[NO ]2 (2 x )2
Kc = = 1.7 x 10-3 =
[ N 2][O2] (2.24 _ x)(0.56 _ x)
2 -3 -3
4x + (4.8 x 10 )x - (2.1 x 10 ) = 0
Use the quadratic formula to solve for x.
_ (4.8 x 10_ 3) ± (4.8 x 10_ 3 )2 _ 4(4)(_ 2.1 x 10_ 3) _ 0.0048 ± 0.1834
x= =
2(4) 8
x = -0.0235 and 0.0223
Discard the negative solution (-0.0235) because it gives a negative NO concentration and
that is impossible.
[N2] = 2.24 - x = 2.24 - 0.0223 = 2.22 M
[O2] = 0.56 - x = 0.56 - 0.0223 = 0.54 M; [NO] = 2x = 2(0.0223) = 0.045 M

13.72 PCl5(g) _ PCl3(g) + Cl2(g)


initial (M) 0.160 0 0
change (M) -x +x +x
equil (M) 0.160 - x x x
2
[PCl3][Cl2] x
Kc = = 5.8 x 10_ 2 =
[PCl5] 0.160 _ x
x2 + (5.8 x 10-2)x - 0.00928 = 0
Use the quadratic formula to solve for x.
(_ 5.8 x 10_ 2) ± (5.8 x 10_ 2 )2 _ 4(1)(_ 0.00928) (_ 5.8 x 10_ 2) ± 0.20
x= =
2(1) 2
x = 0.071 and -0.129
Discard the negative solution (-0.129) because it gives negative concentrations of PCl3
and Cl2 and that is impossible.
[PCl3] = [Cl2] = x = 0.071 M; [PCl5] = 0.160 - x = 0.160 - 0.071 = 0.089 M

(0.100)(0.040)
13.73 Qc = = 0.020, Qc < Kc therefore the reaction proceeds from reactants to
(0.200)
products to reach equilibrium.
PCl5(g) _ PCl3(g) + Cl2(g)
initial (M) 0.200 0.100 0.040
change (M) -x +x +x
equil (M) 0.200 - x 0.100 + x 0.040 + x
[ ][ ] (0.100 + x) (0.040 + x)
Kc = PCl3 Cl2 = 5.8 x 10_ 2 =
[PCl5] 0.200 _ x
2 -3
x + 0.198x - (7.60 x 10 ) = 0
Use the quadratic formula to solve for x.
(_ 0.198) ± (0.198 )2 _ 4(1)(_ 7.60 x 10_ 3) (_ 0.198) ± 0.264
x= =
2(1) 2
x = 0.033 and -0.231

342
Chapter 13 - Chemical Equilibrium
______________________________________________________________________________

Discard the negative solution (-0.231) because it gives negative concentrations of PCl3
and Cl2 and that is impossible.
[PCl3] = 0.100 + x = 0.100 + 0.033 = 0.133 M
[Cl2] = 0.040 + x = 0.040 + 0.033 = 0.073 M
[PCl5] = 0.200 - x = 0.200 - 0.033 = 0.167 M

[ ][ O] (x)(12.0)
13.74 (a) Kc = CH3 CO 2 C2 H5 H 2 = 3.4 = ; x = 6.8 moles CH3CO2C2H5
[CH3 CO 2 H][C2 H5 OH] (4.0)(6.0)
Note that the volume cancels because the same number of molecules appear on both
sides of the chemical equation.
(b) CH3CO2H(soln) + C2H5OH(soln) _ CH3CO2C2H5(soln) + H2O(soln)
initial (mol) 1.00 10.00 0 0
change (mol) -x -x +x +x
equil (mol) 1.00 - x 10.00 - x x x
2
Kc = 3.4 = x
(1.00 _ x)(10.00 _ x)
2.4x2 - 37.4x + 34 = 0
Use the quadratic formula to solve for x.
_ (_ 37.4) ± (_ 37.4 )2 _ 4(2.4)(34) 37.4 ± 32.75
x= =
2(2.4) 4.8
x = 0.969 and 14.6
Discard the larger solution (14.6) because it leads to negative concentrations and that is
impossible.
mol CH3CO2H = 1.00 - x = 1.00 - 0.969 = 0.03 mol
mol C2H5OH = 10.00 - x = 10.00 - 0.969 = 9.03 mol
mol CH3CO2C2H5 = mol H2O = x = 0.97 mol

13.75 When equal volumes of two solutions are mixed together, their concentrations are cut in half.
CH3Cl(aq) + OH-(aq) _ CH3OH(aq) + Cl-(aq)
initial (M) 0.05 0.1 0 0
assume complete reaction (M) 0 0.05 0.05 0.05
assume small back reaction (M) +x +x -x -x
equil (M) x 0.05 + x 0.05 - x 0.05 - x
_ 2
[CH3 OH][Cl ] (0.05 _ x )
Kc = _
= 1016 = ; Because Kc is very large, x << 0.05.
[CH3 Cl][OH ] x(0.05 + x)
(0.05 )2
10 ≈ x = 5 x 10-18
16
;
x(0.05)
[CH3Cl] = x = 5 x 10-18 M; [OH-] = [CH3OH] = [Cl-] ≈ 0.05 M

13.76 ClF3(g) _ ClF(g) + F2(g)


initial (atm) 1.47 0 0
change (atm) -x +x +x

343
Chapter 13 - Chemical Equilibrium
______________________________________________________________________________

equil (atm) 1.47 - x x x


(PClF)(PF2) (x)(x)
Kp = = 0.140 = ; solve for x.
(PClF3) 1.47 _ x
x2 + 0.140x - 0.2058 = 0
Use the quadratic formula to solve for x.
_ (0.140) ±
2
(0.140 ) _ (4)(1)(_ 0.2058)
x=
2(1)
_ 0.140 ± 0.918
x=
2
x = 0.389 and -0.529
Discard the negative solution (-0.529) because it gives negative partial pressures and that
is impossible.
PClF = P F2 = x = 0.389 atm
PClF3 = 1.47 - x = 1.47 - 0.389 = 1.08 atm

13.77 Fe2O3(s) + 3 CO(g) _ 2 Fe(s) + 3 CO2(g)


initial (atm) 0.978 0
change (atm) -3x +3x
equil (atm) 0.978 - 3x 3x
3 3
(P ) (3 x )
Kp = CO2 3 = 19.9 = ; take the cube root of both sides and solve for x.
(PCO ) (0.978 _ 3 x )3
3 3x
19.9 = 2.71 =
(0.978 _ 3 x)
2.65 - 8.13x = 3x
2.65 = 11.13x
x = 2.65/11.13 = 0.238 atm
PCO = 0.978 - 3x = 0.978 - 3(0.238) = 0.264 atm
PCO2 = 3x = 3(0.238) = 0.714 atm

Le Châtelier's Principle

13.78 (a) Cl- (reactant) added, AgCl(s) increases


(b) Ag+ (reactant) added, AgCl(s) increases
(c) Ag+ (reactant) removed, AgCl(s) decreases
(d) Cl- (reactant) removed, AgCl(s) decreases
 l 
Disturbing the equilibrium by decreasing [Cl-] increases Qc  Qc = + _ 
 to a
 [ Ag ] [ ]
t Cl t 

value greater than Kc. To reach a new state of equilibrium, Qc must decrease, which
means that the denominator must increase; that is, the reaction must go from right to left,
thus decreasing the amount of solid AgCl.

344
Chapter 13 - Chemical Equilibrium
______________________________________________________________________________

13.79 (a) ClNO (product) added, NO2 concentration decreases


(b) NO (reactant) added, NO2 concentration increases
(c) NO (reactant) removed, NO2 concentration decreases
(d) ClNO2 (reactant) added, NO2 concentration increases
 [ClNO][ NO2] 
Adding ClNO2 decreases the value of Qc  Qc =  . To reach a new state of
 [ClNO2][NO] 
equilibrium, the reaction must go from left to right, thus increasing the concentration of
NO2.

13.80 (a) Because there are 2 mol of gas on the left side and 3 mol of gas on the right side of
the balanced equation, the stress of an increase in pressure is relieved by a shift in the
reaction to the side with fewer moles of gas (in this case, to reactants). The number of
moles of reaction products decreases.
(b) Because there are 2 mol of gas on both sides of the balanced equation, the
composition of the equilibrium mixture is unaffected by a change in pressure. The
number of moles of reaction product remains the same.
(c) Because there are 2 mol of gas on the left side and 1 mol of gas on the right side of
the balanced equation, the stress of an increase in pressure is relieved by a shift in the
reaction to the side with fewer moles of gas (in this case, to products). The number of
moles of reaction products increases.

13.81 As the volume increases, the pressure decreases at constant temperature.


(a) Because there is 1 mol of gas on the left side and 2 mol of gas on the right side of the
balanced equation, the stress of an increase in volume (decrease in pressure) is relieved
by a shift in the reaction to the side with the larger number of moles of gas (in this case,
to products).
(b) Because there are 3 mol of gas on the left side and 2 mol of gas on the right side of
the balanced equation, the stress of an increase in volume (decrease in pressure) is
relieved by a shift in the reaction to the side with the larger number of moles of gas (in
this case, to reactants).
(c) Because there are 3 mol of gas on both sides of the balanced equation, the
composition of the equilibrium mixture is unaffected by an increase in volume (decrease
in pressure). There is no net reaction in either direction.

13.82 CO(g) + H2O(g) _ CO2(g) + H2(g) ∆Ho = - 41.2 kJ


The reaction is exothermic. [H2] decreases when the temperature is increased.
As the temperature is decreased, the reaction shifts to the right. [CO2] and [H2] increase,
[CO] and [H2O] decrease, and Kc increases.

13.83 Because ∆Ho is positive, the reaction is endothermic.


heat + 3 O2(g) _ 2 O3(g)

345
Chapter 13 - Chemical Equilibrium
______________________________________________________________________________

[ ]2
Kc = O3 3
[O2 ]
As the temperature increases, heat is added to the reaction, causing a shift to the right.
The [O3] increases, and the [O2] decreases. This results in an increase in Kc.

13.84 (a) HCl is a source of Cl- (product), the reaction shifts left, the equilibrium [CoCl42-]
increases.
(b) Co(NO3)2 is a source of Co(H2O)62+ (product), the reaction shifts left, the
equilibrium [CoCl42-] increases.
(c) All concentrations will initially decrease and the reaction will shift to the right, the
equilibrium [CoCl42-] decreases.
(d) For an exothermic reaction, the reaction shifts to the left when the temperature is
increased, the equilibrium [CoCl42-] increases.

13.85 (a) Fe(NO3)3 is a source of Fe3+. Fe3+ (reactant) added; the FeCl2+ concentration increases.
(b) Cl- (reactant) removed; the FeCl2+ concentration decreases.
(c) An endothermic reaction shifts to the right as the temperature increases; the FeCl2+
concentration increases.
(d) A catalyst does not affect the composition of the equilibrium mixture; no change in
FeCl2+ concentration.

13.86 (a) The reaction is exothermic. The amount of CH3OH (product) decreases as the
temperature increases.

(b) When the volume decreases, the reaction shifts to the side with fewer gas molecules.
The amount of CH3OH increases.
(c) Addition of an inert gas (He) does not affect the equilibrium composition. There is
no change.
(d) Addition of CO (reactant) shifts the reaction toward product. The amount of
CH3OH increases.
(e) Addition or removal of a catalyst does not affect the equilibrium composition. There
is no change.

13.87 (a) An endothermic reaction shifts to the right as the temperature increases. The amount
of acetone increases.
(b) Because there is 1 mol of gas on the left side and 2 mol of gas on the right side of the
balanced equation, the stress of an increase in volume (decrease in pressure) is relieved
by a shift in the reaction to the side with the larger number of moles of gas (in this case,
to products). The amount of acetone increases.
(c) The addition of Ar (an inert gas) with no volume change does not affect the
composition of the equilibrium mixture. The amount of acetone does not change.
(d) H2 (product) added; the amount of acetone decreases.
(e) A catalyst does not affect the composition of the equilibrium mixture. The amount
of acetone does not change.

346
Chapter 13 - Chemical Equilibrium
______________________________________________________________________________

Chemical Equilibrium and Chemical Kinetics

13.88 A + B _ C
ratef = kf[A][B] and rater = kr[C]; at equilibrium, ratef = rater
kf [C]
kf[A][B] = kr[C]; = = Kc
k r [A][B]

13.89 An equilibrium mixture that contains large amounts of reactants and small amounts of
products has a small Kc. A small Kc has kf < kr (c).

0.13
13.90 Kc = k f = _4
= 210
k r 6.2 x 10

6 x 10_ 6 M _1 s _1
13.91 Kc = k f ; kr = k f = 16
= 6 x 10-22 M-1s-1
kr Kc 10

13.92 kr increases more than kf, this means that Ea (reverse) is greater than Ea (forward). The
reaction is exothermic when Ea (reverse) > Ea (forward).

13.93 kf increases more than kr, this means that Ea (forward) is greater than Ea (reverse). The
reaction is endothermic when Ea (forward) > Ea (reverse).

General Problems
0.500 mol
13.94 (a) [N2O4] = = 0.125 M
4.00 L
N2O4(g) _ 2 NO2(g)
initial (M) 0.125 0
change (M) -(0.793)(0.125) +(2)(0.793)(0.125)
equil (M) 0.125 - (0.793)(0.125) (2)(0.793)(0.125)
At equilibrium, [N2O4] = 0.125 - (0.793)(0.125) = 0.0259 M
[NO2] = (2)(0.793)(0.125) = 0.198 M
2 2
[ ] (0.198 )
Kc = NO2 = = 1.51
[ N 2 O4] (0.0259)
∆n = 2 - 1 = 1 and Kp = Kc(RT)∆n; Kp = Kc(RT) = (1.51)(0.082 06)(400) = 49.6

(b)

13.95 Kc is very large. The reaction goes essentially to completion.


1 1 1
Kc = ; [CO2] = = 24
= 6.2 x 10-25 M
[CO2] Kc 1.6 x 10

347
Chapter 13 - Chemical Equilibrium
______________________________________________________________________________

[ NH3 ]2
13.96 Kc = = 0.291
[ N 2][H 2 ]3
At equilibrium, [N2] = 1.0 x 10-3 M and [H2] = 2.0 x 10-3 M
[NH3] = [ N 2] x [H 2 ]3 x K c = (1.0 x 10_ 3)(2.0 x 10_ 3 )3 (0.291) = 1.5 x 10-6 M

( )2
13.97 (a) Kp = P F = 7.83 at 1500 K
P F2
PF = K p PF2 = (7.83)(0.200) = 1.25 atm
(b) F2(g) _ 2 F(g)
initial (atm) x 0
change (atm) -y +2y
equil (atm) x-y 2y
2y = 1.25 so y = 0.625
x - y = 0.200; x = 0.200 + y = 0.200 + 0.625 = 0.825
0.625
f= = 0.758
0.825
(c) The shorter bond in F2 is expected to be stronger. However, because of the small
size of F, repulsion between the lone pairs of the two halogen atoms are much greater in
F2 than in Cl2.
13.98 2 HI(g) _ H2(g) + I2(g)
[ ][ ] (0.13)(0.70)
Calculate Kc. Kc = H 2 I22 = 2
= 0.0206
[HI ] (2.1 )
0.20 mol
[HI] = = 0.40 M
0.5000 L
2 HI(g) _ H2(g) + I2(g)
initial (M) 0.40 0 0
change (M) -2x +x +x
equil (M) 0.40 - 2x x x
2
[H 2][I2] x
Kc = 0.0206 = =
[HI ]2 (0.40 _ 2 x )2
Take the square root of both sides, and solve for x.
2
x x
0.0206 = 2
; 0.144 = ; x = 0.045
(0.40 _ 2 x ) 0.40 _ 2 x
At equilibrium, [H2] = [I2] = x = 0.045 M; [HI] = 0.40 - 2x = 0.40 - 2(0.045) = 0.31 M

13.99 Note the container volume is 5.00 L


[H2] = [I2] = 1.00 mol/5.00 L = 0.200 M
[HI] = 2.50 mol/5.00 L = 0.500 M
H2(g) + I2(g) _ 2 HI(g)
initial (M) 0.200 0.200 0.500
change (M) -x -x +2x

348
Chapter 13 - Chemical Equilibrium
______________________________________________________________________________

equil (M) 0.200 - x 0.200 - x 0.500 + 2x


2 2
[HI ] (0.500 + 2 x )
Kc = = 129 =
[H 2][I2] (0.200 _ x )2
Take the square root of both sides, and solve for x.
(0.500 + 2 x )2 0.500 + 2 x
129 = 2
; 11.4 = ; x = 0.133
(0.200 _ x ) 0.200 _ x
[H2] = [I2] = 0.200 - x = 0.200 - 0.133 = 0.067 M
[HI] = 0.500 + 2x = 0.500 + 2(0.133) = 0.766 M

6.00 mol
13.100 [H2O] = = 1.20 M
5.00 L
C(s) + H2O(g) _ CO(g) + H2(g)
initial (M) 1.20 0 0
change (M) -x +x +x
equil (M) 1.20 - x x x
2
[CO][H 2] x
Kc = = 3.0 x 10_ 2 =
[H2 O] 1.20 _ x
2 -2
x + (3.0 x 10 )x - 0.036 = 0
Use the quadratic formula to solve for x.
_ (0.030) ± (0.030 ) _ 4(_ 0.036) _ 0.030 ± 0.381
2

x= =
2(1) 2
x = 0.176 and -0.206
Discard the negative solution (-0.206) because it leads to negative concentrations and
that is impossible.
[CO] = [H2] = x = 0.18 M; [H2O] = 1.20 - x = 1.20 - 0.18 = 1.02 M

13.101 (a) Because Kp is larger at the higher temperature, the reaction has shifted toward
products at the higher temperature, which means the reaction is endothermic.
(b) (i) Increasing the volume causes the reaction to shift toward the side with more mol
of gas (product side). The equilibrium amounts of PCl3 and Cl2 increase while that of
PCl5 decresases.
(ii) If there is no volume change, there is no change in equilibrium concentrations.
(iii) Addition of a catalyst does not affect the equilibrium concentrations.

13.102 A decrease in volume (a) and the addition of reactants (c) will affect the composition of
the equilibrium mixture, but leave the value of Kc unchanged.
A change in temperature (b) affects the value of Kc.
Addition of a catalyst (d) or an inert gas (e) affects neither the composition of the
equilibrium mixture nor the value of Kc.

13.103 (a) Addition of a solid does not affect the equilibrium composition. There is no change
in the number of moles of CO2.
(b) Adding a product causes the reaction to shift toward reactants. The number of moles

349
Chapter 13 - Chemical Equilibrium
______________________________________________________________________________

of CO2 decreases.
(c) Decreasing the volume causes the reaction to shift toward the side with fewer mol of
gas (reactant side). The number of moles of CO2 decreases.
(d) The reaction is endothermic. An increase in temperature shifts the reaction toward
products. The number of moles of CO2 increases.

13.104 2 monomer _ dimer


(a) In benzene, Kc = 1.51 x 102
2 monomer _ dimer
initial (M) 0.100 0
change (M) -2x +x
equil (M) 0.100 - 2x x
[dimer] x
Kc = = 1.51 x 102 =
[monomer ]2 (0.100 _ 2 x )2
604x2 - 61.4x +1.51 = 0
Use the quadratic formula to solve for x.
_ (_ 61.4) ± (_ 61.4 )2 _ (4)(604)(1.51) 61.4 ± 11.04
x= =
2(604) 1208
x = 0.0600 and 0.0417

Discard the larger solution (0.0600) because it gives a negative concentration of the
monomer and that is impossible.
[monomer] = 0.100 - 2x = 0.100 - 2(0.0417) = 0.017 M; [dimer] = x = 0.0417 M
[dimer] 0.0417 M
= = 2.5
[monomer] 0.017 M
(b) In H2O, Kc = 3.7 x 10-2
2 monomer _ dimer
initial (M) 0.100 0
change (M) -2x +x
equil (M) 0.100 - 2x x
[dimer] x
Kc = 2
= 3.7 x 10_ 2 = 2
[monomer ] (0.100 _ 2 x )
0.148x2 - 1.0148x + 0.000 37 = 0
Use the quadratic formula to solve for x.
_ (_ 1.0148) ± (_ 1.0148 ) _ (4)(0.148)(0.00037) 1.0148 ± 1.0147
2

x= =
2(0.148) 0.296
x = 6.86 and 3.7 x 10-4
Discard the larger solution (6.86) because it gives a negative concentration of the
monomer and that is impossible.
[monomer] = 0.100 - 2x = 0.100 - 2(3.7 x 10-4) = 0.099 M; [dimer] = x = 3.7 x 10-4 M
[dimer] 3.7 x 10_ 4 M
= = 0.0038
[monomer] 0.099 M

350
Chapter 13 - Chemical Equilibrium
______________________________________________________________________________

(c) Kc for the water solution is so much smaller than Kc for the benzene solution because
H2O can hydrogen bond with acetic acid, thus preventing acetic acid dimer formation.
Benzene cannot hydrogen bond with acetic acid.

13.105 C(s) + CO2(g) _ 2 CO(g)


initial (M) excess 1.50 mol/20.0 L 0
change (M) -x +2x
equil (M) 0.0750 - x 2x
-2
[CO] = 2x = 7.00 x 10 M; x = 0.0350 M
(a) [CO2] = 0.0750 - x = 0.0750 - 0.0350 = 0.0400 M
[CO ]2 (7.00 x 10_ 2 )2
(b) Kc = = = 0.122
[CO2] (0.0400)

13.106 (a) [PCl5] = 1.000 mol/5.000 L = 0.2000 M


PCl5(g) _ PCl3(g) + Cl2(g)
initial (M) 0.2000 0 0
change (M) -(0.2000)(0.7850) +(0.2000)(0.7850) +(0.2000)(0.7850)
equil (M) 0.0430 0.1570 0.1570
[ ][ ] (0.1570)(0.1570)
Kc = PCl3 Cl2 = = 0.573
[PCl5] (0.0430)
∆n = 1 and Kp = Kc(RT) = (0.573)(0.082 06)(500) = 23.5
[ ][ ] (0.150)(0.600)
(b) Qc = PCl3 Cl2 = = 0.18
[PCl5] (0.500)
Because Qc < Kc, the reaction proceeds to the right to reach equilibrium.
PCl5(g) _ PCl3(g) + Cl2(g)
initial (M) 0.500 0.150 0.600
change (M) -x +x +x
equil (M) 0.500 - x 0.150 + x 0.600 + x
[PCl3][Cl2] (0.150 + x)(0.600 + x)
Kc = = 0.573 = ; solve for x.
[PCl5] (0.500 _ x)
x2 + 1.323x - 0.1965 = 0
_ (1.323) ± (1.323 )2 _ (4)(1)(_ 0.1965) _ 1.323 ± 1.593
x= =
2(1) 2
x = -1.458 and 0.135
Discard the negative solution (-1.458) because it will lead to negative concentrations and
that is impossible.
[PCl5] = 0.500 - x = 0.500 - 0.135 = 0.365 M
[PCl3] = 0.150 + x = 0.150 + 0.135 = 0.285 M
[Cl2] = 0.600 + x = 0.600 + 0.135 = 0.735 M

(2.00)(1.50)
13.107 Qp = = 1.00, Qp < Kp therefore the reaction proceeds from reactants to
(3.00)

351
Chapter 13 - Chemical Equilibrium
______________________________________________________________________________

products to reach equilibrium.


PCl5(g) _ PCl3(g) + Cl2(g)
initial (atm) 3.00 2.00 1.50
change (atm) -x +x +x
equil (atm) 3.00 - x 2.00 + x 1.50 + x
(P )(P ) (2.00 + x) (1.50 + x)
Kp = PCl3 Cl2 = 1.42 =
(PPCl5) 3.00 _ x
2
x + 4.92x - 1.26 = 0
Use the quadratic formula to solve for x.
(_ 4.92) ± (4.92 )2 _ 4(1)(_ 1.26) (_ 4.92) ± 5.41
x= =
2(1) 2
x = 0.245 and -5.165
Discard the negative solution (-5.165) because it gives negative partial pressures and that
is impossible.
P PCl5 = 3.00 - x = 3.00 - 0.245 = 2.76 atm
P PCl3 = 2.00 + x = 2.00 + 0.245 = 2.24 atm
PCl2 = 1.50 + x = 1.50 + 0.245 = 1.74 atm
Ptotal = P PCl5 + P PCl3 + PCl2 = 2.76 + 2.24 + 1.74 = 6.74 atm
[ ][ ]
13.108 (a) K c = C2 H6 C2 H 4 K p = (PC2 H6) (PC2 H4)OVERPC4 H10
[C4 H10]
 1   1 
(b) Kp = 12; ∆n = 1; Kc = Kp   = (12)  = 0.19
 RT   (0.082 06)(773) 

(c) C4H10(g) _ C2H6(g) + C2H4(g)


initial (atm) 50 0 0
change (atm) -x +x +x
equil (atm) 50 - x x x
2
Kp = 12 = x ; x2 + 12x - 600 = 0
50 _ x
Use the quadratic formula to solve for x.
(_ 12) ± (12 )2 _ 4(1)(_ 600) _ 12 ± 50.44
x= =
2(1) 2
x = -31.22 and 19.22
Discard the negative solution (-31.22) because it leads to negative concentrations and
that is impossible.
19.22
% C4H10 converted = x 100% = 38%
50
Ptotal = PC4H10 + PC2H6 + PC2 H4 = (50 - x) + x + x = (50 - 19) + 19 + 19 = 69 atm
(d) A decrease in volume would decrease the % conversion of C4H10.

352
Chapter 13 - Chemical Equilibrium
______________________________________________________________________________

13.109 (a) Because ∆n = 0, Kp = Kc = 1.0 x 105


Ppropene Ppropene 5.0 atm
(b) Kp = 1.0 x 105 = ; Pcyclopropane = 5
= 5
= 5.0 x 10-5 atm
Pcyclopropane 1.0 x 10 1.0 x 10
(c) The ratio of the two concentrations is equal to Kc. The ratio (Kc) cannot be changed
by adding cyclopropane. The individual concentrations can change but the ratio of
concentrations can't.
Because there is one mole of gas on each side of the balanced equation, the composition
of the equilibrium mixture is unaffected by a decrease in volume. The ratio of the two
concentrations will not change.
(d) Because Kc is large, kf > kr.
(e) Because the C–C–C bond angles are 60o and the angles between sp3 hybrid orbitals
are 109.5o, the hybrid orbitals are not oriented along the bond directions. Their overlap
is therefore poor, and the C–C bonds are correspondingly weak.

 1   1 
13.110 (a) Kp = 3.45; ∆n = 1; Kc = Kp   = (3.45)  = 0.0840
 RT   (0.082 06)(500) 
(b) [(CH3)3CCl] = 1.00 mol/5.00 L = 0.200 M
(CH3)3CCl(g) _ (CH3)2C=CH2(g) + HCl(g)
initial (M) 0.200 0 0
change (M) -x +x +x
equil (M) 0.200 - x x x
2
Kc = 0.0840 = x ; x2 + 0.0840x - 0.0168 = 0
0.200 _ x
Use the quadratic formula to solve for x.
(_ 0.0840) ± (0.0840 )2 _ 4(1)(_ 0.0168) _ 0.0840 ± 0.272
x= =
2(1) 2
x = -0.178 and 0.094
Discard the negative solution (-0.178) because it leads to negative concentrations and
that is impossible.
[(CH3)2C=CCH2] = [HCl] = x = 0.094 M
[(CH3)3CCl] = 0.200 - x = 0.200 - 0.094 = 0.106 M
(c) Kp = 3.45
(CH3)3CCl(g) _ (CH3)2C=CH2(g) + HCl(g)
initial (atm) 0 0.400 0.600
change (atm) +x -x -x
equil (atm) x 0.400 - x 0.600 - x
(0.400 _ x)(0.600 _ x)
Kp = 3.45 =
x
x2 - 4.45x + 0.240 = 0
Use the quadratic formula to solve for x.

353
Chapter 13 - Chemical Equilibrium
______________________________________________________________________________

_ (_ 4.45) ± (_ 4.45 )2 _ 4(1)(0.240) 4.45 ± 4.34


x= =
2(1) 2
x = 0.055 and 4.40
Discard the larger solution (4.40) because it leads to negative partial pressures and that is
impossible.
Pt-butyl chloride = x = 0.055 atm; Pisobutylene = 0.400 - x = 0.400 - 0.055 = 0.345 atm
PHCl = 0.600 - x = 0.600 - 0.055 = 0.545 atm

13.111 (a) The Arrhenius equation gives for the forward and reverse reactions
_ / RT
k f = Af e Ea,f and k r = A r e_ Ea,r / RT
Addition of a catalyst decreases the activation energies by ∆Ea, so
_ ∆ ) / RT
_(
k f = Af e Ea,f Ea = Af e_ Ea,f / RT x e∆ Ea / RT

_ ∆ ) / RT
and _(
k r = Ar e Ea,r Ea = A r e _ Ea,r / RT x e∆ Ea / RT
Therefore, the rate constants for both the forward and reverse reactions increase by the
same factor, e∆ Ea / RT .
_ Ea,f / RT
(b) The equilibrium constant is given by Kc = k f = Af e _ E / RT = A f e _ ∆ E / RT
k r A r e a,r Ar
where ∆E = Ea,f - Ea,r. Addition of a catalyst decreases the activation energies by ∆Ea.

_ Ea,f / RT
x e∆ Ea / RT A f _ ∆ E / RT
So, Kc = k f = A f e_ E / RT = e
k r A r e a,r x e∆ Ea / RT A r
Kc is unchanged because of cancellation of e∆ Ea / RT , the factor by which the two rate
constants increase.

13.112 The activation energy (Ea) is positive, and for an exothermic reaction, Ea,r > Ea,f.

kf = Af e _ Ea,f / RT
, kr = Ar e _ Ea,r / RT

_
Ea,f / RT
Kc = k f = A f e _ E / RT = A f e( Ea,r _ Ea,f ) / RT
k r A r e a,r Ar

(Ea,r - Ea,f) is positive, so the exponent is always positive. As the temperature increases,
the exponent, (Ea,r _ Ea,f ) / RT , decreases and the value for Kc decreases as well.

13.113 (a) PV = nRT


 L • atm 
(0.974 mol) 0.082 06 (1000 K)
o nRT  K • mol 
P Br2 = = = 80 atm
V 1.00 L

354
Chapter 13 - Chemical Equilibrium
______________________________________________________________________________

 L • atm 
(1.22 mol) 0.082 06 (1000 K)
o nRT  K • mol 
P H2 = = = 100 atm
V 1.00 L
Because Kp is very large, assume first that the reaction goes to completion and then is
followed by a small back reaction.

H2(g) + Br2(g) _ 2 HBr(g)


before (atm) 100 80 0
100% reaction (atm) -80 -80 +2(80)
after (atm) 20 0 160
back reaction (atm) +x +x -2x
after (atm) 20 + x x 160 - 2x

PH2 = 20 + x ≈ 20 atm
PHBr = 160 - 2x ≈ 160 atm
(P HBr )2 (160 )2
Kp = = 2.1 x 106 =
(P H2)(P Br2) (20)(x)
(160 )2
PBr2 = x = 6
= 6.1 x 10-4 atm
(20)(2.1 x 10 )
(b) (ii) Adding Br2 will cause the greatest increase in the pressure of HBr. The very
large value of Kp means that the reaction goes essentially to completion. Therefore, the
reaction stops when the limiting reactant, Br2, is essentially consumed. No matter how
much H2 is added or how far the the equilibrium is shifted (by lowering the temperature)
to favor the formation of HBr, the amount of HBr will ultimately be limited by the
amount of Br2 present. So more Br2 must be added in order to produce more HBr.

PV (0.588 atm)(1.00 L)
13.114 (a) PV = nRT, n total = = = 0.0239 mol
RT  L• atm 
 0.082 06 (300 K)
 K • mol 
2 NOBr(g) _ 2 NO(g) + Br2(g)
initial (mol) 0.0200 0 0
change (mol) -2x +2x +x
equil (mol) 0.0200 - 2x 2x x

ntotal = 0.0239 mol = (0.0200 - 2x) + 2x + x = 0.0200 + x


x = 0.0239 - 0.0200 = 0.0039 mol
Because the volume is 1.00 L, the molarity equals the number of moles.
[NOBr] = 0.0200 - 2x = 0.0200 - 2(0.0039) = 0.0122 M
[NO] = 2x = 2(0.0039) = 0.0078 M
[Br2] = x = 0.0039 M
[NO ]2 [Br 2] (0.0078 )2 (0.0039)
Kc = 2
= 2
= 1.6 x 10-3
[NOBr ] (0.0122 )

355
Chapter 13 - Chemical Equilibrium
______________________________________________________________________________

(b) ∆n = (3) - (2) = 1, Kp = Kc(RT) = (1.6 x 10-3)(0.082 06)(300) = 0.039

13.115 NO2, 46.01 amu


1 mol NO2
mol NO2 = 4.60 g NO2 x = 0.100 mol NO2
46.01 g NO2
[NO2] = 0.100 mol/10.0 L = 0.0100 M
2 NO2(g) _ N2O4(g)
initial (M) 0.0100 0
change (M) -2x +x
equil (M) 0.0100 - 2x x

[ ] x
Kc = N 2 O42 = 4.72 =
[ NO2 ] (0.0100 _ 2 x )2
18.88x2 - 1.189x + 0.000 472 = 0
Use the quadratic formula to solve for x.
_ (_ 1.189) ± (_ 1.189 )2 _ 4(18.88)(0.000 472) 1.189 ± 1.1739
x= =
2(18.88) 37.76
-4
x = 0.0626 and 4.00 x 10
Discard the larger solution (0.0626) because it leads to a negative concentration of NO2
and that is impossible.

ntotal = (0.0100 - 2x + x)(10.0 L) = (0.0100 - x)(10.0 L)


= [0.0100 mol/L - (4.00 x 10-4 mol/L)](10.0 L) = 0.0960 mol
o
100 C = 100 + 273 = 373 K
 L • atm 
(0.0960 mol) 0.082 06 (373 K)
n total RT  K • mol 
Ptotal = = = 0.294 atm
V 10.0 L

13.116 (a) W(s) + 4 Br(g) _ WBr4(g)


P WBr4
Kp = 4
= 100, P WBr4 = (P Br )4 (100) = (0.010 atm)4(100) = 1.0 x 10-6 atm
(PBr )
(b) Because Kp is smaller at the higher temperature, the reaction has shifted toward
reactants at the higher temperature, which means the reaction is exothermic.
(1.0 x 10_ 6)
(c) At 2800 K, Qp = 4
= 100, Qp > Kp so the reaction will go from products
(0.010 )
to reactants, depositing tungsten back onto the filament.

13.117 (a) (NH4)(NH2CO2)(s) _ 2 NH3(g) + CO2(g)


initial (atm) 0 0
change (atm) +2x +x
equil (atm) 2x x

356
Chapter 13 - Chemical Equilibrium
______________________________________________________________________________

Ptotal = 2x + x = 3x = 0.116 atm


x = 0.116 atm/3 = 0.0387 atm
P NH3 = 2x = 2(0.0387 atm) = 0.0774 atm
PCO2 = x = 0.0387 atm
2 2 -4
K p = (P NH3 ) (PCO2) = (0.0774) (0.0387) = 2.32 x 10
(b) (i) The total quantity of NH3 would decrease. When product, CO2, is added, the
equilibrium will shift to the left.
(ii) The total quantity of NH3 would remain unchanged. Adding a pure solid,
(NH4)(NH2CO2), to a heterogeneous equilibrium will not affect the position of the
equilibrium.
(iii) The total quantity of NH3 would increase. When product, CO2, is removed, the
equilibrium will shift to the right.
(iv) The total quantity of NH3 would increase. When the total volume is increased, the
reaction will shift to the side with more total moles of gas, which in this case is the
product side.
(v) The total quantity of NH3 would remain unchanged. Neon is an inert gas which will
have no effect on the reaction or on the position of equilibrium.
(vi) The total quantity of NH3 would increase. Because the reaction is endothermic, an
increase in temperature will shift the equilibrium to to the right.

13.118 2 NO2(g) _ N2O4(g)


∆n = (1) - (2) = -1 and Kp = Kc(RT)-1 = (216)[(0.082 06)(298)]-1 = 8.83
P N 2 O4
Kp = 2
= 8.83
(P NO2 )
Let X = P N2O4 and Y = P NO2 .
X
Ptotal = 1.50 atm = X + Y and 2 = 8.83. Use these two equations to solve for X and Y.
Y
X = 1.50 - Y
1.50 _ Y
2
= 8.83
Y
8.83Y2 + Y - 1.50 = 0
Use the quadratic formula to solve for Y.
_ (1) ± (1) _ 4(8.83)(_ 1.50) _ 1 ± 7.35
2

Y= =
2(8.83) 17.7
Y = -0.472 and 0.359
Discard the negative solution (-0.472) because it leads to a negative partial pressure of
NO2 and that is impossible.
Y = P NO2 = 0.359 atm
X = P N2O4 = 1.50 atm - Y = 1.50 atm - 0.359 atm = 1.14 atm

13.119 500oC = 500 + 273 = 773 K and 840oC = 840 + 273 = 1113 K

357
Chapter 13 - Chemical Equilibrium
______________________________________________________________________________

Calculate the undissociated pressure of F2 at 1113 K.


P2 = P1 ; P1 T 2 = (0.600 atm)(1113 K) = 0.864 atm
P2 =
T 2 T1 T1 773 K
F2(g) _ 2 F(g)
initial (atm) 0.864 0
change (atm) -x +2x
equil (atm) 0.864 - x 2x

Ptotal = (0.864 atm - x) + 2x = 0.864 atm + x = 0.984 atm


x = 0.984 atm - 0.864 atm = 0.120 atm
PF2 = (0.864 atm - x) = (0.864 atm - 0.120 atm) = 0.744 atm
PF = 2x = 2(0.120 atm) = 0.240 atm
( P F )2 (0.240 )2
Kp = = = 0.0774
P F2 0.744

13.120 N2(g) + 3 H2 _ 2 NH3


initial (mol) 0 0 X
change (mol) +y +3y -2y
equil (mol) y 3y X - 2y
y = 0.200 mol
Because the volume is 1.00 L, the molarity equals the number of moles.
[N2] = y = 0.200 M; [H2] = 3y = 3(0.200) = 0.600 M
[ NH3 ]2 [ NH3 ]2
Kc = = = 4.20 , solve for [NH3]eq
[ N 2][H 2 ]3 (0.200)(0.600 )3
2
[ NH3 ]eq = [ N 2][H 2 ]3 (4.20) = (0.200)(0.600 )3 (4.20)
[ NH3 ]eq = [ N 2][H 2 ]3 (4.20) = (0.200)(0.600 )3 (4.20) = 0.426 M
[NH3]eq = 0.426 M = X - 2(0.200) = [NH3]o - 2(0.200)
[NH3]o = 0.426 + 2(0.200) = 0.826 M
0.826 mol of NH3 were placed in the 1.00 L reaction vessel.

Multi-Concept Problems
1 mol H 2 O
13.121 (a) H2O, 18.015 amu; 125.4 g H2O x = 6.96 mol H2O
18.015 g H 2 O
Given that mol CO = mol H2O = 6.96 mol
 L • atm 
(6.96 mol) 0.082 06 (700 K)
nRT  K • mol 
PCO = PH2 O = = = 40.0 atm
V 10.0 L
CO(g) + H2O(g) _ CO2(g) + H2(g)
initial (atm) 40.0 40.0 0 0
equil (atm) 9.80 9.80 40.0 - 9.80 40.0 - 9.80
= 30.2 = 30.2

358
Chapter 13 - Chemical Equilibrium
______________________________________________________________________________

(PCO2)(PH2) (30.2)(30.2)
Kp = = = 9.50
(PCO )(PH2 O ) (9.80)(9.80)

1 mol H 2 O
(b) 31.4 g H2O x = 1.743 mol H2O
18.015 g H 2 O
 L • atm 
(1.743 mol) 0.082 06 (700 K)
nRT  K • mol 
P H2 O = = = 10.0 atm
V 10.0 L
PH2 O has been increased by 10.0 atm; a new equilibrium will be established.
CO(g) + H2O(g) _ CO2(g) + H2(g)
initial (atm) 9.80 9.80 +10.0 30.2 30.2
change (atm) -x -x +x +x
equil (atm) 9.80 - x 19.8 - x 30.2 + x 30.2 + x
(PCO2)(P H2) (30.2 + x)(30.2 + x)
Kp = = 9.50 =
(PCO )(P H2 O) (9.80 _ x)(19.80 _ x)
2
8.50x - 341.6x + 931.34 = 0
Use the quadratic formula to solve for x.
_ (_ 341.6) ± (_ 341.6 ) _ 4(8.50)(931.34) 341.6 ± 291.6
2

x= =
2(8.50) 17.0
x = 37.25 and 2.94
Discard the larger solution (37.25) because it leads to negative partial pressures and that
is impossible.
PCO = 9.80 - x = 9.80 - 2.94 = 6.86 atm
PH2 O = 19.8 - x = 19.8 - 2.94 = 16.9 atm
PCO2 = PH2 = 30.2 + x = 30.2 + 2.94 = 33.1 atm
PV (33.1 atm) (10.0 L)
n H2 = = = 5.76 mol H2
RT  L • atm 
 0.082 06 (700 K)
 K • mol 
5.76 mol H 2 1L 1 mL 6.022 x 1023 H 2 molecules
x x 3
x = 3.47 x 1020 H2
10.0 L 1000 mL 1 cm 1 mol H 2
3
molecules/cm

13.122 (a) CO2, 44.01 amu; CO, 28.01 amu


1 mol CO 2
79.2 g CO2 x = 1.80 mol CO2
44.01 g CO 2

CO2(g) + C(s) _ 2 CO(g)


initial (mol) 1.80 0
change (mol) -x +2x
equil (mol) 1.80 - x 2x

359
Chapter 13 - Chemical Equilibrium
______________________________________________________________________________

total mass of gas in flask = (16.3 g/L)(5.00 L) = 81.5 g


81.5 = (1.80 - x)(44.01) + (2x)(28.01)
81.5 = 79.22 - 44.01x + 56.02x; 2.28 = 12.01x; x = 2.28/12.01 = 0.19
n CO2 = 1.80 - x = 1.80 - 0.19 = 1.61 mol CO2; n CO = 2x = 2(0.19) = 0.38 mol CO
 L • atm 
(1.61 mol) 0.082 06 (1000 K)
nRT  K • mol 
PCO2 = = = 26.4 atm
V 5.0 L
 L • atm 
(0.38 mol) 0.082 06 (1000 K)
nRT  K • mol 
PCO = = = 6.24 atm
V 5.0 L
2 2
( ) (6.24 )
Kp = PCO = = 1.47
(PCO2) (26.4)
(b) At 1100K, the total mass of gas in flask = (16.9 g/L)(5.00 L) = 84.5 g
84.5 = (1.80 - x)(44.01) + (2x)(28.01)
84.5 = 79.22 - 44.01x + 56.02x; 5.28 = 12.01x; x = 5.28/12.01 = 0.44
n CO2 = 1.80 - x = 1.80 - 0.44 = 1.36 mol CO2; n CO = 2x = 2(0.44) = 0.88 mol CO
 L • atm 
(1.36 mol) 0.082 06 (1100 K)
nRT  K • mol 
PCO2 = = = 24.6 atm
V 5.0 L

 L • atm 
(0.88 mol) 0.082 06 (1100 K)
nRT  K • mol 
PCO = = = 15.9 atm
V 5.0 L
(PCO )2 (15.9 )2
Kp = = = 10.3
(PCO2) (24.6)
(c) In agreement with Le Châtelier’s principle, the reaction is endothermic because Kp
increases with increasing temperature.

13.123 CO2, 44.01 amu; CO, 28.01 amu; BaCO3, 197.34 amu
1 mol CO2
1.77 g CO2 x = 0.0402 mol CO2
44.01 g CO2

CO2(g) + C(s) _ 2 CO(g)


initial (mol) 0.0402 0
change (mol) -x +2x
equil (mol) 0.0402 - x 2x
1 mol BaCO3 1 mol CO2
3.41 g BaCO3 x x = 0.0173 mol CO2
197.34 g BaCO3 1 mol BaCO3
mol CO2 = 0.0173 = 0.0402 - x; x = 0.0229
mol CO = 2x = 2(0.0229) = 0.0458 mol CO

360
Chapter 13 - Chemical Equilibrium
______________________________________________________________________________

 L • atm 
(0.0173 mol) 0.082 06 (1100 K)
nRT  K • mol 
PCO2 = = = 1.562 atm
V 1.000 L

 L • atm 
(0.0458 mol) 0.082 06 (1100 K)
nRT  K • mol 
PCO = = = 4.134 atm
V 1.000 L
2 2
( ) (4.134 )
Kp = PCO = = 11.0
(PCO2) (1.562)

13.124 (a) N2O4, 92.01 amu


1 mol N 2 O4
14.58 g N2O4 x = 0.1585 mol N2O4
92.01 g N 2 O4

PV = nRT
 L • atm 
(0.1585 mol) 0.082 06 (400 K)
nRT  K • mol 
P N 2 O4 = = = 5.20 atm
V 1.000 L

N2O4(g) _ 2 NO2(g)
initial (atm) 5.20 0
change (atm) -x +2x
equil (atm) 5.20 - x 2x
P total = P N 2 O4 + P NO2 (5.20 - x) + (2x) = 9.15 atm
=
5.20 + x = 9.15 atm
x = 3.95 atm
P N2O4 = 5.20 - x = 5.20 - 3.95 = 1.25 atm
P NO2 = 2x = 2(3.95) = 7.90 atm
(P NO2 )2 (7.90 )2
Kp = = = 49.9
(P N2O4) (1.25)
 1  (49.9)
∆n = 1 and Kc = Kp  = = 1.52
 RT  (0.082 06)(400)
(b) ∆Horxn = [2 ∆Hof(NO2)] - ∆Hof(N2O4)
∆Horxn = [(2 mol)(33.2 kJ/mol)] - [(1mol)(9.16 kJ/mol)] = 57.2 kJ
PV = nRT
PV (3.95 atm)(1.000 L)
moles N2O4 reacted = n = = = 0.1203 mol N2O4
RT  L • atm 
 0.082 06 (400 K)
 K • mol 

q = (57.24 kJ/mol N2O4)(0.1203 mol N2O4) = 6.89 kJ

361
Chapter 13 - Chemical Equilibrium
______________________________________________________________________________

13.125 C10H8(s) _ C10H8(g)


(a) Kc = [C10H8] = 5.40 x 10-6
room volume = 8.0 ft x 10.0 ft x 8.0 ft = 640 ft2
3 3
 12 in   2.54 cm   1.0 L 
room volume = 640 ft2 x   x  x  = 18122.8 L
 1 ft   1 in   1000 cm 
3

C10H8 molecules = (5.40 x 10-6 mol/L)(18122.8 L)(6.022 x 1023 molecules/mol)


= 5.89 x 1022 C10H8 molecules
(b) C10H8, 128.17 amu
mol C10H8 = (5.40 x 10-6 mol/L)(18122.8 L) = 0.0979 mol C10H8
128.17 g C10 H8
mass C10H8 = 0.0979 mol C10H8 x = 12.55 g C10H8
1 mol C10 H8
moth ball: r = 12.0 mm/2 = 6.0 mm = 0.60 cm
volume of moth ball = (4/3)πr3 = (4/3)π(0.60cm)3 = 0.905 cm3
mass of moth ball = (0.905 cm3/moth ball)(1.16 g/cm3) = 1.05 g/moth ball
12.55 g C10 H8
number of moth balls = = 12 moth balls
1.05 g C10 H8 /moth ball

13.126 The atmosphere is 21% (0.21) O2; PO2 = (0.21) (720 mm Hg ) = 0.199 atm
2 O3(g) _ 3 O2(g)
(PO2 )3 ( P O 2 )3 (0.199 )
3
Kp = 2
; PO3 = = 57
= 2.46 x 10-30 atm
(PO3 ) Kp 1.3 x 10
3
 100 cm 
6 3 1 L
vol = 10 x 10 m x   x = 1.0 x 1010 L
 1 m 
3
1000 cm
P V (2.46 x 10 atm) (1.0 x 1010 L)
_ 30
n O3 = = = 1.0 x 10-21 mol O3
RT  L • atm 
 0.082 06 (298 K)
 K • mol 
6.022 x 1023 O3 molecules
O3 molecules = 1.0 x 10-21 mol O3 x = 6.0 x 102 O3 molecules
1 mol O3

13.127 250.0 mL = 0.2500 L


[CH3CO2H] = 0.0300 mol/0.2500 L = 0.120 M
2 CH3CO2H _ (CH3CO2H)2
initial (M) 0.120 0
change (M) -2x +x
equil (M) 0.120 - 2x x

[dimer] x
Kc = 2
= 1.51 x 102 = 2
[monomer ] (0.120 _ 2 x )
604x2 - 73.48x + 2.1744 = 0

362
Chapter 13 - Chemical Equilibrium
___________________________________________________________________________
___

Use the quadratic formula to solve for x.


_ (_ 73.48) ± (_ 73.48 ) _ 4(604)(2.1744) 73.48 ± 12.08
2

x= =
2(604) 1208
x = 0.0708 and 0.0508
Discard the larger solution (0.0708) because it leads to a negative concentration and that
is impossible.
[monomer] = 0.120 - 2x = 0.120 - 2(0.0508) = 0.0184 M
[dimer] = x = 0.0508 M
(b) 25oC = 25 + 273 = 298 K
 L • atm 
Π = MRT = (0.0184 M + 0.0508 M)  0.082 06 (298 K) = 1.69 atm
 K • mol 

13.128 PCl5(g) _ PCl3(g) + Cl2(g)


∆n = (2) - (1) = 1 and at 700 K, Kp = Kc(RT) = (46.9)(0.082 06)(700) = 2694
(a) Because Kp is larger at the higher temperature, the reaction has shifted toward
products at the higher temperature, which means the reaction is endothermic. Because
the reaction involves breaking two P–Cl bonds and forming just one Cl–Cl bond, it
should be endothermic.
(b) PCl5, 208.24 amu
1 mol PCl5
mol PCl5 = 1.25 g PCl5 x = 6.00 x 10-3 mol
208.24 g PCl5
 L • atm 
(6.00 x 10_ 3 mol) 0.082 06 (700 K)
nRT  K • mol 
PV = nRT, P PCl5 = = = 0.689
V 0.500 L
atm
Because Kp is so large, first assume the reaction goes to completion and then allow for a
small back reaction.
PCl5(g) _ PCl3(g) + Cl2(g)
before rxn (atm) 0.689 0 0
100% rxn (atm) -0.689 +0.689 +0.689
after rxn (atm) 0 0.689 0.689
back rxn (atm) +x -x -x
equil (atm) x 0.689 - x 0.689 - x
(P )(P ) (0.689 _ x )2 (0.689 )2
Kp = PCl3 Cl2 = 2694 = ≈
PPCl5 x x
2
(0.689 )
x = PPCl5 = = 1.76 x 10-4 atm
2694
Ptotal = PPCl5 + PPCl3 + PCl2
-4
Ptotal = x + (0.689 - x) + (0.689 - x) = 0.689 + 0.689 - 1.76 x 10 = 1.38 atm

363
Chapter 13 - Chemical Equilibrium
___________________________________________________________________________
___

(PPCl5 )o _ (PPCl5) 0.689 _ (1.76 x 10_ 4)


% dissociation = x 100% = x 100% = 99.97%
(PPCl5 )o 0.689

(c)
The molecular geometry is trigonal bipyramidal. There is no dipole moment because of a
symmetrical distribution of Cl’s around the central P.

The molecular geometry is trigonal pyramidal. There is a dipole moment because of the
lone pair of electrons on the P and an unsymmetrical distribution of Cl’s around the
central P.

364
365
16 Applications of Aqueous Equilibria

(a) HNO2(aq) + OH (aq) Ω NO2 (aq) + H2O(l); NO2 (basic anion), pH > 7.00
S S S
16.1
(b) H3O+(aq) + NH3(aq) Ω NH4+(aq) + H2O(l); NH4+ (acidic cation), pH < 7.00
(c) OHS(aq) + H3O+(aq) Ω 2 H2O(l); pH = 7.00

(a) HF(aq) + OH (aq) Ω H2O(l) + F (aq)


S S
16.2
3.5 x 10 _ 4
Kn = K a = _14
= 3.5 x 1010
K w 1.0 x 10
(b) H3O+(aq) + OH (aq) Ω 2 H2O(l)
S

1 1
Kn = = _14
= 1.0 x 1014
K w 1.0 x 10
(c) HF(aq) + NH3(aq) Ω NH4+(aq) + F (aq)
S

(3.5 x 10 _ 4)(1.8 x 10 _ 5)
Kn = K a K b = _14
= 6.3 x 105
Kw 1.0 x 10
The tendency to proceed to completion is determined by the magnitude of Kn. The larger
the value of Kn, the further does the reaction proceed to completion.
The tendency to proceed to completion is: reaction (c) < reaction (a) < reaction (b)

HCN(aq) + H2O(l) Ω H3O+(aq) + CN (aq)


S
16.3
initial (M) 0.025 ~0 0.010
change (M) Sx +x +x
equil (M) 0.025 S x x 0.010 + x
+
[ ][ _ ] x(0.010 + x) x(0.010)
Ka = H 3O CN = 4.9 x 10 _10 = ≈
[HCN] 0.025 _ x 0.025
S9 S9 +
Solve for x. x = 1.23 x 10 M = 1.2 x 10 M = [H3O ]
pH = Slog[H3O+] = Slog(1.23 x 10S9) = 8.91
1.0 x 10 _14
[OHS] = K w + = _9
= 8.2 x 10S6 M
[H 3O ] 1.23 x 10
+ S
[Na ] = [CN ] = 0.010 M; [HCN] = 0.025 M
[HCN ]diss 1.23 x 10 _ 9 M
x 100% = 4.9 x 10 6 %
S
% dissociation = x 100% =
[HCN ]initial 0.025 M

433
Chapter 16 S Applications of Aqueous Equilibria
______________________________________________________________________________

0.10 mol
16.4 From NH4Cl(s), [NH4+]initial = = 0.20 M
0.500 L
NH3(aq) + H2O(l) Ω NH4+(aq) + OH (aq)
S

initial (M) 0.40 0.20 ~0


change (M) Sx +x +x
equil (M) 0.40 S x 0.20 + x x
+ _
[ ][ ] (0.20 + x)(x) (0.20)(x)
Kb = NH 4 OH = 1.8 x 10 _ 5 = ≈
[ NH 3] (0.40 _ x) (0.40)
S S5
Solve for x. x = [OH ] = 3.6 x 10 M
1.0 x 10 _14
[H3O+] = K w_ = = 2.8 x 10 10 M
S
_5
[OH ] 3.6 x 10
pH = Slog[H3O+] = Slog(2.8 x 10S10) = 9.55

16.5 Each solution contains the same number of B molecules. The presence of BH+ from
BHCl lowers the percent dissociation of B. Solution (2) contains no BH+, therefore it
has the largest percent dissociation. BH+ is the conjugate acid of B. Solution (1) has the
largest amount of BH+ and it would be the most acidic solution and have the lowest pH.

S
16.6 (a) (1) and (3). Both pictures show equal concentrations of HA and A .
S
(b) (3). It contains a higher concentration of HA and A .

HF(aq) + H2O(l) Ω H3O+(aq) + F (aq)


S
16.7
initial (M) 0.25 ~0 0.50
change (M) Sx +x +x
equil (M) 0.25 S x x 0.50 + x
[H 3O + ][F _ ] x(0.50 + x) x(0.50)
Ka = = 3.5 x 10 _ 4 = ≈
[HF] 0.25 _ x 0.25
S4 +
Solve for x. x = 1.75 x 10 M = [H3O ]
For the buffer, pH = Slog[H3O+] = Slog(1.75 x 10S4) = 3.76
(a) mol HF = 0.025 mol; mol FS = 0.050 mol; vol = 0.100 L
100%
F (aq) + H3O+(aq)  HF(aq) + H2O(l)
S

before (mol) 0.050 0.002 0.025


change (mol) S0.002 S0.002 +0.002
after (mol) 0.048 0 0.027
[HF]  0.27 
[H3O+] = K a
S4
= (3.5 x 10 _ 4)   = 1.97 x 10 M
 0.48 
_
[F ]
pH = Slog[H3O+] = Slog(1.97 x 10S4) = 3.71

S
(b) mol HF = 0.025 mol; mol F = 0.050 mol; vol = 0.100 L

434
Chapter 16 S Applications of Aqueous Equilibria
______________________________________________________________________________
100%
HF(aq) + OH (aq)  F (aq) + H2O(l)
S S

before (mol) 0.025 0.004 0.050


change (mol) S0.004 S0.004 +0.004
after (mol) 0.021 0 0.054
[HF]  0.21 
[H3O+] = K a = (3.5 x 10 _ 4) 
S4
 = 1.36 x 10 M
 0.54 
_
[F ]
pH = Slog[H3O ] = Slog(1.36 x 10 4) = 3.87
+ S

HF(aq) + H2O(l) Ω H3O+(aq) + F (aq)


S
16.8
initial (M) 0.050 ~0 0.100
change (M) Sx +x +x
equil (M) 0.050 S x x 0.100 + x
[H 3O + ][F _ ] x(0.100 + x) x(0.100)
Ka = = 3.5 x 10 _ 4 = ≈
[HF] 0.050 _ x 0.050
+ S4
Solve for x. x = [H3O ] = 1.75 x 10 M
pH = Slog[H3O+] = Slog(1.75 x 10 4) = 3.76
S

mol HF = 0.050 mol/L x 0.100 L = 0.0050 mol HF


mol FS = 0.100 mol/L x 0.100 L = 0.0100 mol FS
mol HNO3 = mol H3O+ = 0.002 mol
100%
Neutralization reaction: FS(aq) + H3O+(aq)  HF(aq) + H2O(l)
before reaction (mol) 0.0100 0.002 0.0050
change (mol) S0.002 S0.002 +0.002
after reaction (mol) 0.008 0 0.007
0.007 mol S 0.008 mol
[HF] = = 0.07 M; [F ] = = 0.08 M
0.100 L 0.100 L
[HF] (0.07)
[H3O+] = Ka _
= (3.5 x 10 _ 4) = 3 x 10S4 M
[F ] (0.08)
+ S4
pH = Slog[H3O ] = Slog(3 x 10 ) = 3.5
This solution has less buffering capacity than the solution in Problem 16.7 because it
contains less HF and FS per 100 mL. Note that the change in pH is greater than that in
Problem 16.7.

16.9 When equal volumes of two solutions are mixed together, the concentration of each
solution is cut in half.
[base] [CO 32 _ ]
pH = pKa + log = pK a + log
[acid] [HCO 3_ ]
For HCO3 , Ka = 5.6 x 10 11, pKa = Slog Ka = Slog(5.6 x 10 11) = 10.25
S S S

 0.050 
pH = 10.25 + log   = 10.25 S 0.30 = 9.95
 0.10 
[base] [CO 32 _ ]
16.10 pH = pKa + log = pK a + log
[acid] [HCO 3_ ]

435
Chapter 16 S Applications of Aqueous Equilibria
______________________________________________________________________________

For HCO3 , Ka = 5.6 x 10 11, pKa = Slog Ka = Slog(5.6 x 10 11) = 10.25


S S S

[ 2_ ] [ 2_ ]
10.40 = 10.25 + log CO3 _ ; log CO3 _ = 10.40 S 10.25 = 0.15
[HCO3 ] [HCO3 ]
2_
[CO 3 ]
_
= 100.15 = 1.4
[HCO 3 ]
To obtain a buffer solution with pH 10.40, make the Na2CO3 concentration 1.4 times the
concentration of NaHCO3.

16.11 Look for an acid with pKa near the required pH of 7.50.
Ka = 10 pH = 10 7.50 = 3.2 x 10 8
S S S

Suggested buffer system: HOCl (Ka = 3.5 x 10 8) and NaOCl.


S

 66 
16.12 (a) serine is 66% dissociated at pH = 9.15 + log   = 9.44
 34 
 5 
(b) serine is 5% dissociated at pH = 9.15 + log   = 7.87
 95 

16.13 (a) mol HCl = mol H3O+ = 0.100 mol/L x 0.0400 L = 0.004 00 mol
S
mol NaOH = mol OH = 0.100 mol/L x 0.0350 L = 0.003 50 mol
H3O+(aq) + OH (aq)  2 H2O(l)
S
Neutralization reaction:
before reaction (mol) 0.004 00 0.003 50
change (mol) S0.003 50 S0.003 50
after reaction (mol) 0.000 50 0
0.000 50 mol
[H3O+] = = 6.7 x 10S3 M
(0.0400 L + 0.0350 L)
pH = Slog[H3O+] = Slog(6.7 x 10 3) = 2.17
S

(b) mol HCl = mol H3O+ = 0.100 mol/L x 0.0400 L = 0.004 00 mol
mol NaOH = mol OHS = 0.100 mol/L x 0.0450 L = 0.004 50 mol
H3O+(aq) + OH (aq)  2 H2O(l)
S
Neutralization reaction:
before reaction (mol) 0.004 00 0.004 50
change (mol) S0.004 00 S0.004 00
after reaction (mol) 0 0.000 50
0.000 50 mol
= 5.9 x 10 3 M
S S
[OH ] =
(0.0400 L + 0.0450 L)
1.0 x 10 _14
[H3O+] = K w_ = = 1.7 x 10S12 M
[OH ] 5.9 x 10 _ 3
pH = Slog[H3O+] = Slog(1.7 x 10 12) = 11.77
S

The results obtained here are consistent with the pH data in Table 16.1
16.14 (a) mol NaOH = mol OHS = 0.100 mol/L x 0.0400 L = 0.004 00 mol
mol HCl = mol H3O+ = 0.0500 mol/L x 0.0600 L = 0.003 00 mol
H3O+(aq) + OH (aq)  2 H2O(l)
S
Neutralization reaction:
before reaction (mol) 0.003 00 0.004 00
change (mol) S0.003 00 S0.003 00

436
Chapter 16 S Applications of Aqueous Equilibria
______________________________________________________________________________

after reaction (mol) 0 0.001 00


0.001 00 mol
= 1.0 x 10 2 M
S S
[OH ] =
(0.0400 L + 0.0600 L)
1.0 x 10 _14
[H3O+] = K w_ = = 1.0 x 10 12 M
S
_2
[OH ] 1.0 x 10
pH = Slog[H3O+] = Slog(1.0 x 10 12) = 12.00
S

S
(b) mol NaOH = mol OH = 0.100 mol/L x 0.0400 L = 0.004 00 mol
mol HCl = mol H3O+ = 0.0500 mol/L x 0.0802 L = 0.004 01 mol
H3O+(aq) + OH (aq)  2 H2O(l)
S
Neutralization reaction:
before reaction (mol) 0.004 01 0.004 00
change (mol) S0.004 00 S0.004 00
after reaction (mol) 0.000 01 0
0.000 01 mol
[H3O+] = = 8.3 x 10 5 M
S

(0.0400 L + 0.0802 L)
pH = Slog[H3O+] = Slog(8.3 x 10 5) = 4.08
S

S
(c) mol NaOH = mol OH = 0.100 mol/L x 0.0400 L = 0.004 00 mol
+
mol HCl = mol H3O = 0.0500 mol/L x 0.1000 L = 0.005 00 mol
Neutralization reaction: H3O+(aq) + OHS(aq)  2 H2O(l)
before reaction (mol) 0.005 00 0.004 00
change (mol) S0.004 00 S0.004 00
after reaction (mol) 0.001 00 0
0.001 00 mol
[H3O+] = = 7.1 x 10 3 M
S

(0.0400 L + 0.1000 L)
pH = Slog[H3O+] = Slog(7.1 x 10S3) = 2.15

16.15 (a) (3), only HA present (b) (1), HA and AS present


S S S
(c) (4), only A present (d) (2), A and OH present

 0.016 mol HOCl   1 mol NaOH 


16.16 mol NaOH required =   (0.100 L)   = 0.0016 mol
 L   1 mol HOCl 
 1L 
vol NaOH required = (0.0016 mol)   = 0.040 L = 40 mL
 0.0400 mol 
40 mL of 0.0400 M NaOH are required to reach the equivalence point.
(a) mmol HOCl = 0.016 mmol/mL x 100.0 mL = 1.6 mmol
mmol NaOH = mmol OHS = 0.0400 mmol/mL x 10.0 mL = 0.400 mmol
HOCl(aq) + OH (aq)  OCl (aq) + H2O(l)
S S
Neutralization reaction:
before reaction (mmol) 1.6 0.400 0
change (mmol) S0.400 S0.400 +0.400
after reaction (mmol) 1.2 0 0.400
1.2 mmol
= 1.09 x 10 2 M
S
[HOCl] =
(100.0 mL + 10.0 mL)
0.400 mmol
[OClS] = = 3.64 x 10S3 M
(100.0 mL + 10.0 mL)

437
Chapter 16 S Applications of Aqueous Equilibria
______________________________________________________________________________

HOCl(aq) + H2O(l) Ω H3O+(aq) + OCl (aq)


S

initial (M) 0.0109 ~0 0.003 64


change (M) Sx +x +x
equil (M) 0.0109 S x x 0.003 64 + x
+ _
[ ][ ] x(0.003 64 + x) x(0.003 64)
Ka = H 3O OCl = 3.5 x 10 _ 8 = ≈
[HOCl] 0.0109 _ x 0.0109
Solve for x. x = [H3O+] = 1.05 x 10 7 M
S

pH = Slog[H3O+] = Slog(1.05 x 10 7) = 6.98


S

S
(b) Halfway to the equivalence point, [OCl ] = [HOCl]
S8
pH = pKa = Slog Ka = Slog(3.5 x 10 ) = 7.46
(c) At the equivalence point the solution contains the salt, NaOCl.
mol NaOCl = initial mol HOCl = 0.0016 mol = 1.6 mmol
1.6 mmol
[OClS] = = 1.1 x 10S2 M
(100.0 mL + 40.0 mL)
Kw 1.0 x 10 _14
= 2.9 x 10 7
S S
For OCl , Kb = = _8
K a for HOCl 3.5 x 10
OClS(aq) + H2O(l) Ω HOCl(aq) + OHS(aq)
initial (M) 0.011 0 ~0
change (M) Sx +x +x
equil (M) 0.011 S x x x
_ 2 2
[HOCl][OH ] x
Kb = = 2.9 x 10 _ 7 = ≈ x
[OCl _ ] 0.011 _ x 0.011
Solve for x. x = [OHS] = 5.65 x 10S5 M
1.0 x 10 _14
[H3O+] = K w_ = _5
= 1.77 x 10S10 = 1.8 x 10S10 M
[OH ] 5.65 x 10
pH = Slog[H3O+] = Slog(1.77 x 10 10) = 9.75
S

16.17 From Problem 16.16, pH = 9.75 at the equivalence point.


Use thymolphthalein (pH 9.4 S 10.6). Bromthymol blue is unacceptable because it
changes color halfway to the equivalence point.

16.18 (a) mol NaOH required to reach first equivalence point


 0.0800 mol H 2SO 3   1 mol NaOH 
=  (0.0400 L)   = 0.003 20 mol
 L   1 mol H 2SO 3 

vol NaOH required to reach first equivalence point


 1L 
= (0.003 20 mol)   = 0.020 L = 20.0 mL
 0.160 mol 
20.0 mL is enough NaOH solution to reach the first equivalence point for the titration of
the diprotic acid, H2SO3.

438
Chapter 16 S Applications of Aqueous Equilibria
______________________________________________________________________________

For H2SO3,
_2 _2
K a1 = 1.5 x 10 , pK a1 = _log K a1 = _log(1.5 x 10 ) = 1.82
_8 _8
K a 2 = 6.3 x 10 , pK a 2 = _log K a 2 = _log(6.3 x 10 ) = 7.20
pK a1 + pK a 2 1.82 + 7.20
At the first equivalence point, pH = = = 4.51
2 2
(b) mol NaOH required to reach second equivalence point
 0.0800 mol H 2SO 3   2 mol NaOH 
=  (0.0400 L)   = 0.006 40 mol
 L   1 mol H 2SO 3 
vol NaOH required to reach second equivalence point
 1L 
= (0.006 40 mol)   = 0.040 L = 40.0 mL
 0.160 mol 
30.0 mL is enough NaOH solution to reach halfway to the second equivalent point.
Halfway to the second equivalence point
pH = p K a 2 = _log K a 2 = _log(6.3 x 10 _ 8) = 7.20
S
(c) mmol HSO3 = 0.0800 mmol/mL x 40.0 mL = 3.20 mmol
volume NaOH added after first equivalence point = 35.0 mL S 20.0 mL = 15.0 mL
mmol NaOH = mmol OHS = 0.160 mmol/L x 15.0 mL = 2.40 mmol
HSO3 (aq) + OH (aq) Ω SO32 (aq) + H2O(l)
S S S
Neutralization reaction:
before reaction (mmol) 3.20 2.40 0
change (mmol) S2.40 S2.40 +2.40
after reaction (mmol) 0.80 0 2.40
0.80 mmol
[HSO3S] = = 0.0107 M
(40.0 mL + 35.0 mL)
2.40 mmol
[SO32 ] =
S
= 0.0320 M
(40.0 mL + 35.0 mL)
HSO3 (aq) + H2O(l) Ω H3O+(aq) + SO32 (aq)
S S

initial (M) 0.0107 ~0 0.0320


change (M) Sx +x +x
equil (M) 0.0107 S x x 0.0320 + x
[ +
][ 32 _ ] x(0.0320 + x) x(0.0320)
Ka = H 3O SO _
= 6.3 x 10 _ 8 = ≈
[HSO 3 ] 0.0107 _ x 0.0107
+ S8
Solve for x. x = [H3O ] = 2.1 x 10 M
pH = Slog[H3O+] = Slog(2.1 x 10 8) = 7.68
S

16.19 Let H2A+ = valine cation


(a) mol NaOH required to reach first equivalence point
 0.0250 mol H 2 A +   1 mol NaOH 
=  (0.0400 L)  + 
= 0.001 00 mol
 L   1 mol H 2 A 
vol NaOH required to reach first equivalence point

439
Chapter 16 S Applications of Aqueous Equilibria
______________________________________________________________________________

 1L 
= (0.001 00 mol)   = 0.0100 L = 10.0 mL
 0.100 mol 
10.0 mL is enough NaOH solution to reach the first equivalence point for the titration of
the diprotic acid, H2A+.
For H2A+,
_3 _3
K a1 = 4.8 x 10 , pK a1 = _ log K a1 = _ log(4.8 x 10 ) = 2.32
_10 _10
K a 2 = 2.4 x 10 , pK a 2 = _ log K a 2 = _ log(2.4 x 10 ) = 9.62
pK a1 + pK a 2 2.32 + 9.62
At the first equivalence point, pH = = = 5.97
2 2
(b) mol NaOH required to reach second equivalence point
 0.0250 mol H 2 A +   2 mol NaOH 
=  (0.0400 L)  + 
= 0.002 00 mol
 L   1 mol H 2 A 
vol NaOH required to reach second equivalence point
 1L 
= (0.002 00 mol)   = 0.0200 L = 20.0 mL
 0.100 mol 
15.0 mL is enough NaOH solution to reach halfway to the second equivalent point.
Halfway to the second equivalence point
pH = p K a 2 = _ log K a 2 = _ log(2.4 x 10 _10) = 9.62
(c) 20.0 mL is enough NaOH to reach the second equivalence point.
At the second equivalence point
S S
mmol A = (0.0250 mmol/mL)(40.0 mL) = 1.00 mmol A
solution volume = 40.0 mL + 20.0 mL = 60.0 mL
S 1.00 mmol
[A ] = = 0.0167 M
60.0 mL
A (aq) + H2O(l) Ω HA(aq) + OH (aq)
S S

initial (M) 0.0167 0 ~0


change (M) Sx +x +x
equil (M) 0.0167 S x x x
_14
Kw 1.0 x 10
Kb = = Kw = _10
= 4.17 x 10S5
K a for HA Ka 2 2.4 x 10
_ 2
[HA][OH ] _5 x
Kb = = 4.17 x 10 =
[A _ ] 0.0167 _ x
2 S5 S7
x + (4.17 x 10 )x S (6.964 x 10 ) = 0
Use the quadratic formula to solve for x.

x=
_ (4.17 x 10 _ 5) ± (4.17 x 10 _ 5) 2 _ (4)(1)(_ 6.964 x 10 _ 7) (_ 4.17 x 10 _ 5) ± (1.67 x 10 _ 3)
=
2(1) 2
S4 S4
x = 8.14 x 10 and S8.56 x 10
Of the two solutions for x, only the positive value has physical meaning because x is the
S
[OH ].

440
Chapter 16 S Applications of Aqueous Equilibria
______________________________________________________________________________

x = [OH ] = 8.14 x 10 4 M
S S

1.0 x 10 _14
[H3O+] = K w_ = _4
= 1.23 x 10S11 M
[OH ] 8.14 x 10
pH = Slog[H3O+] = Slog(1.23 x 10 11) = 10.91
S

(a) Ksp = [Ag+][Cl ] (b) Ksp = [Pb2+][I ]2


S S
16.20
(c) Ksp = [Ca2+]3[PO43S]2 (d) Ksp = [Cr3+][OHS]3

Ksp = [Ca2+]3[PO43 ]2 = (2.01 x 10 8)3(1.6 x 10 5)2 = 2.1 x 10


S S S S33
16.21

[Ba2+] = [SO42 ] = 1.05 x 10 5 M; Ksp = [Ba2+][SO42 ] = (1.05 x 10 5)2 = 1.10 x 10 10


S S S S S
16.22

AgCl(s) Ω Ag+(aq) + Cl (aq)


S
16.23 (a)
equil (M) x x
Ksp = [Ag+][Cl ] = 1.8 x 10 10 = (x)(x)
S S

molar solubility = x = K sp = 1.3 x 10 5 mol/L


S

AgCl, 143.32 amu


 _5 143.32 g 
1.3 x 10 mol x 
solubility = 
1 mol 
= 0.0019 g/L
1L

(b) Ag2CrO4(s) Ω 2 Ag+(aq) + CrO42S(aq)


equil (M) 2x x
Ksp = [Ag ] [CrO4 ] = 1.1 x 10 = (2x) (x) = 4x3
+ 2 2S S12 2

1.1 x 10 _12
molar solubility = x = 3 = 6.5 x 10S5 mol/L
4
Ag2CrO4, 331.73 amu
 _5 331.73 g 
 6.5 x 10 mol x 
solubility = 
1 mol 
= 0.022 g/L
1L
Ag2CrO4 has both the higher molar and gram solubility, despite its smaller value of Ksp.

16.24 Let the number of ions be proportional to its concentration.


For AgX, Ksp = [Ag+][XS] % (4)(4) = 16
For AgY, Ksp = [Ag+][Y ] % (1)(9) = 9
S

For AgZ, Ksp = [Ag ][Z ] % (3)(6) = 18


+ S

(a) AgZ (b) AgY

16.25 [Mg2+]0 is from 0.10 M MgCl2.


MgF2(s) Ω Mg2+(aq) + 2 F (aq)
S

initial (M) 0.10 0


change (M) +x +2x
equil (M) 0.10 + x 2x

441
Chapter 16 S Applications of Aqueous Equilibria
______________________________________________________________________________

Ksp = 7.4 x 10S11 = [Mg2+][FS]2 = (0.10 + x)(2x)2 . (0.10)(4x2)


x = 1.4 x 10S5, molar solubility = x = 1.4 x 10S5 M

16.26 Compounds that contain basic anions are more soluble in acidic solution than in pure
water. AgCN, Al(OH)3, and ZnS all contain basic anions.

[Cu2+] = (5.0 x 10 3 mol)/(0.500 L) = 0.010 M


S
16.27
Cu2+(aq) + 4 NH3(aq) Ω Cu(NH3)42+(aq)
before reaction (M) 0.010 0.40 0
assume 100 % reaction (M) S0.010 S 4(0.010) +0.010
after reaction (M) 0 0.36 0.010
assume small back reaction (M) +x +4x Sx
equil (M) x 0.36 + 4x 0.010 S x
2+
[Cu( NH 3) 4 ] (0.010 _ x) 0.010
Kf = 4
= 5.6 x 1011 = ≈
2+
[Cu ][ NH 3] (x)(0.36 + 4 x ) x(0.36) 4
4

Solve for x. x = [Cu2+] = 1.1 x 10 12 M


S

AgBr(s) Ω Ag+(aq) + Br (aq) Ksp = 5.4 x 10 13


S S
16.28
Ag (aq) + 2 S2O3  Ag(S2O3)2 (aq)
+ 2S 3S
Kf = 4.7 x 1013
AgBr(s) + 2 S2O32 (aq) Ω Ag(S2O3)23 (aq) + Br (aq)
S S S
dissolution
reaction
K = (Ksp)(Kf) = (5.4 x 10S13)(4.7 x 1013) = 25.4
AgBr(s) + 2 S2O32 (aq) Ω Ag(S2O3)23 (aq) + Br (aq)
S S S

initial (M) 0.10 0 0


change (M) S2x x x
equil (M) 0.10 S 2x x x
3_ _ 2
[Ag(S2O 3) 2 ][Br ] x
K= = 25.4 =
[S2O32 _ ]2 (0.10 _ 2 x ) 2
Take the square root of both sides and solve for x.
2
x x
25.4 = ; 5.04 = ; x = molar solubility = 0.045 mol/L
(0.10 _ 2 x ) 2 0.10 _ 2 x

16.29 On mixing equal volumes of two solutions, the concentrations of both solutions are cut
in half.
For BaCO3, Ksp = 2.6 x 10 9
S

(a) IP = [Ba2+][CO32S] = (1.5 x 10S3)(1.0 x 10S3) = 1.5 x 10S6


IP > Ksp; a precipitate of BaCO3 will form.
(b) IP = [Ba2+][CO32 ] = (5.0 x 10 6)(2.0 x 10 5) = 1.0 x 10 10
S S S S

IP < Ksp; no precipitate will form.

[base] [ NH 3]
16.30 pH = pKa + log = pK a + log
[acid] [ NH +4 ]

442
Chapter 16 S Applications of Aqueous Equilibria
______________________________________________________________________________

For NH4+, Ka = 5.6 x 10 10, pKa = Slog Ka = Slog(5.6 x 10 10) = 9.25


S S

(0.20)
= 9.25; [H3O+] = 10 pH = 10 9.25 = 5.6 x 10 10 M
S S S
pH = 9.25 + log
(0.20)
1.0 x 10 _14
[OH ] = K w + = = 1.8 x 10 5 M
S S
_10
[H 3O ] 5.6 x 10
(25 mL)(1.0 x 10 _ 3 M)
[Fe2+] = [Mn2+] = = 1.0 x 10 4 M
S

250 mL
For Mn(OH)2, Ksp = 2.1 x 10S13
IP = [Mn2+][OHS]2 = (1.0 x 10S4)(1.8 x 10S5)2 = 3.2 x 10S14
IP < Ksp; no precipitate will form.
For Fe(OH)2, Ksp = 4.9 x 10 17
S

IP = [Fe2+][OH ]2 = (1.0 x 10 4)(1.8 x 10 5)2 = 3.2 x 10 14


S S S S

IP > Ksp; a precipitate of Fe(OH)2 will form.

16.31 MS(s) + 2 H3O+(aq) Ω M2+(aq) + H2S(aq) + 2 H2O(l)


[ 2+ ][H 2 S]
Kspa = M
[H 3O + ]2
For ZnS, Kspa = 3 x 10 2; for CdS, Kspa = 8 x 10 7
S S

2+ 2+
[Cd ] = [Zn ] = 0.005 M
Because the two cation concentrations are equal, Qc is the same for both.
[M 2+]t[H 2 S ]t (0.005)(0.10)
Qc = + 2
= 2
= 6 x 10S3
[H 3O ]t (0.3)
Qc > Kspa for CdS; CdS will precipitate. Qc < Kspa for ZnS; Zn2+ will remain in
solution.
S
16.32 This protein has both acidic and basic sites. H3PO4-H2PO4 is an acidic buffer. It
protonates the basic sites in the protein making them positive and the protein migrates
S
towards the negative electrode. H3BO3-H2BO3 is a basic buffer. At basic pH's, the
acidic sites in the protein are dissociated making them negative and the protein migrates
towards the positive electrode.

16.33 To increase the rate at which the proteins migrates toward the negative electrode,
increase the number of basic sites that are protonated by lowering the pH. Decrease the
[HPO42 ]/[H2PO4 ] ratio (less HPO42 , more H2PO4 ) to lower the pH.
S S S S

Understanding Key Concepts

16.34 A buffer solution contains a conjugate acid-base pair in about equal concentrations.
(a) (1), (3), and (4)
(b) (4) because it has the highest buffer concentration.

443
Chapter 16 S Applications of Aqueous Equilibria
______________________________________________________________________________
S
16.35 (a) (2) has the highest pH, [A ] > [HA]
(3) has the lowest pH, [HA] > [AS]

(b) (c)

S
16.36 (4); only A and water should be present

16.37 (a) (1) corresponds to (iii); (2) to (i); (3) to (iv); and (4) to (ii)
(b) Solution (3) has the highest pH; solution (2) has the lowest pH.

16.38 (a) (i) (1), only B present (ii) (4), equal amounts of B and BH+ present
(iii) (3), only BH+ present (iv) (2), BH+ and H3O+ present
(b) The pH is less than 7 because BH+ is an acidic cation.

16.39 (2) is supersaturated; (3) is unsaturated; (4) is unsaturated

16.40 Let the number of ions be proportional to its concentration.


For Ag2CrO4, Ksp = [Ag+]2[CrO42 ] % (4)2(2) = 32
S

For (2), IP = [Ag ] [CrO4 ] % (2)2(4) = 16


+ 2 2S

For (3), IP = [Ag+]2[CrO42S] % (6)2(2) = 72


For (4), IP = [Ag+]2[CrO42 ] % (2)2(6) = 24
S

A precipitate will form when IP > Ksp. A precipitate will form only in (3).

16.41 (a) The lower curve represents the titration of a strong acid; the upper curve represents
the titration of a weak acid.
(b) pH = 7 for titration of the strong acid; pH = 10 for titration of the weak acid.
(c) Halfway to the equivalence point, the pH = pKa ~ 6.3.

Additional Problems
Neutralization Reactions

16.42 (a) HI(aq) + NaOH(aq)  H2O(l) + NaI(aq)


net ionic equation: H3O+(aq) + OH (aq)  2 H2O(l)
S

The solution at neutralization contains a neutral salt (NaI); pH = 7.00.

(b) 2 HOCl(aq) + Ba(OH)2(aq)  2 H2O(l) + Ba(OCl)2(aq)

444
Chapter 16 S Applications of Aqueous Equilibria
______________________________________________________________________________

net ionic equation: HOCl(aq) + OHS(aq)  H2O(l) + OClS(aq)


The solution at neutralization contains a basic anion (OClS); pH > 7.00
(c) HNO3(aq) + C6H5NH2(aq)  C6H5NH3NO3(aq)
net ionic equation: H3O+(aq) + C6H5NH2(aq)  H2O(l) + C6H5NH3+(aq)
The solution at neutralization contains an acidic cation (C6H5NH3+); pH < 7.00.
(d) C6H5CO2H(aq) + KOH(aq)  H2O(l) + C6H5CO2K(aq)
net ionic equation: C6H5CO2H(aq) + OH (aq)  H2O(l) + C6H5CO2 (aq)
S S

S
The solution at neutralization contains a basic anion (C6H5CO2 ); pH > 7.00.

16.43 (a) HNO2(aq) + CsOH(aq)  H2O(l) + CsNO2(aq)


net ionic equation: HNO2(aq) + OH (aq)  H2O(l) + NO2 (aq)
S S

S
The solution at neutralization contains a basic anion (NO2 ); pH > 7.00
(b) HBr(aq) + NH3(aq)  NH4Br(aq)
net ionic equation: H3O+(aq) + NH3(aq)  H2O(l) + NH4+(aq)
The solution at neutralization contains an acidic cation (NH4+); pH < 7.00
(c) HClO4(aq) + KOH(aq)  H2O(l) + KClO4(aq)
net ionic equation: H3O+(aq) + OHS(aq)  2 H2O(l)
The solution at neutralization contains a neutral salt (KClO4); pH = 7.00
(d) HOBr(aq) + NH3(aq)  NH4OBr(aq)
net ionic equation: HOBr(aq) + NH3(aq)  NH4+(aq) + OBr (aq)
S

The solution at neutralization contains the salt NH4OBr.


Ka(NH4+) = 5.6 x 10S10 and Kb(OBrS) = 5.0 x 10S5; Kb(OBrS) > Ka(NH4+); pH > 7.00

1 1
16.44 (a) Strong acid - strong base reaction Kn = = = 1.0 x 1014
Kw 1.0 x 10 _14
3.5 x 10 _ 8
(b) Weak acid - strong base reaction Kn = K a = _14
= 3.5 x 106
K w 1.0 x 10
4.3 x 10 _10
(c) Strong acid - weak base reaction Kn = K b = _14
= 4.3 x 104
K w 1.0 x 10
6.5 x 10 _ 5
(d) Weak acid - strong base reaction Kn = K a = _14
= 6.5 x 109
Kw 1.0 x 10
(c) < (b) < (d) < (a)

4.5 x 10 _ 4
16.45 (d) Weak acid - strong base reaction Kn = K a = _14
= 4.5 x 1010
Kw 1.0 x 10
1.8 x 10 _ 5
(c) Strong acid - weak base reaction Kn = K b = _14
= 1.8 x 109
K w 1.0 x 10
1 1
(a) Strong acid - strong base reaction Kn = = _14
= 1.0 x 1014
K w 1.0 x 10
(2.0 x 10 _ 9)(1.8 x 10 _ 5)
(d) Weak acid - weak base reaction Kn = K a K b = =
Kw 1.0 x 10 _14
3.6

445
Chapter 16 S Applications of Aqueous Equilibria
______________________________________________________________________________

(d) < (b) < (a) < (c)

16.46 (a) After mixing, the solution contains the basic salt, NaF; pH > 7.00
(b) After mixing, the solution contains the neutral salt, NaCl; pH = 7.00
Solution (a) has the higher pH.

16.47 (a) After mixing, the solution contains the neutral salt, NaClO4; pH = 7.00
(b) After mixing, the solution contains the acidic salt, NH4ClO4; pH < 7.00
Solution (b) has the lower pH.

(1.3 x 10 _10)(1.8 x 10 _ 9)
16.48 Weak acid - weak base reaction Kn = K a K boverK w = =
1.0 x 10 _14
2.3 x 10 5
S

Kn is small so the neutralization reaction does not proceed very far to completion.

(8.0 x 10 _ 5)(4.3 x 10 _10)


16.49 Weak acid - weak base reaction Kn = K a K b = = 3.4
Kw 1.0 x 10 _14
Because Kn is close to 1, there will be an appreciable amount of aniline present at
equilibrium.

The CommonSIon Effect

16.50 HNO2(aq) + H2O(l) Ω H3O+(aq) + NO2S(aq)


S
(a) NaNO2 is a source of NO2 (reaction product). The equilibrium shifts towards
reactants, and the percent dissociation of HNO2 decreases.
(c) HCl is a source of H3O+ (reaction product). The equilibrium shifts towards reactants,
and the percent dissociation of HNO2 decreases.
(d) Ba(NO2)2 is a source of NO2S (reaction product). The equilibrium shifts towards
reactants, and the percent dissociation of HNO2 decreases.

16.51 NH3(aq) + H2O(l) Ω NH4+(aq) + OHS(aq)


(a) KOH is a strong base, and it increases the [OHS]. The pH increases.
(b) NH4NO3 is a source of NH4+ (reaction product). The equilibrium shifts towards
S
reactants, and the [OH ] decreases. The pH decreases.
(c) NH4Br is a source of NH4+ (reaction product). The equilibrium shifts towards
S
reactants, and the [OH ] decreases. The pH decreases.
(d) KBr does not affect the pH of the solution.

(a) HF(aq) + H2O(l) Ω H3O+(aq) + F (aq)


S
16.52
LiF is a source of FS (reaction product). The equilibrium shifts toward reactants, and the
[H3O+] decreases. The pH increases.
(b) Because HI is a strong acid, addition of KI, a neutral salt, does not change the pH.
(c) NH3(aq) + H2O(l) Ω NH4+(aq) + OHS(aq)
NH4Cl is a source of NH4+ (reaction product). The equilibrium shifts toward reactants,

446
Chapter 16 S Applications of Aqueous Equilibria
______________________________________________________________________________
S
and the [OH ] decreases. The pH decreases.

(a) NH3(aq) + H2O(l) Ω NH4+(aq) + OH (aq)


S
16.53
NH4Cl is a source of NH4+ (reaction product). The equilibrium shifts toward reactants,
and the [OHS] decreases. The pH decreases.
(b) HCO3 (aq) + H2O(l) Ω H3O+(aq) + CO32 (aq)
S S

2S
Na2CO3 is a source of CO3 (reaction product). The equilibrium shifts toward reactants,
and the [H3O+] decreases. The pH increases.
(c) Because NaOH is a strong base, addition of NaClO4, a neutral salt, does not change
the pH.

16.54 For 0.25 M HF and 0.10 M NaF


HF(aq) + H2O(l) Ω H3O+(aq) + F (aq)
S

initial (M) 0.25 ~0 0.10


change (M) Sx +x +x
equil (M) 0.25 S x x 0.10 + x
+ _
[ ][ ] x(0.10 + x) x(0.10)
Ka = H 3O F = 3.5 x 10 _ 4 = ≈
[HF] 0.25 _ x 0.25
+ S4
Solve for x. x = [H3O ] = 8.8 x 10 M
pH = Slog[H3O+] = Slog(8.8 x 10 4) = 3.06
S

16.55 On mixing equal volumes of two solutions, both concentrations are cut in half.
[CH3NH2] = 0.10 M; [CH3NH3Cl] = 0.30 M
CH3NH2(aq) + H2O(l) Ω CH3NH3+(aq) + OH (aq)
S

initial (M) 0.10 0.30 ~0


change (M) Sx +x +x
equil (M) 0.10 S x 0.30 + x x
+ _
[ ][ ] (0.30 + x) x (0.30) x
Kb = CH 3 NH 3 OH = 3.7 x 10 _ 4 = ≈
[CH 3 NH 2] 0.10 _ x 0.10
S S4
Solve for x. x = [OH ] = 1.2 x 10 M
1.0 x 10 _14
[H3O+] = K w_ = _4
= 8.1 x 10S11 M
[OH ] 1.2 x 10
pH = Slog[H3O+] = Slog(8.1 x 10 11) = 10.09
S

16.56 For 0.10 M HN3:


HN3(aq) + H2O(l) Ω H3O+(aq) + N3S(aq)
initial (M) 0.10 ~0 0
change (M) Sx +x +x
equil (M) 0.10 S x x x
[H 3O + ][ N 3_ ] x
2 2
Ka = = 1.9 x 10 _ 5 = ≈ x
[HN 3] 0.10 _ x 0.10
Solve for x. x = 1.4 x 10S3 M

447
Chapter 16 S Applications of Aqueous Equilibria
______________________________________________________________________________

[HN 3]diss 1.4 x 10 _ 3 M


% dissociation = x 100% = x 100% = 1.4%
[HN 3]initial 0.10 M
For 0.10 M HN3 in 0.10 M HCl:
HN3(aq) + H2O(l) Ω H3O+(aq) + N3 (aq)
S

initial (M) 0.10 0.10 0


change (M) Sx +x +x
equil (M) 0.10 S x 0.10 + x x
[H 3O + ][ N 3_ ] (0.10 + x)(x) (0.10)(x)
Ka = = 1.9 x 10 _ 5 = ≈ =x
[HN 3] 0.10 _ x 0.10
Solve for x. x = 1.9 x 10 5 M
S

[HN 3]diss 1.9 x 10 _ 5 M


% dissociation = x 100% = x 100% = 0.019%
[HN 3]initial 0.10 M
The % dissociation is less because of the common ion (H3O+) effect.

NH3(aq) + H2O(l) Ω NH4+(aq) +


S
16.57 OH (aq)
initial (M) 0.30 0 ~0
change (M) Sx +x +x
equil (M) 0.30 S x x x
+ _ 2 2
[ ][ ]
Kb = NH 4 OH = 1.8 x 10 _ 5 = x ≈ x
[ NH 3] 0.30 _ x 0.30
S S3
Solve for x. x = [OH ] = 2.3 x 10 M
1.0 x 10 _14
[H3O+] = K w_ = = 4.3 x 10 12 M
S
_3
[OH ] 2.3 x 10
pH = Slog[H3O+] = Slog(4.3 x 10S12) = 11.37
Add 4.0 g of NH4NO3.
 1 mol 
 4.0 g x 
[NH4+] = molarity of NH4NO3 = 
80.04 g 
NH4NO3, 80.04 amu; = 0.50 M
0.100 L

NH3(aq) + H2O(l) Ω NH4+(aq) + OH (aq)


S

initial (M) 0.30 0.50 ~0


change (M) Sx +x +x
equil (M) 0.30 S x 0.50 + x x
+ _
[ ][ ] (0.50 + x) x (0.50) x
Kb = NH 4 OH = 1.8 x 10 _ 5 = ≈
[ NH 3] 0.30 _ x 0.30
S S5
Solve for x. x = [OH ] = 1.1 x 10 M
1.0 x 10 _14
[H3O+] = K w_ = = 9.1 x 10 10 M
S
_5
[OH ] 1.1 x 10
pH = Slog[H3O+] = Slog(9.1 x 10 10) = 9.04
S

The % dissociation decreases because of the common ion effect.

Buffer Solutions

448
Chapter 16 S Applications of Aqueous Equilibria
______________________________________________________________________________

16.58 Solutions (a), (c) and (d) are buffer solutions. Neutralization reactions for (c) and (d)
result in solutions with equal concentrations of HF and FS.

16.59 Solutions (b), (c) and (d) are buffer solutions. Neutralization reactions for (b) and (d)
result in solutions with equal concentrations of NH3 and NH4+.
S
16.60 Both solutions buffer at the same pH because in both cases the [NO2 ]/[HNO2] = 1.
Solution (a), however, has a higher concentration of both HNO2 and NO2S, and therefore
it has the greater buffer capacity.

16.61 Both solutions buffer at the same pH because in both cases the [NH3]/[NH4+] = 1.5.
Solution (b), however, has a higher concentration of both NH3 and NH4+, and therefore it
has the greater buffer capacity.

16.62 When blood absorbs acid, the equilibrium shifts to the left, decreasing the pH, but not by
much because the [HCO3S]/[H2CO3] ratio remains nearly constant. When blood absorbs
base, the equilibrium shifts to the right, increasing the pH, but not by much because the
S
[HCO3 ]/[H2CO3] ratio remains nearly constant.

H2PO4 (aq) + H2O(l) Ω H3O+(aq) + HPO42 (aq)


S S
16.63
For H2PO4S, K a 2 = 6.2 x 10 _ 8, pK a 2 = _log K a 2 = 7.21
[HPO 24 _ ] [HPO 24 _ ]
pH = 7.4 = p K a 2 + log = 7.21 + log
[H 2 PO 4_ ] [H 2 PO 4_ ]
2_ 2_
[ ] [ ]
To maintain pH near 7.4, need log HPO 4 _ = 0.19 and HPO 4 _ = 100.19 = 1.5
[H 2 PO 4 ] [H 2 PO 4 ]
The principal buffer reactions are:
H3O+(aq) + HPO42S(aq)  H2PO4S(aq) + H2O(l)
OH (aq) + H2PO4 (aq)  HPO42 (aq) + H2O(l)
S S S

[base] [CN _ ]
16.64 pH = pKa + log = pK a + log
[acid] [HCN]
For HCN, Ka = 4.9 x 10 , pKa = Slog Ka = Slog(4.9 x 10 10) = 9.31
S10 S

 0.12 
pH = 9.31 + log   = 9.09
 0.20 
The pH of a buffer solution will not change on dilution because the acid and base
concentrations will change by the same amount and their ratio will remain the same.

16.65 NaHCO3, 84.01 amu; Na2CO3, 105.99 amu


 1 mol 
 4.2 g x 
S
[HCO3 ] = molarity of NaHCO3 =  84.01 g 
= 0.25 M
0.20 L

449
Chapter 16 S Applications of Aqueous Equilibria
______________________________________________________________________________

 1 mol 
 5.3 g x 105.99 g 
[CO32S] = molarity of Na2CO3 =   = 0.25 M
0.20 L
[base] [CO32 _ ]
pH = pKa + log = pKa + log
[acid] [HCO3_ ]
For HCO3 , K a 2 = 5.6 x 10 11, p K a 2 = _log K a 2 = Slog(5.6 x 10 11) = 10.25
S S S

[0.25]
pH = 10.25 + log = 10.25
[0.25]
The pH of a buffer solution will not change on dilution because the acid and base
concentrations will change by the same amount and their ratio will remain the same.

[base] [ NH 3]
16.66 pH = pKa + log = pK a + log
[acid] [ NH +4]
For NH4+, Ka = 5.6 x 10 10, pKa = Slog Ka = Slog(5.6 x 10 10) = 9.25
S S

(0.200)
For the buffer: pH = 9.25 + log = 9.25
(0.200)
(a) add 0.0050 mol NaOH, [OHS] = 0.0050 mol/0.500 L = 0.010 M
NH4+(aq) + OH (aq) Ω NH3(aq) + H2O(l)
S

before reaction (M) 0.200 0.010 0.200


change (M) S0.010 S0.010 +0.010
after reaction (M) 0.200 S 0.010 0 0.200 + 0.010
[ NH 3] (0.200 + 0.010)
pH = 9.25 + log = 9.25 + log = 9.29
[ NH +4 ] (0.200 _ 0.010)
(b) add 0.020 mol HCl, [H3O+] = 0.020 mol/0.500 L = 0.040 M
NH3(aq) + H3O+(aq) Ω NH4+(aq) + H2O(l)
before reaction (M) 0.200 0.040 0.200
change (M) S0.040 S0.040 +0.040
after reaction (M) 0.200 S 0.040 0 0.200 + 0.040
[ NH 3] (0.200 _ 0.040)
pH = 9.25 + log = 9.25 + log = 9.07
[ NH +4 ] (0.200 + 0.040)

[base] [SO 32 _ ]
16.67 pH = pKa + log = pK a + log
[acid] [HSO 3_ ]
For HSO3S, Ka = 6.3 x 10S8, pKa = Slog Ka = Slog(6.3 x 10S8) = 7.20
(0.300)
For the buffer: pH = 7.20 + log = 6.98
(0.500)
(a) add (0.0050 L)(0.20 mol/L) = 0.0010 mol HCl = 0.0010 mol H3O+
S
mol HSO3 = (0.300 L)(0.500 mol/L) = 0.150 mol
mol SO32S = (0.300 L)(0.300 mol/L) = 0.0900 mol
SO32 (aq) + H3O+(aq) Ω HSO3 (aq) +
S S
H2O(l)
before reaction (mol) 0.0900 0.0010 0.150

450
Chapter 16 S Applications of Aqueous Equilibria
______________________________________________________________________________

change (mol) S0.0010 S0.0010 +0.0010


after reaction (mol) 0.0900 S 0.0010 0 0.150 + 0.0010
[ 2_ ] (0.0900 _ 0.0010)
pH = 7.20 + log SO3 _ = 7.20 + log = 6.97
[HSO 3 ] (0.150 + 0.0010)
S
(b) add (0.0050 L)(0.10 mol/L) = 0.00050 mol NaOH = 0.00050 mol OH
HSO3 (aq) + OH (aq) Ω SO32 (aq) + H2O(l)
S S S

before reaction (mol) 0.150 0.00050 0.0900


change (mol) S0.00050 S0.00050 +0.00050
after reaction (mol) 0.150 S 0.00050 0 0.0900 + 0.00050
[ 2_ ] (0.0900 + 0.00050)
pH = 7.20 + log SO3 _ = 7.20 + log = 6.98
[HSO 3 ] (0.150 _ 0.00050)

16.68 Acid Ka pKa = Slog Ka


5.8 x 10 10
S
(a) H3BO3 9.24
(b) HCO2H 1.8 x 10S4 3.74
S8
(c) HOCl 3.5 x 10 7.46
The stronger the acid (the larger the Ka), the smaller is the pKa.

16.69 (a) Ka = 10 _ pK a = 10S5.00 = 1.0 x 10S5 (b) Ka = 10 _ pK a = 10S8.70 = 2.0 x 10S9


(b) is the weaker acid

[base] [HCO 2_ ]
16.70 pH = pKa + log = pK a + log
[acid] [HCO 2 H]
For HCO2H, Ka = 1.8 x 10S4; pKa = Slog Ka = Slog(1.8 x 10S4) = 3.74
(0.50)
pH = 3.74 + log = 4.04
(0.25)

[base] [HCO 3_ ]
16.71 pH = pKa + log = pKa + log
[acid] [H 2CO 3]
For H2CO3, Ka = 4.3 x 10 ; pKa = Slog Ka = Slog(4.3 x 10 7) = 6.37
S7 S

_ _
[ ] [ ]
7.40 = 6.37 + log HCO 3 ; 1.03 = log HCO 3
[H 2CO 3] [H 2CO 3]
_
[HCO 3 ] [H 2CO 3]
= 101.03 = 10.7; = 0.093
[H 2CO 3] [HCO 3_ ]

[base] [ ]
16.72 pH = pKa + log = pKa + log NH +3
[acid] [ NH 4 ]
For NH4 , Ka = 5.6 x 10 ; pKa = Slog Ka = Slog(5.6 x 10S10) = 9.25
+ S10

[ ] [ ] [ NH 3]
9.80 = 9.25 + log NH +3 ; 0.550 = log NH +3 ; +
= 100.55 = 3.5
[ NH 4 ] [ NH 4 ] [ NH 4 ]
The volume of the 1.0 M NH3 solution should be 3.5 times the volume of the 1.0 M
NH4Cl solution so that the mixture will buffer at pH 9.80.

451
Chapter 16 S Applications of Aqueous Equilibria
______________________________________________________________________________

_
[base] [ ]
16.73 pH = pKa + log = pKa + log CH 3CO 2
[acid] [CH 3CO 2 H]
For CH3CO2H, Ka = 1.8 x 10 5; pKa = Slog Ka = Slog(1.8 x 10 5) = 4.74
S S

_ _
[ ] [ ]
4.44 = 4.74 + log CH 3CO 2 ; S0.30 = log CH 3CO 2
[CH 3CO 2 H] [CH 3CO 2 H]
_
[CH 3CO 2 ]
= 10S0.30 = 0.50
[CH 3CO 2 H]
S
The solution should have 0.50 mol of CH3CO2 per mole of CH3CO2H. For example,
you could dissolve 41g of CH3CO2Na in 1.00 L of 1.00 M CH3CO2H.

16.74 H3PO4, K a1 = 7.5 x 10 _ 3; pK a1 = _log K a1 = 2.12


S
H2PO4 , K a 2 = 6.2 x 10 _ 8; pK a 2 = _log K a 2 = 7.21
HPO42S, K a 3 = 4.8 x 10 _13; pK a 3 = _log K a 3 = 12.32
The buffer system of choice for pH 7.00 is (b) H2PO4 S HPO42 because the pKa for
S S

H2PO4S (7.21) is closest to 7.00.

HSO4 , Ka2 = 1.2 x 10 2; pKa2 = Slog Ka2 = 1.92


S S
16.75
HOCl, Ka = 3.5 x 10 8; pKa = Slog Ka = 7.56
S

C6H5CO2H, Ka = 6.5 x 10 5; pKa = Slog Ka = 4.19


S

S
The buffer system of choice for pH = 4.50 is (c) C6H5CO2H - C6H5CO2 because the pKa
for C6H5CO2H (4.19) is closest to 4.50.

pH Titration Curves

16.76 (a) (0.060 L)(0.150 mol/L)(1000 mmol/mol) = 9.00 mmol HNO3


 1 mmol NaOH   1 mL NaOH 
(b) vol NaOH = (9.00 mmol HNO3)    = 20.0 mL NaOH
 1 mmol HNO 3   0.450 mmol NaOH 
(c) At the equivalence point the solution contains the neutral salt NaNO3. The pH is 7.00.

(d)

452
Chapter 16 S Applications of Aqueous Equilibria
______________________________________________________________________________

16.77

mmol NaOH = (50.0 mL)(1.0 mmol/mL) = 50 mmol


mmol HCl = mmol NaOH = 50 mmol
 1.0 mL 
vol HCl = (50 mmol)   = 50 mL
 1.0 mmol 
50 mL of 1.0 M HCl is needed to reach the equivalence point.
S
16.78 mmol OH = (20.0 mL)(0.150 mmol/mL) = 3.00 mmol
mmol acid present = mmol OHS added = 3.00 mmol
3.00 mmol
[acid] = = 0.0500 M
60.0 mL

16.79 mmol OHS = (60.0 mL)(0.240 mmol/mL) = 14.4 mmol


1 mmol acid
mmol acid present = 14.4 mmol OHS x = 7.20 mmol acid
2 mmol OH _
7.20 mmol
[acid] = = 0.288 M
25.0 mL

HBr(aq) + NaOH(aq)  Na+(aq) + Br (aq) + H2O(l)


S
16.80
(a) [H3O+] = 0.120 M; pH = Slog[H3O+] = Slog (0.120) = 0.92
(b) (50.0 mL)(0.120 mmol/mL) = 6.00 mmol HBr
(20.0 mL)(0.240 mmol/mL) = 4.80 mmol NaOH
6.00 mmol HBr S 4.80 mmol NaOH = 1.20 mmol HBr after neutralization
1.20 mmol
[H3O+] = = 0.0171 M
(50.0 mL + 20.0 mL)
pH = Slog[H3O+] = Slog(0.0171) = 1.77
(c) (24.9 mL)(0.240 mmol/mL) = 5.98 mmol NaOH
6.00 mmol HBr S 5.98 mmol NaOH = 0.02 mmol HBr after neutralization
0.02 mmol
[H3O+] = = 3 x 10S4 M
(50.0 mL + 24.9 mL)
pH = Slog[H3O+] = Slog(3 x 10 4) = 3.5
S

453
Chapter 16 S Applications of Aqueous Equilibria
______________________________________________________________________________

(d) The titration reaches the equivalence point when 25.0 mL of 0.240 M NaOH is
added. At the equivalence point the solution contains the neutral salt NaBr. The pH is
7.00.
(e) (25.1 mL)(0.240 mmol/mL) = 6.024 mmol NaOH
6.024 mmol NaOH S 6.00 mmol HBr = 0.024 mmol NaOH after neutralization
0.024 mmol
= 3.2 x 10 4 M
S S
[OH ] =
(50.0 mL + 25.1 mL)
1.0 x 10 _14
[H3O+] = K w_ = = 3.1 x 10 11 M
S

[OH ] 3.2 x 10 _ 4
pH = Slog[H3O+] = Slog(3.1 x 10 11) = 10.5
S

(f) (40.0 mL)(0.240 mmol/mL) = 9.60 mmol NaOH


9.60 mmol NaOH S 6.00 mmol HBr = 3.60 mmol NaOH after neutralization
S 3.60 mmol
[OH ] = = 0.040 M
(50.0 mL + 40.0 mL)
1.0 x 10 _14
[H3O+] = K w_ = = 2.5 x 10S13 M
[OH ] 0.040
pH = Slog[H3O ] = Slog(2.5 x 10 13) = 12.60
+ S

Ba(OH)2(aq) + 2 HNO3(aq)  Ba2+(aq) + 2 NO3 (aq) + 2 H2O(l)


S
16.81
S
(a) [OH ] = 2(0.150 M) = 0.300 M
1.0 x 10 _14
[H3O+] = K w_ = = 3.33 x 10 14 M
S

[OH ] 0.300
pH = Slog[H3O+] = Slog(3.33 x 10 14) = 13.48
S

(b) (40.0 mL)(0.150 mmol/mL) = 6.00 mmol Ba(OH)2


2 mmol OH _ S
6.00 mmol Ba(OH)2 x = 12.0 mmol OH
1 mmol Ba(OH ) 2
(10.0 mL)(0.400 mmol/mL) = 4.00 mmol HNO3
S S
12.0 mmol OH S 4.00 mmol HNO3 = 8.00 mmol OH after neutralization
S 8.00 mmol
[OH ] = = 0.160 M
(40.0 mL + 10.0 mL)

454
Chapter 16 S Applications of Aqueous Equilibria
______________________________________________________________________________
_14
[H3O+] = K w = 1.0 x 10 = 6.25 x 10S14 M
[OH _ ] 0.160
pH = Slog[H3O ] = Slog(6.25 x 10 14) = 13.20
+ S

(c) (20.0 mL)(0.400 mmol/mL) = 8.00 mmol HNO3


S S
12.0 mmol OH S 8.00 mmol HNO3 = 4.00 mmol OH after neutralization
S 4.00 mmol
[OH ] = = 0.0667 M
(40.0 mL + 20.0 mL)
1.0 x 10 _14
[H3O+] = K w_ = = 1.50 x 10S13 M
[OH ] 0.0667
pH = Slog[H3O ] = Slog(1.50 x 10 13) = 12.82
+ S

(d) The titration reaches the equivalence point when 30.0 mL of 0.400 M HNO3 is
added. At the equivalence point the solution contains the neutral salt Ba(NO3)2. The pH
is 7.00.
(e) (40.0 mL)(0.400 mmol/mL) = 16.0 mmol HNO3
16.0 mmol HNO3 S 12.0 mmol OH = 4.00 mmol H3O+ after neutralization
S

4.00 mmol
[H3O+] = = 0.0500 M
(40.0 mL + 40.0 mL)
pH = Slog[H3O+] = Slog(0.0500) = 1.30

16.82 mmol HF = (40.0 mL)(0.250 mmol/mL) = 10.0 mmol


mmol NaOH required = mmol HF = 10.0 mmol
 1.00 mL 
mL NaOH required = (10.0 mmol)   = 50.0 mL
 0.200 mmol 

455
Chapter 16 S Applications of Aqueous Equilibria
______________________________________________________________________________

50.0 mL of 0.200 M NaOH is required to reach the equivalence point.


For HF, Ka = 3.5 x 10S4; pKa = Slog Ka = Slog(3.5 x 10S4) = 3.46
(a) mmol HF = 10.0 mmol
mmol NaOH = (0.200 mmol/mL)(10.0 mL) = 2.00 mmol
HF(aq) + OH (aq)  F (aq) + H2O(l)
S S
Neutralization reaction:
before reaction (mmol) 10.0 2.00 0
change (mmol) S2.00 S2.00 +2.00
after reaction (mmol) 8.0 0 2.00
8.0 mmol S 2.00 mmol
[HF] = = 0.16 M; [F ] = = 0.0400 M
(40.0 mL + 10.0 mL) (40.0 mL + 10.0 mL)
HF(aq) + H2O(l) Ω H3O+(aq) + FS(aq)
initial (M) 0.16 ~0 0.0400
change (M) Sx +x +x
equil (M) 0.16 S x x 0.0400 + x
[H 3O + ][F _ ] x(0.0400 + x) x(0.0400)
Ka = = 3.5 x 10 _ 4 = ≈
[HF] 0.16 _ x 0.16
+ S3
Solve for x. x = [H3O ] = 1.4 x 10 M
pH = Slog[H3O+] = Slog(1.4 x 10 3) = 2.85
S

(b) Halfway to the equivalence point,


pH = pKa = Slog Ka = Slog(3.5 x 10 4) = 3.46
S

(c) At the equivalence point only the salt NaF is in solution.


S 10.0 mmol
[F ] = = 0.111 M
(40.0 mL + 50.0 mL)
FS(aq) + H2O(l) Ω HF(aq) + OHS(aq)
initial (M) 0.111 0 ~0
change (M) Sx +x +x
equil (M) 0.111 S x x x
_14
S Kw 1.0 x 10 = 2.9 x 10S11
For F , Kb = = _4
Ka for HF 3.5 x 10
[HF][OH _ ] x
2
x
2
Kb = = 2.9 x 10
_11
= ≈
[F _ ] 0.111 _ x 0.111
Solve for x. x = [OH ] = 1.8 x 10 6 M
S S

1.0 x 10 _14
[H3O+] = K w_ = = 5.6 x 10 9 M
S

[OH ] 1.8 x 10 _ 6
pH = Slog[H3O+] = Slog(5.6 x 10 9) = 8.25
S

(d) mmol HF = 10.0 mmol


mol NaOH = (0.200 mmol/mL)(80.0 mL) = 16.0 mmol

Neutralization reaction: HF(aq) + OHS(aq)  FS(aq) + H2O(l)


before reaction (mmol) 10.0 16.0 0
change (mmol) S10.0 S10.0 +10.0
after reaction (mmol) 0 6.0 10.0
S
After the equivalence point, the pH of the solution is determined by the [OH ].

456
Chapter 16 S Applications of Aqueous Equilibria
______________________________________________________________________________

6.0 mmol
= 5.0 x 10 2 M
S S
[OH ] =
(40.0 mL + 80.0 mL)
1.0 x 10 _14
[H3O+] = K w_ = _2
= 2.0 x 10S13 M
[OH ] 5.0 x 10
pH = Slog[H3O+] = Slog(2.0 x 10 13) = 12.70
S

16.83 mmol CH3NH2 = (100.0 mL)(0.100 mmol/mL) = 10.0 mmol


mmol HNO3 required = mmol CH3NH2 = 10.0 mmol
 1.00 mL 
vol HNO3 required = (10.0 mmol)   = 40.0 mL
 0.250 mmol 
40.0 mL of 0.250 M HNO3 are required to reach the equivalence point.
CH3NH2(aq) + H2O(l) Ω CH3NH3+(aq) + OH (aq)
S
(a)
initial (M) 0.100 0 ~0
change (M) Sx +x +x
equil (M) 0.100 S x x x
+ _ 2
[ ][ ]
Kb = CH 3 NH 3 OH = 3.7 x 10 _ 4 = x
[CH 3 NH 2] 0.100 _ x
2 S4 S5
x + (3.7 x 10 )x S (3.7 x 10 ) = 0
Use the quadratic formula to solve for x.
_ (3.7 x 10 _ 4) ± (3.7 x 10 _ 4) _ (4)(_ 3.7 x 10 _ 5) _ 3.7 x 10 _ 4 ± 0.0122
2

x= =
2(1) 2
x = 0.0059 and S0.0063
Of the two solutions for x, only the positive value of x has physical meaning because x is
the [OHS].
S
[OH ] = x = 0.0059 M
1.0 x 10 _14
[H3O+] = K w_ = = 1.7 x 10 12 M
S

[OH ] 5.9 x 10 _ 3
pH = Slog[H3O+] = Slog(1.7 x 10S12) = 11.77
(b) 20.0 mL of HNO3 is halfway to the equivalence point.
Kw 1.0 x 10 _14
For CH3NH3+, Ka = = _4
= 2.7 x 10S11
Kb for CH 3 NH 2 3.7 x 10
pH = pKa = Slog(2.7 x 10 11) = 10.57
S

(c) At the equivalence point only the salt CH3NH3NO3 is in solution.


mmol CH3NH3NO3 = (0.100 mmol/mL)(100.0 mL) = 10.0 mmol
10.0 mmol
[CH3NH3+] = = 0.0714 M
(100.0 mL + 40.0 mL)
CH3NH3+(aq) + H2O(l) Ω H3O+(aq) + CH3NH2(aq)
initial (M) 0.0714 ~0 0
change (M) Sx +x +x
equil (M) 0.0714 S x x x

457
Chapter 16 S Applications of Aqueous Equilibria
______________________________________________________________________________
+ 2 2
[ ][ 2]
Ka = H 3O CH 3 NH = 2.7 x 10
_11
= x ≈ x
[CH 3 NH 3+ ] 0.0714 _ x 0.0714
+ S6
Solve for x. x = [H3O ] = 1.4 x 10 M
pH = Slog[H3O+] = Slog(1.4 x 10 6) = 5.85
S

(d) mmol CH3NH2 = (0.100 mmol/mL)(100.0 mL) = 10.0 mmol


mmol HNO3 = (0.250 mmol/mL)(60.0 mL) = 15.0 mmol
Neutralization reaction: CH3NH2(aq) + H3O+(aq)  CH3NH3+(aq) + H2O(l)
before reaction (mmol) 10.0 15.0 0
change (mmol) S10.0 S10.0 +10.0
after reaction (mmol) 0 5.0 10.0
After the equivalence point the pH of the solution is determined by the [H3O+].
5.0 mmol
[H3O+] = = 3.1 x 10 2 M
S

(100.0 mL + 60.0 mL)


pH = Slog[H3O+] = Slog(3.1 x 10S2) = 1.51

For H2A+, Ka1 = 4.6 x 10 3 and Ka2 = 2.0 x 10 10


S S
16.84
(a) (10.0 mL)(0.100 mmol/mL) = 1.00 mmol NaOH added = 1.00 mmol HA produced.
(50.0 mL)(0.100 mmol/mL) = 5.00 mmol H2A+
5.00 mmol H2A+ S 1.00 mmol NaOH = 4.00 mmol H2A+ after neutralization
4.00 mmol
[H2A+] = = 6.67 x 10 2 M
S

(50.0 mL + 10.0 mL)


1.00 mmol
[HA] = = 1.67 x 10S2 M
(50.0 mL + 10.0 mL)
[HA] S3  1.67 x 10 _ 2 
pH = pKa1 + log = S log(4.6 x 10 ) + log  6.67 x _ 2  = 1.74
[H 2A +]  10 
(b) Halfway to the first equivalence point, pH = pKa1 = 2.34
pK a1 + pK a 2
(c) At the first equivalence point, pH = = 6.02
2
(d) Halfway between the first and second equivalence points, pH = pKa2 = 9.70
(e) At the second equivalence point only the basic salt, NaA, is in solution.
Kw 1.0 x 10 _14
Kb = = Kw = _10
= 5.0 x 10S5
K a for HA K a 2 2.0 x 10
S
mmol A = (50.0 mL)(0.100 mmol/mL) = 5.00 mmol
5.0 mmol
= 3.3 x 10 2 M
S S
[A ] =
(50.0 mL + 100.0 mL)
A (aq) + H2O(l) Ω HA(aq) + OH (aq)
S S

initial (M) 0.033 0 ~0


change (M) Sx +x +x
equil (M) 0.033 S x x x
[HA][OH _ ] (x)(x) x
2
Kb = = 5.0 x 10
_5
= ≈
[A _ ] 0.033 _ x 0.033
Solve for x.

458
Chapter 16 S Applications of Aqueous Equilibria
______________________________________________________________________________

(5.0 x 10 _ 5)(0.033) = 1.3 x 10 3 M


S S
x = [OH ] =
1.0 x 10 _14
[H3O+] = K w_ = = 7.7 x 10 12 M
S

[OH ] 1.3 x 10 _ 3
pH = Slog[H3O+] = Slog(7.7 x 10S12) = 11.11

For H2CO3, Ka1 = 4.3 x 10 7 and Ka2 = 5.6 x 10 11


S S
16.85
(a) (25.0 mL)(0.0200 mmol/mL) = 0.500 mmol H2CO3
(10.0 mL)(0.0250 mmol/mL) = 0.250 mmol KOH added
S
0.500 mmol H2CO3 S 0.250 mmol KOH = 0.250 mmol HCO3 produced
This is halfway to the first equivalence point where pH = pKa1 = Slog(4.3 x 10S7) = 6.37
pK a1 + pK a 2
(b) At the first equivalence point, pH = = 8.31
2
(c) Halfway between the first and second equivalence points, pH = pKa2 = 10.25
(d) At the second equivalence point only the basic salt, K2CO3, is in solution.
_14
Kb = Kw = K w = 1.0 x 10 = 1.8 x 10S4
_ _11
K a for HCO 3 K a 2 5.6 x 10
mmol CO32S = mmol H2CO3 = 0.500 mmol
0.500 mmol
[CO32 ] =
S
= 0.00769 M
(25.0 mL + 40.0 mL)
CO32S(aq) + H2O(l) Ω HCO3S(aq) + OHS(aq)
initial (M) 0.00769 0 ~0
change (M) Sx +x +x
equil (M) 0.00769 S x x x
[HCO3_ ][OH _ ] (x)(x)
Kb = 2_
= 1.8 x 10 _ 4 =
[CO3 ] 0.00769 _ x
Use the quadratic formula to solve for x.
x=
_ (1.8 x 10 _ 4) ± (1.8 x 10 _ 4) 2 _ (4)(1)(_ 1.4 x 10 _ 6) (_1.8 x 10 _ 4) ± (2.37 x 10 _ 3)
=
2(1) 2
S3 S3
x = S1.27 x 10 and 1.09 x 10
Of the two solutions for x, only the positive value of x has physical meaning because x is
S
the [OH ].
[OH ] = x = 1.09 x 10 3 M
S S

1.0 x 10 _14
[H3O+] = K w_ = = 9.2 x 10 12 M
S
_3
[OH ] 1.09 x 10
pH = Slog[H3O+] = Slog(9.2 x 10 12) = 11.04
S

(e) excess KOH


S
(50.0 mL S 40.0 mL)(0.025 mmol/mL) = 0.250 mmol KOH = 0.250 mmol OH
0.250 mmol
= 3.33 x 10 3 M
S S
[OH ] =
(25.0 mL + 50.0 mL)

459
Chapter 16 S Applications of Aqueous Equilibria
______________________________________________________________________________
_14
[H3O+] = K w = 1.0 x 10 = 3.0 x 10 12 M
S
_ _3
[OH ] 3.33 x 10
pH = Slog[H3O+] = Slog(3.0 x 10 12) = 11.52
S

16.86 When equal volumes of acid and base react, all concentrations are cut in half.
(a) At the equivalence point only the salt NaNO2 is in solution.
[NO2S] = 0.050 M
Kw 1.0 x 10 _14
For NO2S, Kb = = _4
= 2.2 x 10S11
K a for HNO 2 4.5 x 10
NO2 (aq) + H2O(l) Ω HNO2(aq) + OH (aq)
S S

Initial (M) 0.050 0 ~0


change (M) Sx +x +x
equil (M) 0.050 S x x x
[HNO 2][OH _ ] (x)(x) 2
Kb = = 2.2 x 10 _11 = ≈ x
[ NO 2_ ] 0.050 _ x 0.050
Solve for x. x = [OH ] = 1.1 x 10 6 M
S S

1.0 x 10 _14
[H3O+] = K w_ = = 9.1 x 10 9 M
S
_6
[OH ] 1.1 x 10
pH = Slog[H3O+] = Slog(9.1 x 10S9) = 8.04
Phenol red would be a suitable indicator. (see Figure 15.4)
(b) The pH is 7.00 at the equivalence point for the titration of a strong acid (HI) with a
strong base (NaOH).
Bromthymol blue or phenol red would be suitable indicators. (Any indicator that
changes color in the pH range 4 S 10 is satisfactory for a strong acid S strong base
titration.)
(c) At the equivalence point only the salt CH3NH3Cl is in solution.
[CH3NH3+] = 0.050 M
Kw 1.0 x 10 _14
For CH3NH3+, Ka = = 2.7 x 10 11
S
= _4
Kb for CH 3 NH 2 3.7 x 10
CH3NH3 (aq) + H2O(l) Ω H3O+(aq) + CH3NH2(aq)
+

initial (M) 0.050 ~0 0


change (M) Sx +x +x
equil (M) 0.050 S x x x
+ 2
[ ][ 2] (x)(x)
Ka = H 3O CH 3 NH +
= 2.7 x 10 _11 = ≈ x
[CH 3 NH 3 ] 0.050 _ x 0.050
+ S6
Solve for x. x = [H3O ] = 1.2 x 10 M
pH = Slog[H3O+] = Slog(1.2 x 10 6) = 5.92
S

Chlorphenol red would be a suitable indicator.

16.87 When equal volumes of acid and base react, all concentrations are cut in half.
(a) At the equivalence point only the salt C5H11NHNO3 is in solution.
[C5H11NH+] = 0.10 M

460
Chapter 16 S Applications of Aqueous Equilibria
______________________________________________________________________________
_14
Kw 1.0 x 10
For C5H11NH+, Ka = = 7.7 x 10 12
S
= _3
K b for C5H11 N 1.3 x 10
C5H11NH+(aq) + H2O(l) Ω H3O+(aq) + C5H11N(aq)
initial (M) 0.10 ~0 0
change (M) Sx +x +x
equil (M) 0.10 S x x x
+ 2
[ ][ N] (x)(x)
Ka = H 3O C5H11+ = 7.7 x 10 _12 = ≈ x
[C 5H11NH ] 0.10 _ x 0.10
+ S7
Solve for x. x = [H3O ] = 8.8 x 10 M
pH = Slog[H3O+] = Slog(8.8 x 10 7) = 6.06
S

Alizarin would be a suitable indicator (see Figure 15.4)


(b) At the equivalence point only the salt Na2SO3 is in solution.
[SO32 ] = 0.10 M
S

Kw 1.0 x 10 _14
For SO32 , Kb = = 1.6 x 10 7
S S
_
= _8
Ka for HSO 3 6.3 x 10
SO3 (aq) + H2O(l) Ω HSO3S(aq) + OHS(aq)
2S

Initial (M) 0.10 0 ~0


change (M) Sx +x +x
equil (M) 0.10 S x x x
_ _ 2
[ ][ ] (x)(x)
Kb = HSO 3 2 _OH = 1.6 x 10 _ 7 = ≈ x
[SO 3 ] 0.10 _ x 0.10
S S4
Solve for x. x = [OH ] = 1.26 x 10 M
1.0 x 10 _14
[H3O+] = K w_ = _4
= 7.9 x 10S11 M
[OH ] 1.26 x 10
pH = Slog[H3O+] = Slog(7.9 x 10 11) = 10.10
S

Thymolphthalein would be a suitable indicator.


(c) The pH is 7.00 at the equivalence point for the titration of a strong acid (HBr) with a
strong base (Ba(OH)2).
Alizarin, bromthymol blue, or phenol red would be suitable indicators. (Any indicator
that changes color in the pH range 4 - 10 is satisfactory for a strong acid - strong base
titration.)
Solubility Equilibria

Ag2CO3(s) Ω 2 Ag+(aq) + CO32 (aq) Ksp = [Ag+]2[CO32 ]


S S
16.88 (a)
PbCrO4(s) Ω Pb2+(aq) + CrO42 (aq) Ksp = [Pb2+][CrO42 ]
S S
(b)
Al(OH)3(s) Ω Al3+(aq) + 3 OH (aq) Ksp = [Al3+][OH ]3
S S
(c)
(d) Hg2Cl2(s) Ω Hg22+(aq) + 2 ClS(aq) Ksp = [Hg22+][ClS]2

(a) Ksp = [Ca2+][OH ]2 (b) Ksp = [Ag+]3[PO43 ]


S S
16.89
(c) Ksp = [Ba2+][CO32 ] (d) Ksp = [Ca2+]5[PO43 ]3[OH ]
S S S

(a) Ksp = [Pb2+][I ]2 = (5.0 x 10 3)(1.3 x 10 3)2 = 8.4 x 10 9


S S S S
16.90

461
Chapter 16 S Applications of Aqueous Equilibria
______________________________________________________________________________

K sp (8.4 x 10 _ 9
= 5.8 x 10 3 M
S S
(b) [I ] = 2+
= _4
[Pb ] (2.5 x 10 )
K sp (8.4 x 10 _ 9)
(c) [Pb2+] = _ 2 = = 0.13 M
[I ] (2.5 x 10 _ 4) 2

16.91 (a) Ksp = [Ca2+]3[PO42S]2 = (2.9 x 10S7)3(2.9 x 10S7)2 = 2.1 x 10S33


K sp 2.1 x 10 _ 33
(b) [Ca2+] = 3
2_ 2
= 3
2
= 2.8 x 10S10 M
[PO 4 ] (0.010)
K sp 2.1 x 10 _ 33
(c) [PO42 ] = = 4.6 x 10 14 M
S S
2+ 3
= 3
[Ca ] (0.010)

Ag2CO3(s) Ω 2 Ag+(aq) + CO32 (aq)


S
16.92
equil (M) 2x x
[Ag+] = 2x = 2.56 x 10 4 M; [CO32 ] = x = (2.56 x 10 4 M)/2 = 1.28 x 10 4 M
S S S S

Ksp = [Ag+]2[CO32 ] = (2.56 x 10 4)2(1.28 x 10 4) = 8.39 x 10 12


S S S S

16.93 (a) [Cd2+] = [CO32S] = 2.5 x 10S6 M


Ksp = [Cd2+][CO32 ] = (2.5 x 10 6)2 = 6.2 x 10 12
S S S

2+ S2
(b) [Ca ] = 1.06 x 10 M
[OH ] = 2[Ca2+] = 2(1.06 x 10 2 M) = 2.12 x 10 2 M
S S S

Ksp = [Ca ][OH ] = (1.06 x 10 )(2.12 x 10 ) = 4.76 x 10 6


2+ S 2 S2 S2 2 S

(c) PbBr2, 367.01 amu


 1 mol 
 4.34 g x 
[Pb2+] = molarity of PbBr2 = 
367.01 g 
= 1.18 x 10 2 M
S

1L
[Br ] = 2[Pb2+] = 2(1.18 x 10 2 M) = 2.36 x 10 2 M
S S S

Ksp = [Pb2+][Br ]2 = (1.18 x 10 2)(2.36 x 10 2)2 = 6.57 x 10 6


S S S S

(d) BaCrO4, 253.32 amu


 -3 1 mol 
 2.8 x 10 g x 253.32 g 
[Ba2+] = [CrO42S] = molarity of BaCrO4 =   = 1.1 x 10S5 M
1L
2+ 2S S5 2 S10
Ksp = [Ba ][CrO4 ] = (1.1 x 10 ) = 1.2 x 10

16.94 (a) BaCrO4(s) Ω Ba2+(aq) + CrO42S(aq)


equil (M) x x
Ksp = [Ba2+][CrO42 ] = 1.2 x 10 10 = (x)(x)
S S

molar solubility = x = 1.2 x 10 _10 = 1.1 x 10 5 M


S

Mg(OH)2(s) Ω Mg2+(aq) + 2 OH (aq)


S
(b)
equil (M) x 2x
2+ S 2 S12 2 3
Ksp = [Mg ][OH ] = 5.6 x 10 = x(2x) = 4x

462
Chapter 16 S Applications of Aqueous Equilibria
______________________________________________________________________________

5.6 x 10 _12
= 1.1 x 10 4 M
S
molar solubility = x = 3
4
Ag2SO3(s) Ω 2 Ag+(aq) + SO32 (aq)
S
(c)
equil (M) 2x x
Ksp = [Ag+]2[SO32 ] = 1.5 x 10 14 = (2x)2x = 4x3
S S

1.5 x 10 _14
= 1.6 x 10 5 M
S
molar solubility = x = 3
4

Ag2CO3(s) Ω 2 Ag+(aq) + CO32 (aq)


S
16.95 (a)
equil (M) 2x x
Ksp = [Ag+]2[CO32S] = 8.4 x 10S12 = (2x)2(x) = 4x3
8.4 x 10 _12
= 1.3 x 10 4 M
S
molar solubility = x = 3
4
Ag2CO3, 275.75 amu
solubility = (1.3 x 10 4 mol/L)(275.75 g/mol) = 0.036 g/L
S

(b) CuBr(s) Ω Cu+(aq) + BrS(aq)


equil (M) x x
Ksp = [Cu+][Br ] = 6.3 x 10 9 = (x)(x)
S S

molar solubility = x = 6.3 x 10-9 = 7.9 x 10 5 M


S

CuBr, 143.45 amu


solubility = (7.9 x 10S5 mol/L)(143.45 g/mol) = 0.011 g/L
Cu3(PO4)2(s) Ω 3 Cu2+(aq) + 2 PO43 (aq)
S
(c)
equil (M) 3x 2x
2+ 3 3S 2 S37 3 2 5
Ksp = [Cu ] [PO4 ] = 1.4 x 10 = (3x) (2x) = 108x
1.4 x 10 _ 37
= 1.7 x 10 8
S
molar solubility = x = 5
108
Cu3(PO4)2 , 380.58 amu
solubility = (1.7 x 10 8 mol/L)(380.58 g/mol) = 6.5 x 10 6 g/L
S S

Factors That Affect Solubility

Ag2CO3(s) Ω 2 Ag+(aq) + CO32 (aq)


S
16.96
(a) AgNO3, source of Ag+; equilibrium shifts left
(b) HNO3, source of H3O+, removes CO32 ; equilibrium shifts right
S

2S
(c) Na2CO3, source of CO3 ; equilibrium shifts left
(d) NH3, forms Ag(NH3)2+; removes Ag+; equilibrium shifts right

16.97 BaF2(s) Ω Ba2+(aq) + 2 FS(aq)


(a) H+ from HCl reacts with FS forming the weak acid HF. The equilibrium shifts to the
right increasing the solubility of BaF2.
S
(b) KF, source of F ; equilibrium shifts left, solubility of BaF2 decreases.
(c) No change in solubility.
(d) Ba(NO3)2, source of Ba2+; equilibrium shifts left, solubility of BaF2 decreases.

463
Chapter 16 S Applications of Aqueous Equilibria
______________________________________________________________________________

PbCrO4(s) Ω Pb2+(aq) + CrO42 (aq)


S
16.98 (a)
equil (M) x x
2+ 2S S13
Ksp = [Pb ][CrO4 ] = 2.8 x 10 = (x)(x)
molar solubility = x = 2.8 x 10 _13 = 5.3 x 10 7 M
S

(b) PbCrO4(s) Ω Pb2+(aq) + CrO42S(aq)


1.0 x 10 3
S
initial(M) 0
S3
equil (M) x 1.0 x 10 + x
Ksp = [Pb ][CrO4 ] = 1.2 x 10 = (x)(1.0 x 10S3 + x) . (x)(1.0 x 10S3)
2+ 2S S10

2.8 x 10 _13
molar solubility = x = = 2.8 x 10S10 M
1 x 10 _ 3

SrF2(s) Ω Sr2+(aq) + 2 F (aq)


S
16.99 (a)
initial (M) 0.010 0
equil (M) 0.010 + x 2x
Ksp = [Sr2+][FS]2 = 4.3 x 10S9 = (0.010 + x)(2x)2 . (0.010)(2x)2 = 0.040 x2
4.3 x 10 _ 9
= 3.3 x 10 4 M
S
molar solubility = x =
0.040
SrF2(s) Ω Sr2+(aq) + 2 F (aq)
S
(b)
initial (M) 0 0.010
equil (M) x 0.010 + 2x
Ksp = [Sr2+][F ]2 = 4.3 x 10 9 = (x)(0.010 + 2x)2 . (x)(0.010)2 = x(0.00010)
S S

4.3 x 10 _ 9
= 4.3 x 10 5 M
S
molar solubility = x =
0.00010

16.100 (b), (c), and (d) are more soluble in acidic solution.
(a) AgBr(s) Ω Ag+(aq) + Br (aq)
S

(b) CaCO3(s) + H3O+(aq) Ω Ca2+(aq) + HCO3 (aq) + H2O(l)


S

(c) Ni(OH)2(s) + 2 H3O+(aq) Ω Ni2+(aq) + 4 H2O(l)


(d) Ca3(PO4)2(s) + 2 H3O+(aq) Ω 3 Ca2+(aq) + 2 HPO42 (aq) + 2 H2O(l)
S

16.101 (a), (b), and (d) are more soluble in acidic solution.
(a) MnS(s) + 2 H3O+(aq) Ω Mn2+(aq) + H2S(aq) + 2 H2O(l)
(b) Fe(OH)3(s) + 3 H3O+(aq) Ω Fe3+(aq) + 6 H2O(l)
(c) AgCl(s) Ω Ag+(aq) + Cl (aq)
S

(d) BaCO3(s) + H3O+(aq) Ω Ba2+(aq) + HCO3S(aq) + H2O(l)

16.102 On mixing equal volumes of two solutions, the concentrations of both solutions are cut in half.
Ag+(aq) + 2 CNS(aq) Ω Ag(CN)2S(aq)
before reaction (M) 0.0010 0.10 0
assume 100% reaction S0.0010 S2(0.0010) 0.0010

464
Chapter 16 S Applications of Aqueous Equilibria
______________________________________________________________________________

after reaction (M) 0 0.098 0.0010


assume small back rxn +x +2x Sx
equil (M) x 0.098 + 2x 0.0010 S x
_
[Ag(CN ) 2 ] (0.0010 _ x) 0.0010
Kf = 3.0 x 1020 = = ≈
[Ag ][CN ] x(0.098 + 2 x ) x(0.098) 2
+ _ 2 2

Solve for x. x = [Ag+] = 3.5 x 10 22 M


S

Cr3+(aq) + 4 OH (aq) Ω Cr(OH)4 (aq)


S S
16.103
before reaction (M) 0.0050 1.0 0
assume 100% reaction S0.0050 S(4)(0.0050) +0.0050
after reaction(M) 0 0.98 0.0050
assume small back rxn +x +4x Sx
equil (M) x 0.98 + 4x 0.0050 S x
_
[Cr(OH ) 4 ] (0.0050 _ x) (0.0050)
Kf = _ 4
= 8 x 10 29 = ≈
3+
[Cr ][OH ] (x)(0.98 + 4 x ) (x)(0.98) 4
4

Solve for x. x = [Cr3+] = 6.8 x 10 33 M = 7 x 10 33 M


S S

[Cr 3+ ] 7 x 10 _ 33 M
fraction uncomplexed Cr3+ = = 1.4 x 10 30 = 1 x 10 30
S S
_
=
[Cr(OH ) 4 ] 0.0050 M

AgI(s) Ω Ag+(aq) + I (aq) Ksp = 8.5 x 10 17


S S
16.104 (a)
Ag+(aq) + 2 CN (aq)  Ag(CN)2 (aq) Kf = 3.0 x 1020
S S

dissolution rxn AgI(s) + 2 CN (aq) Ω Ag(CN)2 (aq) + I (aq)


S S S

S17 20 4
K = (Ksp)(Kf) = (8.5 x 10 )(3.0 x 10 ) = 2.6 x 10
Al(OH)3(s) Ω Al3+(aq) + 3 OH (aq)
S S33
(b) Ksp = 1.9 x 10
Al3+(aq) + 4 OH (aq)  Al(OH)4 (aq) Kf = 3 x 1033
S S

dissolution rxn Al(OH)3(s) + OH (aq) Ω Al(OH)4 (aq)


S S

K = (Ksp)(Kf) = (1.9 x 10 33)(3 x 1033) = 6


S

Zn(OH)2(s) Ω Zn2+(aq) + 2 OH (aq)


S S17
(c) Ksp = 4.1 x 10
Zn2+(aq) + 4 NH3(aq)  Zn(NH3)42+(aq) Kf = 7.8 x 108
dissolution rxn Zn(OH)2(s) + 4 NH3(aq) Ω Zn(NH3)42+ + 2 OH (aq)
S

K = (Ksp)(Kf) = (4.1 x 10 )(7.8 x 108) = 3.2 x 10 8


S17 S

Zn(OH)2(s) Ω Zn2+(aq) + 2 OH (aq) Ksp = 4.1 x 10 17


S S
16.105 (a)
Zn (aq) + 4 OH (aq)  Zn(OH)4 (aq)
2+ S 2S
Kf = 3 x 1015
dissolution rxn Zn(OH)2(s) + 2 OH (aq) Ω Zn(OH)42 (aq)
S S

K = (Ksp)(Kf) = (4.1 x 10S17)(3 x 1015) = 0.1


Cu(OH)2(s) Ω Cu2+(aq) + 2 OH (aq) Ksp = 1.6 x 10 19
S S
(b)
Cu (aq) + 4 NH3(aq)  Cu(NH3)4 (aq)
2+ 2+
Kf = 5.6 x 1011

dissolution rxn Cu(OH)2(s) + 4 NH3(aq) Ω Cu(NH3)4 + 2 OH (aq)


2+ S

465
Chapter 16 S Applications of Aqueous Equilibria
______________________________________________________________________________

K = (Ksp)(Kf) = (1.6 x 10 19)(5.6 x 1011) = 9.0 x 10 8


S S

AgBr(s) Ω Ag+(aq) + Br (aq) Ksp = 5.4 x 10 13


S S
(c)
Ag+(aq) + 2 NH3(aq)  Ag(NH3)2+(aq) Kf = 1.7 x 107
dissolution rxn AgBr(s) + 2 NH3(aq) Ω Ag(NH3)2+(aq) + Br (aq)
S

K = (Ksp)(Kf) = (5.4 x 10 13)(1.7 x 107) = 9.2 x 10 6


S S

AgI(s) Ω Ag+(aq) + I (aq)


S
16.106 (a)
equil (M) x x
+ S S17
Ksp = [Ag ][I ] = 8.5 x 10 = (x)(x)
molar solubility = x = 8.5 x 10 _17 = 9.2 x 10S9 M
(b) AgI(s) + 2 CNS(aq) Ω Ag(CN)2S(aq) + IS(aq)
initial (M) 0.10 0 0
change (M) S2x +x +x
equil (M) 0.10 S 2x x x
K = (Ksp)(Kf) = (8.5 x 10 17)(3.0 x 1020) = 2.6 x 104
S

[Ag(CN ) 2_ ][I _ ] x
2
K = 2.6 x 104 = =
[CN _ ]2 (0.10 _ 2 x ) 2
Take the square root of both sides and solve for x.
molar solubility = x = 0.050 M

16.107 Cr(OH)3(s) + OHS(aq) Ω Cr(OH)4S(aq)


initial (M) 0.50 0
change (M) Sx +x
equil (M) 0.50 S x x
S31 29
K = (Ksp)(Kf) = (6.7 x 10 )(8 x 10 ) = 0.54
_
[Cr(OH ) 4 ] x
K = 0.54 = _
=
[OH ] 0.50 _ x
0.27 S 0.54x = x
0.27 = 1.54x
0.27
molar solubility = x = = 0.2 M
1.54

Precipitation; Qualitative Analysis

16.108 For BaSO4, Ksp = 1.1 x 10 10


S

Total volume = 300 mL + 100 mL = 400 mL


(4.0 x 10 _ 3 M)(100 mL)
[Ba2+] = = 1.0 x 10 3 M
S

(400 mL)
(6.0 x 10 _ 4 M)(300 mL)
[SO42 ] = = 4.5 x 10 4 M
S S

(400 mL)
IP = [Ba ]t[SO4 ]t = (1.0 x 10S3)(4.5 x 10S4) = 4.5 x 10S7
2+ 2S

IP > Ksp; BaSO4(s) will precipitate.

466
Chapter 16 S Applications of Aqueous Equilibria
______________________________________________________________________________

16.109 On mixing equal volumes of two solutions, the concentrations of both solutions are cut in
half.
For PbCl2, Ksp = 1.2 x 10 5 = [Pb2+][Cl ]2
S S

2 S7
IP = (0.0050)(0.0050) = 1.2 x 10
IP < Ksp; no precipitate will form.
S K sp 1.2 x 10 _ 5
[Cl ] = = = 0.049 M
[Pb 2+ ] 5.0 x 10 _ 3
A [ClS] just greater than 0.049 M will result in precipitation.

16.110 BaSO4, Ksp = 1.1 x 10 10; Fe(OH)3, Ksp = 2.6 x 10 39


S S

Total volume = 80 mL + 20 mL = 100 mL


(1.0 x 10 _ 5 M)(80 mL)
[Ba2+] = = 8.0 x 10 6 M
S

(100 mL)
[OH ] = 2[Ba ] = 2(8.0 x 10 6) = 1.6 x 10 5 M
2+
S S S

2(1.0 x 10 _ 5 M)(20 mL)


[Fe3+] = = 4.0 x 10 6 M
S

(100 mL)
3(1.0 x 10 _ 5 M)(20 mL)
[SO42S] = = 6.0 x 10S6 M
(100 mL)
For BaSO4, IP = [Ba2+]t[SO42 ]t = (8.0 x 10 6)(6.0 x 10 6) = 4.8 x 10 11
S S S S

IP < Ksp; BaSO4 will not precipitate.


For Fe(OH)3, IP = [Fe3+]t[OH ]t3 = (4.0 x 10 6)(1.6 x 10 5)3 = 1.6 x 10 20
S S S S

IP > Ksp; Fe(OH)3(s) will precipitate.

(2.0 x 10 _ 3 M)(0.10 mL)


16.111 (a) [CO32S] = = 8.0 x 10S7 M
(250 mL)
Ksp = 5.0 x 10 9 = [Ca2+][CO32 ]
S S

IP = [Ca ][CO3 ] = (8.0 x 10 )(8.0 x 10S7) = 6.4 x 10S10


2+ 2S S4

IP < Ksp; no precipitate will form.


(b) Na2CO3, 106 amu; 10 mg = 0.010 g
 1 mol 
 0.010 g x 
2S
[CO3 ] =  106 g 
= 3.8 x 10 4 M
S

0.250 L
IP = [Ca2+][CO32 ] = (8.0 x 10 4)(3.8 x 10 4) = 3.0 x 10 7
S S S S

IP > Ksp; CaCO3(s) will precipitate.

16.112 pH = 10.80; [H3O+] = 10SpH = 10S10.80 = 1.6 x 10S11 M


1.0 x 10 _14
[OH ] = K w + = = 6.2 x 10 4 M
S S
_11
[H 3O ] 1.6 x 10
For Mg(OH)2, Ksp = 5.6 x 10 12
S

IP = [Mg2+]t[OH ]t2 = (2.5 x 10S4)(6.2 x 10S4)2 = 9.6 x 10S11


S

IP > Ksp; Mg(OH)2(s) will precipitate

467
Chapter 16 S Applications of Aqueous Equilibria
______________________________________________________________________________

16.113 Mg(OH)2, Ksp = 5.6 x 10S12; Al(OH)3, Ksp = 1.9 x 10S33


pH = 8; [H3O+] = 10SpH = 10S8 = 1 x 10S8 M
1.0 x 10 _14
[OHS] = K w + = _8
= 1 x 10S6 M
[ H 3O ] 1 x 10
For Mg(OH)2, IP = [Mg2+][OH ]2 = (0.01)(1 x 10 6)2 = 1 x 10 14
S S S

IP < Ksp; no Mg(OH)2 will precipitate.


For Al(OH)3, IP = [Al3+][OHS]3 = (0.01)(1 x 10S6)3 = 1 x 10S20
IP > Ksp; Al(OH)3 will precipitate.

[ 2+ ][ S]
16.114 Kspa = M H+ 22 ; FeS, Kspa = 6 x 102; SnS, Kspa = 1 x 10 5
S

[H 3O ]
Fe and Sn2+ can be separated by bubbling H2S through an acidic solution containing the
2+

two cations because their Kspa values are so different.


(0.01)(0.10)
= 1.1 x 10 2
S
For FeS and SnS, Qc = 2
(0.3)
For FeS, Qc < Kspa, and no FeS will precipitate.
For SnS, Qc > Kspa, and SnS will precipitate.

[Co 2+][H 2 S]
16.115 CoS, Kspa = =3
[H 3O + ]2
(i) In 0.5 M HCl, [H3O+] = 0.5 M
[Co 2+ ]t[H 2 S ]t (0.10)(0.10)
Qc = = = 0.04; Qc < Kspa; CoS will not precipitate
[H 3O + ]2t (0.5)
2

(ii) pH = 8; [H3O+] = 10SpH = 10S8 = 1 x 10S8 M


[Co 2+ ]t[H 2 S ]t (0.10)(0.10)
Qc = + 2
= _8 2
= 1 x 1014; Qc > Kspa; CoS(s) will precipitate
[H 3O ]t (1 x 10 )

S
16.116 (a) add Cl to precipitate AgCl
(b) add CO32 to precipitate CaCO3
S

(c) add H2S to precipitate MnS


(d) add NH3 and NH4Cl to precipitate Cr(OH)3
S S S
(Need buffer to control [OH ]; excess OH produces the soluble Cr(OH)4 .)
S
16.117 (a) add Cl to precipitate Hg2Cl2
(b) add (NH4)2HPO4 to precipitate MgNH4PO4
(c) add HCl and H2S to precipitate HgS
(d) add ClS to precipitate PbCl2

General Problems

468
Chapter 16 S Applications of Aqueous Equilibria
______________________________________________________________________________

16.118 Prepare aqueous solutions of the three salts. Add a solution of (NH4)2HPO4. If a white
precipitate forms, the solution contains Mg2+. Perform flame test on the other two
solutions. A yellow flame test indicates Na+. A violet flame test indicates K+.
S
16.119 (a), solution contains HCN and CN
S
(c), solution can contain HCN and CN
S
(e), solution can contain HCN and CN

16.120 (a), solution contains H2CO3 and HCO3S


(b), solution contains HCO3 and CO32
S S

S 2S
(d), solution contains HCO3 and CO3

16.121

(a) The pH for the weak acid is higher.


(b) Initially, the pH rises more quickly for the weak acid, but then the curve becomes
more level in the region halfway to the equivalence point.
(c) The pH is higher at the equivalence point for the weak acid.
(d) Both curves are identical beyond the equivalence point because the pH is determined
S
by the excess [OH ].
(e) If the acid concentrations are the same, the volume of base needed to reach the
equilavence point is the same.

469
Chapter 16 S Applications of Aqueous Equilibria
______________________________________________________________________________

16.122 (a)

(b) mol NaOH required =


 0.010 mol HA   1 mol NaOH 
  (0.0500 L)   = 0.000 50 mol
 L   1 mol HA 
 1L 
vol NaOH required = (0.000 50 mol)   = 0.050 L = 50 mL
 0.010 mol 
(c) A basic salt is present at the equivalence point; pH > 7.00
(d) Halfway to the equivalence point, the pH = pKa = 4.00

AgBr(s) Ω Ag+(aq) + Br (aq)


S
16.123 (a)
(i) HBr is a source of BrS (reaction product). The solubility of AgBr is decreased.
(ii) unaffected
(iii) AgNO3 is a source of Ag+ (reaction product). The solubility of AgBr is decreased.
(iv) NH3 forms a complex with Ag+, removing it from solution. The solubility of AgBr is
increased.
BaCO3(s) Ω Ba2+(aq) + CO32 (aq)
S
(b)
(i) HNO3 reacts with CO32 , removing it from the solution. The solubility of BaCO3 is
S

increased.
(ii) Ba(NO3)2 is a source of Ba2+ (reaction product). The solubility of BaCO3 is
decreased.
(iii) Na2CO3 is a source of CO32S (reaction product). The solubility of BaCO3 is
decreased.
(iv) CH3CO2H reacts with CO32 , removing it from the solution. The solubility of BaCO3
S

is increased.

Kw 1.0 x 10 _14
16.124 For NH4+, Ka = = 5.6 x 10 10
S
= _5
K b for NH 3 1.8 x 10
pKa = Slog Ka = Slog(5.6 x 10 10) = 9.25
S

[ ] [ ]
pH = pKa + log NH+3 ; 9.40 = 9.25 + log NH+3
[ NH 4 ] [ NH 4 ]
[ ] [ NH 3]
log NH +3 = 9.40 S 9.25 = 0.15; +
= 100.15 = 1.41
[ NH 4 ] [ NH 4 ]

470
Chapter 16 S Applications of Aqueous Equilibria
______________________________________________________________________________

mol NH 3
Because the volume is the same for both NH3 and NH4+, = 1.41.
mol NH +4
mol NH3 = (0.20 mol/L)(0.250 L) = 0.050 mol NH3
mol NH 3 0.050
mol NH4+ = = = 0.035 mol NH4+
1.41 1.41
 1L 
vol NH4+ = (0.035 mol)  = 0.012 L = 12 mL
 3.0 mol 
12 mL of 3.0 M NH4Cl must be added to 250 mL of 0.20 M NH3 to obtain a buffer
solution having a pH = 9.40.

16.125 H2PO4 (aq) + H2O(l) Ω H3O+(aq) + HPO42 (aq)


S S

(a) Na2HPO4, source of HPO42S, equilibrium shifts left, pH increases.


(b) Addition of the strong acid, HBr, decreases the pH.
(c) Addition of the strong base, KOH, increases the pH.
(d) There is no change in the pH with the addition of the neutral salt KI.
S
(e) H3PO4, source of H2PO4 , equilibrium shifts right, pH decreases.
(f) Na3PO4, source of PO4 , decreases [H3O+] by forming HPO42 , pH increases.
3S S

16.126 pH = 10.35; [H3O+] = 10SpH = 10S10.35 = 4.5 x 10S11 M


1.0 x 10 _14
[OHS] = K w + = = 2.2 x 10S4 M
[H 3O ] 4.5 x 10 _11
_
[ ] 2.2 x 10 _ 4
[Mg2+] = OH = = 1.1 x 10 4 M
S

2 2
Ksp = [Mg2+][OH ]2 = (1.1 x 10 4)(2.2 x 10 4)2 = 5.3 x 10 12
S S S S

16.127 mmol Hg22+ = (0.010 mmol/mL)(1.0 mL) = 0.010 mmol


mmol ClS = (6 mmol/mL)( 0.05 mL) = 0.3 mmol
Assume complete reaction.
Hg22+(aq) + 2 Cl (aq)  Hg2Cl2(s)
S

before reaction (mmol) 0.010 0.3


change (mmol) S0.010 S2(0.010)
after reaction (mmol) 0 0.28
0.28 mmol
[ClS] = = 0.27 M
1.05 mL
Allow Hg2Cl2 to establish a new equilibrium.
Hg2Cl2(s) Ω Hg22+(aq) + 2 Cl (aq)
S

initial (M) 0 0.27


equil (M) x 0.27 + 2x
Ksp = [Hg22+][Cl ]2 = 1.4 x 10 18 = x(0.27 + 2x)2 . x(0.27)2
S S

1.4 x 10 _18
x = [Hg22+] = = 2 x 10 17 mol/L
S

(0.27) 2
Hg22+, 401.18 amu
Hg22+ concentration = (2 x 10 17 mol/L)(401.18 g/mol) = 8 x 10 15 g/L
S S

471
Chapter 16 S Applications of Aqueous Equilibria
______________________________________________________________________________

1 mol
16.128 NaOH, 40.0 amu; 20 g x = 0.50 mol NaOH
40.0 g
(0.500 L)(1.5 mol/L) = 0.75 mol NH4Cl
NH4+(aq) + OHS(aq) Ω NH3(aq) + H2O(l)
before reaction (mol) 0.75 0.50 0
change (mol) S0.50 S0.50 +0.50
after reaction (mol) 0.25 0 0.50
This reaction produces a buffer solution.
[NH4+] = 0.25 mol/0.500 L = 0.50 M; [NH3] = 0.50 mol/0.500 L = 1.0 M
[base] [ NH 3]
pH = pKa + log = pK a + log
[acid] [ NH +4]
Kw 1.0 x 10 _14
For NH4+, Ka = = _5
= 5.6 x 10S10; pKa = Slog Ka = 9.25
Kb for NH 3 1.8 x 10
 1.0 
pH = 9.25 + log   = 9.55
 0.5 

16.129 (a) AgCl, Ksp = [Ag+][Cl ] = 1.8 x 10 10


S S

K sp 1.8 x 10 _10
= 6.0 x 10 9 M
S S
[Cl ] = +
=
[Ag ] 0.030
(b) Hg2Cl2, Ksp = [Hg22+][Cl ]2 = 1.4 x 10 18
S S

K sp 1.4 x 10 _18
[ClS] = 2+
= = 6.8 x 10S9 M
[Hg 2 ] 0.030
(c) PbCl2, Ksp = [Pb2+][Cl ]2 = 1.2 x 10
S S5

S K sp 1.2 x 10 _ 5
[Cl ] = = = 0.020 M
[Pb 2+ ] 0.030
AgCl(s) will begin to precipitate when the [ClS] just exceeds 6.0 x 10S9 M. At this ClS
concentration, IP < Ksp for PbCl2 so all of the Pb2+ will remain in solution.
Kw 1.0 x 10 _14
16.130 For NH4+, Ka = = 5.6 x 10 10; pKa = Slog Ka = 9.25
S
= _5
K b for NH 3 1.8 x 10
[ ] (0.50)
pH = pKa + log NH +3 = 9.25 + log = 9.47
[ NH 4 ] (0.30)
[H3O+] = 10 pH = 10 9.47 = 3.4 x 10 10 M
S S S

2+
[ ][ 2 S]
For MnS, Kspa = Mn H + 2
= 3 x 1010
[H 3O ]
+ 2 2
2+ K spa[H 3O ] (3 x 1010)(3.4 x 10 _10)
= 3.5 x 10 8 M
S
molar solubility = [Mn ] = =
[H 2 S] (0.10)
MnS, 87.00 amu; solubility = (3.5 x 10 mol/L)(87.00 g/mol) = 3 x 10S6 g/L
S8

[H3O+] = 10
SpH S9.00
= 1.0 x 10 9 M
S
16.131 pH = 9.00; = 10

472
Chapter 16 S Applications of Aqueous Equilibria
______________________________________________________________________________
_14
S
[OH ] = K w = 1.0 x 10 = 1.0 x 10S5 M
[H 3O + ] 1.0 x 10 _ 9
Mg(OH)2(s) Ω Mg2+(aq) + 2 OHS(aq)
equil (M) x 1.0 x 10S5 (fixed by buffer)
2+ S 2 S12 S5 2
Ksp = [Mg ][OH ] = 5.6 x 10 = x(1.0 x 10 )
5.6 x 10 _12
molar solubility = x = = 0.056 M
(1.0 x 10 _ 5) 2

16.132 60.0 mL = 0.0600 L


1.00 mol H 3PO 4
mol H3PO4 = 0.0600 L x = 0.0600 mol H3PO4
1.00 L
0.100 mol LiOH
mol LiOH = 1.00 L x = 0.100 mol LiOH
1.00 L

H3PO4(aq) + OH (aq)  H2PO4 (aq) + H2O(l)


S S

before reaction (mol) 0.0600 0.100 0


change (mol) S0.0600 S0.0600 +0.0600
after reaction (mol) 0 0.040 0.0600

H2PO4 (aq) + OH (aq)  HPO42 (aq) + H2O(l)


S S S

before reaction (mol) 0.0600 0.040 0


change (mol) S0.040 S0.040 +0.040
after reaction (mol) 0.020 0 0.040
The resulting solution is a buffer because it contains the conjugate acid-base pair,
H2PO4S and HPO42S, at acceptable buffer concentrations.
For H2PO4 , Ka2 = 6.2 x 10 8 and pKa2 = S log Ka2 = S log (6.2 x 10 8) = 7.21
S S S

2_
[ ] (0.040 mol / 1.06 L)
pH = pKa2 + log HPO 4 _ = 7.21 + log
[H 2 PO 4 ] (0.020 mol / 1.06 L)
(0.040)
pH = 7.21 + log = 7.21 + 0.30 = 7.51
(0.020)

16.133 (a) The mixture of 0.100 mol H3PO4 and 0.150 mol NaOH is a buffer and contains
mainly H2PO4 and HPO42 from the reactions:
S S

H3PO4(aq) + OHS(aq)  H2PO4S(aq) + H2O(l)


before (mol) 0.100 0.150 0
change (mol) S0.100 S0.100 +0.100
after (mol) 0 0.050 0.100

H2PO4S(aq) + OHS(aq)  HPO42S(aq) + H2O(l)


before (mol) 0.100 0.050 0
change (mol) S0.050 S0.050 +0.050
after (mol) 0.050 0 0.050

473
Chapter 16 S Applications of Aqueous Equilibria
______________________________________________________________________________

If water were used to dilute the solution instead of HCl, the pH would be equal to pKa2
because [H2PO4S] = [HPO42S] = 0.050 mol/1.00 L = 0.050 M
H2PO4 (aq) + H2O(l) Ω H3O+(aq) + HPO42 (aq) Ka2 = 6.2 x 10 8
S S S

S8
pKa2 = Slog K2a = Slog(6.2 x 10 ) = 7.21
2_
[ ]
pH = pKa2 + log HPO 4 _ = pKa2 + log(1) = pKa2 = 7.21
[H 2 PO 4 ]

The pH is lower (6.73) because the added HCl converts some HPO42 to H2PO4 .
S S

HPO42S(aq) + H3O+(aq)  H2PO4S(aq) + H2O(l)


before (M) 0.050 x 0.050
change (M) Sx Sx +x
after (M) 0.050 S x 0 0.050 + x
[HPO42 ] + [H2PO4 ] = (0.050 S x) + (0.050 + x) = 0.100 M
S S

2_
[ ]
pH = pKa2 + log HPO 4 _
[H 2 PO 4 ]
2S S
[HPO4 ] = 0.100 S [H2PO4 ]
(0.100 _ [H 2 PO 4_ ])
6.73 = 7.21 + log
[H 2 PO 4_ ]
(0.100 _ [H 2 PO 4_ ])
6.73 S 7.21 = S0.48 = log
[H 2 PO 4_ ]
(0.100 _ [H 2 PO 4_ ])
10 0.48 = 0.331 =
S

[H 2 PO 4_ ]
(0.331)[H2PO4S] = 0.100 S [H2PO4S]
S
(1.331)[H2PO4 ] = 0.100
S
[H2PO4 ] = 0.100/1.331 = 0.075 M
2S S
[HPO4 ] = 0.100 S [H2PO4 ] = 0.100 S 0.075 = 0.025 M

H3PO4(aq) + H2O(l) Ω H3O+(aq) + H2PO4 (aq)


S S3
Ka1 = 7.5 x 10
+ _
[ ][ ]
Ka1 = H 3O H 2 PO 4
[H 3PO 4]
+ _
[ ][ ]
[H3PO4] = H 3O H 2 PO 4
K a1
[H3O+] = 10 pH = 10 6.73 = 1.86 x 10 7 M
S S S

(1.86 x 10 _ 7)(0.075)
= 1.9 x 10 6 M
S
[H3PO4] = _3
7.5 x 10

If distilled water were used and not HCl, the mole amounts of both H2PO4 and HPO42
S S
(b)
would be 0.050 mol. The HCl converted some HPO42 to H2PO4 .
S S

HPO42S(aq) + H3O+(aq)  H2PO4S(aq) + H2O(l)


before (mol) 0.050 x 0.050
change (mol) Sx Sx +x
after (mol) 0.050 S x 0 0.050 + x

474
Chapter 16 S Applications of Aqueous Equilibria
______________________________________________________________________________

From part (a), [HPO42 ] = 0.025 M


S

mol HPO42S = (0.025 mol/L)(1.00 L) = 0.025 mol = 0.050 S x


x = mol H3O+ = mol HCl inadvertently added = 0.050 S 0.025 = 0.025 mol HCl

16.134 For CH3CO2H, Ka = 1.8 x 10 5 and pKa = Slog Ka = Slog(1.8 x 10 5) = 4.74


S S

The mixture will be a buffer solution containing the conjugate acid-base pair, CH3CO2H
S
and CH3CO2 , having a pH near the pKa of CH3CO2H.
_
[ ]
pH = pKa + log CH 3CO 2
[CH 3CO 2 H]
_ _
[ ] [ ]
4.85 = 4.74 + log CH 3CO 2 ; 4.85 S 4.74 = log CH 3CO 2
[CH 3CO 2 H] [CH 3CO 2 H]
_ _
[ ] [CH 3CO 2 ]
0.11 = log CH 3CO 2 ; = 100.11 = 1.3
[CH 3CO 2 H] [CH 3CO 2 H]
In the Henderson-Hasselbalch equation, moles can be used in place of concentrations
because both components are in the same volume so the volume terms cancel.
20.0 mL = 0.0200 L

Let X equal the volume of 0.10 M CH3CO2H and Y equal the volume of 0.15 M
S
CH3CO2 . Therefore, X + Y = 0.0200 L and
Y x [CH 3CO 2_ ] Y (0.15 mol/L)
= = 1.3
X x [CH 3CO 2 H] X (0.10 mol/L)
X = 0.0200 S Y
Y (0.15 mol/L)
= 1.3
(0.020 _ Y)(0.10 mol/L)
0.15 Y
= 1.3
0.0020 _ 0.10 Y
0.15Y = 1.3(0.0020 S 0.10Y)
0.15Y = 0.0026 S 0.13Y
0.15Y + 0.13Y = 0.0026
0.28Y = 0.0026
Y = 0.0026/0.28 = 0.0093 L
X = 0.0200 S Y = 0.0200 S 0.0093 = 0.0107 L
X = 0.0107 L = 10.7 mL and Y = 0.0093 L = 9.3 mL
You need to mix together 10.7 mL of 0.10 M CH3CO2H and 9.3 mL of 0.15 M
NaCH3CO2 to prepare 20.0 mL of a solution with a pH of 4.85.

16.135 [H3O+] = 10 pH = 10 2.37 = 0.004 27 M


S S

H3Cit(aq) + H2O(l) Ω H3O+(aq) + H2CitS(aq)


+ _
[ ][ ]
Ka1 = 7.1 x 10S4 = H 3O H 2Cit
[H 3 Cit]
S4 S
(7.1 x 10 )[H3Cit] = (0.004 27)[H2Cit ]
[H3Cit] = (0.004 27)[H2CitS]/(7.1 x 10S4) = (6.01)[H2CitS]

475
Chapter 16 S Applications of Aqueous Equilibria
______________________________________________________________________________

H2Cit (aq) + H2O(l) Ω H3O+(aq) + HCit2 (aq)


S S

+ 2_
[ ][ ]
Ka2 = 1.7 x 10 5 = H 3O HCit
S
_
[H 2Cit ]
(1.7 x 10 )[H2Cit ] = (0.004 27)[HCit2 ]
S5 S S

[HCit2 ] = (1.7 x 10 5)[H2Cit ]/(0.004 27) = (0.003 98)[H2Cit ]


S S S S

S 2S 3S
[H3Cit] + [H2Cit ] + [HCit ] + [Cit ] = 0.350 M
Now assume [Cit3 ] . 0, so [H3Cit] + [H2Cit ] + [HCit2 ] = 0.350 M and then by
S S S

substitution:
(6.01)[H2CitS] + [H2CitS] + (0.003 98)[H2CitS] = 0.350 M
S
(7.01)[H2Cit ] = 0.350 M
S
[H2Cit ] = 0.350 M/7.01 = 0.050 M
S
[H3Cit] = (6.01)[H2Cit ] = (6.01)(0.050 M) = 0.30 M
[HCit ] = (0.003 98)[H2Cit ] = (0.003 98)(0.050 M) = 2.0 x 10 4 M
2S S S

HCit2 (aq) + H2O(l) Ω H3O+(aq) + Cit3 (aq)


S S

+
[ ][ 3 _ ]
Ka3 = 4.1 x 10 7 = H 3O Cit
S

[HCit 2 _ ]
2_
( )[ ] (4.1 x 10 _ 7)(2.0 x 10 _ 4)
[Cit3 ] = K a 3 HCit = 1.9 x 10 8 M
S S
+
=
[H 3O ] (0.004 27)

16.136 (a) HCl is a strong acid. HCN is a weak acid with Ka = 4.9 x 10S10. Before the titration,
the [H3O+] = 0.100 M. The HCN contributes an insignificant amount of additional
H3O+, so the pH = Slog[H3O+] = Slog(0.100) = 1.00
(b) 100.0 mL = 0.1000 L
0.100 mol HCl
mol H3O+ = 0.1000 L x = 0.0100 mol H3O+
1.00 L
add 75.0 mL of 0.100 M NaOH; 75.0 mL = 0.0750 L
S 0.100 mol NaOH S
mol OH = 0.0750 L x = 0.00750 mol OH
1.00 L

H3O+(aq) + OHS(aq)  2 H2O(l)


before reaction (mol) 0.0100 0.0075
change (mol) S0.0075 S0.0075
after reaction (mol) 0.0025 0
0.0025 mol H 3O +
[H3O+] = = 0.0143 M
0.1000 L + 0.0750 L
pH = Slog[H3O+] = Slog(0.0143) = 1.84
(c) 100.0 mL of 0.100 M NaOH will completely neutralize all of the H3O+ from 100.0
mL of 0.100 M HCl. Only NaCl and HCN remain in the solution. NaCl is a neutral salt
and does not affect the pH of the solution. [HCN] changes because of dilution. Because
the solution volume is doubled, [HCN] is cut in half.

476
Chapter 16 S Applications of Aqueous Equilibria
______________________________________________________________________________

[HCN] = 0.100 M/2 = 0.0500 M

HCN(aq) + H2O(l) Ω H3O+(aq) + CN (aq)


S

initial (M) 0.0500 ~0 0


change (M) Sx +x +x
equil (M) 0.0500 S x x x
[H 3O + ][CN _ ] 2 2
Ka = = 4.9 x 10 10 =
S x . x
HCN 0.0500 _ x 0.0500
[H3O+] = x = (0.0500)(4.9 x 10 _10) = 4.95 x 10 6 M
S

pH = Slog[H3O+] = Slog(4.95 x 10 6) = 5.31


S

(d) Add an additional 25.0 mL of 0.100 M NaOH.


25.0 mL = 0.0250 L
S 0.100 mol NaOH S
additional mol OH = 0.0250 L x = 0.00250 mol OH
1.00 L
0.0500 mol HCN
mol HCN = 0.200 L x = 0.0100 mol HCN
1.00 L
HCN(aq) + OHS(aq)  CNS(aq) + H2O(l)
before reaction (mol) 0.0100 0.00250 0
change (mol) S0.00250 S0.00250 +0.00250
after reaction (mol) 0.0075 0 0.00250
The resulting solution is a buffer because it contains the conjugate acid-base pair, HCN
S
and CN , at acceptable buffer concentrations.
For HCN, Ka = 4.9 x 10 10 and pKa = S log Ka = S log (4.9 x 10 10) = 9.31
S S

[ _] (0.00250 mol / 0.2250 L)


pH = pKa + log CN = 9.31 + log
[HCN] (0.0075 mol / 0.2250 L)
(0.00250)
pH = 9.31 + log = 9.31 _ 0.48 = 8.83
(0.0075)

Cd(OH)2(s) Ω Cd2+(aq) + 2 OH (aq)


S
16.137 (a)
initial (M) 0 ~0
equil (M) x 2x
Ksp = [Cd ][OH ] = 5.3 x 10 = (x)(2x) = 4x3
2+ S 2 S15 2

5.3 x 10 _15
= 1.1 x 10 5 M
S
molar solubility = x = 3
4
[OHS] = 2x = 2(1.1 x 10S5 M) = 2.2 x 10S5 M
1.0 x 10 _14
[H3O+] = _5
= 4.5 x 10 10 M
S

2.2 x 10
pH = Slog[H3O+] = Slog(4.5 x 10 10) = 9.35
S

(b) 90.0 mL = 0.0900 L


mol HNO3 = (0.100 mol/L)(0.0900 L) = 0.009 00 mol HNO3
The addition of HNO3 dissolves some Cd(OH)2(s).
Cd(OH)2(s) + 2 HNO3(aq)  Cd2+(aq) + 2 H2O(l)

477
Chapter 16 S Applications of Aqueous Equilibria
______________________________________________________________________________

1.1 x 10 5
S
before (mol) 0.100 0.009 00
change (mol) S0.0045 S2(0.0045) 1.1 x 10S5 + 0.0045
after (mol) 0.0955 0 ~0.0045
total volume = 100.0 mL + 90.0 mL = 190.0 mL = 0.1900 L
[Cd2+] = 0.0045 mol/0.1900 L = 0.024 M
Ksp = 5.3 x 10 15 = [Cd2+][OH ]2 = (0.024)[OH ]2
S S S

5.3 x 10 _15
= 4.7 x 10 7 M
S S
[OH ] =
0.024
1.0 x 10 _14
[H3O+] = = 2.1 x 10 8 M
S
_7
4.7 x 10
pH = Slog[H3O+] = Slog(2.1 x 10 8) = 7.68
S

2 mol HNO3 1.00 L 1000 mL


(c) volume HNO3 = 0.0100 mol Cd(OH)2 x x x = 200
1 mol Cd(OH ) 2 0.100 mol 1.00 L
mL
Zn(OH)2(s) Ω Zn2+(aq) + 2 OH (aq)
S
16.138 (a)
initial (M) 0 ~0
equil (M) x 2x
Ksp = [Zn ][OH ] = 4.1 x 10 = (x)(2x) = 4x3
2+ S 2 S17 2

4.1 x 10 _17
molar solubility = x = 3 = 2.2 x 10S6 M
4
(b) [OH ] = 2x = 2(2.2 x 10 M) = 4.4 x 10 6 M
S S6 S

1.0 x 10 _14
[H3O+] = = 2.3 x 10 9 M
S
_6
4.4 x 10
pH = Slog[H3O+] = Slog(2.3 x 10S9) = 8.64
Zn(OH)2(s) Ω Zn2+(aq) + 2 OH (aq) Ksp = 4.1 x 10 17
S S
(c)
Zn2+(aq) + 4 OH (aq) Ω Zn(OH)42 (aq) Kf = 3 x 1015
S S

Zn(OH)2(s) + 2 OH (aq) Ω Zn(OH)42 (aq) K = Ksp≅Kf = 0.123


S S

initial (M) 0.10 0


change (M) S2x +x
equil (M) 0.10 S 2x x
2_
[Zn(OH ) 4 ] x
K= 2
= 0.123 = 2
[OH _ ] (0.10 _ 2 x )
0.492x2 S 1.0492x + 0.00123 = 0
Use the quadratic formula to solve for x.
_ (_ 1.0492) ± (_ 1.0492) _ (4)(0.492)(0.00123) 1.0492 ± 1.0480
2

x= =
2(0.492) 0.984
x = 2.1 and 1.2 x 10S3
Of the two solutions for x, only 1.2 x 10S3 has physical meaning because the other
S
solution leads to a negative [OH ].
molar solubility of Zn(OH)42 in 0.10 M NaOH = x = 1.2 x 10 3 M
S S

478
Chapter 16 S Applications of Aqueous Equilibria
______________________________________________________________________________

16.139 (a) Fe(OH)3(s) Ω Fe3+(aq) + 3 OH (aq) Ksp = 2.6 x 10 39


S S

H3Cit(aq) + H2O(l) Ω H3O+(aq) + H2CitS(aq) Ka1 = 7.1 x 10S4


H2CitS(aq) + H2O(l) Ω H3O+(aq) + HCit2S(aq) Ka2 = 1.7 x 10S5
HCit2 (aq) + H2O(l) Ω H3O+(aq) + Cit3 (aq) Ka3 = 4.1 x 10 7
S S S

Fe3+(aq) + Cit3 (aq) Ω Fe(Cit)(aq) Kf = 6.3 x 1011


S

3 [H3O+(aq) + OHS(aq) Ω 2 H2O(l)] (1/Kw)3 = 1.0 x 1042


Fe(OH)3(s) + H3Cit(aq) Ω Fe(Cit)(aq) + 3 H2O(l)

K = Ksp Ka1 Ka2 Ka3Kf(1/Kw)3 = 8.1


(b) Fe(OH)3(s) + H3Cit(aq) Ω Fe(Cit)(aq) + 3 H2O(l)
initial (M) 0.500 0
change (M) Sx +x
equil (M) 0.500 S x x

[Fe(Cit)] x
K= = 8.1 =
[H 3 Cit] 0.500 _ x
8.1(0.500 S x) = x
4.05 S 8.1x = x
4.05 = 9.1x
x = molar solubility = 4.05/9.1 = 0.45 M

Multi-Concept Problems

16.140 (a) HA (aq) + H2O(l) Ω H3O+(aq) + A2 (aq) Ka2 = 10 10


S S S

HA (aq) + H2O(l) Ω H2A(aq) + OH (aq) Kb = K w = 10 10


S S S

K a1
2
2 HAS(aq) Ω H2A(aq) + A2S(aq) K = K a = 10S6
K a1
2 H2O(l) Ω H3O+(aq) + OHS(aq) Kw = 1.0 x 10S14

The principal reaction of the four is the one with the largest K, and that is the third
reaction.
+ _ +
[ ][ ] [ ][ 2 _ ]
(b) Ka1 = H 3O HA and Ka2 = H 3O _A
[H 2 A] [HA ]
_
1 [ A] 2[ ]
[H3O+] = K a H_2 and [H3O+] = K a 2HA _
[HA ] [A ]
_
K a1 [H 2 A] x K a 2 [HA ] = [H O+]2; K a1 K a 2 [H 2 A] = [H O+]2
_ 2_ 3 3
[HA ] [A ] [A 2 _ ]
Because the principal reaction is 2 HAS(aq) Ω H2A(aq) + A2S(aq), [H2A] = [A2S].
Ka1 Ka2 = [H3O+]2
log Ka1 + log Ka2 = 2 log [H3O+]

479
Chapter 16 S Applications of Aqueous Equilibria
______________________________________________________________________________

log K a1 + log K a 2 _ log K a1 + (_ log K a 2)


= log [H 3O + ] ; = _ log [H 3O + ]
2 2
p K a1 + p K a 2
= pH
2
(c) 2 HAS(aq) Ω H2A(aq) + A2S(aq)
initial (M) 1.0 0 0
change (M) S2x +x +x
equil (M) 1.0 S 2x x x

[H 2 A][A 2 _ ] S6 x
2
K= = 1 x 10 =
[HA _ ]2 (1.0 _ 2 x )
2

Take the square root of both sides and solve for x.


x = [A2 ] = 1 x 10 3 M
S S

mol A2S = (1 x 10S3 mol/L)(0.0500 L) = 5 x 10S5 mol A2S


number of A2S ions = (5 x 10S5 mol A2S)(6.022 x 1023 ions/mol) = 3 x 1019 A2S ions

16.141 (a) (i) en(aq) + H2O(l) Ω enH+(aq) + OHS(aq)


initial (M) 0.100 0 ~0
change (M) Sx +x +x
equil (M) 0.100 S x x x
+ _
[ ][ ] (x)(x)
Kb = enH OH = 5.2 x 10 _ 4 =
[en] 0.100 _ x
2 S4 S5
x + (5.2 x 10 )x S (5.2 x 10 ) = 0
Use the quadratic formula to solve for x.
_ (5.2 x 10 _ 4) ± (5.2 x 10 _ 4) 2 _ 4(1)(_ 5.2 x 10 _ 5) _ 5.2 x 10 _ 4 ± 0.01443
x= =
2(1) 2
x = S0.0075 and 0.0070
Of the two solutions for x, only the positive value of x has physical meaning because x is
S
the [OH ].
S
[OH ] = x = 0.0070 M
1.0 x 10 _14
[H3O+] = K w_ = = 1.43 x 10 12 M
S

[OH ] 0.0070
pH = Slog[H3O ] = Slog(1.43 x 10 12) = 11.84
+ S

(ii) (30.0 mL)(0.100 mmol/mL) = 3.00 mmol en


(15.0 mL)(0.100 mmol/mL) = 1.50 mmol HCl
S
Halfway to the first equivalence point, [OH ] = Kb1
1.0 x 10 _14
[H3O+] = K w_ = = 1.92 x 10S11 M
[OH ] 5.2 x 10 _ 4
pH = Slog[H3O+] = Slog(1.92 x 10 11) = 10.72
S

480
Chapter 16 S Applications of Aqueous Equilibria
______________________________________________________________________________

p K a1 + p K a 2
(iii) At the first equivalence point pH = = 9.14
2
(iv) Halfway between the first and second equivalence points, [OH ] = Kb2 = 3.7 x 10 7
S S

M
_14
[H3O+] = K w = 1.0 x 10 = 2.70 x 10S8 M
_ _7
[OH ] 3.7 x 10
pH = Slog[H3O+] = Slog(2.70 x 10 8) = 7.57
S

(v) At the second equivalence point only the acidic enH2Cl2 is in solution.
_14
For enH22+, Ka = Kw = K w = 1.0 x 10 = 2.70 x 10S8
+ _7
K b for enH K b 2 3.7 x 10
3.00 mmol
[enH22+] = = 0.0333 M
(30.0 mL + 60.0 mL)
enH22+(aq) + H2O(l) Ω H3O+(aq) + enH+(aq)
initial (M) 0.0333 ~0 0
change (M) Sx +x +x
equil (M) 0.0333 S x x x
+ + 2
[ ][ ] (x)(x)
Ka = H 3O enH 2+
= 2.70 x 10S8 = ≈ x
[enH 2 ] 0.0333 _ x 0.0333
Solve for x. x = [H3O+] = (2.70 x 10 _ 8)(0.0333) = 3.00 x 10 5 M
S

pH = Slog[H3O+] = Slog(3.00 x 10 5) = 4.52


S

(vi) excess HCl


(75.0 mL S 60.0 mL)(0.100 mmol/mL) = 1.50 mmol HCl = 1.50 mmol H3O+
1.50 mmol
[H3O+] = = 0.0143 M
(30.0 mL + 75.0 mL)
pH = Slog[H3O+] = Slog(0.0143) = 1.84

481
Chapter 16 S Applications of Aqueous Equilibria
______________________________________________________________________________

(b)
Each of the two nitrogens in ethylenediamine can
accept a proton.

(c) Each nitrogen is sp3 hybridized.

16.142 (a) The first equivalence point is reached when all the H3O+ from the HCl and the H3O+
form the first ionization of H3PO4 is consumed.
pK a1 + pK a 2
At the first equivalence point pH = = 4.66
2
[H3O+] = 10 pH = 10( 4.66) = 2.2 x 10 5 M
S S S

(88.0 mL)(0.100 mmol/mL) = 8.80 mmol NaOH are used to get to the first equivalence
point
(b) mmol (HCl + H3PO4) = mmol NaOH = 8.8 mmol
mmol H3PO4 = (126.4 mL S 88.0 mL)(0.100 mmol/mL) = 3.84 mmol
mmol HCl = (8.8 S 3.84) = 4.96 mmol
4.96 mmol 3.84 mmol
[HCl] = = 0.124 M; [H3PO4] = = 0.0960 M
40.0 mL 40.0 mL
(c) 100% of the HCl is neutralized at the first equivalence point.
H3PO4(aq) + H2O(l) Ω H3O+(aq) + H2PO4 (aq)
S
(d)
initial (M) 0.0960 0.124 0
change (M) Sx +x +x
equil (M) 0.0960 S x 0.124 + x x
+ _
[ ][ ] (0.124 + x)(x)
Ka1 = H 3O H 2 PO 4 = 7.5 x 10 3 =
S

[H 3PO 4] 0.0960 _ x
2 S4
x + 0.132x S (7.2 x 10 ) = 0
Use the quadratic formula to solve for x.
_ (0.132) ± (0.132) _ 4(1)(_ 7.2 x 10 _ 4) _ 0.132 ± 0.142
2

x= =
2(1) 2
x = S0.137 and 0.005
Of the two solutions for x, only the positive value of x has physical meaning because the
other solution would give a negative [H3O+].
[H3O+] = 0.124 + x = 0.124 + 0.005 = 0.129 M
pH = Slog[H3O+] = Slog(0.129) = 0.89
(e)

482
Chapter 16 S Applications of Aqueous Equilibria
______________________________________________________________________________

(f) Bromcresol green or methyl orange are suitable indicators for the first equivalence
point. Thymolphthalein is a suitable indicator for the second equivalence point.

16.143 (a) PV = nRT; 25oC = 298 K


 1.00 atm 
 732 mm Hg x 760 mm Hg  (1.000 L)
PV  
n HCl = = = 0.0394 mol HCl
RT  L • atm 
 0.082 06  (298 K)
 K • mol 
Na2CO3, 105.99 amu
1 mol Na 2CO3
mol Na2CO3 = 6.954 g Na2CO3 x = 0.0656 mol Na2CO3
105.99 g Na 2CO3
CO32S(aq) + H3O+(aq)  HCO3S(aq) + H2O(l)
before reaction (mol) 0.0656 0.0394 0
change (mol) S0.0394 S0.0394 +0.0394
after reaction (mol) 0.0656 S 0.0394 0 0.0394

mol CO32S = 0.0656 S 0.0394 = 0.0262 mol and mol HCO3S = 0.0394 mol
Therefore, we have an HCO3S/CO32S buffer solution.
[ 2_ ] 0.0262 mol/V
pH = pKa2 + log CO3 _ = S log(5.6 x 10S11) + log
[HCO3 ] 0.0394 mol/V
pH = 10.25 S 1.77 = 10.08

(b) mol Na+ = 2(0.0656 mol) = 0.1312 mol


mol CO32 = 0.0262 mol
S

S
mol HCO3 = 0.0394 mol
S
mol Cl = 0.0394 mol
total ion moles = 0.2362 mol
 o
C • kg  0.2362 mol 
∆Tf = Kf Α m, ∆Tf = 1.86 
o
 = 1.76 C
 mol  0.2500 kg 
Solution freezing point = 0 C S ∆Tf = S1.76oC
o

483
Chapter 16 S Applications of Aqueous Equilibria
______________________________________________________________________________

(c) H2O, 18.02 amu


1 mol H 2 O
mol H2O = 250.0 g x = 13.87 mol H2O
18.02 g H 2 O
mol H 2 O 13.87 mol
Xsolv = = = 0.9833
mol H 2 O + mol ions 13.87 mol + 0.2362 mol
Psoln = Psolv ≅ Xsolv = (23.76 mm Hg)(0.9833) = 23.36 mm Hg

16.144 25oC = 298 K


 1.00 atm 
 74.4 mm Hg x 
Π  760 mm Hg 
Π = 2MRT; M = = = 0.00200 M
2 RT  L • atm 
(2)  0.082 06  (298 K)
 K• mol 
[M+] = [X ] = 0.00200 M
S

Ksp = [M+][X ] = (0.00200)2 = 4.00 x 10 6


S S

16.145 (a) HCO3 (aq) + OH (aq)  CO32 (aq) + H2O(l)


S S S

S S
(b) mol HCO3 = (0.560 mol/L)(0.0500 L) = 0.0280 mol HCO3
S S
mol OH = (0.400 mol/L)(0.0500 L) = 0.0200 mol OH
HCO3 (aq) + OH (aq)  CO32 (aq) + H2O(l)
S S S

before reaction (mol) 0.0280 0.0200 0


change (mol) S0.0200 S0.0200 +0.0200
after reaction (mol) 0.0280 S 0.0200 0 0.0200
S
mol HCO3 = 0.0280 S 0.0200 = 0.0080 mol
0.0080 mol 0.0200 mol
[CO32 ] =
S S
[HCO3 ] = = 0.080 M = 0.200 M
0.1000 L 0.1000 L
HCO3S(aq) + H2O(l) Ω H3O+(aq) + CO32S(aq)
initial (M) 0.080 ~0 0.200
change (M) Sx +x +x
equil (M) 0.080 S x x 0.200 + x
+ 2_
[ ][ 3 ] x(0.200 + x) x(0.200)
Ka = H 3O CO _
= 5.6 x 10 _11 = ≈
[HCO 3 ] 0.080 _ x 0.080
+ S11
Solve for x. x = [H3O ] = 2.24 x 10 M
pH = Slog[H3O+] = Slog(2.24 x 10 11) = 10.65
S

S
Because this solution contains both a weak acid (HCO3 ) and its conjugate base, the
solution is a buffer.
(c) HCO3 (aq) + OH (aq)  CO32 (aq) + H2O(l)
S S S

∆H rxn = [∆H f(CO3 ) + ∆H f(H2O)] S [∆Hof(HCO3 ) + ∆Hof(OH )]


o o 2S o S S

∆Horxn = [(1 mol)(S677.1 kJ/mol) + (1 mol)(S285.8 kJ/mol)]


S [(1 mol)(S692.0 kJ/mol) + (1 mol)(S230 kJ/mol)]
o
∆H rxn = S 40.9 kJ
S S
0.0200 moles each of HCO3 and OH reacted.
heat produced = q = (0.0200 mol)(40.9 kJ/mol) = 0.818 kJ = 818 J

484
Chapter 16 S Applications of Aqueous Equilibria
______________________________________________________________________________

(d) q = m x specific heat x ∆T


q 818 J
∆T = = = 2.0oC
m x specific heat (100.0 g)[4.18 J/(g •o C)]
Final temperature = 25oC + 2.0oC = 27oC

16.146 (a) species present initially:


NH4+ CO32
S
H2O
acid base acid or base
2H2O(l) Ω H3O+(aq) + OH (aq)
S

NH4+(aq) + H2O(l) Ω NH3(aq) + H3O+(aq)


CO32S(aq) + H2O(l) Ω HCO3S(aq) + OHS(aq)

NH3, Kb = 1.8 x 10 5
S

NH4+, Ka = 5.6 x 10 10
S

CO32 , Kb = 1.8 x 10 4
S S

HCO3 , Ka = 5.6 x 10S11


S

In the mixture, proton transfer takes place from the stronger acid to the stronger base, so
the principal reaction is NH4+(aq) + CO32S(aq) Ω HCO3S(aq) + NH3(aq)

(b) NH4+(aq) + OHS(aq) Ω NH3(aq) + H2O(l) K1 = 1/Kb(NH3)


CO3 (aq) + H2O(l) Ω HCO3 (aq) + OH (aq)
2S
K2 = Kb(CO32 )
S S S

NH4+(aq) + CO32 (aq) Ω HCO3 (aq) + NH3(aq) K = K1≅K2


S S

initial (M) 0.16 0.080 0 0.16


change (M) Sx Sx +x +x
equil (M) 0.16 S x 0.080 S x x 0.16 + x
_
[ ][ 3] 1.8 x 10 _ 4 x(0.16 + x)
K = HCO+ 3 NH 2_
= _5
= 10 =
[ NH 4 ][CO 3 ] 1.8 x 10 (0.16 _ x)(0.080 _ x)
2
9x S 2.56x + 0.128 = 0
Use the quadratic formula to solve for x.
_ (_ 2.56) ± (_ 2.56) 2 _ (4)(9)(0.128) 2.56 ± 1.395
x= =
2(9) 18
x = 0.220 and 0.0647
Of the two solutions for x, only 0.00647 has physical meaning because 0.220 leads to
negative concentrations.
[NH4+] = 0.16 S x = 0.16 S 0.0647 = 0.0953 M = 0.095 M
[NH3] = 0.16 + x = 0.16 + 0.0647 = 0.225 M = 0.22 M
[CO32 ] = 0.080 S x = 0.080 S 0.0647 = 0.0153 M = 0.015 M
S

S
[HCO3 ] = x = 0.0647 M = 0.065 M
The solution is a buffer containing two different sets of conjugate acid-base pairs. Either
pair can be used to calculate the pH.
For NH4+, Ka = 5.6 x 10S10 and pKa = 9.25

485
Chapter 16 S Applications of Aqueous Equilibria
______________________________________________________________________________

[ ] (0.225)
pH = pKa + log NH +3 = 9.25 + log = 9.62
[ NH 4 ] (0.0953)
[H3O+] = 10SpH = 10S9.62 = 2.4 x 10S10 M
1.0 x 10 _14
= 4.2 x 10 5 M
S S
[OH ] = _10
2.4 x 10
_ +
[ ][ ] (0.647)(2.4 x 10 _10)
[H2CO3] = HCO 3 H 3O = = 3.6 x 10 4 M
S
_7
Ka (4.3 x 10 )
(c) For MCO3, IP = [M ][CO3 ] = (0.010)(0.0153) = 1.5 x 10 4
2+ 2S S

Ksp(CaCO3) = 5.0 x 10 9, 103 Ksp = 5.0 x 10 6


S S

S9 3 S6
Ksp(BaCO3) = 2.6 x 10 , 10 Ksp = 2.6 x 10
Ksp(MgCO3) = 6.8 x 10 6, 103 Ksp = 6.8 x 10 3
S S

IP > 10 Ksp for CaCO3 and BaCO3, but IP < 103 Ksp for MgCO3 so the [CO32S] is large
3

enough to give observable precipitation of CaCO3 and BaCO3, but not MgCO3.
(d) For M(OH)2, IP = [M2+][OH ]2 = (0.010)(4.17 x 10 5)2 = 1.7 x 10 11
S S S

Ksp(Ca(OH)2) = 4.7 x 10 , 103 Ksp = 4.7 x 10 3


S6 S

Ksp(Ba(OH)2) = 5.0 x 10 3, 103 Ksp = 5.0


S

Ksp(Mg(OH)2) = 5.6 x 10 12, 103 Ksp = 5.6 x 10 9


S S

IP < 103 Ksp for all three M(OH)2. None precipitate.


CO32 (aq) + H2O(l) Ω HCO3 (aq) + OH (aq)
S S S
(e)
initial (M) 0.08 0 ~0
change (M) Sx +x +x
equil (M) 0.08 S x x x
[HCO 3_ ][OH _ ] x
2
= 1.8 x 10 4 =
S
Kb = 2_
[CO 3 ] (0.08 _ x)
x2 + (1.8 x 10 4)x S (1.44 x 10 5) = 0
S S

Use the quadratic formula to solve for x.


x=
_ (1.8 x 10 _ 4) ± (1.8 x 10 _ 4) _ (4)(1)(_ 1.44 x 10 _ 5) _ (1.8 x 10 _ 4) ± 7.59 x 10 _ 3
2

=
2(1) 2
x = 0.0037 and S0.0039
Of the two solutions for x, only 0.0037 has physical meaning because S0.0039 leads to
negative concentrations.
[OH ] = x = 3.7 x 10 3 M
S S

For MCO3, IP = [M2+][CO32 ] = (0.010)(0.08) = 8.0 x 10 4


S S

For M(OH)2, IP = [M ][OH ] = (0.010)(3.7 x 10 ) = 1.4 x 10 7


2+ S 2 S3 2 S

Comparing IP=s here and 103 Ksp=s in (c) and (d) above, Ca2+ and Ba2+ cannot be
separated from Mg2+ using 0.08 M Na2CO3. Na2CO3 is more basic than (NH4)2CO3 and
Mg(OH)2 would precipitate along with CaCO3 and BaCO3.

16.147 (a) H2SO4, 98.09 amu


Assume 1.00 L = 1000 mL of solution.
mass of solution = (1000 mL)(1.836 g/mL) = 1836 g
mass H2SO4 = (0.980)(1836 g) = 1799 g H2SO4

486
Chapter 16 S Applications of Aqueous Equilibria
______________________________________________________________________________

1 mol H 2SO 4
mol H2SO4 = 1799 g H2SO4 x = 18.3 mol H2SO4
98.09 g H 2SO 4
[H2SO4] = 18.3 mol/ 1.00 L = 18.3 M
(b) Na2CO3, 105.99 amu; 1 kg = 1000 g = 2.2046 lb
H2SO4(aq) + Na2CO3(s)  Na2SO4(aq) + H2O(l) + CO2(g)
2000 lb 1000 g
mass H2SO4 = (0.980)(36 tons) x x = 3.20 x 107 g H2SO4
1 ton 2.2046 lb
1 mol H 2SO 4
mol H2SO4 = 3.20 x 107 g H2SO4 x = 3.26 x 105 mol H2SO4
98.09 g H 2SO 4
1 mol Na 2CO 3 105.99 g Na 2CO3
mass Na2CO3 = 3.26 x 105 mol H2SO4 x x x
1 mol H 2SO 4 1 mol Na 2CO3
1 kg
= 3.5 x 104 kg Na2CO3
1000 g
1 mol CO 2
(c) mol CO2 = 3.26 x 105 mol H2SO4 x = 3.26 x 105 mol CO2
1 mol H 2SO 4
o
18 C = 18 + 273 = 291 K
PV = nRT
 L • atm 
(3.26 x 105 mol)  0.082 06  (291 K)
nRT  K • mol 
V= = = 7.9 x 106 L
P  1.00 atm 
 745 mm Hg x 760 mm Hg 
 

16.148 Pb(CH3CO2)2, 325.29 amu; PbS, 239.27 amu


1 mol Pb(CH 3CO 2) 2
(a) mass PbS = (2 mL)(1 g/mL)(0.003) x x
325.29 g Pb(CH 3CO 2) 2
1 mol PbS 239.27 g PbS
x x (30 /100) = 0.0013 g
1 mol Pb(CH 3CO 2) 2 1 mol PbS
= 1.3 mg PbS per dye application

(b) [H3O+] = 10 pH = 10 5.50 = 3.16 x 10 6 M


S S S

PbS(s) + 2 H3O+(aq) Ω Pb2+(aq) + H2S(aq) + 2 H2O(l)


3.16 x 10 6
S
initial (M) 0 0
change (M) S2x +x +x
equil (M) 3.16 x 10S6 S 2x x x
[Pb 2+][H 2 S] x
2
x
2
≈ = 3 x 10 7
S
Kspa = + 2
= _6 2 _6 2
[ H 3O ] (3.16 x 10 _ 2 x ) (3.16 x 10 )
2 S6 2 S7 S18
x = (3.16 x 10 ) (3 x 10 ) = 3.0 x 10
x = 1.7 x 10S9 M = [Pb2+] for a saturated solution.
mass of PbS dissolved per washing =

487
Chapter 16 S Applications of Aqueous Equilibria
______________________________________________________________________________

239.27 g PbS
(3 gal)(3.7854 L/1 gal)(1.7 x 10 9 mol/L) x = 4.7 x 10 6 g PbS/washing
S S

1 mol PbS
Number of washings required to remove 50% of the PbS from one application =
(0.0013 g PbS)(50 /100)
_6
= 1.4 x 102
(4.7 x 10 g PbS/washing)
washings
(c) The number of washings does not look reasonable. It seems too high considering
that frequent dye application is recommended. If the PbS is located mainly on the
surface of the hair, as is believed to be the case, solid particles of PbS can be lost by
abrasion during shampooing.

488
Thermodynamics:
17 Entropy, Free Energy, and
Equilibrium

17.1 (a) spontaneous; (b), (c), and (d) nonspontaneous

17.2 (a) H2O(g) → H2O(l)


A liquid is more ordered than a gas. Therefore, ∆S is negative.
(b) I2(g) → 2 I(g)
∆S is positive because the reaction increases the number of gaseous particles from 1 mol
to 2 mol.
(c) CaCO3(s) → CaO(s) + CO2(g)
∆S is positive because the reaction increases the number of gaseous molecules.
(d) Ag+(aq) + Br-(aq) → AgBr(s)
A solid is more ordered than +1 and -1 charged ions in an aqueous solution. Therefore,
∆S is negative.

17.3 (a) A2 + AB3 → 3 AB


(b) ∆S is positive because the reaction increases the number of gaseous molecules.

17.4 (a) disordered N2O


(b) silica glass (amorphous solid, more disorder)
(c) 1 mole N2 at STP (larger volume, more disorder)
(d) 1 mole N2 at 273 K and 0.25 atm (larger volume, more disorder)

17.5 CaCO3(s) → CaO(s) + CO2(g)


∆So = [So(CaO) + So(CO2)] - So(CaCO3)
∆So = [(1 mol)(39.7 J/(K⋅ mol)) + (1 mol)(213.6 J/(K ⋅ mol))]
- (1 mol)(92.9 J/(K ⋅ mol)) = +160.4 J/K

17.6 From Problem 17.5, ∆Ssys = ∆So = 160.4 J/K


CaCO3(s) → CaO(s) + CO2(g)
∆Ho = [∆Hof(CaO) + ∆Hof(CO2)] - ∆Hof(CaCO3)
∆Ho = [(1 mol)(-635.1 kJ/mol) + (1 mol)(-393.5 kJ/mol)]
- (1 mol)(-1206.9 kJ/mol) = +178.3 kJ
_ ∆ H o _ 178,300 J
∆Ssurr = = = -598 J/K
T 298 K
∆Stotal = ∆Ssys + ∆Ssurr = 160.4 J/K + (-598 J/K) = -438 J/K
Because ∆Stotal is negative, the reaction is not spontaneous under standard-state
conditions at 25oC.

17.7 (a) ∆G = ∆H - T∆S = 57.1 kJ - (298 K)(0.1758 kJ/K) = +4.7 kJ


Because ∆G > 0, the reaction is nonspontaneous at 25oC (298 K)

489
Chapter 17 - Thermodynamics: Entropy, Free Energy, and Equilibrium
______________________________________________________________________________

(b) Set ∆G = 0 and solve for T.


∆H 57.1 kJ
0 = ∆H - T∆S; T= = = 325 K = 52oC
∆ S 0.1758 kJ/K

17.8 (a) ∆G = ∆H - T∆S = 58.5 kJ/mol - (598 K)[0.0929 kJ/(K ⋅ mol)] = +2.9 kJ/mol
Because ∆G > 0, Hg does not boil at 325oC and 1 atm.
(b) The boiling point (phase change) is associated with an equilibrium. Set ∆G = 0 and
solve for T, the boiling point.
∆ H vap 58.5 kJ/mol
0 = ∆Hvap - T∆Svap; Tbp = = = 630 K = 357oC
∆ Svap 0.0929 kJ/(K • mol)

17.9 ∆H < 0 (reaction involves bond making - exothermic)


∆S < 0 (the reaction becomes more ordered in going from reactants (2 atoms) to
products (1 molecule)
∆G < 0 (the reaction is spontaneous)

17.10 From Problems 17.5 and 17.6: ∆Ho = 178.3 kJ and ∆So = 160.4 J/K = 0.1604 kJ/K
(a) ∆Go = ∆Ho - T∆So = 178.3 kJ - (298 K)(0.1604 kJ/K) = +130.5 kJ
(b) Because ∆G > 0, the reaction is nonspontaneous at 25oC (298 K).
(c) Set ∆G = 0 and solve for T, the temperature above which the reaction becomes
spontaneous.
∆H 178.3 kJ
0 = ∆H - T∆S; T= = = 1112 K = 839oC
∆ S 0.1604 kJ/K

17.11 2 AB2 → A2 + 2 B2
(a) ∆So is positive because the reaction increases the number of molecules.
(b) ∆Ho is positive because the reaction is endothermic.
∆Go = ∆Ho - T∆So
For the reaction to be spontaneous, ∆Go must be negative. This will only occur at high
temperature where T∆So is greater than ∆Ho.

17.12 (a) CaC2(s) + 2 H2O(l) → C2H2(g) + Ca(OH)2(s)


∆Go = [∆Gof(C2H2) +∆Gof(Ca(OH)2)] - [∆Gof(CaC2) + 2 ∆Gof(H2O)]
∆Go = [(1 mol)(209.2 kJ/mol) + (1 mol)(-898.6 kJ/mol)]
- [(1 mol)(-64.8 kJ/mol) + (2 mol)(-237.2 kJ/mol)] = -150.2 kJ
This reaction can be used for the synthesis of C2H2 because ∆G < 0.
(b) It is not possible to synthesize acetylene from solid graphite and gaseous H2 at 25oC
and 1 atm because ∆Gof(C2H2) > 0.

17.13 C(s) + 2 H2(g) → C2H4(g)


P (0.10)
Qp = C2 H42 = 2
= 1.0 x 10-5
(P H2 ) (100 )
∆G = ∆Go + RT ln Qp

490
Chapter 17 - Thermodynamics: Entropy, Free Energy, and Equilibrium
______________________________________________________________________________

∆G = 68.1 kJ/mol + [8.314 x 10-3 kJ/(K ⋅ mol)](298 K)ln(1.0 x 10-5) = +39.6 kJ/mol
Because ∆G > 0, the reaction is spontaneous in the reverse direction.

17.14 ∆G = ∆Go + RT lnQ and ∆Go = 15 kJ


(P AB )2
For A2(g) + B2(g) = 2 AB(g), Qp =
(P A2)(P B2)
Let the number of molecules be proportional to the partial pressure.
(1) Qp = 1.0 (2) Qp = 0.0667 (3) Qp = 18
(a) Reaction (3) has the largest ∆G because Qp is the largest. Reaction (2) has the
smallest ∆G because Qp is the smallest.
(b) ∆G = ∆Go = 15 kJ because Qp = 1 and ln (1) = 0.

17.15 From Problem 17.10, ∆Go = +130.5 kJ


∆Go = -RT ln Kp
_ ∆G o _ 130.5 kJ/mol
ln Kp = = = -52.7
RT [8.314 x 10_ 3 kJ/(K • mol)](298 K)
Kp = e-52.7 = 1 x 10-23

17.16 H2O(l) _ H2O(g)


Kp = P H2 O ; Kp is equal to the vapor pressure for H2O.
∆Go =∆Gof(H2O(g)) - ∆Gof(H2O(l))
∆Go = (1 mol)(-228.6 kJ/mol) - (1 mol)(-237.2 kJ/mol) = +8.6 kJ
∆Go = -RT ln Kp
_ ∆G o _ 8.6 kJ/mol
ln Kp = = = -3.5
RT [8.314 x 10_ 3 kJ/(K • mol)](298 K)
Kp = P H2 O = e-3.5 = 0.03 atm

17.17 ∆Go = -RT ln K = -[8.314 x 10-3 kJ/(K ⋅ mol)](298 K) ln (1.0 x 10-14) = 80 kJ/mol

17.18 Photosynthetic cells in plants use the sun’s energy to make glucose, which is then used
by animals as their primary source of energy. The energy an animal obtains from glucose
is then used to build and organize complex molecules, resulting in a decrease in entropy
for the animal. At the same time, however, the entropy of the surroundings increases as
the animal releases small, simple waste products such as CO2 and H2O. Furthermore,
heat is released by the animal, further increasing the entropy of the surroundings. Thus,
an organism pays for its decrease in entropy by increasing the entropy of the rest of the
universe.

17.19 You would expect to see violations of the second law if you watched a movie run
backwards. Consider an action-adventure movie with a lot of explosions. An explosion
is a spontaneous process that increases the entropy of the universe. You would see an
explosion go backwards if you run the the movie backwards but this is impossible
because it would decrease the entropy of the universe.

491
Chapter 17 - Thermodynamics: Entropy, Free Energy, and Equilibrium
______________________________________________________________________________

Understanding Key Concepts

17.20 (a)

(b) ∆H = 0 (no heat is gained or lost in the mixing of ideal gases)


∆S > 0 (the mixture of the two gases is more disordered)
∆G < 0 (the mixing of the two gases is spontaneous)
(c) For an isolated system, ∆Ssurr = 0 and ∆Ssys = ∆STotal > 0 for the spontaneous process.
(d) ∆G > 0 and the process is nonspontaneous.

17.21 ∆H > 0 (heat is absorbed during sublimation)


∆S > 0 (gas is more disordered than solid)
∆G < 0 (the reaction is spontaneous)

17.22 ∆H < 0 (heat is lost during condensation)


∆S < 0 (liquid is more ordered than vapor)
∆G < 0 (the reaction is spontaneous)

17.23 ∆H = 0 (system is an ideal gas at constant temperature)


∆S < 0 (there is more order in the smaller volume)
∆G > 0 (compression of a gas is not spontaneous)

17.24 (a) 2 A2 + B2 → 2 A2B


(b) ∆H < 0 (because ∆S is negative, ∆H must also be negative in order for ∆G to be
negative)
∆S < 0 (the reaction becomes more ordered in going from reactants (3 molecules) to
products (2 molecules))
∆G < 0 (the reaction is spontaneous)

17.25 (a) For initial state 1, Qp < Kp


(more reactant (A2) than product (A) compared to the equilibrium state)
For initial state 2, Qp > Kp
(more product (A) than reactant (A2) compared to the equilibrium state)

(b) ∆H > 0 (reaction involves bond breaking - endothermic)


∆S > 0 (equilibrium state is more disordered than initial state 1)
∆G < 0 (reaction spontaneously proceeds toward equilibrium)

(c) ∆H < 0 (reaction involves bond making - exothermic)


∆S < 0 (equilibrium state is more ordered than initial state 2)

492
Chapter 17 - Thermodynamics: Entropy, Free Energy, and Equilibrium
______________________________________________________________________________

∆G < 0 (reaction spontaneously proceeds toward equilibrium)


(d) State 1 lies to the left of the minimum in Figure 17.10. State 2 lies to the right of the
minimum.

17.26 (a) ∆Ho > 0 (reaction involves bond breaking - endothermic)


∆So > 0 (2 A's are more disordered than A2)
(b) ∆So is for the complete conversion of 1 mole of A2 in its standard state to 2 moles of
A in its standard state.
(c) There is not enough information to say anything about the sign of ∆Go. ∆Go
decreases (becomes less positive or more negative) as the temperature increases.
(d) Kp increases as the temperature increases. As the temperature increases there will be
more A and less A2.
(e) ∆G = 0 at equilibrium.

17.27 (a) Because the free energy decreases as pure reactants form products and also decreases
as pure products form reactants, the free energy curve must go through a minimum
somewhere between pure reactants and pure products. At the minimum point, ∆G = 0
and the system is at equilibrium.
(b) The minimum in the plot is on the left side of the graph because ∆Go > 0 and the
equilibrium composition is rich in reactants.

[X] [Y] [Z]


17.28 ∆Go = -RT ln K where K = or or
[A] [A] [A]
Let the number of molecules be proportional to the concentration.
(1) K = 1, ln K = 0, and ∆Go = 0.
(2) K > 1, ln K is positive, and ∆Go is negative.
(3) K < 1, ln K is negative, and ∆Go is positive.

17.29 The equilibrium mixture is richer in reactant A at the higher temperature. This means the
reaction is exothermic (∆H < 0). At 25oC, ∆Go < 0 because K > 1 and at 45oC, ∆Go > 0
because K < 1. Using the relationship. ∆Go = ∆Ho - T∆So, with ∆Ho < 0, ∆Go will
become positive at the higher temperature only if ∆So is negative.

Additional Problems
Spontaneous Processes

17.30 A spontaneous process is one that proceeds on its own without any external influence.
For example: H2O(s) → H2O(l) at 25oC
A nonspontaneous process takes place only in the presence of some continuous external
influence.
For example: 2 NaCl(s) → 2 Na(s) + Cl2(g)

17.31 Spontaneous does not mean instantaneous. Even though the decomposition can occur (is
spontaneous), the rate of decomposition is determined by the kinetics of the reaction.

493
Chapter 17 - Thermodynamics: Entropy, Free Energy, and Equilibrium
______________________________________________________________________________

17.32 (a) and (d) nonspontaneous; (b) and (c) spontaneous

17.33 (a) and (c) spontaneous; (b) and (d) nonspontaneous.

17.34 (b) and (d) spontaneous (because of the large positive Kp's)

17.35 (a) and (d) nonspontaneous (because of the small K's).

Entropy

17.36 Molecular randomness or disorder is called entropy. For the following reaction, the
entropy (disorder) increases: H2O(s) → H2O(l) at 25oC.

17.37 Exothermic reactions can become nonspontaneous at high temperatures if ∆S is negative.


Endothermic reactions can become spontaneous at high temperatures if ∆S is positive.

17.38 (a) + (solid → gas) (b) - (liquid → solid)


(c) - (aqueous ions → solid) (d) + (CO2(aq) → CO2(g))

17.39 (a) + (increase in moles of gas)


(b) - (decrease in moles of gas and formation of liquid)
(c) + (aqueous ions to gas)
(d) - (decrease in moles of gas)

17.40 (a) - (liquid → solid)


(b) - (decrease in number of O2 molecules)
(c) + (gas is more disordered in larger volume)
(d) - (aqueous ions → solid)

17.41 (a) + (solid dissolved in water) (b) + (increase in moles of gas)


(c) + (mixed gases are more disordered) (d) + (liquid to gas)

17.42 S = k ln W, k = 1.38 x 10-23 J/K


(a) S = (1.38 x 10-23 J/K) ln (412) = 2.30 x 10-22 J/K
(b) S = (1.38 x 10-23 J/K) ln (4120) = 2.30 x 10-21 J/K
(c) S = (1.38 x 10-23 J/K) ln ( 46.02 x 10 ) = 11.5 J/K
23

If all C–D bonds point in the same direction, S = 0.

17.43 S = k ln W, k = 1.38 x 10-23 J/K


(a) W = 1; S = k ln (1) = 0
(b) W = 32, = 9; S = k ln (32) = 3.03 x 10-23 J/K
(c) W = 1; S = k ln (1) = 0
(d) W = 33 = 27; S = k ln (33) = 4.55 x 10-23 J/K
(e) W = 1; S = k ln (1) = 0
23 23
(f) W = 36.02 x 10 ; S = k ln (36.02 x 10 ) = 9.13 J/K

494
Chapter 17 - Thermodynamics: Entropy, Free Energy, and Equilibrium
______________________________________________________________________________

 
∆S = R ln  Vf  = (8.314 J/K)ln 3 = 9.13 J/K The results are the same.
 Vi 

17.44 (a) H2 at 25oC in 50 L (larger volume)


(b) O2 at 25oC, 1 atm (larger volume)
(c) H2 at 100oC, 1 atm (larger volume and higher T)
(d) CO2 at 100oC, 0.1 atm (larger volume and higher T)

17.45 (a) ice at 0oC, because of the higher temperature.


(b) N2 at STP, because it has the larger volume.
(c) N2 at 0oC and 50 L, because it has the larger volume.
(d) water vapor at 150oC and 1 atm, because it has a larger volume and higher temperature.

Standard Molar Entropies and Standard Entropies of Reaction

17.46 The standard molar entropy of a substance is the entropy of 1 mol of the pure substance at
1 atm pressure and 25oC.
∆So = So(products) - So(reactants)

J
17.47 (a) Units of So = (b) Units of ∆So = J/K
K • mol
Standard molar entropies are called absolute entropies because they are measured with
respect to an absolute reference point, the entropy of the substance at 0 K.
J
So = 0 at T = 0 K.
K • mol

17.48 (a) C2H6(g); more atoms/molecule


(b) CO2(g); more atoms/molecule
(c) I2(g); gas is more disordered than the solid
(d) CH3OH(g); gas is more disordered than the liquid.

17.49 (a) NO2(g); more atoms/molecule


(b) CH3CO2H(l); more atoms/molecule
(c) Br2(l); liquid is more disordered than the solid
(d) SO3(g); gas is more disordered than the solid

17.50 (a) 2 H2O2(l) → 2 H2O(l) + O2(g)


∆So = [2 So(H2O(l)) + So(O2)] - 2 So(H2O2)
∆So = [(2 mol)(69.9 J/(K ⋅ mol)) + (1 mol)(205.0 J/(K ⋅ mol))]
- (2 mol)(110 J/(K ⋅ mol)) = +125 J/K (+, because moles of gas increase)

(b) 2 Na(s) + Cl2(g) → 2 NaCl(s)


∆So = 2 So(NaCl) - [2 So(Na) + So(Cl2)]
∆So = (2 mol)(72.1 J/(K ⋅ mol)) - [(2 mol)(51.2 J/(K ⋅ mol)) + (1 mol)(223.0 J/(K ⋅ mol))]
∆So = -181.2 J/K (-, because moles of gas decrease)

495
Chapter 17 - Thermodynamics: Entropy, Free Energy, and Equilibrium
______________________________________________________________________________

(c) 2 O3(g) → 3 O2(g)


∆So = 3 So(O2) - 2 So(O3)
∆So = (3 mol)(205.0 J/(K ⋅ mol)) - (2 mol)(238.8 J/(K ⋅ mol))
∆So = +137.4 J/K (+, because moles of gas increase)
(d) 4 Al(s) + 3 O2(g) → 2 Al2O3(s)
∆So = 2 So(Al2O3) - [4 So(Al) + 3 So(O2)]
∆So = (2 mol)(50.9 J/(K ⋅ mol)) - [(4 mol)(28.3 J/(K ⋅ mol)) + (3 mol)(205.0 J/(K ⋅ mol))]
∆So = -626.4 J/K (-, because moles of gas decrease)

17.51 (a) 2 S(s) + 3 O2(g) → 2 SO3(g)


∆So = 2 So(SO3) - [2 So(S) + 3 So(O2)]
∆So = (2 mol)(256.6 J/(K ⋅ mol)) - [(2 mol)(31.8 J/(K ⋅ mol)) + (3 mol)(205.0 J/(K ⋅ mol))]
∆So = -165.4 J/K (-, because moles of gas decrease)
(b) SO3(g) + H2O(l) → H2SO4(aq)
∆So = So(H2SO4) - [So(SO3) + So(H2O)]
∆So = (1 mol)(20 J/(K ⋅ mol)) - [(1 mol)(256.6 J/(K ⋅ mol)) + (1 mol)(69.9 J/(K ⋅ mol))]
∆So = -306 J/K (-, because of the conversion of a gas and water to an aqueous solution)
(c) AgCl(s) → Ag+(aq) + Cl-(aq)
∆So = [So(Ag+) + So(Cl-)] - So(AgCl)]
∆So = [(1 mol)(72.7 J/(K ⋅ mol)) + (1 mol)(56.5 J/(K ⋅ mol))] - (1 mol)(96.2 J/(K ⋅ mol))]
∆So = +33.0 J/K (+, because a solid is converted to ions in aqueous solution)
(d) NH4NO3(s) → N2O(g) + 2 H2O(g)
∆So = [So(N2O) + 2 So(H2O)] - So(NH4NO3)
∆So = [(1 mol)(219.7 J/(K ⋅ mol)) + (2 mol)(188.7 J/(K ⋅ mol))] - (1 mol)(151.1 J/(K ⋅ mol))
∆So = +446.0 J/K (+, because moles of gas increase)

Entropy and the Second Law of Thermodynamics

17.52 In any spontaneous process, the total entropy of a system and its surroundings always
increases.

17.53 For a spontaneous process, ∆Stotal = ∆Ssys + ∆Ssurr > 0. For an isolated system, ∆Ssurr = 0,
and so ∆Ssys > 0 is the criterion for spontaneous change. An example of a spontaneous
process in an isolated system is the mixing of two gases.

_ ∆H
17.54 ∆Ssurr = ; the temperature (T) is always positive.
T
(a) For an exothermic reaction, ∆H is negative and ∆Ssurr is positive.
(b) For an endothermic reaction, ∆H is positive and ∆Ssurr is negative.

17.55 ∆Ssurr ∝
1
T
Consider the surroundings as an infinitely large constant-temperature bath to which heat
can be added without changing its temperature. If the surroundings have a low
temperature, they have only a small amount of disorder, in which case addition of a given

496
Chapter 17 - Thermodynamics: Entropy, Free Energy, and Equilibrium
______________________________________________________________________________

quantity of heat results in a substantial increase in the amount of disorder (a relatively


large value of ∆Ssurr). If the surroundings have a high temperature, they already have a
large amount of disorder, and addition of the same quantity of heat produces only a
marginal increase in the amount of disorder (a relatively small value of ∆Ssurr). Thus, we
expect ∆Ssurr to vary inversely with temperature.

17.56 N2(g) + 2 O2(g) → N2O4(g)


∆Ho = ∆Hof(N2O4) = 9.16 kJ
∆Ssys = ∆So = So(N2O4) - [So(N2) + 2 So(O2)]
∆Ssys = (1 mol)(304.2 J/(K⋅ mol))
- [(1 mol)(191.5 J/(K ⋅ mol)) + (2 mol)(205.0 J/(K ⋅ mol))] = -297.3 J/K
_ ∆ H o _ 9.16 kJ
∆Ssurr = = = -0.0307 kJ/K = -30.7 J/K
T 298 K
∆Stotal = ∆Ssys + ∆Ssurr = -297.3 J/K + (-30.7 J/K) = -328.0 J/K
Because ∆Stotal < 0, the reaction is nonspontaneous.

17.57 Cu2S(s) + O2(g) → 2 Cu(s) + SO2(g)


∆Ho = ∆Hof(SO2) - ∆Hof(Cu2S)
∆Ho = (1 mol)(-296.8 kJ/mol) - (1 mol)(-79.5 kJ/mol) = -217.3 kJ
∆Ssys = ∆So = [2 So(Cu) + So(SO2)] - [So(Cu2S) + So(O2)]
∆Ssys = [(2 mol)(33.1 J/(K ⋅ mol)) + (1 mol)(248.1 J/(K ⋅ mol))]
- [(1 mol)(120.9 J/(K ⋅ mol)) + (1 mol)(205.0 J/(K ⋅ mol))] = -11.6 J/K
_ (_ 217,300 J)
∆Ssurr = _ ∆ H ooverT = = +728.8 J/K
298.15 K
∆Stotal = ∆Ssys + ∆Ssurr = -11.6 J/K + 728.8 J/K = +717.2 J/K
Because ∆Stotal is positive, the reaction is spontaneous under standard-state conditions at
25oC.

_ ∆ H vap _ 30,700 J/mol


17.58 (a) ∆Ssurr = = = -89.5 J/(K ⋅ mol)
T 343 K
∆Stotal = ∆Svap + ∆Ssurr = 87.0 J/(K ⋅ mol) + (-89.5 J/(K ⋅ mol)) = - 2.5 J/(K ⋅ mol)

_ ∆ H vap _ 30,700 J/mol


(b) ∆Ssurr = = = -87.0 J/(K ⋅ mol)
T 353 K

∆Stotal = ∆Svap + ∆Ssurr = 87.0 J/(K ⋅ mol) + (- 87.0 J/(K ⋅ mol)) = 0

_ ∆ H vap _ 30,700 J/mol


(c) ∆Ssurr = = = -84.6 J/(K ⋅ mol)
T 363 K

∆Stotal = ∆Svap + ∆Ssurr = 87.0 J/(K ⋅ mol) + (- 84.6 J/(K ⋅ mol)) = +2.4 J/(K ⋅ mol)
Benzene does not boil at 70oC (343 K) because ∆Stotal is negative.

497
Chapter 17 - Thermodynamics: Entropy, Free Energy, and Equilibrium
______________________________________________________________________________

The normal boiling point for benzene is 80oC (353 K), where ∆Stotal = 0.

_ ∆ Hfusion _ 30,200 J/mol


17.59 (a) ∆Ssurr = = = -28.8 J/(K ⋅ mol)
T 1050 K
∆Stotal = ∆Ssys + ∆Ssurr = 28.1 J/(K ⋅ mol) + (-28.8 J/(K ⋅ mol)) = -0.7 J/(K ⋅ mol)

_ ∆ Hfusion _ 30,200 J/mol


(b) ∆Ssurr = = = -28.1 J/(K ⋅ mol)
T 1075 K

∆Stotal = ∆Ssys + ∆Ssurr = 28.1 J/(K ⋅ mol) + (-28.1 J/(K ⋅ mol)) = 0

_ ∆ Hfusion _ 30,200 J/mol


(c) ∆Ssurr = = = -27.5 J/(K ⋅ mol)
T 1100 K

∆Stotal = ∆Ssys + ∆Ssurr = 28.1 J/(K ⋅ mol) + (-27.5 J/(K ⋅ mol)) = +0.6 J/(K ⋅ mol)
NaCl melts at 1100 K because ∆Stotal > 0.
The melting point of NaCl is 1075 K, where ∆Stotal = 0.

Free Energy

17.60 ∆H ∆S ∆G = ∆H - T∆S Reaction Spontaneity


- + - Spontaneous at all temperatures
- - - or + Spontaneous at low temperatures
where  ∆H >  T∆S
Nonspontaneous at high temperatures
where  ∆H <  T∆S
+ - + Nonspontaneous at all temperatures
+ + - or + Spontaneous at high temperatures
where T∆S > ∆H
Nonspontaneous at low temperature
where T∆S < ∆H

17.61 When ∆H and ∆S are both positive or both negative, the temperature determines the
direction of spontaneous reaction. See Problem 17.60 for an explanation.

17.62 (a) 0oC (temperature is below mp); ∆H > 0, ∆S > 0, ∆G > 0


(b) 15oC (temperature is above mp); ∆H > 0, ∆S > 0, ∆G < 0

17.63 (a) ∆H = 0
∆S = R ln Vfinal = (8.314 J/K) ln 2 = 5.76 J/K
Vinitial
∆G = ∆H - T∆S
Because ∆H = 0, ∆G = -T∆S = -(298 K)(5.76 J/K) = -1717 J = -1.72 kJ

498
Chapter 17 - Thermodynamics: Entropy, Free Energy, and Equilibrium
______________________________________________________________________________

(b) For a process in an isolated system, ∆Ssurr = 0. Therefore, ∆Stotal = ∆Ssys > 0, and the
process is spontaneous.

17.64 ∆Hvap = 30.7 kJ/mol


∆Svap = 87.0 J/(K ⋅ mol) = 87.0 x 10-3 kJ/(K ⋅ mol)
∆Gvap = ∆Hvap - T∆Svap
(a) ∆Gvap = 30.7 kJ/mol - (343 K)(87.0 x 10-3 kJ/(K ⋅ mol)) = +0.9 kJ/mol
At 70oC (343 K), benzene does not boil because ∆Gvap is positive.
(b) ∆Gvap = 30.7 kJ/mol - (353 K)(87.0 x 10-3 kJ/(K ⋅ mol)) = 0
80oC (353 K) is the boiling point for benzene because ∆Gvap = 0
(c) ∆Gvap = 30.7 kJ/mol - (363 K)(87.0 x 10-3 kJ/(K ⋅ mol)) = -0.9 kJ/mol
At 90oC (363 K), benzene boils because ∆Gvap is negative.

17.65 ∆Hfusion = 30.2 kJ/mol; ∆Sfusion = 28.1 x 10-3 kJ/(K ⋅ mol)


∆Gfusion = ∆Hfusion - T∆Sfusion
(a) ∆Gfusion = 30.2 kJ/mol - (1050 K)(28.1 x 10-3 kJ/(K ⋅ mol)) = +0.7 kJ/mol
At 1050 K, NaCl does not melt because ∆Gfusion is positive.
(b) ∆Gfusion = 30.2 kJ/mol - (1075 K)(28.1 x 10-3 kJ/(K ⋅ mol)) = 0
1075 K is the melting point for NaCl because ∆Gfusion = 0.
(c) ∆Gfusion = 30.2 kJ/mol - (1100 K)(28.1 x 10-3 kJ/(K ⋅ mol)) = -0.7 kJ/mol
At 1100 K, NaCl does melt because ∆Gfusion is negative.

17.66 At the melting point (phase change), ∆Gfusion = 0


∆Gfusion = ∆Hfusion - T∆Sfusion
∆ 17.3 kJ/mol
0 = ∆Hfusion - T∆Sfusion; T = Hfusion = = 395 K = 122oC
∆ Sfusion 43.8 x 10 kJ/(K • mol)
_3

17.67 128oC = 401 K


At the melting point (phase change), ∆Gfusion = 0
∆Gfusion = ∆Hfusion - T∆Sfusion
0 = ∆Hfusion - T∆Sfusion
∆Hfusion = T∆Sfusion = (401 K)[47.7 x 10-3 kJ/(K ⋅ mol)] = 19.1 kJ/mol

Standard Free-Energy Changes and Standard Free Energies of Formation

17.68 (a) ∆Go is the change in free energy that occurs when reactants in their standard states are
converted to products in their standard states.
(b) ∆Gof is the free-energy change for formation of one mole of a substance in its
standard state from the most stable form of the constituent elements in their standard
states.

17.69 The standard state of a substance (solid, liquid, or gas) is the most stable form of a pure
substance at 25oC and 1 atm pressure. For solutes, the condition is 1 M at 25oC.

499
Chapter 17 - Thermodynamics: Entropy, Free Energy, and Equilibrium
______________________________________________________________________________

17.70 (a) N2(g) + 2 O2(g) → 2 NO2(g)


∆Ho = 2 ∆Hof(NO2) = (2 mol)(33.2 kJ/mol) = 66.4 kJ
∆So = 2 So(NO2) - [So(N2) + 2 So(O2)]
∆So = (2 mol)(240.0 J/(K ⋅ mol)) - [(1 mol)(191.5 J/(K ⋅ mol)) + (2 mol)(205.0 J/(K ⋅ mol))]
∆So = -121.5 J/K = -121.5 x 10-3 kJ/K
∆Go = ∆Ho - T∆So = 66.4 kJ - (298 K)(-121.5 x 10-3 kJ/K) = +102.6 kJ
Because ∆Go is positive, the reaction is nonspontaneous under standard-state conditions at 25oC.
(b) 2 KClO3(s) → 2 KCl(s) + 3 O2(g)
∆Ho = 2 ∆Hof(KCl) - 2 ∆Hof(KClO3)
∆Ho = (2 mol)(-436.7 kJ/mol) - (2 mol)(-397.7 kJ/mol) = -78.0 kJ
∆So = [2 So(KCl) + 3 So(O2)] - 2 So(KClO3)
∆So = [(2 mol)(82.6 J/(K ⋅ mol)) + (3 mol)(205.0 J/(K ⋅ mol))] - (2 mol)(143 J/(K ⋅ mol))
∆So = 494.2 J/(K ⋅ mol) = 494.2 x 10-3 kJ/(K ⋅ mol)
∆Go = ∆Ho - T∆So = -78.0 kJ - (298 K)(494.2 x 10-3 kJ/(K ⋅ mol)) = -225.3 kJ
Because ∆Go is negative, the reaction is spontaneous under standard-state conditions at 25oC.
(c) CH3CH2OH(l) + O2(g) → CH3CO2H(l) + H2O(l)
∆Ho = [∆Hof(CH3CO2H) + ∆Hof(H2O)] - ∆Hof(CH3CH2OH)
∆Ho = [(1 mol)(-484.5 kJ/mol) + (1 mol)(-285.8 kJ/mol)] - (1 mol)(-277.7 kJ/mol) = - 492.6 kJ
∆So = [So(CH3CO2H) + So(H2O)] - [So(CH3CH2OH) + So(O2)]
∆So = [(1 mol)(160 J/(K ⋅ mol)) + (1 mol)(69.9 J/(K ⋅ mol))]
- [(1 mol)(161 J/(K ⋅ mol)) + (1 mol)(205.0 J/(K ⋅ mol))]
∆So = -136.1 J/(K ⋅ mol) = -136.1 x 10-3 kJ/(K ⋅ mol)
∆Go = ∆Ho - T∆So = -492.6 kJ - (298 K)(-136.1 x 10-3 kJ/(K ⋅ mol)) = - 452.0 kJ
Because ∆Go is negative, the reaction is spontaneous under standard-state conditions at 25oC.

17.71 (a) 2 SO2(g) + O2(g) → 2 SO3(g)


∆Ho = 2 ∆Hof(SO3) - 2 ∆Hof SO2)
∆Ho = (2 mol)(-395.7 kJ/mol) - (2 mol)(-296.8 kJ/mol) = -197.8 kJ
∆So = 2 So(SO3) - [2 So(SO2) + So(O2)]
∆So = (2 mol)(256.6 J/(K ⋅ mol)) - [(2 mol)(248.1 J/(K ⋅ mol)) + (1 mol)(205.0 J/(K ⋅ mol))]
∆So = -188.0 J/K = -188.0 x 10-3 kJ/K
∆Go = ∆Ho - T∆So = -197.8 kJ - (298 K)(-188.0 x 10-3 kJ/K) = -141.8 kJ
Because ∆Go is negative, the reaction is spontaneous under standard-state conditions at 25oC.
(b) N2(g) + 2 H2(g) → N2H4(l)
∆Ho = ∆Hof(N2H4)
∆Ho = (1 mol)(50.6 kJ/mol) = 50.6 kJ
∆So = So(N2H4) - [So(N2) + 2 So(H2)]
∆So = (1 mol)(121.2 J/(K ⋅ mol)) - [(1 mol)(191.5 J/(K ⋅ mol)) + (2 mol)(130.6 J/(K ⋅ mol))]
∆So = -331.5 J/K = -331.5 x 10-3 kJ/K
∆Go = ∆Ho - T∆So = 50.6 kJ - (298 K)(-331.5 x 10-3 kJ/K) = +149.4 kJ
Because ∆Go is positive, the reaction is nonspontaneous under standard-state conditions at 25oC.
(c) CH3OH(l) + O2(g) → HCO2H(l) + H2O(l)
∆Ho = [∆Hof(HCO2H) + ∆Hof(H2O)] - ∆Hof(CH3OH)
∆Ho = [(1 mol)(-424.7 kJ/mol) + (1 mol)(-285.8 kJ/mol)] - (1 mol)(-238.7 kJ/mol) = - 471.8 kJ

500
Chapter 17 - Thermodynamics: Entropy, Free Energy, and Equilibrium
______________________________________________________________________________

∆So = [So(HCO2H) + So(H2O)] - [So(CH3OH) + So(O2)]

∆So = [(1 mol)(129.0 J/(K ⋅ mol)) + (1 mol)(69.9 J/(K ⋅ mol))]


- [(1 mol)(127 J/(K ⋅ mol)) + (1 mol)(205.0 J/(K ⋅ mol))]
∆S = -133.1 J/K = -133.1 x 10-3 kJ/K
o

∆Go = ∆Ho - T∆So = -471.8 kJ - (298 K)(-133.1 x 10-3 kJ/K) = - 432.1 kJ


Because ∆Go is negative, the reaction is spontaneous under standard-state conditions at 25oC.

17.72 (a) N2(g) + 2 O2(g) → 2 NO2(g)


∆Go = 2 ∆Gof(NO2) = (2 mol)(51.3 kJ/mol) = +102.6 kJ
(b) 2 KClO3(s) → 2 KCl(s) + 3 O2(g)
∆Go = 2 ∆Gof(KCl) - 2 ∆Gof(KClO3)
∆Go = (2 mol)(- 409.2 kJ/mol) - (2 mol)(-296.3 kJ/mol) = -225.8 kJ
(c) CH3CH2OH(l) + O2(g) → CH3CO2H(l) + H2O(l)
∆Go = [∆Gof(CH3CO2H) +∆Gof(H2O)] -∆Gof(CH3CH2OH)
∆Go = [(1 mol)(-390 kJ/mol) + (1 mol)(-237.2 kJ/mol)] - (1 mol)(-174.9 kJ/mol) = - 452 kJ

17.73 (a) 2 SO2(g) + O2(g) → 2 SO3(g)


∆Go = 2 ∆Gof(SO3) - 2 ∆Gof(SO2)
∆Go = (2 mol)(-371.1 kJ/mol) - (2 mol)(-300.2 kJ/mol) = -141.8 kJ
(b) N2(g) + 2 H2(g) → N2H4(l)
∆Go = ∆Gof(N2H4) = (1 mol)(149.2 kJ/mol) = 149.2 kJ
(c) CH3OH(l) + O2(g) → HCO2H(l) + H2O(l)
∆Go = [∆Gof(HCO2H) +∆Gof(H2O)] - ∆Gof(CH3OH)
∆Go = [(1 mol)(-361.4 kJ/mol) + (1 mol)(-237.2 kJ/mol)] - (1 mol)(-166.4 kJ/mol)
∆Go = - 432.2 kJ

17.74 A compound is thermodynamically stable with respect to its constituent elements at 25oC
if ∆Gof is negative.
∆Gof (kJ/mol) Stable
(a) BaCO3(s) -1138 yes
(b) HBr(g) -53.4 yes
(c) N2O(g) +104.2 no
(d) C2H4(g) +68.1 no

17.75 A compound is thermodynamically stable with respect to its constituent elements at 25oC
if ∆Gof is negative.
∆Gof (kJ/mol) Stable
(a) C6H6(l) +124.5 no
(b) NO(g) +86.6 no
(c) PH3(g) +13 no
(d) FeO(s) -255 yes

17.76 CH2=CH2(g) + H2O(l) → CH3CH2OH(l)


∆Ho =∆Hof(CH2CH2OH) - [∆Hof(CH2=CH2) + ∆Hof(H2O)]

501
Chapter 17 - Thermodynamics: Entropy, Free Energy, and Equilibrium
______________________________________________________________________________

∆Ho = (1 mol)(-277.7 kJ/mol) - [(1 mol)(52.3 kJ/mol) + (1 mol)(-285.8 kJ/mol)]


∆Ho = - 44.2 kJ
∆So = So(CH3CH2OH) - [So(CH2=CH2) + So(H2O)]
∆So = (1 mol)(161 J/(K ⋅ mol)) - [(1 mol)(219.5 J/(K ⋅ mol)) + (1 mol)(69.9 J/(K ⋅ mol))]
∆So = -128 J/(K ⋅ mol) = -128 x 10-3 kJ/(K ⋅ mol)
∆Go = ∆Ho - T∆So = - 44.2 kJ - (298 K)(-128 x 10-3 kJ/K) = -6.1 kJ
Because ∆Go is negative, the reaction is spontaneous under standard-state conditions at 25oC.
The reaction becomes nonspontaneous at high temperatures because ∆So is negative.
To find the crossover temperature, set ∆G = 0 and solve for T.
∆ H o _ 44,200 J
T= = = 345 K = 72oC
∆S o
_ 128 J/K
The reaction becomes nonspontaneous at 72oC.

17.77 2 H2S(g) + SO2(g) → 3 S(s) + 2 H2O(g)


∆Ho = 2 ∆Hof(H2O) - [2 ∆Hof(H2S) + ∆Hof(SO2)]
∆Ho = (2 mol)(-241.8 kJ/mol) - [(2 mol)(-20.6 kJ/mol) + (1 mol)(-296.8 kJ/mol) = -145.6 kJ
∆So = [3 So(S) + 2 So(H2O)] - [2 So(H2S) + So(SO2)]
∆So = [(3 mol)(31.8 J/(K ⋅ mol)) + (2 mol)(188.7 J/(K ⋅ mol))]
- [(2 mol)(205.7 J/(K ⋅ mol)) + (1 mol)(248.1 J/(K ⋅ mol))]
∆So = -186.7 J/K = -186.7 x 10-3 kJ/K
∆Go = ∆Ho - T∆So = -145.6 kJ - (298 K)(-186.7 x 10-3 kJ/K) = -90.0 kJ
Because ∆Go is negative, the reaction is spontaneous under standard-state conditions at 25oC.
The reaction becomes nonspontaneous at high temperatures because ∆So is negative.
To find the crossover temperature set ∆G = 0 and solve for T.
∆ H o _ 145,600 J
T= = = 780 K = 507oC
∆S o
_ 186.7 J/K
The reaction becomes nonspontaneous at 507oC.

17.78 3 C2H2(g) → C6H6(l)


∆Go =∆Gof(C6H6) - 3 ∆Gof(C2H2)
∆Go = (1 mol)(124.5 kJ/mol) - (3 mol)(209.2 kJ/mol) = -503.1 kJ
Because ∆Go is negative, the reaction is possible. Look for a catalyst.
Because ∆Gof for benzene is positive (+124.5 kJ/mol), the synthesis of benzene from
graphite and gaseous H2 at 25oC and 1 atm pressure is not possible.

17.79 CH2ClCH2Cl(l) → CH2=CHCl(g) + HCl(g)


∆Go = [∆Gof(CH2=CHCl) +∆Gof(HCl)] - ∆Gof(CH2ClCH2Cl)
∆Go = [(1 mol)(51.9 kJ/mol) + (1 mol)(-95.3 kJ/mol)] - (1 mol)(-79.6 kJ/mol) = +36.2 kJ
Because ∆Go is positive, the reaction is nonspontaneous under standard-state conditions at 25oC.

CH2ClCH2Cl(l) → CH2=CHCl(g) + HCl(g)


Sum: NaOH(aq) + HCl(g) → Na+(aq) + Cl-(aq) + H2O(l)
CH2ClCH2Cl(l) + NaOH(aq) → CH2=CHCl(g) + Na+(aq) + Cl-(aq) + H2O(l)
∆Go = [∆Gof(CH2=CHCl) +∆Gof(Na+) + ∆Gof(Cl-) + ∆Gof(H2O)]

502
Chapter 17 - Thermodynamics: Entropy, Free Energy, and Equilibrium
______________________________________________________________________________

- [∆Gof(CH2ClCH2Cl) + ∆Gof(NaOH)]
∆Go = [(1 mol)(51.9 kJ/mol) + (1 mol)(-261.9 kJ/mol)
+ (1 mol)(-131.3 kJ/mol) + (1 mol)(-237.2 kJ/mol)]
- [(1 mol)(-79.6 kJ/mol) + (1 mol)(-419.2 kJ/mol)] = -79.7 kJ
Using NaOH(aq), ∆Go = -79.7 kJ and the reaction is spontaneous. (More generally, base
removes HCl, driving the reaction to the right.)
The synthesis of a compound from its constituent elements is thermodynamically feasible
at 25oC and 1 atm pressure if ∆Gof is negative.
Because ∆Gof(CH2=CHCl) = +51.9 kJ, the synthesis of vinyl chloride from its elements is
not possible at 25oC and 1 atm pressure.

Free Energy, Composition, and Chemical Equilibrium

17.80 ∆G = ∆Go + RT ln Q

17.81 ∆G = ∆Go + RT ln Q
(a) If Q < 1, then RT ln Q is negative and ∆G < ∆Go.
(b) If Q = 1, then RT ln Q = 0 and ∆G = ∆Go.
(c) If Q > 1, then RT ln Q is positive and ∆G > ∆Go.
As Q increases the thermodynamic driving force decreases.

o  (PSO3 )2 
17.82 ∆G = ∆G + RT ln  2 
 (PSO2 ) (PO2) 
 (1.0 )2 
(a) ∆G = (-141.8 kJ/mol) + [8.314 x 10-3 kJ/(K ⋅ mol)](298 K)ln  2  = -176.0 kJ/mol
 (100 ) (100) 

 (10 )2 
(b) ∆G = (-141.8 kJ/mol) + [8.314 x 10-3 kJ/(K ⋅ mol)](298 K)ln  2  = -133.8 kJ/mol
 (2.0 ) (1.0) 
(c) Q = 1, ln Q = 0, ∆G = ∆Go = -141.8 kJ/mol

[ 2]

17.83 ∆G = ∆Go + RT ln  NH 2 CONH2 
 (P NH3 ) (PCO2) 
 1.0 
(a) ∆G = -13.6 kJ/mol + [8.314 x 10-3 kJ/(K ⋅ mol)](298 K)ln  2  = -30.7 kJ/mol
 (10 ) (10) 
Because ∆G is negative, the reaction is spontaneous.
 1.0 
(b) ∆G = -13.6 kJ/mol + [8.314 x 10-3 kJ/(K ⋅ mol)](298 K)ln  2  = +3.5 kJ/mol
 (0.10 ) (0.10) 
Because ∆G is positive, the reaction is nonspontaneous.

17.84 ∆Go = -RT ln K


(a) If K > 1, ∆Go is negative. (b) If K = 1, ∆Go = 0.

503
Chapter 17 - Thermodynamics: Entropy, Free Energy, and Equilibrium
______________________________________________________________________________

(c) If K < 1, ∆Go is positive.

_ ∆G o

17.85 K = e RT (a) If ∆Go is positive, K is small. (b) If ∆Go is negative, K is large.


17.86 ∆Go = -RT ln Kp = -141.8 kJ
_ ∆G o _ (_ 141.8 kJ/mol)
ln Kp = = = 57.23
RT [8.314 x 10_ 3 kJ/(K • mol)](298 K)
Kp = e57.23 = 7.1 x 1024

17.87 ∆Go = - RT ln K = -13.6 kJ


_ ∆G o _ (_ 13.6 kJ/mol)
ln K = = = 5.49
RT [8.314 x 10_ 3 kJ/(K • mol)](298 K)
K = e5.49 = 2.4 x 102

17.88 C2H5OH(l) _ C2H5OH(g)


∆Go = ∆Gof(C2H5OH(g)) - ∆Gof(C2H5OH(l))
∆Go = (1 mol)(-168.6 kJ/mol) - (1 mol)(-174.9 kJ/mol) = +6.3 kJ
∆Go = -RT ln K
_ ∆G o _ (6.3 kJ/mol)
ln K = = = -2.54
RT [8.314 x 10_ 3 kJ/(K • mol)](298 K)
K = e-2.54 = 0.079; K = Kp = PC2H5 OH = 0.079 atm

17.89 ∆Go = -RT ln Ka


∆Go = -[8.314 x 10-3 kJ/(K ⋅ mol)](298 K)ln(3.0 x 10-4) = +20.1 kJ/mol

17.90 2 CH2=CH2(g) + O2(g) → 2 C2H4O(g)


∆Go = 2 ∆Gof(C2H4O) - 2 ∆Gof(CH2=CH2)
∆Go = (2 mol)(-13.1 kJ/mol) - (2 mol)(68.1 kJ/mol) = -162.4 kJ
∆Go = -RT ln K
_ ∆G o _ (_ 162.4 kJ/mol)
ln K = = = 65.55
RT [8.314 x 10_ 3 kJ/(K • mol)](298 K)
K = Kp = e65.55 = 2.9 x 1028

17.91 CO(g) + 2 H2(g) _ CH3OH(g)


∆Go = ∆Gof(CH3OH) - ∆Gof(CO)
∆Go = (1 mol)(-161.9 kJ/mol) - (1 mol)(-137.2 kJ/mol) = -24.7 kJ
∆Go = -RT ln Kp
_ ∆G o _ (_ 24.7 kJ/mol)
ln Kp = = = 9.97
RT [8.314 x 10_ 3 kJ/(K • mol)](298 K)
Kp = e9.97 = 2.1 x 104
∆G = ∆Go + RT ln Q
 20 
∆G = -24.7 kJ/mol + [8.314 x 10-3 kJ/(K ⋅ mol)](298 K) ln  2
= -39.5 kJ/mol
 (20)(20 ) 

504
Chapter 17 - Thermodynamics: Entropy, Free Energy, and Equilibrium
______________________________________________________________________________

General Problems

17.92 The kinetic parameters [(a), (b), and (h)] are affected by a catalyst. The thermodynamic
and equilibrium parameters [(c), (d), (e), (f), and (g)] are not affected by a catalyst.

17.93 (a), (c), and (d) are nonspontaneous; (b) is spontaneous.

17.94 (a) Spontaneous does not mean fast, just possible.


(b) For a spontaneous reaction ∆Stotal > 0. ∆Ssys can be positive or negative.
(c) An endothermic reaction can be spontaneous if ∆Ssys > 0.
(d) This statement is true because the sign of ∆G changes when the direction of a reaction
is reversed.

17.95 Point Total Possible Ways Number of Ways


2 (1+1) 1
3 (2+1)(1+2) 2
4 (1+3)(2+2)(3+1) 3
5 (1+4)(2+3)(3+2)(4+1) 4
6 (1+5)(2+4)(3+3)(4+2)(5+1) 5
7 (1+6)(2+5)(3+4)(4+3)(5+2)(6+1) 6
8 (2+6)(3+5)(4+4)(5+3)(6+2) 5
9 (3+6)(4+5)(5+4)(6+3) 4
10 (4+6)(5+5)(6+4) 3
11 (6+5)(5+6) 2
12 (6+6) 1
Because a point total of 7 can be rolled in the most ways, it is the most probable point total.

17.96

17.97 (a) Q = 1, ln Q = 0, ∆G = ∆Go = +79.9 kJ


Because ∆G is positive, the reaction is spontaneous in the reverse direction.
(b) ∆G = ∆Go + RT ln Q; Q = [H3O+][OH-] = (1.0 x 10-7)2 = 1.0 x 10-14
∆G = 79.9 kJ/mol + [8.314 x 10-3 kJ/(K ⋅ mol)](298 K) ln(1.0 x 10-14) = 0
Because ∆G = 0, the reaction is at equilibrium.
(c) ∆G = ∆Go + RT ln Q

505
Chapter 17 - Thermodynamics: Entropy, Free Energy, and Equilibrium
______________________________________________________________________________

Q = [H3O+][OH-] = (1.0 x 10-7)(1.0 x 10-10) = 1.0 x 10-17


∆G = 79.9 kJ/mol + [8.314 x 10-3 kJ/(K ⋅ mol)](298 K) ln(1.0 x 10-17) = -17.1 kJ/mol
Because ∆G is negative, the reaction is spontaneous in the forward direction.
The results are consistent with Le Châtelier's principle. When the [H3O+] and [OH-] are
larger than the equilibrium concentrations (a), the reverse reaction takes place. When the
product of [H3O+] and the [OH-] is less than the equilibrium value, the forward reaction is
spontaneous.
∆Go = -RT ln K
_ ∆G o _ 79.9 kJ/mol
ln K = = = -32.25
RT [8.314 x 10_ 3 kJ/(K • mol)](298 K)
K = Ka = e-32.25 = 9.9 x 10-15

17.98 At the normal boiling point, ∆G = 0.


∆ H vap 38,600 J
∆Gvap = ∆Hvap - T∆Svap; T= = = 351 K = 78oC
∆ Svap 110 J/K

17.99 At the normal boiling point, ∆Gvap = 0. 61oC = 334 K


∆ H vap 29,240 J
∆Gvap = ∆Hvap - T∆Svap; ∆Svap = = = 87.5 J/K
T 334 K

17.100 ∆G = ∆H - T∆S
(a) ∆H must be positive (endothermic) and greater than T∆S in order for ∆G to be
positive (nonspontaneous reaction).
(b) Set ∆G = 0 and solve for ∆H.
∆G = 0 = ∆H - T∆S = ∆H - (323 K)(104 J/K) = ∆H - (33592 J) = ∆H - (33.6 kJ)
∆H = 33.6 kJ
∆H must be greater than 33.6 kJ.

17.101 NH4NO3(s) → N2O(g) + 2 H2O(g)


(a) ∆Go = [∆Gof(N2O) + 2 ∆Gof(H2O)] - ∆Gof(NH4NO3)
∆Go = [(1 mol)(104.2 kJ/mol) + (2 mol)(-228.6 kJ/mol)] - (1 mol)(-184.0 kJ/mol)
∆Go = -169.0 kJ
Because ∆Go is negative, the reaction is spontaneous.
(b) Because the reaction increases the number of moles of gas, ∆So is positive.
∆Go = ∆Ho - T∆So
As the temperature is raised, ∆Go becomes more negative.
(c) ∆Go = -RT ln K
_ ∆G o _ (_ 169.0 kJ/mol)
ln K = = = 68.21
RT [8.314 x 10_ 3 kJ/(K • mol)](298 K)
K = Kp = e68.21 = 4.2 x 1029
(d) Q = (P N2 O)(P H2 O )2 = (30)(30)2 = (30)3
∆G = ∆Go + RT ln Q
∆G = -169.0 kJ/mol + [8.314 x 10-3kJ/(K ⋅ mol)](298 K) ln[(30)3] = -143.7 kJ/mol

506
Chapter 17 - Thermodynamics: Entropy, Free Energy, and Equilibrium
______________________________________________________________________________

17.102 (a) 2 Mg(s) + O2(g) → 2 MgO(s)


∆Ho = 2 ∆Hof(MgO) = (2 mol)(-601.7 kJ/mol) = -1203.4 kJ
∆So = 2 So(MgO) - [2 So(Mg) + So(O2)]
∆So = (2 mol)(26.9 J/(K ⋅ mol)) - [(2 mol)(32.7 J/(K ⋅ mol)) + (1 mol)(205.0 J/(K ⋅ mol))]
∆So = -216.6 J/K = -216.6 x 10-3 kJ/K
∆Go = ∆Ho - T∆So = -1203.4 kJ - (298 K)(-216.6 x 10-3 kJ/K) = -1138.8 kJ
Because ∆Go is negative, the reaction is spontaneous at 25oC. ∆Go becomes less
negative as the temperature is raised.
(b) MgCO3(s) → MgO(s) + CO2(g)
∆Ho = [∆Hof(MgO) + ∆Hof(CO2)] - ∆Hof(MgCO3)
∆Ho = [(1 mol)(-601.1 kJ/mol) + (1 mol)(-393.5 kJ/mol)] - (1 mol)(-1096 kJ/mol) = +101 kJ
∆So = [So(MgO) + So(CO2)] - So(MgCO3)
∆So = [(1 mol)(26.9 J/(K ⋅ mol)) + (1 mol)(213.6 J/(K ⋅ mol))] - (1 mol)(65.7 J/(K ⋅ mol))
∆So = 174.8 J/K = 174.8 x 10-3 kJ/K
∆Go = ∆Ho - T∆So = 101 kJ - (298 K)(174.8 x 10-3 kJ/K) = +49 kJ
Because ∆Go is positive, the reaction is not spontaneous at 25oC. ∆Go becomes less
positive as the temperature is raised.
(c) Fe2O3(s) + 2 Al(s) → Al2O3(s) + 2 Fe(s)
∆Ho = ∆Hof(Al2O3) - ∆Hof(Fe2O3)
∆Ho = (1 mol)(-1676 kJ/mol) - (1 mol)(-824.2 kJ/mol) = -852 kJ
∆So = [So(Al2O3) + 2 So(Fe)] - [So(Fe2O3) + 2 So(Al)]
∆So = [(1 mol)(50.9 J/(K ⋅ mol)) + (2 mol)(27.3 J/(K ⋅ mol))]
- [(1 mol)(87.4 J/(K ⋅ mol)) + (2 mol)(28.3 J/(K ⋅ mol))]
∆So = -38.5 J/K = -38.5 x 10-3 kJ/K
∆Go = ∆Ho - T∆So = -852 kJ - (298 K)(-38.5 x 10-3 kJ/K) = -840 kJ
Because ∆Go is negative, the reaction is spontaneous at 25oC. ∆Go becomes less negative
as the temperature is raised.
(d) 2 NaHCO3(s) → Na2CO3(s) + CO2(g) + H2O(g)
∆Ho = [∆Hof(Na2CO3) + ∆Hof(CO2) + ∆Hof(H2O)] - 2 ∆Hof(NaHCO3)
∆Ho = [(1 mol)(-1130.7 kJ/mol) + (1 mol)(-393.5 kJ/mol)
+ (1 mol)(-241.8 kJ/mol)] - (2 mol)(-950.8 kJ/mol) = +135.6 kJ
∆S = [S (Na2CO3) + So(CO2) + So(H2O)] - 2 So(NaHCO3)
o o

∆So = [(1 mol)(135.0 J/(K ⋅ mol)) + (1 mol)(213.6 J/(K ⋅ mol))


+ (1 mol)(188.7 J/(K ⋅ mol))] - (2 mol)(102 J/(K ⋅ mol))
o -3
∆S = +333 J/K = +333 x 10 kJ/K
∆Go = ∆Ho - T∆So = +135.6 kJ - (298 K)(+333 x 10-3 kJ/K) = +36.4 kJ
Because ∆Go is positive, the reaction is not spontaneous at 25oC. ∆Go becomes less
positive as the temperature is raised.

17.103 (a) ∆Hvap/Tbp


ammonia 120 J/K
benzene 87 J/K
carbon tetrachloride 85 J/K

507
Chapter 17 - Thermodynamics: Entropy, Free Energy, and Equilibrium
______________________________________________________________________________

chloroform 87 J/K
mercury 90 J/K
(b) All processes are the conversion of a liquid to a gas at the boiling point. They
should should all have similar ∆S values. ∆Hvap/Tbp is equal to ∆Svap.
(c) NH3 deviates from Trouton's rule because of hydrogen bonding. Because NH3(l) is
more ordered than the other liquids, ∆Svap is larger.

17.104 (a) 6 C(s) + 3 H2(g) → C6H6(l)


∆Sof = So(C6H6) - [6 So(C) + 3 So(H2)]
∆Sof = (1 mol)(172.8 J/(K ⋅ mol)) - [(6 mol)(5.7 J/(K ⋅ mol)) + (3 mol)(130.6 J/(K ⋅ mol))]
∆Sof = -253 J/K = -253 J/(K ⋅ mol)
∆Gof = ∆Hof - T∆Sof
∆ H of _ ∆ G of 49.0 kJ/mol _ 124.5 kJ/mol
∆Sof = = = -0.253 kJ/(K ⋅ mol)
T 298 K
∆Sof = -253 J/(K ⋅ mol)
Both calculations lead to the same value of ∆Sof.
(b) Ca(s) + S(s) + 2 O2(g) → CaSO4(s)
∆Sof = So(CaSO4) - [So(Ca) + So(S) + 2 So(O2)]
∆Sof = (1 mol)(107 J/(K ⋅ mol))
- [(1 mol)(41.4 J/(K ⋅ mol)) + (1 mol)(31.8 J/(K ⋅ mol)) + (2 mol)(205.0 J/(K ⋅ mol))]
∆Sof = -376 J/K = -376 J/(K ⋅ mol)
∆Gof = ∆Hof - T∆Sof
∆ H of _ ∆ G of _ 1434.1 kJ/mol _ (_ 1321.9 kJ/mol)
∆Sof = = = -0.376 kJ/(K ⋅ mol)
T 298 K
∆Sof = -376 J/(K ⋅ mol)
Both calculations lead to the same value of ∆Sof.
(c) 2 C(s) + 3 H2(g) + 1/2 O2(g) → C2H5OH(l)
∆Sof = So(C2H5OH) - [So(C) + So(H2) + 1/2 So(O2)]
∆Sof = (1 mol)(161 J/(K ⋅ mol))
- [(2 mol)(5.7 J/(K ⋅ mol)) + (3 mol)(130.6 J/(K ⋅ mol)) + (0.5 mol)(205.0 J/(K ⋅ mol))]
∆Sof = -345 J/K = -345 J/(K ⋅ mol)
∆Gof = ∆Hof - T∆Sof
∆ H of _ ∆ G of _ 277.7 kJ/mol _ (_ 174.9 kJ/mol)
∆Sof = = = -0.345 kJ/(K ⋅ mol)
T 298 K
∆Sof = -345 J/(K ⋅ mol)
Both calculations lead to the same value of ∆Sof.

17.105 MgCO3(s) → MgO(s) + CO2(g)


From Problem 17.102(b)
∆Ho = +101 kJ; ∆So = 174.8 J/K = 174.8 x 10-3 kJ/K
The equilibrium pressure of CO2 is equal to Kp = PCO2 . Kp is not affected by the
quantities of MgCO3 and MgO present. Kp can be calculated from ∆Go.
∆Go = ∆Ho - T∆So
∆Go = -RT ln Kp

508
Chapter 17 - Thermodynamics: Entropy, Free Energy, and Equilibrium
______________________________________________________________________________

(a) ∆Go = 101 kJ - (298 K)(174.8 x 10-3 kJ/K) = +49 kJ


_ ∆G o _ 49 kJ/mol
ln Kp = = = - 19.8
RT [8.314 x 10 kJ/(K • mol)](298 K)
_3

Kp = PCO2 = e-19.8 = 3 x 10-9 atm


(b) ∆Go = 101 kJ - (553 K)(174.8 x 10-3 kJ/K) = 4.3 kJ
_ ∆G o _ 4.3 kJ/mol
ln Kp = = = -0.94
RT [8.314 x 10_ 3 kJ/(K • mol)](553 K)
Kp = PCO2 = e-0.94 = 0.39 atm
(c) PCO2 = 0.39 atm because the temperature is the same as in (b).

17.106 ∆Go = -RT ln Kb


At 20 oC: ∆Go = -[8.314 x 10-3 kJ/(K ⋅ mol)](293 K) ln(1.710 x 10-5) = +26.74 kJ/mol
At 50 oC: ∆Go = -[8.314 x 10-3 kJ/(K ⋅ mol)](323 K) ln(1.892 x 10-5) = +29.20 kJ/mol
∆Go = ∆Ho - T∆So

26.74 = ∆Ho - 293∆So


29.20 = ∆Ho - 323∆So Solve these two equations simultaneously for ∆Ho and ∆So.

26.74 + 293∆So = ∆Ho


29.20 + 323∆So = ∆Ho Set these two equations equal to each other.

26.74 + 293∆So = 29.20 + 323∆So


26.74 - 29.20 = 323∆So - 293 ∆So
-2.46 = 30∆So
∆So = -2.46/30 = -0.0820 = -0.0820 kJ/K = -82.0 J/K
26.74 + 293∆So = 26.74 + 293(-0.0820) = ∆Ho = +2.71 kJ

17.107 (a) ∆Ho = 2 ∆Hof(NH3) = (2 mol)(- 46.1 kJ/mol) = -92.2 kJ


∆Go = 2 ∆Gof(NH3) = (2 mol)(-16.5 kJ/mol) = -33.0 kJ
∆Go = ∆Ho - T∆So
∆Ho - ∆Go = T∆So
∆H o _ ∆G o _ 92.2 kJ _ (_ 33.0 kJ)
∆So = = = -0.199 kJ/K = -199 J/K
T 298 K
(b) ∆So is negative because the number of mol of gas molecules decreases from 4 mol
to 2 mol on going from reactants to products.
(c) The reaction is spontaneous because ∆Go is negative.
(d) ∆Go = ∆Ho - T∆So = -92.2 kJ - (350 K)(-0.199 kJ/K) = -22.55 kJ
∆Go = -RT ln Kp
_ ∆G o _ (_ 22.55 kJ/mol)
ln Kp = = = 7.749
RT [8.314 x 10_ 3 kJ/(K • mol)](350 K)
Kp = e7.749 = 2.3 x 103
∆n = 2 - (1 + 3) = -2

509
Chapter 17 - Thermodynamics: Entropy, Free Energy, and Equilibrium
______________________________________________________________________________
∆n _2
 1   1  3 2
Kc = K p   = (2.3 x 103)   = (2.3 x 10 )(RT)
 RT   RT 
Kc = (2.3 x 10 )[(0.082 06)(350)]2 = 1.9 x 106
3

17.108 (a) ∆Ho = [∆Hof(Ag+(aq)) + ∆Hof(Br-(aq))] - ∆Hof(AgBr(s))


∆Ho = [(1 mol)(105.6 kJ/mol) + (1 mol)(-121.5 kJ/mol)] - (1 mol)(-100.4 kJ/mol) = +84.5 kJ
∆So = [So(Ag+(aq)) + So(Br-(aq))] - So(AgBr(s))
∆So = [(1 mol)(72.7 J/(K ⋅ mol)) + (1 mol)(82.4 J/(K ⋅ mol))]
- (1 mol)(107 J/(K ⋅ mol)) = +48.1 J
∆Go = ∆Ho - T∆So = 84.5 kJ - (298 K)(48.1 x 10-3 kJ/K) = +70.2 kJ
(b) ∆Go = -RT ln Ksp
_ ∆G o _ 70.2 kJ/mol
ln Ksp = = = -28.3
RT [8.314 x 10_ 3 kJ/(K • mol)](298 K)
Ksp = e-28.3 = 5 x 10-13
(c) Q = [Ag+][Br-] = (1.00 x 10-5)(1.00 x 10-5) = 1.00 x 10-10
∆G = ∆Go + RTlnQ
∆G = 70.2 kJ/mol + [8.314 x 10-3 kJ/(K ⋅ mol)](298 K) ln(1.00 x 10-10) = 13.2 kJ/mol
A positive value of ∆G means that the forward reaction is nonspontaneous under these
conditions. The reverse reaction is therefore spontaneous, which is consistent with the
fact that Q > Ksp.

17.109 (a) ∆Go = ∆Ho - T∆So and ∆Go = -RT ln K


Set the two equations equal to each other.
-RT ln K = ∆Ho - T∆So
∆ H o _ T ∆S o _ ∆ H o T ∆S o _ ∆ H o ∆S o
ln K = ; ln K = + ; ln K = +
_RT RT RT RT R
_ ∆ H  1  ∆S
o o
ln K =  + This is the equation for a straight line (y = mx + b).
R T R
∆H o 1 ∆S o
y = ln K; m=- = slope; x= ; b= = intercept
R T R
(b) Plot ln K versus 1/T
∆Ho = -R(slope) ∆So = R(intercept)
(c) For a reaction where K increases with increasing temperature, the following plot
would be obtained:

The slope is negative.

510
Chapter 17 - Thermodynamics: Entropy, Free Energy, and Equilibrium
______________________________________________________________________________

Because ∆Ho = -R(slope), ∆Ho is positive,


and the reaction is endothermic.

This prediction is in accord with LeChâtelier's principle because when you add heat
(raise the temperature) for an endothermic reaction, the reaction in the forward direction
takes place, the product concentrations increase and the reactant concentrations
decrease. This results in an increase in K.

17.110 Br2(l) _ Br2(g)


∆So = So(Br2(g)) - So(Br2(l))
∆So = (1 mol)(245.4 J/(K ⋅ mol)) - (1 mol)(152.2 J/(K ⋅ mol)) = 93.2 J/K = 93.2 x 10-3 kJ/K
∆G = ∆Ho - T∆So
At the boiling point, ∆G = 0.
0 = ∆Ho - Tbp∆So
∆H o
Tbp =
∆S o
∆Ho = Tbp ∆So = (332 K)(93.2 x 10-3 kJ/K) = 30.9 kJ
 1 atm 
Kp = P Br2 =  227 mm Hg x  = 0.299 atm
 760 mm Hg 
∆Go = -RT ln Kp and ∆Go = ∆Ho - T∆So (set equations equal to each other)
∆Ho - T∆So = -RT ln Kp (rearrange)
_ ∆ H o 1 ∆S o
ln K p = + (solve for T)
R T R
 _ ∆H o  _ 30.9 kJ/mol 
   
 R   8.314 x 10 kJ/(K • mol) 
_3
T= = = 299 K = 26oC
 ∆S  
o
93.2 x 10 kJ/(K • mol) 
_3
 ln K p _   ln (0.299) _ 
 R   8.314 x 10_ 3 kJ/(K • mol) 
Br2(l) has a vapor pressure of 227 mm Hg at 26oC.

17.111 For PbI2, Ksp = [Pb2+][I-]2


PbI2(s) _ Pb2+(aq) + 2 I-(aq)
initial (M) 0 0
equil (M) x 2x
Ksp = x(2x)2 = 4x3, where x = molar solubility
At 20oC = 20 + 273 = 293 K, Ksp = 4(1.45 x 10-3)3 = 1.22 x 10-8
At 80oC = 80 + 273 = 353 K, Ksp = 4(6.85 x 10-3)3 = 1.29 x 10-6
_ ∆ H o ∆S o
From problem 17.109, ln K = +
RT R

511
Chapter 17 - Thermodynamics: Entropy, Free Energy, and Equilibrium
______________________________________________________________________________

_ ∆H o
∆S o  _ ∆ H o ∆S o 
ln K1 - ln K2 = + - + 
RT1 R  RT 2 R 
_ ∆H o _ ∆H o _ ∆H o  1 1  ∆H o  1 1
ln K1 = - =  _ =  _ 
K2 RT1 RT 2 R  T1 T 2  R  T 2 T1 
[ln K1 _ ln K 2] R
∆Ho =
 1 1
 _ 
 T 2 T1 
[ln(1.22 x 10_ 8) _ ln(1.29 x 10_ 6)][8.314 x 10_ 3 kJ/(K • mol)]
∆Ho = = 66.8 kJ/mol
 1 1 
 _ 
 353 K 293 K 
∆Go = -RT ln Ksp = - [8.314 x 10-3 kJ/(K ⋅ mol)](293 K) ln(1.22 x 10-8) = 44.4 kJ/mol
∆Go = ∆Ho - T∆So
∆Ho - ∆Go = T∆So
∆H o _ ∆G o
∆So =
T
66.8 kJ/mol _ 44.4 kJ/mol
∆So = = 0.0765 kJ/(K ⋅ mol) = 76.5 J/(K ⋅ mol)
293 K

17.112 ∆Ho = [2 ∆Hof(Cl-(aq))] - [2 ∆Hof(Br-(aq))]


∆Ho = [(2 mol)(-167.2 kJ/mol)] - [(2 mol)(-121.5 kJ/mol)] = -91.4 kJ
∆So = [So(Br2(l)) + 2 So(Cl-(aq))] - [2 So(Br-(aq)) + So(Cl2(g))]
∆So = [(1 mol)(152.2 J/(K ⋅ mol)) + (2 mol)(56.5 J/(K ⋅ mol))]
- [(2 mol)(82.4 J/(K ⋅ mol)) + (1 mol)(223.0 J/(K ⋅ mol))] = -122.6 J/K
o
80 C = 80 + 273 = 353 K
∆Go = ∆Ho - T∆So = -91.4 kJ - (353 K)(-122.6 x 10-3 kJ/K) = - 48.1 kJ
∆Go = -RT ln K
_ ∆G o _ (_ 48.1 kJmol)
ln K = = = 16.4
RT [8.314 x 10_ 3 kJ/(K • mol)](353 K)
K = e16.4 = 1.3 x 107

17.113 CS2(l) _ CS2(g)


∆Ho = ∆Hof(CS2(g)) - ∆Hof(CS2(l))
∆Ho = [(1 mol)(116.7 kJ/mol)] - [(1 mol)(89.0 kJ/mol)] = 27.7 kJ
∆So = So(CS2(g)) - So(CS2(l))
∆So = [(1 mol)(237.7 J/(K ⋅ mol))] - [(1 mol)(151.3 J/(K ⋅ mol))] = 86.4 J/K
∆G = ∆Ho - T∆So
At the boiling point, ∆G = 0.
0 = ∆Ho - Tbp∆So
∆H o 27.7 kJ
Tbp = = = 321 K
∆S o
86.4 x 10_ 3 kJ/K
Tbp = 321 K = 321 - 273 = 48oC

512
Chapter 17 - Thermodynamics: Entropy, Free Energy, and Equilibrium
______________________________________________________________________________

17.114 35oC = 35 + 273 = 308 K


∆Go = ∆Ho - T∆So = -352 kJ - (308 K)(-899 x 10-3 kJ/K) = -75.1 kJ
∆Go = -RT ln Kp
_ ∆G o _ (_ 75.1 kJ/mol)
ln Kp = = = 29.33
RT [8.314 x 10_ 3 kJ/(K • mol)](308 K)
Kp = e29.33 = 5.5 x 1012
1
Kp = 6
= 5.5 x 1012
( P H2 O )
1
P H2 O = 6 = 0.0075 atm
5.5 x 1012
760 mm Hg
P H2 O = 0.0075 atm x = 5.7 mm Hg
1 atm

17.115 2 KClO3(s) → 2 KCl(s) + 3 O2(g)


∆Ho = 2 ∆Hof(KCl) - 2 ∆Hof(KClO3)
∆Ho = (2 mol)(- 436.7 kJ) - (2 mol)(-397.7 kJ) = -78.0 kJ
25oC = 25 + 273 = 298 K
∆Go = ∆Ho - T∆So
∆Ho - ∆Go = T∆So
∆H o _ ∆G o _ 78.0 kJ _ (_ 225.8 kJ)
∆So = = = 0.496 kJ/K = 496 J/K
T 298 K
∆So = [2 So(KCl) + 3 So(O2)] - 2 So(KClO3)
496 J/K = [(2 mol)(82.6 J/(K ⋅ mol)) + (3 mol)So(O2)] - (2 mol)(143 J/(K ⋅ mol))
(3 mol)So(O2) = 496 J/K - (2 mol)(82.6 J/(K ⋅ mol)) + (2 mol)(143 J/(K ⋅ mol))
(3 mol)So(O2) = 616.8 J/K
So(O2) = (616.8 J/K)/(3 mol) = 205.6 J/(K ⋅ mol) = 206 J/(K ⋅ mol)

17.116 N2O4(g) _ 2 NO2(g)


∆Ho = 2 ∆Hof(NO2) - ∆Hof(N2O4) = (2 mol)(33.2 kJ) - (1 mol)(9.16 kJ) = 57.2 kJ
∆So = 2 So(NO2) - So(N2O4) = (2 mol)(240.0 J/(K ⋅ mol)) - (1 mol)(304.2 J/(K ⋅ mol))
∆So = 175.8 J/K = 175.8 x 10-3 kJ/K
∆Go = ∆Ho - T∆So and ∆Go = -RT ln Kp; Set these two equations equal to each other
and solve for T.
∆Ho - T∆So = -RT ln Kp
∆Ho = T∆So - RT ln Kp = T(∆So - R ln Kp)
∆H o
T=
∆ S o _ R ln K p
(a) P N2O4 + P NO2 = 1.00 atm and P NO2 = 2 P N2O4
P N2O4 + 2 P N2O4 = 3 P N2O4 = 1.00 atm
P N2O4 = 1.00 atm/3 = 0.333 atm
P NO2 = 1.00 atm _ P N2O4 = 1.00 - 0.333 = 0.667 atm

513
Chapter 17 - Thermodynamics: Entropy, Free Energy, and Equilibrium
______________________________________________________________________________

(P NO2 )2 (0.667 )2
Kp = = = 1.34
P N2 O4 (0.333)
∆H o
T=
∆ S o _ R ln K p
57.2 kJ/mol
T= = 330 K
[175.8 x 10 kJ/(K • mol)] _ [8.314 x 10_ 3 kJ/(K • mol)] ln (1.34)
_3

T = 330 K = 330 - 273 = 57oC


(b) P N2O4 + P NO2 = 1.00 atm and P NO2 = P N2O4 so P NO2 = P N2O4 = 0.50 atm
(P NO2 )2 (0.500 )2
Kp = = = 0.500
P N2 O4 (0.500)
∆H o
T=
∆ S o _ R ln K p
57.2 kJ/mol
T= = 315 K
[175.8 x 10 kJ/(K • mol)] _ [8.314 x 10_ 3 kJ/(K • mol)] ln (0.500)
_3

T = 315 K = 315 - 273 = 42oC

Multi-Concept Problems

17.117 N2O4(g) _ 2 NO2(g)


∆Ho = 2 ∆Hof(NO2) - ∆Hof(N2O4) = (2 mol)(33.2 kJ) - (1 mol)(9.16 kJ) = 57.2 kJ
∆So = 2 So(NO2) - So(N2O4) = (2 mol)(240.0 J/(K ⋅ mol)) - (1 mol)(304.2 J/(K ⋅ mol))
∆So = 175.8 J/K = 175.8 x 10-3 kJ/K
∆Go = ∆Ho - T∆So = 57.2 kJ - (373 K)(175.8 x 10-3 kJ/K) = -8.4 kJ
(P )2
Kp = NO2
P N2 O4
o
∆G = -RT ln Kp
_ ∆G o _ (_ 8.4 kJ/mol)
ln Kp = = = 2.71
RT [8.314 x 10_ 3 kJ/(K • mol)](373 K)
Kp = e2.71 = 15
N2O4(g) _ 2 NO2(g)
initial (atm) 1.00 1.00
change (atm) -x +2x
equil (atm) 1.00 - x 1.00 + 2x
2 2
(P ) (1.00 + 2 x )
Kp = NO2 = 15 =
P N2O4 (1.00 _ x)
2
4x + 19x - 14 = 0
Use the quadratic formula to solve for x.
_ (19) ± (19 )2 _ (4)(4)(_ 14) _ 19 ± 24.2
x= =
2(4) 8

514
Chapter 17 - Thermodynamics: Entropy, Free Energy, and Equilibrium
______________________________________________________________________________

x = 0.65 and -5.4


Of the two solutions for x, only 0.65 has physical meaning because -5.4 would lead to a
negative partial pressure for NO2.
P N2O4 = 1.00 - x = 1.00 - 0.65 = 0.35 atm; P NO2 = 1.00 + 2x = 1.00 + 2(0.65) = 2.30 atm

17.118 N2(g) + 3 H2(g) _ 2 NH3(g)


∆Ho = 2 ∆Hof(NH3) - [∆Hof(N2) + 3 ∆Hof(H2)] = (2 mol)(- 46.1 kJ) - [0] = -92.2 kJ
∆So = 2 So(NH3) - [So(N2) + 3 So(H2)]
∆So = (2 mol)(192.3 J/(K ⋅ mol))
- [(1 mol)(191.5 J/(K ⋅ mol)) + (3 mol)(130.6 J/(K ⋅ mol))] = -198.7 J/K
∆Go = ∆Ho - T∆So = -92.2 kJ - (673 K)(-198.7 x 10-3 kJ/K) = 41.5 kJ
∆Go = -RT ln Kp
_ ∆G o _ 41.5 kJ/mol
ln Kp = = = -7.42
RT [8.314 x 10_ 3 kJ/(K • mol)](673 K)
Kp = e-7.42 = 6.0 x 10-4
Because Kp = Kc(RT)∆n, Kc = Kp(RT)-∆n
Kc = Kp(RT)2 = (6.0 x 10-4)[(0.082 06)(673)]2 = 1.83
N2, 28.01 amu; H2, 2.016 amu
Initial concentrations:
 1 mol   1 mol 
(14.0 g)  (3.024 g) 
 28.01 g   2.016 g 
[N2] = = 0.100 M and [H2] = = 0.300 M
5.00 L 5.00 L

N2(g) + 3 H2(g) _ 2 NH3(g)


initial (M) 0.100 0.300 0
change (M) -x -3x +2x
equil (M) 0.100 - x 0.300 - 3x 2x
2 2
[ NH3 ] (2 x ) 4 x2
Kc = = = = 1.83
[ N 2] [H 2 ]3 (0.100 _ x)(0.300 _ 3 x )3 27(0.100 _ x )4

2
 x  (27)(1.83) x
 
 (0.100 _ x )2  = 4
= 12.35;
(0.100 _ x )2
= 12.35 = 3.515
 

3.515x2 - 1.703x + 0.03515 = 0


Use the quadratic formula to solve for x.
_ (_ 1.703) ± (_ 1.703 )2 _ (4)(3.515)(0.03515) 1.703 ± 1.551
x= =
2(3.515) 7.030
x = 0.463 and 0.0216
Of the two solutions for x, only 0.0216 has physical meaning because 0.463 would lead
to negative concentrations of N2 and H2.
[N2] = 0.100 - x = 0.100 - 0.0216 = 0.078 M
[H2] = 0.300 - 3x = 0.300 - 3(0.0216) = 0.235 M

515
Chapter 17 - Thermodynamics: Entropy, Free Energy, and Equilibrium
______________________________________________________________________________

[NH3] = 2x = 2(0.0216) = 0.043 M

17.119 2 SO2(g) + O2(g) _ 2 SO3(g)


∆Ho = 2 ∆Hof(SO3) - 2 ∆Hof(SO2)
∆Ho = (2 mol)(-395.7 kJ/mol) - (2 mol)(-296.8 kJ/mol) = -197.8 kJ
∆So = 2 So(SO3) - [2 So(SO2) + So(O2)]
∆So = (2 mol)(256.6 J/(K ⋅ mol)) - [(2 mol)(248.1 J/(K ⋅ mol)) + (1 mol)(205.0 J/(K ⋅ mol))]
∆So = -188.0 J/K = -188.0 x 10-3 kJ/K
∆Go = ∆Ho - T∆So = -197.8 kJ - (800 K)(-188.0 x 10-3 kJ/K) = - 47.4 kJ
∆Go = -RT ln Kp
_ ∆G o _ (_ 47.4 kJ/mol)
ln Kp = = = 7.13
RT [8.314 x 10_ 3 kJ/(K • mol)](800 K)
Kp = e7.13 = 1249
SO2, 64.06 amu; O2, 32.00 amu
At 800 K:
 1 mol  L • atm 
192 g x  0.082 06 (800 K)
nRT  64.06 g  K • mol 
PSO2 = = = 13.1 atm
V 15.0 L

 1 mol  L • atm 
 48.0 g x  0.08206 (800 K)
nRT  32.00 g  K • mol 
PO2 = = = 6.57 atm
V 15.0 L

2 SO2(g) + O2(g) _ 2 SO3(g)


initial (atm) 13.1 6.57 0
assume complete rxn (atm) 0 0 13.1
assume a small back rxn +2x +x -2x
equil (atm) 2x x 13.1 - 2x
2 2 2
[SO3 ] (13.1 _ 2 x ) (13.1 )
Kp = 1249 = 2
= 2

[SO2 ] [O2] (2 x ) (x) (2 x )2 (x)
Solve for x. x3 = 0.0343; x = 0.325
Use successive approximations to solve for x because 2x is not negligible compared with
13.1.

Second approximation:
[13.1 _ (2)(0.325) ]2
1249 = ; Solve for x. x3 = 0.0310; x = 0.314
(2 x )2 (x)

Third approximation:
[13.1 _ (2)(0.314) ]2
1249 = ; Solve for x. x3 = 0.0311; x = 0.315 (x has converged)
(2 x )2 (x)

516
Chapter 17 - Thermodynamics: Entropy, Free Energy, and Equilibrium
______________________________________________________________________________

PSO2 = 2x = 2(0.315) = 0.63 atm


PO2 = x = 0.32 atm
PSO3 = 13.1 - 2x = 13.1 - 2(0.315) = 12.5 atm
(b) The % yield of SO3 decreases with increasing temperature because ∆So is negative.
∆Go becomes less negative and Kp gets smaller as the temperature increases.
(c) At 1000 K:
∆Go = ∆Ho - T∆So = -197.8 kJ - (1000 K)(-188.0 x 10-3 kJ/K) = -9.8 kJ
∆Go = -RT ln Kp
_ ∆G o _ (_ 9.8 kJ/mol)
ln Kp = = = 1.179
RT [8.314 x 10_ 3 kJ/(K • mol)](1000 K)
Kp = e1.179 = 3.25
 1 mol  L • atm 
192 g x  0.082 06 (1000 K)
nRT  64.06 g  K • mol 
PSO2 = = = 16.4 atm
V 15.0 L
 1 mol  L • atm 
 48.0 g x  0.082 06 (1000 K)
nRT  32.00 g  K • mol 
PO2 = = = 8.2 atm
V 15.0 L

2 SO2(g) + O2(g) _ 2 SO3(g)


initial (atm) 16.4 8.2 0
assume complete rxn (atm) 0 0 16.4
assume a small back rxn +2x +x -2x
equil (atm) 2x x 16.4 - 2x
[SO3 ]2 (16.4 _ 2 x )2 (16.4 )2
Kp = 3.25 = = ≈
[SO2 ]2 [O2] (2 x )2 (x) (2 x )2 (x)
Solve for x. x3 = 20.7; x = 2.7
Use successive approximations to solve for x because 2x is not negligible compared with
16.4.

Second approximation:
[16.4 _ (2)(2.7) ]2
3.25 = 2
; Solve for x. x3 = 9.31; x = 2.1
(2 x ) (x)

Third approximation:
[16.4 _ (2)(2.1) ]2
3.25 = ; Solve for x. x3 = 11.4; x = 2.3
(2 x )2 (x)

Fourth approximation:
[16.4 _ (2)(2.3) ]2
3.25 = ; Solve for x. x3 = 10.7; x = 2.2 (x has converged)
(2 x )2 (x)

517
Chapter 17 - Thermodynamics: Entropy, Free Energy, and Equilibrium
______________________________________________________________________________

PSO2 = 2x = 2(2.2) = 4.4 atm


PO2 = x = 2.2 atm
PSO3 = 16.4 - 2x = 16.4 - 2(2.2) = 12.0 atm
Ptotal = PSO2 + PO2 + PSO3 = 4.4 + 2.2 + 12.0 = 18.6 atm
On going from 800 K to 1000 K, Ptotal increases to 18.6 atm (because Kp decreases, but
P increases with temperature at constant volume).

17.120 Pb(s) + PbO2(s) + 2 H+(aq) + 2 HSO4-(aq) → 2 PbSO4(s) + 2 H2O(l)


(a) ∆Go = [2 ∆Gof(PbSO4) + 2 ∆Gof(H2O)] - [∆Gof(PbO2) + 2 ∆Gof(HSO4-)]
∆Go = (2 mol)(-813.2 kJ/mol) + (2 mol)(-237.2 kJ/mol)]
- [(1 mol)(-217.4 kJ/mol) + (2 mol)(-756.0 kJ/mol)] = -371.4 kJ
(b) C = 5/9(oF - 32) = 5/9(10 - 32) = -12.2oC;
o
-12.2oC = 261 K
∆Ho = [2 ∆Hof(PbSO4) + 2 ∆Hof(H2O)] - [∆Hof(PbO2) + 2 ∆Hof(HSO4-)]
∆Ho = [(2 mol)(-919.9 kJ/mol) + (2 mol)(-285.8 kJ/mol)]
- [(1 mol)(-277 kJ/mol) + (2 mol)(-887.3 kJ/mol)] = -359.8 kJ
∆So = [2 So(PbSO4) + 2 So(H2O)] - [So(Pb) + So(PbO2) + 2 So(H+) + 2 So(HSO4-)]
∆So = [(2 mol)(148.6 J/(K ⋅ mol)) + (2 mol)(69.9 J/(K ⋅ mol))]
- [(1 mol)(64.8 J/(K ⋅ mol)) + (1 mol)(68.6 J/(K ⋅ mol))
+ (2 mol)(132 J/(K ⋅ mol))] = 39.6 J/K = 39.6 x 10-3 kJ/K
∆G = ∆H - T∆S = -359.8 kJ -(261 K)(39.6 x 10-3 kJ/K) = -370.1 kJ at 261 K
o o o

HSO4-(aq) + H2O(l) _ H3O+(aq) + SO42-(aq)


initial (M) 0.100 0.100 0
change (M) -x +x +x
equil (M) 0.100 - x 0.100 + x x

+
[ ][ 24 _ ] (0.100 + x) x
Ka2 = H3 O SO _
= 1.2 x 10-2 =
[HSO4 ] 0.100 _ x
2 -3
x + 0.112x - (1.2 x 10 ) = 0
Use the quadratic formula to solve for x.
_ (0.112) ± (0.112 )2 _ (4)(1)(_ 1.2 x 10_ 3) _ 0.112 ± 0.132
x= =
2(1) 2
x = -0.122 and 0.010
Of the two solutions for x, only 0.010 has physical meaning because -0.122 would lead
to negative concentrations of H3O+ and SO42-.
[H+] = 0.100 + x = 0.100 + 0.010 = 0.110 M
[HSO4-] = 0.100 - x = 0.100 - 0.010 = 0.090 M
1
∆G = ∆Go + RT ln + 2
[H ] [HSO4_ ]2
1
∆G = (-370.1 kJ/mol) + [8.314 x 10-3 kJ/(K ⋅ mol)](261 K) ln
(0.110 ) (0.090 )2
2

∆G = -350.1 kJ/mol

518
Chapter 17 - Thermodynamics: Entropy, Free Energy, and Equilibrium
______________________________________________________________________________

17.121 CaCO3(s) _ Ca2+(aq) + CO32-(aq)


∆Ho = [∆Hof(Ca2+) + ∆Hof(CO32-)] - ∆Hof(CaCO3)
∆Ho = [(1 mol)(-542.8 kJ/mol) + (1 mol)(-677.1 kJ/mol)] - (1 mol)(-1206.9 kJ/mol)
∆Ho = -13.0 kJ
∆So = [So(Ca2+) + So(CO32-)] - So(CaCO3)
∆So = [(1 mol)(-53.1 J/(K ⋅ mol)) + (1 mol)(-56.9 J/(K ⋅ mol))] - (1 mol)(92.9 J/(K ⋅ mol))
∆So = -202.9 J/K = -202.9 x 10-3 kJ/K
50oC = 50 + 273 = 323 K
∆G = ∆Ho - T∆So = -13.0 kJ - (323 K)(-202.9 x 10-3 kJ/K) = +52.54 kJ
∆G = -RT ln Ksp
_ ∆G _ 52.54 kJ/mol
ln Ksp = = = -19.56
RT [8.314 x 10_ 3 kJ/(K • mol)](323 K)
Ksp = e-19.56 = 3.2 x 10-9

20oC = 20 +273 = 293 K


 1.00 atm 
 731 mm Hg x (1.000 L)
PV  760 mm Hg 
n CO2 = = = 0.0400 mol CO2
RT  L • atm 
 0.082 06 (293 K)
 K • mol 
Ca(OH)2, 74.09 amu
1 mol Ca(OH )2
mol Ca(OH)2 = 3.335 g Ca(OH)2 x = 0.0450 mol Ca(OH)2
74.09 g Ca(OH )2
CO2(g) + H2O(l) → H2CO3(aq)

Ca(OH)2(aq) + H2CO3(aq) → CaCO3(s) + 2 H2O(l)


before (mol) 0.0450 0.0400 0
change (mol) -0.0400 -0.0400 +0.0400
after (mol) 0.0050 0 0.0400

500.0 mL = 0.5000 L
[Ca(OH)2] = [Ca2+] = 0.0050 mol/0.5000 L = 0.010 M

CaCO3(s) _ Ca2+(aq) + CO32-(aq)


initial (M) 0.010 0
change (M) +x +x
equil (M) 0.010 + x x
Ksp = [Ca ][CO3 ] = 3.2 x 10 = (0.010 + x)x ≈0.010x
2+ 2- -9

x = molar solubility = 3.2 x 10-9/0.010 = 3.2 x 10-7 M


Because ∆Ho is negative (exothermic), the solubility of CaCO3 is lower at 50oC.

17.122 PV = nRT

519
Chapter 17 - Thermodynamics: Entropy, Free Energy, and Equilibrium
______________________________________________________________________________

 1.00 atm 
 744 mm Hg x (1.00 L)
PV  760 mm Hg 
n NH3 = = = 0.0400 mol NH3
RT  L • atm 
 0.082 06 (298.1 K)
 K • mol 
500.0 mL = 0.5000 L
[NH3] = 0.0400 mol/0.5000 L = 0.0800 M
NH3(aq) + H2O(l) _ NH4+(aq) + OH-(aq)
∆Ho = [∆Hof(NH4+) + ∆Hof(OH-)] - [∆Hof(NH3) + ∆Hof(H2O)]
∆Ho = [(1 mol)(-132.5 kJ/mol) + (1 mol)(-230.0 kJ/mol)]
- [(1 mol)(-80.3 kJ/mol) + (1 mol)(-285.8 kJ/mol)] = +3.6 kJ
∆S = [S (NH4+) + So(OH-)] - [So(NH3) + So(H2O)]
o o

∆So = [(1 mol)(113 J/(K ⋅ mol)) + (1 mol)(-10.8 J/(K ⋅ mol))]


- [(1 mol)(111 J/(K ⋅ mol)) + (1 mol)(69.9 J/(K ⋅ mol))] = -78.7 J/K
T = 2.0oC = 2.0 + 273.1 = 275.1 K
∆Go = ∆Ho - T∆So = 3.6 kJ - (275.1 K)(-78.7 x 10-3 kJ/K) = 25.3 kJ
∆Go = -RT ln Kb
_ ∆G o _ 25.3 kJ/mol
ln Kb = = = -11.06
RT [8.314 x 10 kJ/(K • mol)](275.1 K)
_3

Kb = e-11.06 = 1.6 x 10-5

NH3(aq) + H2O(l) _ NH4+(aq) + OH-(aq)


initial (M) 0.0800 0 ~0
change (M) -x +x +x
equil (M) 0.0800 - x x x

+ _ 2 2
[ ][ ]
at 2oC, Kb = NH 4 OH = 1.6 x 10-5 = x ≈ x
[ NH3] 0.0800 _ x 0.0800
2 -5
x = (1.6 x 10 )(0.0800)
x = [OH-] = (1.6 x 10_ 5)(0.0800) = 1.13 x 10-3 M
1.0 x 10_14
[H3O+] = _3
= 8.85 x 10-12 M
1.13 x 10
pH = -log[H3O+] = -log(8.85 x 10-12) = 11.05

17.123 (a) I2(s) → 2 I-(aq)


[I2(s) + 2 e- → 2 I-(aq)] x 5 reduction half reaction

I2(s) → 2 IO3-(aq)
I2(s) + 6 H2O(l) → 2 IO3-(aq)
I2(s) + 6 H2O(l) → 2 IO3-(aq) + 12 H+(aq)
I2(s) + 6 H2O(l) → 2 IO3-(aq) + 12 H+(aq) + 10 e- oxidation half reaction

520
Chapter 17 - Thermodynamics: Entropy, Free Energy, and Equilibrium
______________________________________________________________________________

Combine the two half reactions.


6 I2(s) + 6 H2O(l) → 10 I-(aq) + 2 IO3-(aq) + 12 H+(aq)
Divide all coefficients by 2.
3 I2(s) + 3 H2O(l) → 5 I-(aq) + IO3-(aq) + 6 H+(aq)
3 I2(s) + 3 H2O(l) + 6 OH-(aq) → 5 I-(aq) + IO3-(aq) + 6 H+(aq) + 6 OH-(aq)
3 I2(s) + 3 H2O(l) + 6 OH-(aq) → 5 I-(aq) + IO3-(aq) + 6 H2O(l)
3 I2(s) + 6 OH-(aq) → 5 I-(aq) + IO3-(aq) + 3 H2O(l)
(b) ∆Go = [5 ∆Gof(I-) + ∆Gof(IO3-) + 3 ∆Gof(H2O(l))] - 6 ∆Gof(OH-)
∆Go = [(5 mol)(-51.6 kJ/mol) + (1 mol)(-128.0 kJ/mol) + (3 mol)(-237.2 kJ/mol)]
- (6 mol)(-157.3 kJ/mol) = -153.8 kJ
(c) The reaction is spontaneous because ∆Go is negative.
(d) 25oC = 25 + 273 = 298 K
∆Go = -RT ln Kc
_ ∆G o _ (_ 153.8 kJ/mol)
ln Kc = = = 62.077
RT [8.314 x 10_ 3 kJ/(K • mol)](298 K)
Kc = e62.077 = 9.1 x 1026
[ _ ]5 [IO3_ ] (0.10 )5 (0.50)
Kc = I = 9.1 x 10 26
=
[OH _ ]6 [OH _ ]6
5
(0.10 ) (0.50)
[OH-] = 6 = 4.2 x 10-6 M
9.1 x 1026
1.0 x 10_ 14
[H3O+] = _6
= 2.38 x 10-9 M
4.2 x 10
pH = -log[H3O+] = -log(2.38 x 10-9) = 8.62

521
522
18 Electrochemistry

18.1 2 Ag+(aq) + Ni(s) → 2 Ag(s) + Ni2+(aq)

There is a Ni anode in an aqueous solution of Ni2+, and a Ag cathode in an aqueous


solution of Ag+. A salt bridge connects the anode and cathode compartment. The
electrodes are connected through an external circuit.

18.2 Fe(s)│Fe2+(aq)║Sn2+(aq)│Sn(s)

18.3 Pb(s) + Br2(l) → Pb2+(aq) + 2 Br-(aq)


There is a Pb anode in an aqueous solution of Pb2+. The cathode is a Pt wire that dips
into a pool of liquid Br2 and an aqueous solution that is saturated with Br2. A salt bridge
connects the anode and cathode compartment. The electrodes are connected through an
external circuit.

18.4 (a) and (b)

(c) 2 Al(s) + 3 Co2+(aq) → 2 Al3+(aq) + 3 Co(s)

523
(d) Al(s)│Al3+(aq)║Co2+(aq)│Co(s)
18.5 Al(s) + Cr3+(aq) → Al3+(aq) + Cr(s)
 96,500 C   1J 
∆Go = -nFEo = -(3 mol e-)  (0.92 V)  = -266,340 J = -270 kJ
1 C • V 
_
 1 mol e 

18.6 oxidation: Al(s) → Al3+(aq) + 3 e- Eo = 1.66 V


reduction: Cr3+(aq) + 3 e- → Cr(s) Eo = ?
overall Al(s) + Cr (aq) → Al (aq) + Cr(s)
3+ 3+
Eo = 0.92 V
The standard reduction potential for the Cr3+/Cr half cell is:
Eo = 0.92 - 1.66 = -0.74 V

18.7 (a) Cl2(g) + 2 e- → 2 Cl-(aq) Eo = 1.36 V


Ag+(aq) + e- → Ag(s) Eo = 0.80 V
Cl2 has the greater tendency to be reduced (larger Eo). The species that has the greater
tendency to be reduced is the stronger oxidizing agent. Cl2 is the stronger oxidizing
agent.

(b) Fe2+(aq) + 2 e- → Fe(s) Eo = -0.45 V


Mg2+(aq) + 2 e- → Mg(s) Eo = -2.37 V
The second half-reaction has the lesser tendency to occur in the forward direction (more
negative Eo) and the greater tendency to occur in the reverse direction. Therefore, Mg is
the stronger reducing agent.

18.8 (a) 2 Fe3+(aq) + 2 I-(aq) → 2 Fe2+(aq) + I2(s)


reduction: Fe3+(aq) + e- → Fe2+(aq) Eo = 0.77 V
oxidation: 2 I-(aq) → I2(s) + 2 e- Eo = -0.54 V
overall Eo = 0.23 V
Because Eo for the overall reaction is positive, this reaction can occur under standard-
state conditions.

(b) 3 Ni(s) + 2 Al3+(aq) → 3 Ni2+(aq) + 2 Al(s)


oxidation: Ni(s) → Ni2+(aq) + 2 e- Eo = 0.26 V
reduction: Al (aq) + 3 e → Al(s)
3+ -
Eo = -1.66 V
overall Eo = -1.40 V
o
Because E for the overall reaction is negative, this reaction cannot occur under standard-
state conditions. This reaction can occur in the reverse direction.

18.9 (a) D is the strongest reducing agent. D+ has the most negative standard reduction
potential. A3+ is the strongest oxidizing agent. It has the most positive standard
reduction potential.
(b) An oxidizing agent can oxidize any reducing agent that is below it in the table. B2+
can oxidize C and D.
A reducing agent can reduce any oxidizing agent that is above it in the table. C can
reduce A3+ and B2+.

(c) Use the two half-reactions that have the most positive and the most negative standard

524
Chapter 18 - Electrochemistry
_____________________________________________________________________________

reduction potentials, respectively.


A3+ + 2 e- → A+ 1.47 V
2 x (D → D+ + e-) 1.38 V
A3+ + 2 D → A+ + 2 D+ 2.85 V

18.10 Cu(s) + 2 Fe3+(aq) → Cu2+(aq) + 2 Fe2+(aq)


Eo = EoCu _ Cu2+ + EoFe3+ _ Fe2+ = -0.34 V + 0.77 V = 0.43 V; n = 2 mol e-
0.0592 V [Cu 2+ ][Fe2+ ]2 (0.0592 V) (0.25)(0.20 )2
E = Eo - log = 0.43 V - log = 0.25 V
n [Fe3+ ]2 2 (1.0 x 10_ 4 )2

18.11 5 [Cu(s) → Cu2+(aq) + 2 e-] oxidation half reaction


2 [5 e + 8 H (aq) + MnO4 (aq) → Mn2+(aq) + 4 H2O(l)]
- + -
reduction half reaction

5 Cu(s) + 16 H+(aq) + 2 MnO4-(aq) → 5 Cu2+(aq) + 2 Mn2+(aq) + 8 H2O(l)


0.0592 V [Cu 2+ ]5 [Mn 2+ ]2
∆E = - log
n [MnO4_ ]2 [H + ]16
(a) The anode compartment contains Cu2+.
0.0592 V (0.01)5 (1 )2
∆E = - log = +0.059 V
10 (1)2 (1)16
(b) The cathode compartment contains Mn2+, MnO4-, and H+.
0.0592 V (1 )5 (0.01 )2
∆E = - log = -0.19 V
10 (0.01)2 (0.01)16

18.12 H2(g) + Pb2+(aq) → 2 H+(aq) + Pb(s)


Eo = EoH2 _ H+ + EoPb2+ _ Pb = 0 V + (-0.13 V) = -0.13 V; n = 2 mol e-
0.0592 V [ O+ ]2
E = Eo - log H23+
n [Pb ](P H2)
(0.0592 V) [ O+ ]2
0.28 V = -0.13 V - log H3 = -0.13 V - (0.0592 V) log [H3O+]
2 (1)(1)
pH = -log[H3O+] therefore 0.28 V = -0.13 V + (0.0592 V) pH
(0.28 V + 0.13 V)
pH = = 6.9
0.0592 V

18.13 4 Fe2+(aq) + O2(g) + 4 H+(aq) → 4 Fe3+(aq) + 2 H2O(l)


Eo = EoFe2+ _ Fe3+ + EoO2 _ H2 O = -0.77 V + 1.23 V = 0.46 V; n = 4 mol e-
0.0592 V
Eo = log K
n
nE o (4)(0.46 V)
log K = = = 31; K = 1031 at 25oC
0.0592 V 0.0592 V

525
Chapter 18 - Electrochemistry
_____________________________________________________________________________

0.0592 V 0.0592 V
18.14 Eo = log K = log (1.8 x 10_ 5) = -0.140 V
n 2

18.15 (a) Zn(s) + 2 MnO2(s) + 2 NH4+(aq) → Zn2+(aq) + Mn2O3(s) + 2 NH3(aq) + H2O(l)


(b) Zn(s) + 2 MnO2(s) → ZnO(s) + Mn2O3(s)
(c) Zn(s) + HgO(s) → ZnO(s) + Hg(l)
(d) Cd(s) + 2 NiO(OH)(s) + 2 H2O(l) → Cd(OH)2(s) + 2 Ni(OH)2(s)

18.16 (a) [Mg(s) → Mg2+(aq) + 2 e-] x 2


O2(g) + 4 H+(aq) + 4 e- → 2 H2O(l)
2 Mg(s) + O2(g) + 4 H+(aq) → 2 Mg2+(aq) + 2 H2O(l)
(b) [Fe(s) → Fe2+(aq) + 2 e-] x 4
[O2(g) + 4 H+(aq) + 4 e- → 2 H2O(l)] x 2
4 Fe2+(aq) + O2(g) + 4 H+(aq) → 4 Fe3+(aq) + 2 H2O(l)
[2 Fe3+(aq) + 4 H2O(l) → Fe2O3 ⋅ H2O(s) + 6 H+(aq)] x 2
4 Fe(s) + 3 O2(g) + 2 H2O(l) → 2 Fe2O3 ⋅ H2O(s)

18.17 (a)

(b) anode reaction 4 OH-(l) → O2(g) + 2 H2O(l) + 4 e-


cathode reaction 4 K+(l) + 4 e- → 4 K(l)
overall reaction 4 K+(l) + 4 OH-(l) → 4 K(l) + O2(g) + 2 H2O(l)

18.18 (a) anode reaction 2 Cl-(aq) → Cl2(g) + 2 e-


cathode reaction 2 H2O(l) + 2 e- → H2(g) + 2 OH-(aq)
overall reaction 2 Cl-(aq) + 2 H2O(l) → Cl2(g) + H2(g) + 2 OH-(aq)

(b) anode reaction 2 H2O(l) → O2(g) + 4 H+(aq) + 4 e-


cathode reaction 2 Cu2+(aq) + 4 e- → 2 Cu(s)
overall reaction 2 Cu2+(aq) + 2 H2O(l) → 2 Cu(s) + O2(g) + 4 H+(aq)

526
Chapter 18 - Electrochemistry
_____________________________________________________________________________

18.19

anode reaction Ag(s) → Ag+(aq) + e-


cathode reaction Ag+(aq) + e- → Ag(s)
The overall reaction is transfer of silver metal from the silver anode to the spoon.

 C  60 min  60 s  9
18.20 Charge = 1.00 x 105 (8.00 h)   = 2.88 x 10 C
 s  h  min 
 1 mol e _ 
Moles of e- = (2.88 x 109 C)   = 2.98 x 104 mol e-
 96,500 C 
cathode reaction: Al + 3 e → Al
3+ -

1 mol Al 26.98 g Al 1 kg
mass Al = (2.98 x 104 mol e-) x _
x x = 268 kg Al
3 mol e 1 mol Al 1000 g

1 mol Ag
18.21 3.00 g Ag x = 0.0278 mol Ag
107.9 g Ag
cathode reaction: Ag+(aq) + e- → Ag(s)
 1 mol e_  96,500 C 
Charge = (0.0278 mol Ag)   _
 = 2682.7 C
 1 mol Ag  1 mol e 
C  2682.7 C 1h 
Time = = x  = 7.45 h
A  0.100 C/s 3600 s 

18.22 When a beam of white light strikes the anodized surface, part of the light is reflected
from the outer TiO2, while part penetrates through the semitransparent TiO2 and is
reflected from the inner metal. If the two reflections of a particular wavelength are out of
phase, they interfere destructively and that wavelength is canceled from the reflected
light.
Because nλ = 2d x sin θ, the canceled wavelength depends on the thickness of the TiO2 layer.

527
Chapter 18 - Electrochemistry
_____________________________________________________________________________

 1 cm  2 3
18.23 volume =  0.0100 mm x (10.0 cm ) = 0.100 cm
 10 mm 
1 mol Al2 O3
mol Al2O3 = (0.100 cm3)(3.97 g/cm3) = 3.892 x 10-3 mol Al2O3
102.0 g Al2 O3
6 mol e _
mole e- = 3.892 x 10-3 mol Al2O3 x = 0.02335 mol e-
1 mol Al2 O3
96,500 C
coulombs = 0.02335 mol e- x = 2253 C
1 mol e _
C 2253 C 1 min
time = = x = 62.6 min
A 0.600 C/s 60 s

Understanding Key Concepts

18.24 (a) - (d)

(e) anode reaction Zn(s) → Zn2+(aq) + 2 e-


cathode reaction Pb2+(aq) + 2 e- → Pb(s)
overall reaction Zn(s) + Pb2+(aq) → Zn2+(aq) + Pb(s)

18.25 (a) anode is Ni; cathode is Pt


(b) anode reaction 3 Ni(s) → 3 Ni2+(aq) + 6 e-
cathode reaction Cr2O72-(aq) + 14 H+(aq) + 6 e- → 2 Cr3+(aq) + 7 H2O(l)
overall reaction Cr2O72-(aq) + 3 Ni(s) + 14 H+(aq) →
2 Cr3+(aq) + 3 Ni2+(aq) + 7 H2O(l)
(c) Ni(s)│Ni2+(aq) ║Cr2O72-(aq), Cr3+│Pt(s)

18.26 (a) The three cell reactions are the same except for cation concentrations.
anode reaction Cu(s) → Cu2+(aq) + 2 e- Eo = -0.34 V
cathode reaction 2 Fe (aq) + 2 e → 2 Fe (aq)
3+ - 2+
Eo = 0.77 V
overall reaction Cu(s) + 2 Fe3+(aq) → Cu2+(aq) + 2 Fe2+(aq) Eo = 0.43 V

528
Chapter 18 - Electrochemistry
_____________________________________________________________________________

(b)

0.0592 V [Cu 2+][Fe 2+ ]2


(c) E = Eo - log 2+ 2
; n = 2 mol e-
n [Fe ]
o
(1) E = E = 0.43 V because all cation concentrations are 1 M.

0.0592 V (1)(5 )2
(2) E = Eo - log = 0.39 V
2 (1)2

0.0592 V
o (0.1)(0.1 )2
(3) E = E - log = 0.46 V
2 (0.1 )2

Cell (3) has the largest potential, while cell (2) has the smallest as calculated from the
Nernst equation.

18.27 (a) - (b)

(c) anode reaction 2 Br-(aq) → Br2(aq) + 2 e-


cathode reaction Cu2+(aq) + 2 e- → Cu(s)
overall reaction Cu2+(aq) + 2 Br-(aq) → Cu(s) + Br2(aq)

529
Chapter 18 - Electrochemistry
_____________________________________________________________________________

18.28 (a) This is an electrolytic cell that has a battery connected between two inert electrodes.

(b)

(c) anode reaction 2 H2O(l) → O2(g) + 4 H+(aq) + 4 e-


cathode reaction Ni2+(aq) + 2 e- → Ni(s)
overall reaction 2 Ni2+(aq) + 2 H2O(l) → 2 Ni(s) + O2(g) + 4 H+(aq)

18.29 (a) & (b)

(c) anode reaction 2 O2- → O2(g) + 4 e-


cathode reaction TiO2(s) + 4 e- → Ti(s) + 2 O2-
overall reaction TiO2(s) → Ti(s) + O2(g)

0.0592 V [ 2+]
18.30 Zn(s) + Cu2+(aq) → Zn2+(aq) + Cu(s); E = Eo - log Zn 2+
2 [Cu ]
2+
[ ]
(a) E increases because increasing [Cu2+] decreases log Zn 2+ .
[Cu ]
(b) E will decrease because addition of H2SO4 increases the volume which decreases

530
Chapter 18 - Electrochemistry
_____________________________________________________________________________

[ 2+]
[Cu2+] and increases log Zn 2+ .
[Cu ]
[ 2 +]
(c) E decreases because increasing [Zn2+] increases log Zn 2+ .
[Cu ]
2+
(d) Because there is no change in [Zn ], there is no change in E.

0.0592 V [Cu 2+ ]
18.31 Cu(s) + 2 Ag+(aq) → Cu2+(aq) + 2 Ag(s); E = Eo - log
2 [Ag+ ]2
(a) E decreases because addition of NaCl precipitates AgCl which decreases [Ag+] and
[Cu 2+]
increases log .
[Ag+ ]2
(b) E increases because addition of NaCl increases the volume which decreases [Cu2+]
[Cu 2+]
and decreases log .
[Ag+ ]2
(c) E decreases because addition of NH3 complexes Ag+, yielding Ag(NH3)2+, which
[ 2 +]
decreases [Ag+] and increases log Cu+ 2 .
[Ag ]
(d) E increases because addition of NH3 complexes Cu2+, yielding Cu(NH3)42+, which
[ 2 +]
decreases [Cu2+] and decreases log Cu+ 2 .
[Ag ]

Additional Problems
Galvanic Cells

18.32 The electrode where oxidation takes place is called the anode. For example, the lead
electrode in the lead storage battery.
The electrode where reduction takes place is called the cathode. For example, the PbO2
electrode in the lead storage battery.

18.33 The oxidizing agent gets reduced and reduction takes place at the cathode.

18.34 The cathode of a galvanic cell is considered to be the positive electrode because
electrons flow through the external circuit toward the positive electrode (the cathode).

18.35 The salt bridge maintains charge neutrality in both the anode and cathode compartments
of a galvanic cell.

531
Chapter 18 - Electrochemistry
_____________________________________________________________________________

18.36 (a) Cd(s) + Sn2+(aq) → Cd2+(aq) + Sn(s)

(b) 2 Al(s) + 3 Cd2+(aq) → 2 Al3+(aq) + 3 Cd(s)

(c) 6 Fe2+(aq) + Cr2O72-(aq) + 14 H+(aq) → 6 Fe3+(aq) + 2 Cr3+(aq) + 7 H2O(l)

532
Chapter 18 - Electrochemistry
_____________________________________________________________________________

18.37 (a) 3 Cu2+(aq) + 2 Cr(s) → 3 Cu(s) + 2 Cr3+(aq)

(b) Pb(s) + 2 H+(aq) → Pb2+(aq) + H2(g)

(c) Cl2(g) + Sn2+(aq) → Sn4+(aq) + 2 Cl-(aq)

533
Chapter 18 - Electrochemistry
_____________________________________________________________________________

18.38 (a) Cd(s)│Cd2+(aq)║Sn2+(aq)│Sn(s)


(b) Al(s)│Al3+(aq)║Cd2+(aq)│Cd(s)
(c) Pt(s)│Fe2+(aq), Fe3+(aq)║Cr2O72-(aq), Cr3+(aq)│Pt(s)

18.39 (a) Cr(s)│Cr3+(aq)║Cu2+(aq)│Cu(s)


(b) Pb(s)│Pb2+(aq)║H+(aq)│H2(g)│Pt(s)
(c) Pt(s)│Sn2+(aq), Sn4+(aq)║Cl2(g)│Cl-(aq)│Pt(s)

18.40 (a)

(b) anode reaction H2(g) → 2 H+(aq) + 2 e-


cathode reaction 2 Ag+(aq) + 2 e- → 2 Ag(s)
overall reaction H2(g) + 2 Ag+(aq) → 2 H+(aq) + 2 Ag(s)

(c) Pt(s)│H2(g)│H+(aq)║Ag+(aq)│Ag(s)

534
Chapter 18 - Electrochemistry
_____________________________________________________________________________

18.41 (a)

(b) anode reaction Zn(s) → Zn2+(aq) + 2 e-


cathode reaction Cl2(g) + 2 e- → 2 Cl-(aq)
overall reaction Zn(s) + Cl2(g) → Zn2+(aq) + 2 Cl-(aq)
(c) Zn(s)│Zn2+(aq)║Cl2(g)│Cl-(aq)│C(s)

18.42 (a) anode reaction Co(s) → Co2+(aq) + 2 e-


cathode reaction Cu2+(aq) + 2 e- → Cu(s)
overall reaction Co(s) + Cu2+(aq) → Co2+(aq) + Cu(s)

(b) anode reaction 2 Fe(s) → 2 Fe2+(aq) + 4 e-


cathode reaction O2(g) + 4 H+(aq) + 4 e- → 2 H2O(l)
overall reaction 2 Fe(s) + O2(g) + 4 H+(aq) → 2 Fe2+(aq) + 2 H2O(l)

535
Chapter 18 - Electrochemistry
_____________________________________________________________________________

18.43 (a) anode reaction Mn(s) → Mn2+(aq) + 2 e-


cathode reaction Pb2+(aq) + 2 e- → Pb(s)
overall reaction Mn(s) + Pb2+(aq) → Mn2+(aq) + Pb(s)

(b) anode reaction H2(g) → 2 H+(aq) + 2 e-


cathode reaction 2 AgCl(s) + 2 e- → 2 Ag(s) + 2 Cl-(aq)
overall reaction H2(g) + 2 AgCl(s) → 2 Ag(s) + 2 H+(aq) + 2 Cl-(aq)

536
Chapter 18 - Electrochemistry
_____________________________________________________________________________

Cell Potentials and Free-Energy Changes; Standard Reduction potentials

18.44 The SI unit of electrical potential is the volt (V).


The SI unit of charge is the coulomb (C).
The SI unit of energy is the joule (J).
1J=1C⋅1V

18.45 ∆G = -nFE; ∆G is the free energy change for the cell reaction
n is the number of moles of e-
F is the Faraday (96,500 C/mol e-)
E is the galvanic cell potential

18.46 E is the standard cell potential (Eo) when all reactants and products are in their standard
states--solutes at 1 M concentrations, gases at a partial pressure of 1 atm, solids and
liquids in pure form, all at 25oC.

18.47 The standard reduction potential is the potential of the reduction half reaction in a
galvanic cell where the other electrode is the standard hydrogen electrode.

18.48 Zn(s) + Ag2O(s) → ZnO(s) + 2 Ag(s); n = 2 mol e-


 96,500 C   1J 
∆G = -nFE = -(2 mol e-)   (1.60 V)   = -308,800 J = -309 kJ
1 C • V 
_
 1 mol e 

18.49 Pb(s) + PbO2(s) + 2 H+(aq) + 2 HSO4-(aq) → 2 PbSO4(s) + 2 H2O(l)

537
Chapter 18 - Electrochemistry
_____________________________________________________________________________

n = 2 mol e-
 96,500 C   1J 
∆Go = -nFEo = -(2 mol e-)  (1.924 V)   = -371,300 J = -371 kJ
1 C • V 
_
 1 mol e 

18.50 2 H2(g) + O2(g) → 2 H2O(l); n = 4 mol e- and 1 V = 1 J/C


∆Go = 2 ∆Gof(H2O(l)) = (2 mol)(-237.2 kJ/mol) = -474.4 kJ
∆Go = -nFEo
_ ∆G o _ (_ 474,400 J)
Eo = = = +1.23 J/C = +1.23 V
nF _  96,500 C 
(4 mol e ) _

 1 mol e 

18.51 CH4(g) + 2 O2(g) → CO2(g) + 2 H2O(l); n = 8 mol e- and 1 V = 1 J/C


o o o o
∆G = [∆G f(CO2) + 2 ∆G f(H2O(l))] - ∆G f(CH4)
∆Go = [(1 mol)(-394.4 kJ/mol) + (2 mol)(- 237.2 kJ/mol)]
- (1 mol)(-50.8 kJ/mol) = -818.0 kJ
_ ∆G o _(_ 818,000 J)
∆Go = -nFEo Eo = = = +1.06 J/C = +1.06 V
nF _  96,500 C 
(8 mol e ) _

 1 mol e 

18.52 oxidation: Zn(s) → Zn2+(aq) + 2 e- Eo = 0.76 V


reduction: Eu3+(aq) + e- → Eu2+(aq) Eo = ?
overall Zn(s) + 2 Eu3+(aq) → Zn2+(aq) + 2 Eu2+(aq) Eo = 0.40 V
The standard reduction potential for the Eu3+/Eu2+ half cell is:
Eo = 0.40 - 0.76 = -0.36 V

18.53 oxidation: 2 Ag(s) + 2 Br-(aq) → 2 AgBr(s) + 2 e- Eo = ?


reduction: Cu2+(aq) + 2 e- → Cu(s) Eo = 0.34 V
overall Cu2+(aq) + 2 Ag(s) + 2 Br-(aq) → Cu(s) + 2 AgBr(s) Eo = 0.27 V
o
E for the oxidation half reaction = 0.27 - 0.34 = -0.07 V
For AgBr(s) + e- → Ag(s) + Br-(aq), Eo = -(-0.07 V) = +0.07 V

18.54 Sn4+(aq) < Br2(l) < MnO4-

18.55 Pb(s) < Fe(s) < Al(s)

18.56 Cr2O72-(aq) is highest in the table of standard reduction potentials, therefore it is the
strongest oxidizing agent.
Fe2+(aq) is lowest in the table of standard reduction potentials, therefore it is the weakest
oxidizing agent.

18.57 From Table 18.1:


Sn2+ is the strongest reducing agent and Fe2+ is the weakest reducing agent.

538
Chapter 18 - Electrochemistry
_____________________________________________________________________________

18.58 (a) Cd(s) + Sn2+(aq) → Cd2+(aq) + Sn(s)


oxidation: Cd(s) → Cd2+(aq) + 2 e- Eo = 0.40 V
reduction: Sn2+(aq) + 2 e- → Sn(s) Eo = -0.14 V
overall Eo = 0.26 V

n = 2 mol e-
 96,500 C   1J 
∆Go = -nFEo = -(2 mol e-)   (0.26 V)   = -50,180 J = -50 kJ
1 C • V 
_
 1 mol e 
(b) 2 Al(s) + 3 Cd2+(aq) → 2 Al3+(aq) + 3 Cd(s)
oxidation: 2 Al(s) → 2 Al3+(aq) + 6 e- Eo = 1.66 V
reduction: 3 Cd2+(aq) + 6 e- → 3 Cd(s) Eo = -0.40 V
overall Eo = 1.26 V
n = 6 mol e-
 96,500 C   1J 
∆Go = -nFEo = -(6 mol e-)   (1.26 V)   = -729,540 J = -730 kJ
1 C • V 
_
 1 mol e 
(c) 6 Fe2+(aq) + Cr2O72-(aq) + 14 H+(aq) → 6 Fe3+(aq) + 2 Cr3+(aq) + 7 H2O(l)
oxidation: 6 Fe2+(aq) → 6 Fe3+(aq) + 6 e- Eo = -0.77 V
reduction: Cr2O7 (aq) + 14 H (aq) + 6 e + → 2 Cr (aq) + 7 H2O(l) Eo = 1.33 V
2- + - 3+

overall Eo = 0.56 V
-
n = 6 mol e
 96,500 C   1J 
∆Go = -nFEo = -(6 mol e-)   (0.56 V)   = -324,240 J = -324 kJ
1 C • V 
_
 1 mol e 

18.59 (a) 3 Cu2+(aq) + 2 Cr(s) → 3 Cu(s) + 2 Cr3+(aq)


oxidation 2 Cr(s) → 2 Cr3+(aq) + 6 e- Eo = 0.74 V
reduction 3 Cu2+(aq) + 6 e- → 3 Cu(s) Eo = 0.34 V
overall Eo = 1.08 V
n = 6 mol e-
 96,500 C   1J 
∆Go = -nFEo = -(6 mol e-)   (1.08 V)   = -625,320 J = -625 kJ
1 C • V 
_
 1 mol e 
(b) Pb(s) + 2 H+(aq) → Pb2+(aq) + H2(g)
oxidation: Pb(s) → Pb2+(aq) + 2 e- Eo = 0.13 V
reduction: 2 H (aq) + 2 e → H2(g)
+ -
Eo = 0.00 V
overall Eo = 0.13 V
-
n = 2 mol e
 96,500 C   1J 
∆Go = -nFEo = -(2 mol e-)   (0.13 V)   = -25,090 J = -25 kJ
1 C • V 
_
 1 mol e 
(c) Cl2(g) + Sn2+(aq) → Sn4+(aq) + 2 Cl-(aq)
oxidation Sn2+(aq) → Sn4+(aq) + 2 e- Eo = -0.15 V
reduction Cl2(g) + 2 e- → 2 Cl-(aq) Eo = 1.36 V
overall Eo = 1.21 V

539
Chapter 18 - Electrochemistry
_____________________________________________________________________________

n = 2 mol e-
 96,500 C   1J 
∆Go = -nFEo = -(2 mol e-)   (1.21 V)   = -233,530 J = -234 kJ
1 C • V 
_
 1 mol e 

18.60 (a) 2 Fe2+(aq) + Pb2+(aq) → 2 Fe3+(aq) + Pb(s)


oxidation: 2 Fe2+(aq) → 2 Fe3+(aq) + 2 e- Eo = -0.77 V
reduction: Pb2+(aq) + 2 e- → Pb(s) Eo = -0.13 V
overall Eo = -0.90 V
Because Eo is negative, this reaction is nonspontaneous.
(b) Mg(s) + Ni2+(aq) → Mg2+(aq) + Ni(s)
oxidation: Mg(s) → Mg2+(aq) + 2 e- Eo = 2.37 V
reduction: Ni2+(aq) + 2 e- → Ni(s) Eo = -0.26 V
overall Eo = 2.11 V
Because Eo is positive, this reaction is spontaneous.

18.61 (a) 5 Ag+(aq) + Mn2+(aq) + 4 H2O(l) → 5 Ag(s) + MnO4-(aq) + 8 H+(aq)


oxidation: Mn2+(aq) + 4 H2O(l) → MnO4-(aq) + 8 H+(aq) + 5 e- Eo = -1.51 V
reduction: 5 Ag+(aq) + 5 e- → 5 Ag(s) Eo = 0.80 V
overall Eo = -0.71 V
Because Eo is negative, this reaction is nonspontaneous.
(b) 2 H2O2(aq) → O2(g) + 2 H2O(l)
oxidation: H2O2(aq) → O2(g) + 2 H+(aq) + 2 e- Eo = -0.70 V
reduction: H2O2(aq) + 2 H+(aq) + 2 e- → 2 H2O(l) Eo = 1.78 V
overall Eo = 1.08 V
Because Eo is positive, this reaction is spontaneous.

18.62 (a) oxidation: Sn2+(aq) → Sn4+(aq) + 2 e- Eo = -0.15 V


reduction: Br2(l) + 2 e → 2 Br (aq)
- -
Eo = 1.09 V
overall Eo = +0.94 V
o 2+
Because the overall E is positive, Sn (aq) can be oxidized by Br2(l).
(b) oxidation: Sn2+(aq) → Sn4+(aq) + 2 e- Eo = -0.15 V
reduction: Ni (aq) + 2 e → Ni(s)
2+ -
Eo = -0.26 V
overall Eo = -0.41 V
Because the overall E is negative, Ni (aq) cannot be reduced by Sn2+(aq).
o 2+

(c) oxidation: 2 Ag(s) → 2 Ag+(aq) + 2 e- Eo = -0.80 V


reduction: Pb (aq) + 2 e → Pb(s)
2+ -
Eo = -0.13 V
overall Eo = -0.93 V
Because the overall E is negative, Ag(s) cannot be oxidized by Pb2+(aq).
o

(d) oxidation: H2SO3(aq) + H2O(l) → SO42-(aq) + 4 H+(aq) + 2 e- Eo = -0.17 V


reduction: I2(s) + 2 e- → 2 I-(aq) Eo = 0.54 V
overall Eo = +0.37 V
o
Because the overall E positive, I2(s) can be reduced by H2SO3.

540
Chapter 18 - Electrochemistry
_____________________________________________________________________________

18.63 (a) oxidation: Zn(s) → Zn2+(aq) + 2 e- Eo = 0.76 V


reduction: Pb2+(aq) + 2 e- → Pb(s) Eo = -0.13 V
overall Eo = 0.63 V
Zn(s) + Pb (aq) → Zn (aq) + Pb(s)
2+ 2+

The reaction is spontaneous because Eo is positive.

(b) oxidation: 4 Fe2+(aq) → 4 Fe3+(aq) + 4 e- Eo = -0.77 V


reduction: O2(g) + 4 H+(aq) + 4 e- → 2 H2O(l) Eo = 1.23 V
overall Eo = 0.46 V
4 Fe2+(aq) + O2(g) + 4 H+(aq) → 4 Fe3+(aq) + 2 H2O(l)
The reaction is spontaneous because Eo is positive.
(c) oxidation: 2 Ag(s) → 2 Ag+(aq) + 2 e- Eo = -0.80 V
reduction: Ni2+(aq) + 2 e- → Ni(s) Eo = -0.26 V
overall Eo = -1.06 V
o
There is no reaction because E is negative.
(d) oxidation: H2(g) → 2 H+(aq) + 2 e- Eo = 0.00 V
reduction: Cd2+(aq) + 2 e- → Cd(s) Eo = -0.40 V
overall Eo = -0.40 V
There is no reaction because Eo is negative.

The Nernst Equation

18.64 2 Ag+(aq) + Sn(s) → 2 Ag(s) + Sn2+(aq)


oxidation: Sn(s) → Sn2+(aq) + 2 e- Eo = 0.14 V
reduction: 2 Ag+(aq) + 2 e- → 2 Ag(s) Eo = 0.80 V
overall Eo = 0.94 V
0.0592 V [Sn 2+] (0.0592 V) (0.020)
E = Eo - log + 2
= 0.94 V _ log = 0.87 V
n [Ag ] 2 (0.010 )2

18.65 2 Fe2+(aq) + Cl2(g) → 2 Fe3+(aq) + 2 Cl-(aq)


oxidation: 2 Fe2+(aq) → 2 Fe3+(aq) + 2 e- Eo = -0.77 V
reduction: Cl2(g) + 2 e- → 2 Cl-(aq) Eo = 1.36 V
overall Eo = 0.59 V
2 2
o 0.0592 V [Fe3+ ] [Cl _ ] (0.0592 V) (0.0010 )2 (0.0030 )2
E=E - log = 0.59 V _ log = 0.91 V
n [Fe 2+ ]2 PCl2 2 2
(1.0 ) (0.50)

18.66 Pb(s) + Cu2+(aq) → Pb2+(aq) + Cu(s)


oxidation: Pb(s) → Pb2+(aq) + 2 e- Eo = 0.13 V
reduction: Cu2+(aq) + 2 e- → Cu(s) Eo = 0.34 V
overall Eo = 0.47 V
0.0592 V [Pb2+ ] (0.0592 V) 1.0
E = Eo - log 2+
= 0.47 V _ log = 0.35 V
n [Cu ] 2 (1.0 x 10- 4)

541
Chapter 18 - Electrochemistry
_____________________________________________________________________________

0.0592 V [Pb 2+ ] (0.0592 V) 1.0


When E = 0, 0 = Eo - log 2+
= 0.47 V _ log
n [Cu ] 2 [Cu 2+ ]
(0.0592 V)
0 = 0.47 V + log [Cu2+]
2
 2 
log [Cu2+] = (-0.47 V)   = -15.88; [Cu2+] = 10-15.88 = 1 x 10-16 M
 0.0592 V 

18.67 Fe(s) + Cu2+(aq) → Fe2+(aq) + Cu(s)


oxidation: Fe(s) → Fe2+(aq) + 2 e- Eo = 0.45 V
reduction: Cu2+(aq) + 2 e- → Cu(s) Eo = 0.34 V
overall Eo = 0.79 V
0.0592 V [Fe2+] (0.0592 V)  0.10 
E = 0.67 V = Eo - log 2+
= 0.79 V _ log  
2+ 
n [Cu ] 2  [Cu ] 
(0.0592 V)
0.67 V = 0.79 V _ (log (0.10) _ log [Cu 2+])
2
log [Cu2+] = -5.05; [Cu2+] = 10-5.05 = 8.9 x 10-6 M

0.0592 V (0.0592 V)
18.68 (a) E = Eo - log [I_ ]2 = 0.54 V _ log (0.020 )2 = 0.64 V
n 2
0.0592 V [Fe2+] (0.0592 V)  0.10 
(b) E = Eo - log = 0.77 V _ log   = 0.77 V
 0.10 
3+
n [Fe ] 1
0.0592 V [Sn 4+ ] (0.0592 V)  0.40 
(c) E = Eo - log = _ 0.15 V _ log   = -0.23 V
 0.0010 
2+
n [Sn ] 2
(d) E = Eo -
0.0592 V [Cr 2 O72 _ ][H + ]14 (0.0592 V)  (1.0)(0.010 )14 
log = _ 1.33 V _ log  
n [Cr 3+ ]2 6  1.0 
 
(0.0592 V)
E = _ 1.33 V _ (14) log (0.010) = -1.05 V
6

0.0592 V P H2
18.69 E = Eo - log ; Eo = 0, n = 2 mol e-, and PH2 = 1 atm
n [H3 O+ ]2
0.0592 V 1
(a) [H3O+] = 1.0 M; E = - log =0
2 (1.0 )2
(b) pH = 4.00, [H3O+] = 10-4.00 = 1.0 x 10-4 M
0.0592 V 1
E=- log 2
= -0.24 V
2 (1.0 x 10_ 4 )
0.0592 V 1
(c) [H3O+] = 1.0 x 10-7 M; E = - log = -0.41 V
2 (1.0 x 10_ 7 )2

542
Chapter 18 - Electrochemistry
_____________________________________________________________________________
_ 14
(d) [OH-] = 1.0 M; [H3O+] = K w = 1.0 x 10 = 1.0 x 10-14 M
[OH _ ] 1.0
0.0592 V 1
E=- log 2
= -0.83 V
2 (1.0 x 10_14 )

18.70 H2(g) + Ni2+(aq) → 2 H+(aq) + Ni(s)


Eo = EoH2 _ H+ + EoNi2+ _ Ni = 0 V + (-0.26 V) = -0.26 V
0.0592 V
o [H3 O+ ]2
E=E - log
n [ Ni2+ ](P H2)
(0.0592 V) [H3 O+ ]2
0.27 V = -0.26 V - log
2 (1)(1)
0.27 V = -0.26 V - (0.0592 V) log [H3O+]

pH = - log [H3O+] therefore 0.27 V = -0.26 V + (0.0592 V) pH


(0.27 V + 0.26 V)
pH = = 9.0
0.0592 V

18.71 Zn(s) + 2 H+(aq) → Zn2+(aq) + H2(g)


Eo = EoH+ _ H2 + EoZn _ Zn2+ = 0 V + 0.76 V = 0.76 V
0.0592 V [ Zn 2+](PH2)
E = Eo - log
n [H3 O+ ]2
(0.0592 V) (1)(1)
0.58 V = 0.76 V - log
2 [H3 O+ ]2
0.58 V = 0.76 V + (0.0592 V) log [H3O+]
pH = - log [H3O+] therefore 0.58 V = 0.76 V - (0.0592 V) pH
_(0.58 V _ 0.76 V)
pH = = 3.0
0.0592 V

Standard Cell Potentials and Equilibrium Constants

18.72 ∆Go = -nFEo


Because n and F are always positive, ∆Go is negative when Eo is positive because of the
negative sign in the equation.
0.0592 V nE o nE o
Eo = log K; log K = ; K = 10 0.0592
n 0.0592 V
If Eo is positive, the exponent is positive (because n is positive), and K is greater than 1.

18.73 If K < 1, Eo < 0. When Eo = 0, K = 1.

18.74 Ni(s) + 2 Ag+(aq) → Ni2+(aq) + 2 Ag(s)


oxidation: Ni(s) → Ni2+(aq) + 2 e- Eo = 0.26 V

543
Chapter 18 - Electrochemistry
_____________________________________________________________________________

reduction: 2 Ag+(aq) + 2 e- → 2 Ag(s) Eo = 0.80 V


overall Eo = 1.06 V
0.0592 V nE o (2)(1.06 V)
Eo = log K; log K = = = 35.8; K = 1035.8 = 6 x 1035
n 0.0592 V 0.0592 V

18.75 2 MnO4-(aq) + 10 Cl-(aq) + 16 H+(aq) → 2 Mn2+(aq) + 5 Cl2(g) + 8 H2O(l)


oxidation: 10 Cl-(aq) → 5 Cl2(g) + 10 e- Eo = -1.36 V
reduction: 2 MnO4-(aq) + 16 H+(aq) + 10 e- → 2 Mn2+(aq) + 8 H2O(l) Eo = 1.51 V
overall Eo = 0.15 V
0.0592 V nE o (10)(0.15 V)
Eo = log K ; log K = = = 25.3; K = 1025.3 = 2 x
n 0.0592 V 0.0592 V
1025

18.76 Eo and n are from Problem 18.58.


0.0592 V nE o
Eo = log K; log K =
n 0.0592 V
(a) Cd(s) + Sn (aq) → Cd (aq) + Sn(s);
2+ 2+
Eo = 0.26 V and n = 2 mol e-
(2)(0.26 V)
log K = = 8.8; K = 108.8 = 6 x 108
0.0592 V
(b) 2 Al(s) + 3 Cd2+(aq) → 2 Al3+(aq) + 3 Cd(s); Eo = 1.26 V and n = 6 mol e-
(6)(1.26 V)
log K = = 128; K = 10128
0.0592 V
(c) 6 Fe2+(aq) + Cr2O72-(aq) + 14 H+(aq) → 6 Fe3+(aq) + 2 Cr3+(aq) + 7 H2O(l)
Eo = 0.56 V and n = 6 mol e-
(6)(0.56 V)
log K = = 57; K = 1057
0.0592 V

18.77 Eo and n are from Problem 18.59.


0.0592 V nE o
Eo = log K; log K =
n 0.0592 V
(a) 3 Cu2+(aq) + 2 Cr(s) → 3 Cu(s) + 2 Cr3+(aq); Eo = 1.08 V and n = 6 mol e-
(6)(1.08 V)
log K = = 109; K = 10109
0.0592 V
(b) Pb(s) + 2 H+(aq) → Pb2+(aq) + H2(g); Eo = 0.13 V and n = 2 mol e-
(2)(0.13 V)
log K = = 4.4; K = 104.4 = 3 x 104
0.0592 V
(c) Cl2(g) + Sn2+(aq) → Sn4+(aq) + 2 Cl-(aq); Eo = 1.21 V and n = 2 mol e-
(2)(1.21 V)
log K = = 40.9; K = 1040.9 = 8 x 1040
0.0592 V

18.78 Hg22+(aq) → Hg(l) + Hg2+(aq)


oxidation: ½[Hg22+(aq) → 2 Hg2+(aq) + 2 e-] Eo = -0.92 V

544
Chapter 18 - Electrochemistry
_____________________________________________________________________________

reduction: ½[Hg22+(aq) + 2 e- → 2 Hg(l)] Eo = 0.80 V


overall Eo = -0.12 V
0.0592 V
Eo = log K
n
nE o (1)(_ 0.12 V)
log K = = = -2.027; K = 10-2.027 = 9 x 10-3
0.0592 V 0.0592 V
18.79 2 H2O2(aq) → 2 H2O(l) + O2(g)
oxidation: H2O2(aq) → O2(g) + 2 H+(aq) + 2e- Eo = -0.70 V
reduction: H2O2(aq) + 2 H (aq) + 2e → 2 H2O(l)
+ -
Eo = 1.78 V
overal Eo = 1.08 V
0.0592 V nE o (2)(1.08 V)
Eo = log K ; log K = = = 36.5; K = 1036.5 = 3 x
n 0.0592 V 0.0592 V
1036

Batteries; Corrosion

18.80 Rust is a hydrated form of iron(III) oxide (Fe2O3 ⋅ H2O). Rust forms from the oxidation
of Fe in the presence of O2 and H2O. Rust can be prevented by coating Fe with Zn
(galvanizing).

18.81 Cr forms a protective oxide coating similar to Al.

18.82 Cathodic protection is the attachment of a more easily oxidized metal to the metal you
want to protect. This forces the metal you want to protect to be the cathode, hence the
name, cathodic protection.
Zn and Al can offer cathodic protection to Fe (Ni and Sn cannot).

18.83 A sacrificial anode is a metal used for cathodic protection. It behaves as an anode and is
more easily oxidized than the metal it is protecting.
An example of a sacrificial anode is Zn for protecting Fe (galvanizing).

18.84 (a)

(b) Anode: Pb(s) + HSO4-(aq) → PbSO4(s) + H+(aq) + 2 e- Eo = 0.296 V

545
Chapter 18 - Electrochemistry
_____________________________________________________________________________

Cathode: PbO2(s) + 3 H+(aq) + HSO4-(aq) + 2 e- → PbSO4(s) + 2 H2O(l) Eo = 1.628 V


Overall Pb(s) + PbO2(s) + 2 H+(aq) + 2 HSO4-(aq) → 2 PbSO4(s) + 2 H2O(l) Eo = 1.924 V
0.0592 V nE o (2)(1.924 V)
(c) Eo = log K; log K = = = 65.0; K = 1 x 1065
n 0.0592 V 0.0592 V
(d) When the cell reaction reaches equilibrium the cell voltage = 0.

18.85 oxidation: 2 H2(g) + 4 OH-(aq) → 4 H2O(l) + 4 e- Eo = 0.83 V


reduction: O2(g) + 2 H2O(l) + 4 e- → 4 OH-(aq) Eo = 0.43 V
2 H2(g) + O2(g) → 2 H2O(l) Eo = 1.23 V
n = 4 mol e- and 1 J = 1 C x 1 V
 96,500 C   1J 
∆Go = -nFEo = -(4 mol e-)   (1.23 V)   = - 474,780 J = - 475 kJ
1 C • V 
_
 1 mol e 
0.0592 V nE o (4)(1.23 V)
Eo = log K ; log K = = = 83.1; K = 1083.1 = 1 x
n 0.0592 V 0.0592 V
1083
0.0592 V 1 0.0592 V 1
E = Eo - log 2
= 1.23 V - log 2
= 1.29 V
n ( P H2 ) ( P O2 ) 4 (25 ) (25)

18.86 Zn(s) + HgO(s) → ZnO(s) + Hg(l); Zn, 65.39 amu; HgO, 216.59 amu
1 mol Zn 1 mol HgO 216.59 g HgO
mass HgO = 2.00 g Zn x x x = 6.62 g HgO
65.39 g Zn 1 mol Zn 1 mol HgO

18.87 Cd(OH)2(s) + 2 Ni(OH)2(s) → Cd(s) + 2 NiO(OH)(s) + 2 H2O(l)


Ni(OH)2, 92.71 amu; Cd, 112.41 amu
1 mol Ni(OH )2 1 mol Cd 112.41 g Cd
mass Cd = 10.0 g Ni(OH)2 x x x = 6.06 g
92.71 g Ni(OH )2 2 mol Ni(OH )2 1 mol Cd
Cd

Electrolysis

546
Chapter 18 - Electrochemistry
_____________________________________________________________________________

18.88 (a)

(b) anode: 2 Cl-(l) → Cl2(g) + 2 e-


cathode: Mg2+(l) + 2 e- → Mg(l)
overall: Mg2+(l) + 2 Cl-(l) → Mg(l) + Cl2(g)

18.89 (a)

(b) anode: 2 H2O(l) → O2(g) + 4 H+(aq) + 4 e-


cathode: 4 H+(aq) + 4 e- → 2 H2(g)
overall: 2 H2O(l) → O2(g) + 2 H2(g)

18.90 possible anode reactions:


2 Cl-(aq) → Cl2(g) + 2 e-
2 H2O(l) → O2(g) + 4 H+(aq) + 4 e-
possible cathode reactions:
2 H2O(l) + 2 e- → H2(g) + 2 OH-(aq)
Mg2+(aq) + 2 e- → Mg(s)
actual reactions:
anode: 2 Cl-(aq) → Cl2(g) + 2 e-
cathode: 2 H2O(l) + 2 e- → H2(g) + 2 OH-(aq)
This anode reaction takes place instead of 2 H2O(l) → O2(g) + 4 H+(aq) + 4 e- because of
a high overvoltage for formation of gaseous O2.
This cathode reaction takes place instead of Mg2+(aq) + 2 e- → Mg(s) because H2O is
easier to reduce than Mg2+.

547
Chapter 18 - Electrochemistry
_____________________________________________________________________________

18.91 (a) K(l) and Cl2(g) (b) H2(g) and Cl2(g). Solvent H2O is reduced in preference to K+.

18.92 (a) NaBr


anode: 2 Br-(aq) → Br2(l) + 2 e-
cathode: 2 H2O(l) + 2 e- → H2(g) + 2 OH-(aq)
overall: 2 H2O(l) + 2 Br-(aq) → Br2(l) + H2(g) + 2 OH-(aq)
(b) CuCl2
anode: 2 Cl-(aq) → Cl2(g) + 2 e-
cathode: Cu2+(aq) + 2 e- → Cu(s)
overall: Cu2+(aq) + 2 Cl-(aq) → Cu(s) + Cl2(g)
(c) LiOH
anode: 4 OH-(aq) → O2(g) + 2 H2O(l) + 4 e-
cathode: 4 H2O(l) + 4 e- → 2 H2(g) + 4 OH-(aq)
overall: 2 H2O(l) → O2(g) + 2 H2(g)

18.93 (a) Ag2SO4


anode: 2 H2O(l) → O2(g) + 4 H+(aq) + 4 e-
cathode: 4 Ag+(aq) + 4 e- → 4 Ag(s)
overall: 4 Ag+(aq) + 2 H2O(l) → O2(g) + 4 H+(aq) + 4 Ag(s)
(b) Ca(OH)2
anode: 4 OH-(aq) → O2(g) + 2 H2O(l) + 4 e-
cathode: 4 H2O(l) + 4 e- → 2 H2(g) + 4 OH-(aq)
overall: 2 H2O(l) → O2(g) + 2 H2(g)
(c) KI
anode: 2 I-(aq) → I2(s) + 2 e-
cathode: 2 H2O(l) + 2 e- → H2(g) + 2 OH-(aq)
overall: 2 I-(aq) + 2 H2O(l) → I2(s) + H2(g) + 2 OH-(aq)

18.94 Ag+(aq) + e- → Ag(s); 1 A = 1 C/s


C 60 s 1 mol e_ 1 mol Ag 107.87 g Ag
mass Ag = 2.40 x 20.0 min x x x _
x =
s 1 min 96,500 C 1 mol e 1 mol Ag
3.22 g

18.95 Cu2+(aq) + 2 e- → Cu(s)


C 60 min 60 s 1 mol e _
mol e- = 100.0 x 24.0 h x x x = 89.5 mol e-
s h min 96,500 C
1 mol Cu 63.54 g Cu 1 kg
mass Cu = 89.5 mol e- x _
x x = 2.84 kg Cu
2 mol e 1 mol Cu 1000 g

18.96 2 Na+(l) + 2 Cl-(l) → 2 Na(l) + Cl2(g)


Na+(l) + e- → Na(l); 1 A = 1 C/s; 1.00 x 103 kg = 1.00 x 106 g

548
Chapter 18 - Electrochemistry
_____________________________________________________________________________

1 mol Na 1 mol e _ 96,500 C


Charge = 1.00 x 106 g Na x x x _
= 4.20 x 109 C
22.99 g Na 1 mol Na 1 mol e
9
4.20 x 10 C 1h
Time = x = 38.9 h
30,000 C/s 3600 s
1 mol Na 1 mol Cl2
1.00 x 106 g Na x x = 21,748.6 mol Cl2
22.99 g Na 2 mol Na
PV = nRT
 L • atm 
(21,748.6 mol) 0.082 06 (273.15 K)
nRT  K • mol 
V= = = 4.87 x 105 L Cl2
P 1.00 atm

18.97 Al3+ + 3 e- → Al; 40.0 kg = 40,000 g; 1 h = 3600 s


1 mol Al 3 mol e_ 96,500 C
Charge = 40,000 g Al x x x = 4.29 x 108 C
26.98 g Al 1 mol Al 1 mol e _
4.29 x 108 C
Current = = 1.19 x 105 A
3600 s
18.98 PbSO4(s) + H+(aq) + 2 e- → Pb(s) + HSO4-(aq)
mass PbSO4
C 3600 s 1 mol e _ 1 mol PbSO 4 303.3 g PbSO 4
= 10.0 x 1.50 h x x x _
x
s 1h 96,500 C 2 mol e 1 mol PbSO 4
mass PbSO4 = 84.9 g PbSO4

18.99 Cr3+(aq) + 3 e- → Cr(s)


1 mol Cr 3 mol e _ 96,500 C
Charge = 125 g Cr x x x = 6.96 x 105 C
52.00 g Cr 1 mol Cr 1 mol e_
6.96 x 105 C 1 min
Time = x = 58.0 min
200.0 C/s 60 s

General Problems

18.100 (a) 2 MnO4-(aq) + 16 H+(aq) + 5 Sn2+(aq) → 2 Mn2+(aq) + 5 Sn4+(aq) + 8 H2O(l)


(b) MnO4- is the oxidizing agent; Sn2+ is the reducing agent.
(c) Eo = 1.51 V + (-0.15 V) = 1.36 V

18.101 2 Mn3+(aq) + 2 H2O(l) → Mn2+(aq) + MnO2(s) + 4 H+(aq)


Eo = 1.51 V + (-0.95 V) = +0.56 V
Because Eo is positive, the disproportionation is spontaneous under standard-state
conditions.

18.102 (a) Ag+ is the strongest oxidizing agent because Ag+ has the most positive standard
reduction potential.

549
Chapter 18 - Electrochemistry
_____________________________________________________________________________

Pb is the strongest reducing agent because Pb2+ has the most negative standard reduction
potential.

(b)

(c) Pb(s) + 2 Ag+(aq) → Pb2+(aq) + 2 Ag(s); n = 2 mol e-


o o o
E = E ox + E red = 0.13 V + 0.80 V = 0.93 V
 96,500 C   1J 
∆Go = -nFEo = -(2 mol e-)   (0.93 V)   = -179,490 J = -180 kJ
1 C • V 
_
 1 mol e 
0.0592 V nE o (2)(0.93 V)
Eo = log K ; log K = = = 31; K = 1031
n 0.0592 V 0.0592 V
0.0592 V [Pb2+ ] 0.0592 V  0.01 
(d) E = Eo - log + 2
= 0.93 V _ log   = 0.87 V
2
n [Ag ] 2  (0.01 ) 

0.0592 V 1
18.103 For Pb2+, E = -0.13 - log
2 [Pb 2+ ]
0.0592 V 1
For Cd2+, E = -0.40 - log
2 [Cd 2+]
Set these two equations for E equal to each other and solve for [Cd2+]/[Pb2+].

0.0592 V 1 0.0592 V 1
-0.13 - log 2+
= -0.40 - log
2 [Pb ] 2 [Cd 2+ ]

550
Chapter 18 - Electrochemistry
_____________________________________________________________________________

0.0592 V 0.0592 V [Cd 2+ ]


0.27 = (log[Cd2+] - log[Pb2+]) = log
2 2 [Pb 2+]
[ 2+ ] (0.27)(2) [ 2 +]
log Cd2+ = = 9.1; Cd2+ = 109.1 = 1 x 109
[Pb ] 0.0592 [Pb ]

18.104 (a)
(b) 2 Al(s) + 6 H+(aq) → 2 Al3+(aq) + 3 H2(g)
Eo = Eoox + Eored = 1.66 V + 0.00 V = 1.66 V
(c)
0.0592 V [Al3+ ]2 (PH2 )
3
(0.0592 V)  (0.10 )2 (10.0 )3 
E = Eo - log = 1.66 V _ log   = 1.59 V
n [H+ ]6 6  (0.10 )
6 
 
 96,500 C   1J 
(d) ∆Go = -nFEo = -(6 mol e-)   (1.66 V)   = -961,140 J = -961 kJ
1 C • V 
_
 1 mol e 
0.0592 V nE o (6)(1.66 V)
Eo = log K ; log K = = = 168; K = 10168
n 0.0592 V 0.0592 V
C 60 s 1 mol e_ 1 mol Al 26.98 g Al
(e) mass Al = 10.0 x 25.0 min x x x _
x =
s 1 min 96,500 C 3 mol e 1 mol Al
1.40 g
18.105 Zn(s) → Zn2+(aq) + 2 e-
C 60 min 60 s 1 mol e_ 1 mol Zn 65.39 g Zn
mass Zn = 0.100 x 200.0 h x x x x x
s h min 96,500 C 2 mol e _ 1 mol Zn
mass Zn = 24.4 g

18.106 2 Cl-(aq) → Cl2(g) + 2 e-


13 million tons = 13 x 106 tons; Cl2, 70.91 amu
907,200 g 1 mol Cl2
13 x 106 tons x x = 1.66 x 1011 mol Cl2
1 ton 70.91 g Cl2
2 mol e _ 96,500 C
Charge = 1.66 x 1011 mol Cl2 x x _
= 3.20 x 1016 C
1 mol Cl2 1 mol e
1 J = 1 C x 1 V; Energy = (3.20 x 10 C)(4.5 V) = 1.44 x 1017 J
16

551
Chapter 18 - Electrochemistry
_____________________________________________________________________________

 1 kWh 
kWh = (1.44 x 1017 J)  6
10
 = 4.0 x 10 kWh
 3.6 x 10 J

18.107 (a) From: B + A+ → B+ + A, A+ is reduced more easily than B+


From: C + A+ → C+ + A, A+ is reduced more easily than C+
From: B + C+ → B+ + C, C+ is reduced more easily than B+
A+ + e- → A
C+ + e- → C
B+ + e- → B
(b) A+ is the strongest oxidizing agent; B is the strongest reducing agent
(c) A+ + B → B+ + A

18.108 (a) oxidizing agents: PbO2, H+, Cr2O72-; reducing agents: Al, Fe, Ag
(b) PbO2 is the strongest oxidizing agent. H+ is the weakest oxidizing agent.
(c) Al is the strongest reducing agent. Ag is the weakest reducing agent.
(d) oxidized by Cu2+: Fe and Al; reduced by H2O2: PbO2 and Cr2O72-

18.109 From Appendix D:


AgBr(s) + e- → Ag(s) + Br-(aq) Eo = 0.07 V
(a) oxidation: C6H4(OH)2(aq) → C6H4O2(aq) + 2 H+(aq) + 2 e- Eo = -0.699 V
reduction: 2[AgBr(s) + e → Ag(s) + Br (aq)]
- -
Eo = 0.07 V
overall: 2 AgBr(s) + C6H4(OH)2(aq) →
2 Ag(s) + 2 Br-(aq) + C6H4O2(aq) + 2 H+(aq)
overall Eo = -0.699 V + 0.07 V = -0.63 V
Because the overall Eo is negative, the reaction is nonspontaneous when [H+] = 1.0 M.
0.0592 V [ _ ]2 [H+ ]2 [C6 H 4 O2]
(b) Eo(in 1.0 M OH-) = Eo(in 1.0 M H+) - log Br
n [C6 H 4 (OH )2]
_ 14
[H+] = K w = 1.0 x 10 = 1.0 x 10-14 M
[OH _ ] 1.0
(0.0592 V) (1)2 (10_ 14 )2 (1)
Eo(in 1.0 M OH-) = -0.63 V - log = +0.20 V
2 (1)

18.110 (a) 3 CH3CH2OH(aq) + 2 Cr2O72-(aq) + 16 H+(aq) →


3 CH3CO2H(aq) + 4 Cr3+(aq) + 11 H2O(l)
oxidation:
3 CH3CH2OH(aq) + 3 H2O(l) → 3 CH3CO2H(aq) + 12 H+(aq) + 12 e- Eo = -0.058V
reduction:
2 Cr2O72-(aq) + 28 H+(aq) + 12 e- + → 4 Cr3+(aq) + 14 H2O(l) Eo = 1.33 V
overall Eo = 1.27 V

552
Chapter 18 - Electrochemistry
_____________________________________________________________________________

0.0592 V [CH3 CO2 H ]3 [Cr 3+ ]4


(b) E = Eo - log
n [CH3 CH 2 OH ]3 [Cr 2 O72 _ ]2 [H + ]16
pH = 4.00, [H+] = 0.000 10 M
(0.0592 V)  (1.0 )3 (1.0 )4 
E = 1.27 V - 
log  3 2

16 
12  (1.0 ) (1.0 ) (0.000 10 ) 
(0.0592 V) 1
E = 1.27 V - log = 0.95 V
12 (0.000 10 )16

18.111 (a) ∆Go = -nFEo


∆Go3 = ∆Go1 + ∆Go2 therefore -n3FEo3 = -n1FEo1 + (-n2FEo2)
n3Eo3 = n1Eo1 + n2Eo2
n o + n 2 Eo 2
Eo3 = 1 E 1
n3
(3)(_ 0.04 V) + (2)(0.45 V)
(b) Eo3 = = 0.78 V
1
(c) Eo values would be additive (Eo3 = Eo1 + Eo2) if reaction (3) is an overall cell
reaction because the electrons in the two half reactions, (1) and (2), cancel. That is, n1 =
n2 = n3 in the equation for Eo3.

18.112 anode: Ag(s) + Cl-(aq) → AgCl(s) + e-


cathode: Ag+(aq) + e- → Ag(s)
overall: Ag+(aq) + Cl-(aq) → AgCl(s) Eo = 0.578 V

For AgCl(s) _ Ag+(aq) + Cl-(aq) Eo = -0.578 V


0.0592 V nE o (1)(_ 0.578 V)
Eo = log K; log K = = = -9.76
n 0.0592 V 0.0592 V
K = Ksp = 10-9.76 = 1.7 x 10-10

18.113 (a) anode: Cu(s) → Cu2+(aq) + 2 e- Eo = -0.34 V


cathode: 2 Ag+(aq) + 2 e- → 2 Ag(s) Eo = 0.80 V
overall: 2 Ag (aq) + Cu(s) → Cu (aq) + 2 Ag(s) Eo = 0.46 V
+ 2+

0.0592 V [Cu 2+ ] (0.0592 V)  1.0 


E = Eo - log + 2
= 0.46 V _ log   = 0.38 V
2
n [Ag ] 2  (0.050 ) 
Ksp 5.4 x 10_13
(b) [Ag+] = _
= = 5.4 x 10-13 M
[Br ] 1.0 M
0.0592 V [Cu 2+ ] (0.0592 V)  1.0 
E = Eo - log + 2
= 0.46 V _ log   = -0.27 V
_ 13 2 
n [Ag ] 2  (5.4 x 10 ) 
The cell potential for the spontaneous reaction is E = 0.27 V.
The spontaneous reaction is: Cu2+(aq) + 2 Ag(s) + 2 Br-(aq) → 2 AgBr(s) + Cu(s)

553
Chapter 18 - Electrochemistry
_____________________________________________________________________________

(c) Cu2+(aq) + 2 e- → Cu(s) Eo = 0.34 V


2 Ag(s) + 2 Br-(aq) → 2 AgBr(s) + 2 e- Eo = ?
Cu2+(aq) + 2 Ag(s) + 2 Br-(aq) → 2 AgBr(s) + Cu(s) Eo = 0.27 V
Eo = ? = 0.27 V - 0.34 V = -0.07 V
For: AgBr(s) + e- → Ag(s) + Br-(aq)
the standard reduction potential is Eo = 0.07 V

18.114 4 Fe2+(aq) + O2(g) + 4 H+(aq) → 4 Fe3+(aq) + 2 H2O(l)

oxidation 4 Fe2+(aq) → 4 Fe3+(aq) + 4 e- Eo = - 0.77 V


reduction O2(g) + 4 H+(aq) + 4 e- → 2 H2O(l) Eo = 1.23 V
overall Eo = 0.46 V
1.00 atm
PO2 = 160 mm Hg x = 0.211 atm
760 mm Hg
0.0592 V [Fe3+ ]4
E = Eo - log
n [Fe2+ ]4 [H+ ]4 (PO2)
0.0592 V (1 x 10_ 7 )4
E = 0.46 V - log
4 (1 x 10_ 7 )4 (1 x 10_ 7 )4 (0.211)
E = 0.46 V - 0.42 V = 0.04 V
Because E is positive, the reaction is spontaneous.

18.115 H2MoO4(aq) + As(s) → Mo3+(aq) + H3AsO4(aq)

H2MoO4(aq) → Mo3+(aq)
H2MoO4(aq) → Mo3+(aq) + 4 H2O(l)
6 H+(aq) + H2MoO4(aq) → Mo3+(aq) + 4 H2O(l)
[3 e- + 6 H+(aq) + H2MoO4(aq) → Mo3+(aq) + 4 H2O(l)] x 5
(reduction half reaction)
As(s) → H3AsO4(aq)
As(s) + 4 H2O(l) → H3AsO4(aq)
As(s) + 4 H2O(l) → H3AsO4(aq) + 5 H+(aq)
[As(s) + 4 H2O(l) → H3AsO4(aq) + 5 H+(aq) + 5 e-] x 3 (oxidation half reaction)

Combine the two half reactions.


30 H+(aq) + 5 H2MoO4(aq) + 3 As(s) + 12 H2O(l) →
5 Mo3+(aq) + 3 H3AsO4(aq) + 15 H+(aq) + 20 H2O(l)
15 H+(aq) + 5 H2MoO4(aq) + 3 As(s) → 5 Mo3+(aq) + 3 H3AsO4(aq) + 8 H2O(l)

5 x [H2MoO4(aq) + 2 H+(aq) + 2 e- → MoO2(s) + 2 H2O(l)] Eo = +0.646 V


5 x [MoO2(s) + 4 H+(aq) + e- → Mo3+(aq) + 2 H2O(l)] Eo = -0.008 V

554
Chapter 18 - Electrochemistry
_____________________________________________________________________________

3 x [As(s) + 3 H2O(l) → H3AsO3(aq) + 3 H+(aq) + 3 e-] Eo = -0.240 V


3 x [H3AsO3(aq) + H2O(l) → H3AsO4(aq) + 2 H+(aq) + 2 e-] Eo = -0.560 V

15 H+(aq) + 5 H2MoO4(aq) + 3 As(s) → 5 Mo3+(aq) + 3 H3AsO4(aq) + 8 H2O(l)

 96,500 C   1J 
∆Go = -nFEo = -(10 mol e-)   (0.646 V)   = -623,390 J = -623.4 kJ
1 C • V 
_
 1 mol e 
 96,500 C   1J 
∆Go = -nFEo = -(5 mol e-)   (-0.008 V)   = 3,860 J = +3.9 kJ
1 C • V 
_
 1 mol e 
 96,500 C   1J 
∆Go = -nFEo = -(9 mol e-)   (-0.240 V)   = 208,440 J = +208.4 kJ
1 C • V 
_
 1 mol e 
 96,500 C   1J 
∆Go = -nFEo = -(6 mol e-)   (-0.560 V)   = 324,240 J = +324.2 kJ
1 C • V 
_
 1 mol e 
∆Go(total) = -623.4 kJ + 3.9 kJ + 208.4 kJ + 324.2 kJ = -86.9 kJ = -86,900 J
1 V = 1 J/C
_ ∆G o _ (_ 86,900 J)
∆Go = -nFEo; Eo = = = +0.060 J/C = +0.060 V
nF _  96,500 C 
(15 mol e ) _

 1 mol e 

18.116 First calculate Eo for the galvanic cell in order to determine Eo1.
anode: 5 [2 Hg(l) + 2 Br-(aq) → Hg2Br2(s) + 2 e- ] Eo1 = ?
cathode: 2 [MnO4 (aq) + 8 H (aq) + 5 e → Mn (aq) + 4 H2O(l)]
- + - 2+
Eo2 = 1.51 V
overall: 2 MnO4-(aq) + 10 Hg(l) + 10 Br-(aq) + 16 H+(aq) →
2 Mn2+(aq) + 5 Hg2Br2(s) + 8 H2O(l)
n = 10 mol e-
0.0592 V [Mn 2+ ]2
E = Eo - log
n [Br _ ]10 [MnO4_ ]2 [H + ]16
(0.0592 V)  (0.10 )2 
1.214 V = Eo - log  10 2

16 
10  (0.10 ) (0.10 ) (0.10 ) 
(0.0592 V) 1
1.214 V = Eo - log 26
= Eo - 0.154 V
10 (0.10 )
o
E = 1.214 + 0.154 = 1.368 V
Eo1 + Eo2 = 1.368 V; Eo1 +1.51 V = 1.368 V; Eo1 = 1.368 V - 1.51 V = -0.142 V
oxidation: 2 Hg(l) → Hg22+(aq) + 2 e- Eo = -0.80 V (Appendix D)
reduction: Hg2Br2(s) + 2 e- → 2 Hg(l) + 2 Br-(aq) Eo = -0.142 V (from Eo1)
overall: Hg2Br2(s) → Hg22+(aq) + 2 Br-(aq) Eo = -0.658 V
0.0592 V nE o (2)(_ 0.658 V)
Eo = log K; log K = = = -22.2
n 0.0592 V 0.0592 V
K = Ksp = 10-22.2 = 6 x 10-23

18.117 oxidation: Cu+(aq) → Cu2+(aq) + e- Eo = -0.15 V

555
Chapter 18 - Electrochemistry
_____________________________________________________________________________

reduction: Cu2+(aq) + 2 CN-(aq) + e- → Cu(CN)2-(aq) Eo = 1.103 V


overall: Cu (aq) + 2 CN (aq) → Cu(CN)2 (aq)
+ - -
Eo = 0.953 V
0.0592 V nE o (1)(0.953 V)
Eo = log K; log K = = = 16.1
n 0.0592 V 0.0592 V
K = Kf = 1016.1 = 1 x 1016

18.118 (a) anode: 4[Al(s) → Al3+(aq) + 3 e-] Eo = 1.66 V


cathode: 3[O2(g) + 4 H+(aq) + 4 e- → 2 H2O(l)] Eo = 1.23 V
overall: 4 Al(s) + 3 O2(g) + 12 H+(aq) → 4 Al3+(aq) + 6 H2O(l) Eo = 2.89 V
2.303 R T [Al3+ ]4
(b) & (c) E = Eo - log 3
nF (PO2 ) [H + ]12
 kJ 
(2.303) 8.314 x 10_ 3 (310 K)
 K • mol   (1.0 x 10_ 9 )4 
E = 2.89 V - log  
(12 mol e _ )(96,500 C/mol e_ )  (0.20 ) (1.0 x _ 7 ) 
3 12
 10 
E = 2.89 V - 0.257 V = 2.63 V

0.0592 V [Cu 2+ ]3 (P NO )2 0.0592 V [Cu 2+](P NO2 )2


18.119 E1 = Eo1
- log and E2 = E 2 -o
log
6 [ NO3_ ]2 [H + ]8 2 [ NO3_ ]2 [H + ]4
0.0592 V (0.10 )3 (1.0 x 10_ 3 )2
(a) E1 = 0.62 V - log = 0.71 V
6 (1.0 )2 (1.0 )8
0.0592 V (0.10)(1.0 x 10_ 3 )2
E2 = 0.45 V - log = 0.66 V
2 (1.0 )2 (1.0 )4
Reaction (1) has the greater thermodynamic tendency to occur because of the larger
positive potential.
0.0592 V (0.10 )3 (1.0 x 10_ 3 )2
(b) E1 = 0.62 V - log = 0.81 V
6 (10.0 )2 (10.0 )8
0.0592 V (0.10)(1.0 x 10_ 3 )2
E2 = 0.45 V - log = 0.83 V
2 (10.0 )2 (10.0 )4
Reaction (2) has the greater thermodynamic tendency to occur because of the larger
positive potential.
(c) Set the two equations equal to each other and solve for x.
0.0592 V (0.10 )3 (1.0 x 10_ 3 )2 0.0592 V
0.62 V - log 2 8
= 0.45 V - log
6 (x ) (x ) 2
(0.10)(1.0 x 10_ 3 )2
(x )2 (x )4
0.0592 V 0.0592 V
0.17 - [(-9) - 10 log x] = - [(-7) - 6 log x]
6 2
0.0516
0.0516 = 0.0789 log x; = log x; 0.654 = log x
0.0789
[HNO3] = x = 100.654 = 4.5 M

556
Chapter 18 - Electrochemistry
_____________________________________________________________________________

Multi-Concept Problems

18.120 (a) 4 CH2=CHCN + 2 H2O → 2 NC(CH2)4CN + O2


3600 s 1 mol e _
(b) mol e- = 3000 C/s x 10.0 h x x = 1119.2 mol e-
1h 96,500 C
mass adiponitrile =
1 mol adiponitrile 108.14 g adiponitrile 1.0 kg
1119.2 mol e- x _
x x = 60.5 kg
2 mol e 1 mol adiponitrile 1000 g
1 mol O2
(c) 1119.2 mol e- x = 279.8 mol O2
4 mol e_
PV = nRT
 L• atm 
(279.8 mol) 0.082 06 (298 K)
nRT  K• mol 
V= = = 7030 L O2
P  1 atm 
 740 mm Hg x 
 760 mm Hg 

18.121 (a) 2 MnO4-(aq) + 5 H2C2O4(aq) + 6 H+(aq) →


2 Mn2+(aq) + 10 CO2(g) + 8 H2O(l)
(b) oxidation: 5[H2C2O4(aq) → 2 CO2(g) + 2 H+(aq) + 2 e-] Eo = 0.49 V
reduction: 2[MnO4-(aq) + 8 H+(aq) + 5 e- → Mn2+(aq) + 4 H2O(l)] Eo = 1.51 V
overall Eo = 2.00 V
(c)
 96,500 C   1J 
∆Go = -nFEo = -(10 mol e-)  (2.00 V)   = -1,930,000 J = -1,930 kJ
1 C • V 
_
 1 mol e 
0.0592 V nE o (10)(2.00 V)
Eo = log K; log K = = = 338; K = 10338
n 0.0592 V 0.0592 V

(d) Na2C2O4, 134.0 amu


1 mol Na 2 C2 O4 2 mol KMnO 4
1.200 g Na2C2O4 x x = 3.582 x 10-3 mol KMnO4
134.0 g Na 2 C2 O4 5 mol Na 2 C2 O4
3.582 x 10_ 3 mol
molarity = = 0.1102 M
0.032 50 L

18.122 (a) Cr2O72-(aq) + 6 Fe2+(aq) + 14 H+(aq) → 2 Cr3+(aq) + 6 Fe3+(aq) + 7 H2O(l)


(b) The two half reactions are:
oxidation: Fe2+(aq) → Fe3+(aq) + e- Eo = -0.77 V
reduction: Cr2O72-(aq) + 14 H+(aq) + 6 e- → 2 Cr3+(aq) + 7 H2O(l) Eo = 1.33 V
At the equivalence point the potential is given by either of the following expressions:

557
Chapter 18 - Electrochemistry
_____________________________________________________________________________

0.0592 V [Cr 3+ ]2
(1) E = 1.33 V - log
6 [Cr 2 O72 _ ][H + ]14
0.0592 V [ 2+]
(2) E = 0.77 V - log Fe 3+
1 [Fe ]
where E is the same in both because equilibrium is reached and the solution can have
only one potential. Multiplying (1) by 6, adding it to (2), and using some stoichiometric
relationships at the equivalence point will simplify the log term.
[Fe 2+][Cr 3+ ]2
7E = [(6 x 1.33 V) + 0.77 V] - (0.0592 V)log
[Fe3+][Cr 2 O72 _ ][H + ]14

At the equivalence point, [Fe2+] = 6[Cr2O72-] and [Fe3+] = 3[Cr3+]. Substitute these
equalities into the previous equation.
6[Cr 2 O72 _ ][Cr3+ ]2
7E = [(6 x 1.33 V) + 0.77 V] - (0.0592 V)log
3[Cr 3+ ][Cr 2 O72 _ ][H + ]14
Cancel identical terms.
6[Cr 3+]
7E = [(6 x 1.33 V) + 0.77 V] - (0.0592 V)log
3[H + ]14
mol Fe2+ = (0.120 L)(0.100 mol/L) = 0.0120 mol Fe2+
1 mol Cr 2 O72 _
mol Cr2O72- = 0.0120 mol Fe2+ x 2+
= 0.00200 mol Cr2O72-
6 mol Fe
1L
volume Cr2O72- = 0.00200 mol x = 0.0167 L
0.120 mol
At the equivalence point assume mol Fe3+ = initial mol Fe2+ = 0.0120 mol
Total volume at the equivalence point is 0.120 L + 0.0167 L = 0.1367 L
0.0120 mol
[Fe3+] = = 0.0878 M; [Cr3+] = [Fe3+]/3 = (0.0878 M)/3 = 0.0293 M
0.1367 L
[H+] = 10-pH = 10-2.00 = 0.010 M
6(0.0293)
7E = [(6 x 1.33 V) + 0.77 V] - (0.0592 V)log = 8.75 - 1.585 = 7.165 V
3(0.010 )14
7.165 V
E= = 1.02 V at the equivalence point.
7

18.123 2 H2(g) + O2(g) → 2 H2O(l)


(a) ∆Ho = 2 ∆Hof(H2O) = (2 mol)(-285.8 kJ/mol) = -571.6 kJ
∆So = 2 So(H2O) - [2 So(H2) + So(O2)]
∆So = (2 mol)(69.9 J/(K ⋅ mol)) - [(2 mol)(130.6 J/(K ⋅ mol)) + (1 mol)(205.0 J/(K ⋅ mol))]
∆So = -326.4 J/K = -0.3264 kJ/K
95oC = 368 K
∆Go = ∆Ho - T∆So = -571.6 kJ -(368 K)(-0.3264 kJ/K) = - 451.5 kJ
1 V = 1 J/C

558
Chapter 18 - Electrochemistry
_____________________________________________________________________________

∆G o _ 451.5 x 103 J
∆Go = -nFEo; Eo = _ =_ = 1.17 J/C = 1.17 V
nF (4)(96,500 C)
2.303 R T 1
(b) E = Eo - log 2
nF ( P H2 ) ( P O2 )
 kJ 
(2.303) 8.314 x 10_ 3 (368 K)
 mol • K   1 
E = 1.17 V - log  
(4 mol e_ )(96,500 C/mol e _ )  (25 ) (25) 
2
 
E = 1.17 V + 0.077 V = 1.25 V

18.124 (a) Zn(s) + 2 Ag+(aq) + H2O(l) → ZnO(s) + 2 Ag(s) + 2 H+(aq)


∆Horxn = ∆Hof(ZnO) - [2 ∆Hof(Ag+) + ∆Hof(H2O)]
∆Horxn = [(1 mol)(-348.3 kJ/mol)] - [(2 mol)(105.6 kJ/mol) + (1 mol)(-285.8 kJ/mol)]
∆Horxn = -273.7 kJ
∆So = [So(ZnO) + 2 So(Ag)] - [So(Zn) + 2 So(Ag+) + So(H2O)]
∆So = [(1 mol)(43.6 J/(K⋅ mol)) + (2 mol)(42.6 J/(K⋅ mol))]
- [(1 mol)(41.6 J/(K⋅ mol)) + (2 mol)(72.7 J/(K⋅ mol)) + (1 mol)(69.9 J/(K⋅ mol))
∆So = - 128.1 J/K
∆Go = ∆Ho - T∆So = - 273.7 kJ - (298 K)(- 128.1 x 10-3 kJ/K) = - 235.5 kJ
(b) 1 V = 1 J/C
_ ∆G o _(_ 235.5 x 103 J)
∆Go = -nFEo Eo = = = 1.220 J/C = 1.220 V
nF _  96,500 C 
(2 mol e ) _

 1 mol e 
0.0592 V nE o (2)(1.220 V)
Eo = log K; log K = = = 41.22
n 0.0592 V 0.0592 V
K = 1041.22 = 2 x 1041
0.0592 V [ + ]2
(c) E = Eo - log H + 2
n [Ag ]
The addition of NH3 to the cathode compartment would result in the formation of the
Ag(NH3)2+ complex ion which results in a decrease in Ag+ concentration. The log term
in the Nernst equation becomes larger and the cell voltage decreases.

On mixing equal volumes of two solutions, the concentrations of both solutions are cut
in half.
Ag+(aq) + 2 NH3(aq) _ Ag(NH3)2+(aq)
before reaction (M) 0.0500 2.00 0
assume 100% reaction -0.0500 -2(0.0500) +0.0500
after reaction (M) 0 1.90 0.0500
assume small back rxn +x +2x -x
equil (M) x 1.90 + 2x 0.0500 - x
+
[Ag( NH3 )2 ] (0.0500 _ x) 0.0500
Kf = 1.7 x 107 = = ≈
[Ag ][ NH3 ] (x)(1.90 + 2 x ) (x)(1.90 )2
+ 2 2

559
Chapter 18 - Electrochemistry
_____________________________________________________________________________

Solve for x. x = [Ag+] = 8.15 x 10-10 M


o 0.0592 V [H + ]2 0.0592 V (1.00 M )2
E=E - log = 1.220 V - log = 0.682
n [Ag+ ]2 2 (8.15 x 10_10 M )2
V

(d) Calculate new initial concentrations because of dilution to 110.0 mL.


x 0.200 M x 10.0 mL
Mi x Vi = Mf x Vf; Mf = [Cl-] = Mi Vi = = 0.0182 M
Vf 110.0 mL
x 0.0500 M x 100.0 mL
Mi x Vi = Mf x Vf; Mf = [Ag+] = Mi Vi = = 0.0455 M
Vf 110.0 mL
x Vi 2.00 M x 100.0 mL
Mi x Vi = Mf x Vf; Mf = [NH3] = Mi = = 1.82 M
Vf 110.0 mL
Now calculate the [Ag+] as a result of the following equilibrium:
Ag+(aq) + 2 NH3(aq) _ Ag(NH3)2+(aq)
before reaction (M) 0.0455 1.82 0
assume 100% reaction -0.0455 -2(0.0455) +0.0455
after reaction (M) 0 1.73 0.0455
assume small back rxn +x +2x -x
equil (M) x 1.73 + 2x 0.0455 - x
+
[Ag( NH3 )2 ] (0.0455 _ x) 0.0455
Kf = 1.7 x 107 = = ≈
[Ag ][ NH3 ] (x)(1.73 + 2 x ) (x)(1.73 )2
+ 2 2

Solve for x. x = [Ag+] = 8.94 x 10-10 M


For AgCl, Ksp = 1.8 x 10-10
IP = [Ag+][Cl-] = (8.94 x 10-10 M)(0.0182 M) = 1.6 x 10-11
IP < Ksp, AgCl will not precipitate.
Now calculate new initial concentrations because of dilution to 120.0 mL.
x 0.200 M x 10.0 mL
Mi x Vi = Mf x Vf; Mf = [Br-] = Mi Vi = = 0.0167 M
Vf 120.0 mL
x 0.0500 M x 100.0 mL
Mi x Vi = Mf x Vf; Mf = [Ag+] = Mi Vi = = 0.0417 M
Vf 120.0 mL
x 2.00 M x 100.0 mL
Mi x Vi = Mf x Vf; Mf = [NH3] = Mi Vi = = 1.67 M
Vf 120.0 mL
Now calculate the [Ag+] as a result of the following equilibrium:
Ag+(aq) + 2 NH3(aq) _ Ag(NH3)2+(aq)
before reaction (M) 0.0417 1.67 0
assume 100% reaction -0.0417 -2(0.0417) +0.0417
after reaction (M) 0 1.59 0.0417
assume small back rxn +x +2x -x
equil (M) x 1.59 + 2x 0.0417 - x
+
[Ag( NH3 )2 ] (0.0417 _ x) 0.0417
Kf = 1.7 x 107 = = ≈
[Ag ][ NH3 ] (x)(1.59 + 2 x ) (x)(1.59 )2
+ 2 2

Solve for x. x = [Ag+] = 9.70 x 10-10 M

560
Chapter 18 - Electrochemistry
_____________________________________________________________________________

For AgBr, Ksp = 5.4 x 10-13


IP = [Ag+][Br-] = (9.70 x 10-10 M)(0.0167 M) = 1.6 x 10-11
IP > Ksp, AgBr will precipitate.

18.125 (a) anode: Fe(s) + 2 OH-(aq) → Fe(OH)2(s) + 2 e-


cathode: 2 x [NiO(OH)(s) + H2O(l) + e- → Ni(OH)2(s) + OH-(aq)]
overall: Fe(s) + 2 NiO(OH)(s) + 2 H2O(l) → Fe(OH)2(s) + 2 Ni(OH)2(s)
(b)
 96,500 C   1J 
∆Go = -nFEo = -(2 mol e-)   (1.37 V)   = -264,410 J = -264 kJ
1 C • V 
_
 1 mol e 
0.0592 V nE o (2)(1.37 V)
Eo = log K; log K = = = 46.3
n 0.0592 V 0.0592 V
K = 1046.3 = 2 x 1046
(c) It would still be 1.37 V because OH- does not appear in the overall cell reaction. The
overall cell reaction contains only solids and one liquid, therefore the cell voltage does
not change because there are no concentration changes.
(d) Fe(OH)2, 89.86 amu; 1 A = 1 C/s
 60 s  1 mol e_ 
mol e- = (0.250 C/s)(40.0 min)    = 6.22 x 10-3 mol e-
 1 min  96,500 C 

1 mol Fe(OH )2 89.86 g Fe(OH )2


mass Fe(OH)2 = (6.22 x 10-3 mol e-) x x = 0.279 g
2 mol e _ 1 mol Fe(OH )2

H2O molecules consumed =


2 mol H 2 O 6.022 x 1023 H 2 O molecules
(6.22 x 10-3 mol e-) x _
x = 3.75 x 1021 H2O
2 mol e 1 mol H 2 O
molecules

18.126 (a) Oxidation half reaction: 2 [C4H10(g) + 13 O2-(s) → 4 CO2(g) + 5 H2O(l) + 26 e-]
Reduction half reaction: 13 [O2(g) + 4 e- → 2 O2-(s)]
Cell reaction: 2 C4H10(g) + 13 O2(g) → 8 CO2(g) + 10 H2O(l)
(b) ∆H = [8 ∆H f(CO2)) + 10 ∆Hof(H2O)] - [2 ∆Hof(C4H10)]
o o

∆Ho = [(8 mol)(-393.5 kJ/mol) + (10 mol)(-285.8 kJ/mol)]


- [(2 mol)(-126 kJ/mol)] = -5754 kJ
∆So = [8 So(CO2) + 10 So(H2O)] - [2 So(C4H10) + 13 So(O2)]
∆So = [(8 mol)(213.6 J/(K ⋅ mol)) + (10 mol)(69.9 J/(K ⋅ mol))]
- [(2 mol)(310 J/(K ⋅ mol)) + (13 mol)(205 J/(K ⋅ mol))] = -877.2 J/K
∆G = ∆H - T∆So = -5754 kJ - (298 K)(-877.2 x 10-3 kJ/K) = -5493 kJ
o o

1 V = 1 J/C
∆G o _ 5493 x 103 J
∆Go = - nFEo; Eo = _ =_ = 1.09 J/C = 1.09 V
nF (52)(96,500 C)
∆Go = -RT ln K

561
Chapter 18 - Electrochemistry
_____________________________________________________________________________

_ ∆G o _ (_ 5493 kJ)
ln K = = = 2217
RT (8.314 x 10_ 3 kJ/K)(298 K)
K = e2217 = 7 x 10962
On raising the temperature, both K and Eo will decrease because the reaction is
exothermic (∆Ho < 0).
(c) C4H10, 58.12 amu; 10.5 A = 10.5 C/s
60 min 60 s 1 mol e_ 2 mol C4 H10
mass C4H10 = 10.5 C/s x 8 hr x x x x x
1 hr 1 min 96,500 C 52 mol e_
58.12 g C4 H10
= 7.00 g C4H10
1 mol C4 H10
1 mol C4 H10
n = 7.00 g C4H10 x = 0.120 mol C4H10
58.12 g C4 H10

20oC = 20 + 273 = 293 K


PV = nRT
 L • atm 
(0.120 mol) 0.082 06 (293 K)
nRT  K • mol 
V= = = 2.69 L
P  1.00 atm 
 815 mm Hg x 
 760 mm Hg 

18.127 cathode:
(1) MnO2(s) + 4 H+(aq) + 2 e- → Mn2+(aq) + 2 H2O(l) Eo = +1.22 V
(2) Mn(OH)2(s) + OH-(aq) → MnO(OH)(s) + H2O(l) + e- Eo = +0.380 V
(3) Mn2+(aq) + 2 OH-(aq) → Mn(OH)2(s) K = 1/Ksp = 1/(2.1 x 10-13) = 4.8 x 1012
(4) 4 x [H2O(l) → H (aq) + OH (aq)]
+ -
K = (Kw)4 = (1.0 x 10-14)4 = 1.0 x 10-56
MnO2(s) + H2O(l) + e- → MnO(OH)(s) + OH-(aq)
 96,500 C   1J 
∆Go1 = -nFEo = -(2 mol e-)   (1.22 V)   = -235,460 J = -235.5 kJ
1 C • V 
_
 1 mol e 
 96,500 C   1J 
∆Go2 = -nFEo = -(1 mol e-)   (0.380 V)   = -36,670 J = -36.7 kJ
1 C • V 
_
 1 mol e 
∆Go3 = - RT ln K = -(8.314 x 10-3 kJ/K)(298 K) ln (4.8 x 1012) = -72.3 kJ
∆Go4 = - RT ln K = -(8.314 x 10-3 kJ/K)(298 K) ln (1.0 x 10-56) = +319.5 kJ
∆Go(total) = -235.5 kJ - 36.7 kJ - 72.3 kJ + 319.5 kJ = -25.0 kJ = -25,000 J
1 V = 1 J/C
_ ∆G o _ (_ 25,000 J)
∆Go = -nFEo; Eo = = = +0.259 J/C = +0.259 V
nF _  96,500 C 
(1 mol e ) _

 1 mol e 
Eocathode = +0.259 V

562
Chapter 18 - Electrochemistry
_____________________________________________________________________________

anode:
(1) Zn(s) → Zn2+(aq) + 2 e- Eo = +0.76 V
(2) Zn2+(aq) + 2 OH-(aq) → Zn(OH)2(s) K = 1/Ksp = 1/(4.1 x 10-17) = 2.4 x 1016
Zn(s) + 2 OH-(aq) → Zn(OH)2(s) + 2 e-
 96,500 C   1J 
∆Go1 = -nFEo = -(2 mol e-)   (0.76 V)   = -146,680 J = -146.7 kJ
1 C • V 
_
 1 mol e 
∆Go2 = -RT ln K = -(8.314 x 10-3 kJ/K)(298 K) ln (2.4 x 1016) = -93.4 kJ
∆Go(total) = -146.7 kJ - 93.4 kJ = -240.1 kJ = -240,100 J
1 V = 1 J/C
_ ∆G o _ (_ 240,100 J)
∆Go = -nFEo; Eo = = = +1.24 J/C = +1.24 V
nF _  96,500 C 
(2 mol e ) _

 1 mol e 
Eoanode = +1.24 V
Eocell = Eocathode + Eoanode = 0.259 V + 1.24 V = 1.50 V

(b) FeO42-(aq) → Fe(OH)3(s)


FeO42-(aq) → Fe(OH)3(s) + H2O(l)
FeO42-(aq) + 5 H+(aq) → Fe(OH)3(s) + H2O(l)
FeO42-(aq) + 5 H+(aq) + 3 e- → Fe(OH)3(s) + H2O(l)
FeO42-(aq) + 5 H+(aq) + 5 OH-(aq) + 3 e- → Fe(OH)3(s) + H2O(l) + 5 OH-(aq)
FeO42-(aq) + 5 H2O(l) + 3 e- → Fe(OH)3(s) + H2O(l) + 5 OH-(aq)
FeO42-(aq) + 4 H2O(l) + 3 e- → Fe(OH)3(s) + 5 OH-(aq)

(c) K2FeO4, 198.04 amu; MnO2, 86.94 amu


1 mol K 2 FeO 4 3 mol e _ 96,500 C
coulombs = 10.00 g K2FeO4 x x x =
198.04 g K 2 FeO 4 1 mol K 2 FeO 4 1 mol e _
1.46 x 104 C from 10.00 g K2FeO4
1 mol MnO2 1 mol e _ 96,500 C
coulombs = 10.00 g MnO2 x x x =
86.94 g MnO2 1 mol MnO2 1 mol e_
1.11 x 104 C from 10.00 g MnO2

18.128 (a) 4 [Au(s) + 2 CN-(aq) → Au(CN)2-(aq) + e-] oxidation half reaction

O2(g) → 2 H2O(l)
O2(g) + 4 H+(aq) → 2 H2O(l)
4 e- + O2(g) + 4 H+(aq) → 2 H2O(l) reduction half reaction

Combine the two half reactions.


4 Au(s) + 8 CN-(aq) + O2(g) + 4 H+(aq) → 4 Au(CN)2-(aq) + 2 H2O(l)
4 Au(s) + 8 CN-(aq) + O2(g) + 4 H+(aq) + 4 OH-(aq)
→ 4 Au(CN)2-(aq) + 2 H2O(l) + 4 OH-(aq)

563
Chapter 18 - Electrochemistry
_____________________________________________________________________________

4 Au(s) + 8 CN-(aq) + O2(g) + 4 H2O(l)


→ 4 Au(CN)2-(aq) + 2 H2O(l) + 4 OH-(aq)
4 Au(s) + 8 CN-(aq) + O2(g) + 2 H2O(l) → 4 Au(CN)2-(aq) + 4 OH-(aq)
(b) Add the following five reactions together. ∆Go is calculated below each reaction.
4 [Au+(aq) + 2 CN-(aq) → Au(CN)2-(aq)] K = (Kf)4
∆G = -RT ln K = -(8.314 x 10 kJ/K)(298 K) ln (6.2 x 1038)4 = -885.2 kJ
o -3

O2(g) + 4 H+(aq) + 4 e- → 2 H2O(l) Eo = 1.229 V


 96,500 C   1J 
∆Go = -nFEo = -(4 mol e-)  (1.229 V)   = - 474,394 J = - 474.4 kJ
1 C • V 
_
 1 mol e 
4 [H2O(l) _ H+(aq) + OH-(aq)] K = (Kw)4
∆G = -RT ln K = -(8.314 x 10 kJ/K)(298 K) ln (1.0 x 10-14)4 = +319.5 kJ
o -3

4 [Au(s) → Au3+(aq) + 3 e-] Eo = -1.498 V


 96,500 C   1J 
∆Go = -nFEo = -(12 mol e-)  (_ 1.498 V)   = +1,734,684 J = +1,734.7 kJ
1 C • V 
_
 1 mol e 

4 [Au3+(aq) + 2 e- → Au+(aq)] Eo = 1.401 V


 96,500 C   1J 
∆Go = - nFEo = -(8 mol e-)  (1.401 V)   = -1,081,572 J = -1,081.6 kJ
1 C • V 
_
 1 mol e 
Overall reaction:
4 Au(s) + 8 CN-(aq) + O2(g) + 2 H2O(l) → 4 Au(CN)2-(aq) + 4 OH-(aq)
∆Go = - 885.2 kJ - 474.4 kJ + 319.5 kJ + 1,734.7 kJ - 1,081.6 kJ = - 387.0 kJ
18.129 The overall cell reaction is:
2 Fe3+(aq) + 2 Hg(l) + 2 Cl-(aq) → 2 Fe2+(aq) + Hg2Cl2(s)
The Nernst equation can be applied to separate half reactions.
One half reaction is for the calomel reference electrode.
2 Hg(l) + 2 Cl-(aq) → Hg2Cl2(s) + 2 e- Eo = -0.28 V
When [Cl-] = 2.9 M,
0.0592 V 1 0.0592 V 1
Ecalomel = Eo - log 2
= -0.28 V - log 2
= -0.25 V
n [Cl _ ] 2 (2.9 )

Balance the titration redox reaction: MnO4-(aq) + Fe2+(aq) → Mn2+(aq) + Fe3+(aq)


[Fe2+(aq) → Fe3+(aq) + e-] x 5

MnO4-(aq) → Mn2+(aq)
MnO4-(aq) → Mn2+(aq) + 4 H2O(l)
MnO4-(aq) + 8 H+(aq) → Mn2+(aq) + 4 H2O(l)
MnO4-(aq) + 8 H+(aq) + 5 e- → Mn2+(aq) + 4 H2O(l)

Combine the two half reactions.


MnO4-(aq) + 5 Fe2+(aq) + 8 H+(aq) → Mn2+(aq) + 5 Fe3+(aq) + 4 H2O(l)

564
Chapter 18 - Electrochemistry
_____________________________________________________________________________

initial mol Fe2+ = (0.010 mol/L)(0.1000 L) = 0.0010 mol Fe2+


mL MnO4- needed to reach endpoint =
1 mol MnO4_ 1.00 L 1000 mL
0.0010 mol Fe2+ x 2+
x _
x = 20.0 mL
5 mol Fe 0.010 mol MnO4 1.00 L
(a) initial mol Fe2+ = (0.010 mol/L)(0.1000 L) = 0.0010 mol Fe2+
mol MnO4- in 5.0 mL = (0.010 mol/L)(0.0050 L) = 0.000 050 mol MnO4-

MnO4-(aq) + 5 Fe2+(aq) + 8 H+(aq) → Mn2+(aq) + 5 Fe3+(aq) + 4 H2O(l)


before (mol) 0.000 050 0.0010 0
change (mol) -0.000 050 -5(0.000 050) (0.000 050)
after (mol) 0 0.000 75 0.000 25
Again, the Nernst equation can be applied to separate half reactions.
The other half reaction is: Fe3+(aq) + e- → 2 Fe2+(aq) Eo = +0.77 V
E for the half reaction after adding 5.0 mL of MnO4- is
o 0.0592 V [Fe2+ ] 0.0592 V (0.000 75)
E Fe3+ / Fe2+ = E - log 3+
= 0.77 V - log = 0.74 V
n [Fe ] 1 (0.000 25)
(Note in the Nernst equation above, we are taking a ratio of Fe2+ to Fe3+ so we can ignore
volumes and just use moles instead of molarity.)
Ecell = E Fe3+ / Fe2+ + Ecalomel = 0.74 V + (-0.25 V) = 0.49 V
(b) initial mol Fe2+ = (0.010 mol/L)(0.1000 L) = 0.0010 mol Fe2+
mol MnO4- in 10.0 mL = (0.010 mol/L)(0.0100 L) = 0.000 10 mol MnO4-

MnO4-(aq) + 5 Fe2+(aq) + 8 H+(aq) → Mn2+(aq) + 5 Fe3+(aq) + 4 H2O(l)


before (mol) 0.000 10 0.0010 0
change (mol) -0.000 10 -5(0.000 10) +5(0.000 10)
after (mol) 0 0.000 50 0.000 50
E for the half reaction after adding 10.0 mL of MnO4- is
o 0.0592 V [Fe 2+ ] 0.0592 V (0.000 50)
E Fe3+ / Fe2+ = E - log 3+
= 0.77 V - log = 0.77 V
n [Fe ] 1 (0.000 50)
Ecell = E Fe3+ / Fe2+ + Ecalomel = 0.77 V + (-0.25 V) = 0.52 V

(c) initial mol Fe2+ = (0.010 mol/L)(0.1000 L) = 0.0010 mol Fe2+


mol MnO4- in 19.0 mL = (0.010 mol/L)(0.0190 L) = 0.000 19 mol MnO4-

MnO4-(aq) + 5 Fe2+(aq) + 8 H+(aq) → Mn2+(aq) + 5 Fe3+(aq) + 4 H2O(l)


before (mol) 0.000 19 0.0010 0
change (mol) -0.000 19 -5(0.000 19) +5(0.000 19)
after (mol) 0 0.000 05 0.000 95
-
E for the half reaction after adding 19.0 mL of MnO4 is
o 0.0592 V [Fe 2+ ] 0.0592 V (0.000 05)
E Fe / Fe
3+ 2 + = E - log 3+
= 0.77 V - log = 0.85 V
n [Fe ] 1 (0.000 95)
Ecell = E Fe3+ / Fe2+ + Ecalomel = 0.85 V + (-0.25 V) = 0.60 V

565
Chapter 18 - Electrochemistry
_____________________________________________________________________________

(d) 21.0 mL is past the endpoint so the MnO4- is in excess and all of the Fe2+ is
consumed.
initial mol Fe2+ = (0.010 mol/L)(0.1000 L) = 0.0010 mol Fe2+
mol MnO4- in 21.0 mL = (0.010 mol/L)(0.0210 L) = 0.000 21 mol MnO4-

MnO4-(aq) + 5 Fe2+(aq) + 8 H+(aq) → Mn2+(aq) + 5 Fe3+(aq) + 4 H2O(l)


before (mol) 0.000 21 0.0010 0 0
change (mol) -0.000 20 -5(0.000 20) +5(0.000 20)
after (mol) 0.000 01 0 0.000 20 0.0010

Because the Fe2+ is totally consumed, there is a new half reaction:


MnO4-(aq) + 8 H+(aq) + 5 e- → Mn2+(aq) + 4 H2O(l) Eo = 1.51 V
The total volume = 100.0 mL + 21.0 mL = 121.0 mL = 0.1210 L
[MnO4-] = 0.000 01 mol/0.1210 L = 0.000 083 M
[Mn2+] = 0.000 20 mol/0.1210 L = 0.001 65 M
We need to determine [H+] in order to determine the half reaction potential.
[H2SO4]dil ⋅ 121.0 mL = [H2SO4]conc ⋅ 100.0 mL
[H2SO4]dil = [(1.50 M)(100.0 mL)]/121.0 mL = 1.24 M
We can ignore the small amount of H+ consumed by the titration itself, because the
H2SO4 concentration is so large.
Consider the dissociation of H2SO4. From the complete dissociation of the first proton,
[H+] = [HSO4-] = 1.24 M.
For the dissociation of the second proton, the following equilibrium must be considered:
HSO4-(aq) _ H+(aq) + SO42-(aq)
initial (M) 1.24 1.24 0
change (M) -x +x +x
equil (M) 1.24 - x 1.24 + x x
+ 2_
[ ][ ] (1.24 + x)(x)
Ka2 = H SO_ 4 = 1.2 x 10_ 2 =
[HSO 4 ] 1.24 _ x
2
x + 1.252x - 0.0149 = 0
Use the quadratic formula to solve for x.
_ (1.252) ± (1.252 )2 _ 4(1)(_ 0.0149) _ 1.252 ± 1.276
x= =
2(1) 2
x = -1.264 and 0.012
Of the two solutions for x, only the positive value of x has physical meaning, since x is
the [SO42-].
[H+] = 1.24 + x = 1.24 + 0.012 = 1.25 M
o 0.0592 V [Mn 2+]
E MnO4_ / Mn2+ = E - log
n [MnO4_ ][H + ]8
0.0592 V (0.001 65)
= 1.51 V - log = 1.50 V
5 (0.000 083)(1.25 )8
Ecell = E MnO4_ / Mn2+ + Ecalomel = 1.50 V + (-0.25 V) = 1.25 V

566
Chapter 18 - Electrochemistry
_____________________________________________________________________________

Notice that there is a dramatic change in the potential at the equivalence point.

567
568
22 Nuclear Chemistry

22.1 (a) In beta emission, the mass number is unchanged, and the atomic number increases
106 0 106
by one. 44 Ru _ _ 1 e + 45 Rh

(b) In alpha emission, the mass number decreases by four, and the atomic number
189 4 185
decreases by two. 83 Bi _ 2 He + 81 Tl

(c) In electron capture, the mass number is unchanged, and the atomic number
204 0 204
decreases by one. 84 Po + _ 1 e _ 83 Bi

22.2 The mass number decreases by four, and the atomic number decreases by two. This is
214 210 4
characteristic of alpha emission. 90 Th _ 88 Ra + 2 He

0.693 0.693
22.3 t1/2 = = = 64.2 h
k 1.08 x 10_ 2 h _1

0.693 0.693
22.4 k= = = 1.21 x 10-4 y _1
t1/ 2 5730 y

N  t   16,230 y 
22.5 ln  = _ 0.693  = _ 0.693  = -1.963
 N0   t1/ 2   5730 y 
N N
= e-1.963 = 0.140; = 0.140; N = 14.0%
N0 100%

N  t  N Decay rate at time t


22.6 ln   = (_ 0.693)  ; =
 N0   t1/ 2  N0 Decay rate at t = 0
 10,860   28.0 d   28.0 d 
ln   = (_ 0.693)  ; -0.436 = (-0.693)  
 16,800   t1 / 2   t1/ 2 
(_ 0.693)(28.0 d)
t1 / 2 = = 44.5 d
(_ 0.436)

657
Chapter 22 - Nuclear Chemistry
______________________________________________________________________________

N  t 
22.7 ln   = (_ 0.693) 
 N0   t1/ 2 
 10   10 d   10 d 
ln   = (_ 0.693)  ; -2.303 = (-0.693)  
 100   t1/ 2   t1/ 2 
(_ 0.693)(10 d)
t1 / 2 = = 3.0 d
(_ 2.303)

59
22.8 Cu _ 10 e + 59
29 28 Ni

16 Cu → 8 Cu → 4 59Cu → 2 59Cu; three half-lives have passed.


59 59

199 173
22.9 Au has a higher neutron/proton ratio and decays by beta emission. Au has a
lower neutron/proton ratio and decays by alpha emission.

22.10 The shorter arrow pointing right is for beta emission. The longer arrow pointing left is
for alpha emission.
A = 15397+ 97 X = 250
97 Bk
152+ 98 250
B = 98 X = 98 Cf
C = 15096+ 96 X = 246
96 Cm
148+ 94 242
D = 94 X = 94 Pu
E = 14692+ 92 X = 238
92 U

22.11 For 168 O :


First, calculate the total mass of the nucleons (8 n + 8 p)
Mass of 8 neutrons = (8)(1.008 66 amu) = 8.069 28 amu
Mass of 8 protons = (8)(1.007 28 amu) = 8.058 24 amu
Mass of 8 n + 8 p = 16.127 52 amu

Next, calculate the mass of a 16O nucleus by subtracting the mass of 8 electrons from
the mass of a 16O atom.
Mass of 16O atom = 15.994 92 amu
-Mass of 8 electrons = -(8)(5.486 x 10-4 amu) = -0.004 39 amu
16
Mass of O nucleus = 15.990 53 amu
Then subtract the mass of the 16O nucleus from the mass of the nucleons to find the
mass defect:
Mass defect = mass of nucleons - mass of nucleus
= (16.127 52 amu) - (15.990 53 amu) = 0.136 99 amu
Mass defect in g/mol:
(0.136 99 amu)(1.660 54 x 10-24 g/amu)(6.022 x 1023 mol-1) = 0.136 99 g/mol
Now, use the Einstein equation to convert the mass defect into the binding energy.
∆E = ∆mc2 = (0.136 99 g/mol)(10-3 kg/g)(3.00 x 108 m/s)2
∆E = 1.233 x 1013 J/mol = 1.233 x 1010 kJ/mol

658
Chapter 22 - Nuclear Chemistry
______________________________________________________________________________

1.233 x 1013 J/mol 1 MeV 1 nucleus MeV


∆E = 23
x _ 13
x = 8.00
6.022 x 10 nuclei/mol 1.60 x 10 J 16 nucleons nucleon

22.12 ∆E = -852 kJ/mol = -852 x 103 J/mol; 1 J = 1 kg ⋅ m2/s2


 kg • m2 
 _ 852 x 103 2 
2 ∆E  s • mol 
∆E = ∆mc ; ∆m = 2 = = -9.47 x 10-12 kg/mol
c (3.00 x 10 m/s)
8 2

kg 1000 g
∆m = -9.47 x 10-12 x = -9.47 x 10-9 g/mol
mol 1 kg

1
22.13 0n + 235
92 U _
137
52 Te + 97
40 Zr + 2 10 n
235
mass 92 U 235.0439 amu
mass 10 n 1.008 66 amu
-mass 137
52 Te -136.9254 amu
97
-mass 40 Zr -96.9110 amu
-mass 2 10 n -(2)(1.008 66) amu

mass change 0.1988 amu

(0.1988 amu)(1.660 54 x 10-24 g/amu)(6.022 x 1023 mol-1) = 0.1988 g/mol


∆E = ∆mc2 = (0.1988 g/mol)(10-3 kg/g)(3.00 x 108 m/s)2
∆E = 1.79 x 1013 J/mol = 1.79 x 1010 kJ/mol

1
22.14 1H + 12 H _ 3
2 He
mass 1H 1.007 83 amu
mass 2H 2.014 10 amu
-mass 3He -3.016 03 amu

mass change 0.005 90 amu

(0.005 90 amu)(1.660 54 x 10-24 g/amu)(6.022 x1023 mol-1) = 0.005 90 g/mol


∆E = ∆mc2 = (0.005 90 g/mol)(10-3 kg/g)(3.00 x 108 m/s)2
∆E = 5.31 x 1011 J/mol = 5.31 x 108 kJ/mol

40 1 40 1
22.15 18 Ar + 1 p _ 19 K + 0 n

238 2 238
22.16 92 U + 1 H _ 93 Np + 2 10 n

N  t  N Decay rate at time t


22.17 ln   = (_ 0.693)  ; =
 N0   t1/ 2  N0 Decay rate at time t = 0

659
Chapter 22 - Nuclear Chemistry
______________________________________________________________________________

 2.4   t 
ln   = (_ 0.693)  ; t = 1.53 x 104 y
 15.3   5730 y 

22.18 Elements heavier than iron arise from nuclear reactions occuring as a result of
supernova explosions.

Understanding Key Concepts

22.19 16 40K → 8 40K → 4 40K; two half-lives have passed.

14
22.20 The isotope contains 8 neutrons and 6 protons. The isotope symbol is 6 C.
14
6 C would decay by beta emission because the n/p ratio is high.

148 148
22.21 69 Tm decays to 68 Er by either positron emission or electron capture.

22.22 The shorter arrow pointing right is for beta emission. The longer arrow pointing left is
for alpha emission.
A = 14794+ 94 X = 241
94 Pu
146+ 95 241
B = 95 X = 95 Am
C = 14493+ 93 X = 237
93 Np
142+ 91 233
D = 91 X = 91 Pa
E = 14192+ 92
X = 233
92 U

22.23 The half-life is approximately 3 years.

Additional Problems
Nuclear Reactions and Radioactivity

22.24 Positron emission is the conversion of a proton in the nucleus into a neutron plus an
ejected positron.
Electron capture is the process in which a proton in the nucleus captures an inner-shell
electron and is thereby converted into a neutron.

22.25 An alpha particle ( 4


2 He
2+
) is a helium nucleus. The He atom has two electrons and is
neutral.

22.26 Alpha particles move relatively slowly and can be stopped by the skin. However,
inside the body, alpha particles give up their energy to the immediately surrounding
tissue.
Gamma rays move at the speed of light and are very penetrating. Therefore they are
equally hazardous internally and externally.

22.27 "Neutron rich" nuclides emit beta particles to decrease the number of neutrons and

660
Chapter 22 - Nuclear Chemistry
______________________________________________________________________________

increase the number of protons in the nucleus.


"Neutron poor" nuclides decrease the number of protons and increase the n/p ratio by
either alpha emission, positron emission, or electron capture.

22.28 There is no radioactive "neutralization" reaction like there is an acid-base


neutralization reaction.

22.29 The nuclei of 24Na and 24Na+ are identical so their nuclear reactions must be the same
even though their chemical reactions are completely different.

126 0 126 210


22.30 (a) 50 Sn _ _1 e + 51 Sb (b) 88 Ra _ 42 He + 20686 Rn
77 0 77 76
(c) 37 Rb _ 1 e + 36 Kr (d) 36 Kr + _01 e _ 35
76
Br

90
22.31 (a) 38 Sr _ _01 e + 90
39Y (b) 247
100Fm _ 4
He + 243
2 98 Cf
49 0 49 37 0 37
(c) 25 Mn _ 1 e + 24 Cr (d) 18 Ar + _ 1 e _ 17 Cl

188 188 0 218 214 4


22.32 (a) 80 Hg _ 79 Au + 1 e (b) 85 At _ 83 Bi + 2 He
234 234 0
(c) 90 Th _ 91 Pa + _1 e

24 24
22.33 (a) 11 Na _ 12 Mg + _01 e (b) 135
60 Nd _ 135
59 Pr + 0
1 e
170 166 4
(c) 78 Pt _ 76 Os + 2 He

162 158 4 138 0


22.34 (a) 75 Re _ 73 Ta + 2 He (b) 62 Sm + _1 e _ 138
61 Pm
188 188 0 165 165 0
(c) 74 W _ 75 Re + _1 e (d) 73 Ta _ 72 Hf + 1 e

157 157 0 126 0


22.35 (a) 63 Eu _ 64 Gd + _1 e (b) 56 Ba + _1 e _ 126
55 Cs
146 142 4 125 125 0
(c) 62 Sm _ 60 Nd + 2 He (d) 56 Ba _ 55 Cs + 1 e

160 185
22.36 W is neutron poor and decays by alpha emission. W is neutron rich and decays
by beta emission.

136
22.37 53 I is neutron rich and decays by beta emission.
122
53 I is neutron poor and decays by positron emission.

241 237
22.38 95 Am _ 93 Np + 42 He
237 233
93 Np _ 91 Pa + 42 He
233 233
91 Pa _ 92 U + _01 e
233 229
92 U _ 90 Th + 42 He
229 225
90 Th _ 88 Ra + 42 He
225 225
88 Ra _ 89 Ac + _01 e
225 221 4
89 Ac _ 87 Fr + 2 He

661
Chapter 22 - Nuclear Chemistry
______________________________________________________________________________
221 217
87 Fr _ 85 At + 42 He
217 213
85 At _ 83 Bi + 42 He
213 213
83 Bi _ 84Po + _01 e
213 209 4
84 Po _ 82 Pb + 2 He
209 209 0
82 Pb _ 83 Bi + _ 1 e

222 218 4
22.39 86 Rn _ 84 Po + 2 He
218 214 4
84 Po _ 82 Pb + 2 He
214 210 4
82 Pb _ 80 Hg + 2 He
210 210 0
80 Hg _ 81 Tl + _1 e
210 210 0
81 Tl _ 82 Pb + _1 e

22.40 Each alpha emission decreases the mass number by four and the atomic number by
two. Each beta emission increases the atomic number by one.
232 208
90 Th _ 82 Pb

Th mass number _ Pb mass number


Number of α emissions =
4
232 _ 208
= = 6 α emissions
4
The atomic number decreases by 12 as a result of 6 alpha emissions. The resulting
atomic number is (90 - 12) = 78.
Number of β emissions = Pb atomic number - 78 = 82 - 78 = 4 β emissions

22.41 Each alpha emission decreases the mass number by four and the atomic number by
two. Each beta emission increases the atomic number by one.
235 207
92 U _ 82 Pb

U mass number _ Pb mass number


Number of α emissions =
4
235 _ 207
= = 7 α emissions
4
The atomic number decreases by 14 as a result of 7 alpha emissions. The resulting
atomic number is (92 - 14) = 78.
Number of β emissions = Pb atomic number - 78 = 82 - 78 = 4 β emissions

Radioactive Decay Rates

22.42 If the half-life of 59Fe is 44.5 d, it takes 44.5 days for half of the original amount of
59
Fe to decay.

22.43 The half-life is the time it takes for one-half of a radioactive sample to decay.

662
Chapter 22 - Nuclear Chemistry
______________________________________________________________________________

The decay constant is the rate constant for the first order radioactive decay.
0.693
k=
t1/ 2

0.693 0.693
22.44 k= = = 0.247 d-1
t1/ 2 2.805 d

0.693 0.693
22.45 k= = = 8.86 x 10-3 h-1
t1 / 2 78.25 h

0.693 0.693
22.46 t1/ 2 = = = 3.04 d
k 0.228 d _1

0.693 0.693
22.47 t1 / 2 = = = 2.41 x 104 y
k 2.88 x 10_ 5 y _ 1

  65 d  
 
N  t    365 d/y  
22.48 After 65 d: ln  = _ 0.693  = _ 0.693 = -0.000 285 5
 N0   t1/ 2  432.2 y 
 
 
 
N N
= e-0.0002855 = 0.9997; = 0.9997; N = 99.97%
N0 100%

N  t   65 y 
After 65 y: ln  = _ 0.693  = _ 0.693  = -0.1042
 N0   t1/ 2   432.2 y 
N N
= e-0.1042 = 0.9010; = 0.9010; N = 90.10%
N0 100%

N  t   650 y 
After 650 y: ln  = _ 0.693  = _ 0.693  = -1.042
 N0   t1/ 2   432.2 y 
N N
= e-1.042 = 0.3527; = 0.3527; N = 35.27%
N0 100%

N  t   24 min 
22.49 After 24 min: ln   = (_ 0.693)  = (_ 0.693)  = -0.1515
 N0   t1 / 2   109.8 min 
N N
= e-0.1515 = 0.8594; = 0.8594; N = 85.94%
N0 100%

663
Chapter 22 - Nuclear Chemistry
______________________________________________________________________________

 60 min 
N  t   24 h x 
After 24 h: ln   = (_ 0.693)  = (_ 0.693) 1 h  = -9.089
 N0   t1/ 2   109.8 min 
 
 
N N
= e-9.089 = 0.000 113 0; = 0.000 113 0; N = 0.011 30%
N0 100%
 24 h 60 min 
N  t   24 d x x 
After 24 d: ln   = (_ 0.693)  = (_ 0.693) 1d 1 h  = -218.1
 N0   t1/ 2   109.8 min 
 
 
N N
= e-218.1 = 1.861 x 10-95; = 1.861 x 10-95; N = 1.861 x 10-93%
N0 100%

N  t   t 
22.50 ln  = (_ 0.693)  ; ln(0.43) = (_ 0.693)  ; t = 6980 y
 N0   t1/ 2   5730 y 

40
22.51 Assume a sample of 19 K containing 100 atoms.
40
19 K + _01 e _ 18 40
Ar
before decay (atoms) 100 0
after decay (atoms) 100 - x x
40
Ar x
40
= = 1.15; Solve for x. x = 53.5
K 100 _ x
N  t 
ln   = (_ 0.693) 
 N0   t1/ 2 
N = 100 - x = 100 - 53.5 = 46.5, the amount of 40K at time t.
N0 = 100, the original amount of 40K.
 46.5   t 
ln   = (_ 0.693)  ; t = 1.41 x 109 y
 100 
9
 1.28 x 10 y 

0.693 0.693
22.52 t1/ 2 = = = 87.83 d
k 7.89 x 10_ 3 d _1
N  t   185 d 
ln  = (_ 0.693)  = (_ 0.693)  = -1.4597
 N0   t1/ 2   87.83 d 
N N
= e-1.4597 = 0.2323; = 0.2323; N = 23.2%
N0 100%

0.693 0.693
22.53 t1 / 2 = = _5 _1
= 2.41 x 104 y
k 2.88 x 10 y

664
Chapter 22 - Nuclear Chemistry
______________________________________________________________________________

N  t   1000 y 
After 1000 y: ln   = (_ 0.693)  = (_ 0.693) 4
 = -0.028 76
 N0   t1/ 2   2.41 x 10 y 
N N
= e-0.02876 = 0.9717; = 0.9717; N = 97.17%
N0 100%

N  t   25,000 y 
After 25,000 y: ln   = (_ 0.693)  = (_ 0.693) 4
 = -0.7189
 N0   t1 / 2   2.41 x 10 y 
N N
= e-0.7189 = 0.4873; = 0.4873; N = 48.73%
N0 100%
N  t   100,000 y 
After 100,000 y: ln   = (_ 0.693)  = (_ 0.693) 4
 = -2.876
 N0   t1/ 2   2.41 x 10 y 
N N
= e-2.876 = 0.0564; = 0.0564; N = 5.64%
N0 100%

22.54 t1/2 = (102 y)(365 d/y)(24 h/d)(3600 s/h) = 3.2167 x 109 s


0.693 0.693
k= = 9
= 2.1544 x 10-10 s-1
t1/ 2 3.2167 x 10 s
 1 mol Po 
N = (1.0 x 10-9 g)   (6.022 x 1023 atoms/mol) = 2.881 x 1012 atoms
 209 g Po 
Decay rate = kN = (2.1544 x 10-10 s-1)(2.881 x 1012 atoms) = 6.21 x 102 s-1
621 α particles are emitted in 1.0 s.

22.55 t1/2 = (3.0 x 105 y)(365 d/y)(24 h/d)(60 min/h) = 1.6 x 1011 min
0.693 0.693
k= = 11
= 4.3 x 10-12 min-1
t1/ 2 1.6 x 10 min
 1 mol 36 Cl 
N = (5.0 x 10-3 g)   (6.022 x 1023 atoms/mol) = 8.4 x 1019 atoms
 36 g 
Decay rate = kN = (4.3 x 10 min-1)(8.4 x 1019 atoms) = 3.6 x 108 min-1
-12

 1 min  1 Ci 
Curies = (3.6 x 108/min)  
-4
 = 1.6 x 10 Ci
 60 s  3.7 x 10 /s 
10

22.56 Decay rate = kN


 1 mol 79 Se 
N = (1.0 x 10-3 g)   (6.022 x 1023 atoms/mol) = 7.6 x 1018 atoms
 79 g 
5
Decay rate 1.5 x 10 /s
k= = 18
= 2.0 x 10-14 s-1
N 7.6 x 10
0.693 0.693
t1 / 2 = = _ 14 _ 1
= 3.5 x 1013 s
k 2.0 x 10 s

665
Chapter 22 - Nuclear Chemistry
______________________________________________________________________________

 1 h  1 d  1 y  6
t1/ 2 = (3.5 x 10 s)    = 1.1 x 10 y
13

 3600 s  24 h  365 d 

22.57 Decay rate = kN


 1 mol Ti 
N = (1.0 x 10-9 g)   (6.022 x 1023 atoms/mol) = 1.37 x 1013 atoms
 44 g Ti 
Decay rate 4.8 x 103 s _1
k= = 13
= 3.50 x 10-10 s-1
N 1.37 x 10
k = (3.50 x 10 s )(3600 s/h)(24 h/d)(365 d/y) = 1.10 x 10-2 y-1
-10 -1

0.693 0.693
t1/2 = = _1
= 63 y
k 1.10 x 10_ 2 y

N  t  N Decay rate at time t


22.58 ln   = (_ 0.693)  ; =
 N0   t1/ 2  N0 Decay rate at time t = 0
 6990   10.0 d 
ln   = (_ 0.693)  ; t1/ 2 = 34.6 d
 8540   t1/ 2 

N  t  N Decay rate at time t


22.59 ln  = (_ 0.693)  ; =
 N0   t1/ 2  N0 Decay rate at time t = 0
 10,980   48.0 h 
ln  = (_ 0.693)  ; t1/2 = 21.0 h
 53,500   t1/ 2 

Energy Changes During Nuclear Reactions

22.60 The loss in mass that occurs when protons and neutrons combine to form a nucleus is
called the mass defect. The lost mass is converted into the binding energy that is used
to hold the nucleons together.

22.61 Energy (heat) is absorbed in an endothermic reaction. The energy is converted to


mass. The mass of the products is slightly larger than the mass of the reactants.

 1.60 x 10_13 J 
22.62 E = (1.50 MeV)   = 2.40 x 10-13 J
 1 MeV 
hc (6.626 x 10_ 34 Jcdot s)(3.00 x 108 m/s)
λ= = _ 13
= 8.28 x 10-13 m = 0.000 828 nm
E 2.40 x 10 J

 1 MeV  1.60 x 10_13 J 


22.63 E = (6.82 keV)  3   = 1.09 x 10-15 J
 10 keV  1 MeV 

666
Chapter 22 - Nuclear Chemistry
______________________________________________________________________________

E 1.09 x 10_15 J
ν= = = 1.65 x 1018/s = 1.65 x 1018 Hz
h 6.626 x 10 J • s
_ 34

22.64 (a) For 52 26 Fe :

First, calculate the total mass of the nucleons (26 n + 26 p)


Mass of 26 neutrons = (26)(1.008 66 amu) = 26.225 16 amu
Mass of 26 protons = (26)(1.007 28 amu) = 26.189 28 amu
Mass of 26 n + 26 p = 52.414 44 amu
Next, calculate the mass of a 52Fe nucleus by subtracting the mass of 26 electrons from
the mass of a 52Fe atom.
Mass of 52Fe atom = 51.948 11 amu
-4
-Mass of 26 electrons = -(26)(5.486 x 10 amu) = -0.014 26 amu
Mass of 52Fe nucleus = 51.933 85 amu
Then subtract the mass of the 52Fe nucleus from the mass of the nucleons to find the
mass defect:
Mass defect = mass of nucleons - mass of nucleus
= (52.414 44 amu) - (51.933 85 amu) = 0.480 59 amu
Mass defect in g/mol:
(0.480 59 amu)(1.660 54 x 10-24 g/amu)(6.022 x 1023 mol-1) = 0.480 59 g/mol

(b) For 92 42 Mo :

First, calculate the total mass of the nucleons (50 n + 42 p)


Mass of 50 neutrons = (50)(1.008 66 amu) = 50.433 00 amu
Mass of 42 protons = (42)(1.007 28 amu) = 42.305 76 amu
Mass of 50 n + 42 p = 92.738 76 amu
92
Next, calculate the mass of a Mo nucleus by subtracting the mass of 42 electrons
from the mass of a 92Mo atom.
Mass of 92Mo atom = 91.906 81 amu
-Mass of 42 electrons = -(42)(5.486 x 10-4 amu) = -0.023 04 amu
Mass of 92Mo nucleus = 91.883 77 amu
Then subtract the mass of the 92Mo nucleus from the mass of the nucleons to find the
mass defect:
Mass defect = mass of nucleons - mass of nucleus
= (92.738 76 amu) - (91.883 77 amu) = 0.854 99 amu
Mass defect in g/mol:
(0.854 99 amu)(1.660 54 x 10-24 g/amu)(6.022 x 1023 mol-1) = 0.854 99 g/mol

32
22.65 (a) For 16 S:
First, calculate the total mass of the nucleons (16 n + 16 p)
Mass of 16 neutrons = (16)(1.008 66 amu) = 16.138 56 amu
Mass of 16 protons = (16)(1.007 28 amu) = 16.116 48 amu
Mass of 16 n + 16 p = 32.255 04 amu
Next, calculate the mass of a 32S nucleus by subtracting the mass of 16 electrons from
the mass of a 32S atom.

667
Chapter 22 - Nuclear Chemistry
______________________________________________________________________________

Mass of 32S = 31.972 07 amu


Mass of 16 electrons = -(16)(5.486 x 10-4 amu) = -0.008 78 amu
Mass of 32S nucleus = 31.963 29 amu
Then subtract the mass of the 32S nucleus from the mass of the nucleons to find the
mass defect:
Mass defect = mass of nucleons - mass of nucleus
= (32.255 04 amu) - (31.963 29 amu) = 0.291 75 amu
Mass defect in g/mol:
(0.291 75 amu)(1.660 54 x 10-24 g/amu)(6.022 x 1023 mol-1) = 0.291 74 g/mol

(b) For 40 20 Ca :

First, calculate the total mass of the nucleons (20 n + 20 p)


Mass of 20 neutrons = (20)(1.008 66 amu) = 20.173 20 amu
Mass of 20 protons = (20)(1.007 28 amu) = 20.145 60 amu
Mass of 20 n + 20 p = 40.318 80 amu
40
Next, calculate the mass of a Ca nucleus by subtracting the mass of 20 electrons from
the mass of a 40Ca atom.
Mass of 40Ca = 39.962 59 amu
-4
-Mass of 20 electrons = -(20)(5.486 x 10 amu) = -0.010 97 amu
Mass of 40Ca nucleus = 39.951 62 amu
40
Then substract the mass of the Ca nucleus from the mass of the nucleons to find the
mass defect:
Mass defect = mass of nucleons - mass of nucleus
= (40.318 80 amu) - (39.951 62 amu) = 0.367 18 amu
Mass defect in g/mol:
(0.367 18 amu)(1.660 54 x 10-24 g/amu)(6.022 x 1023 mol-1) = 0.367 17 g/mol

22.66 (a) For 5828 Ni :

First, calculate the total mass of the nucleons (30 n + 28 p)


Mass of 30 neutrons = (30)(1.008 66 amu) = 30.259 80 amu
Mass of 28 protons = (28)(1.007 28 amu) = 28.203 84 amu
Mass of 30 n + 28 p = 58.463 64 amu
Next, calculate the mass of a 58Ni nucleus by subtracting the mass of 28 electrons from
the mass of a 58Ni atom.
Mass of 58Ni atom = 57.935 35 amu
-4
-Mass of 28 electrons = -(28)(5.486 x 10 amu) = -0.015 36 amu
Mass of 58Ni nucleus = 57.919 99 amu
Then subtract the mass of the 58Ni nucleus from the mass of the nucleons to find the
mass defect:
Mass defect = mass of nucleons - mass of nucleus
= (58.463 64 amu) - (57.919 99 amu) = 0.543 65 amu

668
Chapter 22 - Nuclear Chemistry
______________________________________________________________________________

Mass defect in g/mol:


(0.543 65 amu)(1.660 54 x 10-24 g/amu)(6.022 x 1023 mol-1) = 0.543 65 g/mol
Now, use the Einstein equation to convert the mass defect into the binding energy.
∆E = ∆mc2 = (0.543 65 g/mol)(10-3 kg/g)(3.00 x 108 m/s)2
∆E = 4.893 x 1013 J/mol = 4.893 x 1010 kJ/mol
4.893 x 1013 J/mol 1 MeV 1 nucleus MeV
∆E = 23
x _ 13
x = 8.76
6.022 x 10 nuclei/mol 1.60 x 10 J 58 nucleons nucleon

(b) For 84 36 Kr :

First, calculate the total mass of the nucleons (48 n + 36 p)


Mass of 48 neutrons = (48)(1.008 66 amu) = 48.415 68 amu
Mass of 36 protons = (36)(1.007 28 amu) = 36.262 08 amu
Mass of 48 n + 36 p = 84.677 76 amu
84
Next, calculate the mass of a Kr nucleus by subtracting the mass of 36 electrons from
the mass of a 84Kr atom.
Mass of 84Kr atom = 83.911 51 amu
-4
-Mass of 36 electrons = -(36)(5.486 x 10 amu) = -0.019 75 amu
Mass of 84Kr nucleus = 83.891 76 amu
Then subtract the mass of the 84Kr nucleus from the mass of the nucleons to find the
mass defect:
Mass defect = mass of nucleons - mass of nucleus
= (84.677 76 amu) - (83.891 76 amu) = 0.786 00 amu
Mass defect in g/mol:
(0.786 00 amu)(1.660 54 x 10-24 g/mol)(6.022 x 1023 mol-1) = 0.786 00 g/mol
Now, use the Einstein equation to convert the mass defect into the binding energy.
∆E = ∆mc2 = (0.786 00 g/mol)(10-3 kg/g)(3.00 x 108 m/s)2
∆E = 7.074 x 1013 J/mol = 7.074 x 1010 kJ/mol
7.074 x 1013 J/mol 1 MeV 1 nucleus MeV
∆E = 23
x _ 13
x = 8.74
6.022 x 10 nuclei/mol 1.60 x 10 J 84 nucleons nucleon

22.67 (a) For 6329 Cu :

First, calculate the total mass of the nucleons (34 n + 29 p)


Mass of 34 neutrons = (34)(1.008 66 amu) = 34.294 44 amu
Mass of 29 protons = (29)(1.007 28 amu) = 29.211 12 amu
Mass of 34 n + 29 p = 63.505 56 amu
Next calculate the mass of a 63Cu nucleus by subtracting the mass of 29 electrons from
the mass of a 63Cu atom.
Mass of 63Cu atom = 62.939 60 amu
-4
-Mass of 29 electrons = -(29)(5.486 x 10 amu) = -0.015 91 amu
Mass of 63Cu nucleus = 62.923 69 amu
63
Then subtract the mass of the Cu nucleus from the mass of the nucleons to find the
mass defect:
Mass defect = mass of nucleons - mass of nucleus
= (63.505 56 amu) - (62.923 69 amu) = 0.581 87 amu

669
Chapter 22 - Nuclear Chemistry
______________________________________________________________________________

Mass defect in g/mol:


(0.581 87 amu)(1.660 54 x 10-24 g/amu)(6.022 x 1023 mol-1) = 0.581 86 g/mol
Now, use the Einstein equation to convert the mass defect into the binding energy.
∆E = ∆mc2 = (0.581 86 g/mol)(10-3 kg/g)(3.00 x 108 m/s)2
∆E = 5.237 x 1013 J/mol = 5.237 x 1010 kJ/mol
5.237 x 1013 J/mol 1 MeV 1 nucleus MeV
∆E = 23
x _ 13
x = 8.63
6.022 x 10 nuclei/mol 1.60 x 10 J 63 nucleons nucleon

(b) For 84 38 Sr :

First, calculate the total mass of the nucleons (46 n + 38 p)


Mass of 46 neutrons = (46)(1.008 66 amu) = 46.398 36 amu
Mass of 38 protons = (38)(1.007 28 amu) = 38.276 64 amu
Mass of 46 n + 38 p = 84.675 00 amu
84
Next, calculate the mass of a Sr nucleus by subtracting the mass of 38 electrons from
the mass of a 84Sr atom.
Mass of 84Sr atom = 83.913 43 amu
-4
-Mass of 38 electrons = -(38)(5.486 x 10 amu) = -0.020 85 amu
Mass of 84Sr nucleus = 83.892 58 amu
84
Then subtract the mass of the Sr nucleus from the mass of the nucleons to find the
mass defect:
Mass defect = mass of nucleons - mass of nucleus
= (84.675 00 amu) - (83.892 58 amu) = 0.782 42 amu
Mass defect in g/mol:
(0.782 42 amu)(1.660 54 x 10-24 g/amu)(6.022 x 1023 mol-1) = 0.782 40 g/mol
Now, use the Einstein equation to convert the mass defect into the binding energy.
∆E = ∆mc2 = (0.782 40 g/mol)(10-3 kg/g)(3.00 x 108 m/s)2
∆E = 7.042 x 1013 J/mol = 7.042 x 1010 kJ/mol
7.042 x 1013 J/mol 1 MeV 1 nucleus MeV
∆E = 23
x _ 13
x = 8.70
6.022 x 10 nuclei/mol 1.60 x 10 J 84 nucleons nucleon

174
22.68 Ir _ 170
77
4
75 Re + 2 He

mass 174
77 Ir 173.966 66 amu
-mass 170
75 Re -169.958 04 amu
4
-mass 2 He - 4.002 60 amu

mass change 0.006 02 amu

(0.006 02 amu)(1.660 54 x 10-24 g/amu)(6.022 x 1023 mol-1) = 0.006 02 g/mol


∆E = ∆mc2 = (0.006 02 g/mol)(10-3 kg/g)(3.00 x 108 m/s)2
∆E = 5.42 x 1011 J/mol = 5.42 x 108 kJ/mol

28 28 0
22.69 12 Mg _ 13 Al + _1 e

670
Chapter 22 - Nuclear Chemistry
______________________________________________________________________________

Reactant: 28
12 Mg nucleus = 1228
Mg atom - 12 e-
Product: 28
13 Al nucleus + e- = ( 13
28
Al nucleus - 13 e-) + e- = 28
13 Al nucleus - 12 e-

Change : ( 28
12 Mg atom - 12 e-) - ( 28
13 Al nucleus - 12 e-) =
Mg atom - 13 28
Al atom 28
12

(electrons cancel)
Mass change = 27.983 88 amu - 27.981 91 amu = 0.001 97 amu

(0.001 97 amu)(1.660 54 x 10-24 g/amu)(6.022 x 1023 mol-1) = 0.001 97 g/mol


∆E = ∆mc2 = (0.001 97 g/mol)(10-3 kg/g)(3.00 x 108 m/s)2
∆E = 1.77 x 1011 J/mol = 1.77 x 108 kJ/mol

∆E 92.2 x 103 J 92.2 x 103 kg• m 2 / s2


22.70 ∆m = 2
= 8 2
= 8 2
= 1.02 x 10-12 kg
c (3.00 x 10 m/s ) (3.00 x 10 m/s )
-9
∆m = 1.02 x 10 g

∆E 131 x 103 J 131 x 103 kg• m 2 / s2


22.71 ∆m = 2
= 8 2
= 8 2
= 1.46 x 10-12 kg
c (3.00 x 10 m/s ) (3.00 x 10 m/s )
-9
∆m = 1.46 x 10 g

22.72 Mass of positron and electron


= 2(9.109 x 10-31 kg)(6.022 x 1023 mol-1) = 1.097 x 10-6 kg/mol
∆E = ∆mc2 = (1.097 x 10-6 kg/mol)(3.00 x 108 m/s)2
∆E = 9.87 x 1010 J/mol = 9.87 x 107 kJ/mol

22.73 2 12 H _ 32 He + 1
0 n
mass 2 12 H 2(2.0141) amu
-mass 32 He -3.0160 amu
-mass 10 n -1.008 66 amu

mass change 0.003 54 amu

(0.003 54 amu)(1.660 54 x 10-24 g/mol)(6.022 x 1023 mol-1) = 0.003 54 g/mol


∆E = ∆mc2 = (0.003 54 g/mol)(10-3 kg/g)(3.00 x 108 m/s)2
∆E = 3.2 x 1011 J/mol = 3.2 x 108 kJ/mol

Nuclear Transmutation

109 4 113 10 4 13 1
22.74 (a) 47 Ag + 2 He _ 49 In (b) 5 B + 2 He _ 7 N + 0 n

671
Chapter 22 - Nuclear Chemistry
______________________________________________________________________________
235 160 72
22.75 (a) 92 U _ 62 Sm + 30 Zn + 3 10 n

235 87 146
(b) 92 U _ 35 Br + 57 La + 2 10 n

209 58 266 1
22.76 83 Bi + 26 Fe _ 109 Mt + 0 n

98 1 99
22.77 42 Mo + 0 n _ 42 Mo

238 12 246 1
22.78 92 U + 6 C _ 98 Cf + 4 0 n

246 12 254
22.79 (a) 96 Cm + 6 C _ 102 No + 4 10 n

253 4 256 1
(b) 99 Es + 2 He _ 101 Md + 0 n

250 11 257
(c) 98 Cf + 5 B _ 103 Lr + 4 10 n

General Problems

232 208
22.80 90Th _ 82 Pb + 6 42 He + 4 _01 e
232 232 -
Reactant: 90 Th nucleus = 90 Th atom - 90 e

-
Product: 208 4
82 Pb nucleus + (6) ( 2 He nucleus) + 4 e
- - -
= ( 208 4
82 Pb atom - 82 e ) + (6)( 2 He atom - 2 e ) + 4 e
-
= 208 4
82 Pb atom + (6)( 2 He atom) - 90 e

- -
Change: ( 232 208 4
90 Th atom - 90 e ) - [ 82 Pb atom + (6)( 2 He atom) - 90 e ]

= 232 208 4
90 Th atom - [ 82 Pb atom + (6)( 2 He atom)] (electrons cancel)
Mass change = 232.038 054 amu - [207.976 627 amu + (6)(4.002 603 amu)]
= 0.045 809 amu

(0.045 809 amu)(1.660 54 x 10-24 g/amu)(6.022 x 1023 mol-1) = 0.045 809 g/mol
∆E = ∆mc2 = (0.045 809 g/mol)(10-3 kg/g)(3.00 x 108 m/s)2
∆E = 4.12 x 1012 J/mol = 4.12 x 109 kJ/mol

22.81 Decay rate = kN


0.693 0.693
k= = 9
= 5.41 x 10-10 y-1
t1 / 2 1.28 x 10 y
KCl, 74.55 amu
N = number of 40K+ ions in a 1.00 g sample of KCl
 1 mol KCl  1 mol K + 
N = (0.000 117)(1.00 g)    (6.022 x 1023 mol-1)
 74.55 g  1 mol KCl 

672
Chapter 22 - Nuclear Chemistry
______________________________________________________________________________

N = 9.45 x 1017 40K+ ions


Decay rate = kN = (5.41 x 10-10 y-1)(9.45 x 1017) = 5.11 x 108/y
 1 y  1 d  1 h 
Disintegration/s = (5.11 x 108/y)     = 16.2/s
 365 d  24 h  3600 s 

241
22.82 94Pu _ 233 4
92 U + 2 2 He + 2 _ 1 e
0

-
Reactant: 241 241
94 Pu nucleus = 94 Pu atom - 94 e
-
Product: 233 4
92 U nucleus + (2)( 2 He nucleus) + 2 e
- - -
= ( 233 4
92 U atom - 92 e ) + (2)( 2 He atom - 2 e ) + 2 e
-
= 233 4
92 U atom + (2)( 2 He atom) - 94 e

Change: ( 241
94 Pu atom - 94 e-) - [ 233 4 -
92 U atom + (2)( 2 He atom) - 94 e ]
241
= 94 Pu atom - [ 233 4
92 U atom + (2)( 2 He atom)]

Mass change = 241.056 845 amu - [233.039 628 amu + (2)(4.002 603 amu)]
= 0.012 011 amu

(0.012 011 amu)(1.660 54 x 10-24 g/amu)(6.022 x 1023 mol-1) = 0.012 011 g/mol
∆E = ∆mc2 = (0.012 011 g/mol)(10-3 kg/g)(3.00 x 108 m/s)2
∆E = 1.08 x 1012 J/mol = 1.08 x 109 kJ/mol

293 289 285


22.83 118 X, 116 Y , and 114 Z

N  t  N Decay rate at time t


22.84 ln  = (_ 0.693)  ; =
 N0   t1/ 2  N0 Decay rate at time t = 0
 100 _ 99.99 
ln  = (_ 0.693)(tover1.53 s ) ; t = 20.3 s
 100 

0.693 0.693
22.85 t1 / 2 = = = 11 s
k 0.063 s _1
N  t  N Decay rate at time t
ln   = (_ 0.693)   ; =
 N0   t1 / 2  N 0 Decay rate at time t = 0
 100 _ 99.99   t 
ln   = (_ 0.693) ; t = 150 s
 100   11 s 

22.86 (a) For 5024 Cr :

First, calculate the total mass of the nucleons (26 n + 24 p)


Mass of 26 neutrons = (26)(1.008 66 amu) = 26.225 16 amu
Mass of 24 protons = (24)(1.007 28 amu) = 24.174 72 amu
Mass of 26 n + 24 p = 50.399 88 amu
Next, calculate the mass of a 50Cr nucleus by subtracting the mass of 24 electrons from

673
Chapter 22 - Nuclear Chemistry
______________________________________________________________________________

the mass of a 50Cr atom.


Mass of 50Cr atom = 49.946 05 amu
-4
-Mass of 24 electrons = -(24)(5.486 x 10 amu) = -0.013 17 amu
Mass of 50Cr nucleus = 49.932 88 amu
Then subtract the mass of the 50Cr nucleus from the mass of the nucleons to find the
mass defect:
Mass defect = mass of nucleons - mass of nucleus
= (50.399 88 amu) - (49.932 88 amu) = 0.467 00 amu
Mass defect in g/mol:
(0.467 00 amu)(1.660 54 x 10-24 g/amu)(6.022 x 1023 mol-1) = 0.467 00 g/mol
Now, use the Einstein equation to convert the mass defect into the binding energy.
∆E = ∆mc2 = (0.467 00 g/mol)(10-3 kg/g)(3.00 x 108 m/s)2
∆E = 4.203 x 1013 J/mol = 4.203 x 1010 kJ/mol
4.203 x 1013 J/mol 1 MeV 1 nucleus MeV
∆E = 23
x _ 13
x = 8.72
6.022 x 10 nuclei/mol 1.60 x 10 J 50 nucleons nucleon

64
(b) For 30 Zn :
First, calculate the total mass of the nucleons (34 n + 30 p)
Mass of 34 neutrons = (34)(1.008 66 amu) = 34.294 44 amu
Mass of 30 protons = (30)(1.007 28 amu) = 30.218 40 amu
Mass of 34 n + 30 p = 64.512 84 amu
Next, calculate the mass of a 64Zn nucleus by subtracting the mass of 30 electrons from
the mass of a 64Zn atom.
Mass of 64Zn atom = 63.929 15 amu
-Mass of 30 electrons = -(30)(5.486 x 10-4 amu) = -0.016 46 amu
64
Mass of Zn nucleus = 63.912 69 amu
Then subtract the mass of the 64Zn nucleus from the mass of the nucleons to find the
mass defect:
Mass defect = mass of nucleons - mass of nucleus
= (64.512 84 amu) - (63.912 69 amu) = 0.600 15 amu
Mass defect in g/mol:
(0.600 15 amu)(1.660 54 x 10-24 g/amu)(6.022 x 1023 mol-1) = 0.600 15 g/mol
Now, use the Einstein equation to convert the mass defect into the binding energy.
∆E = ∆mc2 = (0.600 15 g/mol)(10-3 kg/g)(3.00 x 108 m/s)2
∆E = 5.401 x 1013 J/mol = 5.401 x 1010 kJ/mol
5.401 x 1013 J/mol 1 MeV 1 nucleus MeV
∆E = 23
x _ 13
x = 8.76
6.022 x 10 nuclei/mol 1.60 x 10 J 64 nucleons nucleon

The 64Zn is more stable.

674
Chapter 22 - Nuclear Chemistry
______________________________________________________________________________

N  t  N Decay rate at time t


22.87 ln   = (_ 0.693)  ; =
 N0   t1/ 2  N0 Decay rate at time t = 0
 2.9   t 
ln   = (_ 0.693)  ; t = 1.38 x 104 y
 15.3   5730 y 

2
22.88 1H + 32 He _ 4
2 He + 1
1 H
mass 12 H 2.0141 amu
mass 32 He 3.0160 amu
-mass 42 He - 4.0026 amu
-mass 11 H -1.0078 amu

mass change 0.0197 amu

(0.0197 amu)(1.660 54 x 10-24 g/amu)(6.022 x 1023 mol-1) = 0.0197 g/mol


∆E = ∆mc2 = (0.0197 g/mol)(10-3 kg/g)(3.00 x 108 m/s)2
∆E = 1.77 x 1012 J/mol = 1.77 x 109 kJ/mol
22.89 t1/2 = 1.1 x 1020 y = (1.1 x 1020 y)(365 d/y) = 4.0 x 1022 d
0.693 0.693
k= = 22
= 1.7 x 10-23 d-1
t1 / 2 4.0 x 10 d
N = 6.02 x 1023 atoms
Decay rate = kN = (1.7 x 10-23 d-1)(6.02 x 1023 atoms) = 10/d
There are 10 disintegrations per day.

238 1 239 0
22.90 92 U + 0 n _ 94 Pu + 2 _1 e

22.91 3.9 x 1023 kJ = 3.9 x 1026 J = 3.9 x 1026 kg ⋅ m2/s2


∆ E 3.9 x 1026 kg • m2 / s2
∆E = ∆mc2; ∆m = 2 = 8 2
= 4.3 x 109 kg
c (3.00 x 10 m/s )
The sun loses mass at a rate of 4.3 x 109 kg/s.

22.92 10
B + 1n → 4He + 7Li + γ

mass 10B 10.012 937 amu


mass 1n 1.008 665 amu
-mass 4He - 4.002 603 amu
-mass 7Li -7.016 004 amu
mass change 0.002 995 amu

(0.002 995 amu)(1.660 54 x 10-24 g/amu) = 4.973 x 10-27 g


∆E = ∆mc2 = (4.973 x 10-27 g)(10-3 kg/g)(3.00 x 108 m/s)2 = 4.476 x 10-13 J

675
Chapter 22 - Nuclear Chemistry
______________________________________________________________________________

1.60 x 10_13 J
Kinetic energy = 2.31 MeV x = 3.696 x 10-13 J
1 MeV
γ photon energy = ∆E - KE = 4.476 x 10-13 J - 3.696 x 10-13 J = 7.80 x 10-14 J

22.93 Each alpha emission decreases the mass number by four and the atomic number by
two. Each beta emission increases the atomic number by one.
237 209
93 Np _ 83 Bi

Np mass number _ Bi mass number


Number of α emissions =
4
237 _ 209
= = 7 α emissions
4
The atomic number decreases by 14 as a result of 7 alpha emissions. The resulting
atomic number is (93 - 14) = 79.
Number of β emissions = Bi atomic number - 79 = 83 - 79 = 4 β emissions

100
22.94 (a) Tc _ 10 e + 100
43 42 Mo (positron emission)
100 0 100
Tc + _1 e _ 42 Mo (electron capture)
43

(b) Positron emission


-
Reactant: 100 100
43 Tc nucleus = 43 Tc atom - 43 e
+ - +
Product: 100
42 Mo nucleus + e =
100
42 Mo atom - 42 e + 1 e

Change: ( 100
43 Tc atom - 43 e-) - ( 100 - +
42 Mo atom - 42 e + 1 e )
100 -
= 43 Tc atom - 100
42 Mo atom - 2 e

Mass change = 99.907 657 amu - 99.907 48 amu - (2)(0.000 5486 amu)
= -0.000 92 amu

(-0.000 92 amu)(1.660 54 x 10-24 g/amu)(6.022 x 1023 mol-1) = -0.000 92 g/mol


∆E = ∆mc2 = (-0.000 92 g/mol)(10-3 kg/g)(3.00 x 108 m/s)2
∆E = -8.3 x 1010 J/mol = -8.3 x 107 kJ/mol

Electron Capture
- -
Reactant: 100 100
43 Tc nucleus + e = 43 Tc atom - 42 e
-
Product: 100 100
42 Mo nucleus = 42 Mo atom - 42 e

Change: ( 100
43 Tc atom - 42 e-) - ( 100 -
42 Mo atom - 42 e )
100
= 43 Tc atom - 100
42 Mo atom (electrons cancel)

Mass change = 99.907 657 amu - 99.907 48 amu = 0.000 177 amu

676
Chapter 22 - Nuclear Chemistry
______________________________________________________________________________

(0.000 177 amu)(1.660 54 x 10-24 g/amu)(6.022 x 1023 mol-1) = 0.000 177 g/mol
∆E = ∆mc2 = (0.000 177 g/mol)(10-3 kg/g)(3.00 x 108 m/s)2
∆E = 1.6 x 1010 J/mol = 1.6 x 107 kJ/mol

Only electron capture is observed because there is a mass decrease and a release of
energy.

226
22.95 (a) α emission: 89 Ac _ 222
87 Fr + 2 He
4

226
β emission: 89 Ac _ 22690 Th + _ 1 e
0

electron capture: 226 0


89 Ac + _ 1 e _
226
88 Ra

0.693 0.693
(b) t1/ 2 = = = 1.25 d
k 0.556 d _1
If 80% reacts, then 20% is left.
N  t   20   t 
ln  = (_ 0.693)  ; ln  = (_ 0.693) 
 N0   t1/ 2   100   1.25 d 
 20 
ln (1.25 d)
t=  100 
= 2.90 d
(_ 0.693)

Multi-Concept Problems

22.96 BaCO3, 197.34 amu


1 mol BaCO3 1 mol C 12.011 g C
1.000 g BaCO3 x x x = 0.060 86 g C
197.34 g BaCO3 1 mol BaCO3 1 mol C
4.0 x 10-3 Bq = 4.0 x 10-3 disintegrations/s
(4.0 x 10-3 Bq = 4.0 x 10-3 disintegrations/s)(60 s/min) = 0.24 disintegrations/min

0.24 disintegrations/min
sample radioactivity = = 3.94 disintegrations/min per gram of
0.060 86 g C
C
N  t  N Decay rate at time t
ln  = (_ 0.693)  ; =
 N0   t1/ 2  N0 Decay rate at time t = 0
 3.94   t 
ln  = (_ 0.693)  ; t = 11,000 y
 15.3   5730 y 

1y
22.97 t1/2 = 138 d = 138 d x = 0.378 y
365 d
0.693 0.693
k= = = 1.83 y-1
t1 / 2 0.378 y

677
Chapter 22 - Nuclear Chemistry
______________________________________________________________________________

1 x 10_ 3 g
0.700 mg x = 7.00 x 10-4 g
1 mg
 1 mol Po 
No = (7.00 x 10-4 g)   (6.022 x 1023 atoms/mol) = 2.01 x 1018 atoms
 210 g Po 
N N
ln   = -kt = - (1.83 y-1)(1 y) = -1.83; = e-1.83 = 0.160
 No  No
N = 0.160 No = (0.160)(2.01 x 10 atoms) = 0.322 x 1018 atoms
18

atoms He = atoms Po decayed


atoms He = 2.01 x 1018 atoms - 0.322 x 1018 atoms = 1.688 x 1018 atoms
1.688 x 1018 He atoms
mol He = = 2.80 x 10-6 mol He
6.022 x 1023 atoms/mol

20oC = 293 K
 L • atm 
(2.80 x 10_ 6 mol) 0.082 06 (293 K)
nRT  K • mol 
P= = = 2.69 x 10-4 atm
V 0.2500 L
760 mm Hg
P = 2.69 x 10-4 atm x = 2.05 mm Hg
1.00 atm

22.98 First find the activity of the 51Cr after 17.0 days.
N  t  N Decay rate at time t
ln  = (_ 0.693)  ; =
 N0   t1/ 2  N0 Decay rate at time t = 0
 N   17.0 d 
ln  = (_ 0.693) 
 4.10   27.7 d 
ln N - ln(4.10) = -0.4253
ln N = -0.4253 + ln(4.10) = 0.9857
N = e0.9857 = 2.68 µCi/mL

(20.0 mL)(2.68 µCi/mL) = (total blood volume)(0.009 35 µCi/mL)


total blood volume = 5732 mL = 5.73 L

22.99 First find the activity (N) that the 28Mg would have after 2.4 hours assuming that none
of it was removed by precipitation as MgCO3.
N  t  N Decay rate at time t
ln  = (_ 0.693)  ; =
 N0   t1/ 2  N0 Decay rate at time t = 0
 N   2.40 h 
ln  = (_ 0.693) 
 0.112   20.91 h 
ln N - ln(0.112) = -0.0795

678
Chapter 22 - Nuclear Chemistry
______________________________________________________________________________

ln N = - 0.0795 + ln(0.112) = -2.27


N = e-2.27 = 0.103 µCi/mL

20.00 mL = 0.020 00 L and 15.00 mL = 0.015 00 L


mol MgCl2 = (0.007 50 mol/L)(0.020 00 L) = 1.50 x 10-4 mol MgCl2
mol Na2CO3 = (0.012 50 mol/L)(0.015 00 L) = 1.87 x 10-4 mol Na2CO3
The mol of CO32- are in excess, so assume that all of the Mg2+ precipitates as MgCO3
according to the reaction:
Mg2+(aq) + CO32-(aq) → MgCO3(s)
initial (mol) 0.000 150 0.000 187 0
change (mol) - 0.000 150 - 0.000 150 + 0.000 150
final (mol) 0 0.000 037 0.000 150
[CO32-] = 0.000 037 mol/(0.020 00 L + 0.015 00 L) = 0.001 06 M

Now consider the dissolution of MgCO3 in the presence of CO32-.


MgCO3(s) _ Mg2+(aq) + CO32-(aq)
initial (M) 0 0.001 06
change (M) +x +x
equil (M) x 0.001 06 + x

The [Mg2+] in the filtrate is proportional to its activity after 2.40 h.


0.007 50 M
x = [Mg2+] = 0.029 µCi/mL x = 0.002 11 M
0.103 µ Ci/mL
[CO32-] = 0.001 06 + x = 0.001 06 + 0.002 11 = 0.003 17 M
Ksp = [Mg2+][CO32-] = (0.002 11)(0.003 17) = 6.7 x 10-6

679
23 Organic Chemistry

23.1

23.2

23.3

23.4 C7H16

23.5 Structures (a) and (c) are identical. They both contain a chain of six carbons with two –
CH3 branches at the fourth carbon and one –CH3 branch at the second carbon. Structure
(b) is different, having a chain of seven carbons.

681
23.6 The two structures are identical. The compound is
23.7 (a) pentane

2-methylbutane

2,2-dimethylpropane
(b) 3,4-dimethylhexane
(c) 2,4-dimethylpentane
(d) 2,2,5-trimethylheptane

23.8 (a) (b)

(c) (d)

23.9 2,3-dimethylhexane

23.10 (a) 1,4-dimethylcyclohexane (b) 1-ethyl-3-methylcyclopentane


(c) isopropylcyclobutane

23.11 (a) (b) (c)

23.12

682
Chapter 23 - Organic Chemistry
______________________________________________________________________________

23.13 (a) (b)

23.14 (a) (b)

23.15 (a) 3-methyl-1-butene (b) 4-methyl-3-heptene (c) 3-ethyl-1-hexyne

23.16 (a) (b)

(c)

23.17 (a) (b) (c)

23.18

23.19

23.20 (a) (b) (c)

683
Chapter 23 - Organic Chemistry
______________________________________________________________________________

23.21 (a) (b) (c)

23.22

23.23 (a) (b)

23.24 (a) (b)

(c)

23.25 (a) (b)

23.26

23.27 (a) (b)

684
Chapter 23 - Organic Chemistry
______________________________________________________________________________

23.28

Understanding Key Concepts

23.29

23.30 (a) (b)

23.31 (a) (b)

23.32 (a) alkene, ketone, ether (b) alkene, amine, carboxylic acid

23.33 (a) 2,3-dimethylpentane (b) 2-methyl-2-hexene

23.34

23.35

685
Chapter 23 - Organic Chemistry
______________________________________________________________________________

23.36 There are many possibilities. Here are two:

23.37

Additional Problems
Functional Groups and Isomers

23.38 A functional group is a part of a larger molecule and is composed of an atom or group of
atoms that has a characteristic chemical behavior. They are important because their
chemistry controls the chemistry in molecules that contain them.

23.39 (a) (b) (c) (d)

23.40 (a) (b) (c)

23.41 (a) (b) (c) (d)

23.42

23.43 (a)

(b)

686
Chapter 23 - Organic Chemistry
______________________________________________________________________________

(c)
(d)

23.44 (a) alkene and aldehyde (b) aromatic ring, alcohol, and ketone

23.45 ester, aromatic ring, and amine

Alkanes

23.46 In a straight-chain alkane, all the carbons are connected in a row. In a branched-chain
alkane, there are branching connections of carbons along the carbon chain.

23.47 An alkane is a compound that contains only carbon and hydrogen and has only single
bonds. An alkyl group is the part of an alkane that remains when a hydrogen is removed.

23.48 In forming alkanes, carbon uses sp3 hybrid orbitals.

23.49 Because each carbon is bonded to its maximum number of atoms and cannot bond to
additional atoms, an alkane is said to be saturated.

23.50 C3H9 contains one more H than needed for an alkane.

23.51 (a) Underlined carbon has five bonds.

(b) Underlined carbon has five bonds.


(c) Underlined carbon has six bonds.

23.52 (a) 4-ethyl-3-methyloctane (b) 4-isopropyl-2-methylheptane


(c) 2,2,6-trimethylheptane (d) 4-ethyl-4-methyloctane

23.53 2,2,4-trimethylpentane

687
Chapter 23 - Organic Chemistry
______________________________________________________________________________

23.54 (a) (b)

(c) (d)

23.55 (a) (b) (c) (d)


23.56 (a) 1,1-dimethylcyclopentane (b) 1-isopropyl-2-methylcyclohexane
(c) 1,2,4-trimethylcyclooctane

23.57 (a) The longest chain contains six carbons and the molecule should be named from a
hexane root; the correct name is 3,3-dimethylhexane.
(b) The longest chain contains seven carbons and the molecule should be named from a
heptane root; the correct name is 3,5-dimethylheptane.
(c) The ring is a cycloheptane ring and the methyl groups are in the 1 and 3 position; the
correct name is 1,3-dimethylcycloheptane.

23.58 The structures are shown in Problem 23.2.


hexane, 2-methylpentane, 3-methylpentane, 2,2-dimethylbutane, and 2,3-dimethylbutane

23.59
heptane 2-methylhexane

3-methyhexane 2,2-dimethylpentane

3,3-dimethylpentane 2,3-dimethylpentane 2,4-dimethypentane

688
Chapter 23 - Organic Chemistry
______________________________________________________________________________

2,2,3-trimethylbutane 3-ethylpentane
23.60
(a)

(b)

(c)

23.61 Reaction (a) is likely to have a higher yield because there is only one possible
monochlorinated substitution product. Reaction (b) has four possible monochlorinated
substitution products, which would result in a lower yield of the one product shown.

Alkenes, Alkynes, and Aromatic Compounds

23.62 (a) sp2 (b) sp (c) sp2

23.63 Alkenes, alkynes, and aromatic compounds are said to be unsaturated because they do
not contain as many hydrogens as their alkane analogs.

23.64 Today the term "aromatic" refers to the class of compounds containing a six-membered
ring with three double bonds, not to the fragrance of a compound.

23.65 An addition reaction is the reaction of an XY molecule with an alkene or alkyne.

689
Chapter 23 - Organic Chemistry
______________________________________________________________________________

23.66 (a) (b) (c)

23.67

23.68 (a) 4-methyl-2-pentene (b) 3-methyl-1-pentene


(c) 1,2-dichlorobenzene, or o-dichlorobenzene
(d) 2-methyl-2-butene (e) 7-methyl-3-octyne

23.69 (a) (b) (c)

23.70
1-pentene 2-pentene

2-methyl-1-butene

Only 2-pentene can exist as


cis-trans isomers.
2-methyl-2-butene 3-methyl-1-butene

23.71
1-pentyne 2-pentyne

3-methyl-1-butyne

23.72 (a) CH2=CHCH2CH2CH2CH3 This compound cannot form cis-trans isomers.


(b) CH3CH=CHCH2CH2CH3 This compound can form cis-trans isomers because
of the different groups on each double bond C.
(c) CH3CH2CH=CHCH2CH3 This compound can form cis-trans isomers because

690
Chapter 23 - Organic Chemistry
______________________________________________________________________________

of the different groups on each double bond C.

23.73 (a)
This compound can form cis-trans isomers because of the different groups on each
double bond C.

(b) This compound cannot form cis-trans isomers.

(c)
This compound can form cis-trans isomers because of the different groups on each
double bond C.

23.74 (a) (b)

(c)

23.75 (a) (b)


5-methyl-2-hexene

2,2-dimethyl-3-hexyne

(c) (d)
2-methyl-1-hexene

1,3-diethylbenzene

691
Chapter 23 - Organic Chemistry
______________________________________________________________________________

23.76 Cis-trans isomers are possible for substituted alkenes because of the lack of rotation
about the carbon-carbon double bond. Alkanes and alkynes cannot form cis-trans
isomers because alkanes have free rotation about carbon-carbon single bonds and
alkynes are linear about the carbon-carbon triple bond.

23.77 Small-ring cycloalkenes don't exist as cis-trans isomers because the trans isomer could
not close the carbon-carbon chain back on itself to form a ring.

23.78 (a)

(b)

(c)

23.79 (a)

(b)

(c)

23.80 (a)

692
Chapter 23 - Organic Chemistry
______________________________________________________________________________

(b)

(c)

(d)

23.81
cyclohexane

Alcohols, Amines, and Carbonyl Compounds

23.82 (a) (b)

(c) (d)

23.83 (a) CH3CH2CH2NH2 (b) (CH3CH2)2NH (c) CH3CH2CH2NHCH3

693
Chapter 23 - Organic Chemistry
______________________________________________________________________________

23.84 Quinine, a base will dissolve in aqueous acid, but menthol is insoluble.

23.85 Pentanoic acid will react with aqueous NaHCO3 to yield CO2, but methyl butanoate will
not.

23.86 An aldehyde has a terminal carbonyl group. A ketone has the carbonyl group located
between two carbon atoms.

23.87 In aldehydes and ketones, the carbonyl-group carbon is bonded to atoms (H and C) that
don't attract electrons strongly. In carboxylic acids, esters, and amides, the carbonyl-
group carbon is bonded to an atom (O or N) that does attract electrons strongly.

23.88 The industrial preparation of ketones and aldehydes involves the oxidation of the related
alcohol.

23.89 Carboxylic acids, esters, and amides undergo carbonyl-group substitution reactions, in
which a group –Y substitutes for the –OH, –OC, or –N group of the starting material.

23.90 (a) ketone (b) aldehyde (c) ketone (d) amide (e) ester

23.91 (a)
N,N-dimethylpropanamide pentanamide
N-methylbutanamide

(b)
methyl pentanoate ethyl butanoate

propyl propanoate

23.92 C6H5CO2H(aq) + H2O(l) _ H3O+(aq) + C6H5CO2-(aq)


initial (M) 1.0 ~0 0
change (M) -x +x +x
equil (M) 1.0 - x x x
[H3 O+][C6 H5 CO2_ ] x
2 2
Ka = = 6.5 x 10_ 5 = ≈ x
[C6 H5 CO2 H] 1.0 _ x 1.0
+
x = [H3O ] = [C6H5CO2H]diss = 0.0081 M
[C6 H5 CO2 H ]diss 0.0081 M
% dissociation = x 100% = x 100% = 0.81%
[C6 H5 CO2 H ]initial 1.0 M

694
Chapter 23 - Organic Chemistry
______________________________________________________________________________

23.93 (a) (b) (c)

23.94 (a) methyl 4-methylpentanoate (b) 4,4-dimethylpentanoic acid


(c) 2-methylpentanamide

23.95 (a) N,N-dimethyl-4-methylhexanamide (b) isopropyl 2-methylpropanoate


(c) N-ethyl-p-chlorobenzamide

23.96 (a) (b)

(c)

23.97 (a) (b)

(c)

23.98 (a)

(b)

(c)

695
Chapter 23 - Organic Chemistry
______________________________________________________________________________

23.99 (a)

(b)

(c)

23.100 amine, aromatic ring, and ester

carboxylic acid alcohol

23.101

Polymers

23.102 Polymers are large molecules formed by the repetitive bonding together of many smaller
molecules, called monomers.

23.103 Polyethylene results from the polymerization of a simple alkene by an addition reaction
to the double bond. Nylon results from the sequential reaction of two difunctional
molecules.

23.104

696
Chapter 23 - Organic Chemistry
______________________________________________________________________________

23.105 (a)

(b)

23.106 (a) (b) (c)

23.107

23.108

23.109

General Problems

23.110 (a) (b)

(c) (d)

697
Chapter 23 - Organic Chemistry
______________________________________________________________________________

(e) (f)

23.111 (a) 2,3-dimethylhexane (b) 4-isopropyloctane


(c) 4-ethyl-2,4-dimethylhexane (d) 3,3-diethylpentane

23.112 Cyclohexene will react with Br2 and decolorize it. Cyclohexane will not react.

23.113 Cyclohexene will react with Br2 and decolorize it. Benzene will not react with Br2
without a catalyst.

23.114 (a) (b) (c)

23.115

Multi-Concept Problems

23.116 (a) Calculate the empirical formula. Assume a 100.0 g sample of fumaric acid.
1 mol C 1 mol H
41.4 g C x = 3.45 mol C; 3.5 g H x = 3.47 mol H
12.01 g C 1.008 g H
1 mol O
55.1 g O x = 3.44 mol O
16.00 g O
Because the mol amounts for the three elements are essentially the same, the empirical
formula is CHO (29 amu).

(b) Calculate the molar mass from the osmotic pressure.


 1.00 atm 
 240.3 mm Hg x 
Π  760 mm Hg 
Π = MRT; M = = = 0.0129 M
RT  L • atm 
 0.082 06  (298 K)
 K • mol 
(0.1000 L)(0.0129 mol/L) = 1.29 x 10-3 mol fumaric acid

698
Chapter 23 - Organic Chemistry
______________________________________________________________________________

0.1500 g
fumaric acid molar mass = = 116 g/mol
1.29 x 10_ 3 mol
molecular mass = 116 amu

molar mass 116


(c) Determine the molecular formula. = =4
empirical formula mass 29
molecular formula = C(1 x 4)H(1 x 4)O(1 x 4) = C4H4O4
From the titration, the number of carboxylic acid groups can be determined.
1 mol C4 H 4 O4
mol C4H4O4 = 0.573 g x = 0.004 94 mol C4H4O4
116 g
mol NaOH used = (0.0941 L)(0.105 mol/L) = 0.0099 mol NaOH
mol NaOH 0.0099 mol
= =2
mol C4 H4 O4 0.004 94 mol
Because 2 mol of NaOH are required to titrate 1 mol C4H4O4, C4H4O4 is a diprotic acid.
Because C4H4O4 gives an addition product with HCl and a reduction product with H2, it
contains a double bond.

(d) The correct structure is

23.117 (a) CO2, 44.01 amu; H2O, 18.02 amu

1 mol CO2
mol CO2 = 0.1213 g CO2 x = 0.00276 mol CO2
44.01 g CO2
1 mol H 2 O
mol H2O = 0.0661 g H2O x = 0.00367 mol H2O
18.02 g H2 O
1 mol C 12.011 g C
mass C = 0.00276 mol CO2 x x = 0.0332 g C
1 mol CO2 1 mol C
2 mol H 1.008 g H
mass H = 0.00367 mol H2O x x = 0.00740 g H
1 mol H 2 O 1 mol H
mass O = 0.0552 g sample - 0.0332 g C - 0.00740 g H = 0.0146 g O
1 mol C
mol C = 0.00276 mol CO2 x = 0.00276 mol C
1 mol CO2

699
Chapter 23 - Organic Chemistry
______________________________________________________________________________

2 mol H
mol H = 0.00367 mol H2O x = 0.00734 mol H
1 mol H 2 O
1 mol O
mol O = 0.0146 g O x = 0.000913 mol O
16.00 g O
C0.00276 H0.00734 O0.000913 (divide each subscript by the smallest)
C0.00276 / 0.000913 H0.00734 / 0.000913 O0.000913 / 0.000913
C3.023 H8.039 O
C3H8O

2 C3H8O(l) + 9 O2(g) → 6 CO2(g) + 8 H2O(l)

(b) C3H8O is a molecular formula because a multiple such as C6H16O2 is not possible.
(c)

(d) Acetone is . The most likely structure for C3H8O is .

(e) C3H8O, 60.10 amu


1 mol C3 H8 O
mol C3H8O = 5.000 g C3H8O x = 0.08320 mol C3H8O
60.10 g C3 H8 O
_ 166.9 kJ
∆Hocombustion = = -2006 kJ/mol = -2006 kJ
0.08320 mol
C3H8O(l) + 9/2 O2(g) → 3 CO2(g) + 4 H2O(l)
∆Hocombustion = [ 3 ∆Hof(CO2) + 4 ∆Hof(H2O)] - ∆Hof(C3H8O)
∆Hof(C3H8O) = [ 3 ∆Hof(CO2) + 4 ∆Hof(H2O)] - ∆Hocombustion
= [(3 mol)(-393.5 kJ/mol) + (4 mol)(-285.8 kJ/mol)] - (-2006 kJ)
= -317.7 kJ
∆Hof = -317.7 kJ/mol

23.118 (a) propanamide

(b)

700
Chapter 23 - Organic Chemistry
______________________________________________________________________________

(c)

(d) An observed trigonal planar N does not agree with the VSEPR prediction. The
second resonance structure is consistent with a trigonal planar N.

701

You might also like